'^^o^ ,4 ^ v^ %■"> r ^ ^ * ^ "^ ^.^'^ .5> .^^^<^ ^'- %.o^ ' A^ . .„ -9^,. ^0 % -0^ s ^ ' " <. ^^^ 0-. '^^-^^^f^ ^^^'^m^Sf> ^^^'j^-% ^^^^- ua ir^ lAu. THE SYSTEM METHOD -¥o. 1. THE System or Branching Method BIDTJO^^TIOlSr- BY / ISAAC ELDRIDGE WILSON, INVENTOR OP THE LEAKNEB'S WOKK-TBEE, AUTHOR OF THE SYSTEM METHOD, A SBBIES OF TEXT-BOOKS FOB SCHOOLS, A SERIES OF STUDENT'S WORK-BOOKS, ETC. #/ ' THE MAN MAY TEACH BY DOING, AND NOT OTHERWISE."— EMERSON. Yolume 1. Learner's Work-Tree Company. kl Ij ID -Ws'i J^. J<^ .^^K^^ COPYRIGHT, 1886, BY ISAAC ELDRIDGE WILSON. ALL RIGHTS RESERVED. Any infringement will be prosecuted to the full extent of the law, "^V^- ^^^' "T^ / Qt^l 97 LIST OF SCIEITCE-TREE GUIS. ij {J ^ ij ij ^^ PAGE Botanical Work Tree 25 Zoological Work-Ti'ce 26 Learner's Work-Tree 98 Orthographic Work-Tree 102 Orthographic Synthesis-Tree 116 Ideographic Work-Tree (Final Letters and Hyphen Branches) 175 Orthographic Work-Tree Complete 202 Ideographic Work-Tree (Capital Letter Branch) 229 Etymological Work-Tree 274 Grammatical Work-Tree Incomplete 488 Grammatical Work-Tree Intermediate 574 Grammatical Work-Tree Complete 494 Arithmetical Work-Tree (Not Corrected) 628 Arithmetical Work-Tree (Corrected) 642 pRKE'i^.ai:. ai i^HE great theme of this volume is the learner's process of k work. That learning is doing, is everywhere cried — doing \^^W^ what\% the difficulty, the estopping question. That learning is doing something has ever been known. Learning is doing what ? has remained till to-day the great unsolved problem of the school world. Nor is this doing process, by which we are to learn the system (science) of orthography, grammar, etc., undeveloped because merely blocked, or burdened : it is undeveloped because practically undiscovered, or unknov/n. For, previous to the discovery that these sciences are tree systems, we knew not zvhat system these science systems are; as a matter of course, we could not do such systems. Now that these sciences are shown to be tree systems {branch systems'), the true process of learning (doing) is at once revealed to be branching — an up-the-tree process. The student's doing process, in learning letter, word, etymon, etc., is branching such letter, word, etymon, etc., up the tree branches, which are its classes organized together as c\^.%?,-branches of a science- tree system. No longer either a mystery or a difficulty, the learner's process of doing (learning, or making) orthography, grammar, etc., is, with the i:)ubli- cation of this first volume of The System or Branching Method^ made known. It has been known only that every science is a "system," a "one," a "whole," a "unified whole," or a "unity." It was not known what system a science system is. Ueberweg : "Science is a whole of knowledge in the form of the system." " Science has its true existence only in the ^j/i-/^;/?/^^/-/;/." A "whole" what? in the "form" of a/Aa/ system? These ques- tions Ueberweg leaves unanswered. Bishop Butler: "A system [science system] is a one or a. whole [vl vi PREFACE. made up of several parts." But what "whole," and what "system," is a science? Herbert Spencer: "Knowledge of the lowest kind is ununified knowledge ; science is partially imified knowledge ; philosophy is completely ?^;z{)f^^ knowledge." "Unified" into what unit? is the difficulty unanswered. Sir Wm. Hamilton: "Generalization (system knowledge) is only the apprehension of the one in the many." Apprehension of the "one" what \n the "many" objects? Anaxagoras : "The mind only knows when it subdues its objects, when it reduces the many to the one^ Reduces the "many" to the "one" what? for reducing the many to the one, must needs be reducing the many to the one something. " The Flatonists : All knowledge is the gathering up into one, and the indivisible apprehension of this unity by the knowing mind." — Hamilt07i' s Metaphysics. Gathering up into a "one" what? is still the unsolved problem. Plotinus : "Our knowledge is perfect as it is one.'" — Id. " One " what ? '•'•Leibnitz, Kant, Aristotle, have defined knowledge by the repre- sentation of multitude in unity. ^^ — Id. Tlius do authorities declare that a science is a "system," a "one," and a "whole." But a system, a one, and a whole must be a system, a one, and a whole oi some kind; oi what kind has been the insuper- able difficulty. It has not been known what kind of a "system," a "one," a "whole," a "unified whole," or "unity" a science is. And because we knew not what system, Avhole, or unit a science system is, we could not ascertain how to do such science (system). To unlock this doing process, — to find the learner's true process of work, — was to find what system a science system is. Now the golden key with which this "doing" difficulty was unlocked (in orthography, grammar, initial and final letters, etymology, botany, zoology, and arithmetic) was the elementary principle that all thinking — all learning or knowing — is branching ^dcCacwX-diX into known (branched) class. By "branching" is meant classifying (first step), organizing (second step). PREFACE. vii The systematizers (scientists) have taken the first step, — merely classifying (the letter in orthography, the word in grammar, etc.,) the particulars. But because they have not " gathered up into one" (tree) these classes so formed ; because they have not thus or- ganized (branched) these classes into tree-system " unity ; " they have known neither the student's process of work (branching) nor his in- strument and guide (the tree). They have not taken the second step, — branching or organiz- ing these classes into a "unilied" ''system" "whole," — without which second step the learner's process of doing, or making, a branch is impossible j just as it is impossible to do or make a building without some knowledge of what the unified system whole of such building is. Such second step organizes (branches) the classes (of letters, words, etymons, etc.,) into a tree-system whole, every class properly be- longing to such science system becoming a branch of such logical tree : so that, by tracing a letter^ a word, etc., upward through its tree system, we therein trace (branch) such letter or word through its classes. But tracing a thing through its classes is learning that thing; and tracing (branching) a thing through its classes as branched together in a "unified" "system" "whole," is learning that thing by means oi system (science). Thus it is that the learner's doing pro- cess is branching (from the tree trunk upward), and that the branch whole (the tree) of the system is his invaluable instrument and guide. How invaluable is this branch-whole (the tree) in learning (doing, or making) such branch, will astonish most teachers. Suffice it here to say that such tree whole (branch-plan) cannot be less invaluable to the learner (doer, or maker,) of such branch than is the building- whole (building-plan) to the builder (doer, or maker,) of a building. If building a building is doing (making) a building, as all agree it is, what think you of builders who know not even what the system whole (plan) of a building is like? If, then, learning a branch is doing (making) a branch, as all unwittingly or otherwise agree it is, what think you of our learners who know not even what the system whole of a branch is like ? And whom does it not startle that we have studied orthography, grammar, etc., not knowing what the system whole is like? What think you of a student who finishes his study of tlie ammal parts, — VUl PREFACE. ulna, radius, tibia, fibula, eye, ear, stomach, liver, heart, back-bone, pelvis, etc., etc., — ignorant what the unified system whole of these animal parts is? What think you of a student of these animal parts who "completes" his study, ignorant that such a thing as an animal (whole) has any existence? What think you, then, of our students who actually finish their study of the tree parts, — sonant, subvocal, digraph, aspirate, abrupt, continuant, dental, labial, etc., — ignorant what the unified system whole of these tree parts is? What think you, further, of a student of these tree parts (branches) who "com- pletes" his study (as our pupils all do), actually ignorant that such a thing as an orthography tree (whole) has any existence? Not knowing the system (whole), it was impossible to find the doing process by which to do the system (science). And this fact, namely, that they know not what system a science is, — this is why no system of doing processes is to be found in the writings of Aristotle, Mon- taigne, Comenius, Locke, Rousseau, Pestalozzi, Jacotot, Spencer, or other educational reformer. But, if learning a branch is doing that branch, should they who profess to show us how to learn it, be them- selves unable to show us how to do it ? If learning orthography is doing it, ought not he who knows how to learn it, to know also how to do it? Teachers have reason, therefore, for protesting against fruitless theorizing and unapplied schemes ; for, even though a theory be pure gold, what is gold worth that cannot be used ? Away with these vain essays and prolix lectures ! Away with any who, professing to know what is to be done to learn, are yet themselves unable to do that thing ! "Every tree that bringeth not forth good fruit is hewn down and cast into the fire. ' ' He that professes to know the adding process, should not be unable to add. I. E. WILSON. P. S. — In justice to himself, the author should state that this entire volume has been written within a period of twelve months, duri:-:g which time he has taught an average of five hours a day. Those v.'ho from experience know what multiplicity of labors crowd upon the teacher to unfit him for such a work, will exercise a charity of judg- ment toward the author for whatever undue repetition or hasty work appears. <30NTi:nTS. The System Method. The Knowing Thought Thinks Paeticulab Through Class 15 The Mental Result op Learning is a Branch (or Tree) System 40 We Learn the Branching, or Organizing, Act onlt bt Doing It 54 A Stupid Question 60 The Educational Guide a Natural Product 64 The Long-Sought-eor Guide in Remembering is the Tree-Ststem Whole . . 66 The Branch Whole — Orthography. The Wliole and the Cat's-tail Part — No Branching Guide Previously Discovered — The Orthostatic Work- Tree — Bird's-Eye View of the Branching Process — Written Analysis by the Worli-Tree — Oral Analysis by the Work-Tree — Progi-am Made up from Work-Tree — The Words Analyzed — No Current Method Correct — Written Synthesis by Work-Tree — Oral Synthesis by Means of Work-Tree — The Student Now Knows Whole and Part — Many Learned in One — Why these 3-step and 4-step Programs are Used — By Programs, the Student Learns by Doing — The Programs Force the Student to Decide — This Method of Branching by Programs Sustained by Author- ity .pp. 79-145 WoRK-BooK OF System Orthography— Grade B. Appendix A: Lessons to be Assigned — Appendix B: Orthographic Chart — Appendix C: Rules for Spelling — Challenge — Prog. 1:1. Voice. 2. Sound. 3. Continuance. — Prog. II: 1. Voice. 2. Sound. 3. Position. — Prog. Ill: 1. Voice. 3. Sound. 8. Continuance. — ^Prog. IV: 1. Letter. 3. Sound. 3. Continuance. — Prog. V: Branching by Learner's Work-Tree — Prog. VI: Branching by Learner's Work-Tree (Continued) — ^Prog. VII: 1. Position. 3. Sound. 3. Letter.— Prog. VIII: 1. Position. 2. Sound. 3. Letter.— Prog. IX: 1. Position. 2. Construction. 3. Continuance. — Prog. X: 1. Letter. 3. Continuance. 3. Obstruction. — Prog. XI : Branching by Learner's Work- Tree — ^Prog. XII: Branching by Work-Tree (Continued) — ^Prog. XIII: Branching by Work-Tree (Continued) — Prog. XIV: 1. Union. 2. Letter. 3. Subdivision. — ^Prog. XV: 1. Union. 3. Construction. 3. Diacritical Mark. — Prog. XVI: 1. Union. 3. Obstruction. 3. Diacritical Mark.— Prog. XVII: X C0NTE27T8. 1. Union. 2. Obstruction. 3. Diacritical Mark. — Prog. XVIII: 1. Position. 2. Accent of Syllable. 3. Diacritical Mark. — ^Prog. XIX: 1. Position. 2. Accent of Syllable. 3. Diacritical Mark. — ^Prog. XX: 1. Position. 2. Inter- ruption. 3. Diacritical Mark. — ^Prog. XXI: "Written Analysis — Prog. XXII: Oral Analysis — Prog. XXIII : Oral Analysis — ^Prog. XXIV : Oral Analysis —Prog. XXV: 1. Number of Noun. 2. Eulefor Spelling. 3. Gender.— Prog. XXVI: 1. Number of Noun. 2. Rule for SpeUing. 3. Gender.— Prog. XXVII: 1. Principal Parts of Verb. 2. Regularity. 3. Rule for Spelling. — ^Prog. XXVIII: 1. Prin. Parts. 2. Inflection. 3. Rule for Spelling.— Prog. XXIX: Consonant Doubled by Learner's Work-Tree — ^Prog. XXX : Vowel Changed ■by Learner's Work-Tree — Prog. XXXI: Plural Sign by Learner's Work- Tree — Prog. XXXII : Rules for Spelling by Learner's Work-Tree — Prog. XXXIII: Rules for Spelling by Work-Tree (Continued)— Prog. XXXIV: Rules for Spelling by Work-Ti-ee (Continued) — Prog. XXXV: 1. Phonic Spelling. 2. Syllabication. 3. Rule for SpeUing.— Prog. XXXVI: 1. Phonic Spelling. 2. Syllabication. 3. Rule for Spelling. — Appendix D : Definitions of Orthographic Terms pp. 147-180 WoRK-BooK OF System Orthography — Grade A. '^-^ Appendix A: Lessons to be Assigned — Appendix B: Orthographic Chart — Appendix C: Rules for Spelling — Challenge — Prog. I: 1. Position. 2. Voice. 3. Sound. 4. Continuance. — Prog. II: 1. Position. 2. Contin- uance. 3. Obstruction. 4. Sound. — Prog. Ill: Branching by Learner's Work-Tree — Prog. IV: Branching by Learner's Work-Tree- — Prog. V: 1. Position. 2. Construction. 3. Letter. 4. Sound. — Prog. VI: 1. Sound. 2. Obstruction. 8. Construction. 4. Letter. — Prog. VII: 1. Letter. 2. Continuance. 3. Obstruction. 4. Interruption. — Prog. VIII: 1. Union. 2. Letter. 3. Subdivision. 4. Obstruction. — Prog. IX: 1. Union. 2. Ob- struction. 3. Subdivision. 4. Diacritical Mark. —Prog. X: 1. Union. 2. Interruption. 3. Subdivision. 4. Diacritical Mark. — Prog. XI: Written Analysis — Prog. XII: Oral Analysis— Prog. XIII: Written Synthesis by Synthesis Tree — Prog. XIV: Written Synthesis from Memory (Mental Tree) — Prog. XV: Oral Synthesis from Memory (Mental Tree) »— Prog. XVI: Oral Synthesis by Position— Prog. XVII: 1. Noun. 3. Current. 3. Origin of Name. 4. Rule for Spelling. — Prog. XVIII: 1. Noun. 2. Current. 3. Origin of Name. 4. Rule for Spelling.- Prog. XIX: 1. Substantive. 2. Case Forms. 3. Number Forms. 4. Rule for Spelling. — Prog. XX: 1. Indicatives. 2. Participles. 3. Inflection. 4. Rule for Spell- ing. — Prog. XXI: 1. Composition. 2. Derivation. 3. Accent. 4. Primary Accent. — Prog. XXII: 1. Syllabication and Dir.critical Spelling. 2. Phonic Spelling. 3. Rules. — Prog XXIII: 1. Syllabication and Diacritical SpeU- ing. 2. Phonic Spelling. 3. Rules. — Prog. XXIV: Rules for speUing by Work-Tree — Prog. XXV : Hyphen, Consonant, and Vowel by the W ork- Tree— Prog. XXVI: Plural and Possessive Signs by the Work-Tree CONTENTS. xi — Prog. XXVII: Final Letters and Hyphen bj- the Work-Tree — Prog. XXVIII: Capital Letters by the Work-Tree — Prog. XXIX: Capital Let- ters by the Work-Tree — Prog. XXX : Capital Letters by the Work-Tree — Prog. XXXI : Final Letters and the Hyphen by the Work-Tree — Prog. XXXII: Final Letters and the Hyphen (Continued) — Prog. XXXIII: 1. Composition. 2. Derivation. 3. Origin of Eoot. 4. Affix. — Prog. XXXIV: 1. Composition. 2. Derivation. 3. Origin of Root. 4. Affix. — Appendix D : Number, Gender, and Case — Appendix E : Piules for Syllabication — Appendix F: Rules for Pronunciation — Appendix G: Rules for the Use of Capitals — Appendix H: Nomenclature of Orthography pp. 181-244 WoRK-BooK OF System Ety:\iology. Appendix A: Lessons to be Assigned — Prog. I: 1. Indicatives. 2. Participles. 3. Inflection. 4. Rule for Spelling. — Prog. II: 1. Indicatives. 2. Participles. 8. Regularity, 4. Rule for Spelling. — ^Prog. Ill: 1. Noun. 2. Current. 3. Origin of Name. 4. Rule for Spelling. — Prog. IV: 1. Plural. 2. Current. 3. Origin of Name. 4. Rule for Spelling. — Prog. V: 1. Composition. 2. Der- ivation. 3. Accent. 4. Primary. — Prog. VI: 1. Substantive. 2. Number Forms. 3. Case Forms. 4. Gender. — ^Prog. VII: 1. Substantive. 2. Case Forms. 3. Plural. 4. Gender. — Prog. VIII: 1. Fundamental. 2. Three Parts. 3. Two Forms. 4. Rule for Spelling. — ^Prog. IX: 1. Prime Prefix. 2. Origin in. 3. Derived Forms. 4. Generic Meaning. — Prog. X: 1. Prime Affix. 2. Origin in. 3. Derived Forms. 4. Generic Meaning. — Prog. XI: 1. Composition. 2. Derivation. 3. Accessory. 4. Accessory Class. — Prog. XII : 1. Composition. 2. Derivation. 3. Accessory. 4. Accessory Class. — ^Prog. XIII: 1. Nature. 2. Consolidation. 3. Connection. 4. Generic Meaning. — Prog. XIV: 1. Nature. 2. ConsoUdation. 3. Connection. 4. Generic Mean- ing.— Prog. XV: 1. Nature. 2. Part of Word. 3. Part of Speech. 4. Generic Meaning.— Prog. XVI: 1. Nature. 2. Part of Word. 3. Part of Speech. 4. Generic j^.Ieaning. - Prog. XVII: 1. Nature. 2. Part of Word. 3. Origin in. 4. Generic Meaning. — Prog. XVIII: 1. Nature. 2. Part of Word. 3. Angli- cization. 4. Origin in. — Prog. XIX: Branching by the Learner's Work-Tree — Prog. XX : Branching by the Learner's Work-Tree — Prog. XXI : Branch- ing by the Learner's Work-Tree — Prog. XXII: 1. Prefix-|-Radical-(-Suffix. 2. Radical from. 3. Affixes from. 4. Etymological Meaning. — Prog. XXIII: 1. Preflx-fRadical-fSuffix. 2. Radical from. 3. Affixes from. 4. Etymologicrd , Meaning. — Prog. XXIV: 1. Preflx-j-Radical-)-Suffix. 2. Radical from. 3. Affixes from. 4. Etymological Meaning. — Prog. XXV: 1. Prefix-{-Radical -[-Suffix. 2. Radical from. 3. Affixes from. 4. Etymological Meaning. — Prog. XXVI: 1. Prefix-|-Radical-j-Suffix. 2. Radical from. 3. Affixes from. 4. Etymological Meaning. — Prog. XXVII: 1. Preflx-}-Radical-j-Suflix. 2. Radical from. 3. Aflaxes from. 4. Etymological Meaning. — Prog. XXVIII: Written Analysis — ^Prog. XXIX: Oral Analysis — Prog. XXX: Written Synthesis from Memory (Mental Tree) — Prog. XXXI: 1. Adjunct-fBase -j-Suffix. 2. Adjunct from. 3. Base from. 4. Etymological Meaning. — Prog. xii CONTENTS. XXXII: 1. Adjunct+Base-|-Suffix. 2. Adjunct fi-om. 3. Base from. 4. Etymological Meaning. — Appendix B : List of Irregular Verbs — Appendix C : Kules for Spelling — Appendix D : Number, Gender, and Case — Appen- dix E : Greek, Latin, and Anglo-Saxon AflBxes — Appendix E : A Catalogue showing the Roots, Affixes, and Etymological Meaning of the Most Im- portant English Words pp. 245-364 English Grammar. No Science System Known — The Potential Mood a Blunder — The Fifteen As- sertions of These Grammarians — Would Make the Verb Belong to the Mood — Potential's Foundation False — May, Can, etc. Usually Indicative — An Annihilating Argument — ^Learner's Process of Work Benighted — The Future Tense a Fallacy — Shall and WiU are Presents — The Correct Theory of Tense — The Current Doctrine of Case — Two Words as One — We Know in System — We are to Remember the System — The Tree in Developing the Science — Whether there is Such a Science as Grammatical Etymology pp. 36.5-402 The System Grammar — Grade A. First Step : Classifying the Words 405 Nouns : Demonstrations I to VI — Verbs : Demonstrations VII to XIV — ^Pro- nouns : Demonstrations XIV to XVI — Adjectives : Demonstrations XVI to XVIII — Adverbs : Demonstration XVIII — ^Prepositions : Demonstration XIX — Conjunctions: Demonstration XX — Interjections: Demonstration XXI. Second step : Branching the Classes (Dems. XXII to XXXI) 485 Analysis, or Dissection (Demonstration XXXI) 539 WoRK-BooK OF System Grammar — Grade B. Challenge — Appendix A: Lessons to Be Assigned — Prog. I: 1. Noun. 2. Number. 8. Thing Named. — Prog. II: 1. Noun. 2. Number. 3. Thing Named. 4. Rule for SpeUing. — Prog. Ill: 1. Noun. 2. Number. 8. Gen- der. 4. Rule for SpeUing. — Prog. IV: 1. Verb. 2. Principal Parts. 3. Regularity. 4. Tense. — Prog. V: 1. Regularity. 2. Principal Parts. 3. In- flection. 4. Tense. — Prog. VI: 1. Regulai'ity. 2. Inflection. 3. Action. 4. Tense.— Prog. VII : 1. Inflection. 2. Time. 3. Tense. 4. Event. — Prog. VIII: 1. Inflection. 2. Time. 3. Event. 4. Mood.— Prog. IX: 1. Inflection. 2. Tense. 3. Time. 4. Mood. Prog. X : 1. Inflection. 2. Tense. 3. Event. 4. Mood.— Prog. XI: 1. Xegulai-ity. 2. Inflection. 8. Tense. 4. Mood. — Prog. XII: 1. Pronoun. 2. Person. 3. Number. 4. Case. —Prog. XIII : 1. Substantive. 2. Person. 3. Number. 4. Gender. 5. Case. —Prog. XIV: Branching by Grammatical Work-Tree — Prog. XIV Continued: Branching of Fundamentals— Prog. XV: 1. Adjective. 2. Class. 3. Accessory to What?— Prog. XVI: 1. Adverb. 2. Class. 3. Accessory to ^Yhafi — Prog. XVII : Branching of Adjuncts, or Accessories CONTENTS. xiii — Prog. XVIII: Branching of Fundamentals and Adjuncts — Prog. XIX: 1. Preposition. 2. Object. 3. Antecedent Term. 4. Object. — Prog. XX: 1. Conjunction, 2. Class. 3. Parts Connected. 4. Likeness of Parts. — Prog. XXI: 1. Interjection, 2, Principle. — Prog. XXII: Branch- ing of Words— Prog. XXIII : Branchingof Verbs— Prog. XXIV: 1. Sub- stantive. 2. Case. 3. Construction. 4, Office. — Prog. XXV: Branch- ing of Substantives — Prog, XXVI: Branching of Accessories — Prog. XXVII: Branching of Connectives — Prog. XXVIII: Branching of Words pp. 555-576 WoRK-BooK OF System Grammar — Grade A. ■ Prog. I: 1. Noun, 2, Current. 3. Thing Named. — Prog. II: 1. Noun. 2. Current. 3. Thing Named. 4. Rule for Spelling. — Prog. Ill: 1. Class of Noun. 2. Number, 3. Gender, 4, Personification. — Prog. IV: 1. Class of Noun. 2. Person. 3. Number, 4. Gender, 5. Personification. — Prog. V: 1. Number. 2. Gender. 3. Case. 4. Case Determined by. — Prog. VI: 1. Number. 2. Gender. 3. Case. 4. Case Determined by. — Prog. VII : 1. Verb. 2. Principal Parts. 3. Regularity. 4. Tense. - — Prog. VIII: 1. Principal Parts. 2. Regularity. 3. Inflection. 4. Tense.— Prog. IX: 1. Regularity. 2. Inflection. 3. Event. 4. Mood. — Prog. X: 1. Regularity. 2. Inflection, 3. Time. 4. Mood. — Prog. XI: 1. Regularity, 2. Inflection. 3. Event. 4. Time. 5. Mood.— Prog. XII: 1. Principal Parts. 2. Regularity, 3. Tense. 4. Mood. — Prog. XIII: 1. Regularity. 2. Inflection, 3. Tense. 4. Mood.— Prog. XIV: 1. Class of Pronoun. 2. Person, 3. Number, 4. Gender. 5. Case. — Prog. XV: 1. Class of Pronoun. 2. Person. 3. Number. 4. Gender. 5. Case. — Prog. XV: Pronouns Continued — Prog. XVI: 1. Adjective. 2. Class, 3. Accessory to What? — Prog. XVII: 1. Adjec- tive. 2. Class. 3. Accessory to What?— Prog, XVIII: 1. Adverb. 2. Class. 3. Accessory to What? — Prog. XIX: 1, Preposition. 2. Ob- jective. 3. Antecedent Term, or Base. 4. Unlikeness in. — Prog. XX: 1. Conjunction. 2. Class. 3. Parts Connected. 4. Likeness of Parts. — Prog. XXI : 1. Interjection. 2. Principle. — Prog. XXII : Branching by the Gram- matical Work-Tree — Prog. XXII Continued: Branching by the Tree - Prog. XXIII : Branching by the Grammar-System Tree Complete — Prog. XXIII Continued : Branching of Infinite Verbs — Prog. XXIV : Branching by the Grammar-System Tree — Prog. XXIV Continued: Branching of Verbs — Prog. XXV: 1. Substantive. 3. Class. 8. Case. 4. Construction. 5. Oflace.— Prog. XXV Continued: Substantives Classified - Prog. XXVI: Branching by the Grammar-System Ti'ee — Prog. XXVI Continued: Branch- ing of Substantives— Prog. XXVI Continued: Branching of Substantives -Prog. XXVI Continued: Branching of Substantives— Prog. XXVII: Branching by the Grammar-System Tree— Prog. XXVII Continued: Branching of Substantives — Prog. XXVIII : Branching by the Grammar- xiv CONTENTS. System Tree — Prog. XXIX: BrancliiDg by the Grammar-Sysicui Tree — Prog. XXIX Continued : Branching of Accessories — Prog. XXX : Branch- ing by the G-rammar-System Tree — ^Prog. XXX Continued : Branching of Words — ^Prog. XXX Continued : Branching of Words — Prog. XXX Con- tinued : Branching of Words — Prog. XXXI : Analysis, or Sentence Dissec- tion pp. 577-622 The Science of Arithmetic Developed. Of Every Branch of Learning There is a One or a Whole — This One-and- Whole System is the Tree System — What is the Unit of a Science — What Would be the Tree Trunk — No-time Division of Percentage — Time Divis- ion of Percentage — The Tree Test Again— Second Set of Solutions Cor- rected by the Tree — Third Set of Solutions CoiTected by the Tree — ^Prob-' lems — System-Method Solutions — The System Method Established — The Question Set at Rest — Short Methods pp. 623-652 , WoRK-BooK OF System Arithmetic — Grade B. Compound Numbers: Reduction Ascending — Empirical and Rational Solu- tions — Comparative Measures — ^Longitude and Time: Class I and Class II Problems — Simple Proportion — Fractions — The System Method — Only Three Cases — ^ Where Should Fractions be Taught — Difference Between the Three Cases and Simple Proportion — ^Percentage — ^Proiit and Loss — Com- mission — ^Insurance — Taxes — Simple Interest pp. 653-688 WoRK-BooK OF System Arithmetic — Grade A. Reduction Ascending — Comparative Measures — Longitude and Time — Stand- ard Unit— Calendar Time — Old Style and New Style — Why Add the Extra Day, February 39 — Islanders' Date Line — International Day Line — Date- Line Problems — Compound Proportion — The System Solution Shortened — Fractions — Inquiry 1st — Inquiry 2d — Percentage — Profit and Loss — Commission — Simple Interest and True Discount — Annual and Com- pound Interest — Bank Discount — Money Exchange - Arbitration of Ex- change — Stock Investments — Arithmetical Algebraic Problems. ... pp. 689-744 Merits of the System Method. It has a Guide — It is Masterly — It has a Process — An Advanced (2d) Step — Divers Other Merits pp. 745-752 THE SYSTEM METHOD, The Knowing Thought Thinks Particular through Glass. — This is but another form of putting a truth well known to every school-boy ; namely, that every sentence has a subject (particular) and a predicate (class). A know- ing thought is a thought knowing or discovering what a thing is. Nothing could be more simple and sure than this elementary truth, that to know a thing is to know what it is. It gives no adequate intelligence concerning an unknown particular, to declare that it has such and such, that it might be so and so. You tell us that the object of which we wish to be informed has a stomach, an eye, or else ; but that is too meager. We inquire what it is. We want to know or think it through the verb is, and without this knowledge we are never satisfied. If, using the verb is instead of has or any other, you divulge that the object is such and such a thing, our curiosity and dissatisfaction are quashed. And we may become even scientifically acquainted with the object of our inquiry if you continue with the verb is till you have shown us all the classes to which it belongs ; say Caucasian, man, mammal, vertebrate, animal, etc. (Read Grevy, page 26, downward on the tree through these five classes.) Starting with the particulars mink, weasel, etc., at the extremities of the branches in cuts, pages 25, 26, let the reader form sentences as they here follow :— [15] 16 THE SYSTEM METHOD. Subject. Predicate. 1. The mink is a flesh-eater. 1. The mink is a mammal. 2. The weasel is a vertebrate. 8. The leopard is an animal. 14. Marcel is a Frenchman, 19. Huxley is a man. 29. The red pine is an exogen. 29. The red pine is a vegetable. He will not fail to perceive that the direction of the mind is from the particular (mink, Marcel, etc.,) downward on the tree through the classes (mammal, man, etc.,) to which such particular belongs. The knowing thought runs, " A mink is a flesh-eater ; " not, " A flesh-eater is a mink." The judgment is from Huxley (the particular) to or through man (the class). We say, "Huxley is a man;" not, "A man is Huxley." We say, " The banana is a phenogam ; " not, "The phenogam is a banana." Let him who would learn to teach with enhanced success and continued delight, — let him contemplate this "tree of knowledge" till he shall fully realize this greatest of all educational and psychological principles ; namely, that every thought thinks (or branches) dowjtward on the tree system from particular through known class. Here, at the outset, the author may inform the reader that all scientific education proceeds by tracing particular through its classes as branched into the tree system ; that every science system is a tree system ; that educational power is simply the power of projecting the tree-system organism into disorganized particulars ; that the crucial test to which every educational scheme is to be put, is the crucial test of the tree system ; that by means of this tree-system organism, not only is the comparative worth of every kind of knowledge to be determined, but by it the true, system, scientific, or organizing, method of teaching and learning is to be determined ; that by it the st\xd&n\.'s process of work in learning — the desideratum of the schools — is to be developed. THE FORM OF TRUTH. 17 Every truth will fit every other ; because all truth is a branch or sub-branch of a grand truth system. And this truth system is a tree system. Think of the universe of truth, or knowledge, as a grand tree of truth, in form like material trees — body comprehending all the branches, myr- iad in number, divided and sub-divided toward the utmost extremities. Now inquire which way on the limbs, sub- limbs, and remote branches of this great tree system, the human mind runs, and you will be surprised to find that you cannot think a sane thought in which the mind does not run from outward inward, from the particular to the more comprehensive part, or branch. Everywhere in the economy of life, the feasible result or function is reached by means of some organism. The great Organizer reaches the result vision through means of the organisms, the brain and the eye, which are connected as parts of one system. The same infinite Organizer reaches the result of touch, in our behalf, through the organisms finger, brain, etc., which are similarly connected. It is gratifying to know that in the realm of truth — in all our knowing — we are but to follow after the example of the Creator. To reach the power of deciding, in our behalf. He employs the organs of sight, hearing, touch, smell, temperature, and weight, which organs are likewise associated with the brain, and mutually, as parts of one sys- tem-whole. We are simply to imitate the Creator. We, too, are to reach the power to decide, which is the mental work of life, by means of an organism, — the tree-system organism, — by tracing the thing to be learned, or explained, through that tree system to which it belongs. But before going farther with these deductions, let us consider further that most elementary and most valuable of all truths ; namely, that in the mental activity called learn- ing, we think from particular through its classes— think of the thing to be learned as belonging to known classes. The discovery of the tree system as the only possible represent- 2 18 THE SYSTEM METHOD. ative of a science system, will, we doubt not, soon revolution- ize the disorganizing or incongruous methods as well as ^> A orthogra.fh;io ^work-tree. Copyright, 1886, by I. E. \^^LSON. much of the text-book matter now taught in the schools. To this science-system representative, the author wishes the reader to direct his attention till he shall become familiar * Or, DIGRAPH, DIPHTHONG, TRIGRAPH. THE FORM OF TRUTH. 19 with it. Observe, then, that what is the truth and the know- ing thought, always runs from branch toward body (higher branch) — from particular through class — classes that are limbs of the tree system. We say, — ibjec /. Predicate. Class. Pari. icular. Z is a sub vocal, not. A subvocal is z. Z is a letter. not, A letter is z. Z is a continuant, not. A continuant is z. A is a vocal. not. A vocal is a. A is a sonant, not. A sonant is a. K is an aspirate, not, An aspirate is k. K is a letter. not, A letter is k. ■ " Z is a subvocal " is truth ; for it goes from particular through class, from branch toward higher branch or body. "A subvocal is z" is not truth, necessarily ; for it goes per- versely backward from class to particular. A subvocal might be r, n, or 1, as well as z. And so universally : The knowing thought thinks from particular toward or through the class of which that particular is part, or branch. Authority. — " Many men, when a proposition is to be proven, spend their time in hunting up authorities and col- lecting the opinions of others. By this they expect men to be convinced, without once asking the question whether they are convinced themselves. I would by no means speak lightly of the learning of the past or of the opinions of em- inent men ; but it must still be apparent that an opinion, whether of an ancient or a contemporary, is worth just as much as the reason on which it is founded." — Waylajzd. If it is true that thinking is through known class, it is also true that learning is a branching act — an act branching the particular investigated through its classes as branched together in science {i. e., tree) system. Moreover, if the learning process is branching, and the long-sought-for guide in learning is the branch system, it follows that the 07'ganiz- ingi or system-guided, method employed in this book further 20 TEE SYSTEM METHOD. on, is the true method of learning and teaching — the desid- eratum of the schools. We invite the reader, therefore, not to read this argument to believe blindly ; nor should he read to disbelieve ; he should only be willing and ready, if there be real force in the argument, to be forced to believe. He will, therefore, himself carefully weigh the proof as below offered from authority. Schuyler: "Thought is the recognition of one thing under or in another, as a species [subvocal, for example] under its genus, or class [sonant]." The declaration that thought is the recognition of a species (or particular) as under a genus, or class, is virtually identical with the declaration that the mind passes down- ward on the tree organism from branch toward body ; since that is the only possible direction it could take to declare the species (or particular) to be under or included in the genus, or class. In recognizing the species subvocal to be included in the genus sonant or the genus letter, the mental act starts with subvocal, and goes downward, thus : " Subvo- cals are sonants ; " " Subvocals are letters ; " for when the mental journey is made the other way, — " Letters are sub- vocals," — the thought is not truthful or sane, since some letters are aspirates. Day: "John is a man. Plato and Socrates were Athe- nians. In all such propositions the assertion declares the subject [particular] to be included under [or to belong to] the class denoted by the predicate." Wayland: "We think in judgments. To conceive of things without forming judgments is to make no progress." "The subject [of the judgment, or sentence] may be either an individual or a species ; the predicate must be a genus ; that is, it must designate a larger class than the subject. In a proposition, we therefore affirm that a particular individual is included within a particular class. Hence, every proposi- tion must be either true or false. The subject is either TEE FORM OF TRUTH. 21 included within the class designated by the predicate or it is not," Read wakefully the vital truth here asserted by Mr. Wayland : Every proposition is true or {■d\'s>Q just accordingly as it does or does not affirm that particular belongs in class, in making which affirmation the mental incursion is necessa- rily downward from branch to body of whatever science sys- tem it may be connected with. It is mortal. — It has been asserted that the exclusive form of knowing a thing, is by knowing of what class it is — to think it through the verb is as belonging to some class. It may be supposed by one not accustomed to abstract thinking that in such judgments as " Man is mortal," " Grass is green," "Riches are perishable," and "Chalk is white," there is a comprehending thought without any reference to class. To this objection, the author would reply, (1.) That the thought, or judgment, is not as comprehensive as it would be if the predicate adjective were replaced by a class noun. "Man is mortal" announces only that man belongs in the mortal class of things. " Man is a mortal " announces, not only that the object referred to belongs in the mortal class of things, but more, that the object is a man — ^belongs to the class man. Thus the class noun mortal is logically as much more comprehensive than the predicate adjective mortal as the man is more comprehensive than the class mortal things ; (2.) The doctrine that the predicate adjective in " Man is mortal," " Chalk is white," etc., does not refer the subject particular to a class, is erroneous. It is not denied that "mortal" and "white" express the respective attributes of " man " and " chalk." But eveiy attribute is a class-maker. And no attribute can be perceived, or recognized, except there is a comparison. This psychological truth is estab- lished. But the act of comparison implies unity plus either unity or a greater number ; that is to say, the act of com- parison implies a class, the individuals of which are for the 22 TEE SYSTEM METHOD. comparison. Hence, every recognition of an attribute is a recognition of a class. Therefore the judgment, " Man is mortal," is a mental act identifying the particular man with the class mortal thijigs. Way land: "Judgment is an act of the mind affirming that a particular individual or species is included in a genus, or class. Thus, I judge snow to be white, grass to be green, avarice to be contemptible ; that is, I judge the particular individuals to be comprehended within the class which I predicate of them." "Whatever is affirmed or denied of a class is affirmed or denied of every individual under that class. Thus, when I say, Snow is white, I mean that snow is com- prehended under the class white [white things], and I affirm this also of all snow whatever." " By the proposition, Caesar was detestable, I affirm that Csesar [the particular] is in- cluded under the class detestable [detestable things]." Reductio ad absurdum. — The foregoing arguments that the mental act of knowing is from particular through class, are conclusive. But to make safety safer still, if possi- ble, let the contrary of the foregoing argument be proved to be false. Assume, then, that the knowing act journeys from class through particular. It is accordingly to be shown that (1.) Endogens are bananas, (2.) Germans are Beethovens, (3.) Animals are lions, are true propositions. But if animals are necessarily lions, then leopards, wolves, and foxes, which are animals, are also lions. But this is absurd. Therefore the proposition that the act of thinking, or knowing, is from class to particular, is false. And so again n if Germans are necessarily Beethovens, then Froebel and Pestalozzi, who are Germans, are not Froebel and Pestalozzi, but two Beetho- vens. And so on. The second form of the knowing proposition is, " Man is mortal," " Snow is white," etc. "Mortal" and "white" are attributes ; but psychologists agree that no attribute can exist apart from an object — apart from some thing. If " Man THE FOEM OF TRUTH. 23 is mortal " is not logically equivalent to " Man is a mortal thing," then whatever thing "is mortal" is not a mortal thing! But this is absurd. Therefore, " Man is mortal" is equivalent to " Man is a mortal thing." It remains now to show that the proposition, "A mortal thing is a man," is not necessarily true. If, then, mortal things are men, beetles and gnats, which are mortal things, are men. But this is absurd. Hence the hypothesis that the knowing thought runs from class to particular, as, " Mortal things are men," is false. Further : to say that the knowing act is from class upward, on the tree branches, to the more particular, is to say that the class is the particular — that mortal things are men; since the knowing thought is through the verb is, the copula. But to say that the class is the particular, is to say that the whole (the class) is identical with its part (the par- ticular), which is absurd. Therefore the thought is not from class toward particular, but ever downward in the science, or tree, system from particular to and through class. Finally, the same absurdity may be shown by the syllogism thus : — 1. Men are Bismarcks. 2. Froebel, Pestalozzi, Beethoven, Goethe, etc., are men. 3. Therefore, Froebel, Pestalozzi, etc., are Bismarcks. Jevons : " In * Dictionaries are useful,' it may be said that the substantive things or books is understood in the predicate, the complete sentence being, ' Dictionaries are useful books!" The Knowing Thought Thinks Particular into Known Class. — I have already proven on the authority of logician, physiologist, and psychologist alike, but especially on the authority of the common judgment of mankind, that the knowing thought thinks from particular to class — from the less to the more comprehensive ; that is to say, I have shown that the mental act of thinking, knowing, learning, etc., runs with the tree-system organism from branch downward to higher branch thus : — 24 TEE SYSTEM METHOD. Foxes are flesh-eaters ; not, Flesh-eaters are foxes. Napoleon is a Frenchman ; not, Frenchmen are Napoleons. Englishmen are Caucasians ; not, Caucasians are English- men. Tyndall is an Englishman ; not, An Englishman is Tyn- dall. Red pines are coniferae ; not, Coniferae are red pines. Pitch pines are gymnosperms ; not, Gymnosperms are pitch pines. Bananas are phenogams ; not, Phenogams are bananas. Pine-apples are endogens ; not, Endogens are pine-apples. Let the reader refer to the tree cuts opposite, and trace the eight propositions above downward on the branches. Now let it be remembered that a thing to us unknown is to us nothing ; and I shall be able quickly to demonstrate that no thought is even possible concerning a thing wholly unlike known classes. Let it be proved that a/l things totally unlike known classes are nothings, and it will then at least be seen that all things that are to us real things are like known class. Now all things are divided into (1.) animals, (2.) plants, (3.) inorganic things. Such unknown thing totally unlike all other things cannot be an (1.) animal ; for to be an animai is to be like animals. But it cannot be a (2.) plant ; since to be a plant is to be like plants. Neither can such unknown thing be any (3.) inorganic thing ; for to be an inorganic thing is to be like other inorganic things. Therefore, since this unknown thing, totally unlike every class, cannot be an (1.) animal, a (2.) plant, (3.) nor any inorganic thing, it can be no thing, and must therefore be nothing. It is thus demonstra- bly shown that no thought or thinking about things totally unlike known class, is possible ; for all thinking thinks the thing thought about into class, and thinking is possible only as it is a throwing of particular into class. To declare it true that foxes are flesh-eaters, Englishmen are Caucasians, is to throw or branch the particulars, or sub- BOTAMIOAIv "WORKI-TREE:. Copyright, 1886, by I. E. WILSON. [35] [26] ZOOIvOGICAIv WORKZ-TREB. (WITH GEOGKAPHICAL EXTENSION.) Copyright, 1886, by I. E. WILSON. THE FORM OF TRUTH. 27 ]ects, foxes and Englishmen, into the classes fiesh-eaters and Caucasians. To say that Tyndall is an Englishman is to throw, conceive, or branch Tyndall in the class EjiglishmeJt. This great trunk truth, that we know all things by classing them with other known things, is so elementary and so close to us that it is hard to be fully realized. It will be made to appear plain in " Reductio ad abs2irdnvi " below. Be the thing to be learned what it may, no human mind will ever fully. learn, know, or understand it till such mind is able to refer it, or class it, with what it already knows — class it with known class. That we cannot learn or know any new particular until we can connect it with something known, is a psychological truth as well established as that men are bipeds, or that cats are animals, or that we exist. It is a scholium of exact science. The Creator has said that knov.d- edge is by making the thing learned branch, as of a tree. (See Gen. 2 : lY.) And neither science so-called nor science real will ever learn or make a thing known except it be made known in the light of things already known, except it be branched into known kinds, or classes, of things. The "compare," "abstract," "generalize," '^ judge," "reason," " systematize," and " imagine " of psychology are absolutely nothing except a repetition of this mental down-the-tree act of throwing particular into known class. Everything becomes knowable by being branched into what is known ; and what is known, if known perfectly, or in system, is tree system in form. What of zoology is knowable is such by virtue of the fact that the great Organizer has created, or organized, all animals into a tree system of classes. " We are lost in the multitude of objects presented to our observation, and it is only by assorting them in classes that we can reduce the infinity of nature to the finitude of mind." — Sir Wm. Hamilton. But we could not assort "the infinity" of animals in " classes," except animals had first been created in classes. 28 TRE SYSTEM METHOD. And it is by virtue of the fact that the great Classifier has created the multitude of animals all in classes, that we can know them at all. All the millions of animals belonging in classes of certain characteristics, we learn the millions of individuals by learning the few characteristics of the classes to which these millions belong. "The essential matter in the first place [in studying animals] is to be quite clear about the different classes." — Huxley. Being "lost" in the "infinity" of animals, the student of zoology is compelled to assort them in classes, or branches, in order to learn, know, and remember them. But this is not all. He is further compelled to assort these first classes, or branches, into higher branches, and these again into still higher branches, and so on, until he has assorted, or branched, this " infinity " of animals into system, or science, — into science system. And this assorting of animals into classes, and branching of these classes into system, results, as we shall hereafter find, in a tree system of animal classes ; that is, a branched system in which every branch, or limb, is a class of animals : so that we not only reduce the " infinity " of animals to classes, but, carrying forward the same systema- tizing work, we reduce these classes to one tree-system whole. Thus, the learner is not only to throw, or branch, the " infinity" of animals into classes, but he is likewise to branch these classes into still higher class-branches, until he sees the whole multitude of animals in the unity of science [the tree) system, the process of learning ever being a branching process — a process throwing (J. e., mentally branching) particular or a lower class, into higher class. To illustrate, refer to the zoological tree, page 26. With weasel (2), dog (5), and tiger (10) for his lesson, learner proceeds upward through the class branches of the tree system thus : — (2.) Weasel= Vertebrate. Mammal. Carnivora. Mustelidae. Putorius. Ermineus. (5.) Dog == Vertebrate. Mammal. Carnivora. Canidae. Canis. Familiaris. (10.) Tiger ==Vertebrate. Mammal. Carnivora. Felidae. Felis. Tigris. TEE FORM OF TRUTH. 29 And so learner is to continue till he is familiar with the system of classes into which all animals are systematized. He is not to remember each individual animal separately ; for that is as impossible as it is opposed to system. He is to learn and remember animals just as animals are created — in class system, in a tree system of classes. And let it not be overlooked that it is the whole purpose of. science, just as it is the only possible work of science, (1.) to assort the multi- tude of individuals into including classes ; (2.) to assort (branch) these classes into higher, or including, classes ; (3.) and so on, till all the classes are branched into one single branch, or tree-system, whole. And let it be perceived that the work of science, or system, is by no means done zvhen the 'multitude of objects is assorted into classes, but that THE VITAL WORK OF SYSTEM, OR SCIENCE, CONSISTS IN BRANCH- ING THESE CLASSES, thus formed, INTO THE BRANCHED, OR TREE, SYSTEM. Why vital f — Because unless the classes be thus branched together into one tree-system whole, THE STU- DENT, OR LEARNER, IS LEFT WITHOUT A PROCESS OF WORK ; for learner proceeds up the tree. He is given animals, — weasel, dog, tiger, etc., — as his lesson ; for he must proceed from what he already does to what he does not know, since (1.) it is animal that he knows more or less already ; and (2.) since it is the classes, or tree-branches, of animals that he does not know and is to learn (or trace weasel, dog, tiger, etc., through) ; and (3.) since he must naturally first trace these animals through, and therein learn such classes as are easiest to learn or trace through. Therefore, learner must needs begin at the trunk of the tree and trace upward through the system. Evidently, since learner must trace his lesson of par- ticulars, — weasel, dog, tiger, etc., — through a class, or branch, system, he must needs begin tracing at the tree trunk, since it is impossible thus to trace a particular in any other way. He cannot trace downward till he finds where to 30 'Z'i^^ SYSTEM METHOD. Start, But to find where to start is first to trace a particular from tree trunk upward to the starting place. Because learner cannot go downward through the system till first he goes upward through it to find where to start, learner must begin by going upward. But since going upward is to trace the particular through all its science-system classes, and therein fully to learn it, therefore learner's process of work is upward through the system. And since going upward through a tree system is impossible and absurd until there exists a tree system through which to go upward, therefore learner's process of work is impossible until these classes are BRANCHED together into system. It is thus made plain that the most important work of science or system is, by all odds, not the first step of as- sorting the multitudinous particulars into classes, but the second step of branching these classes into science system ; that is to say, into tree system,, since the tree system is the only organic system in nature that will represent the classes arranged together into one whole. Thus it was in the dis- covery that every science system is a tree system, that the author discovered likewise the learner s process of work in learning — a thing before not at all understood. The tree is thus seen to be the learner's tool, or instrument, with which to learn — the long-sought-for educational guide. But we must return to the elementary truth out of which grew this discovery and invention of the learner's process of work and work-tree guide. Since the knowing thought car- ries the thing thought about to some known class, we should naturally presume that to fully know that particular, or thing, the mind would carry it to, or identify it with, all the classes to which its characteristics assign it. This is entirely true. Take, for example, the particular / in the word " let." (See the orthographic tree, page 18 ■) To detect such characteristic in /as assigns it to the class letter, is to be able to announce the judgment that / is a letter ; and in ascertaining this one char- TEE FOBM OF TBUTE. 31 acteristic alone, the mind has gone a long way toward perfect knowledge of /. But, by closer scrutiny, we find another at- tribute, characteristic, or class-mark in /that assigns it to the class sonant, as distinguished from aspirate. By another ex- amination, we find still another attribute, or class-mark, by which we class / with subvocals as opposed to vocals. And so on, till finally, by a minute inspection, we detect the peculiar characteristics, or class-marks, that render it a con- tijiuant, dental, lingual, and liquid, as distinguished from abrupt and coalescent, from labial, palatal, and guttural, from nasal and sibilant. It is thus seen that we know the particular / fully only when we have found in it six characteristics. It is seen, fur- ther, that these six. characteristics of / are simply six attri- butes, or class-marks ; that / is systematizable, or classifi- able, into the six classes (1.) letter, (2.) sonant, (3.) sub-- vocal, (4.) continuant, (5.) dental, and (6.) lingual and liq- uid, by virtue or means of its six characteristics ; that to find the characteristics, or attributes, of a thing is to find its classes, and vice versa. And who can deny that when we have found all, including the minutest and most insignifi- cant characteristics, or attributes, of a thing, we know and understand it ? Not, surely, any man born with human reason. Who, then, will deny or question that the knowing act is an act throwing, or conceiving, the particular unknown in known class ? Not, surely, any man not born with brutish reason. Who will deny or question that when we know all about Englishmen, all about Germanic peoples, all about Aryans, all about Caucasians, all about rneji, all about the higher classes, mammal, vertebrate, organic, natural, qualita- tive, and finally, thing, and all about all other classes into which Tyndall may be thrown, — who will deny that we un- derstand the particular Tyndall } Not, surely, any man who knows that we know a thing when we know it. Psychological demonstration. — What is true has the 32 THE SYSTEM METHOD. form of truth. We may, therefore, be certain that the best possible illustration in developing a clear conception of the act of knowing, will be that crucial criterion, the tree-system organism. See the figure, page 34. Let the reader con- stantly return to this figure as the explanation advances. The child sees the oak tree, the great stone, the dog, and the house standing together. At an early age it is able to perceive the tree, say, as a different object from the others. This perception, while very vague at first, is nevertheless a recognition of some characteristic, or mark, that renders the oak tree a different thing from the stone, the dog, and the house. Let the reader wake widely to the truth — quite plain when seen at all — that there is no perception to the child until or except it perceive the tree as a different thing from the other objects. But to perceive the oak tree is to con- ceive a particular as belonging to a class — to conceive the particular oak tree as belonging to the class thing. It may be objected that the child does not perceive the tree as being a thing (that is, as belonging to the class thing) ; to which objection it is an overthrowing argument to reply that the objector agrees that the child does actually perceive the tree. But if the child does actually perceive the tree, he must perceive it either as a thing or as not a thing ; that is to say, he must either perceive the tree as a thing or as nothing ; that is to say, if the child perceives the tree at all, as the objector agrees he actually does, he must perceive it as a thing. But to perceive that the tree is a thing is to perceive that the tree belongs in the class thing, be that class as vague to the child as it may. That the judgment " Tree is a thing," runs from particu- lar to class, has already been immovably established ; nor can any argument to the contrary vitiate or disturb that truth, not more than the play of thistle down can overturn Bunker Hill monument ; not more than a cricket can kick the earth out of its orbital path. Mark well that the child THE FORM OF TBUTE. 33 does actually perceive the tree, as all agree. Mark especially that the child perceives the tree as either a thing or as not a thing — either as belonging to the class thing or the class nothing. But to say that he perceives the tree as belonging to the class nothings is to say he perceives nothing. And to say he perceives nothing is to say he does not perceive. Since, therefore, he does actually perceive the tree, he must of necessity perceive it as a thing — as belonging to the class thing. For, if he does not perceive the tree as a thing, he does not perceive it at all. Hence the grand consummate truth. We can know nothing, cannot even perceive a thing, except as we identify that thing with some known class, however vague such class may be to the mind ; that is to say, all knowable or perceivable things are knowable and perceivable by virtue of the possibility of a mental act iden- tifying, or connecting, them with known classes ; that is, nothing whatever can be known except through a mental act classing it with known things. Having now shown that the very first perception, the first knowing or learning act of the child, is throwing, or branch- ing, particular into kind, or class, let us observe how all knowing or learning is but a repetition of the same branch- building act. The child already knows tree in the light of the class thi?ig ; but he now observes an ash tree. He then takes two steps (unconsciously, of course) : He says, (1.) Ash is a thing ; (2.) Ash is a tree. He is now beginning to learn or know — getting certain things known in mind by means of which he may hereafter learn to know the unknown things. Let him who would teach by following science sys- tem ; let him who would guide the learner as the great Brancher guides him, — let him observe how from its birth the child is a science-system, that is, a tree-system, builder. Again, the child observes a clump of trees. His mental acts of learning are now, (1.) This is a thing ; (2.) It is a tree ; (3.) It is an oak ; and (4.), as compared with the others in the 3 34 THE SYSTEM METHOD. clump, a red oak. And so from thing to v/hite oak and burr oak ; and so to ash, mountain and white. Arid so by branch- ing particular in known class, in all knozuing. Thus the act of systematic knowing or learning may be defined to be an act finding all the classes of the particular to be learned, beginning with the tree-trunk branch thing, and going up- ward through the tree-system organism, our unconscious guide. Through this class-system organism alone, we can develop the truly scientific, organizing, or system, method of teaching and learning ; for by it alone can we determine either what the student's process of work is, or what the system whole of any branch of learning is. But whether we have a scien- tific, that is, a system, or truly organizing method, or whether we have a method unscientific and disorganizing, will be easy to determine by projecting this tree-system organism into the subject. If thereby the method to be tested is seen to be the same in process as going upward through this tree system, it is the true, or system, method. If it is different from this system-guided procedure, it is therein unsystematic, disorganizing, difficult, and false or indirect. In this way, for instance, it will be shown that the current text-book methods of solving arithmetical problems are sadly lacking TEE FORM OF TRUTH. 35 in system, and that, therefore, they are not only difficult and confusing to learn, but soon to be forgotten outside of the school-room. It will also be shown that all arithmetical problems, when easily, certainly, and systematically solved, are solved by one and the same method, by a simple repeti- tion of the three-step solution. It may also be shown by this tree-system test that the rules and definitions of the ordinary grammars are generally vicious and often even false. Authority. — Schuyler: "However different two classes neither of which is the highest class, both are subordinate to the same higher class, though not necessarily co-ordinate with each other ; for, ultimately, every class may be referred to that of being [or thing], the highest class. Hence, things the most dissimilar must, in certain respects, be similar^ — Law of Homogeneity. " Every class contains other classes under it. In thought, therefore, THE divisions of classes give classes, not individuals. Hence, things the most similar must, in certain respects, be dissimilar. Thus take any two classes with a small difference. Now, this difference can be divided, thus giving new classes distinguished by this partial difference ; and so on, ad infijzitum." — Law of Heterogeneity. Mr. Schuyler wishes to say that things are similar because classifiable — similar by virtue of their having been created in classes ; which is identical with the grand elementary truth that things are knowable only by being classed. (See the cuts, pages 25 and 26.) Were Spencer and banana, for instance, classifiable in no respect, they could be similar in no respect. Granting it to be known that the banana is a thing, were it not that Spencer is classifiable with banana, Spencer would be a nothi7tg. Spencer is thus to us a thing only by virtue of this elementary truth that he is classifiable with other known things, — with the banana for one thing. Of Spencer and the banana we may say, — 36 THE 8YBTEM METHOD. 1. Spencer belongs in the class thing with banana. 2. Spencer " " " qualitative " 3. Spencer " " " natural " 4. Spencer " " " organic " And could we not know Spencer as belonging in, at least, the class thing, we could only say, Spencer is nothing; that is, Spencer is not a thing. Thus do we know that Spencer is like, or similar to, banana and other things by virtue of his being classifiable with such other things. But it can be easily shown that all things knowable are knowable by virtue of their being similar to known things. This last proposition proved, and it will be plain that anything knowable is know- able by virtue of being classed with known class. To understand this truth is easy. Let us undertake to know, learn, or get knowledge of, a thing not similar in some fespect to some other thing known. (Refer to the tree, page 98.) The thing unknown and to be learned cannot be a man ; for man is a thing known. It cannot be an animal ; for animal is a thing known. This unknown thing cannot be a plant ; for plant is a thing known. Finally, this unknown thing cannot be a thing; for that, too, is a thing known. But since the thing of which we are trying to get knowledge, cannot be a thing, evidently it is nothing. And of what is not a thing, — of nothing, — we can have no knowledge. Of what, then, is not similar to some other thing known, we can have no knowledge whatever. But the only thing of which we have absolutely no knowledge, is nothing. Of what, then, is not similar to some other object known, to us there is nothing ; that is to say, things are knowable only as they are similar — knowable by virtue of their being similar. We have therefore, finally, the following syllogism :^ 1. Similar things are to us similar because classifiable. 2. Knowable things are to us knowable because similar. 3. Therefore knowable things are to us knowable because classifiable. THE FORM OF TRUTH. 37 It may also be perceived in this connection that the great elementary fact that all things are similar in some respects, and yet dissimilar in other respects, is the ultimate basis of all knowledge. Similarity may be called the hinge by means of which we know. Schuyler: " Thought is the recognition of one thing under or in another as a species [particular] under its genus [class.] " This is an unqualified concession that every knowable species or particular has a corresponding class ; that all par- ticulars are created, and therefore found, in classes. But none Vv^ill deny that thought is a knowing act. What Mr. Schuyler really says, therefore, is that the knowing act, called thought, is the recognition of one thing as belonging in a class — the recognition of things as they were created ; namely, in classes. Again : " An object is defined by (1.) referring it to the class immediately containing it, and (2.) by distinguishing it from other objects of the class by means of its differential [special or individual] quality." Now refer to the orthographic and zoological trees, pages 18 and 26. Siibvocal, for example, is defined by saying, (1.) that it belongs in the class sonant, or voiced sound ; (2.) that it belongs in the class obstrncted sound. So man is defined by saying, (1.) that it belongs in the higher class mammal; (2.) that it belongs in the class erect, two-handed animal. But to define a thing is to make it known precisely as it is ; therefore, to render a thing known or knowable precisely as it is, is to refer it to known classes. In other words, the knowing act is an act throwing the thing made known into known class. Again : " The highest genus [class] is not contained under [in or by] a higher genus [class], nor has it a differen- tial quality [class-mark]. It cannot, therefore, be defined." What Mr. Schuyler wishes herein to say is simple ; 38 THE SYSTEM METHOD. namely, there is no class higher, or more extensive, than the highest class, thing. This highest class, thing, cannot, there- fore, be satisfactorily defined ; that is to say, thing (as thing merely) can be only vaguely known ; because there is no higher class into which we might throw it in order thereby to make it known ; since it is by thus throwing a thing into known class that it can be defined, or made known. Aristotle: "Whatever may be affirmed or denied of a class [of particulars] may be affirmed or denied of whatever [particular] comes under that class." This famous dictum of Aristotle is but the recognition that all things are created in, and knowable only in the light of, their classes. Men, as a class, are two-handed. Knowing this, we conclude that every particular, — Bismarck, Glad- stone, Victoria, etc., — is two-handed. Why 1 — Because every particular is known in the light of a class. Hopkins: "A logical definition always consists of the genus, that is, of the class above, and the specific difference. Thus, Cato is a man, with the specific differences that make him Cato. A man is a mammal, with the specific differences that make him man. A mammal is an animal, with the specific differences that make it a mammal, and so on till we come to being, or thing, which cannot be defined, because, as is commonly said, there is no higher genus above it." To make a thing known is to make known not what it has, but what it is. Accordingly, Dr. Hopkins says that man is defined, or made known, by declaring, not that he has, but that he is, a mammal. But to declare that man is a mammal is to declare that man belongs in the class mammal; there- fore, man is defined, or made known, by his being referred to known class. Wayland: "Judgment is an act of the mind in which we affirm that a particular individual or species is included in a particular genus, or class. Thus, I judge snow to be white, grass to be green, avarice to be contemptible ; that is, I judge THE FORM OF TRUTH. 39 these particular individuals to be comprehended within the class which I predicate of them." " We think in judgments ; that is, we are always afhrming one thing of another, a7zd we do not consider anything else to be thinking. To conceive of things without forrning judg- ments is to make no progress [in knowing, learning, or knowledge.] " This is equivalent to the declaration that the repeated act by which we make progress in knowing, or learning, is an act " affirming that a particular individual or species is included in a particular genus, or class." Again : " The syllogism is not a mode, it is the mode, of reasoning. It is the peculiar process of the reasoning faculty." But " a syllogism is a series of judgments, the last of which affirms the conclusion at which we have arrived." Thus does Dr. Hopkins — good authority — virtually aver that all our conclusions (and it is the business of life to make conclusions) are reached by repeating the act of judgment — an act, as he himself declares, affirming that a particular is included in known class. Miscellaneous. — The practice of authors vying with each other in making the most perfect and exhaustive classi- fication, each in his chosen subjects, — this for the purpose of rendering both part and whole easily knowable, — is a testi- mony too conclusive to be overlooked ; nor is the evidence of language, the material dress of this down-the-organism- from-particular-to-class act, less conclusive. And what an interesting hint in such casual expressions as " branch of knowledge" (knowing)^ When we have gotten our feet upon this vicious, knowledge-destroying doctrine of ours, that education lies in a thing called knowledge, and have realized that education in all branches whatever (like educa- tion in writing, reading, talking, doing) lies in the act (mark it, the action^ of knowing,— a mental act directed by the organism of the Creator, — then we shall be able to see that 40 THE SYSTEM METHOD. our concession, "branch of knowledge," is an unwitting acknowledgment, not only that the knowing act discourses from particular to class in the organism of truth, but that that same organism, heretofore unknown, is the necessary copy by which to test or judge the merits of all educational schemes. Reductio ad absurdum. — Let it be supposed that the knowing act is not an act organizing particular into class, and that it is true that a thing may be known, or perceived, without reference to class. Take any object as the thing supposed to be unknown and yet unperceived, say the fells domestlca. To perceive that this fells doniestlca is a flesh- eater, is to perceive it as belonging in a class ; but that is for- bidden by the hypothesis. To perceive it as a mammal or a vertebrate or an animal, is likewise to perceive it in the light of a class, which is forbidden by the hypothesis. Further, to perceive it as an organic being, or thing, is to perceive it in the light of a class. And, finally, to perceive it as a thing is to perceive it as one belonging to a class — the class tJilng; but to perceive it as belonging to any class is forbidden by the hypothesis. We cannot, therefore, perceive or know it either as mammal, animal, or thing. But what we do not know, or perceive, as a thing (that is, something), we know, or perceive, as not a thing — as nothing. We do therefore know and perceive this thing as notJiuig; that is to say, we can knozv, or perceive, nothing zvlthoiit reference to class. Therefore, the doctrine of the hypothesis is absurd. It is exactly parallel with the hypothesis that something is nothing — that a thing is not a thing^ The Mental Result of Learning, or Thinking, any Branch is a Branch System, i. e., a Tree System. — The reader should now keep in mind that all men are somewhat lost in the multitude of the objects presented to our intelli- gence, and that it is because we are thus lost that we educate ourselves ; so that all education begins and proceeds by sys- THE B RANGE SYSTEM. 41 teniatizi7tg. " We are lost in the multitude of the objects presented to our observation, and it is only by assorting them in classes [say branching them through their branch system] that we can reduce the infinity of nature to the fin- itude of mind." — Sir Wm. Hamilton. Now, this "assorting" the objects, with which any branch of learning has to do, into "classes," is called learning them through science, or system. And all these " classes " taken collectively are called, or are to be called, a science system. All scientists have taught that these classes taken collectively form a science, or system ; but they have never yet discovered WHAT system a science system is. Of systems there are nu- merous kinds, — animal, plant, and inorganic systems ; solar and stellar systems ; political, religious, and industrial sys- tems ; digestive, circulatory, nervous, osseous, and muscular systems ; river and lake systems ; and so forth. The question to be solved is not whether a "branch of learning" is a sys- tem, but what system is a branch of learning. Ueberweg: " Science is a whole of knowledge in the form of the system." "Science has its true existence only in the systematic [say branch-system] form." A " whole " what? In the form of what system .■' That is the great educational problem. Bishop Butler: "A system [science system] is a one or a whole made up of several parts." A "one or a whole" what .-• — not whether a system is a one or a whole, nor whether a science is a system, but what whole and what system is a science .'' Neither has the question been answered by any. Herbert Spencer : " Knowledge of the lowest kind is ununified knowledge ; science is partially unified knowledge ; philosophy is completely unified knowledge." "Unified" into luhat unit? Sir Wm. Hamilto?i: "Generalization (knowledge of sys- tem) is only the apprehension of the one in the many.'" Ap- prehension of the "one" tvhat in the many objects .■* 42 THE SYSTEM METHOD. Anaxagoras : "The mind only knotvs when it subdues its objects, when it reduces the many to the one." Reduces the many to the " one " w/z^s:/' f There can be neither a "one" nor a " whole " nor a " system " that is not a one, a whole, or a system, of something, or some kind. " The Platonists: All knowledge is the gathering up into one, and the indivisible apprehension of this unity, by the knowing mind." — Hamilton s Metaphysics. Gathering up into a " one " what ? is still the unanswered question. " Plotimis : Our knowledge is perfect as it is one." — Id. " One " what ? "■Leibnitz, Kaiit, Aristotle, have defined knowledge by the representation of multitude in unity." — Id. The upshot of these testimonies is that science is a "sys- tem," " a one and a whole." Bishop Butler declares that a science is a " one and a whole," but does not tell us what that one or whole is. Neither does Ueberweg see what sys- tem the perfect science system is. Discerning that science is a "whole," and that the form of that whole is some system, he still fails to divulge what system, what kind of system. I repeat : the great problem is and has been, not whether a science is a " one," a " whole," a " unified whole," " unity," or "system," but what kind oi z. one, whole, unity, or system is a science } Is a science an animal system .? a planetary sys- tem 1 a landscape system .? a tree system .? or something else } Now, the key with which to unlock this difficulty — the time-worn problem of the schools — is the elementary princi- ple that all learning, or knowing, is the simple recognition of particular as belonging in some class. This truth, that learning or knowing any particular consists in passing that particular through its classes, I have demonstrated and set at rest, showing that it rests on the same foundation as the self-evident truth that a thing is and must be a thing, and cannot be nothing. THE BRANCH SYSTEM. 43 We have before us, then, two plain truths, by the concur- rence of which, the author of "The System Method" first discovered zvhat system a science system is, and therein the student's process of work and the true method of teaching, so far as science, or system, is concerned : — 1. Learning a thing is. learning the classes to which it belongs. 2. Learning a thing is learning the science system to which it belongs. The first proposition I have demonstrated over and over again. That the second is true, we have the universal testi- mony of the learned. Lideed, our authors vie with each other as discoverers of system with which to explain the multitude of objects to be explained under any system, or science. Since, therefore, /is known in the light of the sci- ence system of orthography to which it belongs ; and since it cannot be known except through a knowledge of its classes, it must needs be that knowing, or learning, / by means of science system, is to learn it through its classes. Therefore, the science system to which / belongs must be a system of classes — a system whole of which the classes to which / belongs are the parts. All agree that / is learned, known, and remembered, when the science system of orthog- raphy is learned, known, and remembered. I have shoAvn that /is learned, known, and remembered, when its classes, — letter, sonant, subvocal, continuant, dental, etc., — are learned, known, and remembered. I have proved that we cannot even think of / except as belonging to some known class. It follows, therefore, that thinking /in system is, or includes, thinking / through its classes. It is now evident that the science system to which / belongs is a system of classes. Similarly, do the scientists agree that, in the sentence whole " I go," / is fully known when the science system of English grammar is known. And I have shown that / is known when its classes, — ivord, fiuidamental, siibstantive, 44 THE SYSTEM METHOD. pronoun, nominative, essejitial, etc., — are known ; so that knowing the science system to which / belongs, and know- ing the classes to which / belongs, amount to the same thing. Hence, learning the science system of / is practi- cally learning the classes of /. In this way do we again conclude that science system is a system of classes. Similarly, still, learning the zoological system to which horse belongs is equivalent to learning the classes to which horse belongs ; namely, animal, vertebrate, mammal, iingn- lata, equidcB, equus, caballus. And so, to become acquainted with the classes to which zvhite pine belongs ; namely, plant, phenogam, exogen, gymnosperm, cojiifercB, pine-apple, pinus, strobus, etc., is to become acquainted with the botanical sys- tem to which white pine belongs. From such considerations and a practical application, after being satisfied that learning a thing is finding its classes, the author quickly saw that a science system is a system of classes — a system of which the parts are classes of the system whole. We have already seen that a science sys- tem is a " one or a whole made up of several parts ; " and the fact that the parts must be classes furnishes a clue to what the whole must be. For of parts that are classes, the including whole can be nothing but a higher class ; just as of parts that are limbs, or branches, the including whole can be nothing but a higher limb, or branch. Thus a system of classes is seen to be a branch, or tree, system ; since both the class divided and the divisions are classes ; since both the including whole and the included parts are alike, each being a class. (See the orthographic tree, page 18.) Ob- serve that the class letter, when divided, gives classes, sonant and aspirate. And so the class sonant, subdivided, gives classes, vocal and subvocal. And ever so : class divided and subdivided is class branched and sub-branched, making the class system a branch system, or tree system ; so that to learn a letter by tracing it through its classes — through the THE BBANGR SYSTEM. 45 class system to which it belongs — is to trace it through a tree system, the branches of which are the classes to which such letter belongs. Tracing a letter through the ortho- graphic tree system, shown in page 18, is tracing that letter through its classes ; and since tracing a thing through its classes is learning it, to learn a particular is to trace it through the tree system to which it belongs. Thus it was that the author discovered that a science system is a tree system. Hence the invention of the " Learner's Work-Tree " to show luhat system a science system is, and to provide the student the true process of work in learning. Mark well that tracing a particular through its classes is finding the attributes of that particular, which assign it to such classes ; so that tracing a particular — say the letter / — through the tree system (see page 18), traces / through its classes and therein through its attributes. Since the classes letter, sonant, subvocal, continuant, dental, are the attributes of /, to trace / through those classes as branched into tree system, is to unravel, or loose, its attributes, which Sir Wm. Hamil- ton, Herbert Spencer, and others declare to be the true process of learning. This process of learning by tracing a thing through its classes as branched into tree system, is learning by true analysis — analysis guided by science-system. The following plain truths show that the system whole of any branch of learning, is a branch, or tree, system : — 1. The tree system is the only system in nature in which every part taken alone is itself a lesser whole of the same features as the including, or great, whole. Every branch part of any higher tree-branch whole possesses all the essen- tial features of such higher, or including, whole. Any divis- ion, or part, of a branch whole is itself a lesser branch whole, no less complete than the greater branch whole of which it is a part. So that either the branch whole or any of its parts is alike branched, or linked, into a branch. Now it is be- 46 THE SYSTEM METHOD. cause every class system {i. e., science system) is just such a branch system, — the class whole and each of its class parts alike being branches of essentially the same features, — that tracing a thing to be learned through its class system [i. e., science system), traces that thing through a tree system. And it is plain because every science system is such a tree system, that the tree system constitutes the only possible representative, or map, of such science system. How neces- sary to an understanding of the systems of orthography, grammar, etymology, botany, etc., how invaluable in all study of these branches, is this tree representative of the science wholes of these branches, will be shown farther on. Suffice it here to say that, in all study of orthography, gram- mar, etc., it is as indispensable — not more so, not less so — to keep before the eyes and mind the tree-system Avholes of these branches, as in the study of the backbone or Iowa it is indispensable to keep before eyes and mind the skeleton whole or United States map whole. 2. The tree system whole is the only system in nattire like the whole of any branch of learning. This long-sought-for demonstration of what the whole of a branch of learning is like, see in the trees themselves, pages 18, 25, 26, 27i, and 494. Do you not there behold the subject of orthography in the likeness of a tree .-^ Are not the very branches of the tree named by the very names included in the nomenclature of orthography .'' If the reader has eyes, does he not there see that the subject of orthography, taken as a whole, is so like the tree, taken as a whole, that the tree actually repre- sents the subject } But if orthography as a whole is pre- cisely like a tree as a whole, then orthography cannot be like anything different from the tree. Therefore there is absolutely nothing except the tree that orthography is like — absolutely nothing except the ensemble, or organization, of the tree that the ensemble, or organization, of orthography is like. Therefore, absolutely nothing except the tree can TEE BRANCH SYSTEM. 47 be a Irue guide in either knowing (that is, constructing, organizing, or doing the organization of) orthography, or in remembering (reconstructing, or reorganizing) orthography. That the ensemble of arithmetic, grammar, botany, and zool- ogy, even of all knowing, is like the tree, see for yourself on pages 26, 98. Therefore, not only is every branch of knowledge like a " tree [a branch] of knowledge," but in all learning and in all remembering, that "tree of knowledge" alone is the true guide. Let the reader undertake to prove to himself that the efisevible of orthography, arithmetic, grammar, botany, or zoology is like some natural and familiar object different from the tree, — say some animal, building, landscape, cloud, etc., — and he will bring about such a humility on his own part as will enable him heartily to indorse as fact what his eyes see on pages 18, 25, 26, and 494 ; namely, that the tree is the only familiar object having Bacon's " root in nature," to which the ensemble of any knowable branch of learning may be compared. For instance, compare the ensemble of orthography as represented by the orthographic tree, page 18, to an elephant. What organ, or limb, of the elephant will represent the tree-trunk letter? Will it be the elephant's fore leg, hind leg, ear, tail, or proboscis } By the time the reader has determined what organ, or limb, represents the tree-trunk letter (see page 18), and then has found v/hat the three branch organs, limbs, or parts of that trunk letter are which represent the three branches "voiced," "whispered," and " aphthongs," I am persuaded that he will realize that he has not only a tail, a fore leg, and a proboscis, but that he has indeed an "elephant," " on his hands" ! Plainly, an elephant cannot represent the ensemble of any subject whatever. And, for the same reason, no object known, ex- cept the tree, which is itself a "branch," can represent the ensemble of any branch of learning. Therefore, the tree, which in its ensemble is like the ensemble of any branch of 48 THE SYSTEM METHOD. knowledge, is alone the true guide in learning, and the true reminder in remembering, any branch of knowledge. But " whatever may be affirmed or denied of the whole may be affirmed or denied of every part contained by the whole," is a principle of psychology disputed by the judg- ment of no man. Since, therefore, the only possible guide in learning and remembering the ensemble, or whole, of a subject, is the tree, that tree is likewise the only possible guide in learning and rem.embering any part or branch of such subject. The reader will be able to see plainly this truth for himself by referring to any one of the tree cuts, say that of orthography. Take, for example, the branch siibvocals, including w, y, ng, r, zh, z, th, n, 1, v, m, g, j, d, and b. Now, to understand these sounds as subvocals is to understand them (1.) as being voiced, or intonated, sounds, (2.) as not being whispered sounds, (3.) as not being aph- thongs, and finally, (4.) as being obstructed voiced sounds as distinguished from unobstructed voiced sounds, i. e., vocals. But thus to understand the subvocals is to understand them as a branch of the tree of orthography. Since, therefore, we can fully understand subvocals only as a branch of the ensemble of orthography, as represented by that which contains the brancJi, namely, the tree; and since we cannot remember what we do not understand, we can fully remember subvocals, therefore, only as a branch of the orthographic tree — only through such tree as guide and means. 3. The tree system is the only familiar system in nature having an indefinite number of parts. (Refer to the cut on p. 26.) Now, I assume that the reader knows, what is no- where denied, and what I have repeatedly proven, (1.) that every known thing is known in the Hght of its organized classes (see pp. 23, 24, 27, 28); (2.) that anything is perfectly known when, but not till, identified with all the organized, or known, classes to which all its attributes, or class-marks, assign it (see pp. 30, 31, 43, M, 49) ; (3.) that THE BBANGE SYSTEM. 49 everything not known to us in the light of some class, is to us a non-existence, a nothing. (See proof of this third proposition on pp. 36, 40). Take Ha// of the cut, p. 26 ; and now to the proof: — Hall is to us scientifically known as belonging (a) in the class being, or thing ; (b) in the class orga7tic, as opposed to (1) inorganic ; (c) in aniina/s, as opposed to (2) plants ; (d) vertebrates, as opposed to (3) articulates, (4) mollusks, (5) radiates, (6) protozoans ; (e) mamma/s, as opposed to (Y) reptiles, (8) birds, (9) fishes ; (f) man, as opposed to (10) hoofed animals, (11) monkeys, (12) flesh-eaters, (13) plant- eaters, (14) whales, (15) bats, (16) insect-eaters, (lY) gnaw- ers, (18) edentates, (19) marsupials, (20) duck-bills, (21) etc. ; (g) Caucasian, as opposed to (22) American, (23) Mon- golian, (24) Ethiopian, (25) Malay ; (h) Indo-European, as opposed to (26) Semitic, (27) Hamitic ; (i) Germanic, as opposed to (28) Romanic, (29) Celtic, (30) Slavonic, (31) Brahmanic, (32) Iranic ; (j) Eng/ish, as opposed to (33) German, (34) Swede, (35) Norwegian, (36) Dutch, (3T) Fleming, (38) Dane. Thus, to perfectly understand Hall is to see him at least in (a to j=)10+38=48 organized classes, not to 'speak of a multitude of other classes — as, honest or dishonest, niortai or imniorta/, earnest or trifling, active or s/othfu/, etc., etc., — entering into every comprehending, or knowing, conception of Hall. But, to be charitable with the objector who cannot yet see the plain fact that the tree alone is the guide in all know- ing or learning, and in all remembering, grant that Ha// is perfectly understood when viewed in relation to 48 organ- ized classes only. Let the reader now ask himself what natural object there is in the knowable universe, except the tree, that has even the 48 branches, or limbs, by means of which these 48 classes can be associated together as mutu- ally dependent parts of one great whole. What object be- sides the tree can have 48 successively subordinated branches } 4 50 THE SYSTEM METHOD. There may be some curious unknown worm or sea-dragon with just 48 legs. But who has heard of even an unknown sea-dragon whose legs branch out of each other! as does the twig out of its limb, twig and its limb out of a higher limb, twig, its limb, and their higher limb out of a still higher limb, and so on as in the tree, which does (as shown on page 26) so branch and re-branch indefinitely as to show the exact likeness of either these 48 branching and re-branching classes or any other indefinite number of classes ? The tree is the only familiar branching thing in the natural universe that has an indefinite number of branches, limbs, or organs. Therefore, the tree is the only familiar object in the natural universe that will represent, or constitute a map of, the parts of any branch of learning as organized, or associated, in one whole. And who will deny that zuJiat is the ONLY POSSIBLE MAP of any branch of learning, is not, FOR THAT REASON, the only possible guide in learning or remembering that branch of learning ? Were there in existence only one map to guide the learner in the study of these United States, but that were perfect, what fool is it that would deny that single map, or guide, to be the only map, or guide, in learning and remembering the topography of these United States.'' What great educator ( ! ), then, will be the first, after the publica- tion of the discovery of the tree as the only possible guide^ or map, in learning and remembering any branch of learning, to denounce the discoverer as a dangerous latitudinarian, seceder, or Latter-day Saint .'' Who, then, is unable to see that the long-sought-for guide in learning and remembering is that which alone, of all nature's products, has an indefinite number of branches, the tree " of knowledge " .-* Upon him it devolves to show that there exists some other object, not only a natural (visi- ble and tangible) product, but an oh]&ct fa^ni liar to mankind, that has an indefinite number of organized branches. When- ever such objector to the organized tree, the true test and THE BRANCH SYSTEM. 51 guide of all organized knowledge and knowing, shall have accomplished his task, he will have proved that there exists a natural object familiar to us all, — say a c/ncken, for instance, — having an indefinite number of branches or limbs ! — a biped familiar to us all, hatched without any definite, but always with an indefinite, number of legs ! Equivalent to the objector's wisdom is the learning of him who should declare that a biped is a five-footed animal without any definite number of feet ! And I challenge any philosopher, American or European, to write a single page against the Creator's tree "of knowledge," as the only true guide in all knowing and remembering, without committing himself to some just such indefinite, and therefore infinite absurdity. I have now established beyond any possibility of error, the grand elementary, science-system-revealing truth that all things are created in branch classes, and that, for this reason, the act of knowing is an act branching particular into class. Knowing or learning or thinking is branching. Knowing is copying the Creator's branch-system organism of truth, of which the tree is the sensible type ; and it is as necessary to him who would learn to know, that he see clearly the organism guide, as it is necessary to him who would learn to write, that he see clearly his written spec- imen, or guide. The specimen is guide to the writer ; the organism is guide to the knower, organizer. Have I not proved that knowing is branching — branching particular through its classes in the form of tree-system branches ? Are not any one of the many proofs already given so solid as, like the solid earth, to furnish ample footing for all men to believe ^ Have I not proved, if knowing, thinking, etc., are not branching particular through known class, that a thing- is nothing? But if knowing is branching, what is branching but copying or imitating or doing the branch, just as writing is copying, imitating, or doing, the written copy .'' " But if perfect method requires us to follow nature's 52 THE SYSTEM METHOD. tree-system organism as guide, the schools are in a sadly defective condition," exclaims the reader ; " for neither Aris- totle, nor Montaigne, nor Comenius, nor Locke, nor Bacon, nor Rousseau, nor Pestalozzi, nor Jacotot, nor Spencer, nor Mann, nor any on this planet, has ever shown us what such science-system, or working-process, organism guide is — not even in those 'common branches,' a fair knowledge of which they hold as necessary to ordinary intelligence in the affairs of life." In such conclusion agree all the educators men- tioned. " A great part of the learning now in fashion, ... a gentleman may be unfurnished with, without any disparage- ment to himself or prejudice to his affairs." — Locke. " We learn, not to live, but to dispute ; and our education fits us rather for the university than for the world." — Id. " Schools have failed ; and instead of teaching the foun- dations, relations, and intentions of all the most important things, they have neglected even the mother tongue, and confined the teaching to Latin ; and yet that has been so badly taught, and so much time has been wasted over grammar, rules, and dictionaries, that from ten to twenty years are spent in acquiring as much knowledge of Latin as is speedily acquired of any modern tongue." — Comenius, as quoted from. " Educational Reformer sT " Take the road directly opposite to that which is in use, and you will almost always do right." — Rousseau. "We [school-teachers] only toil and labor to stuff the memory, and in the meantime leave the conscience and the understanding unfurnished and void. And as birds who fly abroad to forage for grain, bring it home in their beaks without tasting it themselves, to feed their young, so our pedants go picking knowledge here and there out of several authors, and hold it at their tongue's end only to spit it out and distribute it amongst their pupils." — Montaigne. " Yet it is the custom of school-masters to be eternally EDUCATION DEFECTIVE. 53 thundering in their pupils' ears, as if they were pouring into a funnel, whilst the pupils' business is only to repeat what the others have said before." — Id. And similarly the others. Bacon pointed out why phi- losophy and education were so unfruitful in his time, by saying that they had lost their " root in nature." But he does not tell us definitely what that root in nature is. So M. Marcel says the "method of nature" is the archetype of all methods. But what is the "method of nature".? What definite guide is there by which to proceed when following the " method of nature " .'' for until we know some guide definite showing us what this " method of nature " is, such very valuable( .'') "method of nature" must remain to us practically a valueless method. M. Marcel gives no hint what the guide is in proceeding by this " method of nature." Mr. Spencer, too, has found that all improvement is in con- formity to the " methods of nature." But " methods " should have been put in the singular, method ; possibly, diversified method. Now, what this " root in nature " and "method of nature" is, the author has discovered in the tree-system and working-process organism or guide — the Creator's device, not man's. Knowing is a branching, or organiziitg, act — an act branching particular to be learned through the science- class, or class-branch, tree. Knowledge is not store in mind, but exclusively skill in the branching, or systematizing act. And there is as much difference between stored-in-mind knowledge and skill-in-branching knowledge as between a man who could tell weight, size, and pedigree of every fish in the sea, and yet be outmanned by a minnow, and a man who could organize a successful scheme for catching and overpowering sea-dragons. Man's superiority over other creatures lies in his branching, or organizing, power. This opportunity ought not to be neglected to say that until educators shall turn their eyes away from that selfish Ego of psychology, and know by what is knowing, — imitating 54 THE SYSTEM METHOD the Organizer's organized, or branched, copy, — that is, until men get their minds off from self and on the organisms of nature, both education and liberty must despair of triumph. "Be not wise in thine own eyes." " Wisdom is a tree [not yours, but the Tree-Maker's] of life." Man is at best an imitator, not a resolver or reader. The best imitator is he who, because most forgetful of self, is most worshipful of his copy. A man can write well only when he forgets what curves he makes. We Learn the Branching, or Organizing, Act only by Doing It. — Every act is to be learned by doing that act. Mankind have ever practiced this truth in the daily activities of every individual ; nor has there ever been any theoretical contention or disagreement about its truthfulness. To some, the necessity of vital doing has seemed plainer than to others, as we shall presently see. Withal, every babe has learned the act of talking by talking , the act of walking by walking ; printing by printing ; writing by writing ; running by running ; sewing by sewing , swimming by swimming Of practical activities learned only by doing, I might copy from our census by the page. It is a proverb old as the years, that we cannot be certain that we know how to do an act till we have done it That learning is a mental act, and that every act is to be learned by doing that act, is realized by all, by pupil as well as by teacher The great question m educational matters is not the simple inquiry whether we learn to talk by talking, to write by writing, to do by doing, but, What is to be done .? All agree that, in order to learn, we are to do something; the question is, just what } Just what shall we do to learn orthography } Just what shall we do to learn arithmetic.'' geography.? grammar.'' history.? physiology .? To suppose that we are to go through the mental activities required of us in learning, without doing these activities, is to suppose we can go through an activity without going through it ! All this ado about " learning to LEARNING BY DOING. 55 do by doing," savors immensely of the wisdom of the doctrine that doing is doing, learning is learning, your nose is your nose ! That we learn "to do by doing" is not only universally accepted and understood, but is likewise universally prac- ticed; Who has learned to walk without walking, to work without working, to cipher without ciphering, to learn zvith- out learning ? It would seem that some modern boasters will yet seriously undertake to prove that heretofore man- kind have learned to do without doing ! learned to see with their eyes shut ! The educational problem is now what it ever has been and what it will always be, not whether we learn to do by doing, but what and how is it that we are to do? In zu hat does such ^' doifig'' consist? What, precisely what, are we to do in order to know, or get solid knowledge of, arithmetic or this or that branch ? And let no reader fail to grasp the nature of the question to be answered ; it is simply. What is getting knowledge ? that is to say, What is knowing ? A root, or subjective, answer to this question of questions can be given by Him alone who is the Root, the great I AM. An objective, or branch, answer only may be attempted by men, of whom it is said, "Ye are the branches." So long as mankind are men, they will be able to give an objective rep- resentation only of what knowing, or learning is. We shall never understand psychologically. Such objective represen- tation of knowing will be found in the science-system organ- ism, the Creator's tree-of-knowledge organism ; and it will be found nowhere else. This candle the author has lighted, not, he trusts, to be seen of men, but that "it may give light to all that are in the house." Knowing is branching particular through its class system, — branching, because every class system is a branch system. Knowing, or learning, orthography is branching letter through its classes as branched, or organized, together in the branch, 56 THE SYSTEM METHOD. or tree, system ; in other words, knowing, or learning, is putting the tree-system tmity into the disorganized, and therefore multitudinous, units of any science. Learning orthography is bringing the multitudinous letters out of their disorganized, or unknown, state, and branching, or organizing, them into the branch-system, that is, the class- system, unity of the tree. "All knowledge is the gatheidng up into one" — Plato. " Leibnitz, Kant, Plotinus, and Aris- totle have defined knowledge by the representatio7i of miilti- tude in [or by] unity T — Sir Wm. HamiltoJi. Emerson: "The man may teach by doing, and not other- wise." By doing what .-' — Evidently what the pupil does in learn- ing to know, in getting knowledge ; because the true and natural way of teaching every act is, as mankind agree, by doing that act before the eyes of the learner, and then having him do it. The natural way to teach sewing is to do sewing before the eyes of the learner, that your act may be a guide to the learner in repeating the act, as mankind have ever done. The natural way of teaching writing is to do writing before the eyes of the learner, and then have the learner repeat the act. And ever so. But the reader will note that while the subtle Emerson perceives plainly that all knowing is doing, he falls infinitely short of the discovery that all knowing is organizing — that the act of knowing is an organ- izing, or, to coin a word, an " organisming" act. He says, virtually. The man may teach the knowing act by doing it before the pupil, and not otherwise ; because every act is to be taught by doing it ; because such knowing act is itself an act ever to be done ; as if we should declare that a man may teach the act of writing by doing it before his pupils, and not otherwise — not by directing those pupils to read about the act, or otherwise ; because such writing act is an act ever to be done, not to be read about merely, — not, necessarily, to be read about at all. LEARNING BY DOING. 5Y Comenius: "Things that have to be done should be learned by doing them." What Emerson perceives above, — that all things to be learned are merely acts, and, for that reason, evidently to be learned, or known, only by doing them, — Comenius seems not to see. " Things that have to be done," says he, " should be learned by doing them ;" as if there were some things to be learned that might be learned without doing them ! But learning, or what is the same thing, the act of knowing repeated, is itself an act, and, like all other acts, is to be done. This fact all men understand. No man holds that we may become skilled, or learned, in any act by contem- plating merely, or merely reading about, such act. All men agree that we become skilled in any act by doing that act, " and not otherwise." Again : " Learning should come to children as swimming to fish, flying to birds, running to animals ; " that is to say, learning comes to children by their doing the learning. The testimony is again, Learning is doing something. But the problem is and has ever been, doing what } Learning is doing precisely how and what } Froebel: *' We learn through doing." Froebel seems to have comprehended that all learning is by doing ; not, indeed, that he tells us what we are to do. On the contrary, Froebel nowhere informs us what this doing is like ; nor has any author, ancient or modern, given us this essential informa- tion. Had Froebel declared that we learn through brafichiiig things through the braitcJi, or science-class, system, thus dis- covering to us the tree organism as the prerequisite, or guide, to all organized, or real, knowledge, we should have known just what to do. And, accordingly, long ago we should have hung class-branched maps of every common and nearly every so-called higher branch of learning on the walls ol every school-house in teachable countries ; nor would deris- ion have neglected to put to shame every text-book maker 58 THE SYSTEM METHOD. not guided by this organic prerequisite. By this natural guide on the school-house walls and in all school text-books, both the disorganized, or system-lacking, learning and the almost disorganized limbs of many school children might have been, the one system-learned and therefore an invalua- ble means to " complete living," the other filled with that red blood which alone, through such invaluable means, should have been competent to make this life a success and a joy. Spencer: " Rule teaching is now condemned as imparting a merely empirical knowledge — as producing an appearance of understanding without the reality. To give the net prod- uct of inquiry without the inquiry [or doing] that leads to it, is found to be both enervating and inefficient. General truths, to be of due and permanent use, must be earned [by doing]!' Paraphrased, the language of this witness would be : Rule teaching is now generally condemned by good teachers as giving by rote the net product of the author's inquiry, or do- ing, without leading the pupil himself to do the inquiring. In other words. Rule teaching is condemned because rules are simply the author's conclusion ready-made ; and to have the pupil take the ready-made conclusion of another, and not himself do the work which leads to the conclusion, is weakening to the mind, not strengthening ; or again, in Mr. Spencer's own words, it is "producing an appearance of understanding without the reality." From all of which it is apparent that Mr. Spencer holds every attempt at learning without doing to be vain, even "weakening." Again : " Children should be led to make their own in- vestigations, and to draw their own inferences. They should be told as little as possible, and induced to discover [them- selves to do] as much as possible." Pay7te: "To tell the child what he can learn [or do] for himself, is to neutralize his efforts, ... to defeat, in short, all the ends of true education." Rousseau: "The pupil is not to know anything because LEARNING BY DOING. 69 you have told it to him, but because he has himself compre- hended it " [because he himself has do7ze the knowing, or com prehending, act]. Paraphrased and cleared of mist, this quotation would be substantially thus : The pupil learns the act of knowing nei- ther by the teacher's nor the book's telli7ig him, but by his doing the act of knowing the thing Having in mind and believing what another — an author, say — says, as, for instance, having in mind definitions, rules, formulas, or tables, — having such things in mind and believ- ing them is not, necessarily, true or real knowing. Any defi- nition or rule or table or formula gotten into mind by one who, in getting it in mind, has not done the work leading up to such definition or rule or table or formula, is as worthless as a definition of whistling gotten in mind by one who, in getting it in mind, has not done the work leading up to whistling ! And what is that skill in whistling worth which is obtained by getting definitions of whistling .? Reductio ad ABSURDUM. — It is an adage, old as the mis- takes and successes of men, that an act is skillfully learned only by doing it. The educational, or knowledge, question, I repeat again, is not whether we learn an act by doing that act, but what kind of an act is the knowledge-getting act } We know what walking is. We know what writing is. We have not known what knowing, or knozuledge, is. The answer to this question is the solution of the difficulty. Walking is an act repeatedly extending the feet forward to prevent the inclined body from falling on the face. Writing is an act copying, or reproducing, known curves with the pen. Know- ing, or getting knowledge, is — ^just what and hozu ? That is the question of education, psychology, and logic. Toward one thing every educational advance goes — toward the con- clusion that true knowledge is absolutely nothing except true knowing, which is an act. True knowledge is absolutely nothing except the power of now doing and hereafter re- 60 THE SYSTEM METHOD. peating an act — the knowing act ; just as true walking is ab- solutely nothing except the power of now doing and here- after repeating the walking act ; or as true writing is abso- lutely nothing except the power of now doing and hereafter repeating the writing act. " Education is the generation of power," said Pestalozzi. But we are yet unwilling to wake up our sleepy brains to this truth. If I put knozvledge and walking and talking in one class, and label them alike, " do- ing power," our shocked brains sleepily reply, " What ! knowledge is not the same sort of thing as walking ! " " Yes, sir," replies true Knowing humbly. " Knowledge, walking, talking, writing, reading, seeing, hearing, sewing, all doing, are as like as twin sisters ; are as veritably twins as are writing and reading, or seeing and hearing, or any other two like acts." '" My stars ! " put in the nonchalant school-boys throughout these States ; " our teachers make as much difference between knowledge and walking or do- ing as between a quadruped and Grover Cleveland ; for every month they examine us by questions to see whether our knowledge is good : and how could they tell by ques- tions whether our walking is good.?" "And, worst of all," continue the judicious heads of the land, "in this indecent way of testing the pupil's knowledge, such true and good knowledge as alone can do, is at discount ; for usually in these examinations, the more of an author's doing and the less of the pupil's own doing, the better the pupil's grade or so-called knowledge standing." I have myself often known examiners to require applicants to quote an author ! And in country and village schools, quoting either the words or the substance is quite prevalent. A Stupid Question. — Now if that stupid inquiry arise whether a pupil may understand who has not been required to get the author's definitions, rules, formulas, or other con- clusions ; or if an applicant for teacher's license or school standing or admission to any college, be required to quote QUOTING NOT DOING. 61 either the words or the substance of authors, let answer be made thus : If through any lack of perception, or perverse education, I should ask you whether your finger nails are spherical or cubical, honest or dishonest, you would reply that I am ignorant of the nature of my question — ignorant of finger nails. And so here, you know not the nature of your question. The difficulties of this world are exclusively difficulties of doing, i. e., working. These difficulties, or this work, the Creator put upon us when he said, " In the szveat of thy brow thou shalt eat bread ; " " Cursed is the ground for thy sake." Now, ever since the Creator's six days with this earth, people have aped the same Creator, and adopted this cardinal doctrine, that the very best thing " for their sakes " is to be well up in this doing, or " sweating of the brow," or working, as it may be called. And so all men and all schools, whether completely wise or less so, have stood by the Creator in this business ; and to this day all schools vie with each other both in deed and in loud advertising, to convince the people, each one, that it is pre-eminently suc- cessful in doing this great doing (work) of doing the aid needful to teachers in doing this work (doing) whicK the Creator put on their shoulders to do. I would further inform this not-doing inquirer or exam- iner, so opposed to this exclusively doing business of the Creator, that these same doing schools, when they follow either their own doing example of life-doing or that of the doing Creator, test all pupils' power of doing the act of know- ing by doing processes (or doings) alone. And if by this time this not-doing inquirer or examiner has not detected that all the doing of this life consists in doing (and that therefore he has no right to demand that I ape the parrot in quoting, or giving, another's conclusion instead of my own), and is not able to correct his to-be-answered-by-quoting question, I should give him my own conclusion, definition, rule, or formula for something which he has not learned by 62 THE SYSTEM METHOD. doing, — say whistling or singing or violin-playing, — and then demand of him to do such whistling or singing or violin-play- ing by means of the definition, rule, formula, or conclusion which I gave for the act. In this way, it is not difficult to bring about such a degree of humility as Avill render even a county examiner tolerant of opinions that conflict v/ith those of the text-book which he has "gotten" so "thoroughly." Or, if the examiner is a great geologist, quote Lord Lyell's so-called "incontestable evidence" that the world was 20,000,000,000,000 years in its formation. But if he is a Biblist, quote the six-day record, and don't fail to get in the fact that " in the space of 170 years the lower Mississippi has shortened itself just 242 miles " by the geologic story. And that since this is just a trifle over one and one-third miles a year, therefore, any person who is not blind or idiotic, can see that in the Old Oolitic Silurian period, just a million years ago next November, the lower Mississippi was upward of 1,300,000 miles long, and stuck out over the Gulf of Mexico like a fishing-rod ! Or, if the examiner is a great comparative anatomist, quote Prof. Marsh's discovery of land animals of 100 ft. in length just at the time when learned anatomists of Europe had demonstrated that no land animal could exist larger than an elephant. Or, if the examiner is great on transcendentalism, quote Joseph Cook on immortality thus : A northern bird instinctively desires a south or warmer region ; therefore, there is a south, and the bird has got it. And so man instinctively desires immortality ; therefore, there is an im- mortality, and man has got it. And not to be too hurried when in the wonders of deductive philosophy : an unmarried man has an instinctive desire for a wife ; therefore, there is a wife, and an unmarried man has got one. Be careful to ascertain what text-book your examiner was educated in. If he is an Olney man in arithmetic, then quote : " The design should be to familiarize the student with QUOTING NOT DOING. 63 the use of formulas, without which nothing can be done in mathematics beyond the mere elements of arithmetic." But if he is a stickler for Wentworth, you will quote thus : " The object should be to teach the pupil 'to learn by doing,' not by memorizing [or becoming familiar with] rules " or formulas. Or, if the examiner talk of the depths of natural science, and you know he is indifferent about the author quoted, only that you agree with him in an atheistic philosophy, you may quote what I here fix up for you : Since, when we have found all the classes in which animals and plants are made in nature, we fully know or understand those animals and plants ; since as soon as we have taken off the cloak, all animals are seen to be made in classes, as vertebrates, moUusks ; mammals, birds ; flesh-eaters, monkeys ; lions, weasels ; etc. ; since it is an utter impossibility for us thus as perfectly and exhaustively to classify them as they are in nature classified, except we have some knowledge, or under- standing, of them ; since it is impossible and absurd to con- ceive of one's thus perfectly classifying millions of animals except that one know, understand, or have intelligence of them ; therefore, evidently ( ! ) that which did so classify them in nature had no knowledge or understanding or intelligence of them ! It is as plain as daylight that that which did thus classify them knew and understood them. Since that which classified them was a knowing Power, therefore they were classified by the unknowing power of law or chance ; and since a knowing God alone could have classified them, therefore they were classified by an unknow- ing law or by chance ; and since these organized classes could not now exist unless heretofore created by a knowing God, therefore God has no existence ! If by such considerations, the examiner cannot be made to realize that quoting either letter or "substance" of books is proving one's skill in doing somethinghy doing nothing; and if examiner perceive not also that quoting books, while 64 THE SYSTEM METHOD. books are, as yet, almost destitute of science system, and therefore destitute of consistent truths, is to be discouraged and disallowed, not encouraged ; finally, if examiner cannot perceive that it is impossible for me to give evidence of my skill or fitness to do, where knowing or instructing is the work to be done, by quoting, I should magnanimously refuse to accept a certificate at his hands, or to teach under his supervision. The Educational Guide a Natural Product.— I have now established by incontestable evidence the fact that learning, or knowing, is branching. It was shown that, if knowing is not a mental branching process, and therefore a process building a branch-system organism, a thing is not a thing — something is nothing ! Reader, let astonishment seize you as it may, the great educational problem is solved. Knowing, learning, and thinking are branching, limbing, or organizing. Remembering is re-branching, re-limbing, or re- organizing. Knowledge and memory are skill in branching and re-branching. Education, the world over and the ages through, has been an enigma, a repeated charade uncon- sciously played by great and little philosopher on the word science-tree, or "tree of knowledge." Each philosopher has played the same science-tree over and over again, repeating both himself and his predecessors only because he could not do otherwise -because every human thought and act is guided by the wires of the truth as exampled in the branched system, the "tree of knowledge." Not detecting, as we play, that our every thought, and therefore our every act, is guided or "jerked by the unseen wires," we have not till this day discovered what such guide is like. Not knowing what such guide must be like, we have hardly dreamed that the Creator actually holds it everywhere before our eyes, in the natural and world-over-distributed tree organism. Next, a way-side deduction. All agree that what is wanted in educational doing is an intelligible guide. It is a THE LACKING GUIDE. 65 learner's guide, that all real or would-be educational reform- ers have labored to discover or invent. Now mark what it is that lacks such help— what it is to which this lacking gmdc is to be supplied. It is the hitman mind. Now let us inquire, and sagaciously consider the answer, Is the human mind a created or an uncreated thing.'' Scientist and Biblist agree that it is a created thing. But did the Creator make the mind lacking something (you say it lacks a learner's guide) .? — No, sir. Here, too, scientist and Biblist agree that all things are created perfect in their respective spheres. Do you not, then, perceive that, if all things are created perfect, and it takes both mind and guide to make a perfect whole, or created thing, — do you not perceive that the guide itself must be a created thing } — Yes, sir. If the learning mind is at an inconvenience without the proper guide, as the learned agree, it is as plain that He who made the mind made also the guide, as it is plain that He who made the arm made also the fingers, without which the arm must be at an inconven- ience. But is it not plain that, if all things are created per- fect, and the mind lacks a guide, as all agree. He who made the mind made also the guide, without which the mind is lacking or imperfect .'' That is as plain, sir, as that it takes all the parts to make the perfect whole. Since, then, mind and guide are complemental parts of which complete and well-regulated living are the whole, where shall we expect to find the guide } inside the other part, the mind, or outside of it.? Why, you do not suppose I would expect to find one part inside of another part, with which it is taken to form a perfect whole ! I would expect, sir, to find the guide out- side of the mind. What, then, is your conclusion in regard to the wisdom of all the psychological attempts of teachers to find this unknown guide inside of the mind of growing babes and children 1 I think, sir, such psychological work is very unreasonable. Through such considerations as the above, whether sound 5 66 THE 8TSTE3£ METHOD. or unsound, the author came to the conclusion that this lack- ing something — lacking guide — must be sought outside the mind. Further, it must be some natural and corporeal prod- uct ; for no incorporeal thing, as magnetism, odor, sound, or electricity, can be seen, mapped, and icsed as such guide. Such guide could be no mental phenomenon for the same reason ; namely, that no mental phenomenon can be seen, mapped, and used as such guide. Absolutely nothing except a natural product can be seen, mapped, and so used and agreed upon. Absolutely nothing, therefore, except some natural product, could form such lacking guide. Thus it ap- pears that this invaluable guide, lacking to learner's mind and looked for by educators everywhere, must be a natural product. Such inferences as this, pointing toward the tree as the representative of the class system of any science, are cumulative. But it has already been shown that learning, knowing, or thinking, is bra^iching ; and the only possible guide in branching is that whose parts are all branches. Let it next be shown that the tree is guide, not only in all knoAv- ing, but also in all re-knowing, or remembering. The Long-Sought-for Gnide in Remembering is the Tree-System Whole. — Psychologist, teacher, and learner, and men generally, agree that we do not and cannot re-walk till we have first walked ; that we cannot rewrite till we have first written ; that we cannot re-know, or remember, till we have first known, or, to coin a word, " membered." All agree, likewise, that when we have talked perfectly, we shall be able to re-talk ; that when we have written perfectly, we shall be able to rewrite ; that when we have known per- fectly, we shall be able to re-know, or remember. Now, it has been shown that the guide in knowing, or branching, is the branched system, the tree, nature's only tangible repre- sentative of a class system. I need not, therefore, write a word more to demonstrate, or put beyond all controversy, the almost matchless truth that, in re-knowing, or reniem- ANALOGICAL PROOF. Q^ beri?ig, the guide is still the tree system. If, to the first act of knowing-, or branching, the tree organism is the only per- fect guide, who will dispute that in remembering, that is, a subsequent act of knowing, or branching, the same tree or- ganism is not the only perfect guide ? And observe that the tree is a natural product, which alone, as Thave just shown, could be made use of as such guide. That the tree is the true educational guide, nature's only index to the act of knowing, the following considerations, and their kind, will, to the thinking mind, plainly show : — First Proof: By no other knowing guide can we see at all, but by the tree, or branched, guide we can see as plainly as human perception is able to see, what the act and re- sult of knowing, or branching, orthography is like. The act = brandling letters, the result = letters branched into the branched system ; that is to say, the act = learning let- ters through the orthographic tree, — branching, — and the re- sult = all letters known and re-known, or remembered, through the tree as the only guide and only means. Now let the reader wake all his sagacity of mind to consider the mutually corroborating character of the two following truths, or theories : — Second Proof: 1. By no other zurititig guide can we see at all, but by the copy guide we can see as plainly as human preception unfettered is able to see, what the act and result of writing copies (of writing) is like. The act = writing, or copying, copies (of writing), or writings; the result = a copy copied, or made, through the copy guide as the only guide and means. Any thinking and unbiased mind skilled in analogical reasoning, and distinguishing between naines and things, will, if it strive to force itself either to belief or disbelief in the tree, find proof of the guideship of the tree by comparing the above with the following : — 2. By no other learning guide can we see at all, but by 68 TUE SYSTEM METHOD. the branch guide we can see as plainly as human perception unfettered is able to see, what the act and result of learning ''branches" (of learning) is like. The ^.z\.-=^ learning, or branching, branches (of learning), or learnings ; the result == a branch branched, or made, through the branch guide as the only guide and m.eans. In order that this analogical proof may appear overt and plain, substitute for branch, the truest name of the guide and process of learning, the name of any other supposed guide. Take the word " elephant," for example, or "animal" or any other whatever, except "branch," and the immediate result is nonsense. Examine the following : — By no other learning guide can we see at all, but by the elephant gnide we can see as plainly as human perception unfettered is able to see, what the act and result oi learning " elephants " (of learning) is like. The act = learning, or elephanting, elephants (of learning), or learnings ! the result = an elepJiant elephanted, or made, through the elephant guide as the only guide and means. Third Proof: By no other learning guide can we see at all, but by the branch, or grammatical-tree, guide we can see as plainly as the unfettered human perception is able to see, what the act and result of learning, or branching, English grammar is like. The act = branching the words through their tree-branch classes; the result = a. familiar branched system, or science, of Avords. Fourth Proof: By no other learnifig guide can we see at all, but by the branch, or botanical-tree, guide we can see as plainly as the unfettered human perception is able to see, what the act and result of learning, or branching, botany is like. The act ^ branching the plants through their tree- branch classes ; the result = a familiar branched system, or science, of plants. Fifth Proof: By no other learning guide can we see at all, but by the branch, or zoological-tree, guide we can see SIXTH PROOF. 09 as plainly as the unfettered perception is able to see, what the act and result of learning, or branching, zoology is like. The act = branching the animals through their tree-branch classes ; the result = a familiar branched system, or science, of animals. And so on. Sixth Proof: All authority, including the common judg- ment of mankind, which is ever the highest authority, agree that every subject or object is best remembered, as it is best known, zji the light of zvhat it is like. But the subject of orthography, taken in part or in whole, is like the tree ; and it is like nothing unlike the tree. But this is true, likewise, of grammar, — English, German, French, Latin, Greek, He- brew, Sanscrit, or Universal ; of botany and zoology ; of all geographynot unsystematic ; of true notations of logic and psychology; of arithmetic and the higher mathematics. Now, the tree system is the only system in nature in which every part, taken alone, is itself a lesser whole of the same features as the including, or complete, whole. Every branch part of any higher tree-branch whole possesses all the essential features of such higher, or including, whole. Every division, or part, of a branch, or tree, whole is itself a lesser branch whole no less complete than the greater branch whole of which it is a part ; so that branch whole (that is, tree whole) and branch part are entirely similar. Now, every science, or branch of learning, is a class system ; and every class system is a branch system — class divided giving classes, just as branch divided gives brandies. It is because every science system is a class system, and because class divided and subdivided gives classes, precisely as branch divided and subdivided gives branches, as in the tree system, that every science system is like a branch system, or tree. But this has already been established. (See from p. 46 to p. 52.) Sevefith Proof: By a careful study of the branched, or organized, wholes of orthography, grammar, etymology, bot- any, etc., as represented by the tree-system maps given on YO THE 8T8TEM METHOD. pages 18, 25, 26, 2Y4, 494, 642, the reader will note that all the parts of branch of learning and tree alike are branches of the unit of study and the tree trunk alike. Mark well — • 1. That in orthography the unit of study is the letter, 2. That in grammar the unit of study is the word. 3. That in etymology the unit of study is the etymon. 4. That in zoology the unit of study is the animal. 5. That in botany the unit of study is the plant. Mark now, especially,, that when we put such system unit (or unit of study) for the trunk of the tree, all the parts of the tree are represented in the parts of the subject ; and, vice versa., all the subject parts, or branches, are repre- sented by the tree parts, or branches, thus showing that the tree-system fits every "branch of learning" just as the thing molded fits the mold. Mark again, that there is no known object in the natural world whose parts are visi- bly all branches. Take, for example, the human system, and suppose the unit of study to be represented by the trunk. Now, grant for fairness' sake — what is not true — that legs and arms may fairly be considered as branches of the trunk ; but what about the head } In order that man shall become a guide, or means, in learning and remembering any branch of learning, all man's parts lying outside of the body trunk, must be branches of that body trunk ; but is the head a branch of the trunk.? — Not a whit more than the trunk is a branch of the head. All anatomists agree that neither head nor trunk is to be considered as branch, but that either is a division merely of that whole by them called the skeleton. Try, then, any other object whatever, say a building. Of any building, who will find, who has ever yet found, either body or branches .'' He who finds in any building what is body and branch, will in reality therein find that such build- ing is the product, not of man's, but of the Creator's hands. And so, though we may "try, try again," we shall find no possible guide in learning and remembering except the ex- PART AND WHOLE COMPARED. 71 clusively branched and sub-branched system called by rea- son thereof, in the inspired record, the "tree of knowledge ;" because there exists nowhere any other object whose parts are all branches and sub-branches of essentially the same features as the including whole. Eighth Proof: By still another examination of the branched, or organized, wholes of orthography, grammar, etymology, botany, etc., the reader will still further observe that not only the unit of study in each subject — letter, word, etymon, plant, as it may be — branches, but that each of these first branches is sub-branched, each sub-branch again sub- branching, and so on, the extraordinarily simple subject of orthography extending, at the least scientific count, through five successively subordinated branches. Therefore, since ev- ery subject of study comprehending a sufficient territory, or number of sub-branching limbs, or parts, to render it worthy the title of " branch of learning, " is a branch system, and since every important branch of learning contains branch and sub-branch several times subordinated, whatever natu- ral object is taken as guide, map, or representative of such subject, must itself have trunk, branches, sub-branches, and subordinate sub-branches. Since the tree is the only known object of such characteristics, the tree, and the tree alone, is the guide in learning and remembering every such important branch of learning. Ninth Proof: Let the four following propositions be demonstrated, that thereby it may be still again proven that the class-branch, or tree-system, organism is the true guide and instrument in all remembering, just as it is the only guide and tool in learning and knowing : — 1. All things in classes are in branches — parts of a tree- system organism. 2. All knowable things are in classes. 3. All rememberable things are known things. 4. Therefore, all rememberable things are parts of a tree- system organism. T2 THE SYSTEM METHOD. First proposition.— All things— absolutely all things- are in classes. This truth rests upon the same solid founda- tion as the simple equality, A thing is a thing. And not- withstanding all men assent to this grandest of all educational truths, namely, that things are created and to be known in classes, yet I have already repeatedly demonstrated it in these pages. Mark, now, that because all things are in classes, all things are in a tree-system of classes ; for, since all things are in classes, therefore classes are in classes. But to say that classes are included in higher classes, is to say that all the classes (of the things classed), when put together into unity or system, form a tree-system of classes. Take the grammar classes of words for illustration. Now of nouns, the classes, common 2.nA proper, are, in system form, branches : — (i) noun Of pronouns the classes, per- sonal, relative, and interrogative, are, in sensible system form, likewise branches: 4 relative cordingly And ac- And finally, interjections, 18. Mark well what science system is ; for it is a tree system. We could never obtain power, or dominion, over words out- KNOWABLE THINGS IN CLASSES. 73 side of classes ; just as we could have no dominion over things generally if they were not in classes. " It is only by 'assorting them [the almost infinite numbers of objects in GRAIVr]VrA.TICAL WORK-TREE. Copyright, 1886, by I. 3. WILSON. nature] in classes ' that we can reduce the infinity of nature to the finitude of mind." — Sir Win. Hamilton. It was by creating them in classes, and by giving us power to class them, that the Creator gave man dominion over all things. 74 THE SYSTEM METHOD. Now consider well that, since we cannot learn, or know, thousands of words, each as an isolated thing- unlike every class, we first " assort " the multitude of words into about eighteen (IS) classes, so that we may understand the thou- sands of words by thoroughly learning these eighteen classes. But we are to know all things in the light of class. So that, for the same reason that the thousands of words are assorted into eighteen classes to be learned, the eighteen classes are themselves assorted into higher classes, or tree branches, to be learned ; and so on, the whole system of classes forming a tree system of classes, because, placed together in system, these classes can form nothing else. Thus it is, because things are created in classes, because thinking knows nothing outside of class, but thinks every particular through class in order to know such particular, that a science system (that is, class system) is a tree system. Since all words are included in classes, and since all these classes are included in classes, as branches are included in higher branches, and so on, till all the higher classes are in- cluded in the highest class, the tree-trunk class, what can be plainer than that the class system, or science, of words is a tree system of classes — a tree system in which each branch and sub-branch is a class of words .-^ None, then, will deny that all things in classes are in a tree system of classes ; that is to say, are parts of the tree-system organism. Second proposition.— It is next to be shown that all knowable things are in classes — that all things are knowable by virtue of their being created in classes. This I have already demonstrated over and over again in the first forty pages of this work. Quickly to re-demonstrate this great fundamental of learning, let the reader undertake to know, or learn, some supposed knowable thing not contained in classes with other similar things. Refer to the cuts, pp. 25, 26, 98. Such supposed knowable thing cannot be an English- man ; for, to be so, it must belong in the class E^iglisJiman. KNOW ABLE THINGS IN CLASSES. ^5 For the same reason, it cannot be a man ; for, to be a man, it must needs be similar to other men, and belong in the class inaji. It cannot be a plant ; for to be so is necessarily to belong in the class of things known z.s plaiits. It cannot be a thing ; for to be a thing is to belong in the class thing. Such supposed knowable thing is therefore a not-thing — ■ nothing ; that is to say, there is no knowable thing not belonging in some known class of things, and the supposition that all things are knowable by virtue of being in known class, is true. This second proposition is therefore again established. Refer again to the trees, pages 26 and 98. Suppose the mink to be utterly out of class — unclassifiable. It is, then, neither flesh-eater, mammal, vertebrate, animal, organic, natural, nor qualitative thing. Therefore, since it is not thing, it is nothing ! That is to say, all things whatsoever, excepting nothings only, are in classes with other things ; that is, a thing is to us a thing by virtue of its belonging to classes ; that is to say, mink and leopard and Draper and Victoria and Wilhelrn I. and red pine and banana and scour- ing rnsJi (see cuts, pp. 25 and 26) and every other thing, are to us things by virtue of their belonging in some class or classes of things. It is, therefore, herein still again proven that all that are to us things, are in classes. Third proposition. — To say that all rememberable things are knowable, or known, things is only to say that no thing can be remembered till it is first known. Remembering is, as the experience and judgment of mankind have ever averred, simply re-knowing. And that a thing cannot be r^-'known till it \s first known is as plain as that a field of corn cannot be replanted till it has been first planted. If the reader, by some mysterious means, has persuaded himself that he can remember something which he has not beforehand known, that is, if he thinks himself able to remember some unknown thing, let him consider what sort of a thing an un- Y6 THE SYSTEM METHOD. known thing really is to us. See the second proposition above, wherein it is shown that all things to us unknown are to us simply not-things — nothings. Whoever, therefore, is able to remember something which is unknown to him, is able to remember something which is nothing ! Strange remembering ! That all rememberable, or remembered, things are known things is to every one too evident to call for any proof or explanation. Babes advance sufficiently in the act of know- ing and remembering, to know that no man ever remembered what he had never known. Babes know that to remember the name or face of, say father, is to recognize, or again know, such name or face. Conclusion. — It has now, in three parts, been estab- lished, — 1. That all things in classes are class branches — parts of a tree-system organism. 2. That all knowable things are in classes. 3. That all rememberable things are known things. 4. It therefore follows that all knowable and rememberable things are class-branches — parts of a tree-system organism. The class-branch, or tree-system, organism is thus proven to be the long-sought-for guide in all knowing and remem- bering — in all teaching and learning. Tell it at Boston ! Tell it at Cambridge and at Leipsic ! Tell it to a world struggling almost vainly to learn how to learn ! Although I have accomplished the proof of the proposi- tion that the tree-system organism is the only true guide in remembering, — that all rememberable things are parts of a tree system, — I cannot refrain from an observation of how " dreadfully defective," how disorganizing and system-repudi- ating, are the learning and the methods of our schools. Now it is self-evident that the tree organism whole can con- tain only such things as are tree-organism parts. The weasel organism, for illustration, is formed of, and can con- SYLLOGISM CONCLUDED. 77 tain, only such things as are weasel-organism parts. The chicken organism may be formed only of such things as are proper chicken-organism parts. Put, if you please, a raccoon's legs and tail on the chicken organism, instead of the chick- en's own legs and tail ; and you will at once perceive what a deplorable absurdity a man would get himself into by deny- ing the plain truth that the chicken whole can be formed only of chicken-organism parts. Just as foolish is he who would deny that the tree-system organism of classes may be formed only of tree-organism, or class, parts. Plainly, since the tree organism can be formed only of tree-organism, z. e., class- branch, parts, all such tree-organism, or class-branch, parts are parts of the tree-system organism. This is as axiomatic as the plain truth that whatever whole is formed of animal- organ parts must be an animal whole. And let not the reader fail here to perceive the further truth that all such class-branch, z. e., tree-organism parts, are necessarily parts of a known tree-system organism. To demonstrate this truth, let us suppose that some class-branch, z. e., tree-system, parts are parts of an unknown organism. Evidently, such un- known organism could not be any animal or any plant ; for these are things known. For the same reason, such un- known organism or thing could be neither any organic nor any inorganic thing ; for these, too, are classes more or less known. Finally, such unknown organism could not be a thing ; for things, too, are more or less known. Now, since such unknown organism cannot be a thing, it is a not-thing — it is nothing ; that is to say, an organism unknown to us is to us nothing, just as everything to us unknown is to us noth- ing. Therefore, — mark the conclusion, — all class-branch, or tree-system, parts which are parts of an unknown organism, are to us parts of nothing! But parts that are parts of noth- ing are not parts. Therefore, since — 1. All class (tree-system, or science-system) parts are parts of the tree-system organism ; and since — 78 THE SYSTEM METHOD. 2. All parts of the tree-system organism, to be system- learned, must be parts of a known tree-system organism ; therefore — 3. All class parts are parts of a known tree-system or- ganism ; that is to say, while branching through the classes (the branches) of a known organism is real branching (real learning), branching through an unknown organism is not real branching — not real learning. Give ear : since learning is branching, and branching into an unknown organism is branching into nothing — not branching ; and since to this day, the tree-system organism, including the classes branched through in learning orthography, grammar, etc., has been to the world actually unknown ; therefore, to the world, real or rational branching, or learning, of these branches at least, has been impossible ! And have I not proven and re-proven again and again that all real learning is branching through known classes (class branches), and that real learning is nothing else.? Have I not even demonstrated that the very act of thinking is impossible except as it is a mental act branching thing thought of into higher branch class.? And is not the proof that learning is branching as strong as those adamantine truth chains that link the worlds together } 1. All rational branching has been impossible because the branched v/hole (tree-system whole) has bee]>. unknown and unused. 2. But rational branching is rational learning. There- fore — 3. All rational learning (of branches orthography, gram- mar, etc.) has been — shall I say — " impossible 1 " And real rational learning must ever be so till the " tree-of-knowl- edge " organism shall be known and used as guide in learn- ing what is a "branch" of learning — in branching what is a branch of branching. The Branch Whole-Orthography. Verily is the tree a tree " of knowledge." And now mark yet once more that our so-called learning of the parts with no fancy even that the tree whole, or guide, exists, must needs be identical with a supposed learning of a cat's tail with no idea that such a thing as a cat ever existed ! Such is not merely a simile ; it is the literal truth. We pretend to learn the orthographic class-branch parts, sicbvocal, aspirate, vocal, abrupt, diphthong, etc., with little idea of what constitutes the orthographic whole. Nay, we study these orthographic tree parts with no idea that such a thing as an orthographic tree, or whole, ever existed ! Is not our study of these or- thographic parts with absolutely no idea that an ortho- graphic whole exists, identical with our supposed study of a cat's part — cat's tail — with absolutely no idea that a cat ex- ists .-' Let the reader here wake all his sagacity of mind to consider. I say we have pretended to learn understandingly the orthographic parts not knowing what the orthographic whole is. No author, so far as I know, has ever mentioned such a thing as an orthographic whole, far from saying what such orthographic whole is like. No educational or philo- sophical writer so much as hints that the orthographic sys- tem whole is a tree. Since no writer has known that ortho- graphic whole is like known thing, such system whole has been to all writers an unknown thing ; but since everything unknown to man, is to man no thing, — nothing, — such ortho- graphic whole has been to all writers simply nothing — has had no existence. The world, therefore, studies siibvocal, vocal, aspirate, abrupt, dipJitJiong, dental, etc., as parts of nothing — as not parts ; precisely what he does to whom, in [79] 80 THE SYSTEM METHOD. the study of a cat's tail, the existence of a cat, past or pres- ent, is unknown ! Do you not begin by this time to see the priceless value of the tree guide ? And behold what it is to learn orthography or the animal^ by studying siibvocal, aspirate, or tail, by itself, and without being conscious that the orthographic whole or feline whole exists ! It is like, it is identical with, learning the cat by studying the parts, foot, tongue, ear, stomach, tail, heart, eye, back-bone, etc., each by itself, without being conscious that such a thing as a cat exists ! Such is literally what we do when we study the parts subvocal, aspirate, dental, etc., each by itself, without being conscious that such a thing as an orthographic-tree whole exists. No wonder that school- learning is almost worthless for the doings of life ! No won- der that, among business or doing men, our "A. B." added should be taken for conclusive evidence that the adder has just passed his " commencement " merely, — has yet hardly begun to learn the life-doings of men. No Branching Guide Previously Discovered. — First PROOF : But have teachers known nothing at all of this sci- ence-system whole of orthography } — Certainly not. Now, if after learning of the discovery of the tree system as the long- sought-for guide in educational work, — a guide not less es- sential to any rational learning of subvocal, aspirate, diph- tJiong, etc., than the known existence of a cat to any rational learning of a cat's tail, leg, or back-bone, — some great A. M. see himself "begin Avith shame to take the lowest room ;" if he then suddenly discover that he himself has akuays known such an orthographic whole, I would grant it for his shame's sake. He would be asked what natural and familiar system whole is the science-system whole. If he answer any known thing except the tree, his declaration is false ; since neither the orthographic whole nor any other branch-of-learning whole can possibly be made into, or be indexed by, the like- ness of any existing product of nature, except the tree, as I have before shown. He could not, therefore, have known what such orthographic whole is like — far from using it as a NO PREVIOUS DISCOVERY. 81 guide — without actually discovering the orthographic tree. But had orthography or any other tree-system whole actu- ally presented itself to him as a tree-system, or class-branch, whole, he could not yet have made any discovery of the needed educational guide, not until it had plainly appeared to him not only that such orthographic, etymological, gram- matical, or other tree whole was 7iczu, but that such tree was also useful, and therefore not to be dispensed with. Had Columbus discovered a land, which, while new, was nev- ertheless useless to the world, Columbus's discovery would hardly have been to this day even recorded — very far would he have been from any right or claim to a real patentable discovery. Hence, if human eyes have ever seen this class- branch representative of a science system, such eyes have nevertheless failed to discover such tree as a useful guide in learning and remembering — a discovery unlocking the hith- erto unsolvable question. What is the learner's true process of work in doing a branch of learning .? Second proof. — If, then, the inquiry arise whether here- tofore any author has ever discovered the tree as the learn- er's guide in the branching act called learning ; and, that query having arisen, if such discovery and invention be looked for in any grammar-tree picture outline (such as Rufus Blanchard's "Grammatical Tree"), the author replies, No ; and here follows an additional proof: — The only possible way such tree can be used is as a guide in branching — branching word through its class branches. Therefore, whoever heretofore used this science-system tree, used it in branching ; since it cannot be used for aught else — is practically useless for any other purpose. But " branch- ing" is a word not to be foiuidxw. educational literature out- side of the System Method (July, 1885). Moreover, neither Mr. Blanchard's nor any other author's device, so far as I know, is so constructed that it is possible to branch a word through it ! Indeed, no author has known that learning is branching ; as a matter of course, no author has dreamed of a guide by means of which to do the branching act. 6 82 THE SYSTEM METHOD. That the learning act is a branch-building act, is his own discovery. This once estabhshed, it follows that the branch- built whole, the science tree, is the learner's long-sought-for guide by which to do his work ; for, that the branch-built whole is the only possible guide to the branch-builder, is self-evident. What fool cannot see that the house-built whole is the true, the only proper or possible, guide to the house-builder.^ What biased teacher, then, is he who will not see that the branch-built whole is the true, the only proper or possible, guide to the branch-builder, the learner } Mr. Blanchard cannot himself, even after taking time for forethought, possibly branch any word through its classes by means of his tree. He has not so constructed his tree as to render it possible to branch a word through it. Try "are" in " They are true," for instance. Doing no violence to the tree as a branching guide, but impartial justice at every step, we proceed according to the tree thus : Are = a verb, a signification, a mode, a tense, and a present ! And if we seek to branch the word " true " through its classes, by the direction of this tree, we get : True = an adjective, a com- parison, and a positive ! We thus see that such trees as this are not at all aimed, or designed, to be used as branching guides. Whether any author ever discovered that a real science system is a class divided and subdivided as a tree-trunk is divided and subdivided, is doubtful. That no author ever discovered the learner's guide in doing a science system to be a tree-trunk class branched and sub-branched, is certain. No writer, realizing that learning a particular is branching that particular through its classes, has aimed to construct a branching guide. This fact, explains why it is impossible to use any of these grammar-tree, genealogic-tree, or Christmas- tree devices as a guide in branching — as guide in doing the learner's work of branching particular through class. Such is the construction of — such was the design in constructing — the writer's class-branch, grammar-science, tree, that, once started at any topmost branch, learner cannot fail to branch NO PREVIOUS DISCOVERY. 83 correctly any word through its proper science-system classes ; for each topmost class is included in each higher (larger) class, just as each topmost branch is included in each higher branch, and finally in the tree trunk. Such is the construc- tion of Mr. Blanchard's tree that, once started at any topmost branch, learner cannot fail to branch incorrectly and falsely any word through its proper science-system classes ; for each topmost branch is not included — no topmost branch is in- cluded — in each higher (larger) class or branch. Mr. Blanch- ard did not dream of constructing a guide in branching ; for he did not know that learning is branching. Now observe that it is because any word which belongs in any topmost branch tJierein belongs in every succeeding higher branch, that any grammar tree is worth anything whatever as a working guide to students ; for it is because what is in any branch is therein in every higher, or including, branch, that any word is traced through its classes by being traced through the tree branches : so that, if a student is given, say a subjunctive (see cut, p. 49-i), to trace down- ward through the system, he cannot fail, if he follow the tree, to trace correctly such verb through its proper higher classes ; namely, invariable, finite or essential, verb, funda- mental, word, lingual (see tree p. 93), qualitative, thing ; for, once started downward at subjunctive, he cannot go wrong, since subjunctive is included in every higher branch down to the highest known class, tJiing. This grammar tree thus constitutes a true notation for the syllogism itself, exampled thus : — 1. Given a word, — that it is a subjunctive, — 2. Since branch subjunctive is included in all the higher branches, invariable, finite, verb, etc. ; 3. Therefore, this given word is an invariable, a finite, a verb, a fundamental, a word, etc. Thus the learner, when familiar with this tree system as constituting the science system of grammar, knows, from one fact alone (that a word is a subjunctive), many additional facts ; namely, 1. That it is invariable ; 2. That it is finite ; 84 THE SYSTEM METHOD. 3. That it is a verb ; 4. That it is fundamental in the sen- tence, etc. Hence, by becoming familiar with the tree, the learner acquires the power of reasoning concerning words through the syllogism itself, knowing not only what classes a word belongs in, but what classes belong in what other higher and still higher classes. Through the instrumentality of the tree, he is thus able at once to decide what is and what is not cor- rect language so far as grammar-science is concerned. fSee The System Method No. 6, Demonstration XXII. and XXIII. , p. 485.) The scientists had taken the first step only — they had distributed the words into classes. But the author believes himself to be the first to see that there was a second step to be taken before the student's Process of Work could be dis- covered, and his guide become known. The author reasoned thus : In grammar it is the design to learn the classes of words ; since it is only by knowing the classes that we can understand the particular words — understand whether a sen- tence is correct. But since learning a word is finding its classes, therefore learning the classes of words is finding their classes — their higher classes. And taking the eighteen classes found on page 72, and thus distributing them into their higher classes, he discovered that they foi'vi a tree. But he did not stop here. The grand principle that the think- ing act thinks particular into class, must now be applied to this tree formed of the classes of words. And this was done as follows : — Since to learn a word is to think it through its classes, and since its classes are tree branches, therefore, to learn a word is to think it through its tree branches — through its classes as branches of a tree, each branch of which is a class of words. Thus it was that the learner's guide in studying grammar Avas discovered to be the class-branch tree, and his process of work to be branching — branching the word through its classes as branches of the science-tree of gram- mar. NO PREVIOUS DTSCOVERY. 85 Therefore, every branch of the author's grammar-science tree was made a c/ass of words — a class included in the tree- trunk class, the word ; so that, by tracing, or branching, a word through it, the learner therein branches that word through its classes, zvhich is true learning, as all now agree. Thus it was that in his effort to find the learner's true proc- ess of work, the writer found also the true nature of the science system — found that the science of grammar is a class- branch tree. Thus it was that in finding the learner's tool, the class-branch tree, he found also what the learner is to learn, the class-branch tree, — a thing hitherto unknown, a thing operated in a hitherto undreamed of iip-tJic-trce way. So far as the writer knows, no author ever hitherto un- dertook to construct a tree by which to branch ; for branch- ing through a tree where each branch is not a class, is im- possible. The truth is, that not knowing that learning is branching, not conceiving that the branching tree could be used as a guide in learning, no author ever undertook to construct a tree by which to do the learning act. All these picture-tree charts were therefore constructed without a fancy that a science system constitutes a class-branch tree, and that, therefore, such class-branch tree is the long-desired learner's guide in doing, or learning, such science system. Again : the tree branches of these trifling picture charts are not aimed to be classes. A verb in the past tense — " went," for example — run on the what-ought-to-be class branches of Rufus Blanchard's tree, runs thus : Went = a verb, a signification, a mode, a tense, and a past ! showing that Mr. B. had no thought of making a tree whose branches should all be classes, and which could therefore be used as guide in branching words through their classes. Therefore, so far from discovering the tree to be the learner's invaluable instrument and guide in doing his Avork, these authors had not even a fancy that the tree could be used in branching ! Third proof. — But there are other conclusive proofs that the tree has never hitherto been used as the learner's guide and tool in doing the learning, or branching, act. 86 THE SYSTEM METHOD. Whoever hitherto used the class-branch grammar tree in branching, used such tree in branching zvord through its classes ; for it is word, which, divided and subdivided, forms the grammar-science tree. So whoever hitherto used the class-branch orthography tree in branching, used such tree in branching letter through its classes ; for it is letter, which, divided and subdivided, forms the orthography-science tree. So whoever hitherto used the class-branch etymology tree in branching, used such tree in branching etymon through its classes ; for it is etymon, which, divided and subdivided, forms the etymology-science tree. So whoever hitherto used the class-branch arithmetic tree in branching, used such tree in branching o-stcp solution through its classes ; for it is 3-step solution, which, divided and subdivided, forms the arithmetic-science tree. But whoever used the grammar-science tree to branch word through its classes ; or whoever used the orthography- science tree to branch letter through its classes ; or whoever used the etymological or the arithmetical tree to branch the etymon or the 2>-step solution through its classes, — such an author used a tree of which the trunk was word or letter or etymon or 3-step solution respectively. Such author, there- fore, discovered — what is not known — that grammar is the science, or system, of the word ; that orthography is the science, or system, of the letter ; that etymology is the science, or system, of the etymon ; that arithmetic is the science, or system, of the 3-step solution. But these defini- tions for these sciences are nowhere to be found ! So far are authors from any knowledge that the tree is the learner's true guide in learning, or doing, these sciences, or systems, that they know not even that they are real sciences, or systems ! So far are the authors from knowing grammar to be the science, or system, of the word, that En- glish grammar is usually defined to be " the art of reading, writing, and speaking the English language correctly." — Goold Brown's Grammar of English Grammars, p. 145, And instead of defining orthography to be the science, or system, NO PREVIOUS DISCOVERT. 87 of the letter, this same author, than whom there is no higher authority among English grammarians, says, " Orthography treats of letters, syllables, separate words, and spelling." — Id., p. 148. Again : " Etymology treats of the different parts of speech, with their classes and modifications." — Id., p. 220. Since, therefore, these grammarians know not that the word is the science trunk, that is, the tree trunk, of grammar, they cannot know that grammar is a tree, — a long way this, from knowing the tree to be the learner's invaluable guide in learning grammar ! If, therefore, any teacher ever knew the orthographic whole to be an orthographic tree, he did, nevertheless, not know such tree to be the invaluable guide in learning and remembering orthography ; and for this reason, that he did not find a useful guide in learning, no record or memorial of such discovery has, to my knowledge, to this day ever been made. Whatever is a great convenience— but the tree is an indispensable guide — to the discoverer as a guide in all learn- ing and remembering, is also a great convenience and an in- valuable guide to the whole teachable world ; for all civilized men have to learn and remember. If, then, I am asked to state in what form the tree organism is to be used, I answer at once, In the form of the orthographic, ideographic, etymologic, grammat- ical, botanical, zoological, and arithmetical trees, or tree- maps, to be found on pages 18, 223, 274, 494, 25, 26, 642, of this book. If I am asked in what school appliances these work-trees are to appear and be used, I answer. In all text-books of these branches, as well as in all text-books of logic and psychology, and all other branches of learning to which the term "branch" maybe appropriately applied. These work-trees are likewise to be placed in all work-books used by students of these branches ; and, enlarged to the di- mensions of geographical wall-maps, they are likewise all to be hung as large wall-maps of these branches on the walls of all school-rooms ; in which text-books, and on which school- room walls, they will be of the same service in learning and THE SYSTEM METHOD. remembering those subjects of which they are tree maps as are the common geographical maps of the topography and political divisions of the earth in learning and remembering such topography and political divisions. I have already shown that, to all rational learning of orthography, the orthographic tree map of the orthographic whole is as essen- tial as the known existence of the cat is to all rational learn- ing of cat's tongue, tail, or back-bone. And I shall hereafter show that the grammatical, etymological, ideographic, botan- ical, and zoological tree maps are not less essential to any real, science-system, or system-guided, learning of these branches, than is the known existence of the United States, as a whole, to any rational learning of the parts, mountain, river, city, plain, etc., contained by such whole. If I am asked to state any further advantages to arise from the discovery and invention of this science-system representative and working guide, I could specify that, — 1. By such tree tool alone can we see what perceiving is. 2. " " ■ " " thinking 3. " " " " comparing 4. " " " " conceiving 5. " " '* " judging 6. " " " " reasoning Y. " " " " defining 8. " " " " learning 9. " " " " knowing 10. " " " " remembering 11. " " " " imagining 12. " " " " systematizing " 13. Such tree instrument or guide, is itself, by virtue of the twelve truths just stated, the only correct, logical, and psy- chological notation. Certainly, it is not claimed that by means of the tree we can absolutely realize what perceiving, thinking, knowing, etc., are. It is only meant that through the instrumentality of the tree alone we can see as plainly as human perception is able to see with the aid of the perfect instrument. .4iY INVALUABLE TOOL. 89 14. By such tree alone can we guide our investigating. 15. By such tree alone can we guide our reciting. 16. By such tree alone can we guide our teaching, so far, at least, as our teaching has to do with science system. lY. By such tree alone can we determine the correct rel- ative order of arithmetical subjects, — whether proportion properly precedes fractions, and these two, compound num- bers ; whether decimals properly precedes common frac- tions and proportion, — where each arithmetical division is properly to be placed. 18. By such tree alone can we ascertain the correct relative order of subjects in orthography, grammar, botany, zoology, and logic. 19. Such tree is, moreover, the only efficient aid in deter- mining what belongs to the empirical stage of learning, and what to the rational, what is proper in primary, intermedi- ate, and advanced grades of learners. 20. By such tree alone can we know by what objective means we know and learn, and what science is. By such in- strument alone, therefore, can we distinguish between real science (/. e., system science) and empirical (/. e., disorgan- ized) science ; between the organizing, easy, or system- guided, method and disorganizing methods. Judge, then, in what vital sense new, of what incalculable use, must be the discovery of this tree " of knowledge " as guide and instrument in almost all school learning. Judge, too, how " dreadfully defective," as Mr. Spencer truly remarks, " the rude, undeveloped character of our education " must be. For I have shown that no previous discovery of this science- system guide hitherto made, could have been weighted with more import?nce as a guide in learning than the Christmas- tree or the illustrative genealogical-tree drawing designed to exhibit some family genealogy. Therein it is seen that no known orthographic, grammatical, etymological, or other tree whole has ever been consciously used as the indispensable instrument in the study of the tree (branch) parts of these branches. And I have pointed out, merely, that to study the 90 THE SYSTEM METHOD. parts of any branch not knowing what the whole is like, — not knowing that such branch parts are parts of a branch whole, — is to study such parts not knowing that a whole exists ; that is to say, it is to study such parts not knowing them to be parts — to study parts as parts of nothing — as not parts ! Yet once more, think what such study is : To study " subvocal," not knowing what the orthographic whole is like, or that such whole exists, is to study subvocal pa7-t not knowing what subvocal's zvliole is like ! to study a tiger's back-bone not knowing what a tiger is like, whether tiger, ostrich, snake, or giraffe ! To study tiger's back-bone or subvocal not knowing that the tiger whole or the orthographic whole exists, is to study such back-bone or subvocal as subvocal and back-bone of nothing ! But what is the study of back-bone as the back-bone of nothing worth? — Nothing, says the trumpet of truth. What, then, is the study of sub- vocal part as part of nothing worth ? — Nothing ! Let the trumpet blow as long as blown by the breath of the truth. Remember to put yourself in the place of the learner, know- ing almost nothing of the subject. Thus situated, imagine yourself to be learning the, to you unknown, thing (not city to you) called Chicago, precisely as students do now learn orthography, grammar, etymology, geography (surface and political divisions excepted), botany, etc., by studying each part separately, and never detecting what the whole is like, if even ever fancying it to exist. Now, while I cannot dispossess you of your great advan- tage, by means of your acquired learning and experience of things, over any such learner, I will place you as a boy of eight years and of ordinary intelligence, reared by your own parents in Patagonia, say, where you have never seen as such any city, city street or plan ; any brick-built, stone- built, or lumber-built business or dwelling-house, or any sidewalk ; any railroad or street-car ; any depot, park, grove, drive, or lake ; just as young learners have never known subvocal, dental, coalescent, diphthong, etc., as such. Now, RIDICULOUS LEARNING. 91 remember that you are not once to see or imagine Chicago as a zv/io/e, just as learners do never (by current methods) see or imagine orthography as a whole ; nay, you are not to know, when you have finished your study, what Chicago as a whole is like, just as learners, having finished their study of orthography, never know what orthography as a whole is like — not even that Chicago exists in the likeness of any known thing, just as learners of orthography know not even that orthography exists in the likeness of any known thing. Remember, especially, that you are not to know or ever de- tect the things which I give you to study, as being parts of any known whole, but merely separate things, just as learn- ers of orthography never know or detect the things studied— subvocal, dental, diphthong, etc. — as being parts of any known whole, but merely as separate things. I give you now a book definition of railroad, lake, and side- Avalk, which you are to commit to memory ; I also point out to you examples of such railroad, lake, and sidewalk, just as learners of orthography are given book definitions of lingual, aspirate, etc., to be committed to memory, examples at the same time being pointed out. A second day I give you book definitions of house, park, and street-car, and point out examples to you, just as modern teachers give, on a second day, book definitions of vocal, coalescent, etc., pointing out corresponding examples. For a third lesson, you again com- mit to memory book definitions of depot, drive, and grove, and review the definitions, or the " substance of what the book says," in your first lesson, just as school children are now, in their third lesson, directed to "get" what a book says about subvocal, abrupt, etc., and review "what the book says" about their first lesson. (Here, not to make the farce as prolix and vicious as it really is, we will stop.) You are now "through." You are now not to know — mark it well — that such railroad, lake, sidewalk, house, park, street-car, depot, drive, grove, etc., are parts of one whole, just as our students of orthography when " through " do not know — mark well this fact, too — that such dental, coalescent, 92 THE SYSTEM METHOD. subvocal, diphthong, etc., are parts of one whole. You are not even to know that any Chicago whole exists, precisely as all school, college, and university students of orthography never know, after getting " through," that any orthographic whole (orthographic tree) exists. You are only to know that you have been stwdymg from a book about Chicago — a thing to you unknown as a whole both before and after your study of railroad, lake, etc. ! just as students of orthography do only know that they have been studying from a book about orthography — a thing to them unknown as a whole both before and after their study of dental, subvocal, diph- thong, etc ! You are not even to know that there exists a Chicago whole, and that it is a Chicago city ! precisely as no student of orthography knows that there exists an ortho- graphic whole, and that it is an orthographic tree ! You are never to see the railroad, lake, depot, etc., as parts of a cit^ (whole), but are simply to hold these city parts in mind by dogmatic definition as parts of no whole — parts of nothing ! just as students of orthography, under " great " teachers, never see subvocal, vocal, dental, diphthong, etc., as parts of the orthographic tree ; just as pupils of "great professors," the world over, by dogmatic definition do hold such parts in mind — till they "drop out" — as parts of nothing! It has been demonstrated that true knowing, or learning, is branching, or organizing, — branching or organizing the science-system, or tree-trunk, unit through the branch (z. c, the tree) whole, using that tree whole as guide in the branch- ing work. But branching tree-trunk letter can only result in making letters into a tree, or branch-organism, whole, through which (from tree trunk upward) all letters may be traced, or branched. And so branching the class zvord through its science, or tree, system, results necessarily in a copy of such tree-system whole. And so branching (z. e., rationally learning) any branch of learning is but making a branch, or tree, of it. That rational, or perfect, branching the branch orthography must needs result in a branch, or tree, of that subject (as shown on pp. 18, 203), is as self-evi- RTDICULOrS LEARNING. 93 dent as that perfect copying of any specimen must needs re- sult in a copy of such specimen. In learning, the process is branching ; therefore, the result must be a branch. In writ- ing, the process is copying ; therefore, the result must be a copy. Mark once again what that copying is worth which, when finished, does not know what the copy made, or copied, is like. Judge, then, what that organizing, or branching (z. e., learning), is worth, which, when finished, does not know what the branch, or organism, made is like. Who, then, cannot yet see the too-great-to-be-estimated value of the tree organism, by which alone, of all nature's products, the sub- ject whole of any branch of learning may be profitably rep- resented .'' Bread-and-butter school-boys know that just to that proportional degree of clearness in which writing learner sees mental copy to be reproduced (z. e., written), just to that extent can he do perfect writing (perfect copying) ; just to that extent is his learning work of any value. What wakeful teacher, then, cannot perceive that just to that pro- portional degree of clearness in which branching learner sees the mental copy, or branch, to be branched ; just to that ex- tent can he do perfect branching (perfect copying of the branch) ; just to that extent is his learning work of any worth.'' "Graduating" learners, or doers, of writing copy who have obtained no mental picture of the written copy, who do not know what such written copy is like, — whether like curve or animal or else, — and who are therefore utterly unable to do (/. e., write) any such copy ! " Graduating," then, learners, or doers, of an orthographic branch copy who have no mental picture of this branched copy, who do not know what such branched copy is like,— whether like tree or animal or else,- — and who are therefore utterly unable to do (z. e., branch) any such copy ! " Graduating," still, learners, or doers, of ideographic, etymological, grammatical, arith- metical, botanical, and zoological branch copies who have obtained no mental picture of the branched copies, who do not know what such branched, or organized, copies are like, 94 THE SYSTEM METHOD. — Avhether like trees or some other organism, — and who are therefore utterly unable rationally to do the branching (learn- ing) of such copies ! Let not the reader fancy this merely a burlesque on the schools. It is the literal truth. " Graduating," once again, learners, or doers (/. c, map-makers), of a map-made copy of the United States who have no mental picture of the map- made copy, who do not know what such United States map- made copy is like, — whether like United States map or animal or what, — and who are therefore utterly unable to do (/. e., make) any such map-made copy ! If you would not "graduate" students, or doers, of United States map-making or writing, who, when " through " the complete course, have absolutely no mental picture of such writing copy or United States, as a whole, who do not know even what such copy or map whole is like, — whether like curve, country, animal, or else, — would you, then, " graduate " student, or doer, of orthographic, grammatical, and botanical branch-making, who, when " through " the course, have absolutely no rational mental picture of such branch copy or whole, who know not even what such orthographic, grammatical, or botanical whole is like } Who, then, Avill not see the too-great-to-be-estimated value of this tree " of knowledge " as the representative of the science-system whole of every branch of learning, and the only possible working instrument by which student may do this branching work called learning } Absolutely all educational authorities agree that the en- tire flowing current of the educational river, the entire bus- iness of learning, is mirroring part in whole, and whole in part. Inductive philosophy, it is declared, thinks from part up to the whole ; deductive philosophy, from whole down to part ; and all philosophy, learning, or knowing, goes one way or the other. The great educational question is and has been. What is the system (/. e., science) whole of letters ? of words .? of plants } of animals .? etc., in which the class di- visions, or branches, of letter, word, plant, animal, etc., are to be mirrored } This whole once known, the learner's proc- MIRRORING WHOLE IN FART. 95 ess of work is inevitable. For if such system whole is a building, the process must be building ; but, if such system whole is a branched whole, the process must be branching. So that the discovery of the system whole reveals the stu- dent's process of work. And if the evidence offered in this publication is not spurious, the learner's true process of work is branching ; and his true and only guide, or instrument, the branch, or tree organism. Thus it is that the discovery of the tree system as the only sensible representative of a science system, divulges the student's process of work, and therein the true method of teaching. And it is not to go unobserved that such discovery " embodied in a new com- bination of parts," as shown in all the tree engravings found in these pages, amounts to a demonstration of what the science-system parts are — what the orthographic parts, sub- vocal, aspirate, vocal, etc., are ; what the grammatical parts, substantive, verb, noun, adjective, infinitive, direct objective, etc., are. Since, by the discovery, we know the system whole to be a tree, or branch, we therein know the system parts to be branches, or tree organs ; and the process of mirroring part in whole, which authority calls learning, we know to be branching. Since absolutely all authority avers that learn- ing is making wholes out of parts (rather, that learning is tracing the multitude of things through their classes so branched together in tree system that each twig class is a branch of every higher class, including the highest class, the tree trunk) ; and since the parts are branches (branch classes), and the wholes are trees, it follows that learning is branch- ing, or organizing, as shown on pages 28, 103, 274, and 603. Thus, by deductive reasoning, we may go from the discov- ered tree-organism whole down to the part to prove that to learn is to branch, which truth I have so repeatedly demon- strated by the opposite, inductive, reasoning, by going from particular to class, or part to whole. But not to digress further : it is thus plain that just to that proportional degree of clearness in which learner sees mentally the tree, or branch, whole to be made (z. e., branched 96 THE SYSTEM METHOD. through), just to that proportional extent is his learning- of any worth. Now, let the reader himself decide how many (how few !) " graduates " of orthography, English grammar, etymology, botany, zoology, and arithmetic have a clear mental picture of such subject whole. But we have seen that no teacher, philosopher, nor man has ever realized what sys- tem such science-system whole is. See pages 41 and 642. If, then, all really rational learning of these branches is what all authority declares it to be, namely, associating their parts into a whole (which I have discoverd to be accomplished by tracing, or branching, thing to be learned, through its classes as branched into a tree of these classes), judge the inestimable value of the discovery, for the first time realizing what such whole is. What would be the value of a mental picture (i. e., known plan) in making, or building, parts into a build- ing whole } What kind of a builder would you really be, if, after having done the so-called building, you did not know what the built whole was like — built thing or what not "^ If all authority should declare your building worthless, and your claim to proficiency simple foolishness, as all author- ity actually does do, would your sagacity of mind be suffi- ciently electrified to enable you to see and realize the abso- lute necessity and invaluable character, in all building, of a definite mental picture, or plan, of the building whole 1 Tell, then, what sort of learners these are, when, after having done the so-called learning (J. e., branching), they know not what the learned (z. c, branched) whole is like — branched tree or what not ! Judge, too, whether, if all authority should de- clare their learning worthless, and their claim to proficiency simple foolishness, as all real authority actually does do, your sagacity of mind would be sufficiently electrified to enable you to see and realize the absolute necessity and in- valuable character, in all learning, of a definite mental pict- ure of the branched whole. " Let no man deceive himself If any among you seemeth to be wise in this world, let him become a fool, that he may be wise." BIRD'8-ETE VIEW 97 The Orthostatic Work-Tree. — Having seen a little of how " dreadfully defective " must be our teaching and school- work in orthography, let it be shown how the invention of the orthostatic tree will lead us quickly into the perfect method. It has been demonstrated that the very knowing act thinks particular into class previously known (branched). But when you think particular into a class itself previously known (branched), z. e., previously thought into a greater known (branched) class, which, in order, is a class itself already thought into a still greater known (branched) class, you are therein thinking, or mentally constructing, a tree system. This also has been many times proven. And now mark once again that since knowing is branching particular into kind, — itself a known, i. e., branched, thing, — nothing but such branched system can constitute the educational guide. No painted or knitted or otherwise-made copy can be guide in writing. Nothing but writing copy can be used as guide in writing. So, in learning, i. e., branching, nothing but branching copy can be used as guide. This branching guide I have named the orthostatic work-tree (from og-&oQ, right, and laTavaL, to stand), as being the only natural whole known whose parts stand together in such form of unity as to indicate the right method and order in learning and teach- ing. I shall, then, in the following pages give a quick sur- vey, or bird's-eye view, of what are the two steps in learning rational orthography by the branching, natural, scientific, or true method as indicated by the orthostatic tree. But in entering upon any branch of learning, or branching, it is desirable and very important to give the student, at the out- set, a general idea of what is the nature of the branch to be pursued. This can be done only by showing the connection of the branch with the other branches of the whole ortho- static tree. Using the tree-map here following, an enlarged copy of which should hang in every school-room, ask your class (as the first exercise) to help you organize the particulars — say cal, f , 5, re, sjiper, pansy, water, b, He, e, SOO, a, rose, o, g in 7 98 THE SYSTEM METHOD. sign, liver, eye, 6, ^, H and O in H-0, Na and CI in NaCl, rtm, of, un and health and ful and ness in unhealthfulness, 1000, or any line, surface, or solid. Go from the body thing up the tree, which is to be your LEARNER'S WORKI-TREE. Copyright, 1886, by I. E. T\TLSON. guide or program of work. As thus Avorked out by teacher and class, and written on the blackboard or merely traced up the limbs of the tree by teacher, this first introductory, or bird's-eye-view, lesson will appear about as follows (the teacher may omit anything not familiar to him) : — BIRD'S-EYE VIEW. 99 Cat = Thing, qualitative. natural. organic, animal. (zoology). Ya = Thing, quantitative. number. discontinuous. fractional. (arithmetic). Re = Thing, qualitative. lingual. etymon, prefix. (etymology). 5 =^ Thing, quantitative, number. discontinuous. integral. (arithmetic). Super ^ Thing, qualitative. lingual. etymon. prefix. (etymologj-) . Pansy = Thing, qualitative. natural. organic. plant, (botany). Water = Thing, qualitative, natural. inorganic, molecular. (physics). B = Thing, qualitative. lingual, letter. orthograph, (orthography) . He = Thing, qualitative. lingual. etymon, suffix, (etymology). E = Thing, qualitative. lingual. letter. orthograph. (orthography). 200 = Thing, quantitative, number. discontinuous, integral, (arithmetic). A ^ Thing, qualitative. lingual. letter. orthograph. (orthography). Rose ^ Thing, qualitative. natural. organic, plant, (botany 1. = Thing, qualitative. lingual, letter, orthograph. (orthography). G =- Thing, qualitative. lingual. apththong, ideograph. (ideography). Liver = Thing, qualitative. natural. organic. animal, (zoology). Eye = Thing, qualitative. natural. organic. animal. (zoology). 6 = Thing, quantitative, number, discontinuous, integral. (arithmetic^ . % = Thing, quantitative, number. discontinuous. , fractional. (arithm.e'.ic). H ^ Thing, qualitative. natural. inorganic. atomic, (chemistry). = Thing, qualitative. natural. inorganic, atomic, (chemistry). Na = Thing, qualitative, natural. inorganic, atomic. (chemistry). CI ^ Thing, qualitative, natural. inorganic, atomic. (chemistry). Run = Thing, qualitative, lingual. vvford, fundamental 1, (grammar) . Of = Thing, qualitative, lingual, word. connective. (grammar). Un = Thing, qualitative. lingual. etymon. prefix. (etymology). Health = Thing, qualitative. lingual. etymon. root. (etymology). Ful = Thing, qualitative. lingual, etymon, suffix. (etymology). Ness ^ Thing, qualitative. lingual. etymon. suffix, (etymology). 1000 = Thing quantitative, number. discontinuous , integral, (arithmetic). A line ^ Thing, quantitative, magnitude. (geometry). A surface = Thing, quantitative, magnitude. (geometry). A sphere = Thing, quantitative. magnitude, (geometry). Thus you lead your class to see what is the place occu- pied by the branch of orthography, upon the study of which they are about to enter. But do not fail to show how the orthographic branch, as shown on page 18, is but the contin- uation of the limb, or branch, marked "orthograph" in this tree. To do this, you might take any letter, say b, and run it entirely up through this general stump tree, and then con- tinue up through the orthographic tree. You would, of course, proceed thus : B = Thing, qualitative, lingual, letter, orthograph, sonant or voiced, subvocal, abrupt, labial, thus reaching the last class to which it belongs. In tracing b in this way, you will use, of course, first the stump tree, and then the orthographic tree, maps of both supposed to hang on the school-room walls. I do not say that your class will fully understand this 100 THE SYSTEM METHOD. work with the stump tree. I have found in every such trial that they understand very little ; but the doctrine that we may " never teach a child what it does not fully understand, is," as William Henkle, of Ohio, and others have declared, " rank educational heresy." This is merely giving the class a " balloon view," so to speak, that they may know of their whereabouts, in the great tree of knowledge, as they proceed with orthography proper. You need not desire or expect them fully, or more than very dimly, to understand, just as you would not expect a student of Chicago fully to under- stand at the first lesson, if, in giving him that first lesson, you should elevate him in a balloon, and furnish him with a powerful telescope that he might see in what part of this great Mississippi Valley Chicago is situated. But, besides giving them such.balloon view of the great tree of knowledge, this exercise will go a great way to establish their confidence in you as a teacher, in your scholarship ; just as it would establish your faith in the proficiency and equipment of your teacher of Chicago, if, at the outset, having furnished you with a powerful telescope, he should pleasantly elevate you in a balloon or otherwise to where you could see the relation of Chicago to lake. State, Mississippi Valley, and United States, the relation of its chief parts to each other, etc. This exercise will further give the class a general notion of what the organizing, branching, or system, method is, — a method the steps in which are determined by and follow the branches of the- orthostatic tree. If, however, the teacher be not com- petent to trace up the stump tree some such particulars as those above traced, let it be wholly omitted. It is in no wise essential to good and superior teaching of orthography. The first lesson essential will, of course, be to trace from six to twenty letters up the orthographic tree proper, or up some special branches of this tree. This introductory, or bird's- eye-view, lesson is only to be given when the teacher is somewhat familiar with all the branches of the great tree of knowledge ; it is to be wholly omitted by others. But if given, it will serve to indicate to the pupils what place INTRODUCTORY LESBOK 101 orthography occupies among the important branches of learning, to establish their faith in the teacher's proficiency and scholarship, and to bring about a realizing sense of their own ignorance of the whole tree of knowledge — an end as worthy as establishing their faith in the teacher's scholarship. Bird's-Eye View of the Branching Process. — The sys- tem method — organizing, natural, scientific, whole-and-part, easy — of learning every branch, is by branching that branch. And this organizing, branching, or whole-and-part, method comprises two processes of work, both guided, of course, by the orthostatic tree, the only representative of a branch whole. These two processes correspond to Sir Wm. Hamilton's Analysis and Synthesis, which, he declares, are " the two nec- essary parts of the same method," the " only possible method of philosophy." Both processes are to be guided by the orthostatic tree ; either is inefficient without it. Analysis is working up the tree whole, synthesis is working down the tree whole, of the subject. Analysis is the first step, synthe- sis is the last step. But the learner is not only to analyze both graphically and orally, but he is likewise to synthesize both graphically and orally. I shall give this bird's-eye, or general, view, then, in the following order : 1. Written Anal- ysis ; 2. Oral Analysis ; 3. Written Synthesis • 4. Oral Synthesis. Written analysis by the work-tree. — The follow- ing will represent the nature of what is to be assigned to students in the first part of the school course in orthogra- phy : (1.) Cat ; (2.) Bed ; (3.) Gate ; (4.) Cane ; (5.) Bay ; (6.) Beef; (Y.) Shop ; (8.) Yeast ; (9.) Bowl ; (10.) Beau ; (11.) Eye ; (12.) Adieu ; (13.) Noble ; (14.) Often ; (15.) Agree- able ; (16.) Nation; (17.) Filial ; (18.) Quack. And the following will represent the nature of the stu- dent's work. Observe that this process of work uses the tree whole as program, or guide, and runs by analysis from the body orthograph up the tree, thus bringing out the at- tributes of, or analyzing (from avakveiv, to unloose again, as the parts or attributes), each particular assigned to be worked 103 THE SYSTEM METHOD. up by the class. The written work done by the student in this first or analytic part of his school course in orthography by this system or organizing method, would appear, in gen- eral, about as follows : — ORTHOGRAFHIC WORIC-TREE. Copyright, 1886, by I. E. "WnLSON. I. C = Letter, aspirate, abrupt. guttural. A = Letter, sonant. vocal, simple. palatal T = Letter, aspirate. abrupt. dental. 2. B = Letter, sonant, subvocal. abrupt, labial. E = Letter, sonant. vocal, simple. palatal. D = Letter, sonant, subvocal. abrupt. dental. Or, DIGRAPH, DIPHTHONG, TRIGRAPH. WRITTEN ANALYSIS. 103 3- G = Letter, sonant. subvocal, abrupt, guttural. A = Letter, sonant, vocal, simple, dental. T := Letter, aspirate, abrupt, dental. E = Letter, aphthong. 4- C = Letter, aspirate, abrupt. guttural. A = Letter, sonant, vocal, simple. dental. N — Letter, sonant, subvocal. continuant, dental. E = Letter, aphthong. 5- B = Letter, sonant, subvocal, abrupt, labial. Ay -= Digraph, sonant, vocal, simple. dental. 6. B = Letter, sonant, subvocal, abrupt, labial. Ee = Digraph, sonant, vocal. simple, dental. F z= Letter, aspirate, continuant, labial. 7- Sh = Digraph, aspirate, continuant. palatal. =z Letter, sonant, vocal. simple. guttural. P = Letter, aspirate. abrupt. labial. 8. Y — Letter, sonant. subvocal. coalescent. dental. Ea = Digraph, sonant. vocal. simple. dental. S = Letter, aspirate, continuant, dental. T = Letter, aspirate, abrupt. dental. 9- B = Letter, sonant. subvocal, abrupt. labial. Ow = Digraph, sonant. vocal, simple. labial. L = Letter, sonant. subvocal, continuant, dental. lO. B =r Letter, sonant, subvocal, abrupt, labial. Eau = Trigraph, sonant, vocal. simple. labial. II. Eye = Trigraph, sonant. vocal. compound. open. 12. A = Letter, sonant, vocal. simple. guttural. D = Letter, sonant, subvocal, abrupt, dental. leu = Trigraph, sonant. vocal, compound. close. 13- N = Letter, sonant, subvocal. continuant, dental. = Letter, sonant, vocal. simple, labial. B = Letter, sonant. subvocal, abrupt, labial. L = Letter, sonant, subvocal, continuant. dental. E = Letter, aphthong. 14. = Letter, sonant, vocal. simple, guttural. F = Letter, aspirate, continuant, labial. T = Letter, aphthong. E = Letter, aphthong. N = Letter, sonant, subvocal, continuant. dental. IS- A = Letter, sonant, vocal. simple, guttural. G = Letter, sonant, subvocal, abrupt, guttural, R = Letter, sonant. subvocal. continuant, palatal. Ee = Digraph, sonant, vocal. simple. dental. A = Letter, sonant. vocal, simple. guttural. B = Letter, sonant, subvocal, abrupt, labial. L = Letter, sonant, subvocal, continuant. d'ental. E = Letter, aphthong. 104 THE SYSTEM METHOD. i6. N = Letter, sonant, subvocal. continuant, dental. A = Letter, sonant. vocal, simple. dental. Ti = Digraph, aspirate. continuant, palatal. O = Letter, sonant, vocal, simple, guttural. N = Letter, sonant, subvocal, continuant, dental. 17. F = Letter, aspirate. continuant. labial. I = Letter, sonant, vocal. simple, palatal. L = Letter, sonant, subvocal, continuant, dental. I = Letter, sonant. subvocal, coalescent, dental. A Letter, sonant. vocal, simple. palatal. L = Letter, sonant. subvocal, continuant, dental. 18. Q = Letter, aspirate, abrupt. guttural. U = Letter, sonant. subvocal, coalescent, labial. A = Letter, sonant. vocal. simple, palatal. C = Letter, aph thong. K = Letter, aspirate. abrupt. guttural. I have now given a general idea only of what is the direction and written form of this first of the two processes by which the student is to proceed in the mastery of orthog- raphy. It is called analysis (avalvsiv, to unloose), because, as the student proceeds up the tree program, or guide, he men- tally unlooses the several attributes, or characteristics, of the particular in hand. Thus of b, the student says, (1.) Letter, 2.^., written representative of an articulate sound ; (2.) sonant, i. e., voiced sound ; (3.) subvocal, i. e., an obstructed voiced sound ; (4.) abrupt, i. e., a sound abruptly cut off; (5.) labial, i. e., a sound obstructed at the lips {labia). In this analytic, or up-the-tree, process, the student thus unlooses in b five distinct attributes, or characteristics ; hence, the process is called analysis, which means unloosing the attributes, or parts, of the whole analyzed. This analytic process, supple- mented by the synthetic, or finishing, process, I have called the System Method, i. e., the truly organizing method ; be- cause every step in either is guided by organizing, or branch- ing, particular into organized, or limbed, class — by limbing the less into the more comprehensive : so that, when the student has finished his study, he carries a mental picture of the whole branch as a branch in reality. I shall not stop here to re-demonstrate the inestimable value of this ortho- static tree in either teaching or constructing the science of ORAL ANALYSIS. 105 orthography. Suffice it here to say that it cannot be less valuable and indispensable in teaching orthography than a map of the United States in teaching the political geography of the United States ; for of this the only representative whole that can be made visible to the eye at one view, is the United States map, while of orthography the only represent- ative whole that can be made visible at one time is the orthostatic tree of orthography. Farther on, after the entire daily work of the student is made out, I hope to show how every demand of logician, psychologist, and educator, — every demand of the perfect method, — is fully met in this invention of the orthostatic work-tree, the logical product of the dis- covery that every possible act of thought limbs particular through and into class. Oral analysis by the work-tree.— Neither am I here to digress to explain how the student is led up to the power of working out this oral analysis, as it here follows. This will be explained farther on. Remember that I wish here to give a bird's-eye view merely, ^ — to give the features of the student's organizing work in dim aspect only, — reserv- ing a full explanation of the superior riches of the method until the complete work of the student shall have been given. The typical form of this oral analysis, then, would be what follows. Observe that it, too, proceeds by coherent speech tip the orthographic tree, unloosing the attributes of the particular in hand precisely in the same order as unloosed in the written analysis. Program Made up from Work-Tree.- — 1. the i. Antecedent. 2. Base. 3. Consequent. 2. Is a I. Vowel, 4. Trigraph, 3. Representing 1. Its own proper sound, or 2. Consonant, 5. Apththong, 2. What sound of what letter. 3. Digraph, 6. Diphthong. 4. Which is a I. Simple vocal, 5. i- Perfectly obstructed at the (I.) Lips, 2. Compound vocal, 2. Partially obstructed at the (2.) Teeth, 3. Subvocal, 3. Molded (not obstructed) at the (3.) Hard palate, 4. Aspirate. (4.) Soft palate. 6. Producing a I. Labial, 7. And an i. Abrupt. 2. Dental, 2. Continuant. 3. Palatal, 3. Coalescent. 4. Guttural. 106 TEE SYSTEM METHOD. Take now, for representative particulars to be assigned to the class to be orally analyzed, the letters of the following words, which are the same words used above in the written analysis : (1.) Cat ; (2.) Gate ; (3.) Boy ; (4.) Beef; (5.) Shop ; (6.) Yeast ; (7.) Beau ; (8.) Eye ; (9.) Often ; (10.) Agree- able ; (11.) Nation ; (12.) Ocean ; (13.) Filial ; (14.) Quack. The Words A7ialyzed.—\. C, the antecedent of the base, is a consonant, representing its own hard sound (enunciate that sound), which is an aspirate perfectly obstructed at the soft palate, producing a guttural and an abrupt. Note. — Observe how, as the language proceeds, the thought con- stantly ascends to the top of the tree, thus unfolding, (i) consonant, (2) aspirate, (3) guttural, (4) abrupt, as marks, or attributes, of c, which marks, taken together, constitute its "whole of content," and completely identify or individualize it. Observe, especially, how- ever, that this oral analysis not only requires the student to discover these four attributes of c, but that it presupposes and prerequires, on the student's part, before recitation hours, a ^^/;z^ mastery of ^. But mark that this doing mastery, which is able thus freely to talk about a problem of any kind, is a mastery of such problem from all sides. Such all-sided work and mastery is not to be attained except by a sys- tem-guided method of work. The perfectly all-sided knowledge of the orthographic materials will, however, only come after the student has been made proficient in the second, or synthetic, process, yet to be illustrated. A, the base, is a vowel, representing its own short sound a (enunciate), which is a simple vocal molded at the hard palate, producing a palatal and a continuant. T, the consequent of the base, is a consonant, repre- senting its own proper sound t (enunciate), which is an aspi- rate perfectly obstructed at the teeth, producing a dental and an abrupt. 2. G, the antecedent of the base, is a consonant, repre- senting its own hard sound g (enunciate), which is a subvocal perfectly obstructed at the soft palate, producing a guttural and an abrupt. A, the base, is a vowel, representing its own long sound a (enunciate), which is a simple vocal molded at the teeth, producing a dental and a continuant. ORAL ANALYSIS. lOY T, the immediate consequent of the base, is a conso- nant, representing its own proper sound t (enunciate), which is an aspirate perfectly obstructed at the teeth, producing a dental and an abrupt. E, the remote consequent of the base, is an aphthong. 3. B, the antecedent of the base, is a consonant, repre- senting its own proper sound b (enunciate), which is a sub- vocal perfectly obstructed at the lips, producing a labial and an abrupt. Oy, the base, is a diphthong, representing its own proper sound ai (enunciate), which is a compound vocal molded at the soft and hard palates, producing a palato-gut- tural and a continuant. 4. B, the antecedent of the base, is a consonant, repre- senting its own proper sound b (enunciate), which is a sub- vocal perfectly obstructed at the lips, producing a labial and an abrupt. Ee, the base, is a vowel digraph, representing the long sound of e (enunciate), which is a simple vocal molded at the teeth, producing a dental and a continuant. F, the consequent of the base, is a consonant, represent- ing its own proper sound f (enunciate), which is an aspirate partially obstructed at the lips, producing a labial and a con- tinuant. 5. Sh, the antecedent of the base, is a consonant digraph, representing its own proper sound sh (enunciate), which is an aspirate partially obstructed at the hard palate, produc- ing a palatal and a continuant. O, the base, is a vowel, representing its own short sound 6 (enunciate), which is a simple vocal molded at the soft pal- ate, producing a guttural and a continuant. P, the consequent of the base, is a consonant, represent- ing its own proper sound p (enunciate), which is an aspirate perfectly obstructed at the lips, producing a labial and an abrupt. 6. Y, the antecedent of the base, is a consonant, repre- senting its own proper sound y (enunciate), which is a sub- 108 THE SYSTEM METHOD. vocal partially obstructed at the teeth, producing a dental and a coalescent. Ea, the base, is a vowel digraph, representing the long sound of e (enunciate), which is a simple vocal molded at the teeth, producing a dental and a continuant. S, the immediate consequent of the base, is a consonant, representing its own proper sound s (enunciate), which is an aspirate partially obstructed at the teeth, producing a dental and a continuant. T, the remote consequent of the base, is a consonant, representing its own proper sound t (enunciate), w^hich is^an aspirate perfectly obstructed at the teeth, producing a den- tal and an abrupt. Y. B, the antecedent of the base, is a consonant, repre- senting its own proper sound b (enunciate), "which is a sub- vocal perfectly obstructed at the lips, producing a labial and an abrupt. Eau, the base, is a vowel trigraph, representing the long sounds of o (enunciate), which is a simple vocal molded at the lips, producing a labial and a continuant. 8. Eye, the base, is a vowel trigraph, representing the long sound of i {= ae, enunciate), which is a compound vocal molded at the soft palate and the teeth, producing a dento-guttural and a continuant. 9. O, the base of the first syllable, is a vowel, represent- ing its own short sound o (enunciate), which is a simple vocal molded at the soft palate, producing a guttural and a continuant. F, the immediate consequent of the first syllable, is a consonant, representing its own proper sound f (enunciate), which is an aspirate partially obstructed at the lips, produc- ing a labial and a continuant. T, the remote consequent of the base of the first sylla- ble, is an aphthong. E, the antecedent of the subvocal base of the second syllable, is an aphthong. N, the subvocal base of the second syllable, is a con- ORAL ANALYSIS. 109 sonant, representing its own proper sound n (enunciate), which is a subvocal partially obstructed at the teeth, produc- ing a dental, a continuant, and a nasal. 10. Ee, the base of the second syllable, is a vowel digraph, representing the long sound of e (enunciate), which is a sim- ple vocal molded at the teeth, producing a dental and a con- tinuant. L, the subvocal base of the third syllable, is a conso- nant, representing its own proper sound 1 (enunciate), which is a subvocal partially obstructed at the teeth, producing a dental, a continuant, and a liquid. E, the consequent of the base of the third syllable, is an aphthong. 11. Ti, the antecedent of the base of the second syllable, is a consonant digraph, representing the sound of sh (enun- ciate), which is an aspirate partially obstructed at the hard palate, producing a palatal and a continuant. 12. Ce, the antecedent of the base of the second syllable, is a consonant digraph, representing the sound of sh (enun- ciate), which is an aspirate partially obstructed at the hard palate, producing a palatal and a continuant. 13. I, the antecedent of the base of the second syllable, is a consonant, representing the sound of y (enunciate), which is a subvocal partially obstructed at the teeth, producing a dental and a coalescent. 14. U, the immediate antecedent of the base, is a conso- nant, representing the sound of w (enunciate), which is a subvocal partially obstructed at the lips, producing a labial and a coalescent. I have now shown, imperfectly, what analysis is, — how by means of the learner's work-tree, by working upward, or ana- lytically, we unloose the attributes of a letter. But to con- stitute that whole-and-part, or truly organizing, method, which I have called the System Method, requires both analy- sis and synthesis. We have to work both up and down the orthographic tree before we are able to see the " one in the many " — before we can fully understand the class parts. 110 THE SYSTEM METHOD. labial, abrupt, subvocal, sonant, letter, etc., as mutually re- lated branch parts of one single tree, or guiding, whole. Ob- serve that all the thinking of life runs down, not up, the tree. We think down the tree : b \s s, labial, b is an abrupt, b is a subvocal, <5 is a sonant, b is an orthograph, b \s 3. letter, thus thinking always down the tree from particular to class. We cannot reverse our thinking, and say, A letter is b, a sonant is b, a subvocal is b, etc. ; because such statements are false. Thus, since all life's doing is guided by thinking, and all thinking is false that goes not down the tree from particular to class, to prepare them for life we are to make our pupils proficient in thinking down the tree ; i. e., in synthesizing, or seeing all the classes (labial, abrupt, subvocal, sonant, and letter) under one and the same view. Thus, in running b down the tree, to get started is to see beforehand every class, or attribute (labial, abrupt, subvocal, sonant, and letter), at one view. To see merely that b \s 3. labial, and start down on any top branch labial, is not suffi- cient ; for/ and m are likewise labials. It is not even suffi- cient at one view to see two attributes — to see b as being (1) labial, (2) abrupt ; for /, also, has both these marks, or attri- butes. It is only sufficient, when we see b in any one class branch, therein to see b in every class branch ; i. e., in the whole tree of class branches ; just as with our eyes fixed on any one part or organ in a familiar whole, say a man whole, we can instantly put our fingers on any other organ or part of such man whole, even without changing our eyes from such part on which they are first fixed. Should student see in b only these two attributes (labial and abrupt), he would be as likely to start down at the /-limb as at the b-Yimb ; but should he start down the /-branch, at the third step he would run into aspirate, and thus make the mistake of calling b an aspirate. In order to get started in the right place, and see and tell the truth of b, the student must see down the tree at least three attributes (class branches) at one view, or mental glance. He must see b as (1) labial, (2) abrupt, (3) subvocal. But the same view that sees these three top STJVTIIESIS AFTEB ANALYSIS. m branches, will always see sonant and letter (or orthograph) as a fourth and a fifth class, or branch ; just as a view that sees a finger, sees also the hand of which that finger is a part. When we see an ear by itself, such ear, we say, " is off" Thus, before the student knows b down the tree as he must think it in life ; that is to say, before the student can know b as he must think b every time he ever thinks about it in a rational and comprehending thought, he must be able to synthesize its attributes (z. e., its classes) — must be able to see these attribute classes (labial, abrupt, subvocal, sonant, and letter) at one mental view. So that the second, last, and elaborating process of learning is synthesis. Neither shall I again digress to demonstrate that all effi- cient synthesis in orthography, is impossible without the aid of the learner's work-tree ; for no mind can effectively con- template five attributes (labial, abrupt, subvocal, sonant, let- ter) together, except as the attributes, or classes, may be seen as corresponding to the connected parts of some natu- ral and familiar whole. But the tree is the only natural whole in the universe of familiar system wholes whose parts are so connected as at all to represent such attributes, or classes, together in one view. By means of the learner's work-tree, however, we are not only able mentally to see these attributes (class branches) at one view, but, by means of such tree, we are able both rationally to learn and tena- ciously to remember how to decide any question concern- ing b which can. be answered by down-the-tree, i. e., sane, thought. No Current Method Correct. — But (1.) since all life's think- ing and remembering of ^-classes (/. e., (^-attributes) is syn- thesis, as I have in different places shown, and as Mr. Spen- cer, Sir Wm. Hamilton, and all authority agree ; and (2.) since all comprehending synthesis of (^-classes (/. e., (^-attri- butes) is impossible without the aid of the work-tree ; there- fore, (3.) no current school method of learning and remem- bering b (or other orthographic units) is the correct one ; for no current school method uses such orthographic tree as 112 THE SYSTEM METHOD. guide, or instrument, by means of which alone such rational and efficient synthesis is possible. Herbert Spejicer: "The analytical habit of mind has to be supplemented by the synthetical habit of mind. Seen in its proper place, analysis has for its chief function to prepare the way for synthesis ; and to keep a due mental balance, there must be not only a recognition of the truth that syn- thesis is the end to which analysis is the means, but there must also be a practice of synthesis along with a practice of analysis." — The Study of Sociology. Sir Wm. Hamilton : " This is the fundamental procedure of philosophy, and is called by a Greek term. Analysis. But though analysis is the fundamental procedure, it is still only a means toward an end. We analyze only that we may comprehend (synthesize, kiiow, or reason) ; and we compre- hend only inasmuch as we are able to reconstruct (synthe- size), in thought, the complex effects which we have ana- lyzed into their elements. This mental reconstruction is, therefore, the final, the consummative, procedure of philoso- phy, and it is familiarly known by the Greek term, Synthesis. Analysis and synthesis, though commonly treated as two different methods, are, if properly understood, only the two necessary parts of the same method. Each is the relative and correlative of the other. Analysis without a subsequent synthesis, is incomplete ; it is a means cut off from its end. Synthesis without a previous analysis, is baseless ; for syn- thesis receives from analysis the elements which it recom- poses. And as synthesis supposes analysis as the prerequi- site of its possibility, so it is also dependent on analysis for the qualities of its existence. The value of every synthesis depends upon the value of the foregoing analysis. . . . " These two relative procedures are thus equally necessary to each other. On the one hand, analysis without synthesis affords only a commenced state, only an incomplete knowl- edge ; on the other, synthesis without analysis is a false knowledge, that is, no- knowledge at all. Both, therefore, are absolutely necessary to philosophy ; and both are, in SYNTHESIS AFTER ANALYSIS. 113 philosophy, as much parts of the same method as, in the an- imal body, inspiration and expiration are of the same vital function. But though these operations are each requisite to the other, yet were we to distinguish and compare what ought only to be considered as conjoined, it is to analysis that the preference must be accorded. An analysis is al- ways valuable ; for though now without a synthesis, this synthesis may at any time be added ; whereas a synthesis without a previous analysis is radically and al^ initio null. "So far, therefore, as regards \\\q first end of philosophy, or the discovery of causes, it appears that there is only one possible method, — that method of which analysis is the foun- dation, and synthesis the completion. In the second place, considering philosophy in relation to its second end, the car- rying up our knowledge into unity, the same is equally ap- parent." — Bowefts Hamilton's Metaphysics, pp. 61, 62. If it is true, as Sir Wm. Hamilton avers, — 1. That analysis and synthesis are " the two necessary parts of the same method ;" 2. That " analysis without synthesis affords only a com- menced state of knowledge ;" 3. That " synthesis without analysis is no knowledge at all ; " 4. That this analysis-synthesis method is " the only pos- sible method " of learning and remembering, — the only possi- ble method in unifying as well as in first discovering the things to be learned and remembered, — If these things are true, how essential, how indispensable, to rational teaching, that teachers understand what rational analysis and rational synthesis are ! To this end, consider earnestly this up-the-tree process ; for it involves true analy- sis, and plainly explains what true analysis is. We are to read up the tree thus : F= (1) thing, (2) qualitative, (3) lin- gual, (4) letter, (5) orthograph, (6) aspirate, (7) continuant, (8) labial. Now, in this up-the-tree inquiry, we have thus unloosed eight attributes of/" by the process of analysis (from avalvELv, to loose again). Mark, however, that when we call 114 THE SYSTEM METHOD. this process " analysis," we suppose there to be eight different and successive acts of the mind, the first determining whether /is a thing ; the second, whether /"is a qualitative thing ; the third, whether/ is a letter ; the sixth, whether/ is an aspi- rate ; the eighth and last, whether / is a labial. Any act that views two, three, or more of these eight attributes, or classes, under one single view — at one single mental act, — is not analysis, but synthesis (from cwridevai, to place together). But that synthesis which is of the greatest value in remem- bering, discovering, or inventing, — the synthesis of the sys- tem, z. e., truly organizing method, — is that which begins at the branch tops and proceeds downward under the guidance of the work-tree. This is the only scientific, or system, syn- thesis, the only perfectly organizing synthesis ; for it is the only synthesis in which the student sees the whole of the branch at one contemplation. The student may see / at one mental glance as both labial and continuant ; but, unless he has a mental picture of /including labial, continuant, and also aspirate, he has no comprehending view of the branch whole ; for there are other branches (as /;« and z^) in the orthographic whole, that, for two branches downward, are likewise in the classes labial and continuant. But mark that that synthesis which sees/ as labial, continuant, and also aspirate, sees /as labial, con- tinuant, and also not-subvocal and not-vocal (for aspirate is both not-subvocal and not-vocal) ; that is, that mental syn- thesis which sees /on the continuous branch running down as labial, continuant, and aspirate, sees the relation of this /-branch to the subvocal (or m-v) branch, and also to the vocal branch ; that is to say, such view sees, in a way, the entire orthographic tree whole, and therein itself, subvocal branch, and vocal branch, as the parts of such whole. And now mark that such synthesis, or part-and-whole view, sees not merely what are the parts and the whole, but also the position of the parts in the whole ; so that, if asked to point to the <5-part (<5-branch), the .y-branch, or the /-branch, the student could instantly point it out in the tree. Thus, when SYKTHESIS SEES THE POSITION. 115 we really see a man whole, we can instantly point out his eye, arm, ear, foot, etc. ; for to see the man whole is to see the position of the parts. So to see the orthographic whole is to see the position of the parts. Such synthesis, therefore, as is guided by the sensible tree, and begins at the tops of the branches, — such synthesis alone gives mental grasp of part and whole. If it be mentioned that all authorities agree that all reasoning about, all thinking of, all remembering of, b, t, s,f, etc., must be real and rational, or false and irra- tional, just accordingly as they are thought of as whole and connected parts, or only as disconnected things, judge what is the value of such synthetic exercises thus leading to such system-knowing knowledge of s, t,f, etc., as enables student to place them in his mental orthographic whole, and therein easily to use them wherever life requires. What the compass is to the mariner, the class-branch work-tree is to the learner — the brancher. Written synthesis by work-tree. — I have now dis- cussed briefly the nature and relation of synthesis and anal- ysis as the two complemental parts of the organizing method — parts called by Hamilton the "necessary parts" of the " only possible method " to be called rational or true. I have given the student's work thus far, only for the first, or ana- lytic, process. I am now to give a bird's-eye view of what the student's work is in this second, last, or synthetic, process of the organizing method. For this, I shall use the same par- ticulars used in giving a view of the general nature of analy- sis : (1.) Cat ; (2.) Bed ; (3.) Gate ; (4.) Cane ; (5.) Boy ; (6.) Beef; (T.) Shop; (8.) Yeast; (9.) Bowl; (10.) Beau; (11.) Eye ; (12.) Adieu ; (13.) Noble ; (14.) Often ; (15.) Agree- able ; (16.) Nation ; (17.) Filial ; (18.) Quack. In this first exercise in written synthesis, the pupil is to be guided by the work-tree, stripped of such attribute, or class, names as are used by the memory in connecting par- ticular with class — particular in hand with the main branches of the orthographic system, or tree. The tree is also best used when reversed thus : — 116 THE SYSTEM METHOD. The student's work in learning will appear, of course, as follows : — T^ , J3 ^ ORTHOGRAPtilC SYNTHESIS TREE. Copyright, 1886, by I. E. WILSON. c = Guttural, abrupt, aspirate. letter. A = Palatal, simple. vocal, sonant. letter T = Dental, abrupt, aspirate, letter. B = Labial, abrupt. subvocal. sonant, letter. E = Palatal, simple. vocal, sonant, letter. D =, Dental, abrupt, subvocal. sonant, letter G = Guttural, abrupt, subvocal, sonant, letter WRITTEN SYNTHESIS. 11^ A = Dental, simple, vocal. sonant, letter. T = Dental, abrupt, aspirate. letter. E = Aphthong, letter. 4- C = Guttural, abrupt, aspirate, letter. A = Dental, simple, vocal. sonant. letter. N = Dental, continuant, subvocal. sonant. letter. E == Aphthong, letter. 5- B = Labial, abrupt. subvocal. sonant. letter. Oy = Open, compound. vocal, sonant. digraph. 6. B ^ Labial, abrupt. subvocal, sonant. letter. Ee = Dental, simple. vocal. sonant. digraph. F = Labial, continuant. aspirate. letter. 7- Sh = Palatal, continuant, aspirate, digraph. = Guttural, simple. vocal, sonant, letter. P == Labial, abrupt. aspirate, letter. 8. Y = Dental, coalescent, subvocal, sonant. letter. Ea = Dental, simple. vocal. sonant. digraph. S = Dental, continuant, aspirate. letter. T = Dental, abrupt. aspirate. letter. 9- B = Labial, abrupt. subvocal. sonant, letter. Ow = Labial, simple, vocal. sonant. digraph. L = Dental, continuant, subvocal, sonant. letter. lO. B = Labial, abrupt, subvocal, sonant. letter. Eau = Labial, simple. vocal. sonant. trigraph, II. Eye = Open, compound. vocal, sonant. trigraph. 12. A = Guttural, simple. vocal. sonant. letter. D = Dental, abrupt, subvocal. sonant. letter. leu = Close, compound. vocal, sonant. trigraph. 13- N = Dental, continuant. subvocal, sonant. letter. = Labial, simple. vocal. sonant. latter. B = Labial, abrupt. subvocal. sonant. letter. L = Dental, continuant. subvocal, sonant. letter. E = Aphthong, letter. 14. = Guttural, simple, vocal. sonant. letter. F = Labial, continuant. aspirate. letter. T = Dental, abrupt, aspirate. letter. E = Aphthong, letter. N = Dental, continuant. subvocal. sonant, letter. 16. N = Dental, continuant. subvocal. sonant. letter. A = Dental, simple. vocal, sonant, letter. Ti = Palatal, continuant. aspirate. digraph. = Guttural, simple, vocal, sonant. letter. N = Dental, continuant. subvocal. sonant, letter. 18. Q = Guttural, abrupt. aspirate. letter. u = Labial, coalescent. subvocal. sonant. letter. A = Palatal, simple, vocal, sonant, letter. C = Aphthong, letter. K = Guttural, abrupt, aspirate, letter. 118 THE SYSTEM METHOD. Of course, in thus doing" this downward, or synthetic, branching, or tracing, of the particulars assigned for his learn- ing exercises, the student is not expected to turn to the analysis tree, which has all the middle class-branches named. He is to be directed by the teacher to use the synthesis tree alone, the value of this written synthesis consisting in this one feature, that the learner would be forced, by the absence of the class names from these middle class-branches, to syn- thesize abrupt, subvocal, and sonant ; coalescent, subvocal, and sonant, etc., etc., in order to trace the particular through at all. This exercise is, as is every system-guided exercise, an advancing to the more difficult to be remembered from the less difficult to be remembered — from the more perfectly known to the less perfectly known — from the known to the unknown. That all the letters assigned are letters cannot be forgotten. That they are likewise labial, dental, palatal, or guttural, cannot be forgotten by the student who has already done the work to be done under the guidance of the analysis tree ; for there he worked out, with his own lips, teeth, and palates, the facts of labial, dental, palatal, and guttural ; and like walking, sewing, violin-playing, or any other such worked-out activity, it will never be forgotten. It is from these top branches, then (labial, dental, palatal, and guttural), that this synthesis is to start in the work of sup- plying the missing branches of the system. It is these mid- dle branches, connecting the particulars with the unit of the science, or system, that bind the orthographic materials into such science-system tree, by means of which alone the mind can either learn or remember them. Since these middle branches, or members, are the parts most difficult to learn, i. e., to branch, and therefore most difficult to remember, i. e., to re-branch, the student is here to be thoroughly drilled in using this synthesis tree, which is the learner's instrument in re-branching, or remembering ; for it is the supplying of missing parts (branches), the re-branching of missing branches, in which this synthesis exercise, as well as all rememoering, consists. The evident reason that a OEAL SYNTHESIS 119 downward branching on the analysis tree would not effect this same end, is because in that tree these middle missing branches are supplied — are already re-branched ^^remem- bered). Oral synthesis by means of work-tree. — It is to be carefully observed that the science-system, or class-branch, tree is not only the end but also the means. Orthography is a tree, — a tree whose trunk is the class letter, and whose branches are all class branches, branches of the tree-trunk class, letter. And because orthography is a tree, or branch, the orthographic branch is not only the science (branch) to be learned (branched), but is also the means by which learn- er's learning — brancher's branching — is to be accomplished. All thinking (branching) is down the branch ; and so, there- fore, is all reasoning, all knowing, all remembering. Now, the object of this synthetic exercise is to think (branch) down the tree, again down the tree, and so continue this branching, till learner (brancher) can hold the whole of the orthographic system in mind as a mental orthographic branch (tree). It is the incalculable disadvantage and the no-process-of-work misfortune of modern science that learner quits his learning (branching) utterly without this mental branch in mind. By all methods, modern or ancient, student learns letters as be- longing in a few classes, — disconnected classes, — and stops there ; whereas, not unless a second step be taken, in which student learns these classes as being branches of higher classes (branches), and these higher branch-classes as branches of the highest class, a branch or tree trunk, — not unless this second step be taken, is learner able either effi- ciently to know, remember, or teach such branch of learning ; for he can never really know what the true process of learn- ing (branching) is, until he gets all the classes branched to- gether in the unity of the tree, so that branching is possible. This up-the-tree process, here called branching, is impossible until the tree exists up which to branch, i. e., up which to trace particular through its classes. In the synthesizing process, the student is able at first to 120 THE SYSTEM METHOD. see together (z. e., synthesize) but two or three branches ; but through the instrumentality of the tree, by practice, he will eventually be able to see {i. e., synthesize) every class branch, and therein every attribute, of the particular with which he starts at the top, down to the highest class branch, the tree trunk. Finally he will be able to synthesize all the attributes {i. e., to synthesize all the classes) of any letter ; that is, to see any top branch entirely through the tree to tree trunk at one view. Then, but not till then, does he see whole in part and part in whole. The orthographic system thus becomes, to the student, more and more a perfectly or- ganized whole, or unit, till finally it appears in the student's mind, or mental picture, as a tree system, a science in reality. The student has now reached the "second end" of Sir Wm. Hamilton's " only possible " perfect knowledge of orthog- raphy ; since his knowledge is thus " carried into unity," and he sees "the one [the tree] in the many" as plainly as he could see the natural tree itself But not to digress further : it has already been shown that that synthesis which sees clear down the tree, sees more or less perfectly the whole tree, or science system. The plain truth that any vision really seeing the whole, must needs see likewise all the parts of which such whole is a whole, was also pointed out. It was further pointed out that to see the whole is not only to see the parts, but also, if the whole is distinctly seen, to see the position of the parts. Now this is the crucial test of the learner's proficiency in any branch : Ask him what is t)\& position of the parts of the whole. And this placing, or instantaneous pointing, of the parts is the work of oral synthesis. Hang the analysis tree map be- fore the student, name for him a word, say cat, and let him spell it over the top of the tree by pointing out the posi- tion, in the tree, of c, a, and t, in their regular order, and as rapidly as he is able. The class, having practiced at their study seats with the words assigned to them by the teacher, are, at the recitation hour, to exhibit to the teacher whatever of rapidity they may have acquired by such practice. With 1. Cat = k a t 2. Bed = b e d 3. Gate = g a t 4. Cane ^ -e a n 5. Boy = b oi 6. Beef = b e f Y. Shop = sh P 8. Yeast = y e s 9. Bowl = b 1 SPELLING OVER THE TREE- TOP. joi the words used above in written synthesis, giving sometimes another letter, if it represents the same sound, the student's pointer would run over the tree top thus : — 10. Beau = b 6 11. Eye = 1 aph. 13. Adieu = a d ii n aph. 13. Noble = n 5 b 1 aph. 14. Often = o f aph. aph. n 15. Nation = n a sh u n 16. Fihal = f i 1 y a 17. Quack = kwa aph. k The reader should not fail to realize that no synthesis is perfect until the pupil sees part and whole so plainly that he can instantly point out the position of any part in the whole. Whoever sees the whole of a horse, sees every part of that horse ; and whoever sees every part of a horse, in the same synthetic act sees the position of every part of that horse. It is plain, even to one not' accustomed to critical thinking, that the student seeing at one mental act every part of a system, or whole, will be able instantly to point out any part included by such whole. Evidently, the true test as to whether the student actually synthesizes all the parts, is the question whether he is able instantly to point out the differ- ent parts in rapid succession. Evidently, therefore, the cru- cial test of whether a student understands the science-system whole of orthography, is whether he is able rapidly to spell a word of letters over the top of the tree. The Student Now Knows Part and Whole.— It has been demonstrated, what every reader well understands, that, by the current methods of teaching orthography, not even the advanced student ever perceives what is the ortho- graphic whole ; and not perceiving the whole, neither teacher nor pupil ever detecting that there is such a thing as an or- thographic whole, the study of orthography is finished, teacher and pupil alike ignorant that the branch-parts. 122 THE SYSTEM METHOD sonant, subvocal, vocal, aspirate, abrupt, continuant, coales- cent, etc., are parts at all. By the current method, without the science-system tree, the student of the orthography tree whole, when " through," knows not — mark it well — that sonant, vocal, dental, coalescent, diphthong, etc., are parts of one tree whole ! What if a student of the New York City whole, when through, should not know that railroad,, lake, house, park, sidewalk, street-car, depot, drive, grove, etc., are parts of one city whole ! Without the sensible tree, by the current method the student of the orthographic tree whole finishes his study, and is "promoted," not even know- ing what such orthographic whole which he has been learn- ing is like — whether like a tree, a city, a man, or what ! What if a student of the New York City whole should finish his study and ask to be promoted, not even knowing what such New York whole which he has been learning is like — whether like a city, a tree, a man, or what ! Without the guiding tree, no student of orthography can know, not even after finishing his learning and securing his promotion (for without the tree no student has ever to this day found out), that such a thing as an orthographic whole has any ex- istence ! What if a student of New York, after " finish- ing" his work of learning, should be found not to know that such a thing as a New York whole has any existence ! Students of orthography, the world over, do never see sonant, vocal, subvocal, coalescent, abrupt, continuant ; aspi- rate, abrupt, continuant, coalescent, etc., as parts of what they really are — parts of the whole orthographic tree ! What if students of New York did never see railroad, lake, depot, park, building, etc., as parts of what to the mind they are — parts of a whole city! What if students of the skeleton did never see tibia, fibula, femur, pelvis, radius, ulna, humerus, occipital, and turbinated bones, etc., etc., as parts of what to the mind they really are — parts of the skeleton ! What if students of Cape Flattery, Mt. Tyndall, Red River, Cape Cod, Lake Champlain, Great Basin, Yellowstone Park, Phil- adelphia, Great Salt Lake, Cape Sable, Mt. Washington, TREE, CITY, U. S. MAP. 123 Dead Man's Valley, Mammoth Cave, New York, Pike's Peak, New York Bay, Pictured Rocks, Mt. St. Bernardino, Niagara Falls, Black Hills, Lake Itasca, 40th parallel, Gulf of Mexico, 122d meridian, etc., etc., — what if students of the United States map did never see these myriad things as parts of what to the mind they really are — parts of a country or map whole ! Modern teachers expect their pupils to hold sonant, subvocal, vocal, aspirate, continuant, etc., in mind as discon- nected, unorganized, and ttnpositioued things, and as parts of nothing ! What if modern teachers of the skeleton ex- pected their pupils to hold in mind tibia, fibula, femur, innominata, ulna, humerus, scapula, etc., etc., as disconnected, unorganized, and unpositioned things, and as parts of noth- ing ! What if modern teachers of the United States surface, or map, should expect their pupils to hold in mind Cape Flattery, Mt. Tyndall, Red River, Lake Champlain, and all this myriad of connected surface parts, as disconnected, un- organized, and unpositioned things ! and as parts of noth- ing ! No wonder that the scientific and more trustworthy world exclaims with — Ueberweg: " Science as such has a true existence only in systematic [tree-system] form." Real " science is a whole [tree] of knowledge in the form of system." GregOTy : "Until a science is so grasped [as a whole], it is not in any proper sense mastered, since the main thing in a science is not its separate facts and truths, but its zvhole of related facts and truths." " The aim in all systematic knowl- edge is to unite [into one sensible whole] the facts of knowl- edge so as to see them in their several bearings." Spencer : " Knowledge of the lowest kind is ununified [not united into one whole] knowledge ; [imperfect] science is partially unified [made-into-whole] knowledge ; philosophy [pure science] is completely unified knowledge." That is to say, the perfection of science in any branch of learning depends upon the question whether the things, or parts, studied are " completely unified " or not — whether the parts are completely made into one. 124 THE SYSTEM METHOD. If it be added that all authority, educational, scientific, and philosophical, agree in the declaration that the energy of all thinking, the soundness of all learning and reasoning, is to be measured by the close connection made between part and whole, — to be measured by the intensity in which parts are mirrored in whole and whole in parts, — if such authority is right, let the reader judge what is the soundness — what is not the weakness ! — of the current teaching and learning of orthography. Of Orthographies, Etymologies, and Grammars, I have before me about one hundred and fifty, including Harvey, Swinton, Reed and Kellogg, Raub, Welsh, and all the other late ones, and not one, so far as I have yet been able to discover, even so much as mentions "part and whole," " organize," or any other term indicative of this true process of learning by making parts into wholes, branches into branch ! On the other hand, by the Organizing, or Whole-and- Part, Method, both teacher and learner are as continually guided by the orthostatic, or true, whole as superintendent and working-men are guided by the architect's plans and specifications in the construction of a building ; and it needs only ordinary judgment to perceive that rational learning of orthography is as much more easily accomplished with than without such guiding whole, or plan, as it is easier to con- struct ready-prepared parts into a definite building whole with than without any knowledge of Avhat such definite whole, or plan, is. Rational building without some planned, or guiding, whole is impossible ; rational learning without some planned, or guiding, whole is surely not possible. But it was shown above how, by means of analysis and synthesis as guided by the tree, there generates in the stu- dent's mind, not merely some knowledge of the orthographic system whole, but such a perfect image of such whole as ena- bles him instantly to point out — give the position of— any part in the whole to which reference may be made. Noth- ing less than such a mental image of the whole as thus sees part in whole and whole in part, should be called finishing, scientific, or organized, knowledge of any branch. MANY LEARNED IN ONE. 125 It will now be shown how, at times, the orthostatic tree itself constitutes the student's daily Program of Work, and how at all other times the student's daily program, or process, of work is derived directly from, or indicated by, such guid- ing, or orthostatic, means. Many Learned in One. — But another great superiority of this system-guided, or organizing, method is that when by it the student has learned one branch, he has therein learned, or acquired the prerequisite in learning, other branches. Having mastered the science system of orthogra- phy, he easily masters the science system of English gram- mar ; for he knows the whole to be a branch, and the proc- ess to be branching. He has now only to find what the unit to be branched is, and little remains, indeed, to be done in mastering the new branch. The current methods of learning and teaching orthography, grammar, etymology, botany, arithmetic, and zoology, do not at all perceive how to use what the student learns while in one branch as an effectual aid in learning or teaching another. By the current meth- ods, orthography is "finished," and then grammar is begun and prosecuted as blindly as if orthography had not been studied. This statement I believe to be the literal truth. I have not found in any school publication or book on teach- ing as much as a reference to this fundamental means of learn- ing any n'ew branch through some branch or branches al- ready known. But a branch is a branch, whether it is a branch of orthography, etymology, arithmetic, zoology, or grammar ; just as a cat is a cat, whether it is domestic or wild ; just as a felis is a felis, whether it is a lion, tiger, pan- ther, jaguar, leopard, lynx, wild cat, or domestic cat. And I have several times demonstrated that all things are knowa- ble by virtue of their belonging together in classes as similar things, just to that extent to which they are, to the mind, similar things. All things are known through their similar- ity as the means. No thinking is possible — the very act of thinking is impossible — except under this circumstance ; namely, that the mind act through similarity as the means. 126 THE SYSTEM METHOD. It Avas likewise proven that every thought is an identify- ing of particular with class ; and that this identifying act is by means of the similarity of such particular to the members of the known class ; and that if no similarity exists, no think- ing or judgment is possible. It is because of the existence of myriad marks of similarity between things that we can know, or learn, them at all. In the world of known things, all diversity is but diversified uniformity, not incongruity. So we learn the whole feline tribe, substantially, by studying the tiger or domestic cat alone. Cat or tiger once known, we quickly learn lion, panther, jaguar, leopard, lynx, and wild cat, by means of their similarity to tiger and cat. Every two animals have more likeness of organization than we per- ceive. Every two problems have more similarity than we can discover. And this similarity is to be used as the all- essential means of all our learning of animals and problems and all else ; for it is the organizing, or system, method of learning. How incomprehensible, disorganizing, and unsolvable to the mind, would be a two-legged tiger or a two-headed cat ! Why so incomprehensible and disorganizing 1 — Simply be- cause not classifiable, or branchable, with others ; because not similar to known tigers, all of which are four-footed. But why so incomprehensible t — Simply because dissimilar and outside of class. But observe that, if dissimilarity ren- ders a thing not solvable and unknowable, similarity must, ac- cordingly, render a thing easily understandable. And so, when once we know the cat, we have essentially learned the whole feline tribe ; since after understanding cat, to learn lion, tiger, jaguar, etc., we have to do little more than merely to say, They are like the cat. Set out, then, in your imagination to learn these similar animal wholes : (1) lion, (2)tiger, (3) panther, (4) jaguar, (5) leopard, (fi) lynx, (7) wild cat, (8) domestic cat, as students do actually learn, or try to learn, these similar branch wholes : (1) orthography, (2) ideography, (3) etymology, (4) grammar, (5) arithmetic, (6) botany, and (7) zoology. What I mean for MA^r LEARNED IN ONE. ±27 you to do, is to set out, in your imagination, to study and learn these animals as wholly disconnected things, and by making no use whatever of their similarity as the all-essential means of learning or understanding them, just as students of the seven branches of study above mentioned, do never use the similarity between these branches as a means of under- standing them. Now, it is well known to mankind that the anatomy and physiology of the cat is practically identical with the anatomy and physiology of the feline tribe, of which cat is a member. Hence, to learn and remember these eight species as dissimilar and therefore disconnected things, is to do eight times as much mental labor as is necessary ; aye, such system-reversing study requires a thousand times more mental labor. And, worst of all, even after expending this thousand times more mental labor in studying (1) cat, (2) lion, (3) tiger, (4) panther, (5) jaguar, (6) leopard, (7) lynx, (8) wild cat, student has not one iota of learning of such animals that is not so much error or fallacy, as I have repeatedly shown ! For, to understand or learn cat or lion not as being like other animals, — to think or understand cat or lion not as like other animals, — is not to understand cat or lion as an animal ! And to think or understand cat or lion as not being animals is not to understand cat or lion ! Thus it is plain to the logical mind that to study, learn, and pretend to understand these eight animals as discon- nected, separate, or dissimilar things, is to understand them as not being animals. Now, I say that thus to learn them, without using their similarity as the means, would require a thousand times more mental labor than is necessary, and that even then the student has not an iota of knowledge con- cerning the feline tribe that is not so much error or fallacy. But after this thousand precious days of youth have been wasted in "learning" these eight animal species under some "professor" who does not "believe" in the system method, the knowledge gained is a thousand times more worthless than total ignorance of such animals. For, to understand or learn animals as not being animals, is to think one's self to be 128 THE SYSTmi METHOD. educated, when and simply because he is professor-duped and school-gulled — because he " knows " a cat or a tiger is not an animal ! Let none fancy this to be merely a declamation against the learning and the methods of the schools. I am treating text-book authors and current school methods, not with trifling or sarcasm, but with sober argument. It is the stern truth in learning and teaching the animal tiger, that unless the great similarity of tiger whole to other animals be used as the all-essential means of such learning and teaching of tiger, the student must quit his learning of tiger, and "grad- uate," ignorant that a tiger is an animal ! aye, a million times worse it is than mere ignorance ; for such student has taken a " course," and being "educated," he "knows" that a tiger is not an animal ! " If ye were blind, ye would have no sin ; but now ye say. We see [we know] ; therefore the sin remaineth." And so, in teaching or learning, z. e., branching, the orthographic branch, it is true that, unless the great similarity of the orthographic tree-system whole to other tree-system or branch wholes be used as the all- essential means of such learning and teaching of orthography, the student must quit his learning of orthography ignorant that orthography is a branch ! aye, a million times worse it is than mere ignorance ; for such a student has taken a "course" in learning, z. e., branching, the orthographic branch, and being "educated" in this branch, he "knows" that this branch is not a branch ! Here I fancy the reader to be mentally inquiring, " Do not the text-books of orthography, grammar, etymology, etc., make use of the great similarity of these branches as the essential means of learning and teaching them.^" and it is to such earnest inquiry that I have just been laboring to lead the reader. To this question I answer. No ; no text-book, no educational writer, no logician, metaphysician, or other text-book author, so far as I can learn, makes mention even of the similarity of these branches as the essential means of rational teaching of them. It is for these very reasons ; namely, — SIMILARITY PROCESSES UNKNOWN. 129 1. That, in theory called perfect, logician, psychologist, and all text-book makers alike aver that no learning, know- ing, or remembering, not even thinking itself, is possible except through the similarity of the things thought of, as the essential means ; and^ 2. That, in practice called perfect, all write text-books on what they call branches of knowledge to be learned, known, and remembered, and yet therein make known to the student not one single such similarity-means process of work in learning, knowing, and remembering, from the beginning to the end of such text-book guide ! — It is for such reasons as these that current methods and popular text-books are assailable. Education (1.) which proudly assumes the responsibility of shaping a nation's methods of learning and teaching ; (2.) which makes and supports the doctrine that no learning, knowing, remember- ing, no thinking even, is possible except through the similar- ity means ; and (3.) which writes text-books without furnish- ing one single process of work by which pupil or teacher might understand such similarity means of learning, — such education deserves to be arraigned before a court of Public Contempt, and there sentenced to a shower of school- children's derision. School-children, if guided by the natu- rally organizing method of the great Organizer, evolve knowl- edge of or power over the whole feline tribe of animals in learning the cat animal alone. Ought not teachers who talk of their "profession" to be able to evolve some knowledge of or power over the branches grammar, etymology, and botany by learning the orthographic branch alone .? Are teachers less proficient " professors " than the children they teach ? But how, just how, may we give the student knowl- edge of or power over grammar, etymology, and botany by teaching him orthography alone .?— By teaching him orthog- raphy, etymology, grammar, ideography, botany, zoology, etc., as being what in reality they are ; namely, class-branch and syllogism-notating tree systems ; for all alike are tree systems ; and one tree system well understood, a dozen 9 130 THE SYSTEM METHOD. others ought to be quickly mastered through the similarity means. It thus quickly appears plain that what is studied, learned,, and remembered, must needs be studied, learned, and re- membered by means of its similarity to known class, or kind \ for it appears plain that whoever learns and remembers a thing without using similarity as the means of such learning and remembering, learns and remembers such thing as not being what it actually is ! learns and remembers that a cat is not an animal! "knows" that knowing is not knowing! What, then, shall be said of the current methods of learning and teaching the branches herein mentioned ? If of learning they are anything, and not naught, of learning they are branches, as I have shown. If studying, learning, and remembering animals without using similarity as the means of such studying, learning, and remembering, is learning and remembering that the animals cat and tiger are not an- imals ( ! ), is not studying, learning, and remembering science systems, i. e., branches, without using similarity as the means of such learning and remembering, — is this not learn- ing and remembering that the branches, arithmetic, grammar, orthography, etc., are not branches .'' not organized science systems, but merely unsystem somewhats ? But I have demonstrated that all learning and remembering of things is efficient just accordingly as learner employs similarity as the means with which to learn, and that they are inefficient or impossible just to that extent to which similarity as the means is neglected, unknown, or not understood. Since, then, the student wholly neglecting similarity as the means of his learning and remembering, must needs quit his study of zoology not only not knowing that cats, tigers, etc., are animals, but "knowing" (in his own "educated" unwisdom) that cats, tigers, etc., are not animals ( ! ), what must be the efficiency of the current methods of teaching the different branches of language, wherein nothing whatever is said about the similarity of such branches as the means of learn- ing and remembering them ? SIMILARITY THE ESSENTIAL MEANS. 131 That the similarity of the science-system, i. e., branched, wholes of orthography, etymology, and grammar, as the all- essential means in teaching, learning, and remembering them, is wholly overlooked in the text-books now used in the public schools of the United States, all intelligent teachers will perceive at once, after that fact is once pointed out ; for no text-book now used in the schools of the United States presumes that it is at all possible in teaching the science system of grammar to employ the student's knowl- edge of the science system of orthography. No text-book of orthography, etymology, grammar, botany, zoology, or arithmetic now used in the United States perceives even that these branches, or branched wholes, are branched wholes ! Therefore, we shall search modern text-books and methods in vain to discover that they have any power of teaching one of these branches by means of its sim- ilarity to another branch already known to the pupil. Since they know not that these branches are in reality branches, containing both "whole of content" and "whole of extent" of all the multitude of particulars with which the branch has to do, at the very beginning of our search we must despair of evidence that any school text-book author has employed the branched similarity of these branches as the means of vitalizing and dispatching the work of learner. Had we never perceived that the tiger is like an animal, we might little hope to make use of tiger's similarity to other animals in teaching and learning tiger. Since text-book makers have not yet perceived that orthography is like tree, or branch, we shall little hope to find them making use of its similarity to grammar, etymology, etc., in teaching and learn- ing orthography. But we found that in learning tiger it is absolutely necessary to study it through its similarity to the classes to which it belongs in nature — in the light of the tree- class (species) tigris, the tree-class (genus) felis, the tree- class (family) felidae, the tree-class (order) carnivora, the tree-class (class) mammalia, the tree-class (branch) vertebrate. Evidently, therefore, we shall find that to vitality and dis- 132 THE SYSTEM METHOD patch in his study of, say grammar, it is absolutely necessary that the student study and learn grammar by means of its similarity to the classes to which grammar belongs ; namely, etymology, orthography, etc. The question now becomes. How does this organizing method — ^just how does this analysis-and-synthesis, whole- and-part, particular-and-organized-class, or system, method of teaching and learning: — manage to give the student knowl- edge of and power over etymology, syntax, and ideography by teaching and learning orthography alone ? I answer. By teaching and learning each of these branches as what in reality it is, a tree-system, branched, or science, whole. If by the natural method we obtain knowledge and power over the feline animal tribe while learning the cat animal alone, is it unreasonable to suppose that, if the organizing, or sys- tem, method is the perfectly natural method, we may obtain knowledge of and power over the lingual-branch tribe while learning the orthography branch alone .'' We shall soon see that the branched whole of any of these branches is a think- ing system, or thinking tree, — a system or tree through which learner is to think in learning, knowing, and remem- bering. We have already seen what analysis and synthesis in orthography are. We have therein had an imperfect view of how to learn orthography by means of this thinking tree, — by thinking the orthographic particulars, z. e., letters, up and down through the thinking tree. Now, the two com- plemental parts of the organizing, system, scientific, or per- fect, method of learning every branch, are analysis and syn- thesis. But analysis and synthesis in etymology and syn- tax are so very similar to analysis and synthesis in orthog- raphy that when the latter is once learned, the former is substantially mastered. (a.) Orthography is learned, branched, by thinking letter through orthographic thinking tree, (b.) Etymology is learned, or branched, by thinking etymon through etymological thinking tree, (c. ) Syntax is learned, or branched, by thinking syntactic word through syntactic thinking tree. The learning process is one in them all — begun by analy- sis, and completed by synthesis. And in all alike, the or- ganizing, or system, method analyzes and synthesizes under SIMILARITY THE ESSENTIAL MEANS. 133 the guidance and by means of the thinking tree. It is thus that the organizing, or system, method is greatly superior to any other possible method, because it does what every anti- organizing or anti-whole-and-part method undoes or tends to undo, — it carries our knowledge into science-system unity, learns or thinks by what thinking is, organizing, or branch- ing, into a branch-system whole. The organizing, or system, method learns, knows, and remembers — 1. Orthography, by thinking the letter, ororthograph, through the thinking tree. 2. Etymology, by thinking the etymon through the thinking tree. 3. Syntax, by thinking the syntactic word through the thinking tree, (a.) Orthography thinks a, b, c, e, d, o, z, etc., through the tree system. (b.) Ideography thinks capital, pos's've sign, hyphen, through the tree system, (c.) Etymology thinks un, health, ful, ness, etc., through the tree system, (d.) Syntax thinks in-sentence word through the tree system. But to think b, d,f, etc., through the tree, or science, sys- tem is quite the same thing as to think zm, ness, ful, and health through the tree system ; and either is identical in method with thinking either syntactic or ideographic partic- ular through the tree or science system. It is thus that, by the system method, the student acquires knowledge of and power over many branches while learning one alone. Orthog- raphy once learned by thinking letter through the tree sys- tem, etymology is easily mastered by the similarity means, — by thinking etymon likewise through the tree system. Mark you, a tree system is a tree system, whether it is a tree of orthography, ideography, etymology, syntax, arithmetic, botany, zoology, or else. Of the incomparably superior value of the organizing, or system, method over the current methods, I have not space again to speak. Suffice it here to say, that, besides having the incomparable advantage of thus learning many while studying one alone, the organizing method has the second still greater advantage ; namely, that that which it learns and "knows in system," it learns well, learns practically and scientifically. "The last end of all training in thinking should be to prepare for and lead to constructive thinking [or the pov/er to "know in system"]. Constructive thinking [or 134 THE SYSTEM METHOD. "knowing in system"] is manifestly the highest act of the human intellect, and should, therefore, be made prominent in the later stages of higher education." — D. S. Gregory s Prac- tical Logic. But further : the invention of the thinking tree not only (1.) learns many in the labor of learning one, and (2.) learns that one a hundred times more thoroughly, or understand- ingly, and (3.) acquires the power to "know in system" the "highest act of the human intellect," but (4.) it also " carries all its knowledge up to unity," and thus satisfies Sir William Hamilton as being the " only possible " method of true learn- ing. I hope again, farther on, to demonstrate (1.) how the organizing, or system, method learns many (orthography, ideography, syntax, etymology, etc.,) in the labor of learn- ing one (etymology, orthography, or syntax alone) ; and (2.) how it learns that one (orthography, say) a hundred times more thoroughly, or understandingly, through the similarity means (the orthography branch being so very similar to etymology, syntax, etc.,) ; and (3.) how it acquires the power to "know in system," — -in the tree system, — which Dr. Gregory calls the "highest act of the human intellect ;" and, lastly, (4.) how it also carries all its learning of orthog- raphy, ideography, syntax, etc., up into unity — up into the unit which I have called the thinking tree ; which method Hamilton declares is the "only possible" perfect method. But we must proceed to the student's work. Before the work-book branching, or organizing, is begun, however, a few words about — Why these 8-step and 4-step programs are used. — Because they are the only possible means, the tree itself ex- cepted, of placing squarely before the student the classes among which he is to distribute, and through which he is to branch, the particulars to be used in learning these branch classes. Refer to Program I. of Work-Book No. 16, page 158., By that program, for example, six branch classes, two having reference to voice, two to sound, two to continuance, are placed before the student in pairs, voiced contrasting WHF EMPLOY PROGRAMS? 135 with whispered, subvocal with aspirate, and abrupt with con- tinuant : so that the learner (brancher of particular in branch class) proceeds almost as directly through the science system of orthography as if he traced (branched) the particular through the orthographic tree itself, as shown on pages 164, 165, and 166, in Programs XL, XII., and XIII. Proof need not be here repeated that the learning act is a mental act identifying particular with organized, or known, class ; nor shall I re-demonstrate that the result of properly learning any branch is a branch, i. e., a fully organized tree. Know- ing, or learning, a branch is making a branch (/. e., branch- ing) by some known branch as guide, just as building is making a building by some known building as guide. Since learning is branching, the result of learning is, of course, a branch — thing branched ; so the result of building is, of course, a thing built ; so the result of copying is a thing copied. Evidently, the only possible guide in copying and building are the copy copied and the building built. Cer- tainly, the only true guide in learning {i. e., branching) is the branch branched. But next to the orthographic class tree, which is the only perfect guide and means, the best guide and means are the programs as given in Work-Books Nos. 16, lY, 18, 19, 20, 22, and 23 ; for such programs are the only means, the tree excepted, of placing the classes to be learned before the student in such a way as to aid him in his work of branching particular through class. It will be plain to the reader that the learner's problem, when guided by the tree itself, is to decide in what class (class branch) the par- ticular in hand belongs. Thus, he is to decide whether r is whispered or voiced ; subvocal or aspirate ; abrupt, continu- ant, or coalescent ; labial, dental, palatal, or guttural. Now, so far as this decision is concerned, the same result may be reached by placing before the student, in place of the tree, a program such as here follows : — Program : /. Voice. 2. Sound. 3. Continuance. 4. Obstruction. a. Whispered, a. Subvocal. a. Abrupt. a. Labial. b. Voiced. b. Aspirate. b. Continuant. b. Dental. c. Vocal. c. Coalescent. c. Palatal. d. Guttural. 136 THE SYSTEM METHOD It cannot be denied, however, that the tree itself possesses a very great advantage over such a program, if other things besides decision are taken into consideration. The tree itself constitutes the only program by which the student may create in his mind a mental image of what the orthographic whole is. And when it is considered that the vigor of think- ing and the certainty of remembering both depend upon how part is mirrored in whole and whole in part, it is plain how important it is that as much work as possible be done with the tree itself as the guiding program. Nevertheless, such a program has, besides the advantage of taking up but little printed space in either text-book or work-book, the great merit of keeping co-ordinate classes continually before. the eyes of student as being usually some branch of the orthographic whole. In all cases, the programs present the co-ordinate classes together as contrasted classes, precisely as, in that respect, they are presented in the tree. Whispered sounds are to be learned by being contrasted with voiced sounds ; subvocals are to be contrasted with vocals, abrupts with con- tinuants, labials with dentals, and so forth. And so, in all learning, what can be defined only by reference to some as- sociated thing, must not be presented to the learner discon- nected from such associated thing. Thus, whoever under- takes to teach the learner concerning the United States map, will find it absolutely necessary to present the United States in connection with British America, Mexico, and the Atlantic and Pacific Oceans ; because the United States map is only to be known or distinguished as the United States map by being that territory which lies between (1.) British America and Mexico ; (2.) the Atlantic and the Pacific. So, for an- other instance, the bone fibula is to be learned and remem- bered by means of its association with tibia and with femur and tarsus. So Indiana is not at all distinguishable or know- able as such, except as it is contrasted with those things with which it is associated. No one can distinguish, or define, Indiana without using Ohio, Illinois, Michigan, Lake Mich- igan, and the Ohio River as the very means of such distinc- DOING BY PROGRAMS. 13Y tion or definition. And this is the merit of these programs, that, by them, such co-ordinates are always presented to- gether as associated parts. Wherever voiced sounds are pre- sented, whispered sounds are likewise presented by contrast. So subvocal is not seen in the programs unassociated with its co-ordinate, aspirate. Digraph is ever associated with letter (monograph), continuant with abrupt, etc., etc. By the Programs, the Student Learns by Doing. — To attempt to learn the science, t. e., branched system, of orthography by the current practice of getting definitions, lists, " what the book says," and the " substance of the book," without doing the learning (J. e., without doing the branching of particular through classes), is the same sort of wisdom that attempts to learn sewing without doing sewing, but by getting definitions of, and reading books about, sewing. Mankind learn what is an act by doing that act. Learning is an act, and is to be done. The only question has been — the great, unanswerable problem has been — and is, What sort of an act is the learning act f And I have demonstrated that learning is a branching act, — an act branching thing learned through its classes as themselves branched into the branched (the tree) system. " We learn through doing." — Froebel. Since learning, all thinking, is branching particular in class, to learn we are to branch — to learn letters we are to branch letters through their branches, i. e., their tree-limb classes. " The man may teach by doing, and not otherwise." — Emerson. " Let things that have to be done, be learned by doing them." — Comenius. It seems to be Comenius's blunder here that he supposes there to be some things to be learned without doing them — some things to be done without doing them ! But that is not the question. The one insur- mountable difficulty has ever been, and is to-day, not whether we learn through doing, but in what does this doing consist. (See pp. 54, 59.) To say that learning is doing, is to say that doing is doing ! whistling is whistling ! your chin is your chin ! That learning orthography is doing orthogra- phy, all know. The question is, What is doing orthography } 138 THE SYSTEM METHOD. ■ And to this great question, these programs and the ortho- graphic tree furnish the first as well as the only true solution. It has been in many ways demonstrated in this volume that doing orthography to learn it, is branching letter-particular (upward chiefly) through orthography as a tree system. And the tree, supplemented by these programs, is the only possi- ble means (but it is a perfect means) of doing this branching (learning). Doing orthography, then, is branching the par- ticulars'of which this branch is a branch, through the ortho- graphic tree system, which is a tree of letter classes in which topmost branches are continuous Avith each succeeding higher branch ; so that any particular belonging in this topmost class-branch, therein belongs in every higher (larger) branch, including the tree trunk itself But, in the absence of the tree, or for learning such classes as are not yet perfectly organized, /. e., branched, into science (tree) system, the programs are the only efficient, nay, the only possible means. Every branch of learning is a branch ; therefore, from the very nature of things, all doing of such branch must be branching. The Programs Force the Student to Decide,— Therein, these programs train the judgment. What trains us to decide, gives us power. What trains us to say " may be," " yes, but," " I guess so," etc., trains us to imbecility, failure, rottenness. Such training is that which directs student to "get the sub- stance of the book," when, in doing so, he is not led to decide. But by this organizing method — by the tree and programs — the student is forced to say " yes " or " no " from twenty to three hundred times in every lesson made out, or organized. Under Program VIII., for example, Work-Book No. 17, he is forced to decide whether each particular is (1.) monograph, digraph, or trigraph ; (2.) tonic, atonic, or aphthong ; (3.) abrupt, continuant, coalescent, simple vocal, or compound vocal ; (4.) labial, dental, palatal, or guttural ; in doing which he makes, at least count, eight decisions, definite conclusions. To do the entire work of a lesson of twenty-five particulars, it is thus seen to require 25 X 8 = 200 acts of the judgment. DECIDING BY PROGRAMS. 139 Educated by this organizing method, the student is believed to quit his study with the best of all endowments, a trained judgment and a habit of thorough decision. In doing this organizing as guided by the programs and the tree, the stu- dent of orthography is led to take the particular into his mouth, and there work with it in order to decide into what classes it should be placed, or organized. Thus to train, the mind, is to train the judgment to weigh the evidence for and against, sagaciously to detect, the marks of likeness and con- trast in particular and class ; in a word, to identify, the life- long engagement of the human mind. A child decides wretchedly, the youth better, all men imperfectly. And it is to strengthen and correct the power to decide, that schools have been established. And that training which leads to skill in identifying, awakens the examining faculty, strengthens the judgment, and corrects the decision. Such training is priceless. First, such judgment and habit of decision are priceless, because, once acquired in any one branch, they are always retained in every other branch. No matter whether the question be — ORTHOGRAPHICAL, Whether e in sue and bade and righteous is a consonant, aphthong, or vowel ; whether // in ball and tee in sue are digraphs ; whether e in ewe is a vowel ; whether c in duck and lock is an aphthong ; whether e in cane may not be called a consonant ; — OR GRAMMATICAL, Whether who in " who is it 1 " and " who are they .? " is the same word in both ; whether S7iuffe7's is one or more than one whole ; whether in " I saw a corpse," corpse is masculine or feminine; whether John in "John, shut the door," is a nominative or an absolute case ; whether is in " He is the man," is itself an indicative mood, or merely in the mood ; whether shall and zvill are always in the so-called " future " tense, and may, can, must, might, could, zvould, and should, always in the so-called "potential" mood; whether sun in "The sun drives his chariots through mid-heaven," is mascu- 140 TEE SYSTEM METHOD. line or neuter ; whether in " Giving is receiving," giving is essential or accessory — a verb or a substantive ; — OR METAPHYSICAL, Whether a brick house is the same house after one brick is removed ; whether a penknife is the same knife after one blade is removed ; whether we see the same rainbow in two succeeding moments ; whether I am the same man I was ten years ago ; whether a stone is the same stone after a corner is knocked off; whether all things are similar ; whether, if all things were totally dissimilar, thinking would be possible ; whether a thing unknown is to us absolutely nothing ; — OR THEOLOGICAL, Whether in the redeemed state a man is the same man as he is here on earth ; whether the soul is material or immaterial ; whether the soul is a separate entity, or merely the life- organization ; whether a man moves himself or is moved ; whether whatever is, is right ; — Or what it be, it is to be decided by this same " yes " or " no," your judgment as to which — the life-long work of every mind. And that method which trains the pupil to decide upon his own judgment, and which considLntly pi^esscs him to belief or disbelief this way or that way, even in orthography, therein prepares him for any decision, even the most momentous. Not what is the question, but whether the student is forced to decision, to belief or disbelief, is the important matter. Secondly, such training by programs is to be desired, because it does away with rule-teaching, or educational "cramming." "Rule-teaching is now condemned as impart- ing merely an empirical knowledge — as producing an appear- ance of understanding without the reality. To give the net product of an inquiry without the inquiry [or doing] that leads to it, is found to be both enervating and inefficient. General truths, to be of due and permanent use, must be earned." — Spencer. In other words, rule and definition teaching is to be condemned because a rule or a definition is but the author's conclusion ready-made ; and to have the PROGRAMS PREVENT " CRAMMING. " 141 pupil take the ready-made conclusion of another, and not himself do the work which leads to the conclusion, is weak- ening to the mind, not strengthening ; or, in Mr. Spencer's own words, it produces the " appearance of understanding without the reality." It is plain that Mr. Spencer holds every attempt at learning without doing to be vain, even "enervating." And it ought to be seen that any definition or rule or table or formula gotten into mind by one, who, in getting it in mind, has not done the work by which such definition, rule, table, or formula was made, is as worthless as a definition of whistling gotten in mind by one, who, in getting it in mind, has not done the work that leads to whistling. And judge how well will he whistle who has learned exclusively by getting definitions of, and reading books about, whistling ! Now this farce of attempting to learn by getting another's conclusion, and without working out one's own conclusion, is likewise completely broken up by means of these programs and the tree. This will be evident to all who will take the pains to survey and carefully examine Work-Books Nos. 16, lY, 18, 19, 20, 22, and 23, as given further on in this treatise. This Method of Branching by Programs Sustained by Authority. — It has been demonstrated that learning is branching, that is, thinking given particulars through the branch branched, or learned. Thinking b, for instance, through the orthographic branch, or system, is thinking b thus: B = (1) letter, or orthograph, (2) sonant, (3) subvocal, (4) abrupt, and (5) labial ; that is, identifying b as belong- ing to five successively subordinated classes. But logicians agree that this identifying act is the very test of rational doing or thinking. Every thought, they declare, thinks sub- ject (particular) into predicate (known class). What practi- cal gold they handle ! How little they know its value ! Physiologists, too, and psychologists have found that the power of coupling subject (particular) to predicate (class) is the real measure, or indicator, of strength and sanity of mind. " Go from the individual to the class, from the branch 142 TS^ SYSTEM METHOD. toward the body, from the more particular to the more com- prehensive," says the great Organizer, "and you shall not lose the truth ; for the wires, or tree branches, are there to guide you." And He has so stringed the human mind to these inwardly converging wires of truth that it can think no sane thought, make known no fact whatever, except by means of this inward, or down-the-tree, thinking. The reason the thought runs, " John is a man," and not, "A man is John," is because the tree-branch system on which the mind is compelled by its nature to think, runs, like a branch, downward into a higher kind or class. The thought by nature is, " John, the particular, is of, or belongs to, the class inanr John is thought, learned, or branched, by its being thought, or branched, into the higher branch man. "A man is John" is not necessarily truth ; for a man might be Henry. "A man is John" runs not with the truth from sub-branch to higher branch, but backward from branch to to sub-branch, from class to particular. To put the thought thus, "A man is John," is falsely to represent the whole class to be included in one particular — to represent the whole body to be included in the branch. By these repeated thoughts, I wish to get this one great elementary truth before the reader's mind ; namely, that every sane thought goes from particular to class (from particular to known class) naturally and by necessity. All things of which we can think, are, by virtue of our being able to think (/. e., branch) them, connectible or branchable into classes already known to us (/. e., branched by us). Now, since every knowing, or learning, thought (every thinking act ; for every thinking act or thought is a knowing and a learning thought) limbs the thing thought about to some known class, we should naturally presume that fully to understand that thing, the mind would limb it to, or classify it with, all the classes to which by nature it belongs. That this is true, I have already demonstrated. Take, for example, the particular zv in the word " wet." When the student is able quickly to see it as belonging in (1) letter, (2) sonant, (3) subvocal, (4) coa- ^1 THE PROGRAMS SUSTAINED. 143 lescent, (5) labial, — classes, or branches, themselves branched into each other, — when he can quickly organize it thus into all the connected kinds to Avhich by nature it belongs, then he fully understands it. Ifthereisone thing above another settled among men, it is that we cannot know or understand a thing until we know what it is like, until we know to what connected kinds — to what system — it belongs. If there is another thing settled, it is the evident truth that what we can organize, or fully classify with known things, we do understand. What we can class with nothing known, we understand not at all. Such a thing is a mysterious thing, — a thing not yet by the mind brought within kind. Inside the pale of known kind, the human mind is sane ; outside of kind, the human mind is insane. Let the reader who desires to understand this method of organizing by programs, — let him observe that the programs are themselves a schedule of connected classes, — co-ordinate limbs severed from the larger containing limbs, — to be learned exclusively by this rational process of think- ing particular, (1.) into known kind, (2.) through that into a higher kind, or limb, (3.) again through this to a third still higher limb, or branch, (4.) and so on to the tree trunk, the sumnium genics. The organizing method is thus seen to have the support of logician, psychologist, and physiologist alike. Better than that, it has the support of almost undreamed-of success on the part of all teachers who have tried it. It will be found simpler, more efficient, and by all odds easier to teach and to learn in any school or college — ten times more efficient and manageable — than any method now in use in any school, let the grade of students be what it may. The teacher has only to show his pupils how to do precisely what is done in Work-Books Nos. 16 and 17. This he is to do, of course, either by putting the work-books in their hands (which is the best plan) or by writing out the programs for them, he doing for them some specimen work. The pupils may use a regular text-book, or by using the appendixes of their v/ork- 144 THE SYSTEM METHOD. books, they may dispense with it. In reciting, the pupils read their conclusions as written in the work-book (or on slate or paper, if the work-books are not used), jast as they would read their written parsings in reciting a parsing exer- cise in English grammar. To school expositions of any kind, the work thus prepared by programs will be found admirably adapted. The Student's Work-Book of System Orthography- Grade B. — I shall now do the work that learner is expected to do in learning by the organizing, or system, method. This work will include also all the devices, or guiding processes, expected of the teacher. Beginning with "System Orthog- raphy—Grade B" (which is No. 16 of the "System-Method" Series), I shall do all the rational doing demanded of student in taking a complete course in the language branch of the great tree of knowledge, punctuation alone excepted. (Punct- uation will be given in Volume II.) This doing work will be graded into five different work-books, each of which is to be fully worked up by the learner taking a full course. These five work-books are as follows : — No. 16. Work-Book of System Orthography — Grade B. No. 17. Work-Book of System Orthography — Grade A. No. 18. Work-Book of System Etymology (Word-Analysis). No. 19. Work-Book of System Grammar — Grade B. No. 20. Work-Book of System Grammar — Grade A. In addition to these, there will be given also the full amount of work required at the student's hands in learning the science system of arithmetic. This will be given in two work-books as follows : — No. 22. Work-Book of System Arithmetic — Grade B. No. 23. Work-Book of System Arithmetic — Grade A. The work in all these work-books will be made out, or done, precisely as the student is expected to do it in learning these branches, the seven work-books thus being a complete and thoroughly organized key to all the branching and solv- ing to be done by the student in learning all these branches. SEVEN W0BK-B00K8. 145 So that, if the teacher be equipped with this first volume, he shall have in it not theory alone, but the actual work to be done by his pupils in taking a complete course in these science systems. It is designed that in this volume teacher shall find not only all the daily work of his pupils for the first term, but their daily work for every school term of work required of them in mastering these branches by the system method. Besides this work to be done by students, these work-books will also contain explanations of, discussions and reviews enforcing, the claims of the method employed in them. 10 \-^ /ce. 2. Sound. 3. Position. a. Voiced. b. Whispered. voiced. a. Vocal. b. Subvocal. c. Aspirate. subvocal. a. Antecedent. b. Base. c. Consequent. antecedent. voiced, vocal. base. voiced, voiced, voiced. subvocal, subvocal, subvocal. consequent, consequent, antecedent. voiced. vocal. base. whispered, whispered, voiced, aspirate, aspirate, vocal. consequent, antecedent base. voiced, whispered, whispered,^ voiced. subvocal, aspirate, aspirate, vocal. consequent, consequent, antecedent, base. whispered, whispered, voiced, aspirate, aspirate, subvocal. consequent, antecedent, antecedent. voiced. vocal. base. voiced, subvocal. consequent. SYSTEM OBTHOGBAPHY— GRADE B. 159 Prog. Ill: 1. W e t 2. H a s 3. Y e t 4. H e H oice. 2. Sound. a. Voiced. b. Whispered. voiced, a. Vocal, b. Sub vocal. c. Aspirate. subvocal. voiced, vocal. whispered, whispered, voiced. aspirate, aspirate, vocal. voiced. subvocal. voiced, subvocal, voiced, vocal. whispered, whispered, voiced. aspirate, aspirate, vocal. voiced, subvocal. whispered, voiced, aspirate, vocal. voiced. subvocal. 3. Continuance. a. Continuant. b. Abrupt. c. Coalescent. coalescent. continuant. abrupt. coalescent. continuant. continuant. coalescent. continuant. abrupt. coalescent. continuant. continuant. coalescent. continuant. abrupt. Prog. IT: /. Letter. 2. Sound, 1. S n 2. T a 1 k 3. K i 1 n 4. K i c k a. Sonant. b. Non-sonant. c. Aphthong. a. Vocal. b. Subvocal. c. Aspirate. non-sonant. aspirate. sonant. vocal. aphthong. sonant. subvocal. non-sonant. aspirate. sonant, vocal, aj)hthong. non-sonant. aspirate. non-sonant. aspirate. sonant. vocal, sonant. subvocal. aphthong. non-sonant. aspirate. sonant, vocal. aphthong. non-sonant. aspirate. 3. Continuance. a. Continuant. b. Abrupt. c. Coalescent. continuant, continuant. continuant. abrupt. continuant. abrupt, abrupt continuant, continuant. abrujDt. continuant. abrupt. 160 THE SYSTEM METHOD. Prog. Y. — Branching by Learner's Work-Tree. JT'ote. — It is the undertaking of science to obtain power over the great and oth- erwise unmanageable multitude of particulars by organizing them — by putting an organic unit, i. e., a system, into them. Now, in developing any science system, two great steps are to be had: (1.) This multitude are to be thoroughly classified; (2.) These classes thus formed are to be branched into what science system is, namely, the tree system, whose branches are all classes of the particulars to be learned (branched) . Mark with extraordinary attention the fact that this second step In Si f> ^ ORXMOGRAPHIC ^ATORKI-TREE. Copyright, 1886, by I. E. WILSON. the development of a science system, is the all-important step ; since, if the classes are not thus branched into the science tree, we remain ignorant of what the stu- dent's true process of work is, thus forced to "rat it" through old holes and rutted ways, even unconscious that we are learning a branch, and that, therefore, our tree process is branching ! Observe below how each particular (letter) is branched up- ward, in the tree branches, through all the important classes to which it belongs. Observe more, that this branching through classes is uncovering the attributes of the letters, which is true learning, as all agree. * Or, DIGRAPH. DIPHTHONG, TRIGRAPH. SYSTEM 0BTH0OBAPH7—OBADE B. 161 Prog. 1. o f t e n 2. C 1 3. W s e 4. T o n s 5. F o X e s 6. P s a 1 m 7. S u b t 1 e 8. D u m b TI.— Branching by =: sonant, Learner's Work-Tree (Continued). vocal, simple, guttural. asj)irate, continuant, labial. aphthong. apbthong. sonant, subvocal. continuant. dental. aspirate. abrupt, guttural. sonant. subvocal. continuant. dental. sonant. vocal. simple. guttural. sonant. subvocal. continuant. dental. sonant. vocal, simple, palatal. aspirate, abrupt. dental. aphtliong. aspirate. coalescent, guttural. sonant. vocal. simple. labial. sonant. subvocal, continuant, dental. aphthong. aspirate, abrupt. dental. sonant. vocal, simple. guttural. sonant. subvocal. continuant, dental. sonant. subvocal. continuant. dental. aspirate. continuant, labial. sonant. vocal, simjDle, guttural. aspirate. continuant. dental. sonant, vocal. simple, palatal. sonant. subvocal. continuant. dental. aphthong. aspirate. continuant, dental. sonant, vocal. simple. guttural. aphthong. sonant. subvocal, continuant. labial. aspirate. continuant. dental. sonant. vocal, simple. guttural. aphthong. aspirate. abrupt. dental. sonant. subvocal, continuant. dental. aphthong. sonant. subvocal. abrupt. dental. sonant. vocal. simple, guttural. sonant, subvocal. continuant. labial. aphthong. 11 162 THE SYSTEM METHOD. Prog-. yii: /. Position. 2. Sound. 3. Letter. a. Antecedent. b. Base. c. Consequent. a. Vocal. b. Subvocal. c. Aspirate. a. Sonant. b. Non-sonant. c. Aphtliong. 1. G = antecedent, subvocal. sonant. 1 = antecedent, subvocal. sonant. a == base, vocal. sonant. d = consequent, subvocal. sonant. e = consequent. aphthong. 2. S = antecedent. aspirate, non-sonant. P = antecedent, aspirate, non-sonant. i == base, vocal. sonant. k = consequent. aspirate. non-sonant. e == consequent. aplitbong. 3. M = antecedent, subvocal. sonant. a = base, vocal, sonant. s = consequent. aspirate. non-sonant. k = consequent. aspirate. non-sonant. 4. S = antecedent, aspirate. non-sonant. P = antecedent. aspirate, non-sonant. u = base, vocal. sonant. r ^ consequent. subvocal. sonant. n = consequent, subvocal. sonant. Prog. Till : /. Position. 2. Sound. 3. Letter. a. Antecedent. b. Base. c. Consequent. a. Vocal. b. Subvocal. c. Aspirate. a. Vowel. b. Consonant. c. Aphthong. 1. B = antecedent, subvocal. consonant. 1 = antecedent. subvocal. consonant. a = base, vocal, vowel. c = consequent. aphthong. k = consequent. aspirate. consonant. 2. S = antecedent. aspirate. consonant. w = antecedent, aphthong. = base. vocal. vowel. r = consequent. subvocal. consonant. d = consequent, subvocal. consonant. 3. F = antecedent, aspirate. consonant. 1 = antecedent. subvocal, consonant. = base. vocal, vowel. c = consequent. aphthong. k = consequent. aspirate. consonant. 4. L = antecedent. subvocal. consonant. a = base, vocal, vowel. n = consequent. subvocal. consonant. k = consequent, aspirate. consonant. SYSTEIf OBTHOGRAPHT—QBADE B. 163 Prog. ,IX: /. Position. 2. Construction, c ?. Continuance. a. Antecedent. b. Base. c. Consequent. a. Vowel. b. Consonant. a. Abrupt. b. Continuant. c. Coalescent. 1. H = antecedent, consonant, coalescent. u = base, vowel, continuant. s = consequent, consonant. continuant. b = antecedent, consonant. abrupt. a = base. vowel, continuant. n = consequent. consonant, continuant. d = consequent. consonant. abrupt. 2. E = base. vowel. continuant. f = consequent, consonant, continuant. f = antecedent. consonant. continuant. e = base. vowel. continuant. c = consequent. consonant. abrupt. t = consequent. consonant. abrupt. 3. P = antecedent. consonant. abrupt. a = base, vowel. continuant. s = consequent. consonant, continuant. t = consequent. consonant, abrupt. 4. D = antecedent. consonant. abrupt. Prog .X : / . Letter. 2. Continuance. 3. Obstructioi a. Sonant, or Tonic. b. Aspirate, or Atonic, c. Aphthong. a. Abrupt. b. Continuant. c. Coalescent. a. Labial. b. Dental. c. Palatal. d. Guttural. 1. s = aspirate. continuant. dental. q = aspirate. abrupt. guttural. u = sonant. coalescent. labial. a = sonant} continuant. palatal. r = sonant, continuant. palatal. e = apbtbong. 2. A = sonant. continuant. palatal. c = aspirate. continuant, dental. i = sonant. continuant, palatal. d = sonant. abrupt, dental. 3. Z = sonant. continuant. dental. e = sonant, continuant. dental. r = sonant. continuant. palatal. = sonant. continuant. labial. 4. C = aspirate. continuant. dental. e = sonant. continuant. dental. d = sonant. abrupt, dental. a = sonant. continuant. guttural. r = sonant. continuant. palatal. 164 THE SYSTEM METHOD. Prog. XI.— Branching by Learner's Work-Tree. (for the program TREE, REFER BACK TO PROG. V.) 1. P = aspirate. abrupt. labial. 1 = sonant. subvocal. continuant, dental. u = sonant, vocal. simple. guttural. m = sonant. subvocal. continuant. labial. 2. G = sonant. subvocal. abrupt. guttural. i = sonant, vocal. simple. palatal. m = sonant, subvocal. continuant. labial. 1 = sonant, subvocal. continuant, dental. e = sonant. vocal. simple. palatal. t = aspirate. abrupt. dental. 3. S = aspirate. continuant. dental. k = aspirate, abrupt. guttural. y = sonant. vocal. compound. open. 1 = sonant. subvocal. continuant. dental. a = sonant. vocal. simple. guttural. r = sonant, subvocal, continuant. palatal. k = aspirate. abrupt. guttural. 4. S = aspirate. continuant. dental. = sonant. vocal. simple. guttural. 1 = sonant. subvocal. continuant. dental. e = sonant. vocal. simple. palatal. m = sonant. subvocal. continuant, labial. n = aplitliong. 5. C = aspirate, abrupt. guttural. a = sonant. vocal. simple. palatal. P = aspirate. abrupt. labial. t = aspirate. abrupt. dental. u = sonant. vocal. compound. close. r = sonant. subvocal. continuant. palatal. e = aplithong. 6. V = sonant. subvocal. continuant. labial. e = sonant. vocal. simple. palatal. X = aspirate. continuant. dental. 7. C = aspirate. continuant. dental. i = sonant. vocal. simple. guttural. r = sonant. subvocal. continuant. palatal. c = aspirate. abrupt. guttural. 1 = sonant. subvocal. continuant. dental. e == apbthong. SYSTEM OETHOOMAPHY— GRADE B. 165 Prog . XII. —Brandling hy Work-Tree (Continued). 1. G = sonant, subvocal. abrupt, guttural. li = aplithong. o = sonant, vocal, simple. labial. s = aspirate, continuant, dental. t = aspirate, abrupt. dental. 2. A = sonant, vocal. simple. palatal. s = sonant. subvocal. continuant. dental. til = aphtbong. m = sonant, subvocal, continuant. labial. a = sonant, vocal. simple. guttural, 3. A = sonant. vocal. simijle. guttural. g = sonant. subvocal. abrupt. guttural, h = apbtbong. a = sonant, vocal. simple. palatal. s = aspirate. continuant. dental. t = aspirate, abrupt. dental. 4. F = aspirate, continuant. labial. r = sonant. subvocal. continuant. palatal. = sonant. vocal. simple. guttural. 1 = sonant. subvocal. continuant. dental. i = sonant. vocal. simple. palatal. c = aspirate. abrupt. guttural. 5. D = sonant. subvocal. abrupt. dental. u = sonant, vocal. simple, guttural. c = apbtbong. k = aspirate. abrupt. guttural. s = aspirate. continuant. dental. 6. S = aspirate. continuant. dental. c = aspirate. abrupt. guttural. r = sonant. subvocal. continuant. palatal. a = sonant. vocal. simple. palatal. o- o = sonant. subvocal. abrupt. guttural. g = sonant. subvocal. abrupt. guttural. y = sonant. vocal. simple. palatal. 7. G = sonant, subvocal. abrupt. guttural. i = sonant. vocal. ' simple. palatal. b = sonant. subvocal. abrupt. labial. b = sonant, subvocal. abrupt. labial. 6 = sonant. vocal. simple. guttural. r = sonant. subvocal. continuant. palatal. 166 THE SYSTEM METHOD. Prog. XIIL— Branching by Work-Tree (Continued). 1. R = sonant, subvocal. continuant. palatal. h = apbtbong. o = sonant. vocal. simple. guttural. m = sonant, subvocal, continuant. labial. b = apbtbong. 2. R = sonant, subvocal, continuant. palatal. b = aphtbong. u == sonant. vocal. compound. close. b == sonant, subvocal. abrupt, labial. a z= sonant, vocal. simple. guttural. r = sonant. subvocal. continuant, palatal. b = sonant. subvocal. abrupt. labial. 3. I = sonant. vocal, simple. palatal. n = sonant. subvocal, continuant, dental. t = aspirate, abrupt. dental. r = sonant. subvocal. continuant. palatal. i = sonant, vocal, simple, dental. g = sonant. subvocal. abrupt. guttural. u = apbthong. e = aphtbong. 4. L = sonant. subvocal. continuant. dental. u = sonant, vocal. compound, close. c = aspirate, abrupt, guttural. r = sonant, subvocal. continuant. palatal. e = ajjb thong. 5. = sonant. vocal, simple. labial. P = aspirate. abrupt, labial. a = sonant, vocal, simple. dental. q = aspirate, abruj)t. guttural. u = apbthong. e — aphtbong. Note, — Let this work of branching, i. e., doing the learning act, be continued until the learner (brancher) has a distinct 7nental picture of this orthographic branch (science tree) ; that is to say, continue this up-the-tree process of branch- ing letters through their classes (branch classes) until the student is able to repro- duce the tree on the blackboard from memory. You have then, not till then, mastered the science (system) of orthography ; for mastery of any science system consists of having in mind that mental picture ; since it is only by means of such mental picture that student will be able, in after life, to solve (i. e., re-branch) any letter. SYSTEM OBTHOGBAPET—OBADE B. 167 Prog.XIY; : /. Union. 2. Leiien. 3. Subdivision. a. Letter, or monograph. b. Digraph. c. Trigraph. d. Diphthong. a. Sonant. a. Abrupt. b. Aspirate, or non-sonant, b. Continuant. c. Aphthong. c. Simple vocal. d. Compound vocal. 1. B = letter. sonant. abrupt. ee = digraph, sonant. simple vocal. f = letter, aspirate. continuant. 2. Sh = digraph. aspirate. continuant. i = letter, sonant, simple vocal. P = letter. aspirate. abrupt. 3. Th = digraph. sonant, continuant. i = letter. sonant, simple vocal. s = letter. aspirate. continuant. 4. Ch = digraph, aspirate. abrupt. u = letter. sonant. simjile vocal. r = letter. sonant, continuant. ch = digraph aspirate. abrupt. 5. B = letter, sonant. abrupt. u = letter. sonant. comp'nd vocal. r = letter, sonant, continuant. eau = trigraph. sonant. simple vocal. Prog. XV: /. Union. 2. Construction. 3. Diacritical Mark a. Monograph. b. Digraph. c. Trigraph. a. Vowel. b. Consonant. c. Aphthong. a. Macron. b. Breve. c. Diaeresis. d. Unmarked. 1. P = monograph, consonant. unmarked. 1 = monograph, consonant, unmarked. ayt = digra2:)h, vowel, f macron. 3. V = monograph. consonant. unmarked. iewf = digraph. vowel. unmarked. 3. M = monograph. consonant. unmarked. ewf = digraph. vowel. unmarked. 4. = monograph, vowel. macron. ce* = digraph. consonant, f unmarked. a = monograph, vowel. unmarked. n = monograph. consonant. unmarked. *Ce, ti, and si, representing the sound sh, have, by reputable authors, been called "combined" digraphs, on the erroneous supposition that the e or the i "combined" with the consonant (c, t, or s) is a vowel. But since the e and the i help to produce an aspirate (sh) sound, and since, on the contrary, every vowel must represent a vocal sound, such e and i are not vowels, but consonants. Ce is therefore not a " combined," but simply a consonant, digraph. f Strictly speaking, ay, iew, ew, ce, etc. are each iwo or three vowels or con- sonants, not one. If the singulars, "vowel" and "consonant," are used, they are to be understood as properly applying to each letter in the combination. 168 THE SYSTEM METHOD. Prog. XTI: / '. Union. 2. 1 Obstruction. 3. Diacritical Mark. a. Monograph. b. Digraph. c. Trigraph. a. Labial. b. Dental. c. Palatal. d. GutturaL e. Aphthong. a. Macron, f. Tilde. b. Breve. g. Cedilla. c. Caret. h. Horizontal. d. Diaaresis. i. Perpendicular. e. Dot. j. Unmarked. 1. s = nionograph. dental, unmarked. t = monograph, dental. unmarked. ea = digrai)b. palatal. breve. 1 = monograiih, dental. unmarked. th = digraph, dental, unmarked. 2. P = monograph. labial. unmarked. r = monograph, palatal. unmarked. ai = digraph, dental. macron. r = monograph, palatal, unmarked. ie = digraph, palatal, breve. 3. M = monograph, labial. unmarked. i = monograph, palatal. breve. s = monograph. dental. unmarked. cli = digraph, palatal. unmarked. ie = digraph. palatal. breve. f = monograjih, labial. unmarked. Prog. XYII: /. Union. 2. Obstruction. 3. Diacritical Mark. a. Monograph. b. Digraph. c. Trigraph. a. Labial. b. Dental. c. Palatal. d. Guttural. e. Aphthong. a. Macron, f. Tilde. b. Breve. g. Cedilla. c. Caret. h. Horizontal. d. Di.-cresis. 1. Perpendicular. e. Dot. j. Unmarked. 1. F = monograph, labial. unmarked. i = monograph, palatal. breve. 1 = monograjA, dental. unmarked. i = monograph. dental. unmarked. a = monograph. palatal. breve. 1 = monograph, dental, unmarked. 2. Ch = digraph, palatal, cedilla. a = monograph. guttural. dot. r = monograph. palatal. unmarked. a = monograph. dental, macron. d = monograph. dental. unmarked. e = monograph. aphthong, unmarked. 3. P = monograjjh. labial. unmarked. a = monograph. palatal. breve. c = monograjih. aphthong. unmarked. k = monograph, guttural. unmarked. a = monograph. dental, macron. g = monograph. palatal. dot. e = monograph. palatal, breve. 8 = monograph, dental. perpendicular. SYSTEM OBTHOOBAPHT— GRADE B. 169 Prog. XYIII : /• Position. 1. B e g n e 2. C a s t o ff 3. I c e c r ea m a. Antecedent. b. Base. c. Consequent. antecedent, base, antecedent, base, consequent, consequent, antecedent, base, consequent, consequent, base, consequent, base, consequent, consequent, antecedent, antecedent, base, consequent. 2. Accent of Syllable. 3. Diacritical Mark. t. Tilde, g. Cedilla, h. Horizontal, i. Perpendicular. 1. Unmarked. a. Primary. b. Secondary. c. Unaccented unaccented, unaccented, primary, primary, primary, primary, primary, primary, primary, unaccented, unaccented, accented, accented, unaccented, unaccented, unaccented, unaccented, a. Macron. b. Breve. c. Caret. d. Diasresis. e. Dot. unmarked. macron. macron. breve. unmarked. unmarlved. horizontal. breve. unmarked. unmarked. caret. unmarked. macron. cedilla. unmarked. horizontal. unmarked. macron. unmarked. Prog. XIX: J. Position. 2. Accent of Syllable. 3. Diacritical Mark. 1. W r th 1 e s t 2. R h o m b u s a. Antecedent. b. Base. c. Consequent. antecedent, base, consequent, antecedent, base, base, consequent, consequent, antecedent, antecedent, base, consequent, antecedent, base, consequent. a. Primary. b. Secondary. c. Unaccented. primary, primary, primary, unaccented, unaccented, unaccented, unaccented, unaccented, accented, accented, accented, unaccented, unaccented, unaccented, a. Macron. b. Breve. c. Caret. d. Diaeresis. e. Dot. unmarked. dot. unmarked. horizontal. breve. breve. unmarked. unmarlied. unmarked. unmarked. breve. unmarked. unmarked. breve. unmarked. f. Tilde. g. Cedilla. h. Horizontal. i. Perpendicular, j. Unmarked. 170 THE SYSTEM METHOD Prog. XX : /. Position. 2. InterrupHon. 3. Diacritical Mark. a. Antecedent. b. Base. c. Consequent. a. Perfectly interrupted. b. Partially interrupted c. Molded merely. d. Aphthong. a. Macron. b. Breve. c. Caret. d. Diceresis. e. Dot. f. Tilde. g. Cedilla. h. Horizontal, i. Perpendicular, j. Unmarked. 1. F = antecedent, partially interrupted. unmarked. i = base, molded merely, macron. r = consequent, partially interruDted, unmarked. e = consequent. aphthong, unmarked. c = antecedent. perfectly interrupted, horizontal. r = antecedent. partially interrupted, unmarked. a = base. molded merely. breve. c = consequent, aphthong, unmarked. k = consequent, perfectly interrupted, unmarked. e = base, molded merely. tilde. r = consequent. partially interrupted. unmarked. s = consequent. partially interrupted, perpendicular. 2. H = antecedent. partially interrupted. unmarked. 11 = base. molded merely. breve. r = consequent. partially interrupted. unmarked. r = antecedent. partially interrupted. unmarked. a = base, molded merely. diaeresis (under). h = consequent. aphthong. unmarked. Prog. XXI: !• the 1. Antecedent, 2. Is a 1. 3. Base, 2. 3. Consequent, Vowel, 3. Representing Consonant, what sound, if any? 1. Perfectly interrupted at the 1. Lips, 3. Partially interrupted at the 3. Teeth, 3. Molded merely 7. And an 1. 3. 3. at the 3. Hard palate, 4. Soft palate. Abrupt. Continuant. Coalescent. 4. Which is a 1. Simple vocal, 5. 2. Compound vocal, 3. Subvocal, 4. Aspirate, 6. Producing a 1. Labial, 3. Dental, 3. Palatal, 4. Guttural. 1. E, (1.) the antecedent, (2.) is a consonant, (3.) representing its own proper sound r (enunciate), (4.) which is a subvocal, (5.) partially interrupted at the hard palate, (6.) producing a palatal (7.) and a continuant. a, (1.) the base, (2.) is a vowel, (3.) representing its own short sound a (enunciate), (4.) which is a simple vocal, (5.) molded merely, at the hard palate, (6.) producing a palatal (7.) and a continuant. t, (1.) the consequent, (2.) is a consonant, (3.) representing its own proper sound t (enunciate), (4.) which is an aspirate, (5.) perfectly interrupted at the teeth, (6.) producing a dental (7.) and an abrupt. Ifote. — For the verbal analysis of more difficult words, see Work-Book No. 17. See also back pages, 105 to 109. SYSTEM ORTHOGRAPHY— ORADE B. lYl Prog. XXII.— Oral Analysis (Continued by Prog. XXI). 1. L, the antecedent, is a consonant, representing its own proper sound 1 (enunciate), wiiich is a subvocal partially interrupted at the teeth, producing a dental and a continuant. a, the base, is a vowel, representing its own short sound a (enunciate), which is a simple vocal molded merely, at the hard palate, producing a palatal and a continuant. n, the immediate consequent, is a consonant, representing its own proper sound n (enunciate), which is a subvocal partially interrupted at the teeth, producing a dental and a continuant. d, the remote consequent, is a consonant, representing its own proper sound d (enunciate), which is a subvocal, perfectly interrupted at the teeth, producing a dental and an abrupt. 2. H, the antecedent of the base, is a consonant, representing its own proper sound h (enunciate), which is an aspirate partially interrupted at the soft palate, producing a guttural and a coalescent. e, the base of the syllable, is a vowel, representing its own short sound e (enunciate), which is a simple vocal molded merely, at the hard palate, producing a palatal and a continuant. 1, the immediate consequent of the base, is a consonant, representing its own proper sound 1 (enunciate), which is a subvocal jiartially interrupted at the teeth, producing a dental and a continuant. p, the remote consequent of the base, is a consonant, representing its own proper sound p (enunciate), which is an aspirate perfectly interrupted at the lips, producing a labial and an abrupt. Prog. XXIII: 1. the 1. Antecedent, 2. Is a 1. Vowel, 3, Representing 2. Base, 2. Consonant, wlaat sound, 3. Consequent, 3. Digraph, if any? 4. Diphthong, 5. Trigraph, 4. Which is a 1. Simple vocal, 5. 1. Perfectly interrupted at the 1. Lips, 2. Compound vocal, 3. Partially interrupt d at the 2. Teeth, 3. Subvocal, 3. Molded at the 8. Hard palate, 4. Aspirate, 4. Soft palate, G. Producing a 1. Labial, 7. And an 1. Abrupt. 2. Dental, 2. Continuant. 3. Palatal, 3. Coalescent. 4. Guttural. 1. S, the antecedent of the base, is a consonant, representing its own proper sound s (enunciate), which is an aspirate partially interrupted at the teeth, producing a dental and a continuant. i, the base, is a vowel, representing its own long sound I (enunciate), which is an open compound vocal molded at the soft palate and the teeth, producing a gutturo-dental and a continuant. g, the immediate consequent, is an aphthong. ^ n, the remote consequent, is a consonant, representing its own proper sound n (enunciate), which is a subvocal partially interrupted at the teeth, producing a dental and a continuant. 172 THE SYSTEM METHOD. Prog. XXiy.— Oral Analysis (Continued by Prog. XXIII). 1. D, the remote antecedent, is a consonant, representing its own proper sound d (enunciate), whicli is an aspirate perfectly interrupted at the teeth, producing a dental and an abrupt. w, the immediate antecedent, is a consonant, representing its own proper sound w (enunciate), which is a subvocal partially interrupted at the lips, producing a labial and a coalescent. 1, the base of the first syllable, is a vowel, representing its own short sound 1 (enunciate), which is a simple vocal molded at the hard palate, pro- ducing a palatal and a continuant. n, the consequent of the base of the first syllable, is a consonant, rep- resenting its own proper sound n (enunciate), which is a subvocal partially interrupted at the teeth, producing a dental and a continuant. d, the antecedent of the base of the second syllable, is a consonant, representing its own proper sound d (enunciate), which is a subvocal per- fectly interrupted at the teeth, producing a dental and an abrupt. 1, the subvocal base of the second syllable, is a consonant, representing its own proper sound 1 (enunciate), which is a subvocal partially interrupted at the teeth, producing a dental and a continuant. e, the consequent of the base of the second syllable, is an aphthong. 3. A, the base of the first syllable, is a vowel, representing its own long sound a (enunciate), which is a simple vocal molded at the teeth, producing a dental and a continuant. c, the remote antecedent of the base of the second syllable, is a con- sonant, representing its own hard sound € (enunciate), which is an aspirate perfectly interrupted at the soft palate, producing a guttural and an abrupt. r, the subvocal base of the second syllable, is a consonant, representing its own proper sound r (enunciate), which is a subvocal partially interrupted at the hard palate, producing a palatal and a continuant. e, the consequent of the base of the second syllable, is an aphthong. 3. N, the antecedent of the base of the first syllable, is a consonant, rep- resenting its own sound n (enunciate), which is a subvocal partially inter- rupted at the teeth, producing a dental and a continuant. 0, base of the first syllable, is a vowel, representing its own long sound o (enunciate), which is a simple vocal molded at the lips, producing a labial and a continuant. b, antecedent to base of second syllable, is a consonant, representing its own proper sound b (enunciate), which is a subvocal perfectly interrupted at the lips, producing a labial and an abrupt. 1, subvocal base of second syllable, is a consonant, representing its own proper sound 1 (enunciate), which is a subvocal partially interrupted at the teeth, producing a dental and a continuant. e, the consequent of base of second syllable, is an aphthong. SYSTEM OBTHOGBAPHY—OMADE B. irs Prog.XXY: /. A ^um. of Noun. J 2. Rule for Spellir )g. 3. Gender. a. Singular. b. Plural. c. Common. a. Rule . b. Irregular. c. Arbitrary. a. Masculine. b. Feminine. c. Neuter. d. Common. Man = singular, irregular. masculine. Boy = singular, rule 10, masculine. Horse = singular, rule 11, common. Tree = singular. rule 10, neuter. William = singular. rule 10, masculine. Aunt = singular. rule 10, feminine. Pith = singular. rule 10, neuter. Kiss = singular, rule 11, neuter. Church = singular, rule 11, neuter. Woman = singular. irregular, feminine. Hawks = plural. rule 10, common. Ohio = singular. rule 10, neuter. Queen = singular. rule 10, feminine. Armies = plural. rule 11, neuter. Fly = singular. rule 11, neuter. Prog. XXYI : /. Hum. of Noun. 2. Rule for Spelling. 3. Gender. a. Singular. b. Plural. c. Common. a. Rule . b. Irregular. c. Arbitrary. d. Foreign. a. Masculine. b. Feminine. c. Neuter. d. Common. Cargo = singular. rule 11, neuter. House = singular. rule 11, neuter. Echo = singular, rule 11, neuter. Calf = singular. rule 11, common. Knife = singular. rule 11, neuter. Feet = plural. irregular. neuter. Indices = plural. foreign. neuter. Basis = singular. foreign. neuter. Scholar = singular. rule 10, common. Mary = singular. rule 11, feminine. Thursday = singular, rule 10, neuter. Fannies = plural. rule 11, feminine. Eagle = singular, rule 10, common. Mother = singular. rule 10, feminine. Biscuit ^ singular, rule 10, neuter. 17. THE SYSTEM METHOD. )g.XXyiI: /. P. Parts of Verb. 2. Regularity. 3. R. for Spelling. a. Imperfect. b. Perfect. c. Imp. Pcarticiple. d. Perf. Participle. a. Ecgular. b. Irregular. c. Both. a. Eule . b. Arbitrary. Abide = imperfect. irregular. rule 5. Arise = imperfect. irregular, rule 5. Am = imperfect. irregular. rule 3. Awake = imperfect. both, rule 5. Beat == imperfect, irregular. rule 4. Beam = imperfect, regular. rule 4. Begin = imperfect. irregular, rule 2. Bend = imperfect both, rule 4. Bereave = imperfect. both. rule 5. Bet = imperfect. both, rule 2. Bid = imperfect, irregular, rule 5. Bind = imperfect, irregular, rule 4. Blend = imperfect. both. rule 4. Dare = imperfect, both, rule 5. Deal = imperfect. both. rule 4. Prog. XXYIll : /• P. Parts. 2. Infection. 3. R. for Spelling, a. Imperfect and b. Perfect and c. Perf. Participle. a. Complete. b. Defective. c. Redundant. a. Rule . b. Arbitrary. Chide = chide, chid, chidden, etc., redundant. rule 2. Forget = forget, forgot, forgotten, etc. redundant, rule 2. Must = must. defective, rule 3. Set = set, set, set, complete. rule 3. Weave = v?eave, vfove, woven, etc.. redundant, rule 5. Wed = wed, wed, wed, etc.. redundant. rule 2. Slay = slay, slew, slain. complete, arbitrary. Wet = wet, wet, wet, etc.. redundant. rule 2. Ought = ought, defective. rule 3. Freeze = freeze, froze, frozen, etc., redundant. rule 5. Choose = choose, chose, chosen. complete, rule 5. Behold = behold, beheld, beheld. complete, rule 3. Cleave = cleave, cleft, cloven, etc.. redundant. arbitrarjr Get = get, got, gotten. complete. rule 2. Prove prove, proved, proven, etc., redundant, rule 5. IDEOGRAPHIC VSTORK-TREE;. Final Letters and Hyphen Branches. Copyright, 1886, by I. E. WILSOX. [175] ire THE SYSTEM METHOD. Progo XXIX. — Consonant Doubled by Learner's Work-Tree. (See previous page.) ITotc. — No other means of learning the rules of spelling is at all to be compared with this organizing process of branching the particulars through the science system of spelling, the Learner's Work-Tree. Mark that in this branching work, the stu- dent is, with every case, to write entirely out the branch ends, in doing which he writes, each time, a rule of spelling complete. The Arabic numbers on the branch tops refer to the unorganized (i. e., unbranched) rules as given on pages 155 and 156. For the work-tree, see previous page. The character v' below = accented vowel. Pass = consonant doubled, f, 1, and s after v' in monosyllables, 1.^ Will = consonant doubled, f, 1, and s after v' in monosyllables, 1. Off = consonant doubled, f, 1, and s after v' in monosyllables, 1. Stab = consonant doubled, none except f, 1, and s in monosyllables, 3. Tab = consonant doubled, none except f, 1, and s in monosyllables, 3. Whiz := consonant doubled, none except f, 1, and s in monosyllables, 3. Robbed = consonant doubled, between single v' and vowel suffix, 2. Foppish = consonant doubled, between single v' and vowel suffix, 2. Boiling = consonant doubled, none after a digraph or diphthong, 4. Compel = consonant doubled, none except f, 1, and s in monosyllables, 8. Task = consonant doubled, none after a consonant, 4. Moralize = consonant doubled, none after unaccented vowel, 4. Emitting = consonant doubled, between single v' and vowel suffix, 2. Solid = consonant doubled, none except f, 1, and s in monosyllables, 2. Prog. XXX. — Towel Changed by Learner's Work-Tree. (See previous page.) Saving = vowel dropped, but e before vowel suffix, 5. Forced = vowel dropped, but e before vowel suffix, 5. Maries = vowel changed, y to i, but before others (than 7 and 8), 7. Glancing = vowel dropped, but e before vowel suffix, 5. Malays = vowel changed, y to i, not after a vowel, 8. Gayety = vowel changed, y to i, not after a vowel, 8. Merrier = vowel changed, y to i, but before others (than 7 and 8), 7. Glorious = vowel changed, y to i, but before others (than 7 and 8), 7, Crying = vowel changed, y to i, not before 1, 7. Hoeing = vowel dropped, but e before vowel suffix, excep. to 5. Shoeing = vowel dropped, but e before vowel suffix, excep. to 5. Reviled = vowel dropped, but e before vowel suffix, 5. Babyish = vowel changed, y to i, not before i, 7. Monkeys = vowel changed, y to i, not after a vowel, 8. Tying = vowel changed, ie to y, before ing, 9. Changeable = vowel dropped, not e in ce and ge before a and o, excep. to 5. Traceable = vowel dropped, not e in ce and ge before a and o, excep. to 5. ^The Arabic numbers refer to the rules for spelling given in Appendix C, page 155. SYSTEM OBTHOGRAPHY— GRADE B. Ill Prog. XXXL- -Plui Patrick Henries = plu. sign, Foxes = plu. sign, Xenophons = plu. sign. Hindoos = plu. sign. Bowels = plu. sign, B's — plu. sign, Echoes = plu. sign. Cantos = plu. sign. Skies = plu. sign, Keys = plu. sign. Chicagoes = plu. sign, Folios = plu. sign, Africas = plu. sign. Optics = plu. sign, Measles = plu. sign, Riches = plu. sign. Frescoes = plu. sign. Cameos = plu. sign, Moneys = plu. sign, Gregories = plu. sign Plural Sign by Learner's Work-Tree. , es to nouns not coalescing with s, 11. , es to nouns not coalescing with s, 11. , s only to nouns coalescing with s, 10. , s only to nouns in o after vowel, 10. , s only to nouns coalescing with s, 10. , 's to letters, figures, etc., 12. , es to nouns in o after consonant, 11. , es to nouns in o after cons., excep. to 11. , es to nouns not coalescing with s, 11. , s to nouns coalescing with s, 10. , es to nouns in o after consonant, 11. , s only to nouns in o after vowel, 10. , s only to nouns coalescing with s, 10. , s only to nouns coalescing with s, 10. , s only to nouns coalescing with s, 10. , es to nouns not coalescing with s, 11. , es to nouns in o after consonant, 11. , s only to nouns in o after vowel, 10. , s only to nouns coalescing with s, 10. sign es to nouns not coalescing with s, 11. Prog. XXXII. — Rules for Spelling by Learner's Work-Tree. Oresteses = plu. sign, es to nouns not coalescing with s, 11. Puff = con. doubled, f, 1, and s after v' in monosyllables, 3. Guess = con. doubled, f, 1, and s after v' in monosyllables, 3. Squatting = con. doubled, between single v' and vowel suffix, 2. Baggage = con. doubled, between single v' and vowel suffix, 2. Preference = con. doubled, none after unaccented vowel, 4. Preferred = con. doubled, between single v' and vowel suffix, 2. Chastely = vowel dropped, not e before consonant suffix, 6. Incited = vowel dropped, but e before vowel suffix, 5. Dying = vowel changed, ie to y before ing, 9. Maries (1) = vowel changed, y to i, but before others, 7. Maries (2) = plu. sign, es to nouns not coalescing with s, 11, Shoeing = vowel dropped, but e before vowel suffix, excep. to 5. Attorneys (1) = plu. sign, s only to nouns coalescing with s, 10. Attorneys (2) = vowel changed, y to i, not after vowel, 8. Rays (1) = plu. sign, s only to nouns coalescing with s, 10. Rays (2) = vowel changed, y to i, not after vowel, 8. Potatoes = plu. sign, es to nouns in o after consonant, 11. Navigation = vowel dropped, but e before vowel suffix, 5. Mindanaos = plu. sign, s only to nouns in o after vowel, 10. 12 1Y8 THE SYSTEM METHOD. Prog. XXXIII.— Rules for Spelling by Work-Tree. (Continued.) Stool = con. doubled, none after digrapli or diphthong, 4. Stayed = vowel changed, y to i, not after vowel, 8. Heading = con. doubled, none after digraph or diphthong, 4. Decays = plu. sign, s only to nouns coalescing with s, 10. Nationalities = plu. sign, es to nouns not coalescing with s, 11. Ourselves = plu. sign, es to nouns not coalescing with s, 11. Foretelling = con. doubled, between single v' and vowel suffix, 2. Gluey = vowel dropped, but e before vowel suffix, 5. Clayey = vowel changed, y to i, not after vowel, 8. Obeyed = vowel changed, y to i, not after vowel, 8. Ruing = vowel dropped, but e before vowel suffix, 5. Tied = vowel dropped, but e before vowel suffix, 5. Tying '= vowel changed, ie to y before ing, 9. Squabbish = con. doubled, between single v' and vowel suffix, 5. Modeled = con. doubled, none after unaccented vowel, 4. Traveler = con. doubled, none after unaccented vowel, 4. - Tracing = vowel dropped, but e before vowel suffix, 5. Enfranchisement = vowel dropped, not e before consonant suffix, 6. Prog. XXXIY.— Rules for Spelling by Work-Tree. (Continued.) Seer = vowel dropped, but e before vowel suffix, 5. Bagging = con. doubled, between single v' and vowel suffix, 2. Truism = vowel dropped, but e before vowel suffix, 5. Business = vowel changed, y to i, but before others, 7. Bluish = vowel dropped, but e before vowel suffix, 5. Twos = plu. sign, s only to nouns coalescing with s, 10. Delays = vowel changed, y to i, not after vowel, 8. Chaunceys = plu. sign, s only to nouns coalescing with s, 10. Dennies = vowel changed, y to i, but before others, 7. Ciceroes = plu. sign, es to nouns in o after consonant, 11. Suing = vowel dropped, but e before vowel suffix, excep. to 5. Biased = con. doubled, none after unaccented vowel, 4. Shoer = vowel dropped, but e before vowel suffix, 5. Died = vowel dropped, but e before vowel suffix, 5. Tinged = vowel dropped, but e before vowel suffix, 5. Tingeing = vowel dropped, but e before vowel suffix, excep. to 5. Chargeable = vowel dropped, not e in ce or ge before a or o, excep. to 5, Grievance = vowel dropped, but e before vowel suffix, 5. SYSTEM OBTHOGBAPRY— GRADE B. 1Y9 Prog. XXXY : /. Phonic Spelling. 2. Syllabication. {Diacritical Spelling.) Deriding = Boxes = David§ = Motleys = Rulable = Turnkeys = Eehoing = Sixes = Raveled = Merriment = Monkeys = Distended = Days = Greys = Guyots = Salable = Harries == deriding, b 6 k s e z, D a V i d z, Motliz, r oo 1 a b 1, 1 11 r n k e z, e k o i n g, s 1 k s e z, r a V e 1 d, merriment, m u n k i z, distended, daz, Graz, Geoz, s a 1 a b 1, H E r r i z. de-rid-ing, box-es, Da-vids, Mot-leys, rul-a-ble, turn-keys, ech-o-ing, six-es, rav-el-ed, mer-ri-ment, mon-keys, dls-tend-ed, days. Greys, Guy-ots, sal-a-ble, Har-ries, 3. R. fon Spelling. a. Eiiles 1-15. b. Arbitrary. rule 5. rule 11. rule 10. rules 10 and 8. rule 5. rules 10 and 8. arbitrary. rule 11. arbitrary. rule 7. rules 10 and 8. rule 4. rules 8 and 10. rules 8 and 10. rule 10. rule 5. rules 7 and 11. Prog. XXXYI: /. Phonic Spelling. 2. Syllabication. 3. {Diacritical Spelling.) Changing = Maries = Civilizing = Tapping = CWckasas = Shoveled = Henries = Decreeing = Dictionaries = Dishonesty = 197 _ = Pianos = Gyulays = Shawnees = Aortas = Mogeses = Potatoes = Driness = changing M a r i z, 9ivilizin tapping, Chikasaz s h Li V e 1 d, Henriz, dekreing, d i k s h u n e r i z, dizonesty, p i a n o z, Giulaz, S h a n e z, a 6 r t a z, M o z e z e z, p o t a t o z, d r i n e s, .ang-u Ma-ries, civ-il-iz-ing, tap-ping, Chick-a-sas, shov-el-ed, Hen-ries, de-cree-ing, dic-tion-a-ries, dis-hon-est-y, ic-y, pi-an-os, Gy-u-lays, Shaw-nees, a-or-tas, Mo-ses-es, po-ta-toes, dri-ness. R. for Spelling. a. Rules 1-15. b. Arbitrary. rule 5. rules 7 and 11. rule 5. rule 2. rule 10. arbitrary, rules 7 and 11. rule 5. rules 7 and 11. arbitrary, rule 5. rule 10. rules 8 and 10. rule 10. rule 10. rule 11. rule 11. rule 7. Appendix D— Defmitioijs of Orthographic Terms. 1. Oegans of speech. — Those parts of the human system used in speaking. 2. Articulatory organs. — Those organs of speech used in joining the sounds used in speaking. They are the lips (labia), teeth (dentes), hard palate (palatum), soft palate (uvula), nasal cavities, and the larynx. 3. Vocal chords. — Two chords extending across the larynx. They are flanked by thin half-moon-shaped membranes, and, by their vibration, produce the voiced sounds. 4. Voice, or voiced sounds. —Any sound made by the vibration of the vocal chords, as b and a in bad. 5. Whisper, or aspirate sound. — Any sound made without the vibration of the vocal chords ; as, e and t in cat. 6. Vocal. — A voiced sound unob- structed by the articulatory organs ; as, a and a in cat and cane. 7. Sub- vocal. — A voiced sound obstructed by the articulatory organs ; as, b and g in beg. 8. Aspirate. — A whispered sound ; as, e and t in cat. 9. Base. — The letter (or letters) of a syllable representing its vocal ; as, e, a, and oi in met, cat, and boil. 10. Antecedent. — A letter preceding the base ; as, m, c, and b in met, cat, and boil. 11. Consequent. — A letter following the base ; as, t and 1 in cat and boil. 12. Ajbrupt, explodent, or mute. A sound that explodes and cannot be continued ; as, t and d in stand. 13. Continuant. — A sound that can be continued until the breath is exhausted ; as, n and s in stand. 14. Vowel. — A letter representing a vocal sound ; as, e, i, and y in set, sit, and fly. 15. Consonant. — A letter rejDresenting a subvocal or an as- pirate sound; as, s, t, f, and 1 in sit and fly. 16. Aphthong, or silent, letter. — A letter having no sound, but used to determine the signification of a word. 17. Digraph. — The union of two letters to represent one sim- ple sound, vocal, subvocal, or aspirate ; as, oa, th, and ti in boat, thin, and nation. 18. Diphthong. — The union of two vowels to represent a com- pound vocal sound ; as, oi and ou in boil and foul. 19. Simple vocal. — A single or elementary vocal ; as, i and u in pin and tub. 20. Compound VOCAL. — A combination of two elementary vocal sounds ; as, oi, ou, i, and u in boil, foul, pine, and tube. 21. Coalescent. — A sound that precedes and unites (coalesces) with a vocal ; as, w, y, wh, and h in wet, yet, what, and hot 22. Trigraph. — The union of three letters to represent one sound ; as, eau in beau. 23. Labial. — A sound molded or obstructed at the lips ; as, b, p, o, and 00 in bit, lip, note, and boot. 24. Dental. — A sound molded or obstructed at the teeth ; as, t, s, e, and a in sit and sedate. 25. Palatal. — A sound molded or obstructed at the hard palate ; as, j, r, e, and a in jug, rut, set, and sat. 26. Guttural. — A sound molded or obstructed at the soft palate ; as, g, k, u, and a in get, keg, tub, and far. 27. Orthography. — The science of the letter. [180] The System Method— No. 17. Work-Book of System Orthography, Grade A, FOR THE USE OF Public Schools, Normal Schools, Teachers' Institutes, and Colleges, AND ESPECIALLY ADAPTED TO Soi3:ooxj ZE^^^osiTioisrs: IN WHICH THE STUDENT LEARNS AND REMEMBERS UNDER THE GUIDANCE OF THE "LEARNER'S WORK-TREE" BY ORGANIZ- ING, OR BRANCHING, THE LETTER-PARTS OF WORDS THROUGH THEIR CLASSES AS CLASS BRANCHES OF SUCH ORTHOGRAPHIC-TREE WHOLE. -^)?(^- BY ISAAC ELDRIDGE WILSON, INVENTOR OP THE LEARNER'S WORK-TREE, AUTHOR OF THE SYSTEM METHOD, A SERIES OF TEXT-BOOKS FOR SCHOOLS, A SERIES OF STUDENT'S WORK-BOOKS, ETC. THE MAN MAY TEACH BY DOING, AND NOT OTHERWISE."— EMERSON. .F Co:: -corvRiG^7f.Pj\ AUG 13 1886 J?) 3 irYMj CHICAGO, Ilvlv.: Learner's Work-Tree Compact. 1SS6. COPYRIGHT, 1886, BY ISAAC ELDRIDGE WILSON. ALL RIGHTS RESERVED. Any infringement will be prosecuted to the full extent of the law. Appendix. A— Lessons to be Assigned. Peog. I. — Rat, lag, mop, clam, sank, brag, left, skip, curl, prompt, brisk, grist, spelt, jump, of, end, go, up, as, acts, hemp, sworn, burst, dwarf, start, clump, cramp, struck, strict, quick, wax, quest, script, quirk, lynx. Prog. II. — "Web, ben, went, busk, bunt, club, stand, want, word, square, wrist, height, psalm, gaze, alms, bribe, stripe, waft, swore, house, glance, trance, flax, text, guide, guise, swamp, squab, skirt, smirk, twirl, wrong. Prog. III. — Horn, scald, clasp, harm, dust, abdomen, judgment, won, quirk, world, stamped, admit, begin, fulfil, expel, yard, spark, plump, sworn, helm, wept, upon, papa, robin, veto, winter, hotel, alas, occur, eddy, nadir, dollar, tutor, wary, convict, fluent, frequent, fresco, influx, kingdom, edict, quinsy, ubiquity. Prog. IV. — Hydra, espy, crypt, cyst, quilt, guilt, tryst, impromptu, symptom, clink, banquet, blanket, sward, topic, Latin, of, index, rancid, vigil, next, helmet, defect, standard, blink, mongrel, conquest, crank, swamp, swan, periwig, calico, postman, cobbler, carpenter, vortex, exorbi- tant, siDecify, function, identify, cynical, gymnast, mystery, pygmy, pretty, women, busy, covert, wonder, tajjioca, saleratus, macaroni, hornet, ibex. Prog. V. — Dock, musk, bride, half, listen, dishonest, apropos, paradigm, indict, abscind, Wednesday, peremptory, wrong, czar, viscount, whom, hostler, prove, type, one, glove, dirge, verse, depot, homage, pumpkin, ex- ecutive, dishonor, dissolve, glance, ogle, myrtle, critique. Prog. VI. — Strangle, sprinkle, acre, conquest, dangle, victuals, hemor- rhage, legislature, limb, ghost, fight, talk, style, whose, delight, benign, as- sign, gnarl, isthmus, rhombus, sombre, intrigue, oblique, staple, title, ink, gangrene, wink, postscript, querist, abscond, quilt, twine, bade, aqueduct, equipage. Prog. VII. — Rhinoceros, anxiety, schism, yacht, dram, might, zinc, drudge, tubes, guile, lyre, build, landscape, capture, unite, transpire, bequest, conscript, damask, quake, swine, vehicle, icicle, balk, qualm, con- sign, rhetoric, metre, decisive, subjunctive, mercantile, navigable, expostu- late, demonstrate, substitute, invincible, quadruple, example, demagogue, synagogue, plum, jostle, contempt, epistle, honesty, linguist, sanguine, ex- hort, exhibit, patriotism. Prog. VIII. — Anxious, coil, filial, vowel, whey, hoist, flambeau, row, wharf, fish, seal, shun, wire, this, brief, thin, house, leap, weed, haunt, full, [183] 184 THE 8T8TEM METHOD. them, pearl, aye, eye, bureau, stream, hearth, view, church, clay, cheat, gaol, yeast, skein, crawl, chaff, staff, hook, yawl, could, hawk, gown, brooch, shout, poem, knitting, why, lying, frying, defied, tied, deuce, skyey, lieu, mew, glue, plateau, does, ewe, mewl, cruise, two, through, whom, yeoman, laugh, chough, sphinx, flees, fees, boys, enough, languor, anguish, ringlet, whelp, saloon, gingham, heinous, clothier, isle, biscuit, mullen, business, circuit, skewer, eulogy, agree, ewer, sinew. Pkog. IX. — Bright-eyed, Prideaux, Charley, filial, alien, Richelieu, brilliant, cookie, positive, Insulate, adamantine, anxious, charade, whistle, dough, chimney, sew, quoit, quack, junior, builder, sword, clayey, session, diction, choir, mustache, physician, chaise, pinion, question, poniard, liquid, hemorrhage, vignette, phthisic, bouquet, island, rye, dew, rheumatism, due, grazier, vizier, pavilion, guinea, breeches, English, marriage, Moreau, righteous. Prog. X. — Cotton, beckon, pond-lilies, blamable, Rochambeau, loco- motion, extraordinary, primary, disarmed, mouths, baths, dictionary, biology, dishonest, idea, carrying, lucid, suture, duty, oaths, albumen, torpedoes, champagne, abscess, psychology, temptation, mistletoe, February, miscel- lany, weasel, shovel, kettle, abrasion, glacier, valiant, parachute, noncha- lance, machinery, chirography, proficient, coercion, enunciate, avalanche, surgeon, tambourine, hackney, spinous, journey, know, gneiss, autumn, knuckle, knout, photograph, trigraph, hosier, sacrilegious, ocean, nauseous, sumach, patrician, specie, nauseate, glazier. Christian, transition, mosquito, neigh, chevalier. Prog. XI. — Lag, lift, brisk, strict, quick, went, square, wrist, height, alms, stripe, trance, flax, swamp, would, church, cheat, stream, blink, spec- ify, dishonest, Wednesday, whom, hostler, critique, strangle, eager, meager, intrigue, sprinkle, yacht, schism, linguist, qualm, bulk, vehicle, rhetoric, example, demagogue, sugar, nauseate. Prog. XII. — Anxious, wharf, hoist, vowel, filial, whey, aye, bureau, purview, yeast, gaol, crawl, chaff, could, hawk, why, shout, deuce, slew, skyey, two, saloon, sphinx, clothier, biscuit, adieu, sinew, business, colonel, righteous, whistle, sew, quoit, choir, quack, junior, session, physi- cian, poniard, liquid, vignette, rye, grazier, pavilion, vizier, English, nausea, celestial, behavior, eye. Prog. XIII. — Black, twine, ewe, convict, plump, yard, frequent, kingdom, ubiquity, tryst, impromptu, conquest, pygmy, gymnast, hurrah, scissors, abscess, buying, marble, subtle, peculiar, glue, hoe, beef, whelp, whey, bureau, coil, wharf, due, dew, alien, whiskey, chaise, chimney, junior, mansion, air, session, nauseous, fusion, science. Prog. XIV. — Yeast, mine, boil, puss, schism, ocean, nation, lieu, text, spelt, go, script, quirk, lynx, hunt, husk, wrong, clash, busy, catarrh, coaL esce, neuter, herbaceous, farinaceous, gneiss, consequential, voracious, reindeer, sphinx, vehicle, anxious, adieu, vizier, righteous, choir, dough, SYSTEM ORTHOORAPHY—QBADE A. 185 bouquet, phoenix, zephyr, eight, specie, schist, rosette, business, isthmus, asthma, rheum, phlegm, myrrh, unguent, pseudonym. Prog. XV. — Thanks, close, sound, lung, lady, geese, foil, why, strict, square, wrist, light, quoit, lieu, vocal, surd, mute, taken, heaven, poet, tyro, silence, nation, table, lever, talent, destroy, distress, geographic, lucre, stifle, minstrel, usage, error, little, apple, abdomen, bronchitis, bureau, sofa, pri- mary, hyphen, omen, being, echo, preoccupy, brass, cast, counsel, ravel, naval, accessory, notoriety, woolen, often, pivot, ransom, rule. Pbog. XVI. — Spirit, stand, cynic, apathy, medium, specific, docile, cousin, cruelty, acid, upon, exist, beckon, dividing, taking, spoken, loaded, beauteous, aorta, chaotic, stereotype, vacuum, geology, coalesce, real, allop- athy, filial, quack, junior, righteous, clothes, balloon, abscess, leaf, diph- thong, digraph, blanch, staff, answer, shovel, gravel, heaven, intimate, blasphemy, women, eaten, method, lesson, disarm, discern, baths, mouths, oaths, anew, again, palatable, paths, philosophy, abdominal, antipodes, chattel, flannel, cancel, vessel, language, buying, bouquet, bronchitis. Prog. XVII. — Snuffers, thanks, oats, Maries, tongs, Guyots, Grays, goings, earnings, ones, inferiors, Lucies, armies, companies, pincers, Jameses, measles, ashes, boxes, churches, Platoes, U. S. Grants, news, movables, deer, blessings, O. P. Mortons, waters, flies. Mackintoshes, mumps, hoes, dues, robbers, odds, yokes. Miss Thomases, Miss Thomsons, shears, hun- dreds, tubs, tasks, men, lice, children, tenths, lungs, beliefs, molasses, couples, sevens, wages, judges, ices. Harries, Kickapoos, ladies, sixes, geese, vermin, gross, compounds, Greeks, Germans, bases. Prog. XVIII. — Geese, women, dregs, checkers, thanks, oats, lungs, sixties, bowels, companies, coffees, p's, soliloquies, lice, ll's, feet, Haineses, Gyulays, peoples, mumps, Daniel Websters, measles, Patrick Henries, scis- sors, Shawnees, Harries, mathematics, Johns, vitals, shears, news, Ptolemies, Hadleys, Welbies, spectacles, pairs, ones, twos, f's, — 's, politics, teas, Turks, trappings, Moseses, ethics, doings, toes, bitters, dozens, pianos, provisos, Aristotles, valleys, Thackerays, hundredths, sheep, swine, Pharisees, deer, General Gateses, gases, passes, justices, Maries, gross, public, hose. Prog. XIX. — Girl, he, they, mouse, sheep, oxen, deer, she, it, child, we, foot, corps, teeth, vermin, knife, swine, boy, fry, goddess, nephew, John, Mary, you, niece, key. Grey, wages, odds, geese, thou, others, doings, who, which, Lucases, uncle, mother, sir, what, us, women, coffees, son-in- law, James G. Blaine, shoes, lady, ladies, fly, Barclay, two, people, wife, Miss Smith, Mr. Mackintosh, me, its, measles, them, thy, conscience, W. F. "Wilson, peace, cherubim, cattle, gross, chemistry, Germans, he-goat, Pow- hatans, Hindoos, testatrix, gypsy, step-mother, myself, Jew's-harp, St. Peter's, St. Vitus's-dance, bees-wax, Miltons, decree, news, paper, to-day, Moses, angels, commander-in-chief. Prog. XX.— Abide, arise, bear, beget, bend, beset, bid, blow, break, build, can, cut, dig, do, fall, fly, get, give, have, heat, hide, hit, kneel, knit. 186 THE SYSTEM METHOD. lay, let, lie, lose, may, meet, onght, must, pass, pay, pen, put, quit, read, rid, ride, rive, say, see, set, shall, shape, spoil, stick, swear, swim, tell, think, wax, wear, weave, wed, weep, wet, whet, will, win, wis, work, wring, write. Prog. XXI. — Enjoyment, oatmeal, providence, make-shift, tyro, en- gineer, accident, silence, accidental, lemon, lemonade, substantive, eyeball, characteristic, inquiring, uprising, without, destroy, disputable, therea- bout, talent, melon, antejDenult, turncoat, upon, legislature, fatal, Mary, artificer, primarily, taking, parliamentary, extratropical, dividing, bacon, beckoning, bee-hive, passover, transact, obituary, acclimated, therein, whereas, confederated, lyceum, superstructure, heart-broken, untrue, inter- cutaneous, oatmeal, enfold, bankrupt, forbid, dare-devil, drawing-room, moreover, antipathy, antepenult, poet, spoken, clashing, advertisement, incomparable, advertising, demonstration, sacred, being, longer, error, ask- ing, deluded, superstructure, goodman, avalanches, inquiries, contraries, highwayman, forefather, inquiring, extraprofessional, raspberry, little, apple, industrious, nation, table, arrow, apple-tree, irrepressible, daybreak, backbone, bank-bill, backslidden, understanding, monarchy, breakfast, ofEspring, epidermis, outlaw, outdo, hemisphere, hypocritical, influx, over- turn, telegraphy, synagogue, extraordinary, literary, metonymy, chro- nology, youngling, copyright, contemporary, toll-bridge, unseaworthy, commissary, dysenterj', missionary, judiciary. Prog. XXII. — Dividing, financial, pamphlet, acre, lion, spoken, phi- losophizing, civilized, breaths, councils, coflBn, vial, cynic, tuning, beck- oned, upon, exist, measured, fatality, disarming, we, evil, amendment, horseman, theorizing, baths, diseased, seminary, enable, transport, bridal, pianos, again, cuckoos, distress, binomial, decreeing, quitted, ornamented, language, acid, Davids, echoes, ocean, duly, legislature, digit, docile, lever, barren, confederated, answering, whizzed, chaotic, February, paths, miscel- lany, raisin, register, loaded, printing, primeval, oaths, geographic, honor- ary, demoniac, breadths, alphabetical, lucre, strophe, apathy, cruelty, baths, hyphens, sobriety, fourths, statuary, gotten, tasked, cousin, devil, geogra- phy, often, leveling, Welbies, manifold, Guyots, shriveling, sixes, Sicilies, wholly, evil, abdomen, bronchitis, disarm, presumed, stifle, minstrel, Ma- loys, sofas, beauteous, Patrick Henries, acclimated, arrival, often, obituary, extraordinary, thanks, goings, you, they, mumps, swine, what, news, public, that, vitals, grouse. Prog. XXIII. — Nobilities, institution, ideas, mulish, Chinese, secretary, albumen, indivisible, basin, rulable, cousin, azure-like, locomotion, duties, chosen, auxiliary, Hindoos, acclimated, engagement, pitiable, delayed, boyish, fleeing, huzzaed, gaining, mumps, Lucases, waxing, worshiped. Motleys, mansion, canceled, Aristotles, awful, women, miscellaneous, aortas, carried, dissolving, raveled, epidemic, negroes, funereal, Shawnees, meridian, dishonesty, cloths, dictionaries, counseling, deaths, historical. SYSTEM OBTHOGBAPET—OBABE A. Igf Mackintoshes, proven, bragging, secondary, ladies, anatomy, carrying, seven, Chippewas, Harries, pitiable, traveling, Canadas, manned, Stamp- Acts, shoveled, leaden, literary, children, we, Vfho, deer, oats, mice, tongs, eaten, biologies, tripods, notoriety, Latinized, culinary, perilous, stolen, sutures, hidden, motioned, direction, raisin, society, facing, discerned, advertised, toiling, exaggeration, changeable, jollity, apparatus, gaining, attorneys, wooing, merriest, Indias, bitumen, Thackerays, echoed, added, primarily, tarred, social, nation, classes, galloped, woolen, artificer, epiph- any, advantageous, dishonored, tongs, criterion, carried, icy, months, forci- ble, visionary, laths, declaration, lengths, hoeing, breadths, defended, heaven, tertiary, Malays, debatable, Gyulays, open, contemporary, us, sheep, which, geese, them, their, Esquimaux, hose, gross, our. Prog. XXIV. — Maimed, taming, bandied, diagrammed, canceled, as- signed, mill, grass, summed, foppish, compelling, infer, leveled, recoiling, moralize, emitted, tab, tail, solidify, affixing, vowel, alas, brass, shriveled, responded, yes, desisted, oddly, talented, if, is, plasterer, extolling, signal- ize, communication, whetted, hoofed, howled, joining, mauling, hailed, lifeless, fancied, allotting, sensible, continuing, giddiness, renewed, pun- iness, violation, boggy, devotion, lodgement, shameful, reviewing, prosecu- tion, allurement, weariness, legation, arrival, absolution, gnat, connivance, stony, contemplation, desirous, surveyed, clog, coercion, convincing, daily, imbued, knell, stoppage, ragged, knob, lilies, lodging, scraggy, gneiss, palatable, roguish, divorcible, perjury, chastisement, allotment. Prog. XXV. — Saved, Maries, Maloys, married, gayety, merrier, fore- ordain, sing-song, glorious, hoeing, crying, Juliuses, Baileys, tying, hiss, reviled, planner, iron-clad, revealed, thus, reveling, us, babyish, bluish, glueyness, monkeyish. Harries, Grays, Sicilies, Motleys, baggage, enjoyed, died, toad, decreeing, whitish, hying, Jew's-harp, bereavement, rarer, Ciceroes, Sir John Falstafls, equipped, vied, Anglo-Saxon, Patrick Henries, Welbies, pilferer, ball, quit, robbed, soliloquies, Gyulays, hateful, by-word, reform, volleys, buying, traceable, duly, tracing, hottest, dauber, perform- ance, judgment, advantageous, manifold, make-believe, humbugged, peri- wigged, stayed, excusable, lovable, iciest, diagrammed, paleness, Barclays, agreeing, reform, quarreled, catalogues, redbreast, whereas, busying, where- ever, incitement, first-born, movement, woeful, moneys. Nancies, alloys, anti- American, to-day, Dennies, attorneys, Gregories, Godfreys, outrageous, mortgageor. Sir Humphrey Davies, supervision, notice, justice, foggy, supraorbital, child-like, pre-occupy, re-collect, coaction, re-echo, drawing- room, to-morrow, co-eternal. Prog. XXVI. — Foxes, Xenophons, Jameses, Maries, statues, whiskeys, waters, Chaunceys, Hindoos, Quincies, Mays, DicTcens's works, bowels, Rosalies, j'MSfe'ces' warrants, Secretary of the Interior's oflBce, Patrick Henries, Mosbies, palfreys, toes, maid-servants, Mileses, jellies, Christmases, An- tonies, mice's holes, races, King James's translation, Humphreys, Peter's book, 188 THE SYSTEM METHOD. brothers-in-law, echoes, coffees, brandies, Martineaus, Leipnitzes, sons-in- law, girls' bats, runaways, Beed and Kellogg' s grammar, turnkeys, son-in-laio' s farm, mottoes, for conscience sake, Moses's laws, negroes, Eickapoos, skies, values, keys, twos, wives, Mary's slate; Maries' bats, Cbicagoes, Uruguays, President Adams's administration, optics, pains, beliefs, Pierce's grammar, bysterics, sheep's borns, ricbes, moneys, Africas, frescoes, griefs, Emories, Godfreys, Oates's command, Cyruses, Victor Hugoes, foc?zes' bon- nets, Gregories, getters-up, R. W. Baileys, deer's bides, Cyntbias, bangers- on, Plinies, loaves, etc.'s, 5's, buzzas, t's, thanks, Penelopes, U. S. Grants history, classes, canaries, valleys. Prog. XXVII. — Rough-hew, Lieutenant- Oovernor's office, catalogued, Bay and Martin's blacking, puff, gearing; agreeing, drier, vied, well-laid- out, flutter, streets, re-enlist, hoeing, judgment, stiff, suing, re-invest, moss, hall, outrageous, lass, forcible, hoed, manageable, repulsive, convalescent, brigadier, Harris's Hermes, gyration, rued, sun-bird, handkerchiefs, ruing, ingathering, yes, proofs, lancing, rebellion, gayety, ceaseless, Moses's writ- ings, abridgement, Ptolemies, gas, calculation, ring-finger, guess, sprout; roll, soldiers' hardships, potatoes, sons-in-law, sprawl, globule, stool, Catbays, has, strut, full, stoppage. Canaries, cockatrice's den, Mississippies, brow- beat, vice-president, far-fetched, AUegbanies, grass, bliss, mobbed, shunned, boil, providence, Ralph Waldo Emerson's writings, veal, sprout, creation, skull, narrative, odds, brot7ie7'-ifi-laic's -place, make-believe, for goodness' sake, finger-ring, mouse-traps, Doctor Lees, children, breakdown. Doctors Lee and Jones, overdone, Milton's "Allegro," pseudo-apostle, Rocky-Mountain park, Webster's and Worcester's dictionaries. Prog. XXVIII. — Chicago is nortli and loest of N'ew York. The North gave freedom to the slaves of the South, /am the Lord's, and he is mine. The thought was, We learn through doing. Behold Charles the Fifth's day. Oh! show me where is He, the high and holy One. We were visiting Brother Cray's. Father Matthew began bis woi-k in 1839. The passage was Englished by an American teacher. Such place Eternal Justice bad prepared. His brother Albert is visiting Governor Sherman. The Son of Man is Lord also of the sabbath. It was sabbath after Wednesday, July 13, in New England. Emerson said, " The man may teach by doing, and not otherwise." Be it enacted. That sidewalks, etc. His name was called, The Word of Life. His maxim was. Enow thyself. The Woman''s National Temperance Union was organized in 1874. The early Fathers were divided in opinion. One of the evangelists says, "Jesus wept." The problems of life are, Where are we? and WJiere are we going? Resolved, TJmt the mind is a real substance. With three steps L compass thy grave, thou who wast so great before. He is my Rock, and there is no unrighteousness in him. /will show thee the bride, the Lamb's wife. Sing unto the Lord, sing praises to his name. The apostrophe only is added, {l.)to abstract nouns that never take the plu- ral; (2.) to plurals ending in s. The Romans reckoned time from the Foun- SYSTEM OBTHOGBAPHY—OBADE A. 189 dation of Borne; the Greeks from the Era of the Olympiads. How proud are we of the Bevolution! James returned Wednesday, September 10th, and died Thursday, November 3d, The Hurons and the Iroquois were allied in a confederacy called The Six Nations. The Methodists and the Catholics are at variance. The advices of President Cleveland were disregarded. General Grant wrote his own history. The planets are Mercury, Venus, Earth, Mars, Jupiter, Saturn, Uranus, and Neptune. The University of Michigan is well established. The Mohegans were an Algonquin tribe. Upon this. Fancy began to bestir herself. There Guilt his anxious revel kept. The central Unity is dishonored by impiety. Whatis matter? Whence is it? and Where to? The Senate sent a deputation to the Eastern emperor Zeno. He undertook a war against Zenoiia, queen of Palmyra. The Egyptian kingdom was divided between Cleopatra and her brother Ptolemy. He resigned the East to his brother Valens. The Empire of the West fell to his son Gratian. Prog. XXIX. — Leo appointed Nepos emperor of the East. Babylon was reduced by Cyrus, king of Persia. He is friendly to Prince Albert. The Sahara or Great Desert lies north of Central Africa. The three historic na- tions are the Aryan, the Semitic, and the Hamitic. The High Court of Par- liament had not sat. Of all my uncles. Uncle Nathan was my favorite. The Fifteenth Amendment reads as follows : " The right of citizens of the United States to vote," etc. Washington Was a Federalist. The currents of the Uni- versal Being circulate through me. The English poets have Shakspearized now for two centuries. The Blue Bibbon Movement was organized in 1871. The Esquimaux live in the far north. Kext the Most High is the Son of Man. To this -ZVflTJiire replies, "J grow." Thank Heaven, we are free. 1 repeat the question, Is your law just? The Bed Biver of the NortJi empties into Lake Winnepeg. The Lady Touchwoods appeal to no moral sense. When it is snowing, Mother Gary is said to be plucking her geese. The daughter of Mother Goose, Elizabeth, was married in June, 1715, to Thomas Fleet, Esquire. The poet JEschylus is 'known as the FatJier of Tragedy. The work entitled, " A History of the Council of Trent," -was written by Father Paul, an ecclesiastic of Venice. The New-England Journal of Education is conducted by Thos. M. Bicknell. The emperor Nero was a cruel tyrant. There slept guilt-born Excess. The Hottentots are an Ethiopian people. The Bevolutionary War was fought for principle's sake. His accent shows him to belong at the South. I was in the Spirit on the Lord's day. When, my countrymen, will you begin to exert your vigor? Behold Charles the Fifth's day. The cruel emperor Tiberius died of excessive profligacy. Mrs. Stoioe is author of Uncle Tom's Cabin. We have two sisters and a brother in loica. Resolved, Thai whatever is, is right. Omaha is icest of New York. The poet Milton wa&hWndi. Will he take us to the sa66ai!A meetings? What means this journeying to the White Hills? The wind blew from the south. The Neto Testament is our chief witness. The Beformation under Martin Luther is a memorable event. The Strait of Gibraltar is wider than the 190 TEE SYSTEM METHOD. IstJimus of Darien. /saw the continent of hope glittering in the vast West. The Polytechnic Institute was founded in 1860. It was granted to the Duke of Yorlc. Progress is from the East to the West. The Biver St. Lawrence is two thousand miles long. The South misunderstood the spirit of the North. The officers of this society shall be a President, a Secretai-y, a Treasurer, and a Sergeant-at-Arms. The Fathers are not authority on scriptural doctrines. He wished to be absorbed into the One — the Infinite. "A Complete Graded Arithmetic, Oral and Written, by the Inductive Method. For Schools and Col- leges." The North and the South are now railroaded together. Shakspeare ridicules Sir Fastolfe, an English general of the times of Henry the Sixth. The people of South America are largely Roman Catholics, We spent the summer in Martha's Vineyard. In Senator Blaine's book, the author eulo- gizes General Grant. This peace recognized Massinissa as king of Numidia. The consul Nero was kept busy watching Hannibal. Peog. XXX. — The White Mountains are in New Hampshire. Hmo wonderful is Death, — Death and his hrotJier Sleep ! Alexander the Great was king of Macedonia. Henry the Fourth crossed the Alps in January. May Providence guide them. He prizes the sahhatJi. Charles the Bold, duke of Burgundy, besieged the town of Nancy, Heaven jjardon Brother Timothy. The letters of Madam De Stael are mourned. There Night and Chaos held eternal anarchy. Neic York was settled by the East India Company. The Father of Waters rises in Lake Itasca, The richest of all lords is Use, And ruddy Health the loftiest muse. The virtues of Lady Washington were emulated by the people. The Shakers regard Mother Ann as a second manifestation of the Deity, A sentence is a thought expressed in words; as, ^.r;; is long. Resolved, That the assembly meet at 8 o'clock. De Gama rounded the Cape of Good Hope, The Teach- ers' Institute convenes in August. Coleridge says, "■Experience is the best school-master." Operation. Face of note, or principal, from August 1, 1881, $1,000.00 Interest from August 1 to April 21, 1882, 43.33 Amount, ^pWZ 21, 1882, |1,043.33 The English hate the French as frivolous. This is the report of Colonel Lee. He is reading the life of the emperor Charles the Fifth. In "King Henry the Fourth," Sir John Falstaff is called Sir John Oldcastle. It was read at a meeting of the British Association. Gender is distinguished, (1.) by suffixes; (S.) by prefixes; (3.) by arbitrary words Your God is a God of gods. The DuJce of York sent out as governor Major Andros, "The Young SYSTEM OBTHOGBAPHT— GRADE A. 191 American : A Lecture read to the Mercantile Library Association, in Boston, February 7, 1884." He believed himself inspired by the Divine Soul. Pres- ident Lincoln was elected in 1860. The Oulf of Mexico is south of the United States. The subject of the lecture was, " The Wisdom of not Aiming at Perfection." A great truth may be expressed in few words; as. Wisdom Is priceless. Here the religion of the East met the philosophy of the West. Know ye, That we, the delegates, do ratify the said Articles of Gonfedoration and Perpetual Union. The question returns. What shall we do? It was about a sabbath-day's journey. Our doings are not unknown to Him. Feudalism and Orientalism had long enough thought it majestic to do noth- ing. They were rapidly Hellenized. The following officers were elected; President, Martin Henry; Secretary, Oliver Tennis; Treasurer, William Den- nison. He returned to England with the Dulee of Monmouth. Gray's Elegy in a Country Ghurch-yard is not excelled in English. It was claimed that the Horih and the South belonged to each other. Regret was there, his eye still cast. With vain repining, on the past. The Two Witnesses are the Old and the New Testaments. Who can compre- hend Him as love? The prophecy of Mother Shipton has failed. Pr.OG. XXSl. — Pre-occupy, co-action, navigation, Mindanaos, pursuing, definition, digging, co-cval, Peleus's son, women's rights, turkeys, congratula- tory, ingathering, mobbed, lookout, maimed, hooted, ox-cart, mottoes, druggist, earthinsas, hardihood, migration, pre-establish, boy-king, Eliza- beth's ruff, player, embargoes, fogginess, start-up, obligatory, children's, send-off, tributary, Uruguays, son-in-law, Waterloos, hallooing, designative. Miss Days, Hsr Majesty, Queen Victoria's government, root-and-branch reform, cribbage, preference, annul, concentration, receivable, ocean-weed, he-goat, Beaumont and Fletcher's plays, baggage, Oresteses, printing-press, peaces sake, re-formation, abhorrence, Joneses, substitution, dogged, decay- ing, diagramming, keyed, nationalities, tried, president of the society's pleas- ure, instilled, inundation, humming-bird, pre-engage, magnetics, whiskeys, Ciceroes, reveling, vice-president, Asiatics, Medes, to-morrow, anti-evangel- ical, fusible, riddance, translator, niatter-of -fact-looking town, preferred, mortgageor, glued, flying, usage, step-father, retaliatory, church-going, Henry Thomas's memory, pilferer, sued, foes, revision, pulleys, obeying, tied, Dufresnoys, obligation, Guyots, Berkeley's, rescuing, traveler, scowled, John Abernethies, biased, relative, emulator, revival, beech-tree, interroga- tory, incited, emigration, moralize, date-tree, peace-offering, revision, to- night, manifold, Felix's library, tinged, shoer, hoeing, hoed, catalogued, rav- eled, stewing, Euripides's works, long-expected news, commander-in-chief, ear-ring. Pko&. XXXII. — Trying, jellies, co-operation, demonstrative, angels' visits, re-collect, opposition, delays, devotion, accompaniment, financier. 192 THE SYSTEM METHOD. well-to-do, to-day, Stoddard & HenM^s algebras, Lasarus's son, puppet- prince, man-loving Americans, demurring, plumage, mortgageor, pre-exist, gluing, righteousness' sake, noticeable, driest, scantiness, Cbaunceys, legalize, ten-foot pole, expected, moderator, reversion, modeled, heart-broken, played, catalogued, re-absorb, tranquilize, worshiped, providence, pine-tree, seer, chastely, pear-tree, repulsive, imagination, dotage, pre-emption, shoed, en- terprising, easy, business, obligatory, Humphreys, cribbage, Charlottes, anti-American, many-colored, narrative, dying, dyeing, died, shoeing, seeing, suing, sued, brewing, biased, canceled, penciled, abetted, owed, priest, sewing, busied, pro-tempore secreta;ry, expedition, dyed, day-book, Anglo- Saxon, book-keeping, New-York directory, forget-me-not, alliance, Sixth- street car, fluctuation, donation, up-hill game, St. F^^!tts'.s-dance, pre-eminent, one-hour rule, globule, to-day, Pharisees' self-conceit, opinion, distributive, sine-qua-non principle, Jew's-harp, a devoutly-to-he-toished consummation, stipulator, night-time, Jesus's name, glow-worm, ArcJiimedes's screw, good- looking, somebody else's business, Henry the Eighth's abilities, my old teacher, Jfr. Smith's instruction. Prog. XXXIH. — A+shore, for-(-give, drunk-|-ard, dis-j-join, wiz-j-ard, be-[-dew, dis-)-please, black-|-en, en-|-trap, be-|-friend, teach-j-able, un-|- school-j-ed, re-|-new+al, health-f-ful, un-|-taught, king-f-dom, beginn-j-er, pay-)-ment, fore-j-tell, direct+or, pre-f-judge, pjiy-f-ee, e+duc+at+ion, blu-j-ish, name+less, mis-|-take, re-|-plant, Sun-)-day, care-|-ful, con-|-cur, pre-j-vent-j-ive, super-|-natur-)-al, intro-J-duce, eagl-|-et, clos-]-et, man-|-ly, dark-|-some, blood-j-y, wild+er-j-ness, op+press+ive, friend+ship, power -j-less-j-ness, skeptic+ism, im-fit+at-f-ion, puri+ty, fruit-^-age, safe-j-ty, im+pure, a+wake, gif+t, bon-(-d, be-|-think, be-j-cause, an-f-onym-|-ous, in-f-ef-f-fici-j-ent, amphi-|-bi-|-ous, inter-|-cede, Ana-^-bapt-j-ist, cardi-j-ac, con-|-ceal-|-ment, opulent, Tartar-(-ean, anti-j-pod+es, phys-)-ics, organ-j- ize, ob-|-liter-|-ate, apo-|-stas-|-y, apo-f-stle, en+thusi-|-asm, cata-f-rrh, joe +ose, variol-|-oid, per+vade, pel+luc-f-id, dia-f-meter, dis-|-syl-|-lable, multi-|-tude, gnost-j-ic, ec-|-clesi-|-ast-|-ic-|-al, di+phthong, ec+centr-f-ici -|-ty, e+nerv+ate, pre-f-dict, liber+ty, pro-t-vid-)-ent, eu-f-dem-j-ic, epi-|- dem-|-ic, re-j-pel, hypo+crite, dram-(-a, trace-j-able, glob+ule, post-|-ure, obs-|-tin-|-acy, di-|-visible, creat-|-ure, retro-|-spect-|-ive, met-]-onym-f-y. For additional particulars, see Appendix F of "The System Method," No. 18, page 305, Prog. XSXIV. — Se-j-greg-f-ate, para-|-dox, con-]-flu-|-ence, ir-(-re-|-sol -j-ute, sapon-j-ace-l-ous, sub+merge, suf-j-fer, peri-|-ostium, savag-f-er+y, per-j-cent-l-age, supra-j-orbit-j-al, sur-f-vey, con-f-vey-f-ance, syn-|-chron-|- ous, pro-j-gram, sy-|-stem, sym-f-path-j-y, as+sist+ant, thef-|-t, tra+verse, dee-|-d, ad-|-versa-|-ry, ac-|-cesso-(-ry, pay-|-ed, hear-j-d, ab+ject, ad-j-voc -j-ate, af+flx, stai-|-d, thrif-f-t, ob-|-serv-|-ato-}-ry, ap-|-plaud, gold-|-en, leather+n, heav-)-en, mod-l-er-|-n, en-j-dear, daw-)-n, am-j-put-f-ate, anti-|- cip+ate, liber-}-at-|-ion, larg-j-er, chatt-j-er, mole-|-cule, mo-|-st, man^- SYSTEM ORTHOGRAPHY— GE ABE A. 193 hood, in+born, bi+ped, bi+gamy, bis-J-cuit, kern-|-el, bow-|-el, circum-t- vent, mis+give, mors+el, tab-j-le, go+ing, leav-f-ing, bagg+ing, wolf-j- ish, soldie+ry, fer+ry, mani-f-kin, n+aught, con-|-nect, cor+rug-j-ate, nob+le, oiit+live, dribb-|-le, inagni-|-fy, gos+ling, aquat+ic, peri+od-|- ic, not-|-ice, utt-j-er, contro+vert, stup+id, puer-|-ile, bumm+ock, ball+ oon, de-|-flect, racc-|-oon, viper-|-ine, don-l-at-|-ion, ladi+es, es-j-tabl-j-ish, grow-|-th, beal+tb, over-]-reach, over+coat, s+melt, dis-j-pel, s+pread, tru+th, de-j-ism, Wesley+an-l-ism, di+verge, Huss+ite, un-|-do, un-|-true, e+dent+ate, beal+th-f-y, ef+flu-|-ent, de-|-lus-[-ive, capt+ive, bab+y, under-f-ling, extra+mur-t-al, eas-j-y, an+y, with-|-stand, witb-f hold, Will -f-ie, ex-|-cresc-|-ence. For additional particulars, see " The System Method," No. 18, Appendix F, page 305. 13 Appendix B— Orthographic Chart. *^^<^i5S==r-&- — 1. Diacritical Marks of I Simple and Compound Vocals. 1. a = fate, ate. 3. 6 = come, other. 3. a = bat, at. 4. o = wolf, bosom. 3. a = pear, hare. 5. o = do, prove. 4. a = far, ah. 6. 6 = form, ought. 5. a = brass, ask. 1. oo = hoot, boom. 6. a = wall, all. 2. do = hook, wool. 7. a = wad, wan. 1. u = tube, iise. 1. e = field, key. 2. u = tub, us. 2. e = pen, let. 3. u = rude, rule. 3. g =1 there, 6re. 4. V = pull, full. 4. e = they, prey. 5. u = burn, urge. 5. e = herd, her. 1. y = sky, fly. 1. 1 = bind, mile. 2. y = nymph, cyst. 2. 1 = tin, hill. Compound Vocals. 3. i = pique, mien. 1. oi = ai in coil, toy. 4. i =; bird, sir. 2. ou = iioo in foul, how. 1. o = vote, old. 3. i = jie in pine, try. 2. 6 = not, odd. 4. u = eoo in tube, Use. 2. Diacritical Marks of Subvocals and Aspirates. 1. 9- soft. = 9ite. 9. ng =: n in sing. 2. €, hard, = call. 10. ph = f in phantom. 3. ch , soft. = 9hais€ !. 11. qu = kw in quack. 4. ch , bard, = ehoir. 12. s, aspirate. = yes. 5. ch , unmarked, = child. 13. §, subvocal. = ha|. 6. g, hard. = get. 14. th, aspirate. = thin. ■7- g. soft. = sem. 15. til, subvocal, = this. 8. n, guttural, = link. 16. X, aspirate, ks = fix. 17. ^, subvocal, gz = e^ert. 3. Physiological Classification of Subvocals and Aspirates. Labials. Dentals. Palatals. Gutturals. 1. Abrupts: b d J g 2. Continuants m, V 1, n, z, th r, zh ng 3. Coalescents: ; w y 4. Abrupts: P t ch k 5. Continuants ;: f s, th sh 6. Coalescents; wh h Names of the Diacritical Marks. 1. a = macron. 6. e = tilde, or wave. 2. a = breve. 7. 9 = cedilla. 3. a = dot. 8. § = perpendicular. 4. a = diaeresis. 9. e = horizontal. 5. a = caret, or circumflex. [1941 Appendix C— Rules for Spelling. (a) Consonant Doubled. 1. Rule I. — In monosyllables, terminal f, 1, or s, preceded by a single vowel, is written double ; as, staff, hill, pass, off, mill. Exceptions: clef, if, of, as, is, has, was, yes, his, this, us, thus, gas, pus. 2. Bule II. — A terminal consonant preceded by a single accented vowel, is doubled before an added suffix beginning with a vowel; as, robH-ed = robbed; fop + ish = foppish; refer -{- ed = referred; quit + ed = quitted , diagram + ed = diagrammed. (b) Consonant Not Doubled. 3. Rule III — All terminal consonants except f, 1, and s are written single; as, cat, man, tar, brag, tab, ham, lap, wax, pod, whiz, task. Exceptions: add, ebb, odd, inn, butt, buzz, Qgg. 4. Rule IV. — A terminal consonant preceded by a digraph, diphthong, another consonant, or an unaccented vowel, is never doubled; as, boil + ing := boiling; defend + ed = defended ; travel + ing = traveling. (c) E Sometimes Dropped. 5. Rule V. — Terminal e is dropped before a suffix beginning with a vowel ; as, love -)- ing = loving; force + ible = forcible ; love + ed := loved; ice + y = icy. Exception 1: -ce and -ge retain e before a and o; as, peace + able = peaceable; courage \ ous = courageous. Exception 2: E as part of a digraph is usually retained ; as, hoe -f- ing = hoeing ; agree -j- ing = agreeing. At other times it is dropped ac- cording to the rule ; as, sue -f- ing = suing ; see -|- er = seer. 6. Rule VI. — Silent terminal e is not dropped before an added suffix beginning with a consonant; as, large + ly = largely; pale + ness = pale- ness; judge -f ment = judgement. (Judgement is also, but arbitrarily, written judgment.) Exceptions: true + ly ^ truly; awe + ful = awful; whole -f ly = wholly. (d) Y and IE Changed. 7. Rule VII. — Except before i, y preceded by a consonant is changed to i before an added suffix; as, carry -f ed = carried; fly -|- es = flies; glory -|- ous = glorious; merry -]- er = merrier; pity -f able = ]3itiable ; many + fold = manifold; Mary -f es = Maries; Ptolemy + es = Ptolemies; Welby + es = 'Welbies. (Maries, Harries, etc., are sometimes written Marys, Harrys, [1951 196 THE SYSTEM METHOD. Welbys, etc. But such a spelling is at discord with the genius of English orthography.) Exceptions: duty + ous = duteous; beauty + ous = beauteous; bounty -\- ous = bounteous; miscellany + ous = miscellaneous. 8. Rule VIII. — Y terminal preceded by a vowel is not changed; as, play 4- iiig = playing; valley + s = valleys; gay -|- ly = gayly; Thackeray + s = Thackerays; Motley + s = Motleys; Gray + s = Grays. 9. Bule IX. — le terminal is changed to y before ing; as, die + ing = dying; tie + ing = tying. (e.) Regulak Plurals in s, es, or 's. 10. Rule X. — Nouns ending in a syllable that coalesces with s, includ- ing those in o preceded by a vowel, add s only for the plural; as, tack-s, day-s, hoe-s, decree-s, water-s, coffee-s, TJ. S. Grant-s, Miss Thompson-s, Grey-s, Maloy-s, Guyot-s, Gyulay-s, Hadley-s, David-s, Canada-s, Shawnee-s, Chippewa-s, Stamp-Act-s, Magna Charta-s, tong-s, vital-s, new-s, physic-s, thank-s, going-s, mump-s, embryo-s, cuckoo-s, Hindoo-s. 11. Rule XL — Nouns ending in a syllable that does not coalesce with s, and nouns ending in o preceded by a consonant, add es for the plural; as, box-es, class-es, six-es, summons-es, fli-es, compani-es, loav-es, liv-es, Mari-es, Sicili-es, ladi-es, Mrs. "Welbi-es, negro-es, echo-es, hero-es, Lucas-es, S. S. Mackintosh-es, James-es, Crumless-es, Cicero-es, Plato-es, hous-es, hors-es. Exceptions: canto-s, quarto-s, piano-s, solo-s, duodecimo-s, tyro-s, proviso-s, octavo-s, memento-s, lasso-s.* 13. Rule XII. — Letters, signs, and figures substitute the apostrophe (') for the e of the es; as, g's, ll's, p's, +'s, — 's, etc. (f.) Irregular, Arbitrary, and Common Plurals. (a.) The following nouns form the plural irregularly: — Child, children. Louse, lice. Ox, oxen. Foot, feet. Man, men. Woman, women. Goose, geese. Mouse, mice. Tooth, teeth. (b.) Pronouns form the plural arbitrarily: — I, we. Thou, you. He, they. My, our. Thy, your. His, their. Me, us. Thee, you. Him, them. (c.) Nouns from foreign languages usually retain their f® reign plurals. Examples are, — Automaton, automata, r. Erratum, errata. Medium, media, r. Axis, axes. Focus, foci. Nebula, nebulae. Basis, bases. Formula, formulae, r. Phenomenon, phenomena. Beau, beaux, r. Genius, genii. Radius, radii. * See note at close of page 304. She, they. It, they. Her, their. Its, their. Her, them. It, them. SYSTEM OBTHOORAPHY— GRADE A. I97 Cherub, cherubim, r. Genus, genera. Stratum, strata. Criterion, criteria. Hypothesis, hypotheses. Vertebra, vertebrse. Ellipsis, ellipses. Lamina, laminae. Virtuoso, virtuosi. Encomium, encomia, r. Magus, magi. Vortex, vortices, r. (d.) The following nouns and pronouns have ticin-Uke (i. e., common, or equivocal) forms for the singular and the plural : — Nouns: sheep, deer, svfine, vermin, hose, fry, gross, neat, grouse, pub- lic. Esquimaux, Japanese, Cyclops, corps, molasses^ Pronouns: who, which, what, that, as. (g.) POSSESSIVES IN 'S. 13. Rule XIII. — The apostrophe (') and s are added, — (a.) To singulars not having the same form for the plural, as, boy's hat, Peter's book, Milton's "Allegro," General Gates's command, Pierce's gram- mar, Harris's "Hermes," cockatrice's den, Moses's writings. (b.) To singulars having the same form for the plural; as, a sheep's fleece, a deer's horns. But the apostrophe should be placed after the s in forming the possessive plurals; as, sheeps' fleeces, a load of deers' horns. (c.) To plurals unlike their singulars and not ending in s; as, men's, children's, oxen's, geese's. (d.) To the last part of any sort of a compound or complex name or substantive phrase; as, Ralph Waldo Emerson's writings; my step-father's residence; my brother-in-law's piace; the Secretary of the Interior's office; every body else' s business; the president of the society's pleasure; the lawyer, Henry Feliafs, library; Henry the Eighth's honor; sergeant-at-arm's duty. (e.) To the last name in a series denoting joint possession; as, Beaumont and Fletcher's plays. Reed and Kellogg's grammars. (f.) To each name in a series denoting separate possession; as, Webster's and Worcester's and Murray's dictionaries. (h.) POSSESSIVES IN THE ('). 14. Rule XIV. — The apostrophe (') only is added, — (a.) To abstract names that never take the plural; as, for conscience' sake; for goodness' sake; for righteousness' sake; for justice' sake; for peace' sake. (b.) To plurals ending in s, as, girls' hats, Pharisees' self-conceit, sol- diers' hardships, eagles' wings, angels' visits, justices' warrants. (i.) The Hyphen (-). 15. Rule XV. — The hyj)hen (-) may be used only when the word parts unite with contrasted accents, to represent one compound idea; as, a sharp-edged instrument; its too-great-to-be-estimated value will then appear. Wherever there are two distinct ideas, and similar accents, the word parts are separate, not hyphened; as, a sharp edged instrument. Mark the differ- ence between " Sixth-street car" and " sixth street-car." 198 THE SYSTEM METHOD. Note. — The question whether in any current (dictionary contained) compound, the hyphen is to be used or the parts written consolidated, is to be settled, of course, only by reference to the dictionary. Rule XV. is only designed to show what words not found in the dictionary either hyphened together or consolidated as one com- pound, may be correctly hyphened. 16. Rule XVI. — The hyphen separates two vowels separately pro- nounced when the first forms part of a prefix; as, pre-engage, re-echo, anti- organizing. 17. Rule XVII. — The hyphen separates re, meaning " again," from any word when consolidating destroj^s its meaning; as, re-collect (means "col- lect again," not recollect), re-memher (means "member again," not remem- ber). SYSTEM OBTHOGBAPHY—OBADE A. I99 CHALLKNOK. (a.) Propositions. — To hasten the development of the perfect method of learning and teaching the science system of orthography, the author of "The System Method" challenges any educator of the United States, Can- ada, England, Germany, or France to prove either of the following propo- sitions: — 1. That this system method is not a better method than he himself uses. 2. That this system method is not the true method. (b.) Conditions. — 1- Such educator shall compress his argument into not to exceed 150 8vo. pages of a 200-page discussion. 2. The author of "The System Method" to have the last 50 pages of such 200-page discussion, in v^hich to reply. 3. The decision to be made by five judges, three of whom shall be James Eussell Lowell, of Boston, A. Wilford Hall, of New York, and the State Superintendent of Indiana; these three to choose a fourth and a fifth. 4. The judges shall make no qualification, but render their decision af- firming or denying the propositions as they stand. 5. The expression, "this system method," of the propositions, to be construed to mean that method of learning and teaching the science system of orthography exemj)lified in this (No. 17) Work-Book, and advocated in this Volume I. of "The System Method," together with the author's 50 last pages of the 200-page discussion. 6. Such educator shall make no use of the Learner's Work-Tree or any counterfeit in establishing any argument. (C.) Offer. — 1. As a summons to earnest exertion, the author of "The System Method " offers to print and deliver to such educator, for his own disposal, and free of all cost to him, 1,000 volumes of such 200-page discus- sion, neatly bound in cloth. 2. The period of this offer extends from July 1st, 1886, to July 1st, 1888. 3. Such educator shall prepare his 150 first pages in proper form to be examined by the judges, in MS. or print, as he may choose. 200 THE SYSTEM METHOD. r( 1. R a I: /. Position. 2. a. Antecedent. b. Base. 0. Consequent. antecedent, base. Voice. a. Voiced. b. Whispered. voiced, voiced. 3. Sound. 4. a. Vocal. b. Subvocal. c. Aspirate. subvocal, vocal. Continuance. a. Abrupt. b. Continuant. continuant, continuant. 2. t L = consequent, antecedent. whispered, voiced. aspirate, subvocal, abrupt, continuant. a = base. voiced. vocal, continuant. 3. g M = consequent, antecedent, voiced, voiced. subvocal, subvocal. abrupt, continuant. = base. voiced. vocal. continuant. 4. P C 1 = consequent, antecedent, antecedent, wbispei-ed, whispered, voiced. aspirate, aspirate, subvocal. abrupt, abrupt, continuant. a = base. voiced. vocal. continuant. ni = consequent. voiced. subvocal, continuant. 5. S a = antecedent, base. whispered, voiced. aspirate, vocal, continuant, continuant. n = consequent. voiced, subvocal. continuant. 6. k B z=z consequent, antecedent, whispered, voiced. aspirate, subvocal. abrupt, abrupt. r = antecedent. voiced. subvocal, continuant. a = base. voiced. vocal. continuant. 7. g L = consequent, antecedent. voiced, voiced, subvocal, subvocal. abrupt, continuant. 6 = base. voiced, vocal. continuant. 8. f t S k i = consequent, consequent, antecedent, antecedent, base, whispered, whispered, whispered, whispered, voiced. aspirate, aspirate, aspirate, aspirate, vocal. continuant. abrupt. continuant. abrupt. continuant. 9. P C u = consequent, antecedent, base. whispered, whispered, voiced. aspirate, aspirate, vocal. abrupt, abrupt, continuant. r = consequent. voiced. subvocal. continuant. 1 = consequent. voiced. subvocal. continuant. iV^ofe to Teacher. — For the words rat, lag, mop, etc., to be assigned to, and worked up by, your classes, refer to Appendix A of this (No. 17) Work-Book. Refer to the appendixes, also, for any definition or explanation desired. J3e sure to show your class how to get started in doing orthography in this way. See note To Teacher, p. 158. Read, "Why These 4-Step Programs Are Used," "By the Pro- grams the Student Learns by Doing," " The Programs Force the Student to Decide," and " This Method of Branching by Programs Sustained by Authority," on pages 134, 137, 138, and 141, respectively. Read also pp. 79-134 inclusive. For the two great steps to be taken in learning any branch, see pp. 29-31, 160 and 161. SYSTEM OBTHOOBAPHY— GRADE A. 201 Prog. II 1. w e b 2. H u s k 3. S q u a r e 4. W r i s t 5. H t 6. P 1 m 7. S "W r e 8. F 1 a X 9. G u i '. Pos/'tfon. 2. Continuance. 3. Obstruction. 4. Sound. a. Antecedent. b. Base. c. Consequent. a. Abrupt. b. Continuant. c. Coalescent. d. Aphthong. a. Obstructed. b. Unobstructed. a. Vocal. b. Subvocal. c. Aspirate. antecedent. coalescent. obstructed. subvocal. base, continuant, unobstructed. vocal. consequent. abrupt. obstructed. subvocal. antecedent, coalescent, obstructed, aspirate. base, continuant. unobstructed. vocal. consequent. continuant, obstructed. aspirate. consequent. abrupt, obstructed, aspirate. antecedent, continuant. obstructed. aspirate. antecedent. abrupt. obstructed. aspirate. antecedent. coalescent. obstructed. subvocal. base. continuant. unobstructed, vocal. consequent. continuant. obstructed. subvocal. consequent. aphthong. antecedent. aphthong. antecedent, continuant. obstructed, subvocal. base. continuant, unobstructed. vocal. consequent. continuant, obstructed. aspirate. consequent. abrupt. obstructed. aspirate. antecedent. coalescent, obstructed. aspirate. base. continuant. unobstructed. vocal. consequent. aphthong. consequent. aphthong. consequent. abrupt, obstructed, aspirate. antecedent. aphthong. antecedent. continuant, obstructed. aspirate. base. continuant. unobstructed. vocal. consequent. aphthong. consequent, continuant. obstructed, subvocal. antecedent. continuant. obstructed. aspirate. antecedent, coalescent, obstructed. subvocal. base, continuant, unobstructed, vocal. consequent. continuant. obstructed. subvocal. consequent. aphthong. antecedent. continuant. obstructed, aspirate. antecedent, continuant. obstructed, subvocal. base. continuant. unobstructed, vocal. consequent. continuant, obstructed, aspirate. antecedent. abrupt. obstructed, subvocal. antecedent. aphthong. base, continuant. unobstructed, vocal. consequent, continuant, obstructed. subvocal. consequent. aphthong. [202] ORTHOGRAPHIC ^^VORK:-TREE; CClVtRLKXE:. Copyright, 1886, by I. E. WILSON. 1. H = 4. SYS'. TEM OETHl JGBAPHT— GRADE A. 203 Prog. III.— Branching by the Learner's Work-Tr ee. H = aspirate, coalescent, guttural. = sonant, vocal, simple. guttural. r = sonant, subvocal, continuant, palatal, lingual. liquid. n = sonant. subvocal, continuant, dental, nasal. S = aspirate. continuant, dental. sibilant. c = aspirate, abrupt. guttural. a = sonant. vocal. simple. guttural. 1 = sonant. subvocal. continuant. dental, lingual. liquid. d = sonant. subvocal, abrupt, dental. W = sonant, subvocal, coalescent. labial. o = sonant, vocal, simple. guttural. n = sonant, subvocal, continuant. dental, nasal. Q = aspirate. abrupt. guttural. u = sonant, subvocal, coalescent, labial. i = sonant, vocal, simple. guttural. r = sonant. subvocal. continuant. palatal, lingual, liquid. k = aspirate. abrupt, guttural. Y = sonant. subvocal, coalescent. dental. a = sonant. vocal. simple. guttural. r = sonant. subvocal. continuant. palatal, lingual. liquid. d = sonant, subvocal. abrupt. dental. F = aspirate. continuant. labial. r = sonant. subvocal, continuant. palatal, lingual. liquid. e = sonant. vocal, simple. dental. q = aspirate, abrupt. guttural. u = sonant. subvocal, coalescent. labial. e = sonant, vocal, simple. palatal. n = sonant, subvocal. continuant, dental, nasal. t = aspirate. abrupt. dental. I = sonant. vocal. simple. palatal. n = sonant, subvocal. continuant. dental, nasal. f = aspirate. continuant. labial. 1 = sonant, subvocal. continuant. dental, lingual. liquid. u = sonant. vocal. simple. guttural. ix = aspirate. continuant. gutturo-dental, sibilant. ^The sound of x in influx is the same as s made from the position k. Ask the pupil to place the articulatory organs in position to make k, then without making k, quicldy pronounce s, and he will perceive why x is a gutturo-dental, i. e., a k-s sound. Because the tree constitutes the orthographic science system, it shows us plainly what is system-like (law-like), and what is outside of system, i. e., lawless, or exceptional, and therefore to be learned empirically, not through the system as a visible means. Thus, the tree informs us that this x is an exceptional letter by the fact that x can- not be branched upward through such science system. (See " The System Method" — No. 6, Demonstration XXII.) 20J: THE SYSTEM METHOD. ] Pro? ?. IT.— Branching by the Learner's Work-Tree. 1. E = sonant, vocal, simple. palatal. s = aspirate, continuant, dental. sibilant. P = aspirate. abrupt. labial. y = sonant, Yocal, simple, palatal. 2. c = aspirate, abrupt. guttural. r = sonant, subvocal. continuant, palatal. lingual, liquid. y = sonant. vocal. simple, palatal. p = aspirate, abrupt. labial. t = aspirate, abrupt. dental. 3. B = sonant. subvocal, abrupt, labial. a = sonant. vocal, simple, palatal. n = sonant, subvocal. continuant, dental, nasal. q = aspirate, abrupt. guttural. u = sonant, subvocal. coalescent. labial. e = sonant, vocal, simple, palatal. t = aspirate. abrupt. dental. 4. I = sonant. vocal. simple. palatal. n = sonant. subvocal. continuant. dental, nasal. d = sonant, subvocal. abrupt, dental. e = sonant, vocal. simple. palatal. X = aspirate. continuant, gutturo-dental , sibilant. 5. R = sonant, subvocal, continuant. palatal. lingual, liquid. a = sonant, vocal. simple, palatal. u = sonant, subvocal. continuant. dental. nasal. c == aspirate. continuant. dental. sibilant. i = sonant. vocal, simple. palatal. d = sonant. subvocal. abrupt. dental. 6. P = aspirate. abrupt. labial. y = sonant. vocal simple. palatal. g = sonant. subvocal. abrupt. guttural m = sonant, subvocal. continuant, labial. nasal. y = sonant, vocal. simple. palatal. 7. w = sonant. subvocal. coalescent. labial. = sonant. vocal. simple. palatal. m = sonant. subvocal, continuant. labial. nasal. e = sonant. vocal. simple. palatal. n = sonant, subvocal. continuant. dental, nasal. 8. B = sonant. subvocal. abrupt. labial. u = sonant. vocal, simple. palatal. s = sonant. subvocal. continuant. dental. sibilant. y = sonant. vocal, simple. palatal. SYSTEM ORTHOGRAPHY— GEADE A. 205 Prog. T: /. Position. 2. Construction. 3. 1. H a 1 f 2. L i s t e ii' 3. D i s h n e s t 4. A P r o P s 5. I n d i c t a. Antecedent. b. Base. c. Consequent. antecedent, base, consequent, consequent, antecedent, base, consequent, consequent, antecedent, base, antecedent, base, consequent, antecedent, base, consequent, base, consequent, consequent, base, consequent, antecedent, base, antecedent, base, consequent, base, consequent, antecedent, base, consequent, consequent. a. Vowel, or Base. b. Consonant. consonant, vowel, consonant, consonant, consonant, vowel, consonant, consonant, vowel, consonant, consonant, vowel, consonant, consonant, vowel, consonant, vowel, consonant, consonant, vowel, consonant, consonant, vowel, consonant, vowel, consonant, vowel, consonant, consonant, vowel, consonant, consonant. Letter. 4. a. Sonant. b. Non-sonant. c. Aphthong. non-sonant, sonant, aphthong. non-sonant, sonant, sonant, non -sonant, aphthong. aphthong. sonant, sonant, sonant, sonant, aphthong. sonant, sonant, sonant, non -sonant, non-sonant, sonant, non-sonant, sonant, sonant, non-sonant, sonant, aphthong. sonant, sonant, sonant, sonant, aphthong. non-sonant. Sound. a. Vocal. b. Subvocal. c. Aspirate. aspirate, vocal. aspirate, subvocal. vocal, aspirate. subvocal. subvocal, vocal, subvocal. vocal. subvocal. vocal. aspirate. aspirate. vocal. aspirate. subvocal. vocal. aspirate. vocal. vocal, subvocal. subvocal. vocal. aspirate. ^In such words as listen (lie-n), acre (a-kr), thistle (this-1), etc., n, r, and 1 though the base of the syllable, are consonants, not vowels ; since they represent subvocal, not vocal, sounds. With this exception, namely, that n, r, and 1 form the base of the second syllable in such words as heaven, sombre, gristle, etc., the base of a syllable is always a vowel representing, of course, a vocal sound. 206 THE SYSTEM METHOD. Prog. YI : /. Sound. 2. Obstruction. 3. Construction. 4. Letter. a. Vocal. b. Subvocal. c. Aspirate. d. Aphthong. a. Unobstructed. b. Obstructed. a. Vowel, or Base, b. Consonant. . a. Sonant. b. Non-sonant, 1. S = aspirate, obstructed, consonant. non-sonant. t = aspirate, obstructed. consonant, non-sonant. r = subvocal, obstructed, consonant, sonant. a = vocal. unobstructed, vowel. sonant. n = subvocal, obstructed, consonant. sonant. g = subvocal, obstructed. consonant, sonant. 1* = subvocal, obstructed, consonant, sonant. e = aphthong. 2. A = vocal. unobstructed. vowel, sonant. c = aspirate, obstructed, consonant. non sonant. r* = subvocal, obstructed. consonant, sonant. e = aphthong. 3. V = subvocal, obstructed. consonant, sonant. i = vocal, unobstructed. vowel. sonant. c = aphthong. t = aspirate. obstructed. consonant, non-sonant. u = aphthong. a = aphthong. 1* = subvocal. obstructed. consonant. sonant. s = subvocal, obstructed. consonant. sonant. 4. G = aphthong. n = subvocal. obstructed, consonant, sonant. a = vocal. unobstructed, vowel. sonant. r = subvocal. obstructed, consonant. sonant. 1 = subvocal. obstructed. consonant, sonant. 5. S = aspirate, obstructed. consonant, non sonant. = vocal, unobstructed, vowel. sonant. m = subvocal. obstructed. consonant, sonant. b = subvocal. obstructed, consonant. sonant. r* = subvocal. obstructed. consonant. sonant. e = aphthong. 6. I = vocal. unobstructed. vowel. sonant. n = subvocal, obstructed. consonant. sonant. t = aspirate. obstructed. consonant. non- sonant. r = aspirate, obstructed. consonant. sonant. i = vocal, unobstructed, vowel. sonant. g = subvocal, obstructed. consonant. sonant. u = aphthong. e = aphthong. * See note on previous page. SYSTEM OBTHOGBAPHY— GRADE A. 207 Prog. VII : /. Letter. 2. Continuant. 3. Obstruction. 4. Interruption. a. Sonant, or tonic. b. Aspirate, or atonic. c. Aphthong. a. Abrupt. b. Continuant c. Coalescent. a. Labial. . b. Dental. c. Palatal. d. Guttural. a. Perf. interrupted. b. Part, interrupted. c. Molded. 1. A = sonant, or tonic, continuant. palatal. molded. n = sonant, or tonic. continuant. guttural, part, interrupted. X = tonic. continuant. gutturo-dental, part, interrupted. i = tonic. continuant. gutturo-dental (ae), molded. e = tonic, continuant. dental. molded. t = aspirate, or atoniC; , abrupt. dental, perf. interrupted. y = tonic, continuant. palatal. molded. 2. S = atonic. continuant, dental, part, interrupted. ch .= aphthongs. i = tonic, continuant. palatal. molded. s = tonic. continuant, dental. part, interrupted. m = tonic. continuant, labial. part, interrupted. 3. G = tonic, abrupt. guttural. perf. interrupted. U2 = aphthong. i = tonic, continuant. gutturo-dental (ae), molded. 1 = tonic, continuant, dental, part, interrupted. e = aphthong. 4. B = tonic. abrupt. labial, perf. interrupted. a = tonic. continuant. guttural. molded. 1 = aphthong. k = atonic. abrupt. guttural. perf. interrupted. 5. E = tonic. continuant. palatal. molded. xi = tonic. continuant. gutturo-dental, part, interrupted. h = atonic. coalescent, guttural. perf. interrupted. = tonic. continuant. guttural. molded. r = tonic. continuant, palatal. part, interrupted. t = atonic. abrupt. dental. perf. interrupted. 1 In such words as exhort, exact, exhaust, etc. , x is pronounced z from the posi- tion g. To understand how x here is both guttural (g) and dental (z), let the stu- dent put the articulatory organs in readiness to make the sound g; then, without making g, suddenly pronounce the sound z. He will then understand why x is a gutturo-dental, i. e., a g-z sound. Accordingly, in influx, x is a gutturo-dental, i. e., k-s aspirate sound. To pronounce it, take the position for k, and then make s with- out making the k-sound. ^ U and i cannot here be taken together to form a diphthong, because the com- pound vocal sound represented (ae) may be represented by 1 alone ; neither could the two together be called a digrai^h, since the vocal represented (ae) is not simple, but compound. See definitions of digraph and diphthong, page 180, and also note 1 under Prog. VIII., page 208. 208 THE SYSTEM METHOD. Prog. VIII : /. Union. 2. Letter. 3. Subdivision. 4. Obstruction. a. Letter. b. Digraph. c. Trigraph. d. Diphthong a. Tonic, or sonant. a. Abrupt. I). Atonic, or non-sonant, b. Continuant, c. Aphthong. c. Coalescent. ;. d. Simple vocal. e. Compound vocal. a. Labial. b. Dental. c. Palatal. d. Guttural. 1. C = letter, atonic, abrupt. guttural. oil ^ diphthong. tonic. compoiind vocal, , gutturo-pal'tal. 1 = letter, tonic, continuant. dental. 2. Wh= digraph, atonic, coalescent. labial. ey2 = digraph, tonic. simple vocal. dental. 3. B = letter. tonic, abrupt. labial. r = letter. tonic. continuant. palatal. ie^ = digraph. tonic, simple vocal, dental. f = letter, atonic. continuant, labial. 4. C = letter. atonic, abrupt. guttural. r = letter. tonic, continuant. palatal. u = letter. tonic. simple vocal. labial. i = letter, aphthong. s = letter. tonic. continuant, dental. e = letter. aphthong. 5. T = letter. atonic. abrupt, dental. w^ = letter. aphthong. = letter. tonic. simple vocal, labial. ^The differences between a vowel digraph and a diphthong are: 1. A diphthong represents a compound vocal sound (thus, in boil, oi=ai), while a digraph represents a simple vocal sound (thus, in feat, ea^e) ; 2. A diphthong represents a sound which cannot be represented by either vowel taken alone (thus, in boil, the sound oi cannot be represented by either o or i taken alone), while a digraph represents a sound that can be represented by one vowel or the other alone (thus, in feat, the sound of ea is often represented by e alone.) 2 The difficult question will arise whether, in whey and brief, the y and the 1 are to be called silent, or whether ey and ie are to be taken together as digraphs, as here above. The principle is: If two letters "cling" in three or more words, they may be called a digraph. Otherwise, one only is to represent the sound, the other to be called an aphthong. Thus e and y in whey are to be called a digraph, since they cUng together in three other words to represent this same sound a : they, prey, con- vey, etc. And so 1 and e in brief are found also in fief, thief, chief, etc. They are therefore a digraph. If, in such words as light, tight, sign, etc., the questions arise whether the gh shall be taken with the following t to form a trigraph, and whether g should be taken with n to form a digraph, observe that what force, or value, is possessed by the gh or the g is exerted on the vowel i, not on the t and the n ; since taking out the gh and the g, changes the sound of i from i to i ; but it no way affects the t-sound. There- fore, the gh and the g are not to be taken with the t and the n. SYSTEM OBTEOQBAPHY— GRADE A. 209 Prog. IX : /. Union. 2. Obstruction. 3. Subdivision. 4. Diacrit. Mark. a. Letter. b. Digraph. c. Trigraph. d. Diphthon a. Labial. b. Dental. c. Palatal, g.d. Guttural. e. Aphthong. a. Abrupt. b. Continuant. c. Coalescent. d. Simple vocal. e. Comp. vocal. a. Macron, f. Tilde. b. Breve. g. Cedilla. c. Dot. h. Perpendicular. . d. DiEeresis. i. Horizontal. e. Caret. j. Unmarked. 1. A = letter, dental. simple vocal, macron. 1 = letter. dental, continuant. unmarked. ii = letter. dental, coalescent, unmarked. e := letter. palatal. simple vocal. breve. n = letter. dental, continuant. unmarked. 2. A = letter. palatal. simple vocal. breve. n3 = letter. guttural (ng). continuant. horizontal. X = letter. gutturo- dental. continuant. unmarked. 11 = letter. dental, coalescent. unmarked. ou^ '= digraph. guttural. simj)le vocal. breve. s = letter. dental, continuant, unmarked. 3. Ch = digraph, palatal, continuant. cedilla. a = letter, guttural. continuant. dot. r = letter. palatal. continuant. unmarked. a = letter, dental. simple vocal. macron. d = letter, dental. abrupt. unmarked. e = letter. aphthong. 4. D = letter. dental. abrupt. unmarked. ou = digraph. labial. simple vocal. macron. gh = letters. aphthongs. 5. Ch = digraph. guttural, abrupt. horizontal. oi = letter. labial. coalescent. unmarked. 1 = letter. gutturo-dental. comp. vocal. macron. - r = letter. palatal. continuant, unmarked. 6. R = letter, palatal. continuant, unmarked. i = letter. gutturo-dental. comp. vocal. macron. gh = letters. aphthongs. t = letter, dental, abrupt. unmarked. el = letter. dental. coalescent. unmarked. ou = digraph, guttural, simple vocal. breve. s = letter. dental. continuant. unmarked. lit is dogmatic and even false to teach that " a, e, i, o, and u are vowels; " for that cannot be a vowel which represents either a sub vocal or an aspirate sound. But in righteous (rlt-yiis), alien (al-yen), choir (kwlr), quack (kwak), e, 1, o, and u repre- sent respectively the subvocal sounds y, y, w, w, ; they are therefore consonants, not vowels. So 1 in anxious (angks-yus) ; and so in ocea7i (o-shiin), nation (na-shim), etc., where e and i are taken with c and t to form consonant digraphs. See note to Prog. XV., of Work-Book No. 16, page 167. 2 See note under Prog. VIII. ^ See note under Prog. X. 14 210 THE SYSTEM METHOD. Prog. X : /. Union. 2. Inierrupiion. 3. Subdivision. 4. Diacnit Mark. a. Letter. a. Pert, interrupted, a. Abrupt. a. Macron, f. Tilde. b. Digraph. b. Part, interrupted, b. Continuant, b. Breve. g. Cedilla. c. Trigraph. c. Molded. c. Coalescent. c. Dot. b. Perpendicular. d. Diphthong, d. Aphthong. d. Simple vocal, d. Diaeresis, i. Horizontal. e. Comp. vocal, e. Caret. j. Unmarked. unmarked. 1. B = a = th = s = 2. D = c = ti = o = n := y 3. P a r q e t G 1 = a : C : ie : r : 5. S : p = e : Ci : e : 6. N : ei : gh: 4. G = letter, perf . interrupted, abrupt, letter, molded, simple vocal, breve, digraph, part, interrupted, continuant, horizontal. ^ letter, part, interrupted, continuant, perpendicular, letter, perf. interrupted, abrupt, unmarked, letter, molded, simple vocal, breve, letter, perf. interrupted, abrupt, horizontal, digraph, part, interrupted, continuant, unmarked, letter, molded, simple vocal, breve, letter, part, interrupted, continuant, unmarked, letter, molded, simple vocal, unmarked, letter, part, interrupted, continuant, unmarked, letter, molded, simple vocal, breve, letter, perf. interrupted, abrupt, unmarked, letter, molded, simple vocal, diaeresis, letter, part, interrupted, continuant, unmarked, letter, perf. interrupted, abrupt, unmarked, letter, aphthong. letter, molded, simple vocal, breve, letter, perf. interrupted, abrupt, unmarked, letter, perf. interrupted, abrupt, macron.^ letter, part, interrupted, continuant, unmarked, letter, molded, simple vocal, macron, letter, part, interrupted, continuant, cedilla, digraph, molded, simple vocal, tilde, letter, part, interrupted, continuant, unmarked, letter, part, interrupted, continuant, unmarked, letter, perf. interrupted, abrupt, unmarked. letter, molded, simple vocal, macron, digraph, part, interrupted, continuant, unmarked, letter, molded, simple vocal, unmarked, letter, part, interrupted, continuant, unmarked, digraph, molded, simple vocal, horizontal, digraph, aph thongs. ^When placed above the letter, the character (-) is conveniently called the "macron ;" when below the letter, it maybe called the "horizontal," for distinc- tion's sake. 2 This silent u is an exception to the law that u after q is a consonant with the sound of w. SYSTEM OBTHOQBAPHY— GRADE A. 211 Prog. XI. — !• the 1. Antecedent, 2. Isal. Vowel, 3, Eepresenting 2. Base, 2. Consonant, what sound, 3. Consequent, if any? 4. Which is a 1. Simple vocal, 5. 1. Perfectly interrupted at the 1. Lips, 2. Compound vocal, 2. I'artially interrupted at the 2. Teeth, 3. Subvocal, 3. Molded at the 8. Hard palate, 4. Aspirate, 4. Soft palate, 6. Producing a 1. Labial, 7. And an 1. Abrupt. 2. Dental, 2. Continuant. 3. Palatal, 3. Coalescent. 4. Guttural, 1. L, (1.) the antecedent, (2.) is a consonant, (3.) representing its own proper sound 1 (enunciate), (4.) which, is a subvocal, (5.) partially interrupted at the teeth, (6.) producing a dental, (7.) and a continuant. a, (1.) the base, (2.) is a vowel, (3.) representing its own short sound a (enunciate), (4.) which is a simple vocal, (5.) molded at the hard palate, (6.) producing a palatal, (7.) and a continuant. g, (1.) the consequent, (2.) is a consonant, (3.) representing its own hard sound g (enunciate), (4.) which is a subvocal, (5.) perfectly interrupted at the soft palate, (6.) producing a guttural, (7.) and an abrupt. 2. Q, the antecedent, is a consonant, representing its own proper sound k (enunciate), which is an asj)irate perfectly interrupted at the soft palate, producing a guttural and an abrupt. u, the antecedent, is a consonant, representing the sound w (enun- ciate), which is a subvocal partially interrupted at the lips, producing a labial and a coalescent. i, the base, is a vowel, representing its own short sound i (enunciate), which is a simple vocal molded at the hard palate, producing a palatal and a continuant. c, the consequent, is an aphthong. k, the consequent, is a consonant, representing its own proper sound k (enunciate), which is an aspirate perfectly interrupted at the soft palate, producing a guttural and an abrupt. 3. A, the base, is a vowel, representing its own Italian sound a (enun- ciate), which is a simple vocal molded at the soft palate, producing a gut- tural and a continuant. 1, the consequent, is an aphthong. m, the consequent, is a consonant, representing its own proper sound m (enunciate), which is a subvocal partially interrupted at the lips, producing a labial and a continuant. s, the consequent, is a consonant, representing its own subvocal sound § (enunciate), which is a subvocal partially interrupted at the teeth, produc- ing a dental and a continuant. 4. F, the antecedent, is a consonant, rejpresenting its own proper sound f (enunciate), which is an aspirate j)artially interrupted at the lips, producing a labial and a continuant. 212 THE SYSTEM METHOD. Prog. XII. — !• thel. Antecedent, 2. Is a 1. Vowel, 3. Kepresenting 2. Base, 2. Consonant, what sound, 3. Consequent, 3. Digraph, if any? 4. Diphthong, 5. Trigraph, 4. Which is a 1. Simple vocal, 5. 1. Perfectly interrupted at the 1. Lips, 2. Compound vocal, 3. Partially interrupted at the 2. Teeth, 3. Subvocal, 3. Molded at the 3. Hard palate, 4. Aspirate, 4. Soft palate, 6. Producing a 1. Labial, 7. And an 1. Abrupt. 2. Dental, 2. Continuant. 3. Palatal, 3. Coalescent. 4. Guttural, 1. Wh, the antecedent of the base, is a digraph, representing its own proper sound wh (enunciate), which is an aspirate partially interrupted at the lips, producing a labial and a coalescent. a, the base, is a vowel, representing its own broad sound a (enunciate), which is a simple vocal molded at the soft palate, producing a guttural and a continuant. r, the immediate consequent, is a consonant, representing its own proper sound r (enunciate), which is a subvocal partially interrupted at the hard palate, producing a palatal and a continuant. f, the remote antecedent, is a consonant, representing its own proper sound f (enunciate), which is an aspirate partially interrupted at the lips, producing a labial and a continuant. 2. H, the antecedent of the base, is a consonant, representing its own proper sound h (enunciate), which is an aspirate partially interrupted at the soft palate, producing a guttural and a coalescent. oi, the base, is a diphthong, representing its own proper sound oi (enunciate), which is a compound vocal molded at the soft ]oalate and teeth, producing a guttur o-palatal and a continuant. s, the immediate consequent, is a consonant, representing its own proper sound s (enunciate), which is an aspirate partially interrupted at the teeth, producing a dental and a continuant. t, the remote consequent, is a consonant, representing its own proper sound t (enunciate), which is an aspirate perfectly interruj)ted at the teeth, producing a dental and an abrupt. 3. V, the antecedent of the base of the first syllable, is a consonant, rep- resenting its own proper sound v (enunciate), which is a consonant partially interrupted at the lips, producing a labial and a continuant. ow, the base of the first syllable, is a diphthong, representing the sound ou (enunciate), which is a compound vocal molded at the soft palate and lij)s, producing a gutturo-labial and a continuant. 6, the base of the second syllable, is a vowel, representing its own short sound e (enunciate), which is a simple vocal molded at the hard palate, producing a palatal and a continuant. SYSTEM OBTHOOBAPHT— GRADE A. 213 Prog. XIII.— Written Synthesis by Synthesis Tree. The student's work in this first step in synthesizing is here omitted, since it was given under "Written Synthesis by Worli-Tree," page 115. Let the reader turn to the student's work there given, and read carefully the explanations there also given. Prog. XIY.— Written Synthesis from Memory (Mental Tree). dental, coalescent, subvocal ; dental, simj)le vocal ; dental, continuant, aspirate; dental, abrupt, aspirate. labial, continuant, subvocal; open, compound vocal; dental, continuant, subvocal; aphthong. labial, abrupt, subvocal; open, compound vocal; dental, con- tinuant, subvocal. labial, abrupt, aspirate; guttural, simple vocal; dental, con- tinuant, aspirate. dental, continuant, aspirate; aphthong; palatal, simple vocal; dental, continuant, subvocal; labial, continuant, subvocal. labial, simple vocal; palatal, continuant, aspirate; guttural, simple vocal ; dental, continuant, subvocal. dental, abrupt, aspirate; palatal, simple vocal; gutturo-dental, continuant, aspirate; dental, abrupt, aspirate. dental, continuant, aspirate; labial, abrupt, aspirate; palatal, simple vocal; dental, continuant, subvocal; dental, etc. guttural, abrupt, subvocal; labial, simple vocal. dental, continuant, subvocal; palatal, simple vocal; guttural, continuant, subvocal; gutturo-dental, continuant, aspirate. guttural, coalescent, aspirate; guttural, simple vocal; dental, continuant, subvocal; dental, abrupt, aspirate. guttural, coalescent, aspirate; guttural, simple vocal; dental, continuant, aspirate; guttural, abrupt, aspirate. aphthong; palatal, continuant, subvocal; guttural, simple vocal; guttural, continuant, subvocal. guttural, abrupt, aspirate; dental, continuant, subvocal; pal- atal, simple vocal; palatal, continuant, aspirate. ^This synthesizing consists in writing /row memory from the topmost branch of the mental copy (of tree, as given on p. 116) downward through three class branches only. This synthesizing of three classes is to be done, not, necessarily, with the synthesis tree before the eyes, but from memory by means of the mental copy of this science-system tree held in mind, Students should be directed to write out their work as above until they become able to do the same work orally. (See " Oral Synthesis," Prog. XV.) Sufficient practice should be given In this written and oral synthesis to fix immovably in student's mind a perfect mental picture of science-system tree of orthography ; for it is the possession of such mental-system whole that enables student afterward to solve quickly and ably any orthographic problem which may be presented to him. ^For X see under Prog. VII., page 207. Yeasfi = Mine = Boil = Puss = Schism = Ocean = Text^ = Spelt = Go = Lynx^ = Hunt = Husk = Wrong = Clash = 214 THE SYSTEM METHOD. Prog. XT.— Oral Synthesis from Memory (Mental Tree). This work is identical with the written synthesis of Prog. XIV., except that students are not to commit the results of their study to writing. They are simply to practice this three-steps-down-the-tree process of work at their study hours, that they may prove, at the recitation hour, their posses- sion of the mental tree-image, by their ability rapidly thus orally to synthe- size the tree classes. Their oral recitation reduced to print would appear as follows: — Thanks = den. con. asp., pal. sim. voc, gut. con. sub., gut. abr. asp., den. con. asp. Close = gut. abr. asp., den. con. sub., lab. sim. voc, den. con. asp., aph. Sound = den. con. asp., open com. voc, den. con. sub., den. abr. sub. Lung = den. con. sub., gut. sim. voc, gut. con. sub. Lady = den. con. sub., den. sim. voc, den. abr. sub., pal. sim. voc. Geese = gut. abr. sub., den. sim. voc, den. con. asjo., aph. Foil = lab. con. asp., open com. voc, den. con. sub. Why = lab. co. asp., open com. voc. Strict = den. con. asp., den. abr. asp., pal. con. sub., pal. sim. voc, gut. abr. asp., den. abr. asp. Square = den. con. asp., gut. abr. asp., lab. co. sub., pal. sim. voc, pal. con. sub., aph. Wrist = aph., pal. con. sub., pal. sim. voc, den. con. asp., den. abr. asp. Light = den. con. sub., open com. voc, aph., den. abr. asp. Quoit = gut. abr. asp., lab. co. sub., open com. voc, den. abr. asp. Lieu = den. con. sub., close com. voc. Vocal = lab. con. sub., lab. sim. voc, gut. abr. asp., pal. sim. voc, den. con. sub. Surd = den. con, asp., gut. sim. voc, pal. con. sub., den. abr. sub. Mute = lab. con. sub., close com. voc, den. abr. asp., aph. Taken = den. abr. asp., den. sim. voc, gut. abr. asp., aph., den. con. sub. Heaven == gut. co. asp., pal. sim. voc, lab. con. sub., aph., den. con. sub. Poet = lab. abr. asp., lab. sim. voc, pal. sim. voc, den. abr. asp. Tyro = den. abr. asp., open com. voc, pal. con. sub., lab sim. voc. Nation = den. con. sub., den. sim. voc, pal. con. asp., gut. sim. voc, den. con. sub. Table = den. abr. asp., den. sim. voc, lab. abr. sub., den. con. sub., aph. Lever = den. con. sub., den. sim. voc, lab. con. sub., gut. sim. voc, pal. con. sub. SYSTEM ORTHOGBAPHT— GRADE A. 215 Prog. XVI. — Oral Synthesis by Position. It has been shown (p. 114) that whoever sees the unit whole, or sys- tem, of orthography, therein sees every part of that system whole ; and that whoever sees the parts distinctly, must needs see the position of the parts. So the proof of pupil's possession of a clear mental image of the science- system unit whole of the orthographic branch, will be his ability instantly to point out the parts of that branch in rapid succession. Who- ever sees the whole of a horse, sees not only every part, but the position of every part, of that horse, and will be able instantly to point to such parts. But pointing out the position of the most distant parts of the orthographic branch is pointing out the branch tops of the tree. Evidently, therefore, the crucial test whether a student really understands the science-system whole, will be whether he is able to spell a word of letters rapidly over the top of the tree. Giving sometimes a different letter, when it is the one that appears in the tree and represents the sound in hand, the student's pointer, at the recitation hour, would run over the tree top thus : — Spirit = s P i r i t Stand = s t a n d Cynic = s i n i k Apathy = a P a th i Medium = m e d i u m Specific = s P e s i f Docile = d 6 s i 1 Cousin = k li z n Cruelty = k r oo e 1 t Acid = a s i d Upon = u P 6 n Exist = e g z 1 s t Beckon = b e k n Dividing = d i V i d i Taking = t a k i ng Spoken = s P o k n Loaded = 1 o d e d Beauteous = b u t e ii s Aorta = a a r t a Chaotic = k a 6 t i k Filial = f i 1 y u 1 Quack = k w a k Junior = J u n y u r Eighteous = r i t y u s Clothes = k 1 o ti. z Balloon = b a 1 oo n Abscess = a b s e s Leaf = 1 e f 216 THE SYSTEM METHOD. Prog. XYII : /. Noun. 2. Current^ 3. Origin of Name. 4. R. for S. a. Common. b. Proper. a. Singular. b. Plural. c. Both. a. In whole. a. Rule . b. In action. b. Arbitrary. c. In part or particle, c. Irregular. d. In substance. d. Common. Snuffers^ = common, plural, in part, rule 10. Oats^ = common. plural. in particle. rule 10. Goings = common, both. in action. rule 10. Armies = common. both, in whole. rule 11. Jameses = proper. both, in whole, rule 11. Measles' = common. plural. in particle, rule 10. Platoes = proper. both, in whole. rule 11. Movables = common. both, in part. rule 10. 0. P. Mortons = proper, both, in whole. rule 10. Waters = common, both. in substance. rule 10. Mumps^ = common. plural, in action. rule 10. Hoes = common, both. in whole. rule 10. Odds = common. plural. in particle, rule 10. Miss Thomases = proper. both. in whole. rule 11. Shears = common, plural. in part. rule 10. Tubs = common. both, in whole, rule 10. Men = common, both. in whole. irregular. Lice = common. both. in whole. irregular. Children = common, both. in whole. irregular. Tenths = common. both. In whole. rule 10. Lungs = common. both, in part, rule 10= Scissors^ = common, plural. in part. rule 10. Molasses* = common, both. in substance, common. Ices = common, both. in substance. rule 11. Harries = proper, both. in whole. rule 11. ^The statements, (1.) that "bellows, scissors, shears, tongs, have no singular forms;" (2.) that "the plural number denotes more than one; as, apples, boys, horses,"— these statements are contradictory, since bellows, shears, scissors, and tongs each denote but one instrument. This mistake the teacher should carefully avoid. It lies in confounding the principles of form and signification. These nouns are in the plural number, not because they denote more than one instrument, but because the name in each case originated in a jJart, which part is repeated to form the whole. Scissors is not in the plural number because it means more than one jjair of scissors, but because in scissors there are two parts. One part = scissor (which is not now currently used) ; two parts = scissors. Observe that scissors = scissor -|- s. Number is a structural change. It is based on structure, not on signification merely. 2 " Current — (a.) singular, (b.) plural, (c.) both," is meant to lead the student to ascertain and decide whecher the singular alone, or the plural only, or whether both the singular and the plural are regularly used (i. e. "current") in the English lan- guage. Thus with "measles in hand, he would decide that only the plural is current ; since the singular, measle, is not used among good authors. 3 See note to Rules X. and XI., page 198. * See Appendix C, (f.), (d.), Nouns. SYSTEM ORTHOGBAPHT— GRADE A 217 Prog. XVIII : /. Noun. 2. Current 3. Origin of N. 4. R. forSpel'g. a. Common. b. Proper. a. Singular. b. Plural. c. Both. a. In whole. a. Eule b. In action. b. Arbitrary. c. In part or particle, c. Irregular. d. In substance. d. Common. Geese = common. both. in whole. irregular. Women = common, both. in whole. irregular. Dregs = common. plural. in particle. rule 10. Checkers = common, both, in whob. rule 10. Thanks = common. plural. in action, rule 10. Oats . = common. plural. in particle. rule 10. Lungs = common. both, in part. rule 10. Sixties'^ = common. both. in whole. rule 11. Bowels = common, both. in part. rule 10. Companies = common. both. in whole. rule 11. Coffees = common. both. in substance. rule 10. Lice = common. both. in whole. irregular. ll's = common. both. in whole, rule 12. Feet = common. both. in- whole, irregular. Haineses = proper, both. in whole. rule 11. Gyulays = j)roper, both, in whole. rule 10. Peoples^ = common. both. in whole. rule 10. Mumps = common. plural, in action. rule 10. Measles = common. plural. in particle. rule 10. Scissors = common. plural. in part. rule 10. Shawnees = proper, both. in whole. rule 10. Harries = proper. both. in whole. rule 11. Johns = proper. both. in whole. rule 10. Vitals = common. plural, in part. rule 10. Shears = common, plural. in part. rule 10. News = common. plural. in part, rule 10. Ptolemies = proper. both in whole. rule 11. Hadleys = proper. both, in whole. rule 10. Pairs = common, both, in whole. rule 10. Ones = common, both. in whole. rule 10. Twos = common, both, in whole, rule 10. P's = common, both. in whole, rule 12 — 's = common. both. in whole, rule 12. Politics = common, plural, in part. rule 10. Teas = common. both. in substance, rule 10. Turks = proper, both. in whole. rule 10. Trappings = common plural. in action. rule 10. Moseses = proper, both. in whole. rule 11. Ethics = common. plural, in part. rule 10. 1 See note to Rules X. and XI., page 198. 218 THE SYSTEM 3IETH0I). Prog. XIX: /• Subsi'ive. 2. Case Forms. 3. Num. Forms. 4. R. forS. a. Noun. a. Nominative, b. Pronoun.b. Possessive, and c. Objective. a. Singular, b. And plural. a. Eule- b. Arbitrary. c. Irregular. d. Common. Girl =noun, He =pronoun, They =pronoun. Mouse :=noun, Sheep^ =noun, Oxen =noun, Deer^ =noun, She =pronoun, It =pronoun, Child =noun. We =pronoun, Foot :=noun, Corps^ =noun, Teeths =noun, Vermin 2=noun, Knife =noun. God^ =noun, Swine^ =noun. Mary =:noun, You =pronoun. Niece =noun, Key :=noun, Grey =noun. Wages* ^noun, Odds =noun, Geese^ =noun, Thou =pronoun, Others =noun, Doings =noun, Who6 =pronoun. Whiche =pronoun, girl, girl's, girl, he, his, him, they, their, them, mouse, mouse's, mouse, sheep, sheep's, sheep, oxen, oxen's, oxen, deer, deer's, deer, she, her, her, it, its, it, child, child's, child, we, our, us, foot, foot's, foot, corps, corps's, corps, teeth, teeth's, teeth, vermin, vermin's, vermin, knife, knife's, knife, God, God's, God, swine, swine's, swine, Mary, Mary's, Mary, you, your, you, niece, niece's, niece, key, key's, key. Grey, Grey's, Grey, wages, wages', wages, odds, odds', odds, geese, geese's, geese, thou, thy, thee, others, others', others, doings, doings', doings, who, whose, whom, , which, which. girl, girls, rules 10, 13 a. he, they, arbitrary, he, they, arbitrary, mouse, mice, irregular, 13 a. sheep, sheep, common, 13 b. ox, oxen, irregular, 18 c. deer, deer, common, 13 b. she, they, arbitrary, it, they, arbitrary, child, children, irregular, 18 a. I, we, foot, feet, corps, corps, tooth, teeth, arbitrary, irregular, 13 a. common, 13 b. irregular, 13 c. vermin, vermin, common, 13 b. knife, knives, rules 11, 13 a. God, swine, swine, Mary, Maries, thou, you, niece, nieces, key, keys. Grey, Greys, wage, wages, odd, odds, goose, geese, thou, you, other, others, doing, doings, who, who. arbitrary, 13 a. common, 13 b. rules 11, 13 a, 7. arbitrary, rules 11, 13 a. rules 10, 13 a, 8. rules 10, 13 a, 8. rules 11,7 14 b. rules 10, 14 b. irregular, 13 c. arbitrary, rules 10, 14 b. rules 10, 14 b. arbitrary. which, which, arbitrary. ^ Some write the possessive plural, sheeps', deers', swines' ; but there is good authority and much reason for, " two sheejfs wool," a load of deer^s horns, etc. * The possessive plurals would be corps' and verm.ins'. 3 Ood^ being a member of no known class, has no plural. * The possessive singular, were it used, would be wage's. 513 d applies both to singulars and to plurals. ^ It is the distinguishing characteristic of a relative pronoun, that it has twin, or common, forms for both numbers. 7 See "The System Method," No. 6, Dem. XIV., b, d, e, (a). SYSTEM mTnOOBAPEY—OEADE A. 219 Prog. XX: /. indicatives '. 2. Participles. 3. Inflection. 4. R. for Spel. a. Imperfect, b. And perfect. a. Imperfect, b. And perfect. a. Complete. b. Defective. c. Redundant a. Rule . b. Arbitrary-c Abide = abide, abode, abiding, abode. complete. rule 5. Arise = arise, arose. arising, arisen. complete. rule 5. Bear = bear, bore, etc.. bearing, borne, redundant. rule 4. Beget = beget, begot, begetting, begotten, etc., redundant, rule 3. Bend = bend, bent, etc., bending, bent, etc.. redundant. rule 4. Beset = beset, beset. besetting, beset. complete. rule 3. Bid = bid, bade, etc.. bidding, bidden, etc :., redundant, rule 3. Blow = blow, blew. blowing, blown. complete. arbitrary. Break = break, broke. breaking, broken. complete. rule 4. Build = build, built, etc.. building, built, etc.. redundant. rule 4. Can = can, could. defective. rule 3. Cut = cut, cut. cutting, cut, complete. rule 3. Dig = dig, dug, etc., digging, dug, etc.. redundant. rule 3. Do = do, did. doing, done. complete. arbitrary. Fall = fall, fell, falling, fallen. complete, rule 1. Fly = fly, flew. flying, flown, complete. rule 7. Get = get, got. getting, gotten. complete. rule 8. Give = give, gave. giving, given, complete. rule 5. Have = have, had. having, had. complete. rule 5. Heat = heat, heat, etc., heating, heat, etc., redundant. rule 4. Hide = hide, hid. hiding, hidden. comjDlete, rule 5. Hit = hit, hit. hitting, hit. complete. rule 3. Kneel = kneel, knelt. kneeling, knelt. complete, rule 4. Knit = knit, knit, etc.. knitting, knit, etc., redundant. rule 3. Lay = lay, laid, etc., laying, laid, etc.. redundant. rule 8. Let = let, let, letting, let. complete. rule 3. Lie = lie, lay, lying, lain, complete. rule 9. Lose = lose, lost. losing, lost, complete. rule 5. May = may, might. defective, rule 3. Meet = meet, met. meeting, met. complete, rule 4. Ought = ought. defective. rule 3. Must = must. defective, rule 3. Pass = pass, past, etc.. passing, past, etc.. redundant. rule 1. Pay = pay, paid, etc.. paying, paid, etc.. redundant. rule 8. Pen = pen, pent, etc., penning, pent, etc., redundant. rule 3. Put = put, put. putting, put. complete. rule 3. Quit = quit, quit, etc.. quitting, quit, etc., redundant. rule 3. Eead = read, read. reading, read. complete. rule 4. Rid = rid, rid, ridding, rid. complete. rule 3. ^For an explanation of "Indicative," "Participle," "Imperfect," and "Per- fect, "look through " The System Method," No. 6, Dems. VIII.-XIIL, inclusive ; and see, especially, Dem. VII., k. 220 TEE SYSTEM METHOD. Prog. XXI :/. Composii'n. 2. Derivat'n . 3. Accent 4. Pni. Accent a. Simple. b. Compound. a. Primitive. b. Derivative. a. Primary. b. Pri. and Sec. a. Ultimate. b. Penult. c. Antepenult. d. Preantepenult. Enjoyment = simple. derivative. primary. penult. Oatmeal = compound. primitive. pri. and sec. penult. Providence = simple. derivative, primary. antepenult. Makeshift = compound. primitive. pri. and sec, penult. Tyro = simple, primitive, pri. and sec. penult. Engineer = simple. derivative. pri. and sec. ultimate. Accident = simple. primitive, primary. antepenult. Silence = simple. primitive. primary. penult. Accidental = simple. derivative. pri. and sec. penult. Lemonade = simple. derivative. pri! and sec, ultimate. Substantive = simple. primitive, primary. antepenult. Eye-ball = compound. primitive. pri. and sec. penult. Characteristic ; = simple, derivative. pri. and sec. penult. Inquiry = simple. derivative, primary. penult. Uprising^ = compound. derivative. primary. penult. Without = compound. primitive. primary. ultimate. Destroy = simple. primitive. primary, ultimate. Disputable = simple. derivative. primary. preantepenult. Thereabout = compound. primitive, pri. and sec. ultimate. Talent = simple. primitive. primary. penult. Melon = simple. primitive. primary. penult. Antepenult = simple, derivative. pri. and sec. ultimate. Turncoat = compound, primitive. pri. and sec. penult. Upon = : compound, primitive. primary, ultimate. Legislature = : simple. derivative. pri. and sec. preantepenult. Fatal = : sim]3le. derivative. primary, penult. Mary = : simple, primitive. primary. penult. Artificer^ = : simple, derivative. primary. antepenult. Primarily = : simple, derivative. primary. preantepenult. Artificial = : simple. derivative. pri. and sec, , penult. Taking = : simple. derivative. primary. penult. Parliamentary= : simple. derivative, pri. and sec, antepenult. 1 By calling uprising compound is meant that it contains two root words, up and rise, separately used, or current, in English. But artificer is to be called simple, be- cause, while also containing two root words (Latin ars and /aeio), those roots are not separately used in English. And so primitive and derivative, as well as simple and compound, are to be used as meaning primitive, simple, etc, in English, not in any older lano-uage. ST8TEM OBTHOGBAPEY— GRADE A. 221 Prog. XXII: /. Syllabicaiion and 2. Phonic Diacritical Spelling. Spelling. Dividing = di-vid-ing, Financial = fi-nan-cial. Pamphlet = pam-phlet, Acre = a-ere. Lion = li-^on. Spoken = spok-en. Philosophizic ig= phMos-o-phiz Civilized = civ-il-ize, Breaths = breaths. Councils = €0un-9il§, Coffin = €of-fin. Vial = vi-^al, Cynic = 9yn-ie, Turning = turn-ing. Beckoned = beck-on-ed, Upon = iip-6n, Exist = ei-ist, Measured = meas-ur-ed. Fatality = fa-tal-i-ty. Disarming = di§-arm-ing, Weevil = wee-v^l, Amendment = a-mend-ment. Horseman = horse-man. Theorizing = the-o-riz-ing. Baths = batii§, Diseased = di§-e^a§-ed. Seminary = sem-i-n%-ry. Enable = en-a-ble. Transport = trans-port. Bridal = bri-dal, Pianos = pi-an-o§, 3. Rules. a. For syllabication. b. For pronunciation. c. For spelling. *1:I:5. di-vi-ding, £i-nan-shul, pam-flet, a-kr, li-iin, spo-kn, fi-16s-o-fi-zing, siv-il-iz, breths, koun-silz, kof-fin, vi-al, sin-ik, ttim-mg, bek nd, iip-6n, eg-zist, mezh-oord, fa-tal-i-ti, diz-arm-ing, we-vl, a-mend-ment, hors-man, the-o-ri-zing, biithz, diz-ezd, sem-i-ner-i, en-a-bl, trans-port, bri-diil, p!-an-oz, ^ In Webster's notation, which is here used, a and o obscure are left unmarked. They are usually pronounced hke indistinct u; as, ]I-on:=li-un; vi-al=vi-iil, etc. 'In ea (in diseased), as in every vowel digraph, one vowel is to be regarded as principal (e here in ea), and the other as modifier (a here in ea). It is often impos- sible to determine just what power, sound, or influence, the modifier has as related to principal ; still, the modifier is never to be regarded as really silent. ^In seminary, dictionary, etc., however, a is equivalent to e. See Rule VI. for pronunciation. * J^ote. — The Arabic characters on the left refer to rules for syllabication, the Roman numbers to the rules for pronunciation, and the right hand Arabics to rules for spelling. 4, 2: VII: 5. 2: VII. 1:1. 1, 2:1, II. 1, 3: XI, 1:5. 1, 3, V, VII, IV, 1:5. 1, 3: VII. :10. XII: 10. 2: XII. 1:1, II. 2: VII. 2: :4. 2: VII, XIV. 2: VII. 2: VII. :5. 1, 2:V, VII. 2:Vn, XVII. 1:1, ex. to XII. 2:111, VII. :6. 1, 3:1, II, IV: 5. XVIII: 10. 2:XVm:5. 2:V, VI, VII. 2, 1:1, VII. vni. 1:1:5. 2:V:ex. to 11. 222 THE SYSTEM METHOD. Prog. XXIII : /. Syllabication and 2. Phonic Diacriiical Spelling. Spelling. 3. Rules. Nobilities Institution Ideas Mulish. Chinese Secretary Albumen Indivisible Basin Eulable Cousin Locomotion Duties Chosen Auxiliary Hindoos Acclimated Engagement Pitiable Delayed Boyish Fleeing Huzzaed Gaining Mumps Lucases Waxing Worshiped Motleys Mansion Canceled Aristotles Awful Women Miscellaneous Aorta Carried y)issolving = no-bil-i-ties, = in-sti-tu-tion, = i-de-as, = mtJl-ish, = Chi-nege, == see-re-ta-ry, = al-bu-men, = in-di-vi§-i-ble, = ba-szn, = rul-a-ble, = eoug-en, = lo-eo-mo-tion, = du-tie§, = cho§-en, = au3j-il-i-a-ry, = Hm-doo§, = ae-eli-mat-ed, = en-gag6-ment, = pit-i-a-ble, = de-lay-ed, = boy-ish, = flee-ing, = huz-za-ed, = gain-ing, = mumps, = Lu-eas-e§, = wax-ing, = wor-ship-ed, = M6t-ley§, = man-sion, = Gan-9el-ed, = Ar-is-tot-le, = aw-ful, = wo-men, = mis-9el-la-ne-oiis, = a-6r-ta, = €ar-ri-ed, = di§-§61v-ing, no-bil-i-tiz, in-sti-tu-shiin, i-de-az, mu-lish, Chi-nez, sek-re-ter-i, al-bu-men, in-di-viz-I-bl, ba-sn, rul-a-bl, kuz-n, lo-ko-mo-shiin du-tiz, cho-zn, ag-zil-i-a-ri, Hin-dooz, ak-kll-ma-ted, en-gaj-ment, pit-i-a-bl, de-lad, boi-ish, fle-ing, hob-zad, ga-ning, mumps, Lu-kas-ez, wak-sing, wur-shipt, Mot-liz, man-shun, kan-seld, Ar-is-tot-l, a-fool, wim-en, mis-sel-la-ne-iis, a-6r-ta, kar-rid, diz-zol-ving. a. For syllabication. b. Tor pronunciation. c. For spelling. n, 2:1, V, VII: 7, 11. 3, 1:V, I, VII, XVI-.5. 1:L 111:10. 1, 3, 3.1:5. 1:L 3, 1:IV, VII, V, VL 3, 1:1 3:V. 1:1, ex. toXIL III, XV: 5. 3: VII, XIL 1:1, IV, XVL 1, 3:1:7, 11. 1, 3:1, XI:5. 2: VII, V, IIL 3: VII: 10. 3, 1:1, 111:5. 3: :6. 3:111, V, VII: 7. 1:1:8. :8. 1:1:5. 3. 3. 1:1:11. 3:VIL 3:VII:4. 3: VII: 8, 10. 3: VII, XVL 3:Vn, IX: 4 3:VIL : ex. to 6. 3: VII, X. 3, 1:L IV, IX:ex. to8. 1:L IIL 3:V:8. 3: VII: 5. ^ See note on previous page. n Final Letters and Hyphen Branches. Copyright, 1886, by I. E. WILSON. [223J 224 THE SYSTEM METHOD. Prog. XXIT.— Eules for Spelling by Work-Tree. Maimed = consonant doubled, none after digraph, 4. Taming = vowel dropped, but e before vowel suflBx, 5. Bandied = vowel changed, y to i, but before others, 7. Diagramm'd = consonant doubled, between single v' and vowel suffix, 3. Canceled = consonant doubled, none after unaccented vowel, 4. Assigned = consonant doubled, none after consonant, 4. Mill = consonant doubled, f, 1, s, after single v' in monosyllables, 1. Grass = consonant doubled, f, 1, s, after single v' in monosyllables, 1= Summed = consonant doubled, between single v' and vowel suffix, 2. Compelling = consonant doubled, between single v' and vowel suffix, 2. Infer = consonant doubled, none except f, 1, s, in monosyllables, 3. Kecoiling := consonant doubled, none after digraph, 4. Moralize = consonant doubled, none after unaccented vowel, 4. Tab = consonant doubled, none except f, 1, s, in monosyllables, 3. Solidify = consonant doubled, none after unaccented vowel, 4. Affixing^ = consonant doubled, between single v' and vowel suffix, 2. Yowel = consonant doubled, none except f, 1, s, in monosyllables, 3. Alas = consonant doubled, none except f, 1, s, in monosyilaoles, 3. Brass = consonant doubled, f, 1, s, after single v' in monosyllables, 1. Shriveled = consonant doubled, none after unaccented vowel, 4. Responded = consonant doubled, none after consonant, 4. Yes, = consonant doubled, f, 1, s, after single v' in mon., ex. to 1. Desisted = consonant doubled, none after consonant, 4. Oddly = consonant doubled, none except f, 1, s, in mon., ex. to 3. If = consonant doubled, f, 1, s, after single v' in mon., ex. to 1. Is = consonant doubled, f, 1, s, after single v' in mon., ex. to 1. Extolling = consonant doubled, between single v' and vowel suffix, 2. Signalize = consonant doubled, none after unaccented vowel, 4. Whetted = consonant doubled, between single v' and vowel suffix, 2. Howled = consonant doubled, none after diphthong, 4. Joining = consonant doubled, none after diphthong, 4. Lifeless = vowel dropped, not e before consonant suffix, 6. Fancied = vowel changed, y to i, but before others, 7. Allotting = consonant doubled, between single v' and vowel suffix, 2. Sensible = vowel dropped, but e before vowel suffix, 5. Giddiness = vowel changed, y to i, but before others, 7. Lodgement = vowel dropped, not e before consonant suffix, 6. Daily = vowel changed, y to i, but before others, 7. Stoppage = consonant doubled, between single v' and vowel suffix, 2. 1 The reason x is never doubled (for it is not doubled, even when It is a terminal consonant preceded by a single accented vowel, as in affixing), is that in itself it represents a double, or k-s, sound. SYSTEM OBTHOGRAPHY— GRADE A. 225 Prog. XXY.— Hyphen, Consonant, and Towel by the Work-Tree. Saved = final, vowel dropped, but e before vowel suffix, 5. Maries = final, plural sign, es to nouns not coalescing with s, 11. Maries = final, vowel cbanged, y to i, but before others, 7. Maloys := final, plural sign, s only to nouns coalescing with s, 10. Maloys = final, vowel changed, y to i, not after a vowel, 8. Married = final, vowel changed, y to i, but before others, 7. Gayety = final, vowel changed, y to i, not after a vowel, 8. Merrier = final, vowel changed, y to i, but before others, 7. Fore-ordain^== hyphen, separates prefix, vowel from vowel, 16. Sing-song = hyphen, unites words contrasted in accent and conjoined in thought, 15. Glorious = final, vowel changed, y to i, but before others, 7. Hoeing = final, vowel dropped, but e before vowel suffix, excep. to 5o Crying = final, vowel changed, y to i, not before 1, 7. Juliuses = final, plural sign, es to nouns not coalescing with s, 11. Baileys = final, plural sign, s only to nouns coalescing with s, 10. Baileys = final, vowel changed, y to i, not after a vowel, 8. Tying = final, vowel changed, ie to y before ing, 9. Hiss = final, consonant doubled, f, 1, s, after single v' in mon., 1. Keviled = final, vowel dropped, but e before vowel suffix, 5. Planner = final, con. doubled, between single v' and vowel suffix, 2. Iron-clad = hyphen, unites words contrasted in accent and conjoined in thought, 15. Revealed = final, consonant doubled, not after digraph, 4. Thus = final, con. doubled, f, 1, s, after single v' in mon., ex. to 1. Reveling = final, consonant doubled, none after unaccented vowel, 4. Us = final, con. doubled, f, 1, s, after single v' in mon., ex. to 1. Babyish = final, vowel changed, y to i, not before i, 7. Bluish = final, vowel dropped, but e before vowel suffix, 5. Glueyness = final, vowel changed, y to i, not after a vowel, 8. Harries = final, plural sign, es to nouns not coalescing with s, 11. Grays = final, plural sign, s only to nouns coalescing with s, 10. Grays = final, vowel changed, y to i, not after a vowel, 8. Motleys = final, plural sign, s only to nouns coalescing with s, 10. Baggage = final, con. doubled, between single v' and vowel suffix, 3. Enjoyed = final, vowel changed, y to i, not after a vowel, 8. Died = final, vowel dropped, but e before vowel suffix, 5. Toed = final, vowel dropped, but e before vowel suffix, 5. Decreeing = final, vowel dropped, but e before vowel suffix, excep. to 5. Jew's-harp^ = hyphen, unites words contrasted in accent and conjoined in thought, 15. 1 See note under Rule XV., Appendix C, page 198. 15 226 THE SYSTEM METHOD. Prog. XXYI. — Plural and Possessive Signs by the Work-Tree. Foxes = plural sign, es to nouns not coalescing with s, 11. Xenophons = plural sign, s only to nouns coalescing with s, 10. Jameses = plural sign, es to nouns not coalescing with s, 11. Maries = plural sign, es to nouns not coalescing with s, 11. Statues = plural sign, s only to nouns coalescing with s, 10. Whiskeys = plural sign, s only to nouns coalescing with s, 10. Waters = plural sign, s only to nouns coalescing with s, 10. Chaunceys = plural sign, s only to nouns coalescing with s, 10. Hindoos = plural sign, s only to nouns coalescing with s, 10. Quincies = plural sound, es to nouns not coalescing with s, 11. Mays = plural sign, s only to nouns coalescing with s, 10. Dickens's = poss. sign, 's added to each part in separate possess- ives, 13 a. Bowels = plural sign, s only to nouns coalescing with s, 10. Rosalies = plural sign, s only to nouns coalescing with s, 10. Justices' = poss. sign, (') only to plurals ending in s, 14 b. Interior's := poss. sign, 's added to last part in compounds, 13 d, Patrick Henries = plural sign, es to nouns not coalescing with s, 11. Mosbies = plural sign, es to nouns not coalescing with s, 11. Palfreys = plural sign, s only to nouns coalescing with s, 10. Toes = plural sign, s only to nouns coalescing with s, 10. Maid-servants = plural sign, s only to nouns coalescing with s, 10. Mileses = plural sign, es to nouns not coalescing with s, 11. Jellies =: plural sign, es to nouns not coalescing with s, 11. Christmases = plural sign, es to nouns not coalescing with s, 11. Antonies = plural sign, es to nouns not coalescing with s, 11, Mice's = poss. sign, 's to each part in separate possessives, 13 c. Races = plural sign, es to nouns not coalescing with s, 11. King James's = poss. sign, 's added to last part in compounds, 13 d. Humphreys = plural sign, s only to nouns coalescing with s, 10. Peter's = poss. sign, 's to each part in separate possessives, 13 a. Brothers-in-law = plural sign, s only to nouns coalescing with s, 10. Girls' = poss. sign, (') only added to plurals ending in s, 14 b. Runaways = plural sign, s only to nouns coalescing with s, 10. Reed and Kellogg's = poss. sign, 's added to lastpart in joint possessives, 13 e. Turnkeys = plural sign, s only to nouns coalescing with s, 10. Son-in-law's = poss. sign, 's added to last part in compounds, 13 d. Mottoes = plural sign, es to nouns in o after a consonant, 11. Conscience' = poss. sign, (') only added to abstracts having no jdIu- ral, 14 a. SYSTEM OBTHOORAPY— GRADE A. 227 Prog.XXTlI. Kough-liew Lieut. -Governor's Lieut. -Governor's Catalogued Day and Martin's Puff Gearing Agreeing Drier Vied Well-laid-out Flutter Streets Re-enlist Hoeing Judgement Stiff Suing Re-invest Moss Hall Outrageous : Lass Forcible Hoed Manageable Repulsive Convalescent Brigadier Harris's : Gyration : Rued Sun-bird Handkerchiefs = Ing9.thering : Yes Proofs Lancing Rebellion = -Final Letters and Hyphen by the Work-Tree. = hyphen, unites words contrasted in accent and con- joined in thought, 15. = final, poss. sign, 's added to last part in compounds, 13 d. = hyphen, unites words contrasted in accent, etc., 15. = final, vow. dropped, but e before vowel sutfix, 5. = final, poss. sign, 's added to last part in joint poss., 13 e. = final, con. doubled, f, 1, s, after single v' in mon., 1. = final, con. doubled, none after digraph. 4. = final, vow. dropped, but e before vowel suffix, ex. to 5. = final, vow. changed, y to i, but before others, 7. = final, vow. dropped, but e before vowel suffix, 5. : hyphen, unites words contrasted in accent, etc., 15. = final, con. doubled, between single v' and vowel suffix, 2. = final, plu. sign, s only to nouns coalescing with s, 10. = hyphen, separates prefix, sometimes re but not others, 17. : final, vow. dropped, but e before vowel suffix, ex. to 5. = final, vow. dropped, not e before consonant suffix, 6. = final, con. doubled, f, 1, s, after single v' in mon., 1. = final, voWo dropped, but e before vowel suffix, 5. = hyphen, separates jirefix, sometimes re but not others, 17. = final, con. doubled, f, 1, s, after single v' in mon., 1. = final, con. doubled, f, 1, s, after single v' in mon., 1. : final, vow. dropped, but e before vowel suffix, excep. to 5. = final, con. doubled, f, 1, s, after single v' in mon. 1. = final, vow. dropped, but e before vowel suffix, 5. = final, vow. dropped, but e before vowel suffix, 5. = final, vow. dropped, but e before vowel suffix, excep. to 5. = final, vow. dropj)ed, but e before vowel suffix, 5. = final, vow. dropped, but e before vowel suffix, 5. = final, vow. dropjDed, but e before vowel suffix, 5. = final, poss. sign, 's added to each part in sep. pos., 13 a. : final, vow. dropped, but e before vowel suffix, 5. = final, vow. dropped, but e before vowel suffix, 5. :hyj)ben, unites words contrasted in accent and con- joined in thought, 15. : final, plu. sign, s only to nouns coalescing with s, 10. : hyphen, separates prefix, sometimes re but not others, 17. : final, con. doubled, f, 1, s, after single v' in monosylla- bles, excep. to 1. -- final, plu. sign, s only to nouns coalescing with s, 10. = final, vow. dropped, but e before vowel suffix, 5. : final, con. doubled, between single v' and vowel suffix, 3. [228] IDEOGRA-PHIO ^?VORK:-TRKEJ. CAPITAL LETTER BRANCH. Copyright, 1886, by I. E. WILSON. SYSTEM ORTHOGBAPET— GRADE A. 229 Prog. XXYIII.- Chicago = proper, North = special West = special New York = proper, The = first of, North = special South = special I = special Lord's = special He = special We = first of. Charles = proper. Fifth's = title. Oh = first of. He = special One = special Brother = term, Grays = proper. Father = special Matthew = proper. Englished = proper. American = proper. Eternal = special Justice = special Brother = term. Albert = proper. Governor = title. Sherman = proper. Son = special Man = special Lord = special Sabbath = special Sabbath Wednesday- July New England = special = special = special proper, -Capital Letters by the Work-Tree. noun, verb, or adjective, ^1. word, four points, direction (s), 14. word, four points, direction (s), 14. noun, verb, or adjective, 1. sentence, 17. word, four points, section, 9. word, four points, section, 9. word, I and O, 6. word, put for God or Christ, 7 word, pronoun of Deity, with antecedent (s) 16. direct quotation or well-known saying, 18. noun, verb, or adjective, 1. epithet added, 24. sentence, 17. word, pronoun of Deity, with no antecedent, 13. word, put for God or Christ, 7. noun prefixed as part of name, 22. noun, verb, or adjective, 1. word, Bible "Father" (c), but "oh" and "sab- bath" (s), 10, 15. noun, verb, or adjective, 1. noun, verb, or adjective, 1. noun, verb, or adjective, 1. word, put for God or Christ, 7. word, put for God or Christ, 7. noun prefixed as not part of name (s), 25= noun, verb, or adjective, 1. noun prefixed as part of name, 22, noun, verb, or adjective, 1. word, name-part separated by " of," 8. word, name-part separated by " of," 8. word, put for God or Christ, 7. word, Bible "Father "(c), but "oh" and "sab- bath" (s), 10, 15. word, Bible "Father" (c), but "oh "and "sab- bath" (s), 10, 15 word, seven days and twelve months, 11. word, seven days and twelve months, 11. two adjectives, two nouns, or one of each, 3. ^ The Arabic numbers refer to the rules for the use of capital letters, Appendix G, page 239. The learner should write entirely out the branch-top ends, since only by doing so -will he acquire an organized knowledge of the capital-letter system, and reach a practical mastery over this sort of language problems. 230 THE SYSTEM METHOD. Prog. XXIX.— Leo = proper, Nepos = proper. East = special Babylon = proper. Cyrus = proper, Persia = proper. Prince = title, Albert = proper, Sahara = proper. Great =: proper, Desert = proper. North = special Central = proper, Africa = proper, Aryan = special Semitic = special Hamitic = special High = special Court = special Parliament = special Uncle = term, Nathan = proper. Fifteenth = special Amendment = special United = proper, States = proper. Washington = proper. Federalist = special Universal = special Being = special English = proper, Shakspearized= proper, Blue := special Ribbon = special Movement = special Esquimaux = special North = special Most = special High = special Son = special Man = special Capital Letters by the Work-Tree. noun, verb, or adjective, 1. noun, verb, or adjective, 1. vyord, four points, section, 9. noun, verb, or adjective, 1. noun, verb, or adjective, 1. noun, verb, or adjective, 1. noun prefixed as part of name, 22. noun, verb, or adjective, 1. noun, verb, or adjective, 1. tw^o adjectives, two nouns, or one of each, 3. tvfo adjectives, two nouns, or one of each, 3. word, four points, direction (s), 14. two adjectives, two nouns, or one of each, 3 two adjectives, two nouns, or one of each, 3, word, nations, tribes, sects, organizations, 13. word, nations, tribes, sects, organizations, 13. word, nations, tribes, sects, organizations, 18. word, nations, tribes, sects, organizations, 13. word, nations, tribes, sects, organizations, 13. word, nations, tribes, sects, organizations, 13. noun prefixed as part of name, 23. noun, verb, or adjective, 1. word, nations, tribes, etc., national enactments, 13. word, nations, tribes, etc., national enactments, 13. two adjectives, two nouns, or one of each, 3. two adjectives, two nouns, or one of each, 3, noun, verb, or adjective, 1. word, nations, sects, organizations, etc., 13. word, put for God or Christ, 7. word, put for God or Christ, 7. noun, verb, or adjective, 1. noun, verb, or adjective, 1. word, nations, tribes, sects, organizations, 13. word, nations, tribes, sects, organizations, 13. word, nations, tribes, sects, organizations, 13. word, nations, tribes, sects, organizations, 18. word, four points, direction (s), 10, 15. word, put for God or Christ, 7. word, put for God or Christ, 7. word, name-parts separated by "of," 8. word, name-parts separated by "of," 8. SYSTEM ORTHOOBAPHY—GRADE A. 231 Prog. XXX. White = proper. Mountains = proper. New = proper. Hampsliire = proper, How = first of. Death = proper. Death = first of, Brother = term. Sleep = proper. Alexander = proper. Great = title. Macedonia = proper. Henry = proper. Fourth = title. Alps = proper. January- = special ' Providence = special Sabbath = special ■ (s), Charles = proper. Bold = title. Burgundy = proper. Nancy = proper. Heaven = special Brother = term, Timothy = proper. Madam = title. De Stael — proper, Night = proper. Chaos = proper. New York = proper. East = special India = special Company = special Father = special Waters = special Lake = term. Itasca = proper. The = first of. Use = proper. And = first of, Health = proper. —Capital Letters by the Work-Tree. two adjectives, two nouns, or one of each, 3. two adjectives, two nouns, or one of each, 3. two adjectives, two nouns, or one of each, 3. two adjectives, two nouns, or one of each, 3. printed line of poetry; 20. by personification, 2. printed line of poetry (or 2), 20. noun prefixed as not part of name (s), 25. by personification, 2. noun, verb, or adjective, 1. epithet added, 24. noun, verb, or adjective, 1. noun, verb, or adjective, 1. epithet added, 24. noun, verb, or adjective, 1. word, seven days and twelve months, 11. word, put for God or Christ, 7- word, Bible " Father" (c), but "oh" and " sabbath" , 10, 15. noun, verb, or adjective, 1. epithet added, 24. noun, verb, or adjective, 1. noun, verb, or adjective, 1. word, put for God or Christ, 7. noun prefixed as part of name, 22. noun, verb, or adjective, 1. noun prefixed as part of name, 22. noun, verb, or adjective, 1. by personification, 2. by personification, 2. noun, verb, or adjective, 1. word, nations, tribes, organizations, etc., 13. word, nations, tribes, organizations, etc., 13. word, nations, tribes, organizations, etc., 13. word, name-parts separated by " of," 8. word, name-parts separated by "of," 8. noun prefixed as part of name, 22. noun, verb, or adjective, 1. printed line of poetry, 20. by personification, 2. printed line of poetry, 20. by personification, 2. 232 TEE SYSTEM METEOD. Prog. XXXI. — Final Letters and the Hyphen by the Work-Tree. Pre-occupy = hyphen, separates prefix, vowel from vowel, 16. Co-action = hyj^hen, separates prefix, vowel from vowel, 16; Navigation = final, vowel dropped, hut e before vowel suflBx, 5. Mindanaos = final, plu. sign, s only to nouns coalescing with s, 10. Pursuing^ = final, vowel dropped, hut e hefore vowel suffix, 5. Definition^ = final, vowel dropped, hut e hefore vowel suffix, 5. Digging = final, con. douhled hetween sing, v' and vowel suffix, 3. Co-eval = hyphen, separates prefix, vowel from vowel, 16, Peleus's = final, poss. sign, 's added to each part, in sep. poss., 13 a. Women's = final, poss. sign, 's added to each part, in sep. poss., 13, c. Turkeys* = final, vowel changed, y to i, not after a vowel, 8. Congratulatory= final, vowel dropped, hut e hefore vowel suffix, 5. Ingathering = hyphen, separates prefix, sometimes re hut not others, 17. Mohhed = final, con. doubled, between sing, v' and vowel suffix, 2. Look-out = hyphen, unites words contrasted in accent, etc., 15. Manned = final, cons, doubled, between sing, v' and vowel suffix, 2. Hooted = final, cons, doubled, none after digraph or diphthong, 4. Ox-cart = hyphen, unites words contrasted in accent, etc., 15. Mottoes = final, plu. sign, es to nouns in o after a consonant, 11. Druggist = final, cons, doubled, between sing, v' and vowel suffix, 2. Earthiness = final, vowel changed, y to i, but before others, 7. Hardihood = final, vowel changed, y to i, but before others, 7. Boy-king = hyphen, unites words contrasted in accent, etc., 15. Elizabeth's = final, poss. sign, 's added to each part in separate poss., 13 a. Player = final, vowel changed, y to i, not after a vowel, 8. Embargoes = final, plu. sign, es to nouns in o after a consonant, 11. Fogginess = final, vowel changed, y to i, hut before others, 7. Start-up = hyphen, unites words contrasted in accent, etc., 15. Obligatory = final, vowel dropped, but e before vowel suffix, 5. Pre-establish = hyphen, separates prefix, vowel from vowel, 16. Send-off = hyphen, unites words contrasted in accent, etc., 15. Tributary = final, vowel dropped, but e before vowel suffix, 5. Uruguays = final, vowel changed, y to i, not after a vowel, 8. Son-in-law = hyphen, unites words contrasted in accent, etc., 15. Waterloos = final, plu. sign, s only to nouns in o after a vowel, 10. Designative := final, vowel dropped, but e before vowel suffix, 5. Miss Days = final, vowel changed, y to i, not after a vowel, 8. Victoria's = poss. sign, 's added to each part in separate poss., 15 a. ' The terminal e in pursue is not to be regarded as silent. 2 Definition = define -|- ite -\- ion. See Appendix F to " The System Method," No. 18. ^ There are involved in turkeys two rules, one for the plural signs, the other for the change of the vowel y. In such cases, the student may pass by the less important. SYSTEM ORTHOOBAPHT—OBADE A. 233 Prog. XXXII.— Final Letters and the Hyphen (Continued). final, vowel changed, y to i, not before i, 7. final, vowel changed, y to i, but before others, 7. hyphen, separates prefix, vowel from vowel, 16. final, vowel dropped, but e before vowel suffix, 5. final, poss. sign, (') only added to plurals ending in s, 14 b. hyphen, separates prefix, sometimes re but not others, 17. final, vowel dropped, but e before a vowel suffix, 5. final, vowel changed, y to 1, not after a vowel, 8. final, vowel dropped, but e before vowel suffix, 5. final, vowel changed, y to i, but before others, 7. final, vowel dropped, but e before vowel suffix, 5. hyphen, unites words contrasted in accent, etc., 15. hyphen, unites words contrasted in accent, etc., 15. final, 's added to each part in separate possessives, 13 a. final, 's added to each part in separate possessives, 13 a. hyphen, unites words contrasted in accent, etc., 15. hyphen, unites words contrasted in accent, etc., 15. final, cons, doubled, between single v' and vowel suffix, 3. final, vowel dropped, but e before a vowel suffix, 5. final, vowel dropped, not e in ce or ge before a or o, ex. to 5. hyphen, separates prefix, vowel from vowel, 16. final, vowel dropped, but e before a vowel suffix, 5. final, poss. sign, (') only added to abstracts having no plural, 14 a. final, vowel dropped, not e in ce or ge before a or o, ex. to 5. final, vowel changed, y to i, but before others, 7. final, vowel changed, y to i, but before others, 7. final, vowel changed, y to i, not after a vowel, 8. final, cons, doubled, none after an unaccented vowel, 4. hyphen, unites words contrasted in accent, etc., 15. final, consonant doubled, none after a consonant, 4. final, vowel dropped, but e before vowel suffix, 5. final, vowel dropped, but e before vowel suffix, 5. final, cons, doubled, none after an unaccented vowel, 4. hyphen, unites words contrasted in accent, etc., 15. final, vowel changed, y to i, not after a vowel, 8. final, vowel dropped, but e before a vowel suffix, 5. hyphen, separates prefix, sometimes re but not others, 17. final, cons, doubled, none after an unaccented vowel, 4. final, cons, doubled, none after an unaccented vowel, 4. final, vowel dropped, but e before a vowel suffix, 5 Trying = Jellies = Co-operation = Demonstrative = Angels' = Ke-collect = Opposition = Delays = Devotion = Accompaniment = Financier = Well-to-do = To-day = Henkle's = Lazarus's = Puppet-prince = Man-loving = Demurring = Plumage = Mortgageor = Pre-exist = Gluing = Righteousness' = Noticeable = Driest = Scantiness = Chaunceys = Legalize = Ten-foot = Expected = Moderator = Revision = Modeled Heart-broken Played Catalogued Re-absorb Tranquilize Worshiped Providence 234 THE SYSTEM METHOD. Prog. XXXIII: /. Composit'n. 2 .Derivat'n. 3. Origin of Root 4. Affix. a. Simple. b. Compound a. Primitive. . b. Derivative. a. Greek. b. Latin. c. Anglo-Saxon. a. Greek. b. Latin. c. Anglo-Saxon. A-|-shore = simple. derivative. Anglo-Saxon, Anglo-Saxon. For+give = simple, derivative. Anglo-Saxon, Anglo-Saxon. Drunk+ard = simple. derivative. Anglo-Saxon, Anglo-Saxon. Dis-|-join = simple, derivative. Latin, Latin. Wiz-j-ard = simple. primitive. Anglo-Saxon, Anglo-Saxon. Be+dew = simple. derivative. Anglo-Saxon, Anglo-Saxon. Dis+please = simple. derivative. Latin, Latin. Black-|-eii = simple. derivative. Anglo-Saxon, Anglo-Saxon. En-|-trap = simple. derivative. Anglo-Saxon, Anglo-Saxon. Be+Mend = simple, derivative. Anglo-Saxon, Anglo-Saxon. Teach.-|-able = simple, derivative. Anglo-Saxon, Latin. Un-j-school+ed = simple. derivative. Greek, Anglo-Saxon. Re+new+al = simple. derivative. Anglo-Saxon, Latin. Health+ful = simple. derivative. Anglo-Saxon, Anglo-Saxon. TJn+tauglit = simple. derivative. Anglo-Saxon, Anglo-Saxon. King+dom = simple. derivative. Anglo-Saxon, Anglo-Saxon. Beginn+eri = simple, derivative, Anglo-Saxon, Anglo-Saxon. Pay-fment = simple, derivative, Latin, Latin. Fore+tell = simple. derivative. Anglo-Saxon, Anglo-Saxon. Di-|-rect-)-or = simple. derivative. Latin, Latin. Pre+ judge = simple. derivative. Latin, Latin. E-|-ducH-at-)-ion = simple. derivative, Latin, Latin. Blu+ish = simple. derivative. Anglo-Saxon, Anglo-Saxon. Name+less = simple. derivative. Anglo-Saxon, Anglo-Saxon. Mis-]-take = simple. derivative. Anglo-Saxon, Anglo-Saxon. Re+plant = simple. derivative, Anglo-Saxon, Latin. Sun-|-day = compound, , primitive. Anglo-Saxon. Care+ful = simple. derivative. Anglo-Saxon, .Anglo-Saxon. Con-|-cur = simple. primitive. Latin, Latin. Pre+vent+ive = simple, derivative. Latin, Latin. Super-|-natur-t-al = simple, derivative. Latin, Latin. Intro-)-duce = simple, primitive. Latin, Latin. Eagl+et = simple. derivative. Latin, Latin. Clos+et = simple, derivative. Latin, Latin. Man+ly = simple. derivative. Anglo-Saxon, Anglo-Saxon. Dark-f-some = simple, derivative. Anglo-Saxon, Anglo-Saxon. Blood+y = simple. derivative. Anglo-Saxon, Anglo-Saxon. Wild-|-er-|-ness = simple. derivative, Anglo-Saxon, Anglo-Saxon. Op+press-f-ive = simple, derivative. Latin, Latin. ^ The n of begin is doubled in accordance with Rule II. for spelling. SYSTEM ORTHOOBAPHY— GRADE A. 235 Prog. XXXIY: /. ComposH'n. 2. Derivat'n. 3. Origin of Root 4. Affix. a. Simple. b. Compound a. Primitive. .. b. Derivative. a. Greek. b. Latin. c. Anglo-Saxon. a. Greek. b. Latin. c. Anglo-Saxon. Se+greg+ate = simple, primitive. Latin, Latin. Para+clox = simple. primitive. Greek, ■ Greek. Con-|-flu+ence = simple. derivative. Latin, Latin. Ir-l-re+sol-|-ute = simple. derivative. Latin, Latin. Sapon-face+ous = simple. primitive. Latin, Latin. Sub+merge = simple. derivative. Latin, Latin. Suf+fer = simple. primitive. Latin, Latin. Peri+osteum = simple. primitive, Greek, Greek. Savag+er+y = simple. derivative. Latin, Latin. Per-|-cent+age = simple. derivative. Latin, Latin. Supra-}- orbit-|- al = simple. derivative, Latin, Latin. Syn+cliron-l-ous . = simple. primitive. Greek, Greek. Pro+gram = simple, derivative. Greek, Greek. Sy+stem = simple. primitive. Greek, Greek. Sym+path-l-y = simple. primitive, Greek, Greek. As-j-sist+ant = simple. derivative, Latin, Latin. Thef+t = simple. primitive, Anglo-Saxon, Anglo-Saxon. Tra-j-verse = simple. primitive. Latin, Latin. Dee+d = simple. primitive, Anglo-Saxon, Anglo-Saxon. Ad+versa-j-ry = simple. derivative, Latin, Latin. Ac-|-cesso-|-ry = simple. derivative. Latin, Latin. Pay+ed = simple. derivative. Latin, Latin. Hear+d = simple. primitive. Anglo-Saxon, Anglo-Saxon. Ab-j-ject = simple. primitive. Latin, Latin. Stai+di = simple. derivative. Latin, Anglo-Saxon. Thrif+t = simple. primitive. Anglo-Saxon, Anglo-Saxon. Ap-f-plaud = simple. primitive. Latin, Latin. Gold+en = simple. derivative. Anglo-Saxon, Anglo-Saxon, Leatber+n = simple. derivative. Anglo-Saxon, Anglo-Saxon. Heav-|-en = simple. primitive. Anglo-Saxon, Anglo-Saxon. Am-|-put-|-ate = simple. primitive. Latin, Latin. Cbatt+er = simple. derivative. Anglo-Saxon, Anglo-Saxon. Mole+cule = simple. derivative. Latin, Latin. Mo+st = simple. primitive. Anglo-Saxon, Anglo-Saxon, Man-j-bood = simple. derivative. Anglo-Saxon, Anglo-Saxon, Bi+ped = simple. primitive. Latin, Latin. Bis-f-cuit = simple, primitive, Latin, Latin. Kern-|-el = simple. primitive. Anglo-Saxon, Anglo-Saxon. Bow-|-el = simple. j)rimitive. Latin, Latin. Circum-|-vent, = simple. primitive. Latin, Latin. J The y of stay is changed to i contrary to Rule VIII. for spelling. Appendix D— Number, Geijder, aijd Case. (a.) Nature of Graiimatical Number. 1. Singular Number. — That which represents the object, part, or action named as not repeated; as, man, bellow, chair, checker, class, company, water, Milton, half, third, grief, people, heathen, seven, couple. 2. Plural Number. — That which represents the object, part, or action named as repeated; as, men, bellows, chairs, checkers, classes, companies, waters, Miltons, halves, thirds, griefs, peoples, heathens, sevens, couples. 3. Origin of Name, (a.) In Wholes: Names originating in wholes are now singular, now plural; as, boy, boys; belief, beliefs; woe, woes; spoon- ful, spoonfuls; hundred, hundreds; s, s's; getter-up, getters-up; army, armies; Kickapoo, Kickapoos; orgy, orgies. Lungs and bowels excepted, names originating in part or action are usually plural. The following names originated in part and action respectively: (b.) In i^art : Tongs, shears, scissors, pincers, tweezers, nippers, snuffers, trousers, pantaloons, drawers, duds, billiards, clothes, eaves, environs, withers, nuptials, news, annals, means, entrails, intestines, vitals, mathematics, ethics, pneumatics, magnetics, movables, goods, rickets, assets, nones, species, victuals, odds, riches, oats, embers, measles, ashes, suds, (c.) In action : Thanks, bellows, skimmings, goings, happenings, sweepings, trappings, tidings, doings, cal- ends, amends, blessings, searchings, earnings, feelings, proceeds, mumps, warblings, surgings, wages, (d.) In substance: Names of substance exhib- ited more than once are, of course, plural. Observe that the plural number indicates repetition, of whole or substance, part or particle, or action. Singulars of substance : gold, tea, coffee, snow, furniture, molasses, etc. Plurals of substance: golds, teas, coffees, snows, molasses, etc. (See "The System Method," No. 6, Dem. I., Obs. b, c, d.) (b.) How THE Singular and Plural are Distinguished, 1. By the regular s, es, or 's: valley-s, hoe-s, Guyot-s, Hadley-s, going-s, box-es, class-es, Mari-es, Mrs. Welbi-es, echo-es, Moses-es, 7's, lO's, t's, p's, etc. 2. By an irregular or changed voicel: man, men ; foot, feet ; goose, geese; louse, lice; mouse, mice; woman, women. Children and oxen are distinguished from their singulars by an added -en. 3. By arbitrary words : I, we; thou, you; he, they; she, they; it, they. 4. By twin-like words: sheep, sheep; deer, deer; who, who; which, which; what, what; that, that; as, as. [236] SYSTEM OBTHOGBAPHT—OBADE A. 237 (c.) How THE Masculine and Feminine aee Distinguished. 1. By suffixes: God, goddess; actor, actress; duke, duchess; heir, heir- ess; lion, lioness; testator, testatrix; hero, heroine; bride, bridegroom. 2. By prefixes: Male, female; he-goat, she-goat; man-servaut, maid- servant. 3. By arbitrary words: Brother, sister; nephew, niece; sir, madam; wizard, witch; he, she. (d.) Case. — The case-forms of any noun or pronoun may be made up from the following table: — Singular: — Nominative, I he who it child girl lady Mary James Possessive, my his whose its child's girl's lady's Mary's Jameses Objective, me him whom it child girl lady Mary James Plural: — Nominative, we they who they children girls ladies Maries Jameses Possessive, our their whose their children's girls' ladies' Maries' Jameses' Objective, us them whom them children girls ladies Maries Jameses Appendix E— Rules for Syllabication. 1. Bule I. — A vocal long usually has its vowel final ;i as, ty-ro, si-lence, Ma-ry, ba-con, po-et, sa-cred, be-ing, na-tion, ta-ble, li-on, tri-umph, la-i-ty, le-ver, e-vil, ab-do-men, bron-chi-tis, bi-as. 2. Bule II. — A vocal short usually has its vowel intermediate,'^ a, i, and y excepted; as, tal-ent, mel-6n, spir-it, cyn-ic, iip-on, ex-ist, beck-on, a-mend-ment, ac-id, doc-ile, dig-it, spe-cif-ic, reg-is-ter, ap-a-thy, me-di-um, la-i-ty, cru-el-ty, coiis-in, dev-il. 3. Bule III. — But, if Rules I. and II. disguise a root, remove^ the con- sonant to the preceding vowel; as, tak-ing, divid-ing, rather than ta-king and di-vi-ding. So spok-en, clasp-ing, ask-ing, delud-ing, trans-port, load-ed, print-ing, presum-ed, dis-arm. 4. Bule IV. — If neither Rule I., II., nor III. is violated thereby, join the consonant or consonants to the folloioing vowel; as, de-stroy, dis-tress, geo-graph-ic, lu-cre, sti-fle, min-strel, er-ror, lit-tle, ap-ple. 1 Final., as here used, means the last letter in the syllable, not necessarily the last letter in the word. ^ Intermediate.^ as here used, means simply »ioi j??iaZ in the syllable. ^ Let it be remembered always that the vowel., not a consonant, Is the body or base of the syllable, the consonants being mere additions to, or joints between, the vowels ; so that we speak, not of moving the vowels to the consonants, but of moving the consonants to the vowel, as ha&e of the syllable. Appendix F- Rules for Pronunciation. (a.) Final Vowels = those last in the syllable. 1. Rule I. — A single vowel final and accented=the vocal long; as, Da- vid, a-sy-lum, au-ro-ra, ab-do-men, bi-tu-men, bron-chi-tis, bu-reau, so-fa, pri-ma-ry, o-men, hy-phen, ta-ble, stro-iDhe, ty-ro, sa-cred, be-ing, na-tion. 3. Bule II. — A single vowel final before a vowel = the vocal long; as, cha-ot-ic, a-or-ta, beati-te-ous, pre-occupy, re-ech-o, co-op-er-ate, la-i-ty, re-al, ster-e-o-tyj)e, vi-al, po-et, vac-u-um, ge-ol-ogy, co-a-lesce, crti-el-ty, bi-as, in-cho-ate. 3. Bule III — A, final, unaccented, and not before a vowel=:a; as, a- gain, a-new, or-na-ment, pal-a-ta-ble, pri-ma-ry, so-fa, In-di-a. 4. Rule IV. — E, o, and u, final and unaccented=e, o, and u; as, ech-o, ac-cu-rate, par-tic-u-lar, e-vent, be-gone, blas-phe-my, the-o-ry, no-to-ri-e-ty, de-stroy, ty-ro, or-tho-dox-y, in-cho-ate, ac-ces-so-ry. 5. Bule V. — I and y, final and unaccented^! and y; as, di-vide, the- o-ry, phi-los-o-phy, fi-nance, in-ti-mate, det-ri-ment, a-crid-i-ty. 6. Rule VI. — A, under secondary accent in -ary=e; as, sec-ond-ary= ery; dic-tion-ary=ery; ad-vers-ary=ery. (b.) Intermediate VowELS=those followed by one or more conso- nants in same syllable. 7. Rule VII. — A single vowel intermediate and accented=the vocal short; as, pam-phlet, det-ri-ment, min-strel, ac-rid, ab-dom-i-nal, ac-ces- so-ry, al-16p-a-thy, an-nun-ci-ate, an-tip-o-des, ap-pren-tice. 8. Rule VIII. — A, intermediate with f, n, or s=a; as, draft, staff, waft, ant, answer, blanch, brass, grasp, vast, task, branch, slant, grass, class, entrance, fast, cast, mischance, France. 9. Rule IX. — E, intermediate with l=e; as, lev-el, can-eel, grav-el, flan-nel, ves-sel, coun-seL Exceptions : be-tel, chat-tel, driv-el, eas-el, grov-d, ha-zd, na-vd, rav-d, shek-d, shov-d, mus-sd, shriv-d, sniv-d, swin-gd, swiv-d, tea-sd, wea-sd. 10. Rule X. — E, intermediate with n, and preceded by 1, m, n, or r=e; as, wom-en, wool-en, o-men, Hel-en, lin-en, Al-len. 11. Rule XI. — E, intermediate with n and not preceded by 1, m, n, or r=e silent; as, heav-en, tak-en, oft-en, giv-en, eat-en, gar-den, gold-en, got-ten. Exceptions : as-pen, chick-en, glu-ten, hy-phen, kitch-en, lich-en, lin-den, mar-ten, mit-ten, pat-ten, row-en, sud-den, yew-en. [238J SYSTEM OBTHOQRAPHY—OBADE A. 239 13. Rule XII. — I, intermediate with 1 or n=i; as, Lat-in, satin, coun- cil, per-il, cof-fin. Exceptions: dev-zl, e-vzl, wee-v^l, ba-sm, cous-m, rai-s^n. 13. Rule XIII. — O, unaccented and before a terminal consonant=u; as, cus-tom=tum; meth-od=:ud; ran-cor:=cur; ran-som=:sum; pig-eon=: un; piv-ot=Tit. 14. Rule XIV. — O, intermediate with n and preceded by a sound k, s, or t=o silent; as, ba-con, beck-on, cot-ton, les-son, etc. 15. Rule XV. — U, intermediate with a sound r, sh, or zh=oo; as, rule, sure, meas-ure, az-ure. (c.) Special Syllables. 16. Rule XVI. — A terminal syllable sounded sh-n=shiin ; as, o-cean= o-shun; nation=na-shun. 17. Rule XVII. — -Dis^^dig in ten words only: dis-arm, dis-aster, dis- cern, disdain, dis-ease, dis-honesr, dis-honor, dis-mal, dis-own, dis-solve. 18. Rule XVIII. — Ths=th§ in eight words only: baths, paths, laths, clothes, oaths, moths, wreaths, mouths. Appendix G— Rules for The Use of Capitals. (a.) Of Proper Terms, Capital Initials Begin, — 1. Rtile I. — Pro]3er nouns, adjectives, and verbs. a. David fought with Qoliatli, the giant of Gatli. b. The passage was Englished by an American teacher. c. The planets are Mercury, Venus, Earth, Mars, Jupiter, Saturn, Uranus, and Neptune. 2. Rule II. — Nouns proper by jjersonification. a. Upon this. Fancy began to bestir herself. b. The richest of all lords is Use, And ruddy Health the loftiest muse. 3. Rule III. — Two adjectives, two nouns, or one adjective and one noun, together forming a proper name. a. The Michigan Central is a prosperous road. b. We stayed on Martha's Vineyard. c. The Wliite Mountains are in New Hampshire. d. The River St. Lawrence Is two thousand miles long. e. The people of South America are largely Roman Catholics. 24:0 THE SYSTEM METHOD. (b.) Of Chief Words, Capitals Begin, — 4. Rule TV. — The chief words in advertisements, synopses, etc. a. A Complete Graded Arithmetic, Oral and Written, on the Inductive Method of Instruction. For Schools and Colleges. b. Specimen Copies sent to Teachers and School Officers free. 5. Rule V. — The chief words in book titles, periodicals, ordinances, Bacred writings, etc. a. "Cray's Elegy in a Country Church-yard" is not excelled in English. b. The Students Work-Booh of System Orthography, for the use of Public Schools, Normal Schools, Teachers' Institutes, and Colleges. c. The New-England Journal of Education is conducted by Thos. M. Bicknell. d. The Two Witnesses are the Old and Neto Testaments. (c.) Of Special Words, Capitals Begin, — 6. Rule ri.—l and O. a. J am, and by me, child, thy body grows. b. With three steps / compass thy grave, thou who wast sd great before! 7. Rule VII. — All terms put for the true God or the true Christ. a. Of a truth it is that your God is a God of gods. b. He hoped to absorb himself in the One — the Infinite. c. I will show thee the bride, the Lamb's wife. d. Such place Eternal Justice had prepared. 8. Rule VIII. — Each of two nouns sej)arated by "of," and together forming one name. a. The Isthmus of Panama is wider than the Strait of Gibraltar^ b. His name was called the Word of Life. c. Next to the Most High is the Son of Man. d. The territory was granted to the Duke of York. e. The Greeks used the Era of the Olympiads; the Babylonians, the Era of Nahonassar . The Romans reckoned from. fiiQ Foundation of Rome. 9. Rule IX. — The words "north," "south," "east," and "west," when referring to territorial sections. a. The religion of the East and the philosophy of the West met. b. The North gave freedom to the slaves of the South. 10. Rule X. — The word "father," when it refers to saint or sage. a. The early Fathers were divided in opinion. b. The Fathers are not authority for scriptural doctrine^ 11. Rule XI. — The names of the seven days and the twelve months. a. Returning in the spring, he died on Thursday, the 13th ol April. b. On Thursday, the 22d of June, he signed his second abdication. SYSTEM OBTHOGBAPHY— GRADE A. 241 12. Bule XII. — Every pronoun referring to God or Christ but having no antecedent expressed in the same sentence. a. Oh! show me where is He, the high and holy One. b. Our doings are not unknown to Him. 13. Bule XIII. — Names of nations, tribes, races, sects, organized bodies, great eras, specific institutions, and national enactments. a. The English, hate the French as frivolous. b. The MoJiegans were an Algonquin tribe. c. The three historic races are the Aryan, the Semitic, and the Hamitic. d. The Baptists joined the Methodists. e. New York was settled by the East India Company. f. How proud is America of Washington and the Bevolution ! g. The Beformation began in 1517. h. The Fifteenth Amendment was adopted in 1870. (d.) Of Special "Words, Small Letters Begin, — 14. Bide XIV. — The words "north," "south," "east," and "west," when referring to direction merely. a. Chicago is icest of New York and east of Omaha. 15. Bule XV. — The words "oh" and "sabbath." a. They upbraided Jesus for plucking corn on the sahhath. b. What is a sabbath-day's journey? 16. Bxile XVI. — Every pronoun referring to God or Christ but having an antecedent expressed in the same sentence. a. I am the Lord's, and he is mine. b. Sing unto the Lord, sing praises to his name. (e.) Of First Words, Capitals Begin.— 17. Bule XVII. — The first of every distinct sentence. a. One of the evangelists says, "Jesus wept." b. The problems of human life are, TF/jere are we? and Where axe we going? 18. Bule XVIII. — The first of every direct quotation or well-known saying. a. The words of Emerson are, " The man may teach by doing, and not otherwise." b. The substance of the essay was, We learn through doing. 19. Bule XIX. — The first of every social resolution or political enact- ment. a. Resolved, That the mind is a real substance. b. Be it enacted. That all cattle found running loose, etc. 20. Bule XX— The first of every line of poetry. 16 242 THE SYSTEM METHOD. a. Sweet day, so calm, so cool, so bright, b. The bridal of the earth and sky, c. The dews shall weej) thy fall to-night, d. For thou must die. 21. Rule XXI. — The first of every separate section or paragraph. a. The 's is added, — (1.) To singulars having the same form for the plural. (3.) To the last part of proper names. b. The apostrophe (') only is added, — (1.) To abstract nouns that never take the plural. (2.) To plurals ending in s. (f.) Of Titles, Capitals Begin, — 22. Bule XXII. — Every title, or other term, which is a noun prefixed as part of the name. a. In Senator Blaine's book the author eulogizes the character of General Grant. b. The prophecy of Mother Shipton has failed. c. Colonel Robert E. Lee was a great soldier. d. "We were visiting at Brother Gray's. 23. Rule XXIII. — Every official title occurring in a formal society report. a. After the constitution was read, the following officers were elected: President, Martin Henry; Secretary, Oliver Tennis; Treasurer, William Dennison. b. The officers of this society shall be a President, a Secretary, a. Treasurer, and a Sergeant-at-Arms. 24. Rule XXIV. — An epithet added to confer distinction. a. Alexander the Great was king of Macedonia. b. And had James the Second no private virtues? 25. Rule XXV. — The titles (or terms) brother, mother, aunt, president, governor, king, queen, when not prefixed as part of the name, are begun. by small letters. a. The queen Cleopatra quickly fascinated Antony. b. Babylon was reduced by Cyrus, king of Persia. c. He resigned the East to his brother Valens. d. The senate sent a deputation to the Eastern em'peror Zeno. e. The consul Nero was busy watching Hannibal. ^^l^^^J^ j^ppendix H— Nomenclature of Orthography. 1. Organs of speech. — Those parts of the human system used in speaking. 2. Articulatort organs. — Those organs of speech used in joining the sounds used in speaking. They are the lips (labia), the teeth (dentes), hard palate (palatum), soft palate (uvula), nasal cavities, and the larynx. 3. Vocal chords. — Two chords extending across the larynx. They are flanked by thin half -moon-shaped membranes, and by their vibra- tion produce the voiced sounds. 4. Voice, or voiced sounds. — Any sound made by the vibration of the vocal chords; as, b and a in bat. 5. Whisper, or aspirate sound. — Any sound made without the vibration of the vocal chords; as, e and t in cat. 6. Vocal. — A voiced sound unob- structed by the articulatory organs; as, a and a in cat and cane. 7. Sub- vocal. — A voiced sound obstructed by the articulatory organs; as, b and g in beg. 8. Aspirate. — A whispered sound; as, e and t in cat. 9. Base. — The letter (or letters) of a syllable representing its vocal; as, e, a, and oi in met, cat, and boil. 10. Antecedent. — A letter preceding the base; as, m, c, and b in met, cat, and boil. 11. Consequent. — A letter following the base; as, t and 1 in cat and boil. 13. Abrupt, explodent, or mute. — A sound that explodes and cannot be continued; as, t and d in stand. 13. Continuant. — A sound that can be continued until the breath is exhausted; as, n and s in st&nd. 14. Vowel. — A letter representing a vocal sound; as, e, i, and y in set, sit, and fly. 15. Consonant. — A letter representing a subvocal or an aspirate sound; as, s, t, f, and 1 in sit and fly. 16. Aphthong, or silent letter. — A letter having no sound, but used to determine the signification of a word. 17. Digraph. — The union of two letters to rejoresent one sim- ple sound, vocal, subvocal, or aspirate; as, oa, th, and ti in boat, thin, and nation. 18. Diphthong. — The union of two vowels to represent a com- pound vocal sound; as, oi and ow in boil and fowl. 19. Simple vocal. — A single or elementary vocal; as, i and li in pin and tub. 20. Compound vo- cal. — A combination of two elementary vocal sounds; as, oi, ou, i and u in boil, foul, pine, and tube. 21. Coalescent. — A sound that precedes and unites (coalesces) with a vocal; as, w, y, and h in wet, yet, and hot. 23. Trigraph. — The union of three letters to represent one sound; as, eau in beau. 23. Labial. — A sound molded or obstructed at the lips; as, b, p, o, and oo in bit, lip, note, and boot. 24. Dental. — A sound molded or ob- structed at the teeth; as, t, s, e, and a in sit and sedate. 25. Palatal. — A sound molded or obstructed at the hard palate; as, j, r, e, and a in jug, rut, set, and sat. 26. Guttural. — A sound molded or obstructed at the soft palate; as, g, k, ti, and a in get, keg, tub, and far. [243] 244 TEE SYSTEM METHOD. 27. Name. — That combination of letters, syllables, or words, by wMch an object is distinguished, or known ; as, John, boy, box, we, you. 38. Noun. — A name that has or may have an affix added ; as, John-s, boy-ish, un-box, fur-s. 29. Peonoun. — A name that has not and will not take an affix ; as, we, you, he, they, I. 30. Proper noun. — One that individual- izes its object ; as, John, Englishman, Greeks, Csesars, Plato, France. 31. Common noun. — One that classifies its object ; as, boy, subvocal, verb, house, animal, letters, words. 32. Gender. — A distinction of names, hav- ing reference to the properties of sex. 33. Masculine gender. — That which represents the object as exhibiting the properties of the male sex. 34. Feminine. — That which represents the object as exhibiting the properties of the female sex. 85. Neuter. — That which represents the object as ex- hibiting the properties of neither sex. 36. Common. — That v/hich repre- sents the object as having sex without distinguishing -ic/ia^ properties of sex it has. 37. Yerb. — A word that asserts the progress or completion of ac- tion. 38. Regular verb. — One that takes ed added to the simplest form. 39. Irregular verb.— One that does not take ed added to its simplest form. 40. Principal parts. — Those most frequently used ; as, see, saw, seen ; talk, talked, talked. The three principal parts may always be found in the blanks, thus : I now, I yesterday, I have . 41. Complete verb. — One having three parts ; as, see, saw, seen. 42. De- fective VERB. — One having fewer than three parts ; as, will, would ; ought, ought. 43. Redundant verb. — One having more than three parts ; as, learn, learned or learnt, learned or learnt. 44. Letter. — A character used in language to represent a sound, or to determine signification. 45. Word. — A sign expressing in letters or sounds an idea. 46. Simple word. — One containing a single separable word. 47, Compound word. — One containing two or more simple words. 48. Primitive word. — One not reducible to a simpler form in its native language. 49. Derivative word. — One reducible to a simpler form in its native language. 50. Root. — A word from which other words are formed. 51. Etymon. — The least number of letters that represent an idea and constitute a word-part ; as, in, stit, ut, ion in institution. 52. Pre- fix. — An etymon placed before a root or another etymon ; as, re-move, cir- c«m-navigate, acZ-vance. 53. Suffix. — An etymon placed after a root or another etymon ; as, r-ei^-id-ly , TdOY-able. 54. Affix. — Either a prefix or a suffix. 55. Radical. — A root modified or unmodified. Thus in 'precede, recede, and intercede the English radical cede is the Latin cedere (to go^ mod- ified. 56. Etymology. — The science (^. e., tree system, or class system) of the etymon ; in a loose way. The study of words as made up of root and affix. 57. Orthography. — The science (tree system, or class system) of the letter ; in a loose way, The study of letters as expressing a language. 58. Word- analysis. — An unscientific term applied to word dissection. Real word-an- alysis would be unloosing the attributes, i. e., finding the classes, of words ; but this would be the doings of grammar. Thh System Method— No. 18. WORK-BOOK OF SYSTEM ETYMOLOGY, FOR THE USB OF 'PubliG Schools, Normal Schools, Teachers' Institutes, and Colleges, AND ESPECIALLY ADAPTED TO Soz-zooi-i IB2^:E^OSITIOn>TS. IN WHICH THE STUDENT LEARNS AND REMEMBERS UNDER THE GUIDANCE OF THE "LEARNER'S WORK-TREE" BY ORGANIZ- ING THE ETYMON-PARTS OF WORDS THROUGH THEIR CLASSES, BRANCHED TOGETHER AS CLASS- BRANCHES OF AN ETYMOLOGICAL- TREE WHOLE. ->^=^-<- J BY ISAAC ELDRIDGE WILSON, IKVENTOE OP THE LEARNEF.S WORK-TEEE, AUTHOR OP THE SYSTEM METHOD, A SERIES OP TEXT-BOOKS POR SCHOOLS, A SERIES OP STUDENT'S WORK-BOOKS, ETC. '• THE MAN MAY TEACH BY DOING, AND NOT OTHERWISE.''— EMEfi..^.. "^ '/ir ' AUG 13 I8BS4?] OHIO A. GO, IIvIv 5 Learner's Work-Tree Company. COPYRIGHT, 1886, BY ISAAC ELDRIDGE WILSON. ALL RIGHTS RESERVED. Any infringement will be prosecuted to the full extent of the law. Appendix A— Lessons to be Assigned. — "^©^-r-i-^-^®^" — Program I. — Arise, love, go, learn, burst, be, awake, dare, cry, bear, prove, eye, eat, beget, knit, sit, begin, swell, ought, must, spin, light, may, shall, set, lie, lay, can, dig, bid, bite, rap, creep, blow, bring, hoe, work, will, build, burn, buy, catch, whip, mean, choose, reave, know, do, drink, drive, write, fly, forbear, have, hide, wind, wet, kneel, lead, wake, think, swear, lose, pen, pay, ring, ride, stride, stay, split, spit, spill, say, shoe, shine, shave, sing, shoot, shut, smite, speak, sink, shrink, slit. Prog. II. — Turn, wedge, sit, set, abide, laugh, shall, fight, quit, smite, may, burst, awake, snatch, gird, give, go, grind, heat, can, build, lie, lay, see, bring, must, know, knit, hew, bless, twist, miss, do, will, crack, ought, gild, burn, blow, seize, compel, instill, rise, beseech, hear, wail, work, hail, sleep, cough, climb, clothe, smell, learn, light, expect, love, fly, tie, hoe, prove, shoe, bloom, dare, bid, dream, fall, eat, knot, spank, scald, walk, whet, mold, bet, steal, wed, wound, stay, spoil, be, craze, smoke, sink, shut, parse, analyze, classify, divide. Prog. III. — Snuffers, earnings, goings, tongs, thanks, oats, doings, in- feriors, pincers, ones, Jameses, armies, Lucies, feelings, measles, ashes, boxes, embers, churches, oxen, R. B. Hayeses, news, movables, blessings, tweezers, scissors, undertakings, sheep, pepples, O. P. Mortons, waters, halves, Miltons, goods, Blanches, Mackintoshes, mumps, warblings, math- ematics, searchings, odds, yokes, Miss Thompsons, acoustics, ethics, masses, shears, getters-up, hundreds, whiskies, tracings, tenths, lungs, bowels, be- liefs, heathens, couples, sevens, checkers, suds, wages, riches, moneys, compounds, plurals, proceeds, Platoes, senses, Greeks, Turks, truths, bricks, Chickasas, Powhatans, gods, Germans, Maries. Prog. IV. — Thanks, men, checkers, oats, lungs, bowels, doings, com- panies, ashes, waters, s's, soliloquies, tens, lO's, geese, peoples. Henries, Joneses, mumps, children. Senator Blaines, Daniel Websters, feet, thirds, spoonfuls, Lucases, measles, warblings, tongs, shears, deer, Greeks, pneu- matics, mathematics, dregs, Johns, halves, flies, pairs, spectacles, politics, ones, twos, f's, affairs, sixes, wages, movables, odds, riches, betters, teas, coffees, tongs, Germans, news, trappings, skimmings, ethics, hoes, bitters, hundredths, Shakspeares, Moseses, Pharisees, justices, entrails, sheep. General Gateses. Prog. V. — Lawful, homespun, nobleman, hailstorm, manliness, friendly, hour-glass, milkmaid, gentleman, gardener, reform, large, quiet, sailor, sea- gull, steamboat, sideboard, ox-goad, book-seller, oat, nut-cracker, stronghold, wooden, warehouse, formal, hilly, trust, classmate, inkstand, brightness, [247] 248 THE SYSTEM METHOD. grammarian, windmill, sharp-edged, waylay, justice, necklace, goodman, time-piece, unfetter, bluish, forget-me-not, apple-tree, coachman, music- teacher, demonstrate, overbalance, footstep, understanding, unlooked-for, redness, watch-key, rib, heart-broken, kingdom, uprising, hardihood, day- star, impossible, shoe-making, plowman, finisher, turncoat, hearsay, far- fetched, by-word, make-believe, workman, brotherhood, goodness, rainbow, redbreast, nowhere, whereas, trickster, misinform, reconstruct, colonize, irrepressible, finger-ring, ring-finger. Prog. VI. — Church, girl, he, mouse, oxen, child, men, we, it, foot, teeth, she, knife, god, goddess, boy, nephew, niece, John, key, wages, odds, doings, others, basis, Mary, they, who, Jameses, uncle, sir, witch, mother, host, widower, teas, sugars, son-in-law, James G. Blaine, geese, molasses, hundreds, twos, people, fives, sheep, deer, girls, wife, babies, Miss Smith, Mackintoshes, Platoes, me, measles, thou, you, thy, them, conscience, peace, nebula, cherubim, cattle, Greek, chemistry, summons, sirs, soliloquy, fly, TJ. S. Grants, Hindoos, landlord, she-goat, Chickasas, brick, Powhatans, testator, which, hoe, relative, friend, gypsy, p, plus, Maries, ladies, father- in-law, getters-up, step-brother, himself, myself, itself, Jew's-harp, New- year's-day, beeswax, newspaper, tradesman, to-night, to-day, to-morrow, ones, commander-in-chief, Moses, angels. Pkog. VII. — Man, geese, boxes, we, girls, church, David, history, god- dess, actor, widow, landlord, mouse, children, bridegroom, brother, she, knife, sister, nephew, thanks, niece, he-goat, landlady, wages, odds, basis, teeth, analyses, sheep, belief, ones, sugars, son-in-law, negro, molasses, Moses, hundred, dozen, census, twos, fives, people, news, snuffers, sober- ness, wife, valley, babies, duplicity. Miss Smiths, James, London, shears, class, company, Mary, Platoes, Spaniard, goods, movables, them, it, you, measles, stratum, vow, their, errata, us, earnings, blessings, thesis, data, nebula, pairs, tongs, seraph, index, medium, focus, honey, deer, cattle, chemistry, truth, summons, bellows, Mobilians, gods, Algonquins, cannon, dregs, scissors, half, staff, cloths, Mussulman. Prog. VIII. — Travel, he, cancel, conscience, be, awake, bear, begin, thou, will, shall, bid, glow, may, can, break, walk, love, ought, she, they, must, choose, sit, set, lie, lay, house, Csesar, woman, crisis, oats, climb, clothe, dare, dig, embers, sheejD, ashes, deer, bellows, means, dream, drive, goods, eat, nippers, fly, who, I, grow, tongs, their, knit, twist, Plato, Moses, know, learn, burst, it, blessings, doings, you, her, light, me, hang. Englishman, write, wet, German, Algonquin, teach, Seminoles, spell, spill, Mary, stratum, negro, shut, data, errata. Prog. IX.— Cata, ad, con, anti, amphi, am, ana, ante, a (Gr.), ab, a (A.-S.), be, bi, circum, dia, for, dis (Gr.), fore, contra, de, in (A.-S.), ec, dis (Lat.), en (Gr.), en (Lat.), mis, epi, ne. ex, extra, in (Lat.=m), in (Lat.= not), inter, intro, retro, off, on, hyper, hypo, non, ob, per, meta, out, post, re, pro, se, over, pre, to, peri, sub, un, syn, under, super, with, trans. WORK-BOOK OF ETYMOLOGY. 249 Prog. X. — A (Gr.), ab, ard, dom, cata, ad, en (Gr.), en (Lat.), en (A.-S), er (Lat.), er (A.-S.), dis (Lat.), ante, circum, con, ec, a (A.-S.), be, an, ful, ate, able, dis (Gr.), ex, mis, ne, ish, kin, cle, ling, eer, in (Lat.=?iot), off, ance, para, ock, ster, ob, inter, ship, in (Lat.=m), id, fly, le, ty, sub, trans, se, epi. Prog. XI. — Lion-hearted, always, old, heavily, open-mouthed, pitch- dark, green, together, greenish, blue-green, backward, three, child-like, elegant, ill-natured, some, politely, sad, lengthways, white, ruinous, severe, immediately, here, rough-hewn, sour, constantly, lengthwise, ripe, remotely, thereabout, little, thereby, many, soon, excellently, somehow, coarse, somewhat, tall, therefor, thereby, now, Socratic, wearisome, headlong, same, fearfully, lonesome, guestwise, triple, yonder, rough, far-fetched, second, overdone, finally. Prog. XII. — Fruit-bearing, everywhere, frequently, well-earned, boldly, when, largest, downward, balmy, anyhow, towards, this, sufficiently, hare- lipped, then, happy, sidewise, weeds, warlike, manly, therefore, trustworthy, greener, necessarily, loveliest, slowly, seaward-gazing, wisely, ill-content, league-long, adjacent, long-haired, the, yonder, inarticulate, excessively, meadowy, wholesome, worm-eaten, upward, half-believing, gloriously, groimdless, eastward, round, same, barefaced, identical, frolicsome, super- fine, self-same, nowadays, a, nevertheless, anywhere, an. Prog. XIII. — Fine-|-apples, a-|-rich-|-inan, ab+solve, heart-(-broken, the-|-most-|-delicious+fruit, contra-j-dict, American-|-policy, be+side, air-|- built, a+scend, tran-|-scend, rough-|-hewn, Siberian-f-snows, termin-{-ate, civil-fize, all-|-wise, blue-j-green, pay-j-er, green-f-leaves, pay-|-ee, act-(-or, con-j-voke, red-j-tape, de+part, Rom-j-an, con+duct, un-|-ripe, i]l-(-bred, new-f hat, lion-1-hearted, publ-|-ish, coin-fage, month-j-ly, black-fen, far-|- fetched, terri-)-fy, dis-|-miss, colon-|-ize, gradu-j-ate, en-(-fold, child-f-like, art-fist, fruit-fbearing, im-f mense, Swiss-f hero, hero-f ism, pleasant-fair, e-f ject, team-f ster, partis-fan, fore-f tell, lion-f ess, Eve-flike, mediatr-f ix, wild-f geese, in-f cise, man-flike, first-fborn, owl-f like, inter-f pose. Prog. XIV. — Un-ftie, publici-fty, intro-fduce, ill-fcontent, mis-f guide, both-f arms, occup-f ancy, ripe-f cherries, good-f looking, vigil-f ance, quie-ftude, male-f fact-f or, Egyptian-f lily, ob-f ject, the-f Austrian-f army, hard-fworking, out-frun, ad-fher-f ence, rude-fness, per-f for-f ate, above -f said, nat-f ion, either-f hand, move-f ment, two-f busy-f men, two-f leaved, seiz+ure, pre-f judge, post-f script, act-five, live-fly, child-f like, care-f less +ly, twelve-f peaches, un-f true, oil-f y, pro-f duce, retro -f grade, avoid-f able, law-f less, one-f legged, hero-f ic, Afric-f an, wool-fen, Rom-fish, matter-f of-ffact, direct-for, trick-f ster, se-fcede, re-fcon+struct, broad-f shoul- dered, per-ffect+ible, poet-f ic-f al, un-fheard-f of, an-f nex, fore-f father, sup-f press, long-f looked-ffor, ir-f re-f press-f ible, ir-fresolute. Prog. XV. — Contempt-fible, tam-fable, be-fstir, drunk-fard, dis-f joln, be-fdew, n-f either, dis-fplease, re-f new-f al, Christi-f an, em-fbody, missiona-fry, black-fen, en-ftrap, wood-fen, fore-ftell, di-frect-for. 250 THE SYSTEM METHOD. preach+er, un-|-ripe, pre+judge, fear-f-ful, educat-fion, blu+ish, mis-l- take, in-|-secure, re-f-plant, civil-|-ize, pre+vent+ive, super-j-natur-(-al, plant-j-let, man-)-ly, banish+ment, friend+ship. Prog. XVI. — A-|-rise, n-|-one, under-}-mine, be-|-set, for-(-give, be-|- friend, teach-|-able, un-f-taught, scholar-j-ly, un+schooled, health-j-ful, health-f-y, kmg-)-dom, liast-)-en, beginn-j-er, pay-|-ment, un-|-paid, pay-j-ee, with-|-stand, over-{-look, bound-)-less, Sun-J-day, Mon-|-day, dai'k-|-some, song-j-ster, Satur+day, book-|-ish, begg-|-ar, Fri-|-day, Wednes-|-day, blood -f-y. wilder-j-ness, Thurs-|-day, Tues-|-day, critic+ize, e-|-duc-|-at-)-or, tranquili-|-ty, knav-|-isb., medicin-|-al, op-(-press-|-ive, power-|-less-|-ness, skeptic-|-ism, nation-1-al, imit+at-|-ion, dis-]-embarrass-)-ment, in-j-sincere, disciplina-j-ry, in-|-ex-}-press-|-ible, safe-)-ty, im-f-pure, liiIl-|-ock, n-|-either. Pkog. XVII. — A-j-path-j-y, gold-fen, syn+tact-|-ic, sym-j-metr-|-y, kitt -j-en, an-j-arch-f-y, a-(-vert, abs-|-tain, ana-|-lyze, cardi+ac, cour-|-age, dia-f- meter, cata-f-logue, anti-j-path+y, preach-j-er, Cat-|-liol-)-ic, dis-)-syl-|-lable, at-|-tain, hermit-|-age, annu-|-al, debt+or, di+phthong, af-|-fix, ap+plaud, dys+pept-|-ic, as-)-sist, a-j-bed, a-j-nse, be-j-take, ec-|-centr-|-ic, mani-)- fold, ex-j-ort-l-ive, en-j-tliusi+ast-j-ic, truth-|-ful, em-|-pbas-|-ize, ag-j-greg +ate, el-|-lipt+ic+al, con-)-sign, epi-|-cycle, gos-fling, vill+ain, vigil-f- ance, eu+log-j-ize, am-|-put-|-ate, anti-)-cip-j-ate, mis-)-ap-|-ply, con-)-nect, co-|-ev-|-al, am-)-put-f-at-)-ion, com-fpress, wilder-l-ness, hill-f-ock, as-j-sist -fant, ad-)-versa-f-ry, leg+ate, pre+vise, hyper-|-crit-|-ic-|-al, liypo-|-tenuse, meta-l-pbys-|-ics, to+day, mani-f-kin, lord+sbip, in-|-iti-|-ate, sta-|-ble, cred-|-ible, ortho-|-dox, un-|-equal, vesi-|-cle, di-j-verge, dif-|-fuse, em-(- body, ex-)-clude, hast-f-en. Prog. XVIII. — Team-)-ster, extra-j-ordina-f-ry, in-|-vade, under-fstand, peri+osteum, animal-|-cule, dif+fid-f-ence, Wesley-|-an, pre+sid-j-ency, de +pend-)-ent, four-)-teeii, pro-}-gnosls, in-|-born, side-)-wise, inagiii-|-fy, peri-f-od-l-ic, im-f-merse, lr-|-radi-}-ate, rig-l-id, con+cept+ion, m-\-et-{- fici-j-ent, ig-|-noble, sym-|-path+y, under-|-go, blood-(-y, sbeep+ish, de-j- ism, dent+ist, intro-f-duce, oc+cur, of-j-fer, war-j-like, majes+ty, retro-|- spect, sub+merge, su-f-spect, sug+gest, pros-|-elyt-f-ize, sym-f phon+y. Prog. XIX. — A -{-shore, n-j-either, con-|-tempt-|-ible, n-|-one, for+give. drunk-fard, dis-j-join, wiz+ard. be-)-dew, dis-|-please, re-f new+al, black-|- en, en-|-trap, be-|- friend, teacb-(-able, un-f-taught, un+school-f-ed, health -(- ful, king-1-dom, beginn-fer, pay-fment, fore-j-tell, di-(-rect-|-or, pre+judge, pay+ee, e-{-duc-|-at+ion, blu+ish, bound+less, mis-|-take, re-)-plant. Sun -[-day, pre+vent+ive, super-f-natur-j-al, Mon-|-day, eagl-j-et, Satur-j-day, clos-|-et, Tues-f-day, dark-|-some, wilder-|-ness, op-|-press-[-ive, friend-|-ship, power-f less-|-ness, skept-^-ic+ism, im-|-pure, im-|-it-)-at-l-ion, puri-j-ty, fruit-fage, safe+ty, crystall-}-oid, blood-|-y. Prog. XX. — A-j-wake, drunk+ard, gift+ed, boun-fd, be-|-think, be-j- cause, an+onym-[-ous, in-|-ef+fici-|-ent, amphi-(-bi-f ous, inter+cede, Ana-|- bapt-fist, squal-j-oid, opu-|-lent, Tartar-}-ean, anti-|-pode-|-s, phys+ics, ethm-1-oid, con-|-ceal-|-ment, organ-j-ize, ob+liter-|-ate, apo-j-stas-|-y, apo-(- WORK-BOOK OF ETYMOLOGY. 251 stle, en+thusi+asm, cata+rrh, joc-f-ose, variol+oid, pel+luc+icl, per-{- vade, dia+meter, dis+syl+lable, multi+tude, gnost+ic, ec+clesi+ast+ic-f al, di+phthong, ec+centr+ici+ty, e+nerv+ate pre+dict, liber+ty, pro+ vid+ent, en+dem-'pic, epi+dem+ic, re+pel, liypo+crite, dram+a, trace-f- able, glob-{-ule, post+ure, obstin+acy, di-[-vis-|-ible, creat-fure, retro-1- spect-f-ive, met-f-onym+y. Prog. XXI. — Se-|-greg-|-ate, para-j-dox, con-|-flu-f-ence, ir-j-re+sol-l- ute, sapon+ace+ous, sub+merge, suf+fer, peri+osteum, savag-j-er-t-y, per-fcent-|-age, supra+orbit+al, sur+vey, con+vey+ance, syn-fchron-j- ous, pro-f-gram, sy-fstem, sym+path-j-y, as+sist+ant, tlief+t, tra-j- verse, dee-f-d, ad+versa+ry, ac+cesso-fry, pay+ed, hear-fd, ab-j-jeot, ad+voc-l-ate, af+fix, stai+d, tlirif-f-t, ob+serv+ato+ry, ap+plaud, gold -f-en, leather+n, heav+en, mod-|-er-l-n, en-j-dear, daw+n, am-fput+ate, anti+cip+ate, liber+at-}-ion, larg+er, chatt+er, mole+cule, mo+st, man -|-liood, in+born, bi+ped, bi+gamy, kern+el, bow+el, bis-f cuit, circum -|-veut, mis-j-give, metall-|-oid. Prog. XXII.— Mors+el, tab+le, go+ing, bagg+ing, wolf+ish, soldie-1- ry, fer-j-ry, mani+kin, n+augbt, con-j-nect, cor+rug+ate, nibb+le, out-j- live, dribb+l3, magni-f fy, gos-fling, aquat+ic, peri+od+ic, not-fice, utt-|- er, contro-|-vert, stup+id, prism+oid, puer+ile, humm+ock, ball+oon, Campbell+ite, de-j-flect, viper+ine, macadam-j-ize, don-|-at-|-ioD, ladi-f- es, e-fstabl-j-isb, grow-|-th, heal+th, over+reacb, over-{-coat, s+melt, dis -|-pel, s+pread, tru+th, de-fism, Wesley+an+ism, di+verge, ex+clude, dent-j-ist, fin+ite, cross+wise, Huss-fite, un-|-do, un-ftrue, e+dent-l-ate, heal+tli-t-y, ef+flu+ent, de+kis-i-ive, capt+ive, bab+y, under+ling, extra -)-mur-|-al, eas+y, an+y, witb+stand, Will+ie, ex-fcresc+ence, leav-|-ing. Prog. XXIII. — Ac+cede, con-j-duct-j-or, dis+cuss+ion, organ-|-ize, in-f-vis-l-ible, libra+ry, intro-j-duc-f-tion, martyr-|-dom, name-|-less, pro-j- sec-f-ute, con-}-sequ-|-ence, ad-(-mon-j-isb, pur-fsue, a-j-voc+at-fion, pro-|- pose, in-j-trus-j-ion, re-f bell+ion, un-t-ten-|-able, re-|-miss-|-ion, lov+able, therapeut-f ics, syn+tact-j-ic, taberna+cle, ampli+tude, con-|-greg-t-at-[-ion, ed+ible, cour-(-age, in-(-toxic+ate, se+cess-j-ion, inter+cede, con-l-sider-|- at+ion, dur-fable, re+mit, trans-(-fer, capit-j-al, di+pbthong, felicit-|-ous, con-j-junct+ion, de+capit-l-ate, e-j-loqu+ent, e-fluc-j-id-j-ate, ab-t-norm-(- al, pract-j-ic+able, ec+centr-|-ic, digni+ty, dif+ficult, di-j-graph, flex-J- ible, feroci+ous, im-t-poss+ible, com4-prehens-|-ible, in+cur-j-able, mar-j- ine, in-|-tent-l-ion, inter-f-vene, con-j-vent+ion, matbemat-|-ics, lion-|-ess, para+grapb, padd+le, pack+et, over+reach, out-|-live, parc-}-el. Prog. XXIV. — Motion, motive, parody, ossify, parentage, nasal, re- flect, ornament, protector, punishment, postponement, pursue, prospect, senior, render, resonant, secede, texture, timidity, subsequent, zodiac, venal, unwind, altitude, ambition, acephalous, beseech, bepraise, arrange, concep- tion, ciiTumstance, degrade, convex, consumption, fraction, evasion, fore- know, Impulsive, grievance, Invincible, inter, magnitude, prophetess, testament, succumb, thermal, technics, underwood, wolfish, verbose, 252 THE SYSTEM METHOD. wooden, wizard, aliment, allusion, amendment, breakage, armful, brevity, canine, boxes, ascend, concoction, collect, dejection, decimal, embalm, filial, exhort, finny, marriage, intrude, luxury, overeat, notify, promotion, piety, scripture, scissors, reprove, sanctify, temjooral, sufficient, tenement, thermal, technics, stomatic, underwood, witness, tyrannize, valor, amendment, ac- quire, illusion, artless, bloody, ardor, deodorize, contortion, decrease, con- vulsion, controvert. Prog. XXV. — "Warlike, translucent, tradition, remorse, supplant, ridicu- lous, comfort, saccharine, stable, ravenous, precision, knavery, method, mental, minute, iterate, justice, military, ignominious, infusion, expression, retrograde, assumjDtion, dissuade, dribble, denticle, describe, incurable, con- stant, baby, cemetery, attention, agent, adjective, vulnerable, vindicate, tor- ment, antagonist, antecedent, common, irksome, precede, patience, pastor, miracle, labial, mercantile, metatarsus, restriction, signal, transgression, tremendous, vocal, missionary, supply, sidewise, premature, perfumery, vo- cal, votary, supremacy, torrid, patriotic, objection, religion, ministerial, synagogue^ resumption, patience, castigate, cartage, quackery, audible, bar- barous, villain, merchant, indomitable, expectancy, expectorate, induction, tradition, vocalize, resurrection, solar, reversion, Presbyterian, reverend, nursery, ignoble, ignorant, dispel, correspond, atheism, disruption, emo- tion, derision. Prog. XXYI. — Revive, revolve, solitude, victory, infant, friction, inflexible, fretful, expel, fugitive, voracity, translation, company, congress, fraudulent, percentage, transmit, saline, revolution, recumbent, realism, pericardium, perjury, permanent, predominate, peddle, laity, kingdom, ointment, pervade, mortal, language, lenity, inherent, infatuate, gamester, explode, extant, iniquity, extortion, garrulous, garment, fundamental, exult, enormous, distraction, epicycle, enactment, doubtful, cynic, cultivate, darling, dexterity, crescent, consonant, beatitude, aviary, cellule, aerate, after, vulgarism, tenant, subterranean, thoracic, rotundity, candidate, statement, spectacle, procrastinate, rabid, prelude, lacteous, production, reflect, obnoxious, persecution, accidental, perfect, perihelion, missive, lacerate, migratory, influence, provision, sympathy, connection, reception, puerile, masculine, coUajase, submit, donation, temporary, trespass, transact, infidel, establish, caution, message. Prog. XXVII. — Itinerant, symmetry, knightly, mitigation, mnemonics, inference, inflammation, expire, insj)ire, atonic, immoral, immortal, coun- teract, furious, insanity, fretful, injurious, corpuscle, animalcule, globule^ captain, inexhaustible, interpose, lunar, mariner, visible, interlude, mana- cle, intellect, insulting, macerate, loquacity, elocution, lobule, levity, nar- cotic, panic, paganism, osculation, plenitude, magnitude, multitude, pro- posal, popery, remonstrate, demonstrate, refulgence, suspect, symphony, vengeance, untrue, altitude, bronchial, boundary, carnal, article, aquatic, contemptible, edict, dilation, didactic, essence, dilute, mundane, nefarious, despotic, cumulative, devotion, cutaneous, depredation, consolidate, com- WORK-BOOK OF ETYMOLOGY. 253 posnre, tactics, territory, syntax, stifle, tedious, stubble, succor, timidity, thimble, ruminant, society, sonorous, reversion, recline, preposition, re- dound, rendition, prevail, resuscitate, presidency, procedure, prodigy, obvi- ate, perigee, pedestrian, obituary, oblivion, obstacle, pentecostaL Prog. XXVIII. — Omission, undersell, convention, moonbeam, omi- nous, monkish, sweetmeat, imbibe, heroic, honorable, cylinder, apathy, afield, workman, withdraw, anticipate, chatter, aversion, circuit, because, cellular, cathedral, circle, baby, avert, banishment, childhood, remorse, thinnest, square, songster, ruminate, revive, proffer, reasonable, reduction, radicle, redaction, perambulate, enthusiasm, permission, occur, oculist, mortal, length, monitor, misery, pear-shaped, inhale, factor, gastric, expo- nent, grateful, funeral, emotion, doctor, enrage, pay-day, debtor, dawn, cripple, darkling, curable, detraction, consanguinity, composition, epidemic, watch-word, submit, collapse, cervical, creamery, provide, confluent, mount- ainous, conscience, antithesis, adjoining, livelong, ashore, adverb, Wes- leyan, windward, receive, watchful, traction, trade-wind, providence. Prog. XXIX. — Cellule, thermal, summon, sacrament, township, shad- owy, plenteous, royal, review, subdue, May-day, sparsest, remove, predic- tion, reflect, postpone, pass-word, quickstep, recumbent, deposit, rectitude, molecule, judgment, religion, peaceable, prescribe, precedence, distract, sing-song, receive, postscript, greenish, wizard, redeem, prevail, justify, redolent, misplace, critic, perusal, onset, persuasion, oration, repulse, non- sense, venal, capital, personate, wine-press, sluggard, modesty, mollify, laureate, insatiable, insanity, deduction, insecure, insipid, reply, garment, innocent, windmill, traction, undergo, torsion, fusion, sunflower, cowhide, inquiry, family, gratify, exult, impression, transport, transatlantic, imped- iment, famous, globule, falsify, fashion, grantee, payee, wishful, friendless, remonstrate, dogmatic, deduce, inkstand, penman, wakeful, bookworm, controversy. PuoG. XXX. — Submission, return, arrival, condescend, admonish, alleviate, punctuation, renewal, confusion, deciduous, suspicion, interjec- tion, dropping, doubtful, epitaph, dissatisfy, emanate, enthusiast, religion, demonstrate, donor, gratify, removal, intervention, equity, depression, con- spiracy, devotion, dispel, companion, consignee, martyrdom, incision, revis- ion, apprehend, cautious, admirable, chatter, Baptist, childhood, testament, subject, tactual, solar, rusticate, squaloid, prismoid, solitary, verbal, rotund- ity, consequence, songster, solitude, matrimony, sidereal, lunar, endorse- ment, spoliation, spherical, salary, revive, recapitulate, redolent, precession, enormous, procurement, redemption, pre-emption, reconcile, decision, on- set, option, odorous, presidency, optics, reference, president, consumption, assumption, raccoon, impediment, production, invent, procrastinate, lat- itude, magnify, justice, impend, lachrymal, doubtful, decapitate, damnation, resuscitate, scribble, resurrection, dukedom, crystallize, cordial, defensive, question, final, recognize, perambulate, perimeter, oration, misery, labora- tory, modesty, circumference, mesmerism, Calvinism. 254 THE SYSTEM METHOD. Program XXXL— Words of Two Roots. 1. AGOGVE=agogeus. Demagogue Pedagogue 2. A:NTnROFY=cmt7iropos, Misantliroi^y Philanthropy 3. Ait.cuY=arche, y. Heptarchy Heterarchy Monarchy Oligarchy 4. A'T'E.=^atUS. Amalgamate Certificate Duplicate Emancipate Triplicate 5. ciDE=caecZc>. Fratricide Homicide Infanticide Matricide Parricide Regicide Suicide 6. cosis.=Gosmos. Macrocosm Microcosm 7. CRACY=/fcratos, y. Aristocracy Democracy Theocracy Timocracy 8. CULTURE :=c^iZ t-mSiSS- 4> 1^' <^ .^-^v^ WOBK-BOOK OF ETYMOLOGY. 275 Prog. XX.— Branching by the Learner's Work-Tree. A =etymon, prefix, Anglicized, A.-S., from. Wake =etymon, root, base. verb. Anglicized, irregular. Drunk : =etymon. root. adjunct. adjective. Ard =etymon, suffix, Anglicized, noun. one who. Gift =etymon, root, base. noun. Anglicized, common. Ed :=etymon, suffix, Anglicized, adjective. with. Boun =etymon. root, base. verb. unanglicized. A.-S. D =etymon, suffix. Anglicized, verb, finished. Be =etymon, prefix. Anglicized, A.-S., be. Think =etymon, root, base, verb, Anglicized, irregular. Be =etynion. prefix, Anglicized, A.-S„ by. Cause =etymon, root. base. noun. Anglicized, common. Ani =etymon, prefix, unanglicized. , Greek, without. Onym —etymon, root. base, noun. unanglicized, Greek. Ous =etymon, suffix. unanglicized , adjective. quality of. In =etymon. prefix. Anglicized, Latin, not. Ef =etynion. prefix. Anglicized, Latin, out of. Fici =etymon, root, base. verb, unanglicized. Latin. Ent^ =etymon, suffix. unanglicized , adjective. ing. Amphi =etymon. prefix. unanglicized , Greek, double. Bi =etymon, root, base, noun. unanglicized, Greek. Ous =etymon. suffix. unanglicized, adjective. quality of. Inter =etymon, prefix, Anglicized, Latin, between. Cede =etymon. root. base. verb. unanglicized. Latin. Ana =etymon, prefix. unanglicized; , Greek, again. Bapt =etymon, root. base, verb. unanglicized. Greek. 1st =etymon. suffix. Anglicized, noun. one who. Squal =:etymon. root, base. noun. unanglicized. Latin. Oid =etymon, suffix. unanglicized, adjective. like. Opu =etynion, root, base. noun. unanglicized. Latin. Lent =etymon. suffix. unanglicized , adjective. full of. Tartar =etymon, root. base. noun. Anglicized. Ean =etymon. suffix. unanglicized, adjective, of. Anti =etymon. prefix. Anglicized, Greek, opposite. Pode =etymon. root, base. noun. unanglicized. Greek. S =etymon, suffix. Anglicized, noun. two or more. Phys =etynion, root. base. noun. unanglicized. Greek. Ics =etynion, suffix. unanglicized; , noun. things of. Ethm =etymon, root, base, noun. unanglicized. Greek. Oid =etymon. suffix. unanglicized, adjective. like. Organ =etymon. root. base, noun. Anglicized, common. Ize^ =etymon, suffix. Anglicized, verb. make. 1 An is a derived form of a. 2 ^>j^ jg derived from arU. ^ Ize is derived from ise. 276 TEE SYSTEM METHOD. Prog. XXI. Se =etynion. Greg =etynion. Ate =etymon. Para =etyHion, Dox =etymon, Con =etymon, Flu =etymon. Ence =etymon, Ir =etymon, Ke =etymon. Sol =etymon, Ute =6tymon, Sapon =etymon, Acei =etymon. Ous =etynion, Sub :=etymon. Merge :=etymon. Suf :=etymon, Fer =etymon, Peri =etynion. Osteurc I := etymon. Savag =etymon, Er =etymon, Y =etymon, Per =etymon. Cent =etymon, Age =etymon. Supra =etymon. Orbit =etymon, Al =etymon. Sur =etymon. Vey =etymon. Con =etymon. Vey =etymon. Ance =etymou, Syn =etymon. Cbron =etymon. Ous =etymon, Pro =etymon. Gram =etymon, root, base, suffix, Anglicized, prefix, Anglicized, prefix, Anglicized, root, base, unanglicized, Latin, make, at side. unanglicized, Greek, together. unanglicized, Latin, act of. not. unanglicized, Latin. ,— Branching by the Learner's Work-Tree, prefix, unanglicized, Latin, apart. root, base, noun, suffix, unanglicized, verb, prefix, unanglicized, Greek, root, base, noun, prefix, Anglicized, Latin, verb, noun, Latin, Latin, verb, suffix, unanglicized, adjective, quality of. root, base, noun, unanglicized, Latin suffix, unanglicized, noun, state of. suffix, unanglicized, adjective, quality of. prefix, Anglicized, Latin, under, root, base, verb, prefix, Anglicized, Latin, root, base, verb, prefix, unanglicized, Greek, root, base, noun, root, adjunct, adjective, suffix, Anglicized, adjective, more, suffix, unanglicized, noun, state of prefix, unanglicized, Latin, root, adjunct, adjective suffix, Anglicized, noun, prefix, AnglicL'Jed, Latin, root, base, noun, suffix, unanglicized, adjective, of. prefix, Anglicized, Latin, under. verb, Latin, noun, noun. Anglicized, regular, under. unanglicized, Latin, around, unanglicized, Greek. through. pay for. over. Anglicized, common, root, base, prefix, Anglicized, root, base, suffix, Anglicized, prefix, unanglicized, Greek, root, base, noun, suffix, unanglicized, adjective, quality of. prefix, unanglicized, Latin, forth. root, base, noun, unanglicized, Greek. *This is the same as aey, the y being changed to e. See exception to Rule VII., Appendix C, Work-Book No. 17, page 196. unanglicized, Latin. with. unanglicized, Latin. act of. with. unanglicized, Greek. WORK-BOOK OF ETTMOLOOT. 27Y Prog. XXII : /. Prefix+Rad- ical-\-Suffix. 2. Radical 3. Affixes 4. Etymolog- from. from, ical meaning. Morsel Table Going Outlive Bagging Wolfish Soldiery Ferry Manikin Naught Connect Corrugate 1 Nibble Leaving Dribble Magnify Gosling Aquatic Periodic Notice Utter Controvert Stupid Puerile Hummock Balloon Campbellite ^ Macadamize ^ Donation a. Greek. b. Latin. c. Anglo-Sax. = mors-|-el, Latin, = tab-j-le, Latin, = go-j-ing, A.-S., = out-flive, A.-S., = bagg+ing, A.-S., = wolf+ish, A.-S., = soldie-|-ry, Latin, = fer-|-ry, Latin, = mani-f-kin, A.-S., = n-|-auglit, A.-S., =^ con-|-nect, Latin, = cor-(-rug-|-ate, Latin, = nibb+le, A.-S., = leav+ing, A.-S., = dribb+le, A.-S., = magni-(-fy, Latin, = gos-j-ling A.-S., = aquat-j-ic, Latin, = peri-|-od-(-ic, Greek, = not+ice, Latin, = utt+er, A.-S., = contro+vert, Latin, = stup-j-id, Latin, = XDuer-|-ile, Latin, = bumm-j-ock, A.-S., = ball-foon, A.-S., = Campbell-|-ite, a. Greek. b. Latin. c. Anglo-Saxon. Latin, little bite. Latin, like board. A.-S., continuing go. A.-S., live beyond. A.-S., continuing bag. A.-S., like wolf. Latin, aggregate of soldiers. Latin, place where bear. A.-S., little man. A.-S., not any thing. Latin, tie together, Latin, make wrinkle with. A.-S., repeatedly nip. A.-S., thing left. A.-S., repeatedly drip. Latin, make great. A.-S., little goose. Latin, of water. Greek, of way around. Latin, thing mark. A.-S., more out. Latin, turn against. Latin, quality of stupor. Latin, pertaining to boy. A.-S., little hump. A,-S., large ball. Latin, one who Campbell-s. Latin, make Mac Adam. Latin, act of make gift. macadam-(-ize, don-j-at+ion, Latin, ^ In all cases, the suffixes ate, ite, and ute are to be treated as suffiuxes, not as a part of the root. This is done that the simplest form of the root may not be dis- guised, but may be seen to be the same in the greatest possible number of words. For example, the words decline and declination are to be divided into de-|-cline (root, cline), and de-j-clin-[-at-|-ion (root, clin), not de-|-clinat-)-ion (root, clinat) ; since the latter division would lead the student to suppose that these two words are made from different roots (cUne and clinat). He would thus miss the great aim of this study, which is to enable the learner to master the multitude of words by reducing their etymon parts to the least possible number of classes. When, however, the meaning of aie, ite, and ute ("make") gives no force to, and cannot be united with, the other parts of the etymological meaning, it may be omitted. ^ See note 2 under Proir. XVIII. 278 THE SYSTEM METHOD. Prog. XXIII : /. Prefix-\-Rad- 2 ical-\- Suffix. Accede Conductor Discussion Organize Invisible Library Introduction Martyrdom Nameless Prosecute Consequence Admonish Pursue Avocation ^ Propose Intrusion Rebellion Untenable Remission Lovable Therapeutics Syntactic Tabernacle Amplitude Congregation ^ Edible Courage Intoxicate Secession Intercede Consideration^ Durable Remit Transfer Cajjital Diphthong Felicitous Conjunction Decapitate = ac+cede, = con-f-duct-l-or, = dis-j-cuss+ion, = organ-j-ize, = in-fvis-j-ible, = libra-f-ry, = intro-f-duct-J-ion, = martyr-j-dom, = name-|-less, = pro+sec+ute, = con-|-sequ-)-ence, = ad-f mon-)-ish, = pur-)-sue, = a-(-voc-|-at+ion, = pro-f-pose, = in+trus-f-ion, = re+bell-|-ion, = un-|-ten-)-able, = re4-miss-)-ion, = lov-(-able, = tlierapeut-f-ics, = syn-|-tact-|-ic, = taberna-)-cle, = ampli-|-tude, = con-j-greg+at-f-ion, = ed-|-ible, = cour-|-age, = in-)-toxic+ate, = se+cess+ion, = inter-fcede, con-|-sider-f-at-j-ion, = dur-f-able, = re-|-mit, = trans+fer, = capit-|-al, = di+phthong, = felicit+ous, = con-{-junct-|-ion, = de-j-capit+ate. . Radical 3. Affixes 4. Eiymolog- from. from, ical meaning. a. Greek, a. Greek. b. Latin, b. Latin. c. A.-Sax. c. A. -Sax. Latin, Latin, go to. Latin, Latin, one who lead with. Latin, Latin, act of shake apart. Greek, Greek, to make organ. Latin, Latin, that can be seen not. Latin, Latin, place where books. Latin, Latin, act of lead into. Greek, A.-S., A.-S., A.-S., state of martyr, without name. Latin, Latin, make follow forth. Latin, Latin, act of follow with Latin, Latin, make warn to. Latin, Latin, follow forth. Latin, Latin, act of call from. Latin, Latin, place forth. Latin, Latin, act of thrust in. Latin, Latin, act of war again. Latin, Latin, that can be held not. Latin, Latin, act of send back. A.-S., Latin, worthy of love. Greek, Greek, things of heal. Greek, Greek, of arrange together. Latin, Latin, little board. Latin, Latin, state of large. Latin, Latin, act of flock together. Latin, Latin, that can be eaten. Latin, Latin, state of heart. Greek, Latin, make arrow in. Latin, Latin, Latin, Latin, act of go apart, go between. Latin, Latin, act of sit with. Latin, Latin, that can last. Latin, Latin, send back. Latin, Latin, bear across. Latin, Latin, of head. Greek, Greek, vocal two. Latin, Latin, Latin, Latin, full of happy. act of join together. Latin, Latin, make head from. ^ See note 1 under Prog. XXII. WORK-BOOK OF ETYMOLOGY. 279 Prog. XXIY /. Prefix-\-Rad- 2. Radical ical-\-Suffix. from. 3. Affixes 4. Eiymolog- from. ical meaning. a. b. c. Greek. Latin. Anglo-Sax. Motion = mot-)-ion, Latin, Motive = mot+ive, Latin, Parody =par-|-ody, Gr., Ossify = ossi-}-fy, Latin, Parentage = parent-j-age. Latin, Nasal := nas-|-al, Latin, Reflect =re-|-flect, Latin, Ornament = orna-|-nient. Latin, Protector =pro-(-tect+or. Latin, Punishment = pun-|-isli-l-ment,Latin, Postponement=post-fpone-|-ment, Latin, Pursue =pur-)-sue, Latin, Prospect =pro+spect. Latin, Senior ^ = seni-|-or, Latin, Render =ren-|-der, Latin, Secede =se-(-cede. Latin, Texture = text-j-ure, Latin, Subsequent ^sub-|-sequ-|-ent, Latin, a. Greek. b. Latin. c. Anglo-Saxon. Latin, act of move. Latin, that which moves. Gr., song at side. Latin, make bone. Latin, state of parent. Latin, of nose. Latin, bend back. Latin, that which adorns. Latin, one who covers fortk. Latin, act of make penalty. Latin, act of place after. Latin, follow forth. Latin, look forth. Latin, more old. Latin, give back. Latin, go apart. Latin, that which woven. Latin, that which follow under. Trog. XXY: /. Prefix+Rad- 2. Radical 3. Mixes 4. Etymolog- ical-\-Suffix. from. from, ical meaning. a. Greek. a. Greek. b. Latin. b. Latin. c. Anglo-Sax. c. Anglo-Saxon. Trog. XXYI: /. Prefix+Rad- 2 ical-\- Suffix. Radical 3. Affixes 4. Etymolog- from. from, ical meaning. a. Greek. a. Greek. b. Latin. b. Latin. c. Anglo-Sax. c. Anglo-Saxon. Trog. XXTII: 1. Prefix-^ R ad- 2. Radical 3. Affixes 4. Eiymolog- ical-{.Suffix. from. from, ical meaning. a. Greek, a. Latin. c. Anglo-Sax. a. Greek. b. Latin. c. Anglo-Saxon. 1 The student's work under Progs. XXV., XXVI., and XXVII., being precisely like that done under Progs. XXII., XXIII., and XXIV., is here omitted. These programs should not, however, be omitted in teaching the subject, as they constitute one of the most important processes of work in the whole science of etymology. The teacher wiU have no difficulty in deciding the questions that may arise, as he can refer to the work done under Progs. XXII., XXm., and XXTV., but always to Appendix F. The particulars to be assigned under these programs are to be found in Appendix A. ^Or = er, more. 280 THE SYSTEM METHOD. Prog. XXTIII. — 1.— is a 1. Simple 1. Primitive word, 2. SpeUed Ortho- 2. Compound 2. Derivative, gi-aphicallj , 3. Phonically , and 4. Etymologically 1 1 . 5. the 1. Prefix, 3. Root, 3. Suffix, 6. Is an 1. Etymon, 7. — 1- Anglicized from the 1. Greek, 8. And signifies — . 2. Separable word, 2. Unanglicized, 2. Latin, 3. Anglo-Saxon, 1. (1.) Omission is a simple primitive word, (2.) spelled orthographic- ally, omission, (3.) phonically, o-mish-un, and (4.) etymologically, o-|- miss-j-ion. (5.) O, the prefix, (6.) is an etymon, (7.) unanglicized from the Latin, (8.) and signifies "out." (5.) Miss, the root, (6.) is an etymon, (7.) unanglicized from the Latin, (8.) and signifies "send." (5.) Ion, the suffix, (6.) is an etymon, (7.) Anglicized from the Latin, (8.) and signifies " act of." 3. (1.) Undersell is a simple derivative word, (2.) spelled orthographic- ally, u n d e r s e 11 , (3.) phonically, iin-der-sel, and (4.) etymologically, under -{-sell. (5.) Under, the prefix, (6.) is an etymon, (7.) Anglicized from the A.-S., (8.) and signifies "beneath." (5.) Sell, the root, (6.) is a separable word, (7.) Anglicized from the A.-S., (8.) and signifies "exchange." 3. (1.) Convention is a simple derivative word, (2.) spelled orthograph- ically, convention, (3.) phonically, kon-ven-shun, and (4.) etymologic- ally, con-J-vent-f-ion. (5.) Con, the prefix, (6.) is an etymon, (7.) Anglicized from the Latin, (8.) and signifies "together." (5.) Vent, the root, (6.) is an etymon, (7.) unanglicized from the Latin, (8.) and signifies "come." (5.) Ion, the suffix, (6.) is an etymon, (7.) Anglicized from the Latin, (8.) and signifies "act of." 4. (1.) Moonbeam is a compound primitive word, (2.) spelled ortho- graphically, moonbeam, (3.) phonically, moon-bem, and (4.) etymologic- ally, moon-j-beam. (5.) Moon, the first root, (6.) is a separable word, (7.) Anglicized from the A.-S., (8.) and signifies "satellite." (5.) Beam, the second root, (6.) is a separable word, (7.) Anglicized from the A.-S., (8.) and signifies "parallel rays." 5. (1.) Ominous is a simple primitive word, (2.) spelled orthographically, ominous, (3.) phonically, om-in-iis, and (4.) etymologically, omin-j-ous. (5.) Omin, the root, (6.) is an etymon, (7.) unanglicized from the Latin, (8.) and signifies "omen." (5.) Ous, the suffix, (6.) is an etymon, (7.) unangli- cized from the Latin, (8.) and signifies "full of." 6. (1.) Monkish is a simple derivative word, (2.) spelled orthographic- ally, monkish, (3.) phonically, milnk-ish, and (4.) etymologically, monk -(-ish. (5.) Monk, the root, (6.) is a separable word, ("7.) Anglicized from the Greek, (8.) and signifies "alone." (5.) Ish, the suffix, (6.) is an etymon, (7.) Anglicized from the A.-S., (8.) and signifies "like." WORK-BOOK OF ETYMOLOGY. ggl Prog. XXIX. — 1. — is a 1. Simple 1. Primitive word, 2. Spelled Orthograph- 2. Compound 2. Derivative, icaUy , 3. Phonically , and 4. EtymologicaUy 1 1 . 5. the 1. Prefix, 2. Root, 3. Suffix, 6. Is an 1. Etymon, 7. — 1- Anglicized from the 1. Greek, 8. And signifies — . 2. Separable word, 2. Unanglicized, 2. Latin, 3. Anglo-Saxon, 1. Cellule is a simple derivative word, spelled ortliograpMcally, cellule, phonically, sel-ul, and etymologically, cell-|-ule. Cell, the root, is a sepa- rable word, Anglicized from the Latin, and signifies "bag." Ule, tlie suf- fix, is an etymon, Anglicized from the Latin, and signifies "little." 2. Thermal is a simple primitive word, spelled orthographically, ther- mal, phonically, ther-miil, and etymologically, therm-j-al. Therm, the root, is an etymon, unanglicized from the Greek, and signifies "heat." Al, the suflQx, is an etymon, unanglicized from the Latin, and signifies "of." 3. Summon is a simple primitive word, spelled orthograjohically, sum- mon, phonically, sum-on, and etymologically, sum-|-mon. Sum\ the pre- fix, is an etymon, Anglicized from the Latin, and signifies "under." Mon, the root, is an etymon, unanglicized from the Latin, and signifies "warn." 4. Sacrament is a simple primitive word, spelled orthographically, sacrament, phonically, sak-ra-ment, and etymologically, sacra-]-ment. Sacra, the root, is an etymon, unanglicized from the Latin, and signifies "sacred." Ment, the suffix, is an etymon, Anglicized from the Latin, and signifies "that which." 5. Review is a simple derivative word, spelled orthographically, re- view, phonically, re-vu, etymologically, re-j-view. Re, the prefix, is an etymon, Anglicized from the Latin, and signifies " again." View, the root, is a separable word, Anglicized from the Latin, and signifies " see." 6. Pass-word is a compound primitive word, spelled orthographically, pass -word, phonically, pas-wurd, and etymologically, pass+word. Pass, the first root, is a separable word, Anglicized from the Latin, and sig- nifies "go." Word, the second root, is a separable word, Anglicized from the A.-S., and signifies "the sign of an idea." 7. Religion is a simple primitive word, spelled orthographically, relig- ion, phonically, re-lij-un, and etymologically, re-f-lig+ion. Re, the prefix, is an etymon, Anglicized from the Latin, and signifies "again." Lig, the root, is an etymon, unanglicized from the Latin, and signifies "bind." Ion, the suffix, is an etymon, Anglicized from the Latin, and sig- nifies "act of." ^ Sum = suh. 282 THE SYSTEM METHOD. Prog. XXX.— 'Written Synthesisfrom Mem'ry (Mental Tree). Submission ^under Latin Anglicized, Latin unanglicized verb, act of noun Anglicized. Return =again Latin Anglicized, regular Anglicized verb. Arrival =to Latin Anglicized, Latin unanglicized noun, act of noun Anglicized. Condescend =witli Latin Anglicized, down Latin unanglicized, Latin un- anglicized verb. Admonish =to Latin Anglicized, Latin unanglicized verb, make verb unanglicized. Alleviate =to Latin Anglicized, adjective adjunct root, make verb un- anglicized. Punctuation =Latin unanglicized noun, make verb unanglicized, act of noun Anglicized. Renevfal =again Latin Anglicized, adjective adjunct root, act of noun Anglicized. Confusion =togetlier Latin Anglicized, Latin unanglicized verb, act of noun Anglicized. Deciduous =down Latin unanglicized, Latin unanglicized verb, quality of adjective unanglicized. Suspicion ^ =under Latin Anglicized, Latin unanglicized verb, act of noun Anglicized. Dropping :=regular Anglicized verb, continuing verb Anglicized. Doubtful =common Anglicized noun, full of adjective Anglicized. Epitaph =upon Greek unanglicized, Greek unanglicized noun. Dissatisfy =not Latin Anglicized, adverb adjunct root, make verb unan- glicized. Enthusiast' =in Greek unanglicized, Greek unanglicized noun, one vrho noun Anglicized. Religion =again Latin Anglicized, Latin unanglicized verb, act of noun Anglicized. Consignee =together Latin Anglicized, regular Anglicized verb, to "whom noun Anglicized. Martyrdom =common Anglicized noun, state of noun Anglicized. Admirable =to Latin Anglicized, Latin unanglicized verb, worthy of ad- jective Anglicized. Chatter =regular Anglicized verb, repeatedly verb Anglicized. Testament =Latin unanglicized noun, that which noun Anglicized. Rusticate =Latin unanglicized noun, like adjective unanglicized, make verb unanglicized. Squaloid =Latin unanglicized noun, like adjective unanglicized. Prismoid =Greek unanglicized noun, like noun Anglicized. ^For explanation of this synthesizing process, see Work-Book No. 17, Prog. XTV"., page 213. ^Sus^^aub. ^Ast:=iHt. WORK-BOOK OF ETYMOLOGY. 283 Prog. XXXI : /. AdJuncf-\-Base 2. Adjunct 3. Base 4. Etymological -\-Suffix. from. from. Meaning. Demagogue Philanthropy Monarchy Fratricide Theocracy Agriculture Orthodox Aqueduct Biennial Satisfaction Auriferous Fortification Cruciform Oviform Misogamy Monogamy Polygamy Hexagon Chirography Ethnography Geography Lithography Telegraphy Equilateral Monologue Zoology Necromancy Barometer Micrometer Monomial Astronomy Audiphone Telephone Microscope Telescope Polytechnic Monotheism Polytheism Archetype Stereotype Carnivorous a. Greek. a. Greek. b. Latin. b. Latin. c. Anglo-Saxon, c. Anglo-Saxon. =dema-|-gogue, Greek, Greek, lead people. =phil-|-anthrop4-y, Greek, Greek, state of man love. =mon-l-arch-(--y, Greek, Greek, state of rule one. =fratri-|-cide, Latin, Latin, kill brother. =theo4-crac-l-y, Greek, Greek, state of strength God. :=agri -j-cult-f-ure, Latin, Latin, state of till field. =ortho-|-dox, Greek, Greek, doctrine straight. =aque-|-duct, Latin, Latin, lead water. =bi-f enni-|-al, Latin, Latin, of year tvi^o. :=satis-l-fact-|-ion, Latin, Latin, act of make enough. =auri-l-fer4-ous, Latin, Latin, quality of bear gold. =forti+fic-}-at+ion, Latin, Latin, act of make strong. =cruci+form, Latin, Latin, form of cross, ^ovi+form, Latin, Latin, form of egg. :=miso-|-gam+y, Greek, Greek, state of marry hate. :=mono-|-gam-[-y, Greek, Greek, state of marry once. =poly-]-gam-)-y, Latin, Greek, state of marry many. =hexa-)-gon, Greek, Greek, angle six. =chiro-|-graph+y, Greek, Greek, state of write with hand. =ethno-|-graph-}-y, Greek, Greek, state of write of nation. =geo+graph+y, =litho+graph-|-y, =tele-|-graph-(-y, =equi+later-|-al, :=mono-(-logue, =zoo+log-|-y, =necro-l-manc-}-y, =:baro+meter, :=micro-]-meter, =mo-|-nomi-)-al, =astro-j-nom-|-y, :=audi-|-phone, =tele-l-phone, =micro-)-scope, =tele+scope, =poly-[-techn-|-ic, =mono-|-the-|-ism, =poly-|-the-)-ism, =arche-|-type, =stereo-]-type, =carni+vor + ous. Greek, Greek, state of write of earth. Greek, Greek, state of write in stone, Greek, Greek, state of write far off. Latin, Latin, of side equal. Greek, Greek, speech alone. Greek, Greek, state of speech animal. Greek, Greek, state of divination dead. Greek, Greek, measure weight. Greek, Greek, measure small. Greek, Greek, of division alone. Greek, Greek, state of law of star. Latin, Greek, sound hear. Greek, Greek, sound far off. Greek, Greek, see small. Greek, Greek, see far off. Greek, Greek, of art many. Greek, Greek, principles of God one. Greek, Greek, principles of god many. Greek, Greek, pattern beginning. Greek, Greek, pattern solid. Latin, Latin, quality of devour flesh. 284 THE 8YHTEM METHOD Prog. XXXII : /. Adjuncf+Base 2. Adjunct 3. +Suffix. from. a. Qrc(!k. a. Gro hand. Appendix B— List of Irregular Verbs. Indicatii ves. Participles. IMI'KKFUCT. rmu'Ecr. iMi'Kiir'Kcri'. Pehfect. Abid(! abode abiding abode Airi was being been Arises aroHe arising arisen Awak(', Y. awoke awaking awoice Bear bore, bare bearing born Bear bore, bare bearing borne Beat beat beating beaten, beat Beget begot begetting l)cgo1,t(!n, begot Begiu began beginning 1)1 'gun Behold belHsld beholding l)(!il(!](l Bend, r. iti'rit bending ))cnt Bereave, r. bereft bereaving bereft Beseieli, r. besought besceeliing besought Beset beset besetting beset Bet, r. bet betting bet Bid bade, bid bidding bidden, bid Bind bound binding bnlllid Bite bit biting bit,l,(:n, bit Bleed bled bleeding bled Blend, r. blent blending blent Blow blew blowing blown Break broke breaking br(;j{<;n Breed bred. breeding bred Bring bi'ought bringing brouglit Build, r. built bnililing built Burn, r. burnt burning burnt Burst, r. bui'st bursting burst Buy bought buying bought Can eould Cast cast casting cast Catch, r. caught catching caught Chide chid chiding chidden, ehid Choose chose choosing chosen Cleave, r. clave cleaving cleaved Cleave cleft, clove, clave cleaving cleft, cloven CUng clung clinging clung Clothe, r. clad clothing clad Come came coming come Cost cost costing cost Creep crept creeping crejjt Crow, r. crew crowing crowed Cut cut cutting cut Dare, r. durst daring dared Deal, r. dealt dealing dealt Dig, r. dug digging dug Do did doing done Draw drew drawing drawn [285] 286 THE SYSTEM METHOD. Dream, r. dreamt dreaming dreamt Dress, r. di'est dressing di-est Drink di'ank drinking di-unk, drank Drive di'ove driving driven Dwell, r. dwelt dwelling dwelt Eat ate, eat eating eaten Engrave, r. engraved engraving engraven FaU feU falling fallen Feed fed feeding fed Feel felt feeling felt Fight fought fighting fought Find found finding found Flee fled fleeing fled FUng flung flinging flung Fly flew flying flown Forbear forbore forbearing forborne Forget forgot forgetting forgotten, forgot Forsake forsook forsaking forsaken Freeze, r. froze freezing frozen Geld, r. gelt gelding gelt Get got getting gotten Gild, r. gilt gilding gilt Gird, r. girt girding girt Give gave giving given Go went going gone Grave, r. graved graving graven Grind ground grinding ground Grow grew growing grown Hang, r. hung hanging hung Have had having had Hear heard hearing heard Heat, r. heat heating heat Heave, r. hove heaving hoven Hew, r. hewed hewing hewn Hide hid hiding hidden Hit hit hitting hit Hold held holding held Hurt hurt hurting hurt Keep kept keeping kept Kneel, r. knelt kneeling knelt Knit, r. knit knitting knit Know knew knowing known Lade, r. laded lading laden Lay, r. laid laying laid Lead led leading led Lean, r. leant leaning leant Leap, r. leapt leaping leapt Learn, r. learnt learning learnt Leave left leaving left Lend lent lending lent Let let letting let Lie lay lying lain Light, r. Ut hghting lit Lose lost losing lost Make made making made May might Mean meant meaning meant Meet met meeting met WORK-BOOK OF ETYMOLOGY. 28T Melt, r. melted melting molten Mow, r. mowed mowing mown Must Ought Pass, r. past passing past Pay, r. paid paying paid Pen, r. pent penning pent Plead, r. pled pleading pled Prove, r. proved proving proven Put put putting put Quit, r. quit quitting quit Rap, r. rapt rapping rapt Read read reading read Reave, r. reft reaving reft Rend rent rending rent Rid rid ridding rid Ride rode riding ridden Ring rung, rang ringing rung Rise rose rising risen Rive, r. rived riving riven Run ran running run Saw, r. sawed sawing sawn Say said saying said See saw seeing seen Seek sought seeking sought Seethe, r. sod seething sodden Sell sold selling sold Send sent sending sent Set set setting set Shake shook shaking shaken ShaU should Shape, r. shaped shaping shapen Shave, r. shaved shaving shaven Shear, r. shore shearing shorn Shed shed shedding shed Shine, r. shone shining shone Shoe shod shoeing shod Shoot shot shooting shot Show, r. showed showing shown Shred shred shredding shred Shrink shrank, shrunk shrinking shrunk, shrunken Shut shut shutting shut Sing sang, sung singing sung Sink sank, sunk sinking sunk Sit sat sitting sat Slay slew slaying slain Sleep slept sleeping slept SUde shd sliding sUdden, sUd SHng slung, slang slinging slung Shnk slunk slinking slunk Slit, r. slit sUtting sUt SmeU, r. smelt smelling smelt Smite smote smiting smitten, smit Sow, r. sowed sowing sown Speak spoke, spake speaking spoken Speed, r. sped speeding sped Spell, r. spelt spelling spelt Spend spent spending spent 288 THE SYSTEM METROB. Spill, r. spilt spilling spilt Spin spun, span si^inning spun Spit spit, spat spitting spit, spitten Split split splitting spht Spoil, r. spoilt spoiling spoilt Spread spread sj^reading spread Spring sprang, sprung springing sprung Stand stood standing stood Stave, r. stove staving stove Stay, r. staid staying staid Steal stole steahng stolen Stick stuck sticldng stuck Sting stung stinging stung Stink stunk, stank stinking stunk Strew, r. strewed strewing strewn Stride strode striding stridden Strike struck striking stricken, struck String, r. strung stringing strung Strive, r. strove striving striven Strow, r. strowed strowing strown Swear swore, sware swearing sworn Sweat, r. sweat sweating sweat Sweep swept sweeping swept SweU, r. swelled swelling swollen Swim swam, swum swimming swum Swing swung swinging swung Take took taking taken Teach taught teaching taught Tear tore, tare tearing torn TeU told telling told Think thought thinking thought Thrive, r. throve thriving thriven Throw, r. threw throwing thrown Thrust thrust thrusting thrust Tread trod treading trodden, trod Wake, r. woke waking woke Wax, r. waxed waxing waxen Wear wore weai'ing worn Weave, r. wove weaving woven Wed, r. wed wedding wed Weep wept weeping wept Wet, r. wet wetting wet Whet, r. whet whetting whet wm would Win won winning won Wind, r. wound winding wound Wis wist Work, r. wrought working wrought Wreathe, r. wreathed wreathing wreathen Wring, r. wrung •RTinging wi-ung Write wi-ote writing wi'itten Appendix C— Rules for Spelling. \ The rules for spelling, having been already once printed, are not here repeated. See Appendix C, Work-Book No. 17, page 195. Appendix D -Number, Geijder, aijd Case. The discussion of Number, Gender, and Case has already been given. See Ap- pendix D, Work-Book No. 17, page 236. 19 [289] Appendix E- Greek, Latin, ^A Aijglo-Saxoij ^ IF IF I x: E s. GREEK PREFIXES. FOR-IVI. ]ME^4k.]SrilSrGr. OP PREFIX. OF RADICAJL. A = without, not. a + pathy = an + onymous = an + omalous = AMPHi = around, double. amphi -j- theater = amphi + bious = amphi + logy = ANA = through, up, again. ana + lyze = ana + tomy = Ana + baptist = ANTi = against, opposite. anti + christian = anti + podes = ant -\- arctic = APO = away, from. apo + stasy = apo + stle = ap + helion = CATA = down, throughout. cata + rrh = cata + logue = cat + echism = cat + holic = DiA = through, apart. dia + meter dia + logue di + seresis = two. dis + syllable di + phthong [390] OF RADICAL. OF PREFIX. feeling without name without regular not theater around life double speech double loose through cut up baptize again Christ against feet opposite arctic opposite stand away send away sun from flow down speech throughout sound down whole throughout measure through speech apart take apart take together two vocal two WORK-BOOK OF ETYMOLOGY. 291 EC = out. ec + clesiastical ex + odus e + nervate EN = in. en + demic em + phasis el + lipsis EPi = upon. epi + demic ep + hemeral HYPER = over. hyper + critical HYPO = under. hypo + crite byp + hen META = beyond, after meta + carpus met + onymy met + hod PARA = at side para + dox par + helion PERI = around. peri + osteum PRO = before. pro + gram SYN = witli, together. syn + chronous syn + agogue sy + stem syl + lable syl + logism sym + pathy call way nerve people show leave people day critical play one wrist name way opinion sun bone write time lead stand take speech feeling out out out in in in upon upon under under beyond after after at side at side around before with together together with together with LA TIN PREFIXES. F O R nvE . OF PREFIX. AB =from, away. ab -f ab + a + abs ■ 4- OP KADICAIi. solve jeet vert tain m: E! A. i>r I N" G- . OP RADICAL. OP PREFIX. loose throw turn stretch from away away from 292 THE SYSTEM METHOD. AD = to. ad + vocate = call to a + vow = vow to ac + Crete = grow to af + fix = fix to ag + gregate = flock to al + lude = play to am + munition = fortify to an + nounce = report to ap + plaud = clap to ar + rive = reacb to as + sist = stand to at + tain = stretch to AM = around. am + putate = cut around amb + ition = go around ambi + dextrous = hand around an + helation = breath around ANTE = before ante + diluvian = flood before anti + cipate = take before Bi = two, twici bi + ped = feet two bi + gamy = marry twice bis + cuit = cook twice ciRCUM = around. circum + vent = come around circu + it = go around CON = with, together. con + nect = tie together con + nate = born with CO + equal = equal with col + lect = gather together com + press = press together cor + rugate = wrinkle together CONTRA = against. contra + diet = speak against contro + vert = turn against counter + act = act against DE =: down, from. de + press = press down de + fleet = bend from Dis = apart, not. dis + pel = drive apart WORK-BOOK OF ETYMOLOGY. 293 dis + please di + verge di + stant dif + fuse dif + ficult EN = in, not. en + noble en + emy em + power EX = out. ex + elude e + dentate ef + fluent EXTRA = beyond. extra + mural IN = in, not. in + vade in + efficient ig + noble il + legal il - + lumine im + merse im + material ir 4- resolute ir -f radiate INTER = between. inter + cede intel + lect rNTKO = into. intro + duce HON = not. non + sense OB = against, out. Ob + ject Ob + literate o + mit oc + cur of + fend op + pose PER = through, thoroughly. per + vade per + plex pel + lucid pil + grim POST = after. post + pone please bend stand pour make noble friend power shut teeth flow wall lead not apart apart apart not in not in out out out beyond go m efficient not noble not legal not lighten in dip in material not resolute not shine in go between select between into not throw against letter out send out run against strike against place against go through tangle thoroughly clear thoroughly territory through place after 294 THE SYSTEM METHOD. PRE = before. pre + diet PRO = forth, out, foi pro + pose pro + long pro + consul pol + lute pru + dent pur + pose RE = hack, again. re + pel re + vive red + eem red + olent KETRO = backwards. retro + spect SE = aside, apart. se + gregate se + cede sed + ition SUB = under. sub + merge sue + cumb suf + fer sug + gest sum + mon sup + pose sur + reptitious sus + tain su + spect SUPER = over. super + natural supra + orbital sur + vey sur + vive TRANS = across. trans + fer tran + scribe tra + montane tra + verse tres + pass before ^-$4^«-^®; place forth lengthen out consul for wash out see forth place forth drive back live again buy back smell again look backwards flock aside go apart go aside dip under lie down under bear under carry under warn under place under creep under stretch under see under nature over orbit over see over live over bear across write across mountain across turn across step across r:s—r WORK-BOOK OF ETYMOLOGY. 295 ANGLO-SAXON PREFIXES. in o E. M: . OP PKEFIX. OF RADICAL. A = on, at, to, from. a + foot a + side a + shore a + wake BE = by, he. be + side be + tliink be + cause be + gnaw EN = in. en + dear en + rage em + blaze em + balm PORE = before fore + tell fore + see FOR = through, again.it. for + bear for + swear fore + doom fore + go tN = in. in + come in + born im + bitter MIS = wrong. mis + speak mis + give NE = not. ne + uter n + aught n + either OFF r=:from. off + set of + fal ON = upon, against. on + set on + slaught m: E ^ ]sr I T>r G- . OF RADICAL. OF PREFIX. foot on side at shore to wake from side by think be cause by gnaw be dear in rage in blaze in balm in tell before see before bear through swear against doom through go against come in bear in bite in give either aught either set fall set dash wrong wrong not not not from from upon against 296 THE 8T8TEM METHOD, OUT = beyond, over , more. out + live out + general out + flatter OVER = too much. beyond, out&r over + reach over + eat over + coat s = make s + melt s + pike s + pread 8 + crub UN = not, back un + clean un + wind un + true un + do UNDER = beneath. under + wood under + sell under + clothes under + ling WITH = against, back. with + stand with + hold GREEK m o R m: . OF RADICAL. OP SUFFIX. AC = of, belonging to. demoni + ac elegi + ac BAN = of, pertaining to. Tartar -|- ean Epicur + ean IC = one who, of, things of cyn + ic gnost + ic spher + ic Satan + ic acoust + ics phys + ics = general over = flatter more = reach beyond = eat too much = coat outer = melt make peg make = broad make = rub make = clean not = wind back = true not do back = wood beneath sell beneath = clothes beneath = person beneath = stand against hold back SUEFIXES. ivi E A. N- 1 isr G- , OF SUFFIX. OP RADICAL of demon = belonging to elegy of Tartarus = pertaining to Epicurus = one wno dog-like = one who know of sphere of Satan = things of sound = things of nature WORK-BOOK OF ETYMOLOGY. 297 mathemat + ics eth + ics ISE = to make. apolog + ise organ + ize ISM = state of, principles of. heathen + ism magnet + ism enthusi + asm 1ST = o)ie who. theor + ist enthusi + ast OID =: like. spher + oid aster + oid variol + oid crystall + oid things of things of quantity custom to make to make apology organ state of principles of state of heathen magnet god in one who one who theory god in like like like like sphere star small-pox crystal LATIN SUFFIXES. form: m: E .A. ]sr I jsr G- . OF RADICAL OF SUFFIX. OF SUFFIX. OF RADICAL. A = two or more. dram + a = two or more act gener + a = two or more genus ABLE = that can be woi-thy of trace + able = that can be traced ami + able = worthy of loYe sta + ble = that can stand divis + ible = that can be divided cred + ible = worthy of belief AC-T = state (■- = y) of{= ac). contum + acy = state of puff obstin + acy = state of stand again AGE = act of. state of, collection of, pay for. marri + age = act of marry parent + age = state of parent foli + age = collection of leaves percent + age = collection of hundredths cart + age = pay for cart AL = of, act /• leg + al = of law ment + al = of mind remov + al = act of remove recit + al = act of recite 298 THE SYSTEM METHOD. AN = of, one who, like. urb + an partis + an vill + ain urb + ane agrari + an ANCE = quality of, , act of, stc vigil + ance convey + ance brilli + anc-y provid + ence conflu + ence convalesc ■ + ence clem + ency ANT = one whc ', ing, , that whi assist + ant ten + ant dorm + ant absorb + ent convalesc + ent ard + ent AR = one who, of 11 + ar angul + ar sol + ar ATE = quality of, one icho, th rose + ate deleg + ate certiflc + ate liber + ate CLE = little. vesi + cle arti + cle animal + cule mole + cule EB = to whom. grant + ee EER = one who , that which. engin + eer brigad + ier chandel + ier EL = little, like. kern + el bow + el mors + el to make. of city like party one who village like city of land quality of vigil act of convey state of shine quality of foresee act of flow together state of grow well state of mild one who assists one who holds ing sleep that which absorb ing grow well ing burn one who lies of angle of p sun quality of rose one who chosen that which certify to make free little vessel little joint little animal little mass to whom grant one who engine one who brigade that which candle little corn little intestine little bite WOBK-BOOK OF ETYMOLOGY. 299 cru frai + + el 1 tab + le ER = one ifiTio, that lo/iich. build + er fend er ESS = female. lion + ess heir + ess songstr ET = little. + ess clos + et eagl stream + + et let branch + let FT = to make. magni forti + + fy ic = of. aquat + ic gener med + + ic ic-al naut + ic-al ICE = tMng. not + ice just artif + + ice ice ID = quality of, thing that. stup + id flu + id rLE = of, that : can be. puer frag mE = o/, like. + + ile ile mar + ine viper sacchar + + ine ine ION = act of. opin rebell donat + + + ion ion ion vis ISE = to make. advert + + ion ise poet legal + + ise ize like blood like break like board one who build that which strike off female lion female heir female singer little close little eagle little stream little branch to make great to make strong of water of genus of healing of sailor thing mark thing right thing make art quality of stupor thing that flow of boy that can be broken of sea like viper like sugar act of think act of war again act of make gift act of see to make advert to make verse to make legal 300 THE SYSTEM METHOD. ISH := to Tnake. publ + ish = establ -{- ieli = ISM = state of, idiom, theory of. fanatic + ism = Gallic -f- ism = Wesleyan + Jsm = De -(- ism = iTE = one who, that which, like, to make. Campbell + ite Huss + ite gran + Ite fin + ite un + ite ivjjj = quality of, one who, tl fest + ive delus + ive capt + ive mot + ive IX ^= female. executr + ix LENT =full of. opu + lent fraudu + lent MENT = state of, act of, that excite + ment conceal + ment gar + ment ali + ment speci + men acu + men MONY = that which, state of. testi -\- mony matri -|- mony OR = one who, quality of, that which. act + or eplend + or mot + or fav + or OSE —full of. acet + ose joe + ose ous =full of, quality of. odor -\- ous enorm + ous to make public to make stand state of fanatic idiom France theory of Wesley theory of God one who Campbell-s one who Huss-es that which grain like end to make one quality of feast quality of delude one who taken that which move female executor full of wealth full of fraud state of excite act of conceal that which protect that which nourish that which to see state of sharp that which witness state of mother one who acts quality of shine that which moves that which befriends full of acid full of joke full of odor quality of without rule WORK-BOOK OF ETYMOLOGY. 301 KY = quality c f, one ^Dho, place where, state Of • epistola + ry = quality of epistle adversa + ry = one who advert avia + ry = place where bird accesso + ry = that which accede derogato + ry = quality of degrading observato + ry = place where observe distille + ly = place where drop slave + ry = state of slave "thieve + ry = practice of thief soldie + ry = aggregate of soldiers fer + ry = place where bear TUDE = state c f. multi + tude = state of many pleni + tude = state of plenty quia + tude = state of quiet TY = state of. liber + ty = state of free densi + ty = state of dense ULE = little. glob + ule = little globe gran + ule = little grain plum + ule = little feather r + ule = little model UBE = state of, that which, act of sut + ure = state of sewed post + ure = state of placed ten + ure = that which holds ere at + ure = that which created fract + ure = act of break depart + ure = act of depart UTB = like, to make. ac + ute = like needle min + ute = like small sal + ute = to make health Y = state of. famil + y = state of servant miser + y = state of wretched savager + y = state of more savage anarch + y = state of without rule JG'?^>. V% / 7 "^ ^^oZ/ CHICAGO, ILL.: SJ^_0f/aSH1^^^^ Leakxer's Work-Tree Company. 18S6. COPYRIGHT, 1886, BY ISAAC ELDRIDGE WILSON. ALL RIGHTS RESERVED. Any infringement will be prosecuted to the full extent of the law. n DEMONSTRATION I. FACT MATTER: a. Name. — That combination of letters, syllables, or words by which an object is distinguished, or known; as, John, boy, box, we, you, senator, mister, belief. b. Noun. — A name that has or may have an affix added; as, John-S, boy-ish, one-s, box-es. C. Pronoun. — A name that has not and will not take an affix; as, we, yon, he, they, I. The, pronouns are I, we; thou, you; he, she, it, tliey; who, whieh, what, that, and as; and compounds of these. d. Other Differences. — 1. The noun indicates repetition by an added s or es; the pronoun indicates repetition by an arbitrary word; as, — Noun: boy, boys; box, boxes; sir, sirs; Lucas, Lucases. Pronoun: I, we; thou, you; he, they; she, they; it, they. 2. The noun indicates possession by an added 's or by the (') alone; the pronoun indicates possession by changing its form; as, — Noun: boy, boy's; girls, girls'; senator, senator's; boys' hats. Pronoun: 1, my; thou, thy; you, your; he, his; they, their; who, whose. e. Common Noun. — One that classifies its object; as, boy, subvocal, verb, house, animal, plant, letters, words. f. Proper Noun. — One that individualizes its object; as, John, Englishmen, Oreeks, Caesars, Plato, France, Ohio. g. GRAMMATICAL! NuMBER. — That Structural change in names that indicates whether the thing named is taken more than once, i. e., repeated. h. Singular Number. — That which represents the thing named as not repeated; as, man, elass, water, checker, Milton, half, people, seven, couple, grief. i. Plural Number. — That which represents the thing named as being repeated; as, men, classes, Waters, checkers, Miltons, halves, peoples, sevens, couples, griefs. OBSER VATIONS : a. Number Based on Structure. — The statements that, (1.) "Bel- lows, scissors, shears, tongs, have no singular forms;" (2.) "The plural num- [405] 406 THE SYSTEM METHOD. ber denotes more than one; as, apples, boys, borses," — these statements are contradictory, since bellows, scissors, sJiears, and tongs each denote but one instrument. This mistake must be avoided: it will bring confusion. It lies in confounding the principles of form and signification. These nouns are in the plural number, not because they denote more than one instrument, but because the name in each case originated in a part, which part is re- peated to form the whole. Scissors is not in the plural number because it means more than one pair of scissors, more than one whole, but because the part named, scissor, is taken twice. In scissors there are two parts; one ^?iXl=scissor, which word is not now used; two parts=scmo?'s. Observe that scissors-=scissor-\-s. Number is properly defined to be a structural change signifying repetition of some sort — of whole, part, particle, action, or substance. b. The Thing Naked. — The thing named and repeated must belong to one of the following classes: (1.) Material objects; (2.) Immaterial ob- jects. Examples are, — 1. Material Objects: boy, army, shears, victuals, foods, gold, teas, tar, snow, scissors, etc. 2. Immaterial Objects: sweetness, safety, wickedness, peace, health, re- sistance, succession, etc. C. Part and Whole. — Names of wholes are now singular, now plural; but, lungs and botcels excepted, names of parts repeated to form the object of thought are usually plural. Let those names following, in which a whole is the thing named, be compared with those in which the thing named is a part or particle: — A WJiole Named: boys, armies, boxes, spoonfuls, companies, etc. A Part or Particle Named: tongs, shears, scissors, pincers, tweezers, nip- pers, news, nuptials, vitals, ethics, goods, movables, rickets, oats, em- bers, ashes, odds, measles, suds, etc. d. Substance. — Names of substance exhibited but once are, of course, singular; but, if the substance is exhibited more than once, the name is plural. Observe that the plural number always indicates some sort of rep- etition, as of whole, part, particle, substance, or action. Singulars: gold, tea, coffee, snow, furniture, jeans, molasses, cider, matter, tar, clay, mortar, flesh, gravel, brick, stone, trout, mackerel, fish, fowl, quail, venison, etc. Plurals: golds, teas, coffees, furnitures, snows, j causes, molasses, etc. e. Action. — The following nouns are names of actions: goings, doings, treatment, thanks, ascent, bellows, skimmings, proceeds, creations, begin- nings, tidings, blessings, mumps, wages, etc. f. References. — For other matters pertaining to number in grammar, refer to Demonstration XXVIII.; also to Webster's or Worcester's Un- abridged Dictionary. g. Tests by which to distinguish between nouns and adjectives or pro- nouns: 1. Only nouns take the added s as a plural sign; 2. Neither the pro- THE WORDS CLASSIFIED. 407 noun nor the adjective takes the (') as a possessive sign. (See Demonstra- tion XVII., i, j, k, 1.) Prog. I: /. Noun. 2. Current. 3. The Thing Named. a. Common. a. Singular. a. Material. b. Proper. b. Plural. b. Immaterial. c. Equivocal. Note to Teacher.— Tl^& student's work in classifying, by this program, the particulars given below is here omitted, as it will be given in full in " The System Method," No. 20, which begins on page 577. Both the programs and the particulars to be worked up by the programs will be given in this, " The System Method," No. 6; but the student's work itself will in all cases be omitted in this, No. 6, to be given in Work-Book, No. 20. FABTICULABS : The hills ^ and the valleys ^ are green. Our botany ^ is all names,* not powers-^ The Enghsh are a nation ^ of humorists.' Whatever surly sweetness ^ possession ^ can give, is tasted in England i° to the dregs.^^ A person's ^^ manners ^^ not infrequently indicate his morals.^* November ^^ is the month i^ when the deer i' sheds its horns.^^ That coach-and-four 19 is its master's 2" sole remaining property.^i The jury^a were not unanimous. The pubhc ^^ are respectfuUy invited to attend. We are to speak in public. 2* The spectacles 25 set them unhappily wrong. A coach-and-six ^^^ is, in our time,^' never seen, except as a part^^ of some pageant.29 We expect the dancing master ^o to teach our children ^^ manners.^^ Where are the Platoes ^^ and Aristotles 3* of modern times ? ^^ All the Maries ^^ in this school ^' are good girls. ^^ The Germans '^ are an industrious people.*" Six*i and three*'* make nine.*^ The army** of the queen *5 mean to seize us. Mankind *^ have, in aU ages,*' attached themselves to a few persons. *8 A man *9 comes to measure his greatness ^ by the regrets,^! envies,^^ and hatreds 5* of his competitors.^* His illustrations ^5 are poetry.56 few knew as much about Nature 5' and her subtle manners, ^s or expressed more subtly her goings.^^ It was well worth the pains ^ that have been taken to procure it. His means ^^ are as admirable as his ends.^^ The means "^ are too strong for them. Your manners ^ are always under examination.^' The rich take up something ^^ of the world ^^ into man's ^ life.^9 It is the interest '" of aU that there should be Captain Cooks '^ to voyage round the world; '2 Rosses,'" Franklins,'* Richardsons,'^ and Kanes '^ to find the magnetic and geographic poles." Our politics '^ are disgusting. The sky '^ is a covering*' for a market ^^ and for the cherubim ^^ and seraphim.^^ He loved the tall deer s* as if he were their father. ^^ The heats ^^ of summer®' destroy its beauties.'^ Young people ^^ in this country s" are prone to melancholy.^i The Irish peerage ^2 drew their names ^3 from a playbook.^* Mankind ^5 divides itself into two classes,^^ — benefactors 9' and malefactors.98 Mankind ^s are very superficial. At times ^"0 the summit 1"^ of the high city 1°'^ fld-hed. These news 1"^ be mine. We are a people i*'* yet. The Japanese ^"^ are a very jealous people.i"^ The Esquimaux 'o' have high cheek-bones ^"^ and small noses.i°9 The army^^'' was sadly neglected. The assembly ^ was tumultuous. Whales ^^ are found in companies,^!^ but mostly in pairs.^^* He plays biUiards^^^ ex- cessively. A herd 11^ of buffaloes i" can only be kept in order i^^ by a guard ^^^ of mounted keepers,i2o armed with lances.i^^ The oats 122 ^yere ripe. The scissors ^^^ are sharp. The market ^^i -^^^s filled with goods.i^* He had had no victuals ^-^ for thirty- six hours.12' No parallel 128 can be found in the annals ^^^ of the world.^^" He had none of the coin ^^^ of the country.^^^ It was buried in the ashes.i^^ This summons i^* 408 THE SYSTEM METHOD. he resolved to obey. Let a gallows ^^^ \,q made. Evil news ^^^ rides fast, while good news 1^' baits. When Khea ^^^ heard these news,!^^ she fled. Why do the heathen i^" rage? Christianity ^^^ was formerly propagated among the heathens. ^^^ They had brick 1^^ for stone.^** The bricks ^*^ are fallen down. His affairs ^*^ went on at sixes i*^ and sevens.^*^ We sold two dozen eggs."* Knock them down by the dozens.^^" Many thousands ^^^ were slain in the war. 1^2 We went up five pair^^s of stairs.i^* Several pairs ^^5 of scissors ^^s lay on the shelf.^^' No pains ^^^ is taken. Your pains i^s are registered. Hear these iU news.^^" They hate us youth. ^^^ The fish i^s are in the pond.i*^ There are fishes ^^ that have wings.^^^ We believe in Woman's ^^^ Rights.i*^^ Vested rights,^*^ are awful things. ^^* DEMONSTRATION 11. FACT MATTER— Rules for Spelling Plurals : a. ^RtTLE X. — Nouns ending in a syllable that coalesces with s, in- cluding those in o preceded by a vowel, add s only for the plural ; as,— 1. Tack-s, cake-s, truth-s, puff-s, youth-s, ember-s, eye-s, swarm-s, water-s, tong-s, new-s, coflee-s, woe-s, tea-s, hoe-s, U. S. Grant-s, Can- ada-s, James Gordon Bennett-s, Miss Johnson-s, Sir Isaac Newton-s, thank-s, brothers-in-law, going-s, redcoat-s, getters-up, mump-s, Guy- ot-s. 2. Day-s, valley-s, turkey-s, Thackeray-s, Motley-s, runaway-s, Gyulay-s, Maloy-s, key-s. 3. Folio-s, cameo-s, embryo-s, cuckoo-s, cacao-s, Hindoo-s, imbroglio-s. 4. Fife-s, grief-s, chief-s, safe-s, scarf-s, belief-s, haudkerchief-s, hoof-s, roof-s, turf-s, dwarf-s, brief-s, reef-s, gulf-s, proof-s, serf-s, surf-s, stripe-s, staff-s, mischief-s, relief-s, fief-s, clef-s, oaf-s, waif-s. 5. Dozen-s, hundred-s, seven-s, pair-s, people-s. 6. Spoonful-s, mouthful-s, armful-s, sackful-s. b. ^RuLB XI. — Nouns in a syllable that does not coalesce with s, and nouns in preceded by a consonant, add es for the plural ; as, — 1. Church-es, class-es, Lucas-es, six-es, S. S. Mackintosh-es, the Mas- ter Crumless-es, buzz-es. Governor Morris-es. 2. Negro-es, echo-es, Cicero-es, tomato-es, Plato-es, zero-es, bravo-es, desperado-es. 3. Nouns inf or fe, except those mentioned under Rule I., 4, change f or fe to V, before es is added ; as, leaf, leav-es ; shelv-es, kniv-es, thiev-es, stav-es, wharv-es, elv-es, liv-es, halv-es, wolv-es. 4. Nouns in y after a consonant change y to i before es is added ; as, fly, fli-es ; Henry, Henri-es ; Mari-es, ladi-es, Welbi-es, Sicili-es, Nan- ci-es, compani-es, twenti-es, soliloqui-es, merci-es. 1 These rules are numbered to accord to their numbers as given on page 196. THE WORDS CLASSIFIED 409 0. Exceptions to Rule XI. — Canto-s, quarto-s, solo-s, piano-s, duodecimo-s, halos, tyros, provisos, juntos, octavos, mementos, armadillos, lassos. C. Rule XII. — Letters, signs, and figures substitute the apostrophe (') for the e of es ; as, In 999 there are three 9's ; Casting out the ll's; Mind your p's and q's ; Dot your i's, and cross your fs- Exception. — For a word coalescing with s, and used merely as the name of a word, the best usage also sanctions a plural in s alone; as, "Hence- forth my wooing mind shall be expressed in russet yeas and honest kersey m^%."—8hakspeare. " The novel is full of ohs, bys, whys, alsos, and nos." OBSERVATIONS : a. Compounds, with or without the hyphen, annex the plural sign to the important part ; as, hangers-on, house-tops, getters-up, after-thoughts, forget-me-nots, brothers-in-law, step-fathers, the Stamp-Acts, the George Washingtons. Men-servantfs, knigJits-templars, and ignes-fatui have both parts pluralized. b. Proper nouns of two or more parts, and proper nouns including a title used as a part of the name, annex the plural sign to the last part ; as, O. P. Morton-s, Thomas Y. Stevens-es, Miss Thomson-s, the General Smith-s, the Mrs. R. B. Hayes-es. C. In references including a plural title and names distinguishing be- tween members of that title, the name and title are not to be regarded as one noun. In such expressions as "the Emperors Napoleon and Alexander," "the Misses Julia and Maria Morton" the names Julia \Mortori\ and Maria Morton are to be regarded each as a proper noun in apposition with the noun Misses. So Napoleon and Alexander. d. The following nouns form the plural irregularly : — child, children. louse, lice. ox, oxen, foot, feet. man, men. woman, women, goose, geese. mouse, mice. tooth, teeth. e. Pronouns form the plural arhiirarily : — I, we. thou, you. he, they. she, they. it, they, my, our. thy, your. his, their. her, their. its, their, me, us. thee, you. him, them. her, them. it, them. f. These nouns have two plurals in different significations : — index, indexes, indices. genius, geniuses, genii, penny, pennies, pence. medium, mediums, media, die, dies, dice. phalanx, phalanxes, phalanges, brother, brothers, brethren. stamen, stamens, stamina, vortex, vortexes, vortices. vertex, vertexes, vertices. g. The following nouns and pronouns have twin-like {i. e., common, or equivocal) forms for the singular and the plural: — Nouns : sheep, deer, swine, vermin, hose, fry, gross, neat, grouse, Esquimaux, Japanese, Cyclops, corps, molasses. Pronouns : who, which, what, that, as. 410 THE SYSTEM METHOD. h. For the most part, nouns from foreign languages retain their native plurals. In case of dou-bt here, or in any other question concerning the plural, refer to Webster's or Worcester's Unabridged Dictionary. i. Nouns in the singular number are sometimes used with a plural verb • as. The jury are agreed. Nouns in the plural are likewise some- times used with singular verbs ; as, The neics is bad. (See Demonstration XXVIII.) \. For other matters pertaining to number, see Demonstration XXVIII. k. Test to distinguish between nouns, and adjectives or pronouns: It is the essential character of the plural number to indicate some sort of repetition, as of object, action, part or particle. But the plural is formed by adding s or es to a name, not to a verb, or a connective, or other kind. Hence, that object, action, part or particle, whose plural is formed, must Mve a name sometime, properly or improperly in use. Thus, trees, boxes, oats, and scissors have tree, box, oat, and scissor, of which the last two are not now properly used, or current. But since it is a name that has a plural, it is a name that is pluralized; and the existence of the plural in s or es is proof that the word in question is a noun. Thus we decide that — WJiatever will take the plural sign therein becomes a noun; as, boy-s, other-s, plural-s, better-s, doing-s, etc. Prog- II : ^- Noun. 2. Current. 3. The Thing Named. 4. Rule for Spelh'ng. a. Common. a. Singular. a. Material. a. Rule . b. Proper. b. Plural. b. Immaterial. b. Irregular. c. Equivocal. c. Equivocal. c. Common. Note to Teacher.— Th.e student's vs^ork-book ("The System Method," No. 20) will show how the student is to proceed under the guidance of these programs. The numbers of the proorams in the work-book correspond to the numbers of the Demonstrations here in the grammar (No. 6). PABTICULABS : James i is a good physician ; ''' Henry ^ is a better one.* What is meant by two- thirds ^ of a unit ? « One 'should defer to his superiors.^ In compounds ^ the sign of the plural 1" is added to the important part.^^ Some nouns ^^ have two plurals ^^ of diilerent significations.^* How many twelfths i^ in three fourths ? ^^ Possessives ^^ are formed by adding 's to the nominatives. ^^ Teach me to feel another's ^^ woe.^" Three times ^^ two'^^ are six.^^ They came by sixes 2* and sevens.^^ The Turks ^6 and Greeks 27 are at war.^s The nos^^ have it. They love each other. 3" Each one^i helps aU the others.^^ Once one^^ is one.^* The strife ^^ -^ag between the Whites 3^ and the Blacks.^'' I care not what others ^^ say. One ^^ has to comply with law.*" We were hstening to the warbling of the young ones.*i They came with the Uttle ones.*^ The young one's *3 warbhng M'as sweet. He hstened not at others' ** words.*^ Many hundreds *^ were slain. He matched them in braces, *' or twos.*^ Be courteous to your betters.*^ The snuffers ^° are not less convenient than the tongs.^i There is less science ^^ in ethics ^^ than in mathematics.^* These were real blessings ^5 and merited more than our mere thanks.^^ These orgies ^7 Kome ^^ practiced In the name ^^ of rengion-fi"^ It is proven by odds ^^ of authority ^^ that there THE WORDS CLASSIFIED. 411 is such a disease ®^ as the measles.®^ The rickets ^^ and mumps ^^ are the wages ^^ of bad victuals ^8 and bad birth.^' That molasses™ is better than the news.'i The crowd '^ were again gathered together. The world '^ is round. The world '* are spectators '^^ of your conduct. The crowd '® was large. A few years '''^ is a matter of little consequence.'^ The public '^ is more disposed to censure than to praise. The public 8" are respectfully invited to attend. The jury^^ were not unanimous. A coach-and-six ^2 is, in our time,^^ never seen, except as a part ^^ of some pageant. ^^ Our politics ^^ are disgusting. Mankind ^'' divides itself into two classes, ^* — benefac- tors ^ and malefactors.^" — Emerson.^^ Mankind ^^ are very superficial. — Ben. Frank- lin.^^ We are a people ^* yet. The cardinal ^^ is not my better ^^ in the field. — Shaks- peare.^'^ Henceforth my wooing mind shall be expressed In russet yeas^® and honest kersey nays.^* — Id. His better ^^ does not breathe upon the earth. His means ^^^ are as admirable as his ends.^"'^ — Fmerson.^°^ That tidings i"* came. — Shakspeare.^'^^ The Esquimaux ^"^ have high cheek-bones '•" and small noses.i°8 a herd ^^ of buflaloes ^^^ can only be kept in order '^^ by a guard ^^'-^ of mounted keepers.^ ^ Where are the Platoes^^* and Aristotles i^^ of modern times ? ^^^ Let a gaUows ^^^ be made. — Esther. Vast num- bers ^^^ of sea fowl ^^^ frequent the rocky chffs.'^" His affairs ^^^ went at sixes ^^^ and sevens.^^^ The fish^^* are in the pond. All the Maries ^^^ jq ti^jg school'^® are good girls.^^' Mankind ^^^ have, in all ages, attached themselves to a few persons.'-^ — Emerson. DEMONSTRATION III FACT MATTER: a. Gender. — A distinction of nouns and pronouns having reference to the properties of sex. There are four genders : — b. Masculine Gender. — That which represents the object as exhibit- ing the properties of the male sex; as, he, sir, his, uncle, eraperor. C. Feminine Gender. — That which represents the object as exhibit- ing the properties of the female sex ; as, she, hers, aunt, empress, sister. d. Neuter Gender. — That which represents the object as not exhibit- ing the properties of either sex; as, it, watch, lamp, widedness, sound. e. Common Gender. — That which represents the object as having sex, without distinguishing what properties of sex it has; as, friend, COUSin, teacher, parent, animal, elephant, scholar. OBSERVATIONS : a. Things personified are represented both by common and by proper nouns. Accordingly, nouns that are names of things personified sometimes are, at other times are not, written with a capital letter. There is no 412 THE SYSTEM METHOD. rigid rule observed by the best writers. Sometimes we find, " Tbe vices are hnt virtues there, each named by her victim here." Again we find, "The Vices are but Virtues there," etc., which last the author prefers, as hightening the effect by attaching a full jiersonality to each vice. b. Contrary to rule, proper nouns arising by personification are often written without the capital; as, "For him light labor spread her whole- some store." "When memory recurs to the past, she wanders among tombs." It would strengthen and beautify such sentences to capitalize the proper noun. C. But observe that nothing is to be regarded as ]oersonification in the noun, save what is set forth in such manner and character as to accord, in the idea of sex and personality, with the pronouns I, he, she, thou, and you. If the sentence, "The eye was the first of artists," formed part of a continued discourse, eye would accord in the context, not with he or she, but rather with the neuter pronoun it; hence, by the nature and definition of personification, eye is not here personified; and is to be regarded as neuter, notwithstanding the fact that artists is of the common gender. d. Mark that nouns are not masculine and feminine because the things named are actually, physically, male and female, but because the language exhibits the thing as male or female. Thus, in the sentence below, irook is masculine, not because the thing named is a male, but because the language of the sentence ("built Mm") is the same as if the brook had actually been a male. (Cf. Obs. a. Demonstration IV.) e. Collective nouns in the plural number, and collective nouns convey- ing the idea of unity, are neuter; as, " Several J'M?'^■es [neuter] have been impaneled;" "The Jury [neuter] gave «^s verdict." But the pronoun they, and collective nouns carrying the idea of plurality, determine the gender from the individuals that compose the collection named; as, "Not alone the fair sex [feminine] was marked, but both sexes [common]." "The stars, they [neuter] shall shine forever." f. Small children and inferior animals, though having in reality the properties of real males or real females, are commonly viewed as being without sex. They are, therefore, commonly represented by the neuter pronoun it. The names of beings viewed in this way (that is, as not exhib- iting the properties of sex) are to be regarded as neuter. Examples are: child, swallow, fox, snake, bird, crab, hound, mosquito, deer, etc. g. Not having a pronoun of the third person singular and common gender, in its stead the English employs 7ie, his, him; as, — - Is any among you afflicted, let him pray. Everybody has his faults. Every person should think for himself. The fox ran in his den. THE WORDS CLASSIFIED. 413 But such generic names as fox, if viewed as not exhibiting the properties of sex, are neuter (of. Obs. f); as, "The/oa; ran in its den." h. The masculine and feminine are distinguished in three ways: — 1. By suffixes: God, goddess; actor, actress; duke, duchess; heir, heiress; lion, lioness; testator, testatrix; hero, heroine; bride, bridegroom. 2. By prefixes: male, female; he-goat, she-goat; man-servant, maid-serv- ant. 3. By arlitrary imrds : Brother, sister; nephew, niece; sir, madam; wiz- ard, witch; he, she. Prog, III : ?■ Class of Noun. 2. Number. 3. Gender. 4. Personification. a. Common. a. Singular. a. Masculine. a. Personified. b. Proper. b. Plural. b. Feminine. b. Unpersonifled. c. Neuter. d. Common. PABTICULABS : " Will you come into my parlor? ^ " said the spider ^ to the fly.^ Cicero * was an orator.5 Horses ^ are animals.'' Children ^ should be careful. Dogs^ kiU sheep.^" The fish^^ which you caught is a trout.^^ The bear^^ growled when he saw the hunter.i* He went to the doctor i5 for advice.^'' Susan's i' motherly jg my aunt. ^^ Go to the ant,2o thou sluggard. ^i Flies ^2 are insects.^^ j ga-^ a flock 2* of wild turkeys.^s The hunters 26 came to the den^^ of a tiger.28 Death 29 lifts the vail so that hides a brighter sphere.31 Thou art the man. ^3 Clarence ^3 has been chosen captain.^* Per- adventure, the old dragon 35 is asleep. His being a foreigner ^6 is no disgrace.^T There smiling spring ^^ itg earhest visit 39 paid. Grim-visaged war*" hath smoothed his wrinkled front. Necessity ^ is the mother ^ of invention. Never did Natm-e *3 be- tray the heart that loved her. Tremble, thou earth,''* at the presence of the Lord.** Procrastination *^ is the thief *'' of time. See, the Summer *8 gay With her green chaplet and ambrosial flowers. Droops into palhd autumn. *9 O eloquent, just, and mighty Death ! ^ whom none could advise, thou hast persuaded. O trouble ! ^^ would I had never known thee. Can Time,^^ his flight reversed, restore the hours? The Moon ^3 sees her unwrinkled face reflected bright. Nature ** discloses with a smile the author ^5 of her beauties. How wonderful is Death,^^ — Death and his brother ^^ Sleep ! =^ Eternal Love ^^ doth keep, In his complacent arms. The earth, the air, the deep. For him Ught labor®" spread her wholesome store. ^^ Remembrance ^^ wakes with all her busy train. Shouting Folly ®3 haUs them from her shore. Rehgion ®* veils her eyes. Ocean, ^^ -^tij ^j^g brine on his gray locks, was there. When memory ^^ recurs to the past, she wanders among tombs. Follow the bier of the dead cold year,^^ And make her grave green with tear on tear. 414 THE SYSTEM METHOD. Lost Echo ^^ sits amid the voiceless mountains, And feeds her grief with his remembered lay. AH crimes shall cease, and ancient frauds shall fail. Returning Justice ^^ lift aloft her scale, Peace ^^ o'er the world her olive wand extend. And white-robed Innocence "^ from heaven '^ descend. Truth, ■'2 crushed to earth, shall rise again ; The eternal years of God ''^ are hers ; But error, '^ wounded, writhes in pain. And dies among his worshipers. When Freedom '^ from her mountain bight Unfurled her standard to the air, She tore the azure robe of night. And set the stars of glory there. The sanguine sunrise,'''' with his meteor eyes. And his burning plumes outspread. Leaps on the back of my sailing rack. When the morning star shines dead. The little brook ''^ heard it, and built him a roof. Man's glory heaven ''^ vouchsafes to call her own. Look not on Nature,^" for her name is fatal. Time,^^ my lord, hath a waUet on his back, wherein he puts alms for obhvion. Grim Darkness ^^ furls his leaden shroud. The Sun^^ holds his fiery course through mid-heaven. The moon^* sheds her pale light on that dismal battle scene. The Vices ^^ are but Virtues *^ there, each named by her victim here. The earth ^'^ in its Eden glory must have been a de- lightful sphere. " Oh happy Moon," ^^ the lady said, "Men love thee for thyself." Everybody's thinks for himself. The fox^" ran in his den. The fox^i hid in it& hole. Every one ^^ has his faults. The jury ^^ gave in its verdict. They are friends ^* of mine. James and Henry are my cousins.^^ Mary and Blanche are my cousins.^® James and Blanche are my cousins.^'' They are teachers. ^^ Both women are par- ents.93 The eye ^"^ was the first of artists. I am thy father's spirit. ^^i The dark- ness ^02 waved wider its sable wings. A little child, M3 dear brother Jim,!"* That lightly draws its breath. And feels its life in every limb. What should it know of death? — Wordsworth. The thunder 'o* Winged with red lightning ^"^ and impetuous rage. Perhaps has spent his shafts. — Milton. Love '""^ in my bosom like a bee Doth suck his sweet ; THE WORDS CLASSIFIED. 415 Now with his wings he plays with me, Now with his feet. — Lodge. Thou SuDjios said I, fau- Ught ! And thou enlightened Earth, ^^ so fresh and gay! Ye hiUs ^^^ and dales,i^i ye rivers,^!^ woods, ^'•^ and plains,^^** And ye that live and move, fair creatures,ii^ tell. Tell, if you saw, how came I thus, how here? — Milton. This same child "^ is he who reigns. Upon this. Fancy ^''^ began to bestir herself. DEMONSTRATION IV. FACT MATTER: a. Person. — That distinction of nouns and pronouns which indicates which part of conversation is taken. b. First Person. — That part represented by the speaker; as, I, we, me; We boys took the apples. C. Second Person. — That part represented by the person or thing spoken to; as, thou, you; O mighty Caesar, dost thou lie so low? d. Third Person. — That part represented by the person or thing- spoken of; as, he, they; What is man? OBSERVATIONS : a. Graiveviatical Person =P art in Conversation. — Let the student perceive, if he can, that language represents things, not as they really are, but only as they are thought to be. Language is the foot-prints of the thought, not of the things represented by the thought. In " The sun holds his fiery course through mid-heaven," we parse sti7i as masculine, not because the thing named is really a male, but because the sentence represents that thing as a male. In "Jam not John," we parse John in the third person, not because he is really in the third part of the conversation, but because the language represents him to be in that part. As a fact, John is in the same part as I. But while the thought represents I to be in the first part, it represents John to be in the third part ; and we parse, as by the language things are represented to be, not as they really are. In conversation, each moment some one takes the part speaking, one takes the part spoken to, one the part spoken of, which are all the parts pos- sible. In "Jam not John," Jis of the first part, or person, John of the third part, the part spoken of. If we change to "Jam John," John does not change parts, but is still in the part spoken of, the third person. John is, in fact, the same object as J, but the thought puts it in a different conver- sational part, or person. 416 THE SYSTEM METHOD. b. A command or entreaty usually employs the second person; but a wish or act of ordination commonly makes use of the third person. Ex- amples of the third person are, — Wish. Act of Ordination. 1. God he with you. 1. Be it enacted by the assembly. 2. So help you Ood. 2. Hear he that hath ears. 3. The heavens speed thee in 3. Be it known to you, etc. thine enterprise. C. The third person is often put for the first. In the invitation to his birthday party. Master Henry writes: "Master Henry C. French requests the pleasure of your company to his birthday celebration, etc.;" "Your daughter pleads; hear, O my father." Prog. IV: /. Class of Noun. 2, Person. 3. Number. 4. Gender. 5. Personification. a. Common. a. First. a. Singular, a. Masculine, a. Personified. b. Proper. b. Second, b. Plural. b. Feminine, b. Unpersonifled. c. Third. c. Neuter. d. Common. PARTICULARS : Nero 1 was a tyrant.^ Children,* obey your parents.* Philip,^ thou art a man.^ I, John,' saw. I have loved thee, ocean.^ O trouble,^ would I had never known thee! Ellen,^" tell your sister" to come home. "We are strangers. ^^ Thou, God,i^ seest me. I, Paul,i* have written it. This is the stone ^^ which was set at naught ^'^ of you builders.!' Thou, Harry,!® art a wicked fellow.!^ What ailed thee, O thou seaj^" that thou fleddest ? I am the physician.^! Worship thy Creator, 22 God; and obey his Son,'-^^ the Master,^* King, 2= and Saviour ^^ of men. Many evils ^^ beset us mortals.^® Fellow citizens, ^^ this occasion is a portentous one. Rome*" was a city^i of fame.^^ We are cowards.** Love ** denies rest *^ to my soul,*^ and slumber*' to my eyes.*® Coming events *^ cast their shadows *" before. Time *^ writes no wrinkles ^^ on thy azure brow.** The snow-drifts** lie breast-high in the fence-corners. "'^ The school-boys*^ run to the play-ground*' with joyful hearts.*® His penknife*^ lies beside the inkstand 5*' on his study-table. ^^ More than a hundred childi'en's ^^ q^^^i. dren ^* rode on his knee.^* They shook the depths ^^ of the desert gloom,^^ With their hymns ^' of lofty cheer.^® I am John.^^ I am not John.^" I am your friend.^i Somebody ^^ ring the bell.^^ So help you God.^* The heavens^ speed thee in thine enterprise.^^ God^' be with you. Hear, every man ^® that hath ears.^^ Be the wretch '" hung. Tour d&ughter '^ pleads ; hear, O my father. '^ Master Henry C. French '* requests the pleasure '* of your com- pany '^ at his birthday celebration,'^ etc. Kneel thou down Philip," but arise more great; Arise Sir Richard'® and Plantagenet." night ^° and darkness ! ®! ye are wondrous strong. THE W0BD8 CLASSIFIED. 417 DEMONSTRATION V. FACT MATTER: a. Case. — The form and posture of a noun or pronoun as a part of the sentence whole. (See Demonstration XXV., g.) b. Nominative Case. — That form and posture by which a being may- be asserted to exist; as, I exist, we exist, he exists, John exists. Observe that we do not say. Him exists. It grates upon the ear, and is never sanctioned by good autliority. Putting him in place of he is like putting the eye in place of the ear; it does not "fit." C. Possessive Case. — That form and posture which indicates owner- ship, sometimes origin or kind; as, my knife, our farm, sun's heat, boys' fun. d. Objective Case. — That form and posture which indicates the ob- ject or end of action; as, I struck him; I struck at them. e. Rule XIII. — The Apostrophe (') and s is added, — 1. To singulars not having the same form for the plural; as, boy's hat, Peter's book, Milton's Allegro, General Gates's command, Pierce's grammar, Harris's Hermes, coekatricc's den, Moses's writings. 2. To singulars having the same form for the plural; as, a sheep's fleece, a deer's horns. But the J') should be placed after the s in forming the possess- ive plurals; as, sheeps' fleeces, a load of deers' horns. 3. To plurals unlike their singulars and not ending in s; as, men's, chil- dren's, oxen's, geese's. 4. To the last part of any sort of a compound or complex name or sub- stantive phrase; as, Ralph Waldo Emerson's writings; my step-father's residence; my brother-in-law's place; the Secretary of the Interior's office; everybody else's bus- iness; the president of the society's pleasure; the lawyer, Henry Felix's library; Henry the Eighth's honor; sergeant-at-arm's duty. 5. To the last name in a series denoting joint possession; as, Beaumont and Fletcher's plays; Reed and Kellogg's grammars. 6. To each name in a series denoting separate possession; as, Webster's and Worcester's and Murray's dictionaries. f. Rule XIV. — The apostrophe (') only is added, — 1. To abstract nouns that never take the plural ; as, for conscience' sake ; for goodness' sake ; for righteousness' sake ; for justice' sake ; for peace' sake. 3. To plurals ending in s ; as, girls' hats, Pharisees' self-conceit, soldiers' hardships, eagles' wings, angels' visits, justices' warrants. g. The Case Determined. — Any child ten years old may learn eas- ily to determine the case. First, let the student fix in mind the following forms of the pronouns with their cases. Thej^ must be immovably fixed in the memory. The following forms of Be must also be thoroughly memor- ized: — 27 418 THE SYSTEM METHOD. Nominative: I, he, we, they, who. Forms i Am, art, is, are. Possessive: my, his, our, their, whose. -j Be, been, being. Objective: me. Mm, US, them, whom. 0/ -Be. (Was, wast, were, wcrt. Next he must fix in the memory, likewise, — h. The PRINCIPX.E OF "Be and Same Meanings." — This is by all odds the most important principle in grammatical science. The student that fails to grasp it, must certainly fail of real progress in grammar, whoever comprehends it may reasonably hope for the highest success. It is this : Names separated by a verb to Be and meaning the same thing, are in the same case: as, lam John; My name is Hassan; We took him to 6e Henry; He might have ieen elected president. The verbs to Be are those given above (see g). They are am, art, is, are ; he, teen, leing ; was, wast, were, wert. i. Directions to Determine the Case. — 1. Apply the possessive test, namely, tho ('). 2. Substitute the pronouns, thus: I, my, mc; he, his, liim; we, onr, us. 3. Aj)ply the principle of " Be and Same Meanings." j. Application op the Tests. — If the student has mastered the prin- ciple of "Be and the Same Meaning," it is believed that he will have no diificulty in deciding the most difficult questions of case. a. Apjilication 1st. — Take Mary in the sentence, "Mary thought Henry to be James." The student will observe, — 1. That Mary has no (') added, and is not, therefore, possessive ; 3. That we say "/thought," not "me thought." Now, since /is nom- inative, and we can substitute it for Mary, we conclude that Mary also is nom- inative. Again, take Henry. We see, — 1. That Henry has no ('j, and is not possessive ; 2. That we say "Mary thought me," not "Mary thought /." Now. since me is objective, and we can substitute it for Henry, we conclude that Henry also is objective. Once more, take James. We observe, — 1. That James is not possessive ; 2. That we are in doubt whether we say "Mary thought Henry to be /" or " Mary thought Henry to be me." But we see, — 3. That James means the same thing as Henry, from which it is separated by be. By direction 3d, therefore, we conclude that James is objective. b. Application 2d. — In the sentence, "James is supposed to have been elected president," we may observe, — 1. That James is not possessive, since it has no C) ; 3. That we say "lie is," not ''Mm is." Now since he is nominative, and we can substitute it for James, we conclude that James is also nominative. Again, we see, — 1. That president is not possessive ; 2. That it means the same thing as James, from which it is separated by is and been. Hence, by direction 3d, we know that president is nominative. c. Application 3d. — In "The gate is three feet high," we see, — 1. That gate is not possessive ; THE WORDS CLASSIFIED. 419 2. That we say "he is," not " Ms or him is." But since Jte can be substi- tuted for gate, we know that gate also is nominative. Again, we observe, — 1. That feet is not i^ossessive ; 2. That it is separated from gate by be, but that it does not mean the same thing. Hence it is not in the same (nominative) case. We conclude, there- fore, — 3. That feet is objective. OBSERVATIONS : a. Excepting such abstract singulars as are enumerated above in Rule XIV., 1, which do not take the plural form, the most judicious usage adds an additionai s to all singulars ending in s or s sounds ; as, Moses's writings, Peleus's son, Adams's ex^jress, Otis's letters, Princes Island, a justice's war- rant, King James's edict, Jir. Williams's oration. (See b below.) b. Since the pronunciation of possessive names formed by adding 's to singular nouns ending in the sound of s or s (as, justice's, Peleus's) is iden- tical with the pronunciation of the plurals formed by adding es to the same names (thus, justices, Peleuses), to reject such a possessive would be to reject also this corresponding plural. But, with both the odds of authority and the common practice of good authors in favor of 's, it is more wise to decide in favor of the plural in es, than to employ the plural but reject the possessive on the ground that the succeeding s's renders it difficult of pronunciation, when the same sibilant sounds must needs render the plural equally difficult. C. In parsing such expressions as "The Secretary of the Interior's of- fice," to show the construction of the words separately it is necessary to transfer the 's to the name of the real possessor, thus making the possess- ive Secretary's and leaving Interior in the objective case with of. In the expression, "The lawyer, Henry Felix's library," the 's is supplied thus, lawyer's; which is then parsed like Henry Felix's — in the possessive. d. The noun limited by the possessive is often understood, and must be supplied in systematizing or parsing, to complete the grammatical con- struction ; as, Bonaparte relied on his own sense, and did not care a bean for other people's [sense] ; We found him at Mr. Tomlinson's, the druggist. e. In systematizing the possessive in such expressions as "She is a wife of my brother's," " O, I see that nose of yours," "That is a picture of my /a^/ifir's," some word, as having, oicning, possessing, making, composing, must be suj)plied to complete the grammatical construction : She is a wife of my brother's having ; O, I see that nose of your having ; That is a picture oi -my father's owning, or ol -mj father' s making, according to the meaning intended. This possessive with of forms one of the most beautiful and forcible idioms in the English language. Prog. Y: 1- Number. 2. Gender. 3. Case. 4. Case Determined by. a. Singular. a. Masculine. a. Nominative, a. The apostrophe. b. Plural. b. Feminine. b. Possessive. b. Substituting pronoun. c. Neuter. c. Objective. c. Be and Same Meaning. d. Common. 420 THE SYSTEM METHOD. FABTICULABS : An idle brain ^ is the devil's ^ workshop. ^ He was paid money.* Homer 5 is Btyled the prince * of poets. '' He caused himself to be proclaimed king. ^ She wan- ders an outcast. ^ Let us make man. i" I do not care a straw. ^^ James ^^ became a good lawyer. '^ They found the party i* to be Henry, i^ The potatoes ^^ measured a bushel. " He was made leader, i^ We were taught a lesson, i^ The hat 2'' is worth a dollar. ''■^ The wall ^^ is three feet ^^ high. Let the angels 2* be our guides. ^^ She is worth him and aU of his connections. ^^ He was called Caesar. ^7 Let high-born seraphs "8 tune the lyre. ^9 Tom^" struts a soldier, ^i John Bunyan^^ was the au- thor ^^ of Pilgrim's ^* Progress. ^^ The general^® was saluted emperor. ^^ She walks a queen. ^^ Pitticus ^' was offered a large sum *" by the king. *i Lady Jane Grey *^ fell a sacrifice ^* to the wild ambition ** of the duke *^ of Northumberland. ^ He remains an idler *'' in the street. ^^ She was promised* the jewels. *9 I was asked a question. ^" The court ^^ would not have tried the man, ^2 if he had not broken jail. ^3 They are called cannibals. ^* They yawned their jaws ^^ out of joint. ^^ She seems a goddess." You wiU dance a jig.** He danced his feet^s tired. The king 8" is a child. ''I They wept their eyes*^ blind. My name ^* is Hassan. ^^ He returned a friend ** who came a foe. ^^ She wrings the clothes *'' dry. It would be a romantic madness ^^ for a man 69 to be a lord '° in his closet. ''^ It is like silver. ^2 To reign is worth ambition. '* I discovered him to be a scholar. '* The Lord '* sitteth King'''' forever. I am the true vine, " and my Father's is the husbandman. '^ Sir Philip Sidney 80 lived and died the darUng^i of the Court. ^^ A mother's "^ tender- ness •** and a father's®* care ^^ are nature's 8' gifts *® for man's ^^ advantage. '■"' By such a change, ^^ thy darkness ^ is made light, ^^ Thy chaos ^* order, ^5 and thy weakness ^^ might. ®' I have an Emerson's ^^ and a Greenleaf 's ^^ arithmetic.!"" And I thy victim i"! now remain. To thy own dogs,io2 a prey ^"^ thou shalt be made. I was eyes i"* to the blind, and feet i"* was I to the lame. He appears the upholder ^^^ of aU. that surrounds him. Our mistakes iw are permitted to be instruments i"® of good ^^^ to us. A sect ii" of freethinkers 111 is a sum^i^ of ciphers, i^* He fell a victim ^i* to his passions. ^^^ ^ few years "^ is a matter of little consequence."^ About one dollar i^^ was paid me. Can an adverb "^ ever be an attribute ? ^^o The reciprocal ^'^^ of a quantity 122 is unity 123 divided by that quantity. Light i-'* was called day. i-* This man i^e is now become a god. i^t What, Celso, I'^s thou turned courtier 1 129 The forces 1^" and the resistances 1^1 are Nature's, i^^ He came a foe, 1^* and returned a fi-iend. 1^* Lord Darnley 1^* turned out a dissolute husband. 1^^ The Atlantic Ocean i^' is three thou- sand miles 12® wide. Six i^^ and three 1*" make nine. 1*1 A man "* comes to measure his greatness 1^^ by the regrets, ^^ envies, 1** and hatreds ^^ of his competitors. ^" — Emerson. The angels 1*® that dwell with them, and are weaving laurels i*^ of life 1*° for their youthful brows, 1*1 are Toil, i*^ and Want, 1*^ and Truth, and mutual Faith. 1** — Id. He loved the tall deer '** as if he were their father. 1*^ We are a people 1*' yet. The Japanese 1^® are a very jealous people, i*^ Where are the Pla- toes 1®" and the Aristotles 1^1 of modern times ? 1^2 Christianity 1^^ was formerly prop- agated among the heathens, i^* We sold two dozen eggs, i^s No pains 1^^ is taken. — Pope. 16' Here these ill news. 1^* — Sfiakspeare. i^^ They hate us youth, i'" — Jd. She THE WORDS CLASSIFIED. 421 was named Penelope. "^ He caused himself to be proclaimed king. ^'^ Hortensius died a martyr, i" A man should not be a silkworm, "* nor a nation ^'^ a tent "^ of caterpillars."' DEMONSTRATION VI . FACT MATTEB : a. Principle II. — A noun or pronoun after "being" or "to be" IS NOMINATIVE, except when meaning the same thing as a preceding object- ive; then it is objective by "Be and the Same Meanings." <'See Demon- stration v., h.) 1. This principle constitutes Principle II. of syntax. (See Demonstra- tion XXVII.) Against it there has ever been a frivolous objection. Er- roneous in itself, the objection is unworthy attention; but as a lurking error imposed upon learners by writers of high repute, it deserves the more se- verely to be condemned. I quote from Goold Brown alone, since from him most other good grammars used in this country have adopted the error. " When a noun is put after an abstract infinitive [to be] that is not transitive, it appears necessarily to be in the objective case, though not governed by the verb [to be] ; for if we supply any noun to which such in- finitive [to be] may be supposed to refer, it [the noun supplied] must be introduced before the verb [to be] by the preposition for; as. To be an En- glishman , . . is no easy matter; that is. For a traveler to be an English- man . . . is no easy matter." — Grammar of English Grammars, t^. 52S. a. Mr. Brown exhibits much ignorance of what is scientific in gram- matical science, by presuming to explain the sentence, "To be an English- man is no easy matter," by explaining another very diflierent sentence, " For a traveler to be an Englishman is no easy matter." And should he succeed to perfection in proving Englishraail to be in the objective in this, he would have no shadow of proof to offer that Englishman is not nominative in that. b. It has been elsewhere clearly established that, " supplying, in pars- ing or systematizing or ' analyzmg,' is justifiable only to show the whole grammatical construction — ^justifiable only for the purpose of filling out the sentence whole." Hence, before arrogating to himself the privilege to sup- ply a word or words, our venerable grammarian must demonstrate that the construction of Ens;Iishraan cannot be determined and explained without sup- plying such words. This he nowhere makes any attempt to do; and thus is he justly chargeable with the commonest of scientific blunders, that of assuming the very fact which he sets out to prove. c. But were the assumption not unjust, the effort at argument is en- tirely fruitless. For, on page 623, he informs us that to be is, in reality, the 422 "THE SYSTEM METHOD. subject of the proposition, "To be an Englishman is no easy matter." Then traveler cannot be governed by to b«, for he himself says that to be is the subject of is. Traveler cannot be governed by, or have any grammatical connection with to be, for these reasons, which he unwittingly announces himself: 1. It is in the objective, which case the intransitive verb does never govern; 2. It is introduced by the preposition for, and is of necessity the object of that preposition, which otherwise could not be what he de- clares it is — a preposition. Now, since Mr. Brown has shown us that trav- eler is not governed by to be, and that it is the object of the preposition for, and that prepositions have an antecedent term as well as a subsequent term (p. 435, Obs. 2), and that the antecedent term "often comes" after the preposition and its object, even "at a great distance" (p. 435, Obs. 6), let us ascertain whether the antecedent term to for, may not be found just where he says it often is; namely, "after the jireposition and its object" (after for and traveler). To do this, our author says (p. 683, Obs. 2), " Place the in- terrogative wM< before the preposition." Following these directions, doing impartial justice at every step and no violence anywhere, we have the ques- tion, "What for a traveler?" to which the only answer possible is, " To be an Englishman is no easy matter 'for a traveler.'" Thus it is proved, by Mr. Brown's own direction and agreement, that the antecedent term to for is matter. And thus do we see that the "noun [traveler] to which the infin- itive [to be] was supposed to refer," has no sort of grammatical reference or connection with to be, but, on the contrary, is the object of the preposition for, by which it is utterly separated from the verb to be, and joined to the noun matter, " at a great distance " after! Hence, supplying the noun traveler, by which unwarranted assumption Mr. B. sought to prove Englishman to be in the objective case, proves noth- ing so far as Englishman is concerned, since traveler has, as Mr. B. is forced to admit, and finally to agree, no manner of grammatical connection with Englishman! which is still left as completely unexplained as before he had written a word. The principle by which the case of Englishman is to be determined is this: Whatever cannot be proven to be objective and has not the possessive sign, is nominative. " Nominative means ' naming ; ' and when we merely name an object, we put the name in the nominative, or naming, case. And ev- ery relation not expressed by one of the other cases, is expressed by the nominative case. Thus the noun is in the nominative case when it is inde- pendent of any verb, or when it is in the predicate [as in " To be an En- glishman is no easy matter"], and is not the object of a transitive verb or a j)reposition." — Noble Butler. 2. Professor Hart and others think the argument from the analogy of the Latin competent to put Englishman in the objective case; to which it will be more than ample to reply that the analogy of the German, which lan- guage is more closely allied to the English than the Latin is, would put it in the nominative. THIS WORDS GLASSIFIEl). 423 b. Principle III. — A noun or pronoun directly attached to an- other, AND SIGNIFYING THE SAME THING, IS IN THE SAME CASE BY APPOSI- TION (apposition = close position); as, Milton, tlie poet, "was blind; The apostle Paul persecuted the Church. Prog. YI: 1- Number. 2. Gender. 3. Case. 4. Case Determined by. a. Singular. a. Masculine. a. Nominative, a. The apostrophe. b. Plural. b. Feminine. b. Possesi?ive. b. Substituting pronoun. c. Equivocal, c. Neuter. c. Objective. c. Be and Same Meanings. d. Common. d. Apposition. e. After Beincj or to Be. rARTICULABS : The learned pagans ^ ridiculed the Jews ^ for being a credulous people.' His being a lawyer* was no reason^ for supposing him to be a rascal.^ There is no mistake 7 about his being the man.^ To be an Englishman^ in London, ^^ a Frenchman ii in Paris,^^ a Spaniard '^ in Madrid/*' is no easy matter.'^ I must be taught in order to be a gi-ammarian.'6 He went out mate/^ but returned captain. ^^ To be an eloquent speaker/^ in the proper sense 2° of the word,^^ is far from being a common or easy at- tainment.^''* What made Luther ^^ a great man '^"^ was his unshaken reliance ^^ upon God.-'5 It is almost as hard a thing '^^ to be a poet^^ in desi^ite^'-* of fortune,'" as it is in despite of nature.^^ I regard him as being a wise man.^^ ' T is most just that thou turn rascal.^' I had a suspicion ^' of the fellow's''^ being a swindler."^ He was taken no notice ^'' of. My being his father '^^ was the cause "^ of my great anxiety.'*" Edgar's *i having become a candidate*^ changed his whole demeanor.** A better proof** of his being a fool*^ is not wanted. Five*" less two*^ are three. *8 They ob- jected on account*" of his being a lawyer.^" The farm^i was talien possession ^^ of. The cakes ^^ were done liberal justice =* to. He was refused the protections^ of the law.s" He conducted himself as an honorable man.^' The most sacred things ^^ may be made an ill use^" of. When the quantity"" is taken twice as a factor,"^ the product "2 is called the second power."* It was added as an appositive."* The pro- gram's j-eads, Music,"" Toasts,"'' Supper,"^ etc. I consider Dr. Johnson"" as an excel- lent moralist.''" Under the Roman law,''i every son " was regarded as a slave. ''•'^ The warrior''* fell back upon the bed'^ a Hfeless corpse.''" The Duke''^ of Norfolk's'^ park '^ in Sussex is fifteen miles ^" in circuit.^' Kneel thou down Philip, ^2 but arise more great: Arise Sir Richard **3 and Plantagenet.^* Ye shall be as gods.^^ jj^ ^g^^ q^^ j^g mate,*" and came back captain.**^ Plato ^^ and Aristotle *s are caUed the two head springs ^" of all philosophy. 'Ji He came a foe,^^ and returned a friend.^'* Lord Darnleyi** turned out a dissolute and insolent husband."5 Sir Phihp Sidney '■'^ hved and died the darling ^^ of the Court, and the gentleman ^^ and idol 39 of the time. The tadpole, i"" or pohiwig,!"' becomes a frog.i"^ John,'"^ the dis- ciple,"* lay on his Master's breast. The petals ^"s of the daisy lor; (day's-eye) i"^ close at night and in rainy weather. He spake as one "* having authority. i"'' One and a hah times 11" one '" is one "■- and a hah."'* 424 THE SYSTEM METHOD. DEMONSTRATION VII . FACT 3IATTEB: Grammar is the science of constructing sentences; just as architecture Is the science of constructing houses, bridges, etc. In civil architecture, the unit of study is the building-part; in grammar, the unit of study is the sentence-part, the word. And we are to study, first, these parts, the mate- rials out of which the sentence is constructed. Of these, the most impor- tant are the name and the verb. Two parts are essential to the structure of any sentence; namely, a name and a verb. Because the verb is one of the structural parts essential to the sentence, and because most adjectives and relative pronouns are in some way dependent upon, or identified through, the verb, it is by odds of reason and advantage investigated or studied be- fore the adjective and the pronoun. The bones and muscles are studied before the lacteals and blood-vessels, etc., because they are the stays of the lacteals and blood-vessels. The noun and verb are studied before the other "parts of speech," because they are the stays of all these parts of speech. The verb is studied before the pronoun, because the chiefest mark of one class of pronouns (relatives, see Demonstration XV.) is that it forvis a link between the verbs. Thus the natural order takes first the two verbs, then the link (pronoun) between them; not first the link (pronoun) and then the things linked. The natural order takes first the two bones, then the link (muscle or ligament) between them; not first the link (the liga- ment), then the things linked. The parts essential to a building are those to which accessory, or de- pendent, parts are attached; namely, the foundation and other main timbers. The parts essential to a sentence are those to which accessory, or de- pendent, parts are attached; namely, the noun and the verb — the "principal elements." Now because, in the architectural structure, all other parts are attached to and made by these essentials (the foundation and other main timbers), therefore they (the essentials) must be first studied and under- stood. And because, in the sentential structure, all other parts are attached to and made by these essentials (the noun and the verb), therefore they (the essential noun and verb) must be first studied and understood. And so on. But the items of fact matter to which the student's attention should be first called, in the study of the verb, are, — • a. Verb. — A word that asserts the progress or completion of action; as, He sees, he saw ; The child falls, the child fell. b. A Regular Verb. — One that takes ed added to the simplest form of the verb; as, walk, walk-ed; lift, lift-ed. e. An Irregular Verb. — One that does not take ed added to the sim- plest form of the verb; but usually changes the sound of the base vowel; as, see, saw; eat, ate. THE WORDS CLASSIFIED. 425 d. Principal Parts. — Those parts most frequently used; as, see, saw, seen; go, went, gone; eat, ate, eaten. e. The principal parts are found by reading the verb into the follow- ing blanks: — 1. I now; and this part is the imperfect tense. 2. I yesterday; and this part is the perfect tense. 3. I have ; and this part is the passive participle. f. Observe that when we change love to loving, final e is dropped by Rule V. for spelling. In the same way, when love is changed to loved, final e is dropped and ed added by the same rule for spelling. g. Tense. — That property of the verb that certifies whether the action is represented to be begun and finished, or only abiding. h. Imperfect Tense. — That form used to represent the action as unended and abiding in fact. It is the simplest form of the verb. Thus in "God is wise," the imperfect is represents the wisdom of God to be un- ended and to abide in fact. So in " Goldsmith writes excellent poetry," the imperfect writes certifies, not that the action abides yet, but that the fact yet abides. Again, in "Mr. Russell goes to Chicago next week," goes certifies, not that the action is now going on, but that the fact of his going now abides. i. Perfect Tense. — That form of the verb which represents a real or supposed event as ended and completed. In the sentence, "God created the world," the perfect tense created shows the act of ci'eation to be ended and completed. j. The following formalized words have so far departed from their original radical sense that students will find it altogether necessary thor- oughly to memorize them with their tenses. Let these forms be so thor- oughly committed that they can never be forgotten : — ■ Imperfect. Perfect. Will would. Shall should. Ought ought. Thus the principal parte of either may or migllt are may, might; of can or could, the principal parts are simply can, could; etc., etc. k. Why are "imperfect" and "perfect" the proper names to apply to what are called the tenses? — Because they are congruent with the nature of grammatical tense; whereas "jiresent" and "past" are incongru- ent with the nature of tense. The name " past tense " misleads the student by suggesting the false idea of past time. But this tense (more properly called the "perfect") does not refer to past time, but to finished, or per- fected, action. (See "The Correct Theory of Tense," a, b, c, d, e, f, pp. 384 and 885.) " Perfect tense " is the proper name. It is a very fitting one, since "perfect" directs to the true nature of the action as completed. Imperfect. Perfect. May might. Can could. Must must. 426 THE SYSTEM METHOt). finished, perfected. "Present tense" is objectionable; for "present" does not suggest the real character of the action as unfinished, or imperfect. 1. Is NOT THE TENSE to be determined from the verb's signification as to time? — No; read carefully "The Correct Theory of Tense," jDages 584 to 388. Prog. VII: I.Verb. 2. Principal Parts. 3. Regularity. 4. Tense. a. Imperfect tense, a. Regular. a. Imperfect. b. Perfect tense, and b. Irregular. b. Perfect. c. Passive participle. c. Equivocal. FABTICULABS : She can ' play.^ Sarah has ^ written * a letter. I love ^ to see ^ the sun shine.' Has 8 he come? » They were ^o reading ^ the book. America was i^ discovered i^ by Columbus. She had^* gone^^ to walk.^s If you wish,^'' I wiU accompany i8 you. She must ^^ learn 2" her lesson. She hoped ^^ to have ^^ gone.^^ I would 2* not have '^^ believed ^g it. Did ^7 you lose ^* your umbrella? Milton wrote ^^ Paradise Lost. The king was s" concealed ^i in a tree. He should ^8 have ^3 told 3* the truth. The burglar might 35 have ^6 been" arrested.^s jjg has 3'' found '«' his knife. The army en- camped *i by the river. He should ^^ have ^^ come *^ home. Were ^^ i rich, I would ^^ purchase*' that property. Do ^^ let *^ me go ^^ to the picnic. I had ^i been ^^ taught.^^ The letter wiU 5* havens been ^s written.^' You might ^s study.59 The man loves «" to see^i it rain.^2 He ought ^^ to have^* goners home. He should ^^ be^T more indus- trious. He has ®^ invented ^3 a velocipede. The army will™ be '^ disbanded. '^ The ship wiU'^ have'* arrived'^ by that time. The sun has'^ arisen." That villain may '8 have'^ stolen s" my knife. Tou might ®i have ^^ seen ^^ the menagerie to-day. The assassins, having s* been ^ condemned,*'' were *' executed.*^ Ought ^^ I to study 9" grammar? The ship would ^^ have^^ becn^^ sunk,^* had^^ not the storm abated. ^^ The coi-n should 3' have 9* been 99 husked w before the rainy season. The lion isi"! subdued.i°2 To-morrow is '"^ Thursday. Mr. RusseU goes i"* to Chicago next week. When the mail arriveSjio^ -we shall 'os hear lo' the news. He wiU "'•'' go.io^ The pupils did 110 succeed.iii A Mon is "^ subdued. ^^^ j would m study "^ if I had i^s my book. I shall 11' see 118 him when he comes.n^ Hadi^" not the storm abated, i^i the ship would 122 have 123 been 12* sunk. 125 I had 126 better go.i^' Henry could i28 speak i29 fluently. He may i^" return i^i to-morrow. The ship sails i32 next week. Has i33 it been 13* decided? > 35 gg kept i^s shipping i3' the horse. The pupils cani38bei39 taught.i^o We soon shall 1*1 reach 1*2 the boundless sea. The poor must i*3 work i" in their grief. I could i-^5 not learn i*'' that lesson. Clarence has "' been i*^ chosen i^'-* captain. They might i^o have i5i finished 1^2 their task yesterday. The Indians are i53 fast disappearing.15* John would i^" havei^^ avoided i^' meeting i58 him, if he could 159 havei''* donei^i so without being i''2 called 1^3 a coward. Thy fame hathii^* preceded i*'^ thee. Liars should i^^ have i''' good memories. You should i''^ have i''' beeni'" working. I'l The danger might "2 havens been"* avoided.i'5 I shall "6 bei'^' contented.1'8 The farm was i'9 sold.i^" Stars have 1^1 disappeared. 1^2 Times will 1^3 surely change.i^* If I should i^5 go^iso you would 1^' missi^^ me. The cock shaUi«9 not crowi-'" tiU thou hasfi^i denied 192 Me thrice. He willies become i^* discouraged 195 before he hasi^^ thoroughly tried i9' it. Thou hasti98 neglected 199 thy duty. They did 200 testify 201 the truth. We should 202 have203 been 20* contented.205 THE WORDS CLASSIFIED. 42^ Your conduct did^os excite 2'"' disturbance. I could ^os break *^°9 that bar of iron. They may^i" have^" forsaken ^^^ their friends. They did^is make ^1* a villainous appearance. Would -^^ they deliver '^i" the message? Did^i''' we circulate ^i* our opinions too freely ? I did ^^^ only dream. ^^o j g^^^ 221 Absalom hanged ^'^ in an oak. The sick man, having ^^^ been^^* sleeping ^^^ for some time, awoke ^^^ in an improved state. We did^^'^ esteem ^^8 i^jqj smitten ^29 of God. The character of the man, having^"* been^^i ascertained,"^^ caused ^^^ his disgrace. Did^^* you return? ^35 He goes^^^ to the city to-morrow. There she would ^37 sit"^^ and weep ^"^ for hours. If he sees 2*0 the signal, he wiU^n come.^*^ We must^*^ go^ii to-day. I can^*^ pay 246 you next week. I will 2*7 pay 2*8 you next week. Has 2*9 he received ^^o his pension? We shall 251 hear 253 the news when the mail arrives.253 I wish 25* to see 255 him before he goes 256 to the city. The pupils certainly did 257 succeed. '■^58 jf j had 259 my book, I would 26O study.26i If ye loved 262 Me, ye would 263 keep 26* My commandments. The lion is 265 subdued.266 The laws are 267 to be 268 obeyed. ^^9 Has 270 it been 271 decided? 273 piato reasons 273 well. The ship sails 27* nest week. He can 275 pay 276 to-morrow. What nature has 277 denied,278 fools will279 pursue.289 — Young. I had 281 rather be 28" a kitten and cry 283 mew Than one of these same ballad-mongers. — Hhakspeare. DEMONSTRATION VIII. FACT MATTER : a. Form is the basis of every grammatical property. In any language not having a change of form to mark the action expressed by the verb as continued or completed, there can be no such thing as tense is in English. b. The imperfect is the simplest form, or root-form, of the verb, and may always be known, except in the participial mood, by its taking to before it as a test; as, to love, to eat. We can write or spell to love, but not to loved; to drink, but not to drank; since neither loved nor drank will justify with to. C. The perfect tense of regular verbs may be known by its adding ed to the imperfect; thus: — Imperfect. Perfect. - Imperfect. Perfect. Trust trusted. Drop dropjDed. Hope hoped. Carry carried. E is first dropped from hope, by Rule V. for spelling, then ed is added. In dropped, the p is doubled by Rule II. for spelling, when the ed is added. When ed is added to carry, y is changed to i by Rule VII. for spelling. 428 THE SYSTEM METHOD. d. The Nature op Tense. — A mistake is often made by confusing the tense of the verb with the time of the sentence. (See "The Correct Theory of Tense," e, p. 385.) e. Contradictories.— See "The Correct Theory of Tense," f, p. 385. f. Structure the Base op Classification. — See "The Correct Theory of Tense," g, p. 386. g. One Verb Consists op a Single Word. — See "The Correct Theory of Tense," h, p. 387. h.. Complete Verb. — One having a form that may be joined to have; thus: I have gone. Examples: go, look, lie, steal, sit. 1 Defective Verb. — One not having a form that may be put adjunct to have; as, will, shall, may, can, must, beware, oaght, quoth, wit, methinks. i Eedundant Verb. — One having more than one form that may properly be put adjunct to have; as, stay, work, learn. Prog. VIII. /. Principal Parts. 2. Regularity. 3. Inflection. 4. Tense. a. Imperfect tense, a. Regular. a. Complete. a. Imperfect. b. Perfect tense, and b. Irregular. b. Defective. b. Perfect. c. Perfect participle. c. Equivocal. c. Redundant. PABTICVLABS: She can ^ play.^ Sarah has ^ written * a letter. Was ^ it a dream we dreamed ? « They are^ prepared » to go.^ The barley was" just reaped.ii When George returns '■^ we wlll^^ send^* you. We may^^ seei^ Venus to-night. The Christian lives " forever. It is ^^ but the other day that a hero died.^^ Yesterday was ^o Tues- day. We shall 21 inform ^^ him when he comes.^a Were ^^ I not Alexander, I would ^5 be 26 Diogenes. It is ^^ necessary that the rules should ^s be ^9 observed. s" Were ^^ I sure he would ^2 come,33 j should s* invite ^5 him. She might 36 go,^^ were ^8 she so dis- posed. ^9 What could*" we ask*i more ? Could" he fail,*' if he should** attempt *5 it 1 The battle would *« seem*' to have*^ been** a bloody one. The boy is so pun- ished.51 The assassins will^^ be^' hanged.^* The horse was 55 rode 56 by Captain Jinks. The melancholy days are 57 coine.58 I wish 59 he lived 6" with us. The mail is 61 arrived. 62 May 63 one be^* pardoned 65 and retain 66 the oifense? Ah, had 67 your faith been 68 joined 69 with mine. As once this pledge appeared ™ the token, These foUies had 'i not then been'^ mine, — My early vows had ^^ not been 7* broken.''5 — Byron. The pupils certainly dido's succeed '''^ in the effort. The clerk is '^^ reading '''9 a decision rendered,^" written,^i signed,^^ sealed,^^ and deUvered ^* to him by the judge. He, having *5 encountered ^s and defeated ^^ his antagonist, returned ^^ umnjured. I have *9 written 90 and mailed 9i the letters. America was 92 discovered 9" by Columbus. Pom- pey was 9* defeated 95 by Caesar. If you wish, 96 T will 9^ accompany 98 you. Ought 99 I to go?!"" WiUi6i he beio^ present? The ship would "^ have^o* been^^s sunk,i<'6 had 19'' not the storm abated. i"* The lion is i"' subdued.""* I have,"! methinks,"^ a fever upon me. What man enters,"' dies."* He wist "5 not what to say."6 The women, thcyfeU"'' to praying."* Beware "9 of him who would i^o beguile i^i you. THE W0BD8 CLASSIFIED. 429 When he came ^^^ to doing the work, he was ^'^^ unwilling. We got i^* to tripping ^^^ each other. There the wicked cease ^-^ fi'om troubling, i"'^ He chooses ^^^ to ride.i^^ They intend ^^^ starting i^i to-morrow. She can ^^^ sew.^^^ What would ^^i yQ^ think 1^^ of me? Does^^'' he choose ^^^ to sit^^^ there? We are^^D going i''' well, but did^^^ not continue ^^^ as he had^^** begun, i^i He has ^^2 awaked 1'^^ and arisen. ^^^ He rose ^''^ up, but is^^^ now lying ^^''' down again. The water is^"^ frozen.i''^ The book is^™ lying i''^ where John laid^™ it. The box isi'^-'' sitting 1'^'' in the desk where you set^'''^ it. He set^^^ the child on the floor. He raised ^'''' himself up. He rose ^'^ up, and then again sat i'' down. Methought ^^o j saw 181 uiy late espoused ^^^ saint. It seems ^^^ to me that I hear^^* it thunder.^^s When wilt^^^ thou arise ^^'^ out of thy sleep? Some say 1^8 the earth Was 188 feverish and did shake. ^8" — Shakspeare. Many minds did^^^ gird^^^ their orbs with beams. — Tennyson. Music in his ear his beating 1®^ heart did^^* make.^^*] DEMONSTRATION IX. FACT MATTER : a. Mood. — The form and posture of a verb as a part of the sentence whole. (See Demonstration XXV., g.) b. Indicative Mood. — That form and posture by which the event is represented to be determined in fact; as, I walk, thou walkest, be walks; 1 wish be lived with us. C. Subjunctive Mood. — That form and posture by which the event is represented to be undetermined or denied in fact; as, I wish he lived with us; If he be bearing bis class, do not interrupt him d. Determined Event. — Any truth or event that can be declared now to be true or to obtain. Thus, at any present moment, we can make one of two declarations, — 1. That it is raining; or, — 2. That it is not raining; for the fact of its raining in time present is always a determined fact, or event But we cannot make, at any present moment, either of these other two declarations : — - 1. That it will begin to rain just 100,000,001 seconds from the present second; or, — 2. That it tcill not begin to rain just 100,000,001 seconds from the present second; for the hypothesis of its raining in future time, at a jiarticular and 430 THE SYSTEM METHOD. distant second, is only hypothesis. This, or such hypothetical event, is called an undetermined fact, or event. e. The iNDiCATrvE imperfect represents an event as already determined in fact, though what that event is, may be known neither to the speaker nor to the hearer. f. The subjunctive imperfect represents an event as not yet deter- mined and therefore contingent — but possible. g. Let the difference in thought in these pairs of sentences be wake- fully studied, the mind dwelling particularly on the different circumstances under which these sentences may be appropriately used:— Ikdicative Imperfect. Subjunctive Imperfect. 1. Though he gets far from home, he 1. Though he get far from home, he always -wTites to us. will have friends. 2. If you are true, I have no doubt you 2. If you be true, you will have have friends. friends. 3. If thou canst talk, speak. 3. If thou can, learn to talk. 4. If he drinks, then I know he swears. 4. If he drink, he will immediately swear. 5. He maintains his integrity, though 5. He wiU maintain his integrity he loses his estate. though he lose his estate. 6. If he is alive, he would write to us. 6. If he be alive, it will be better than we shall expect. 7. If I am wrong, why did you not tell 7. If I be ^^Tong, you wiU be there to me before ? apologize for me. 8. If he does drink, and she could have 8. If he do drink at the festival, she learned that fact, she would have re- will renounce him. nounced him. h. The table here following will be the chief instrument and best means that the instructor can find with which to lead the class to the foundation idea of mood; and it should be immovably fixed in the memory of every member of the class. It constitutes the best test for determining the mood: — Indicative Imperfect. Subjunctive Imperfect. 1. I am, go, love, sec. i- I be, go, love, see. 2. Thou art, goest, lovest, scest. 3. Thou be, go, love, see. 3. He is, goes, loves, sees. 3. He be, go, love, see. i. Were there no structural appearance, or foot-print, of the indicative, it were not studied as a mood. A mood is to be traclied — as a rabbit in the snow, if you please. The foot-prints of the indicative are st in the second person — thou may-st — and s in the third — he love-S. When these marks or tracks appear, the verb is indicative. When the verb is indicative, these can be made to appear by substituting thou and he for the nominative just preceding the verb. Thus, in "I go to school," we may show that go is in- dicative in this way: substituting thon for the nominative I, we get thou gO-est, THE WORDS CLASSIFIED. 431 in which the indicative st appears ; therefore go, in "I go to school," is in- dicative. j. Let the student draw two parallel lines on his slate or paper, refer to the examples of the subjunctive imperfect already given, and write the verbs so that the subjunctives will come under one another and between the two lines : thus:-— 1. Though he 2. Though he 3. Whether he 4. If he get lose go do far from home, he will have friends. his estate, he will maintain his integrity. or come, do not fail to inform me. drink at the festival, she will renounce him. Then let the student fail not to see that no s or st (indicative signs) appears to the right of the second line, but that the form of the subjunctive is al- ways the simple (infinitive) form, the one used with I; as, 1 go, I love, etc., etc. ; not I goes, nor I gocst. Then, placing the examples of a few indicatives between the parallel lines, observe that s or st (tracks of the indicative) do appear, thus: — 1. Though he 2. If he 3. Though he 4. If he 5. If thou 6. If thou 7. He 8. He 9. It 10. Thou get do lov go can may do like rain do es es st st cs s s. st far from home, he always writes to us. not hear, I am much mistaken. his estate, he maintains his integrity. to heaven, I must meet him. talk, pray speak. ask, get permission, if that is possible. not hear, and you are mistaken. to give. harm me. k. Mood, like case, is based on attitude-form — on the form and posture of the verb as a part of the sentence whole. But to understand the form and posture of a verb as a part of the sentence whole, — as a part joined to other parts, — we must understand the joints thus formed. These two tests, then, will always enable us to cUstinguis/i between the indicative and the sub- junctive : — 1. Substitute for the nominative of the verb, !, tlioa, and he. If S or st appears, the verb is an indicative ; but if s or st is not thus made to appear, — 2. Substitute other verbs for the verb in question, when, if the verb is indicative, the s or st will ajDpear. 3. If they do not appear at either of the above steps, and the verb has a nominative, it must be a subjunctive. Prog. IX : — /. Regularity. a. Regular. b. Irregular. 2. Inflection. a. Complete. b. Defective. c. Redundant. 3. Event. 4. Mood. a. Determined. a. Indicative. b. Undetermined, b. Subjunctive. 432 THE SYSTEM METHOD. PABTICULABS : If he bei hearing his class, do not interrupt him. If he is^ hearing his class, why do ^ we not hear their voices ? If thou read * this, O Cisesar, thou mayst ^ live. If, then, that friend demands " why Brutus rose "^ against Caesar, this is ^ my answer. He will ^ maintain his integrity though he lose i" his estate. He maintains ^^ his in- tegrity, though he loses ^^ his estate. If he do ^^ not hear, I shall i* be much mis- taken. If he does ^^ not hear, I am i" much mistaken. Go on, and see whether Bru- tus be^'^ alive or dead. If he be^^ so resolved, I can^^ o'ersway him. Though he gets '^^ far from home, he always writes ^^ to us. Though he get ^^ far from home, he will ^3 have friends. I have 2* no doubt that you have ^^ fi-iends, if you are ^^ true. If your messenger find^^ him not there, seek him in the other place yourself. Love not sleep, lest thou come^* to poverty. If his head do^^ not ache, you should ^*> ask him to go. If his head aches, ^^ do not ask him to go. Tou must ^^ ascertain whether it be 23 the correct time. Determine whether it is ^^ the correct time. A man does ^^ not know until ^he has "^ tried. If he refuses ^'^ the gold, I will 2* not send him the silver. If he refuse ^^ the gold, do not give him the silver. If his feet are ** large, he wiU*i not sell well. If his feet be*^ large, he wiU*^ not sell well. ■ If thou see** the signal, thou shalt*^ come. Here will**' I stand till Ccesar pass*'' along. See thou to it that thine own fineness undo *^ thee not. See thou follow *^ me. The love of all thy people comfort thee, Till God's love set^* thee at his side again. — Tennyson. Though He slay ^^ me, yet will ^^ I trust Him. His soul thou canst ^^ not have. If he were ^* dead, what would ^^ betide me ? I thought thou hadst ^^ been resolute. If the process of learning be^'' branching, the result of learning will^* be a branch. DEMONSTRATION X. FACT MATTER: a. Events are Either Determined or Undetermined. — Either it is, or it is not, now raining. That fact is determined. But we cannot de- clare that it will begin raining just 3,000,001 seconds from the present sec- ond. For that is an undetermined, contingent, event. That a frog now full-grown either did or did not develop from a tad-pole is a fact now deter- mined. That a suppositious egg, or spawn, to be laid by a supjiositious frog, itself not yet born, or existing, icill develop into a frog, is undetermined. That this same egg will not develop into a frog is also undetermined. Thus, either that this egg will, or that it will 7iot, develoj) into a frog, is undeter- mined. We now come to the specific jjoint ; namely, that, not considering the perfect subjunctive, we may say — 1. Every determined action, or event, is represented by tlie indicative Jiiood ; 3, The subjunctive mood always refers to an undetermined or a supposed event. THE WORDS CLASSIFIED. 433 b. Let the student again give his attention to the difference in signifi- cation between the indicative and the subjunctive, and particularly to the fact that the circumstances under which they can be appropriately used are different. Indicative Imperfect. 1. If he has the book, he will give it to you. 2. If he does not hear, I am much mis- taken. 3. If he is hearing his class, do not in- terrupt him. 4. If he refuses the gold, I wiU not send him the silver. 5. If he is better, I have hope of his recovery. 6. If his head aches, do not ask him to go. 7. If his head does not ache, you should ask him to go. 8. I wiU perform the operation if he desires it. 9. I shall not perform the operation if he does not desire it. 10. If he denies the charge, I wiU prove it. Subjunctive Imperfect. 1. If he have the hook, he wiU give it to you. 2. If he do not hear, I shall be much mistaken. 3. If he be hearing his class, do not in- terrupt him. 4. If he refuse the gold, do not give him the silver. 5. If he be better, you may stay with him an hour. 6. If his head ache, do not ask him to go. 7. If his head do not ache, you should ask him to go. 8. I wiU perform the operation if he desire it. 9. I shall not perform the operation if he do not desire it. 10. If he deny the charge, I will prove it. C. Commit the English portion of the following table, which gives the changes of form and posture through the indicative, subjunctive, and im- perative moods of the verb : — Be, with the Anglo-Saxon Equivalent. Indicative^ Imperfect Tense. Subjunctive, Imperfect Tensi Singular. Plural. Singular. Plural. 1 I am. 1 We are. 1 I be. 1 We be. 2 Thou art. 2 You are. 2 Thou be. 2 You be. 3 He is. 3 They are. 3 Hebe. 3 They be. 1 Ic beon. 1 We beoth. 1 Ic beo. 1 We beon. 2 Thu hist. 2 Ge beoth. 2 Thu beo. 2 Ge beon. 3 He bith. 3 Hi beoth. 3 He beo. 3 Hi beon. Perfect Tense. Perfect Tense. 1 I was. 1 We were. 1 I were. 1 We were. 2 Thou wast. 2 Tou were. 2 Thou were. 2 You were. 3 He was. 3 They were. 3 He were. 3 They were. 1 Ic waes. 1 We waeron. 1 Ic waere. 1 We waeren. 2 Thu waere. 2 Ge waeron. 2 Thu waere. 2 Ge waeren. 3 He waes. 3 Hi waeron. 3 He waere. 3 Hi waeren. 28 434 THE SYSTEM METHOD. Imperative, Imperfect Tense, 1 Bel. 1 Be we. 2 Be thou. 2 Be you. 3 Be he. 3 Be they. 1 1 2 Thuheo. 2 Gebeoth. 3 3 Infinitive Mood. Simple infinitive, t)e=beon. Gerundial infin., being=[to] beonne. Participial mood. Active, being^beonde. Perfect, been^gewesen. rrom this table let the student or reader observe,— 1. That the subjunctive has no change or variation for person and number, having the sxDelling be for each person and both numbers in the imperfect tense, and were for all persons and numbers in the perfect tense. 2. That the indicative has a variation for both person and number. 3. That the imperative imperfect and the subjunctive imperfect are alike in form but different in posture, the imperative usually coming before its nominative, the subjunctive always after the nominative. d. Next, the student will fix in his memory if possible, in his compre- hension at all events, the English portion of the conjugation of — Love, with the Anglo-Saxon Equivalent. Subjunctive, Imperfect Tense. Indicative, Imperfect Tense. Singular. 1 I love. 2 Thou lovest. 3 He loves. 1 Ic luflge. 2 Thulufast. 3 Helufath. Perfect. 1 I loved. 2 Thou lovedst. 3 He loved. 1 Ic lufode. 2 Thulufodest. 3 He lufode. Plural. 1 "We love. 2 Tou love. 3 They love. 1 Weluflath. 2 Geluflath. 3 Hi lufiath. 1 We loved. 2 You loved. 3 They loved. 1 We lufodon. 2 Ge lufodon. 3 Hi lufodon. Singidar. 1 I love. 2 Thou love. 3 He love. 1 Ic luflge. 2 Thu luflge. 3 He luflge. 1 I loved. 2 Thou loved 3 He loved. 1 Ic lufode. 2 Thu lufode. 3 He lufode. Plural. 1 We love. 2 You love. 3 They love. 1 We lufigen. 2 Ge lufigen. 3 Hi lufigen. Perfect, 1 We loved. 2 You loved. 3 They loved. 1 We lufoden. 3 Ge lufoden. 3 Hi lufoden. Imperative, Imperfect Tense. 1 Love I. 2 Love thou. 8 Love he. 1 2 Thulufa. 3 1 Love we. 2 Love you. 3 Love they. 1 2 Ge lufiath. 3 Infinitive Mood. Simple infinitive, love:=lufian. Gerundial infinitive, loving^ [to] lufianne. Participial Mood. Active, loving=lufigende. Perfect, loved=lufod. In the conjugation of love, as in that of be, it will be noticed that the one characteristic of the English and Anglo-Saxon subjunctives is that they have no variation whatever for person. THE WORDS CLASSIFIED. 435 e. Every effort should be made, in the demonstration, or preliminary drill, to get these facts before the mind: — 1. That the indicative adds s in the imperfect tense third singular (except for am, may, can, must, will, shall, ougUt.) 2. That the subjunctive never adds s in the imperfect tense third singular. 3. That the indicative imperfect has a variation, that is, a change of spell- ing, for both person and number. 4. That the subjunctive imperfect has no variation for person or numbei*. 5. That the indicative adds st or est in the imperfect tense second singu- lar. 6. That the subjunctive never adds st or est in the imperfect tense second singular. 7. That the third singular indicative adds s to the third plural. 8. That every grammatical property, or modification, is somewhere marked by a change of form. f. The Tests Repeated. 1. Substitute for the nominative of the verb, I, tllOH, lie. If s or st appears, the verb is indicative. If they do not appear, — 3. Substitute, for the verb in hand, other verbs, when if s or st appears, the verb is indicative. If s or st do not yet appear, and the verb has a nom- inative before it, — 3. It is subjunctive. Prog. X: /. Regularity. 2. Inflection. 3. Time. 4. Mood. a. Regular. a. Complete. a. Present. a. Indicative. b. Irregular. b. Defective. b. Future. b. Subjunctive. c. Redundant. c. Past. FABTICULABS: If this is 1 not obvious, you may consult the essays. Though you must ^ ride, I will 3 walk. If he loves * her not, and is ^ not from his reason fallen thereon, let me be no assistant for a state. He will ^ not succeed, unless he exert ^ himself. Though this he * madness, yet there is ^ method in it. If he hut shrink,^'' I know ^^ my course. If he has ^^ the book, he will give it to you. If she find i^ him not, to England send him. If his head do ^^ not ache, you should ask him to go. I will perform the opera- tion if he desires 15 ^^ jf j^g steal ^^ aught, I will pay the theft. If he have^^ tbe book, he wiU give it to you. If he be ^^ better, you may stay with him an hour. For if the king hke ^^ not the comedy, he hkes ^o It not. If he does ^^ not hear, I am much mistaken. If his head aches,^^ do not ask him to go. If my duty be ^^ too bold, my love is ^* too unmannerly. If he is ^^ better, I have hope of his recovery. If your messenger find ^^ him not there, seek him in the other place yourself. If he refuse ^'' the gold, do not give him the silver. If my love thou holdst ^* at aught, thou mayst ^^ not scorn my action. If it please 3" his Majesty, ' t is the breathing-time of day with me. If your mind dishke ^^ anything, obey it. If it be ^^ not now, yet it wiU come If it be 23 so, Hamlet is of the faction that is wronged. If thou didst ^^ ever hold me in thy heart, absent thee from f ehcity awhile. Thus thou must ^5 do, if thou have ^^ it. If he do 3^ bleed, I wiU gild the faces of the grooms withal. You shall ^^ digest the venom of your spleen, though it do ^^ split you. Remember that thou magnify *•* 436 THE SYSTEM METHOD. his work. If He cut ^^ off, and shut ^^ up, or gather *^ together, then who can ** hin- der Him? Doubtless thou art *^ our father, though Abraham be *^ ignorant of us, and Israel acknowledge *^ us not. I can ** not doubt that they are *^ what they appear 5* to be. He must ^^ fail, except God be ^^ with him. She should ^^ act now, lest she forget 5* her duty. Call now, if there be ^^ any that wiU ^^ answer thee; and to which of the saints wilt ^7 thou turn? If I say,58 I am ^^ perfect, it shall ^^ also prove me per- verse. Is 8^ it good unto thee that thou shouldst ^^ oppress ? So man lieth ^^ down and riseth 8* not : till the heavens be ^^ no more, they shall ^^ not awake, nor be raised out of their sleep. If a man die,^'' shall ^^ he live again? All the days of my ap- pointed time will ^^ I wait till my change come.'''" DEMONSTRATION XL FACT MATTER: a. Perfect Tense. — That form of the verb whicli represents a real or supposed event as ended and completed. In " God created the vrorld," the perfect created establishes the act of creating as ended and completed. b. The indicatiye perfect represents a completed event affirmed in fact. C. The subjunctive perfect represents a completed event as merely- supposed. d. The So-called Future Tense. — If inquiry be made, hereabout, concerning the so-called future tense, let the inquirer be informed that it will be explained further on in the Syntax. (See "The Future Tense a Fallacy," "Two Words as One," "Learner's Process of Work Benighted," and "One Verb a Single Word," pp. 378, 393, 377, 387.) e. Let the students sense, as thoroughly as they can, the following sen- tences, the instructor carefully explaining the difference in signification be- tween the indicative perfect and the subjunctive perfect, and especially dwelling upon the difference in the circumstances under which they would be appropriately used: — Subjunctive Perfect. Indicative Perfect. 1. Though it thundered he wouldn't 1. Though it thundered, he didn't hear hear it. it. 2. If I had a pen, I would write. 2. If I had the pen, I must have left it there. 3. If ye loved Me, ye would keep My 3. Though ye loved Me, ye did not keep commandments. My commandments. 4. If he died I would not go to help. 4. He died when it was better not to live. 5. If it were not so, I would not have 5. If it was not true, why does he not told you. ' deny it? THE WORDS CLASSIFIED. 437 From the sentences above, it will be observed, — 1. That the subjunctive mood represents an event as either undeter- mined or merely supposed. 2. That the indicative mood represents an event determined in fact. 3. That the imperfect tense of the subjunctive represents an event un- determined in fact and therefore contingent. 4. That the imperfect tense of the indicative represents an event de- termined in fact, though not necessarily known. 5. That the subjunctive perfect represents a completed event as merely supposed. 6. That the indicative perfect represents a completed event affirmed in fact. f. The following outline embraces those items of fact matter of most importance, and should, hereabout, be carefully studied by each member of the class, they reviewing, if necessary fully to grasp it. g. The Indicative and Sub.junctive in Outline. V lidicative— 1.^ In the English idiom for declarative sentences, follows a nominative. 3.^ Can always be used with its own nominative to form a direct em- phatic assertion. 3.2 Has two tenses. 1.^ Imperfect tense of — 1.* Adds %i in changing I to thOU. 2.* Adds s in changing I to he. 3.* In the sentence, represents an event as determined and abiding in fact, though not necessarily known. 4.* Can be used with its -own nominative to refer to present time. 2.^ Perfect tense of — 1.* Adds st in changing \ to fchoa. 2.* Does not add § in changing I to he. 3.* Can be used with its own nominative to assert an event as com- pleted and ended in fact. 4.2 Implies an event determined in fact. 5.2 Has all the persons and numbers. 2.^ Subjunctive— 1.2 In idiomatic English, follows a nominative. 2.2 Can never be used with thOH to form a direct assertion. 8.2 Has two tenses. 1.^ Imperfect tense of — 1.* Does not add st in changing I to thou. 2.* Does not add s in changing I to he. 3.* In the sentence, represents an event as neither begun nor ended in fact, therefore undetermined and contingent. 4.* Represents the time as usually future, since the event is unde- termined and contingent. 438 THE SYSTEM METHOD. 2.3 Perfect tense of — 1.* Does not add st in changing I to thou. 2.* Does not add s in changing I to he. 3.* Represents an event as supposed, and therefore undetermined and denied in fact. 4.^ Implies an event undetermined or merely supposed. 5.*^ Has none of the persons or numbers. (Person and number will be discussed in Syntax, Demonstration XXIV., c.) h. Note to Tbachee.— The result and aim of true education in grammar or elsewhere is the power to find the general in the particular; the law in the instance; the whole in the part; the classifiable (class) nature of all units (except of course the creating Unit). It is in pursuit of this power to comprehend the general or whole (of the indicative and the subjunc- tive) that we have studied the particulars set forth in the outline above. Through the partic- ulars of the outline, either of these moods may now be seen by that student who has done his work well in the fact matter. The instructor should now assign this outline to the class as a lesson, each member of the class to prepare to talk about these two moods, each to say what- ever he himself thinks is most important to be said. The instructor may give each member two minutes, say, in which to tell all he can tell, after preparation, about the indicative and subjunctive moods. That student who can talk most freely from the outline (let the student keep the outline before his eyes, while talking, that he may not be compelled to memorize its contents), — that student who has the most discursive thought and speech in this report, has studied and grasped the fact matter most thoroughly, and is best "educated" in these two moods. Success at continued and coherent speech upon the general, or whole, the particulars of which have been previously studied, is the proper evidence for, and principle of, promotion. Prog. XI: /. Regularity. 2. Inflection. 3. Event. 4. Time. 5. Mood. a. Regular. a. Complete, a. Determined. a. Present. a. Indicative. b. Irregular. b. Defective, b. Undetermined, b. Past. b. Subjunctive. c. Redundant. c. Future. d. Equivocal. PARTICULARS : If he have ^ the book, he wiU ^ give it to you. If he has ^ the book, why does * he deny having it? If he is ^ better, I have^ hope of his recovery. If he be ' better, you may^ stay with him an hour. Though it thundered' he would n't i" hear it. Though it thundered, ^^ he did n't ^^ hear it. If I had^^ a pen, I would i* wi'ite. If I had 1^ the pen at aU, I must i^ have left it there. If this were ^'' true, then should ^^ I know this secret. Such an exploit have ^^ I in hand, Ligarius, Had 2" you a healthful ear to hear it. — Shakspeare. If the man were ^^ ahve, I would ^^ make him eat a piece of my sword. — Id. If ye loved ^^ Me, ye would ^^ keep My commandments. Though ye loved, "^ ye did ^^ not do my wiU. If it were ^^ not so, I would ^^ have told you. If it was ^' not true, why does 3" he deny it? Here wast ^^ thou bayed, brave hart ; Here didst ^^ thou faU. — Shakspeare. Strike as thou didst ^^ at Caesar ; for, I know, When thou didst ^^ hate him worst, thou lovedst ^s him better Than ever thou lovedst ^^ Cassius. — Id. THE WORDS CLASSIFIED. 439 If thou read ^'' this, O CsBsar, thou mayst ^s live. — Id. Here will ^^ I stand till Caesar pass ^o along. — Id. Go on, and see whether Brutus be *^ alive or dead. — Id. If he died*^ I would not go to reUeve him. He died *3 when it was ** better not to live. If he was ^^ ill, he did*^ ^ot make it known. Though he was ^^ ill, he did *8 not make it known. If he were ^^ ill, he would ^^ make it known. Had ^i it not been for Dryden, we should ^^ never have known a Pope. Though he may^^ study, he makes 5* poor progress. Were ^^ I not Alexander, I would ^^ be Diogenes. Whether he had ^^ heard the news before you told ^^ him, I do ^^ not know. Csesar, thou canst ^^ not die by traitors' hands. Unless thou bringest ^i them with thee. — Shakspeare. Would thou hadst^^ less deserved. — Id. O young and noble Cato, art ^^ thou down? "Why, now thou diest ^* as bravely as Titinius ; And mayst ^^ be honored, being Cato's son. — Id. DEMONSTRATION XII. FACT MATTER: a. The Imperative Mood. — One that represents an event not to be brought to reality, but only to be put in command, wish, hypothesis, or entreaty; as, God save you, sir. — Shakfspeare; Cursed be my tribe, if I for- give him. — Id; Be you content. — Id; Mary, sing for us. Oh ! write it not, my hand ! his name appears Already written : blot it out, my tears ! — Po2}e. b. The imperative is found onlj^ in the imperfect tense. The person of the imperative will be explained in the next Demonstration. C. The Infinitive Mood. — One that designates an event neither as determined in fact nor contingent, but as merely named; as, Man is born to die; What did she do besides rock herself? I should laugh to giggle; I feel my pulse beat; I must go. d. The Participial Mood. — One that designates an event either as de- termined and ended without reference to time or as merely named in ing; as, I have money concealed; He came tumbling down the stairs; By acting quickly, we saved his life; He is preparing to be examined. (A fuller explana- tion of these three moods will be given in Demonstrations XIII. and XXIII.) e. How TO Ascertain the Mood. 1. Determine whether it is the imperative — used in a command, wisb<. hypothesis, emotion, entreaty. Next — 440 TEE SYSTEM METHOD. 2. Determine whether it is the indicative, which, in the English idiom, follows a nominative directly, changes its form for person, and can be used with /, thou, or he to form a direct emphatic statement. 3. Whether it is subjunctive, which, in the English idiom, follows a nominative directly, but does not change the form for person. 4. If it is neither of these, it is then the infinitive, provided it justifies with to. If it will not justify with to, it is the participial mood, or the par- ticiple. • r. V T, f. A SIMPLE APPLICATION OF THESE POUR TESTS glVCU abOVC, WhlCh will enable any teacher who will carefully examine it easily to decide any question of mood : — He died^ loved ^ by all. He was ^ ordered * to go.^ I saw" him fall.'' Somebody ring^ the bell. If he go,^ say " good-by. lean" go.^^ 1. By direction 1, died is not imperative, since it does not contain the property of a command or entreaty. By direction 2, it is indicative, since it follows the nominative he, changes its form for person (thus, I died, thon diedst, he died), and is the same as used in a direct emphatic statement. 2. By direction 1, loved is not imperative. By directions 2 and 3, it is neither indicative nor subjunctive, since both follow the nominative in construction. By direction 4, it is participial, or the participle, since it will not justify with to. (To is never used with loved; thus, to loved. It would justify with love; thus, to love.) 3. By direction 1, was is not imperative. By direction 2, it is indicative, since it follows the nominative he, changes the form for person (thus, I was, thou wast, he Was), and is the same as used in a direct emphatic statement. 4. By direction 1, ordered is not imperative. By directions 1 and 2, it is neither indicative nor subjunctive, since it does not follow the nominative in construction, was coming between. By direction 4, it is participial, since it will not justify with to. You can say to order, but not to ordered. 5. By direction 1, go is not imperative. By directions 2 and 8, it is neither indicative nor subjunctive. By direction 4, it is infinitive, since it justifies with tO. In fact, the to is here directly before it. Next go to the 11th word. 11. By direction 1, can is not imperative. By direction 2, it is indicative, since it follows the nominative I, changes its form for person (thus, I can, thou canst, he can), and is the same form as used in a direct emphatic statement, "he can ": "I know he can do THE WORDS CLASSIFIED. 441 it." (That ean is not potential, see proven in " The Potential Mood a Blun- der," "Would Make the Verb Belong to the Mood," "Potential's Founda- tion False," etc., pp. 373, 374. See also "Learner's Process of Work Be- nighted," p. 377.) 13. By direction 1, go is not imperative. By direction 3 and 3, it is neither indicative nor subjunctive. By direction 4, it is infinitive, since it justifies with to. Prog. XII : /. Principal Parts. 2. Regularity. 3. Tense. 4. Mood. a. Imperfect tense, a. Regular. a. Imperfect, a. Imperative. b. Perfect tense, and b. Irregular. b. Perfect. b. Indicative. c. Perfect participle, c. Equivocal. c. Subjunctive. d. Infinitive. e. Participial. PARTICULARS : If he study,! he will^ improve.^ He will* maintain ^ his integrity. He maintains ^ his integrity, though he loses ^ his estate. If I am * wrong, why have^ you not toldi" me. If I he^^ wrong, wiU^^ you not correct ^^ me? If he does^* not hear,i5 I am^^ much mistaken.^'' He might ^^ have^^ been'^" elected.^i Leave ^2 your theory and flee.'^^ The ship is ^* to sail ^^^ nest week. If he refuse ^^ the gold, do '^'^ not give ^^ him the silver. If he refuses ^^ the gold, I wiU 2" not send ^i him the silver. I am ^^ to go ^^ to-morrow. What was ^^ it that moved ^5 pale Cassius to conspire? ^^ What's to do? ^^ Go ^^ bid^^ the priests do *" present sacrifice. — Shaks- peare. Go *i love^ thy infant; love*^ thy wood-chopper; be** good-natured and modest. — Emerson. Tou wronged ^^ yourself to write *^ in such a case. Why should *''' the poor be *8 flattered? *9 Let so there be^i light. Cursed 52 be 53 I that did 5* so. Somebody caU^^ my wife. Be 56 it how it will, 57 (lo5S right now. — Emerson. Sup- pose 59 they are ^^ right, are ^^ we wrong? HaUowed ^^ be ^^ Thy name ; Thy kingdom come ;6* Thy wiU be ^5 done ^^ in earth as it is ^"^ in heaven. To give ^^ is ^^ better than to receive.™ Be'^^ it known '"■^ to you that I obey ^^ the eternal law. They intend'''* starting '''5 to-morrow. Compel ''^ him to laugh. ^'^ He prefers ''^ riding. '^^ They must ^^ do their duty. I enjoy ^^ inhahng^^ the crisp air. The ratthng^^ Qf musketry is 8* said ^5 to cause *^ horses to grow ^'' restive. Let ^^ us go ^^ thank ^^ him and encour- age ^1 him. Perchance the maiden smiled ^^ to see ^^ Yon parting s* hngerer wave ^5 adieu, And stop ^^ and turn "^ to wave ^^ anew. A mutual relation leads ^^ us to respect ^^o each other. A mutual relation leads ^^i us to respect for each other. What has ^^^ she done ^^s except laugh ? i"* Man is i°' born i*'s to die.^"? j should ^^s laugh to gigglc^^a jyjy soul turn ^i" from them, tiunm we to survey.^12 — Goldsmith. Speak ^^^ any man with us, and we wiU i^* obey.^^s — Emerson. The heavens speed ^^^ thee in thine enterprise. — Shakspeare. The worm of conscience still begnaw ^^'^ thy soul. — Id. From the steep promontory gazed "^ The stranger, raptured ^^^^ and amazed.^*^" Did 121 he know i23 it, he might i^s make i^* a better investment. 442 THE SYSTEM METHOD. DEMONSTRATION XIII . FACT MATTEB: a. Hitherto most grammarians have taught that the imperative is lim- ited to a subject in the 2d person. The examples that follow, selected from the most beautiful parks of the literary forests, will show that either the first or the third person of the imperative is good idiomatic English: — Examples of the Imperative Mood. 1. Cursed be J that did so. — Shakspeare. 2. (lOd save you, sir. — Id. 3. Cursed be my tribe, if I forgive him. — Id. 4. Somebody call my wife. — Id. 5. Somebody ring the bell. 6. Be it how it will; do right now. — Emerson. 7. Sappose they are right, are we wrong? — Id. 8. Be he a king, he is still a dependent creature. 9. Well, then, be it so; may he ever prosper. 10. Hallowed be Thy name ; Thy kingdom come ; Thy will be done in earth as it is in heaven. — Bible. 11. Laugh those that can, weep those that may. 12. Swift fly the years, and rise the expected morn. 13. Retire we to our chamber. — Shakspeare. 14. Success be yours alway. 15. My soul turn from them, turn we to survey. — Goldsmith. 16. Fall he that must beneath his rival's arm. — Pope. 17. Be it how it will. — Emerson. 18. Speak any man with us, and we will obey. — Id. 19. Please it your grace to go to bed. — Shakspeare. 20. Be it known to you that I obey the eternal law. — Emerson. 21. Be it scroll, or be it book, Into it, knight, thou must not look. — Scott. 22. The mighty (iod defend thee. — Shakspeare. 23. Then walk we forth even to the market place. — Id. 24. All the charms of Sycorax, toads, beetles, bats, be on you. — Id. 25. Come we now to this translation of the Iliad. — Pope. 26. Black night o'ershade thy day. — Shakspeare. 27. The worm of conscience still begnaw thy soul. — Id. 28. Long live the king, And Gilpin, long live he. — Gowper. 29. Peace be with you. 30. Exalt we his name. — Bible. ^ THE WORDS CLASSIFIED. 443 31. Praise be to God. — Id. 32. "And rest we here," Matilda said. 33. Commence we now that higher state, Now do thy will as angels do. — Montgomery. 34. Send he his vassal train— himself advance, Here will I take my stand — decide our swords the chance. — Hunts Tasso. 35. With virtue be we armed. — Id. 36. Do AVe what duty bids. — Id. 37. Be it so ! — Mrs. Browning. 38. God save thee, weeping queen, With blessing more divine ! And till with happier love than earth's, That tender heart of thine ! — Id. 39. look on who Will in apathy, and stifle they who can The sympathies, the hopes, the words, that make man truly man. — Loicell. 40. May she bear in mind that she walks with pride In the winding-sheet where the silk-worm died. — Hannah F. Oould, . 41. Oh! write it not, my hand! his name appears Already written: blot it out, my tears. — Poj^e. 43. And all the gods go with you. Upon your sword Sit laureled victory! and smooth success Be strewed before your feet! — SJiakspeare. 48. Despatch we the business we have talked of. — Id. 44. Find we a time for frighted peace to pant. — Id. 45. Stand we to defend our rights. — Noble Butler. 46. Fall not that curse upon us. — Id. 47. Be this our motto. — Id. 48. Rise we by morning light. — Id. b. Like the subjunctive, the imperative has no variation for the person and number of its nominative. Unlike the subjunctive, it has no perfect tense, but is found in the imperfect tense only. We had decided to extend the examples of the imperative to quotations from a large number of authors, but from lack of time have collected only from a few, — enough, it is supposed, to satisfy the reader that, so far from being limited to the 2d person, as many grammarians teach, both the 1st and 3d persons of the imperative are good English. The first and second persons were also used in both Old English and Middle English. He that hath eeris of herynge, hear he. — Wicliffe. Give he to her a libel. — Id. But fall I first Amongst my sorrows, ere my treacherous hand Touch holy things. — Beaumont and Fletcher. Si thin nama gehalgod=Be thy name hallowed. 444 TRE SYSTEM METHOD. Fare we on tunas=()lo we to the towns. C. Characteristics of the Infinitive Mood. — These are, — 1. That it has only the imperfect tense; hence — 2. That it justifies with the preposition to. (To and drink may be con- structed together, but not to and draflk.) 3. It has neither person nor number; it cannot take a grammatical sub- ject. 4. It is used in two different constructions. (These will be presented in Demonstration XXIII., in connection with the tree-system whole.) d. Characteristics of the Participial Mood. — These are, — 1. That it does not, usually, justify with to. 3. That it has neither person nor number; it cannot take a grammatical subject. 3. That it has the two tenses, the imperfect and the perfect. 4. That the imperfect (or active) participle ends in ing; the perfect usually in d, s, t, n, g, k. 5. That it has four different constructions. (These are presented with the infinitive constructions in Demonstration XXIII.) They constitute, in the tree, the four sub-branches of the limb participial. Prog. XIII: 1- Regularity. 2. Inflection. 3. Tense. 4. Mood. a. Regular. a. Complete. a. Imperfect, a. Imperative. b. Irregular. b. Defective. b. Perfect. b. Indicative. c. Redundant. c. Subjunctive. d. Infinitive. e. Participial. rABTICULABS : What can^ she do^ besides sing?^ He was* inclined^ to sleep.^ It must be a bitter esperience to be more accustomed to hate '' than to love.* We could ^ but laugh.!** Better not be^^ at aU than not to be^^ noble. It will go^^ near to be^* thought!^ so. Bring 18 me word, boy, if thy lord look^''' well. — Shaks2}eare. He were ^^ no lion, were i® not Komans hinds. Though it thundered ^* he would not hear 21 it. Though it thundered,^^ he did^^ not hear it. If it were ^^ not so, I would ^5 have^® told 2'^ you. If he was ^* iU, he did '■^^ not make it known.so Conmience ^i we now that higher state, Now do ^2 thy wiU as angels do.^^ — 3£ontgom.ery. Send 2* he his vassal train — himself advance,-^^ Here will I take my stand — decide ^'^ our swords the chance. — RicnVs Tasso. Look ^'^ on who wiU in apathy, and stifle ^* they who can ^^ The sympathies, the hopes, the words, that make man truly man. — Loioell. Who could refrain,*" That had a heart to love,*i and in that heart Courage to make ^ his love known. *^ — Shakspeare. Why do** you dress *^ me in borrowed *8 robes? — Id. Warren died*'' loved *^ by all. I want to be*9 quiet and to be ^^ let ^i alone. We did not think of being ^2 defeated. ^^ THE WOBDS CLASSIFIED. 445 Somebody ring •^•' the bell. All the charms of Sycorax, toads, beetles, bats, be^^on you. Playing ^^ tricks on students and hazing ^7 are imknown here. Four and twenty happy boys came bounding ^^ out of school. Ill rest betide ^^ the chamber where thou liest.^'* They were sealed with a seal never to be ^^ broken.^^ Well, sit *3 we down, and let^* us hear^^ Bernardo speak ^^ of this. — Shakspeare. Thank ^^ Heaven, we are free. Had ^^ I three ears, I'd ^^ hear thee. If the face were ™ made ''^ of glass, or of air, and the thoughts were '^^ written '^^ on steel tablets within, it could not pubhsh '^* more truly its meaning than now. I could not die in peace till this were ^^ told.''^ We saw the storm approaching.^^ I hear the blue-bird prophesying '^^ spring. They stood terrified.''* jjg owned himself defeated.^** You may imagine me sitting 81 there. Remembering ^^ matter pays ^^ the debt. How 'scaped I killing ** when I crossed you so? — Shakspeare. Are^^ the Ethiopians styled*^ civilized? ^^ Let ^8 us never bow^* and apologize*" more. A noun is a word, which, when spoken, *i may be heard, and when written, ^^ seen.*^ How do ** you do? ^^ He does'^ not like ^'^ to be ^^ made ** fun of. Would i"" God I had died 101 for thee ! Whether he is a ^'^ sinner or not I cannot ^"^ telL^*** Methought ^^^ I was incarcerated beneath the mighty deep. Be ^'^^ thou an example of the believers. O Lord ! methought ^^^ what great pain it was to drown ! 1"^ Trust i"* God and be^^" doing, ^^^ and leave the rest with Him. When Antony found Julius Caesar dead. He cried almost to roaring. ^^^ — Shakspeare. And then the people fell a shouting. ^^^ — j^_ Think ^1* you, I am no stronger than my sex, Being 11^ so fathered i^^ and so husbanded? ^^'^ — Id. How this world is given to lying! ^^^ — Id. It is to the interest of all that there be ^i* Captain Cooks to voyage '-* round the world. Men are born to write.i^^ I at first was near to laugh. i"^^ SkiU to do^^^ comes 1^* of doing. ^^^ Huge on a huge red horse,- and all in mail Burnished 1^^ to bhnding^^'' shone ^^^ the Noonday Sun. — Tennyson. Weak-built hopes persuade him to abstaining. i"* Seeing i^o that he is so obsti- nate, let 121 us leave him. Granting i^^ that he had the right, he was very rash. See- ing,i23 J ga^ jjo^. hearing 12* not, I heard. The river suddenly rising,!^^ much in- jury was sustained. Man is siich an ignorant peacock, that he goes bustling i36 yp and down, and hits i^'' on extraordinary discoveries. — Emerson. We are born believ- ijjgi38 It is passing 13* strange. It is scalding 1*" hot. The horses came galloping. 1*1 No recipe can be given for the making 1^ of a Shakspeare. She upi*^ with her fist and took him on the nose. The branch, being 1** broken i*^ off , fell. Supposing i*^ he werei*^ here, what would i** you do?i** Dr. White spoke as foUows.i^" What man enters 1^1 dies.i^^ Up! up! my friend, and quit^^^ your books. Would i^* he were 1^5 fatter. The beginner should be accustomed to giving each i^s step. I have, methinks,!^'' a fever upon me. He wist 1^* not what to say.i^* Be 1^" this as it may, 1*1 his works enjoy a very smaU share of popularity. There are, say,!^^ a thousand dialects. They kept me going.i^^ -^g all fell to eating.i^* The women, they fell to praying. 165 Leave wringing i^^ of your hands. — Shakspeare. M6 THE SYSTEM METHOD. And all the gods go ^^'' with you. Upon your sword Sit 1^8 laureled victory ! and smooth success Be ^^^ strewed before your feet. — Id. Dispatch i'''" we the business we have talked of. — Id. Oh ! write ^''1 it not, my hand! His name appears Already written : blot ^^^ it out, my tears. — Poije. Searching ^^^ the window for a flint, I found This paper, thus sealed i^* up. — Shakspeare. And then I swore thee, saving ^^^ of thy life, That, whatsoever I did bid^'^^ thee do,^^^ Thou shouldst "» attempt ™ it.— Id. Many sons lie scattered ^^^ on the bleeding ^^^ ground. Many a widow's husband groveling ^^'-^ Ues.^^^ — Id. Woe worth i^* the chase, woe worth ^^^ the day, That cost thy life, my gallant gray. — Scott. What he gives thee, see thou keep ; ^^^ Stay 18'^ not thou for food and sleep. Be 1^^ it scroll, or be ^*' it book, Into it, knight, thou must not look. — Id. Thou to the untamed i^" horse Didst 19^ use the conquering ^^^ bit ; And here the well-shaped oar. By skiUed"3 hands deftly plied, i^* Still leaped i^* through the sea. Following i^** in wondrous guise. The fair Nereids with their hundred feet. —Plumptre' s Oedipus at Colonics. What is in will^^^ out.^^^ — Emerson. We go to Europe to be Americanized.!^^ — Umerson. A rising ^o" of the sea, say^oi an inch in a century, will bury all your towns, Meseems ^•'^ that here is much discourtesy. — Tennyson. I thou"''^ thee, thou traitor. — Coke. Thank °^* him who isled^''^ us here. — Tennyson. Diamond ^"^ me no diamonds, prize 207 me no prizes. — Id. Language has as much occasion to adjective^** the distinct significations of the verb as it has to adjective ^"^ time. — Tooke. The particulars are as follows.^^*' I shall consider his censures so far only as concerns 211 my friend's conduct. With sanguine drops the walls are 212 rubied ^is round. And Nature in the tangles soft envolved ^i* Of death-like sleep. It were ^15 good she were^i^ spoken with. Though I were^i^ perfect, yet would ^i* I not know ^1^ my soul. THE WORDS CLASSIFIED. 447 DEMONSTRATION XIV. FACT MATTER : a. Pronouns. — See Demonstration I., b, c, d. The doctrine that a "pronoun is a word used instead of a noun," is not strictly true. So far from being "used instead of," or "substituted for," any noun are the following italicized pronouns, that no noun, or name, can be found for which or instead of which they could be substituted: — "In that and all things will we show our duty." "We doubt it nothing; heartily farewell." — Shakspeare. Love is a silly infatuation, depend upon it; It is raining; It is important what he is; I tell you what, that's a fine horse; It is John and Mary; Who or what is it? "The common definition of a pronoun, that it is a 'word used instead of a noun,' is not correct. A pronoun is simply a noun, expressing its pecul- iar meaning as completely as a noun of any other class expresses its own meaning." — NohU Butler. "Pronouns are not used 'to avoid the too frequent repetition of nouns.' There is no noun expressing the same relation that is expressed by I, for instance; and therefore there is no noun for which /may be substituted. It is true that other words may be employed to denote the person speaking; as when Samuel says, 'Speak; for thy servant heareth;' but here the speaker merely uses &form representing him as speaking of himself as if he were speaking of some one else, and this is indicated by the form of the verb, heareih. With /, a different form, Jiear, would be required." — Id. " ' John studies, and he will improve.' Here he is emjjloyed, not because it prevents a repetition of the noun JoJin, but because it is the word that expresses the intended relation. If we say, 'John studies, and John wiW improve,' we use a noun instead of a pronoun; but when we say, 'John studies, and he-will improve,' the pronoun he takes the place that belongs to it; he expresses a relation to the noun Jo7in, while the second John does not. 'John studies, and John will improve,' mig/Uhe said of two different Johns, which is not the case when the appropriate word he is used." — Id. b. Personal Pronoun. — One always of one and the same grammat- ical number: I, thou, he, she, and it are always singular ; you or ye and they are always plural. C. Interrogative Pronoun. — One not always of one and the same grammatical number, and used in asking direct questions; as. Who is here? Who are here? The interrogatives are who, which, and what, introducing direct questions. d. Relative Pronouns. — One not always of one and the same gram- matical number, and used only with two or more verbs; as. The boy who 448 ■ TEE SYSTEM METHOD. laughs grows fat; He ashed what it was; We knew not what to do; The relatives are who, which, what, that, and as. e. To Distinguish the PERsoNAii from the Relative. — Observe — a. That the personal, by definition, is either always singular or always plural in form, but that the relative is sometimes singular, sometimes plural: I who speak will lead the way; These are they who came from afar. b. That the personal can be used with a single verb (as, caii) to form a direct emphatic statement: we ca% she can, it can; but that the relative joined with can, does not make a direct emphatic statement: what can, as can. c. That the personal can be accompanied with an index, as the finger pointing; as, he, it (the finger pointing); but that the relative cannot be accompanied with a pointer; as, who, as, which; these do not point out. d. That the personal may be found in sentences having only one verb: studies, she sweeps it through the streets; but that the relative is found only in sentences having at least two verbs: He who studies learns; What he wanted he took; Whoever gii^es receives; It is important what Ido ; I tell thee what, Corporal, I could tear her; I heard wlisit prevented his coming. e. That the personal often forms a part of an independent statement; as, He is divine; but that the relative forms part of a dependent, or accessory, statement; as, He who rules the world is divine. "Who rules the world" is accessory to he, and would not make an independent statement of itself. f. To Distinguish the Relative from the Interrogative. a. The essential signature of a relative is the fact that there must be two verbs, at least, in the sentence; whereas both the personal and the inter- rogative are used in sentences having but a single verb. Belatives : I tell you what, that is a fine horse; "We knew not what to do; You decided Yi\^ it was; I A;?iow who was the first president; ^^atnx gives re- ceives; Take what you want. Interrogatives: '^hX prevented his, commg'^. Who^■she? Which ^« Uranus? What is man? Personals: Who is he? It is I; She sweeps it through the streets; It rains; It is well with my soul. b. The essential evidence of an interrogative is always that it introduces a direct question; which it may do with a single verb; as Who was the man? A direct question is often incorporated in a sentence, and is then, usually, inclosed within quotation marks, thus: He asked, "Who was the man?" Who is here an interrogative, since it introduces a direct question. But if the direct be changed to the indirect discourse, the order of the words will also be changed, and who will then become a relative; as. He asked who the man was. g. Properties of the Pronoun. — The properties of the pronoun are the same as those of nouns, — person, number, gender, and case. THE WORDS CLASSIFIED. 449 h. Declension op Simple Peksonal Pronouns.^ — The student should fix the following forms in the memory — by committing, if necessary: — Common Gender. First Person. Second Person. Singular. Plural. Singular. Plural. Rominative : I. We. Thou. You. Possessive : My, mine. Our, ours. Thy, thine. Tour, yom-s. Objective : Me. Us. Third Thee. Person. You. Singular. Plural. Masc. Fern. Neu. Com. and Neu. Nominative : He. She. It. They. Possessive : His. Her, hers. Its. Their, theirs. Objective ; Him. Her. It. Them. i. Declension of the Relative Pronouns. — These have the singular and plural alike. The interrogatives are declined precisely like the rela- tives who, which, and what. Nominative: Who. Which. What. As. Possessive : Whose. Objective: Whom. Which. What. As. OBSERVATIONS : a. The plural you is now used in familiar address to denote either one or more than one. But let the student observe that it is always grammat- ically plural. (See Demonstration XV.) b. Rulers, reviewers, editors, leaders in prayer, and other^s, who speak for their constituents, their readers, or an assembly, use we in lieu of I to preclude the objection of the "fallibility of self," on the one hand, and the "responsibility of self," on the other hand. Says the king, in Hamlet, "Therefore our sometime sister, now our queen . . . have we . , , taken to wife." C. Names of children, and such animals as, to the mind, present no ex- hibition of their properties of sex, are to be regarded as neuter; and hence are represented by the neuter pronoun it. (Cf. Demonstration III., Obs. e, f.) d. In parsing the possessive in such expressions as "He is a friend of mine," "Your weeping sister is no wife of mine," some word, as, possess- ing, owning, liamng, must be supplied. (Cf. Demonstration V., Obs. e.) e, 'Which, what, and as are used in the third person only. Who and that may represent any of the conversational parts, or persons; as, — I, loho am his pupil, do not believe it. God of our salvation, who art the confidence of all the earth. 1 will bless the Lord, who hath given me counsel. I that bring the news made not the match. Thou icho stealest fire, strengthen me, enlighten me. 29 ^50 THE SYSTEM METHOD. I that denied thee gold, will give my heart. Thou that inhabitest the praises of Israel. Dark anthracite that reddenest on my hearth. f. Wlio is used for persons and things personified, occasionally for things not personified; as, — The most advanced nations are those wJio navigate the most. — Em- erson. The nineteenth century, whose speculative genius. — Id. Want is a growing giant whose coat. — Id. Inventory of the Southern Heavens, a work whose value. — Id. 1 could a tale unfold whose lightest word. — Shakspeare. g. Than Whom.^ — In such expressions as, " My father, than whom no man is wiser, approves that course," whom is an adverbial object of the comparative adjective wiser, and than is a subordinate conjunction connect- ing word governed {ichom) and word governing {loiser). (See Demonstra- tion XXVI., b 6 b; and Demonstration XX., note 2.) h. Which is used for animals and things, sometimes for persons; as, — "The anecdote of handsome captives which.'''' — Emerson. " Hostile to geniuses which seeing and using ways of their own." — Id. In Old English, tchich displaces wJio; as, — "Let them be ashamed which transgress without cause." " Our Father ^Dhich art in heaven." But this is not now regarded as good English, which would require, " Our Father w7io art in heaven." i. That is used for persoiis, animals, and things; as, — " Wake all ye that sleep." "How should I that am a king, give my own daughter saving to a king." "The evil that men do lives after them." j. Tests to distinguish between pronouns and nouns. (See Demon- stration I., b, c, d.) Prog. XIT : 1. Class of Pronoun. 2. Person. 3. Number. 4. Gender. 5. Case. a. Personal. a. First. a. Singular, a. Masculine, a. Nominative. b. Relative. b. Second, b. Plural. b. Feminine, b. Objective. c. Interrogative. c. Third. c. Neuter. c. Possessive. d. Common. PAMTICULARS : It^ might have been he,^ but could not have been I.^ I* understood it ^ to be them.^ We ^ took it * to be the man who ^ had gone before us.^'^ Who ^^ say ye ^^ that I ^2 am? If it^* was not he,^^ whom^Mo you^'' suppose It^^ to have been? Has he^^ received his^" pension? Whose ^^ house is on fire? Which ^^ is Venus? What^^ do you want? I '^^ esteem the boy who"^ loves study. What^'' do I^'^ know? Thou 2^ to whom -^ all creatures bow, How mighty is Thy ^'^ name ! 1 THE WORDS CLASSIFIED. 451 And thou,^^ too, whosoe'er ^^ thou ^3 art, That ^* readest this brief psalm ! They ^^ saw themselves ^^ deceived. Is it ^^ far to Chicago ? It ^^ is well with my ^^ soul. It*" is the same to him*i who ^ wears a shoe as if the whole earth were cov- ered with leather. See it ^^ hghtening. I ^ that *^ denied thee *^ gold, wiU give my ^'' heart. Who*^ are you*^ that^" have no task to keep you^i at home? It^'** was now a matter of curiosity who ^s the old gentleman was. Tou,^* John, yourself ,^5 ^j-e in fault. Who 58 and what ^7 are you ^s that ^9 would lay the ghastly anatomy bare? What *''* is the ocean but cubic miles of water? Ye,^^ therefore, who®^ love mercy, teach your^^ sons to love it ^- too. There stood by me ^^ this night the angel of God, whose ^^ I ^"^ am, and whom ^^ I ^^ serve. This same child is he '"^ who ^^ reigns. Who "^^ should be king save him '^^ who ''^ makes us ''^ free? 1 7S have dogs, my ^^ lord, WiU rouse the proudest panther in the chase. — Shakspeare. Come, come, and sit you '^^ down. — Id. We ^^ hunt not, we,^" with horse nor hound. — Id. 1 ^1 care not, I,^^ knew she ^^ and aU the world. — Id. Myself ^* have to mine ^^ own turned enemy. — Id. Myself ^^ have letters of the self-same tenor. — Id. What^^ touches us *^ ourseK *^ shaU be last served. — Id. How should I "" that ^^ am a king. Give my^^ own daughter saving to a king. — Tennyson. Who ^^ is he? 9* What ^^ is man? Which ^^ is yours? ^'^ She struts it ^^ through the streets. He^^ is a friend of mlne.^o" Your^"^ weeping sister is no wife of mine.!"^ j^i03 -(vho^"* am his pupil, do not believe it.i''^ I^"^ that^'^^ bring the news made not the match. The sun seemed shorn of his ^^^ beams. Never did Nature betray the heart that loved her. 109 Tremble, thou ^i' earth, at the presence of the Lord. Can Time, his^ flight reversed, restore the hours? How wonderful is Death, — Death and his ^^^ brother Sleep. Folly hails them ^^^ from her ^^^ shore. ReUgion veils her ^^^ eyes. Jura answers, through her ^^^ misty shroud, Back to the joyous Alps, who i^'' call to her ^^^ aloud. I "9 that speak to thee ^^o am he.^'-^i He ^^a that 1^3 endureth to the end shall be saved. Whom 124 the gods would destroy, they i^s first make mad. Art thou i^^ that traitor angel, art thou^^'' he?!^* Truth, crushed to earth, shall rise again ; The eternal years of God are hers;!^^ But Error, wounded, writhes in pain, And dies among his i^o worshipers. The thunder Winged with red Ughtning and impetuous rage, Perhaps has spent his i^i shafts. — Milton. 452 TEE SYSTEM METHOD. DEMONSTRATION XV. FACT MATTER— It^ Thou, You, As, and miat : a. It is used, in the absence of an explicit name, to indicate that con- cerning which we have no comprehending thought. For an instance, con- sider, "It is raining." Could the mind at one view comprehend that to which the word it refers, we should have a specific name for that thing, just as we have for a house. We can write neither " cloud is raining," ''weather is raining," nor "elements is raining;" nor anything save " it is raining." The only express word to be employed is it. It is an erroneous and pernicious notion to call it an "expletive" in any sentence: as if mankind were demented, preserving with the greatest care, in the very parks of the literary forests, the most useful and graceful it, because it is an "expletive," a "superfluous" and "useless" element! It is always a pronoun, personal, representing its own special and substantial value. The word it is used to indicate, — 1. A certain condition of fortune or lot; as, Think of me when it shall be well with thee; It was good to be there; It is well with my soul. 3. A certain condition of welfare or happiness; as, It is better to suffer wrong than to do wrong {to do is an adverbial element belonging to bet- ter); I believe it to be necessary that I should go. 3. A certain condition of destiny or providence; as. It was for Alexander to die in the prime of life. 4. A certain condition of experience or action; as. It is useless to complain of what is irremediable; It is impossible to please every one; It is pleasant to play; It is difficult to write well; It came to blows between them; It is the same to him who wears a shoe as if the whole earth were covered with leather. 5. A certain condition of the weather or climate; as, It is sultry; It is Unheal thful there; It is warm. 6. A certain condition or action of the elements; as, It is raining; See it lightening. 7. A certain measure of time or space; as. Is it far to the city? It is more than one hundred years since the nation was born; It is forty miles to Chicago. 8. A certain imitation of action or conduct; as, He is disposed to lord it over them; He struts it through the streets; Come, and trip it as you go. 9. Or, finally, to indicate whatever vague and derived thing invisible to the eye, is yet visible to the mind or fancy — whether thought can name it or not; as. Love is a silly infatuation, depend upon it; It is important what be is. THE WORDS CLASSIFIED. 453 "In that and all things will we show our duty." " We doubt it nothing; heartily farewell." — ShaTcspeare. It was in the tree that the rabbit ran; It is to you that we speak; It is they; It is John and Mary; It is possible that we have erred; It is a man; It is doubtful whether he go home; It was the day that we came home; It is enough if you do your best; When the wicked are cut off, thou shalt see it; I said he was a fool, and am. sorry for it. b. It is frequently the base of an explanatory appositive; as. It is a curious fact in modern history, the growth of the machine &h.o^.— Emerson; It is the foible of American youth, pretension. — Id; I will walk through fire to gain it, your full leave to go. — Tennyson; He declared it to be wrong, the tiling which I did. Tlaou. — Thou is now used only in solemn address, and by the Fiiends. Formerly, it was employed universally in familiar address, denoting any degree of intimacy or familiarity. This explains why, in familiar discourse, we now employ the plural you in the singular as well as in the plural sense; for, since treating persons with undue familiarity is an efficient means of expressing contempt, it was but natural to employ tJiou for that purpose. Thus, Coke addressed Sir Walter Raleigh to insult him: "All that Lord Cobham did was at thy instigation, thoU viper! for I thott thee, thou traitor!" Thus, from a caution that has long been practiced to avoid a contemptuous remark, and from the consequent aversion to the word thou, and from that politeness and flattery always avouched when plurals are donated to indi- viduals, we have come to use the plural you in the singular as well as in the plural sense. You Always Plural. — ISTot the side-show application which a word may acquire, but that sense in which its form was conceived, and in which the word has been longest maintained, determines the grammatical property. Von is plural in grammatical number because its form was first conceived in the idea of two or more, and because it has ever since been used with a plural verb. \m has been used in the singular sense only since the middle of the sixteenth century or thereabout. We say, You are, never You art; You can, never You canst; and can never use it with a singular verb. It has been supposed that the fact that you is applied to a single individ- ual, is proof that it is singular in number. But the same sort of proof would establish we, also, in the singular number; as, Wc, Victoria, queen of England, do proclaim; When we set oiir hand to this great work, we purposed with ourself never to wed. — Tennyson. If you is to be regarded as singular because it may be used to denote one, then we shall parse we in the singular for the same reason. So far as grammar is concerned, both you and we are always plural; yet both may be used to refer to an individual thing. As. — a. As is a relative pronoun, — 1. When it refers to a preceding noun; as, I have read such bOOks as please me; He has as many books as can be read with profit; There is as great a 454 THE SYSTEM METHOD. variety as can be found in any other country; He then read the conditions as follow. — Nutting's Grammar; We made as naany bricks as the material ren- dered possible. 2. When it refers to a preceding adjective; as, The business is such as concerns me; Avoid sucll as are vicious; They are the same as escaped from us; He collected as mucll as is due; As many as choose, may remain; The positions were sucll as appear incontrovertible. — Murray. Reconciliation was offered on conditions as moderate as were consistent with a permanent union; We made as mucll of him as was possible; They are as fruitful as we saw growing. 3. When it forms part of a subordinate sentence accessory to the verb Be; as. The words are as follow. — Addison; The conditions are as follow. — Nut- ting's Grammar; The particulars are as follow. — Buckets Grammar; His words are as follow. — Steele; The conditions of the agreement were as fol- lows. — CamphelVs PMlosopTiy of Rhetoric; The arguments advanced were nearly as follows. — Murray; The circumstances were as follow. — Gromhie. 4. When it forms part of a sentence accessory to another sentence; as, The wicked shall perish, as all the world knows; The point in dispute was, as all the world knows, to which the said spectacles ought to belong. b. As is an adverb when it forms part of any clause accessory to any verb except Be; as. The arguments were advanced as follows; The sentence should be 2:»unctuated as follows; Dr. White si^oke as follows; He trembled as he spake; He acted as he was directed; You will reap as you sow. e. As is a conjunction when it corresponds to a preceding adverb ; as, I am as tall as you ; I shall consider his censures so far only as concerns my friend's conduct ; I had as lief go as stay. The as's in the following sen- tences are also conjunctions : It was as black as night ; As I knew him to be gone, I did not call ; The English hate the French as frivolous ; He was re- garded as dangerous ; They regarded him as dangerous ; Brutes have the least sense of actions as distinguished from events ; You were as innocent of it as I ; His brother writes as well as he ; No lamb was e'er so mild as he; There I brought her as pitying her usage. — Massinger ; Virtue considered as virtue, is purely hight ; He seems as if he were inspired ; He fell as though he had been shot ; A noun is a name ; as, man, eagle ; I chose him as the most worthy ; I assume it as a fact ; As it was then, so it is now ; The widow's enjoyment was wrapped up, as it were, in her only child ; Lib- erty was theirs as men['s] ; He acted as governor ; He came as a business man ; As a dramatic poet, Shakspeare excels ; He went out as mate ; As mayor of the city, I felt much aggrieved ; The beggar is greater as a man than is the man merely as a king ; Currency, considered as paper, has little value ; As to your statement, I am doubtful ; As for me, give me liberty or give me death ; Phrases and clauses, as adverbial elements, are used with much greater precision ; The word means to place in an equal state as to obligation ; He often vaunts himself as victorious ; As yet, science has THE WORDS CLASSIFIED. 455 hardly penetrated through the surface of nature. (See Demonstration XX., d, note.) d. As IS Never a Preposition. — Whatever may connect the object- ive case to a verb may become a preposition. A word that cannot connect the objective case to a verb, cannot do the service of a preposition, and hence cannot be a preposition. In the sentences, "He came as an embas- sador," and "She reigns as queen," both embassador and queen are nomina- tive by the rule of "Be and the Same Meanings." But a preposition gov- erns not the nominative, but the objective case. Hence as cannot be a preposition, but is merely a conjunction in either sentence. (See Demon- stration XIX.) What, — a. What is a simple relative used where the word thing intro- duced before it requires which in lieu of what, and is exactly like which both in its properties and construction ; as, Buy what you need ; Do what is right ; It is important what he is. If thing is introduced, what is changed to which; thus, Buy the thing which you need ; Do the thing which is right. b. What Parsed. — Parse exactly as if thing were introduced, and you were parsing lohich. Thus, in the derived sentence, ' ' Do the thing which is right," we see that — 1. Whieh=pro., rel., 3d, sing., neu., nom., subject of is. Hence the pars- ing of — 2. What=pro., rel., 3d, sing., neu., nom., subject of is. And so in "Buy what you need," it is seen that which would be objective, object of need. Hence — 3. What=pro., rel., sim., 3d, sing., neu., obj., object of need. And so in "It is important what he is," — 4. What=pro., rel., sim., 3d, sing., neu., nom., meaning the same as he before is. C. What a Simple, not a "Double," Relative. — In view of the wide- spread error concerning the nature of what, I have discussed it somewhat at length. The common practice of expanding the sentence by substituting for what other words of equivalent meaning, should be denounced by all good teachers of grammar as the coxcomical science of the copier. That what is equivalent in meaning to "the thing which," is true, but that what is equivalent in parsing, or grammatical construction, to "the thing which," is not true. For — d. Equivalency in Meaning is not Sameness of Grammatical Con- struction. — Examine, 1. Those books are hers, and 2. Those books are her books ; in which sentences hers and her books are presented as identical in meaning, but radically diflEerent and even contradictory in grammatical construction, as indicated in the parsings, thus : — 1. Hers=pro., per., 3d, sing., fern., poss., limiting [books] omitted. f Her=pro., per., Bd, sing., fern., poss., limiting books. 1 Books=noun, 3d, plu., neu., nom., meaning same as boohs before are. 456 TEE SYSTEM METEOB. Now, if parsing the tldng icMch parses what, then parsing her books parses hers; for in each case it is parsing the equivalent meaning. But if the parsing, or grammatical construction, of hers is the same as the grammatical con- struction of her books, hers is both a noun and a pronoun! both singular and plural! both feminine and neuter! both possessive and nominative! But the student has now observed that the grammatical construction of her and hers is the same, the parsing being identical. Her is used when the noun is expressed, and kers when the noun is omitted. In the same way, the grammatical construction of lohich and what is the same, which being used when the noun thing is expressed, and what when the noun is omitted. That which and what are in the same construction will be apparent when it is understood to be the general law of language forms that — e. The Same Grammatical Construction mat Eequire Different Forms for Different Circumstances. — If we change the circumstances, in the sentences above, by supplying hooks, this law requires the form hers to be changed to her, eliding the s, though the parsing of the two words is the same. In the same way, if we change the circumstance, by supplying thing in the sentence "Do what is right," this law changes what to loMch, thus, " Do the thing icMch is right," though the parsing of the two words is the same. We could point to numerous examples of other pronouns, nouns, adjectives, where different forms arise by change of circumstances ; but, in all cases, those different forms are parsed alike, — are alike in grammatical construction, just as what and which are. Examples are, — This is the man that I spoke of. This is the man of whom I spoke. None knew thee but to love thee. No person knew thee but to love thee. We consulted the lawyer's library. We consulted the lawyer, Henry Felix's library. All the world was Olirs once more. All the world was our possession once more. That evidence is from an historical source. That evidence is from a history. f. It is evident that to parse or describe a word is to give its own prop- erties ; not to describe, or give the properties of, some other word. Man- ifestly, then, if pronouns have their own person, number, gender, and case, in parsing what, we shall find its own person, its own number, its own gen- der, its own case. And to pretend to be guided in parsing, or giving the properties of, the pronoun what, by giving the properties of the adjective the, the noun thing, and the relative which, is to take the cars for New York by embarking in the ship Pretension for the Island of Lanterns. THE WORDS CLASSIFIED. 457 Difference betvreen a Relative and an Interrogative. — The gTammarians teach that the two points upon which any word is cata- logued as a relative pronoun are, — 1. That it represents an antecedent noun or pronoun; 2. That it binds that antecedent to itself. And they aver that in such sentences as, " It is important wkat I do," "I got what I wanted," "I tell you what, that is a fine horse," what is a relative pro- noun. But taking "noun" to mean "name," — its proper signification, — let us ask. What antecedent name does what represent? It must be evident, even to an ordinary student, that what represents no antecedent name, either expressed or understood. But since there is no antecedent (1.) which it could represent, it does not bind (2.) an antecedent to itself. What, then, if it is a relative at all, is such by virtue of some other principle than the two couched by the grammarians. To estaJblish a principle of classification in science is to find at least one point — a. In which a number of objects are alike; and — b. In which those objects differ from all others. And the class name of these similar objects has reference to that point only, and leaves all other circumstances out of view. Now the principle in which one relative is like any other relative, and unlike a personal or an interrogative pronoun, consists in this, that the relative establishes a relation beticeen verbs, not that it represents and binds to itself an antecedent noun or pronoun. We now see why, in either of the sentences quoted, what should be reckoned a relative: because it establishes a relation between verbs. With a single verb, the sense in either is incomplete. Thus, both "I found what" and "I tell thee what" are unfinished without, and therefore demand, a second verb; whereas an interrogative demands but a single verb; as. What was wanted? It would seem that the dispute about the pronoun in indirect questions in such sentences as "I know what prevented his coming," should be forever settled by this consideration; since the only objection here to calling what a relative, is that it does not represent (1.) an antecedent noun or pronoun which it could bind (2.) to itself, which objection, as we have just seen, is no objection at all. And it does do what every relative must do, and what no interrogative can do, namely, establish a relation between two verbs. It is incidentally true, indeed, that every relative must have, or must not have, reference to an antecedent; just as it is true that every quadruped must be horned or hornless. And it would l^e just as scientific, and not less laughable, to declare that what is a relative (properly indicating a relation between verbs) because it represents an antecedent noun, as to declare that a horse is a quadruped because it represents a two-eared animal! The horse is a quadruped, not because it has two ears, but because it has four feet. The relative is a relative, not because it represents and is bound to an ante- 458 THE SYSTEM METHOD. cedent noun or pronoun, but because it connects (establisbes a relation be- tween) two verbs. Prog. XV: /. Class of Pronoun. 2. Person. 3. Number. 4. Gender. 5. Case. a. Personal. a. First. a. Singular, a. Masculine, a. Nominative. b. Relative. b. Second, b. Plural. b. Feminine. b. Possessive. c. Interrogative. c. Third. c. Neuter. c. Objective. d. Common. PABTICULABS : It 1 was good to be there. She sweeps it ^ through the streets. Come, and trip it 3 as you go. It * is important what ^ he ^ is. It "^ is they.* It ^ is John and Mary. It 1" is enough, if you " do your ^^ best. I ^^ said he i* was a fool, and am sorry for it.^^ It ^8 is the foible of American youth, pretension. He ^"^ declared it i* to be wrong, the thing which ^^ I ''^'^ did. We,^^ Victoria, queen of England, do proclaim. "When we^^ set our "3 hand to this great work, we^* purposed with ourself ^^ never to wed. — Ten- nyson. Buy what "^ you need. Do what ^^ is right. Buy the thing which ^^ you need. Do the thing which 29 is right. Those books are hers.^o That book is hers,^! not yours. 32 This is the man that ^^ I spoke of. AU the world was ours ^^ once more. I got what 23 1 wanted. I ^^ teU you what,^'' that 's a fine horse. I know who ^^ he ^9 is. I know what *" prevented his coming. There are who ^ put their ^ trust in riches. I know who*3 was the first president. Take what^ you^^ want. Thou^^ who*^ stealest fire, strengthen me, ^* enlighten me. Wake, all ye*^ that^o sleep. What^i^ touches us ourself 32 shall be last served. — Shakspeare. Whatever 33 is, is right. Who- ever 3* did it,35 ought to be punished. I knew not what 36 to do. What 37 he 38 says, is true. We see what 39 he is doing. Whoever ^o gives, receives. Whoever ^^ trusts him will be deceived. I knew not what ^2 it was. They have furnished what ^^ will deUght us. He 's not what ''* I took him ^3 to be. When it ^6 struck ten, it ^7 began to rain. TeU what ^8 it is to be wise. Whatever ''^ he does, you mind what ^^ I teU you. We could not conceive whaf^^ had made him what ''2 we found him. Whatever '^^ thy '* hand findeth to do, do it with thy might. Whatever ^3 i do, I can- not please you. I asked what ™ he knew about it. We all knew what '''^ the difficulty was. I have read such books as ''^ please me. He has as many books as ^^ can be read with profit. There is as great a variety as 89 can be found in any other country. The business is such as 8i concerns me. Avoid such as 82 are vicious. He collected as much as 88 is due. As many as84 choose, may remain. We made as many bricks as 83 the material rendered possible. The positions were such as 86 appear incontro- vertible. Reconciliation was offered on conditions as moderate as 87 were consistent with a permanent union. The conditions are as 88 foUow. His words are as 89 foUow. The conditions of the agreement were as ''" follows. The wicked shall perish, as 9i aU the world knows. He then read the conditions as ^2 foUow. The point in dispute was, as 93 all the world knows, to which the said spectacles ought to belong. Man, proud man, Drest in a little brief authority. Plays such fantastic tricks before high Heaven As ^* makes the angels weep. — Shakspeare. Whence and what ^3 art thou,^^ execrable shape? She soured to what 97 she ^8 ig. What 99 can you do with an eloquent man? Man is such asi"** his afEection and THE WORDS CLASSIFIED. 459 thought are. Such as ^"^ have deprived themselves of charity, wander and flee. The modern world is theirs.i*'^ Whati''^ is there that^"* he cannot do? You know not what^os you (Jo. — SJiakspeare. It^^^ is now struck twelve. — Id. What^"^ are the angles of a plane triangle whose ^'^ sides are 432, 543, and 654? — Loomis''s Trigo- nometnj. The most advanced nations are always those who ^^^ navigate the most. — Emerson. Nor is any language complete whose ^i" verbs have not tenses. — Harris'' s Hermes. Those adjectives whose ^^^ signification does not admit of different degrees, cannot be compared. Is it true, as^^^ is said, that mood refers to the manner of the assertion ? DEMONSTRATION XVI. All sciences are systems of what is a pa?'i! of an including tchole. Or- thography is the system of the letter (etymon-jjart); etymology, of the etymon (word-part); grammar, of the word (sentence-part). In the same way, civil architecture is the science, or system, of the building-part. Grammar, the science of the sentence-part, or word, is much like architecture: the sentence is to be thought of as a building, — a building formed of words (sentence- parts). The sentence is a wonderful building of complex parts, some essen- tial, some merely accessory to the essential parts, others, still, being inde- pendent of the body of the sentence. The essential parts of the sentence are usually a name and a verb. These correspond to the main timbers in the frame-work of a building. The accessory parts are the adjective and the adverb, corresponding in the sentence to those parts that are attacJie he was predisposed to melancholy. The 9* EngUsh -^ bate the^^ French®* as frivolous. "^ What®* a®^ wealth has England earned. Other®* refuge have I none.®® AUi**® was quiet. i"* Knowledge, so my daughter held, was an*®^ in aU.i®^ He was not worthy i®* suchi®^ a 106 ioye_ The*®'' slow,*®* sweet*®® hours bring us all**® things good.*** My good*'*^ lords both,**3 I shall do so. O, what*** portents are these.**^ On Wednesday next,*** our Council we wiU hold at Windsor. None **'' is so deaf *** as he that wiU not hear. The wicked **® flee when no *~® man pursueth, but the *2* righteous i^^ are bold *2* as a lion. The*2* upright *25 shall dwell in the*^® land, and the*^'' perfect *2* shah remain in it. He came unto his own,*^® and his own *^® received him not. Ear through the *** memory shines a**^ happy **^ day, Cloudless *** of care, down-shod *2^ to every *3* sense, And simply perfect **'' from its own *** resource. — Lowell. *See Demonstration XXIX., c, note. THE WORDS CLASSIFIED. 463 DEMONSTRATION XVII. FACT MATTER : a. Grammatical Comparison. — A- variation in the adjective by which a measuring principle is established for objects and attributes, as follows: — 1. Between two objects: James is taller than John; London is larger than New York. 2. Between one object and all others under view: London is the largest of cities; Moses was the meekest of men; The rose is the fairest of flowers. 3. Between attributes: She is more nice than wise; He is stronger than wise. 4. Between measures of the same attribute: Thou canst the wisest Wiser make; He is happier at home than he is abroad; I grew greater. There are three forms of comparable adjectives, as follows: — b. The Positive Form. — That in which the measuring principle is applied to a single object or attribute; as, He is a good man; The 7iouse is large. C. The Comparative Form. — That in which the measuring principle is applied to two objects only or two attributes only; as, Thou art a goodly youth and worth a goodlier boo7i; London is larger than New York. Jffofe.—TMs form frequently governs a noun in the objective case. (See Demonstration XX., d, note 2.) d. The Superlative Form. — That in which the measuring principle is applied to the whole number of objects or attributes under the mental view; as. Woman is the fairest of the sexes ; The sun is the brightest of the two lights made to rule the earth. e. Regular Comparison. — Adjectives are compared regularly by add- ing to the positive, er for the comparative, and est for the superlative. Ex- amples are, — ositive. Compar. Super. Positive. Compar. Super. Bright, brighter. brightest. Nice, nicer, nicest. Crooked, crookeder. crookedest Mild, milder. mildest. Happy, happier, happiest. Tall, taller. tallest. Worthy, worthier, worthiest. Wise, wiser, wisest. f. Irregular Comparison. — Adjectives are compared irregularly by different or modified words. Examples are, — Positive. Comparative. Superlative. Aft, after. aftermost. Bad, ill, evil^ worse. worst. Far, farther, farthest, farthermost. further. furthest, furthermost. Good, better. best. 464 THE SYSTEM METHOD. Fore, former, End, Eastern, Southern, In, inner, Under, hither, hinder. Hind, Little, less. Much, naany. more. Late, later, latter, Low, lower, Near, nigh, nearer. TlPtllPT Old, older, elder, Out, outer, utter, Up, upper, first, foremost, endermost. easternmost, southernmost, inmost, innermost, undermost, hithermost. hindmost, hindermost. least, most. latest, last, lowest, lowermost, nearest, next, nethermost, oldest, eldest. outermost, uttermost, utmost, uppermost, upmost. g. Logical Comparison. — Logical comparisons are made both through the three grammatical forms and by prefixing more, most, or less, least; thus, beautiful, more beautiful, most beautiful; beautiful, less beautiful, least beau- tiful. In regard to logical superlatives, it should be observed that in Eng- lish no word is absolutely superlative except what has its grammatical form in st. The best authors habitually write " most perfect," "most supreme," "most permanent," "most right," "roundest," "straightest," "sincerest," "truest," "chiefest," "extremest," "supremest," etc. h. Most Perfkct, Extremest. — It has been a dispute whether Addi- son's sentence, " Our sight is the most perfect ... of all our senses," and its sort are proper. Perfect is from ]}er and facio {" thorough " and " to make "), and means simply "thoroughly made," or "thoroughly finished;" hence, most perfect cannot be less proper than "most thoroughly finished," which phrase is certainly good thought sensibly conceived and well expressed. And so extreme=" at the outer," not necessarily the "utmost," edge; hence the classic "extremest : " " The extremest verge of the swift brook." — ShaJcs- peare; "On the sea's extremest border stood." — Addison; "To extremest thrill of agony." — Pollok. i. One and Other Sometimes Nouns. — It has been shown (See Dem- onstration II., Obs. k) that whatever takes the plural sign is a noun. The added s alone is adequate to identify one and other as nouns in the following examples. But it will plainly appear to him who attends to these examples, that not only do one and other exhibit the plural by adding the s, but they exhibit likewise the possessive by taking the ('). And if we add that they have the constructions and the properties of nouns, it cannot be questioned that they are nouns; for thus to decide is only to aver that what has the uses and the properties and the peculiarities of a noun, must be a noun. THE WORDS CLASSIFIED. 465 Hence, only by avouching the laughable doctrine that that is not a noun which has the varied uses, the several properties, and the distinguishing peculiarities of a noun, could we deny that one and other, in the following examples, are nouns: — 1. Nominative singular, subject of verb: One has to comply with law; What an other may say does not concern me. 2. Nom. plu., subject of verb: The old birds flew away, but the young ones remained in the nest; I know not what others say. 3. Obj. sing., object of verb: Labor makes one merry; The one aided the other. 4. Obj. plu., object of verb: He admires virtuous statesmen, but despises corrupt ones ; No man can command others, until he has himself learned to obey. 5. Obj. sing., object of preposition: Time gets the better of one ; I am the sister of one Claudio; You should never covet the goods of an other. 6. Obj. plu., object of preposition: They came with the little ones; Re- spect the rights of others. 7. Poss. sing., accessory to a noun: Evil hides behind one's back; The in- stinct to protect one's self comes by nature; Teach me to feel an other's WOe. 8. Poss. plu., accessory to a noun: One may profit by otliers' experience; "Black-man" is the little ones' game. 9. Poss. plu., accessory to a participle: The young ones' warbling was sweet; I care not for others' scoffing. 10. Object of preposition, and base of appositive: The witness referred to others,— James Little and Edward Finch; I am the sister of one Claudio. 11. Appositive to noun or pronoun: It was Cain, the one that killed his brother; The oflicers caught little and Finch, the others. 13. One And Othee, Declined: — Singular. Plural. Nominative: One, other. Ones, others. Possessive: One's, other's. Ones', others'. Objective: One, other. . Ones, others. 13. Noble Butler says, " Among personal pronouns may be placed one and other." And Goold Brown thinks, very feebly, that others is a noun and "ought to be parsed as such, . . . unless it be called a pronoun " ! But the pronoun theory cannot be more satisfactory than Mr. Brown's con- founding language. The essential difference between a noun and a pronoun lies in their syntheses of properties, and nowhere else. In the noun- synthesis, we may observe two things always, — (1.) that the regular plural is formed in s or es; (3.) That the possessive is formed, the singular in 's, the ]3lural in the ('); while in the synthesis of the pronoun, both the possessive and the plural lie covert and undistinguishable in the root of the name-word. Thus, in the pronouns my, his, our, their, we can separate the possessive and plural signs neither from each other nor from the naming word. But in one's, ones', other's, others', boy's, boys', trunk's, trunks', we can at once distin 30 40G THE SYSTEM METHOD. guish 's as the possessive singular, (') as the possessive plural, and s as the sign of the plurul number ; hence the doctrine that others and ones are nouns. (See the definitions of noun and pronoun, Demonstrations I. and XIV.) j. One and Otiiek as Adjectives. — One is an adjective save when used as above, or vphen used abstractly to denote number. Examples of one are, — 1. As an adjective: Yfe, being many, are one head and one body; The dream of Pharaoh is one; One i)ound of energy is worth ten pounds of intel- lect; One time one is one; Either failed to make the kingdom one; There is none other God but one. 2. As a noun: One and one are two; Two times one are two; One time one is one. other is an adjective when not having the plural s, and referred to an object somewhere named in the sentence; as, Other refuge have I none; The king had one fair daughter and none Other cJhild. k. Ones, OxnEiis, Nominatives, Plurals, Hundreds, Tenths, Doz- ens, Twos, Superiors, Betters, etc. — It is the essential character of the plural number to indicate some sort of concrete repetition of object, action, part or particle. And the formation of the plural is by the addition of s or es to some name in vogue at the present, or at some past time. Hence that object, action, part or particle, whose plural is formed, must itself neces- sarily have a name — sometime properly or im])roperly used — before a plural could be formed. Thus, trees, boxes, oats, scissors, have tree, box, oat, scissor, of which the last two are not now properly used. Then it is that the fact of the existence of a plural in s or cs is indubitable proof of the existence of the necessary name, or noun; and we at once decide that what takes the plural sign, therein becomes a noun; as witnessclh ones, others, and the other nouns at the head of this Observation, and their like, of which the following are examples; Be courteous to your betters; Their betters would hardly be found; Some nouns have two plurals of different signiiications; In compounds, the sign of the plural is added to the important part; What is meant by two thirds of a unit? How many twelfths in three fourths? The possessives are formed by adding 's to the nominatives; One should defer to his superiors. 1. Each, Else, All. — We are now ready to observe that each, else, and all never add the s or es of the noun plural, but that they are always adjec- tives. In " They loved each other," each is a quantitative adjective, accessory to person, which is to be supplied to complete a sentence whole. Other is a noun, object of loved, which is also to be supplied. The complete sentence would be, "They loved, each ^)crso72. ZowcfZ the other." The sentence, "Each loved the other," is in apposition with the sentence, "They loved," to which it is accessory as an adjective element of the third class. Else is always a demonstrative adjective postpositive; as. Who else is coming? What else shall I give? Do you expect anything else? Do you THE WORDS CLASSIFIED. 467 expect anybody else? In each case, else is a demonstrative adjective in the appositive constructipn (see Demonstration XXIX.), limiting the interroga- tive ])ronouns who and tpluit, and the nouns anyildng and anybody. In such exj)ressions as, "It was nobody else's business," "The \)06k. was nobody else's," we should parse else, not else's ; since the possessive sign ('s) is no part of the adjective, but is merely placed to it, only because the adjective stands nearest the noun limited by the possessive nobody-'s. (Cf. Dem- onstration v., Obs. c.) All is never a noun. (Sec above.) In "She robbed me of my hopes, my heart, my all," iill is a numeral adjective, limiting some such noun as concerns or possessions understood, — " She robbed me of my hopes, my heart, my [concerns] all." And so in " God shall be all in all," — "God shall be all [concern] in all [concerns]," which renders the parsing easy, and is neces- sary to show the grammatical construction of all. Some does not take the plural sign, but is always an adjective prepos- itive; as, Some blushed, and others dazed. m. What a, Good a, Many a, etc. — Not the properties of the syllable, nor the properties of tlie sentence, but the j^roperties and logical value of the word in the sentence, constitute the basis of the grammatical distinction of "parts of speech." Consequently, unless the properties and logical value of the one arc identical with those of the other, two words should be treated sejiarately; for neither the plea of convenience and dispatch, nor the blurt- ing ignorance of a grammaticaster, can cope with the desirability of con- sistency. In " So good a man is not among us," " He was not worthy such a love," "What a })iece of work is man," each word preceding a, is an adjec- tive, descriptive, while in each sentence, a is an adjective, numeral. Hence a must be treated separately from the preceding adjective, with which it is identice,l in neither properties nor logical value. And so many und a in "Many a time he came." (See Demonstration XXIII., p.) Prog. XYll: 1- Adjective. 2. Class. 3. Accessory to What? a. Quantitative. b. Qualitative. c. Demonstrative. PARTICULABS : A "• bright ^ day followed a ^ gloomy * night. I prefer a New-England ^ winter to an Australian " summer. Wc took only a bird's-eye '' view. Save his good ^ broad- sworii, ho weapon had none.'' Eachi" to each^^ is dearest ^^ brother. Thou art lonelier ^^ than whate'cr ^* dread haunts the knolls of Ida. Thou kuowest what ^^ a ^^ man I am. Was there nothing else?!''' What^* stufE is this? She is more uice^' than wise.^'' London is larger ^^ than New York. Thou canst the^^ wisest '^^ wiser '^^ make. I seemed to grow greater. ^^ We know him to be true.^* We regarded it as possible.^'' Thou art a goodly ^^ youth, and worth a goodlier ^-^ boon. Our sight is the^** most perfect ^^ of our senses. She robbed me of my hopes, my heart, my all.''^ We, being many,^^ are one ^* head and one "■'' body. They loved each ^^ other. They 468 THE SYSTEM METHOD. loved one an ^7 other. Who else ^8 is coming? Do you expect anybody else? ^9 The book was nobody else's*" (see Dem. V., Obs. c). God shall be all" in aU.*^ Somei^ blushed, and others dazed. So good ** a ^^ man is not among us. What *« a *7 piece of work is man. Many ^^ a *» time he came. Much •''» time is wasted by those si six ^2 boys. Five ^^ hundred ''* men were slain. Ten ^^ thousand •''^ men, women, and chil- dren perished. We purchased two " dozen ^8 eggs (see Dem. XXVIII., h). The life of Howard was that ^9 of a philanthropist. A •*" great ^i many ^3 of his followers de- serted him. I ought to go upright ^"^ and vltal.<5* That ^^ apple tastes bitter. "^ This ^^ board feels smooth. ^^ He grew very cautious."' She turned pale.''" Time hangs heavy '"^ in the hall. I have bread enough '''^ for us all.^^ The child has been made happy.''* Virtue consists in being good,^^ not in appearing good-''^ His end was that ■''' of a good man. Pride alone ''* urges him on. It is his only ''9 son. The En- glish 80 regard the French 8i as frivolous.^^ These 83 ten 8* soldiers were among the first 85 to cuter the city. The twelve 86 last 87 are to my purpose. — AdcUmn. The 88 four 89 greatest ^° names in English '■■ poetry are almost the four '^ first '^ -^'e come to. JIazUtt. The two ^* last "^ may enter Carleton or any"" other "^ house, and the two"* first "" are limited to the opera. — Byron. Procure a manuscript of the ten i"" or twenty 1"! first "2 lines. — Johnson. The whale is the largest ^"8 of animals. Both"* the^"^ apples are ripe. AU^"" the^"'' books have been studied. This happened some i"8 fifty i"" years ago. The veriest "" child could not have been guilty i" of a worse "2 or a sillier "8 blunder. John was tall"* — taUer"^ than either"" of his brothers. He came the last,"' neglected and apart."* it is a choice out of half a"" million 12" books alli"^ alike.^^s We do it wrong, being so majestical,i23 To offer it the show of violence. — Shaksjjeare. With many ^^i a ^-^ weary ^^s step, and many ^^^ a iss groan, Up the high ^^s hiU he heaves a huge '80 round ^^^ stone. As Caesar loved me, I weep for him; as he was fortunate,'82 1 rejoice at it; as he was valiant,i''8 j honor him; but as he was ambitious, '8* I slew him. — Shaksjjeare. HaK- way down a by-street of one of our New-England towns, stands a rusty, wooden house, with seven acutely-peaked i^^ gables, facing toward various points of the compass, and a huge, clustered chimney in the midst. — Hawthorne. To be great ^86 is to be good,i8T To be truly happy ^88 is to be truly true.i89 1 THE WORDS CLASSIFIED. 469 DEMONSTRATION XVIII. FACT MATTER: a. Adverb. — A word accessory to a verb, an adjective, or another adverb. b. Classes. — There are five important circumstances connected with every event; namely, the time, the place, the degree, the manner, and the reason. Of these five particulars, what is requisite to the thought and not fully set forth through the properties of the verb, is portrayed by the adverb. There are, corresponding to these particulars, five classes of adverbs: — C. Temporal Adverb. — One that pertains to time; it answers to the idea of w/iew, how soon, hoio often, ox how long; as, immediately, already, here- after, first, eternally, etc. d. Local Adverb. — One that pertains to location, or place; it answers to the idea of where, whereabout, lohence, ov toMther ; as, around, everywhere, abroad, forward, off, remotely, secondly, etc. e. Intensive Adverb. — One that pertains to degree; it answers to the idea of hoio much, how little, more, or less ; as, entirely, chiefly, competently, least, merely, precisely, etc. f. Modal Adverb. — One that pertains to mode, or manner; it answers to the idea of how, with what condition ; as, foolishly, certainly, perhaps, yes, no, hesitatingly, etc. g. Causal Adverb. — One that pertains to cause or reason; it answers to the idea of why ; as, why, wherefore, therefore, reasonably, etc. OBSER VA TIONS : a. This distribution of adverbs into five classes has no reference to the construction of adverbs. In Syntax it will be determined that of the con- structions of adverbs there are two, — modifying and conjunctive. b. Yesterday, to-day, to-morroio, are always nouns, never adverbs; for there is not a great author among the grammarians who can deny that there is such a thing as to-day ; and of that thing, if to-day is not the name, what is? But every name is a noun. Observe that cawft'o-ws;?/ is not a noun because there is no such a thing as a cautiously. " To-day is as yesterday, and to-morrow will be as to-day." C. Let it be noted that the mere fact that a word may, occasionally, in addition to its own regular use, assume that of some other part of speech, does not constitute it such other part of speech — not more than the mere fact that a man's coat may occasionally be used as a pillow, constitutes such coat a pillow. Thus in the following sentences, 07ice and how, though used like nouns, — in being limited by the adjectives this and some, — are yet adverbs, not nouns: "Please sing for us just this once;" " Some hoto, I can- 4Y0 THE SYSTEM ME TITO A not understand it." Once and how are not here to lose their name as ad- verbs, and be called nouns; because in assuming the use of nouns they have not totally lost their own adverbial value, or construction, — since once still attaches to smg and 7iow to understand. d. If, however, in assuming the use of a noun, an adverb totally loses its own adverbial peculiarities, so that it no longer attaches to verb, adjec- tive, or other adverb, such word has really become a noun. Examples are the following :- By far : Better by far than two hearts estranged. Is a low grave starred with daisies. For aye: Ah! 'twere a lot too blest, To roam and dream for aye. — Bryant. From far: For divers of them came from/«?'. From within : It proceeds from witldn outward. From above : He must be born from above. From beloto : A voice came from beloic. Till now : Till now he knew not defeat. Till then : Till t7ien who knew the force of those dire ms? e. Observation c (above) is equally ap|)licable to those participles some- times employed with the value of adverbs. The participle remains a parti- ciple notwithstanding that it has the construction of an adverb. Examples: staring mad, sopping wet, scorching hot ; My clothes are dripping wet; We were all freezing cold; The water was scalding hot; It is passing strange. A man he was to all the country dear. And i)assi7ig rich with forty pounds a year. — Goldsmith. He came tumbling dowu the stairs. (See Demonstration XXIII. , k, o.) f. From several phrases composed of a preposition, an adjective, and a noun, the noun has been dropped. This has been done partly for dispatch, but chiefly to relieve the phrase of a clumsiness that must needs attach to the expression in full. From the continued disuse of the noun, the prep- osition and adjective of the part remaining, have, in some cases, acquired a structural twist, or gnarl, rendering it difficult now to supply the noun. In every case, however, some noun must be supplied to complete the gram- matical construction. Examples are, — At last, at the last=:at the last juncture, stage, resort; as. At last, the pas- sengers forsook the burning vessel. Almost, at the most=at the most avail, result, jjart, import; as. The enemy can only take our lives at most. At least, at the least^at the least estimate, condition, result; as. He who tempts, though vain, at least asperses. At best, at the best=at the best state, result, condition ; as. Life is, at best, very short. THE WORDS CLASSIFIED 471 At first, at the flrst=at the first time consideration; as, At first, his courage failed him. At present, at the present=at the present time, condition; as, Illinois is, at present, the greatest corn-producing State. ^^ a;^=at all events, estimation; as, He has no ambition at all. Above a?Z=above all things, considerations; as, Aboi)e all, teach your chil- dren to be useful. Before long=^hefore a long season, time; as, It will rain before long. Ere long=eve long time; as, Ere long, all upon the earth must leave it. For good=tor good time; as. The good woman never died after this, till she came to die for good and all. From, first, from, the first=from the first time, trial, consideration; From the first his conduct has been suspicious. In short=in short phrase, fashion; as. In sJiort, he is a saint. In general, in particular^in general, in particular, way, matters, sense; as. It is a very cold humanity that loves the world in general, and nobody in particular. In private, in publicum private, in public, manner, things, sense; as. He prays in private, or in secret; He spolse in public. In full=m full state, estimate, expression; as, These lectures have been published in full. Of old=ot old times, memory; as, A land there is, Hesperia named of old. Of late=ot late times, memory; as, The practice is of late uncommon. On high=on high place, estate; as. He set up 07i Mgli those that be low. On a sudden^on a sudden turn, throe; as. He seemed to recollect on a sud- den that he had made a promise to his father. g. The common practice of calling almost every irregular combination of words an "adverbial phras.e " to be "parsed as single adverbs " is but the expedient of the grammaticaster to evade the difiiculty of explanation, and is worthy to be condemned by all good teachers, At random, at length, in conclusion, of course, by no means, for the most part, etc., are all prepositional phrases, in which preposition, adjective, and noun each has its own pecul- iar oifice and individual properties: They were taken at random; they were taken at sight; they were taken at sea. If at random is to be "parsed as a single adverb," we must parse at sight and at sea in the same way! It is the iDord — not the letter, nor the etymon, nor the sentence — that constitutes the science-system, or tree-trunk, unit of grammar; it is the word, therefore, that we distribute into "j^arts of speech." Hence, every single word is of itself some part of speech. This pernicious and shallow practice of treating two or three words as one must be broken up, before it will be possible for the stu- dent to understand the science system of grammar, without which his study of grammar must be almost useless. (See " One Verb Consists of a Single Word" and "Two Words as One," pages 387 and 393. See, also. Demon- stration XXIII.) 472 THE SYSTEM METHOD. h. Hand in Hand, Side by Side, Word by Word, etc. — In explain- ing such expressions as, "They walk 7iand in hand," " They sit side by side," let the student give no heed to the wisdom of that author or copyist who would call these single, separable, and uncompouuded words a "single ad- verb" or "an inseparable phrase." Let him, likewise, consider whether it will not be indiscreet to supply an assumed and forced ellipsis; seeing that it is by arrogating to themselves that liberty that writers have cursed the science of grammar with interminable disputes. The naked fact will then appear, I think, to be this, that the Introductory words Jiand and side are simply nominatives by accompanying circumstance. (See Demonstration XXVI., a 4.). The parsing of the remaining parts of these phrases is sim- ple. So, also, "The earth is cooling, day by day;" "The mountains rose, MgM above Mglit" etc. i. All adverbial phrases should be fully analyzed. In to and fro there are two adverbs and the co-ordinate conjunction and; as. The lunatic paces to and fro continually. In by and by are the same parts; as. When persecu- tion ariseth, by and by he is offended. "By the bye, there is a little matter that we must consider at once :" by the bye^"'bj the way;" and bye is a noun. "I wind about, and in and out; " in and out are here local adverbs accessory to iciiid. " The stranger told me an out-and-out falsehood : " out- and-out is here an adjective, as are all words joined with a hyphen, and ac- cessory to a noun. Analyze ever and anon, long ago, all along, ever so, over and over, again and again, many a time and oft, over again, now and then, far and near, etc. j. Adverbs sometimes govern nouns ; as. He came an hour sooner; We will start an hour hence; A hundred years ago, railroads were unknown. In the first example, sooner is an adveib modifying came; and Jiour is in the objective case, governed by sooner. That ago is an adverb mod- ifying unknown, will appear from the following sentences: He died long ago-^ He died a long ti7ne ago. Team's and time are governed by the adverb ago, in the same way that miles is governed by the adjective long, in " The lake is ten miles long." (See Demonstration XXVI., b 6 c.) k. Words used merely as expedients of transition from one sentence to another are to be regarded as conjunctions, rather than adverbs; as, N'ow, Barabbas was a robber; W7iy, you told me so yourself; If I cannot perform my promise, why, I will regret having made it; Well, it is a fine morning. 1, The responsives, yes, no, certainly, etc., are parsed as modal adverbs accessory to the verbs to which they respond. Thus, in "Is he a Quaker? — iVb;" "Have you read the book? — Certainly," no and certainly are parsed as adverbs accessory to, or modifying, is and have respectively. In such ex- pressions as, " The more quietly it is done, the better," "Is she tJie less wor- thy because she is a woman?" the is an intensive adverb. m. Comparison op Adverbs. — Comparison does not belong to adverbs in general; hence it need not be mentioned in parsing. THE WORDS CLASSIFIED. 4Y3 1. A few adverbs are compared regularly, after the manner of adjectives; as, soon, sooner, soonest; fast, faster, fastest; long, longer, longest; often, oftener, oftenest. 2. A few are compared irregularis^ ; as, well, better, best; badly or ill, worse, worst; little, less, least; much, more, most; forth, further, furthest. 3. Most adverbs ending in ly admit the comparative adverbs more, most, less, least, before them. This is a virtual comparison of the latter adverb, but the grammatical comparison, or change of form, belongs only to the word prefixed. n. Adverbs Modify Prepositions. — The prepositions were originally adverbs, and are so-called, not because they now precede their objects, but because tJien they formed the first part of the verb. Many of them still re- tain an adverbial bond, or value. And it is by means of this bond that ad- verbs are still sometimes attached to them. In " He threw the stone al- most over the river," almost is an adverb modifying tlie preposition over. Other examples: The stone struck ^ws.!! beneath the window; This mode of pro- nunciation runs considerably beyond ordinary discourse. — Goold Brown; He sees let down from the ceiling, exactly over his head, a sword; We are al- most at our journey's end; The wrecli lay right aiMvart the stream; The price of a virtuous woman is far above rubies. Prog. XYIII: /• Adverb. 2. Class. 3. Accessor/ io What? a. Temporal. b. Local. c. Intensive. d. Modal. c. Causal. FARTICULAMS : How^ often 2 have you been there? ^ He studies sometimes* very^well. ^ They live very "^ happily. ^ He speaks fluently ^ and coherently. ^^ Come often, ^^ and stay long. ^^ It is full enough, i^ She writes correctly. ^^ The hook is wherein you laid it. He studies whenis jq^ piay. Never" can she be more^* happy. Often 19 have I seen them walking together, ^o I have forgotten my part, and I am out. 2^- He grew very ^^ thin. The actor scowled ominously. ^^ He leaped far 2* beyond the mark. It is very^s natural for him to move gracefully. ^^ The rain was falling fast. 27 The wreck lay right ^8 athwart the stream. The price of a virtuous woman is far 29 above rubies. The feast was over,30 i^ Branksome tower. They cannot go while ^1 1 am away. ^^ Never ^3 two loved as ^^ they do. Out 35 steps, with cautious foot and slow, And quick, keen glances to^e and fro,^^ The outlaw. Do not 38 use adverbs needlessly. 39 He seldom *" went home sober, if ever. *i Begin it over ^ again. *3 It is pretty ^ nearly ^^ finished. He expects rapidly *6 to recruit. I wiU not go out " but once. Do not speak up *» promptly. *9 This is a remarkably ^o cold winter. I came as^^ I was directed. Then^^ -^j^q knew the force of those dire arms? The lunatic paces to''3 and fi-o^i continuaUy. ^5 When persecution ariseth, by ^6 and by" he is offended. I wind about, ^-8 and in 59 and out. «" He 4H THE SYSTEM METHOD. came an hour sooner. ®i We will start an hour hence. "^ A hundred years ago,*^ raikoads were unknown. On my way hither, I saw her come forth. ^^ There ^^ is DO night there.'''' — Bible, The ''^ faster ^^ the ball falls to the sun, the force to fly oif^isbyso much augmented. — Emerson. The''" louder'''^ the thief talked of his honor, the '^ faster '^ we counted our spoons. — Id. But even then '* the morning cock crew loud. '^ Titinius's face is upward. ''^ The storm is up, '^' and all is on the hazard. Wherever '^^ snow falls, there "^^ is usually ^^ civil liberty. Luther died where ^^ he was born. Error must always *^ yield, however ^^ ingeniously ** it may be defended. He put her by. ^^ He died not long before. ^^ She was laughed at. *'' He gave up ^^ that plan. He was made fun of. ^^ The season is over.^" The sun is up. ^^ Why ^^ do you speak ? DEMONSTRATION XIX. FACT MATTEB: a. Objective. — Any word or part used like a noun in the objective case; as, We met him; We could not avoid going; I love to sing; They all fell to praying; They start to-morrow. b. Preposition. — A connective governing an objective. The parts connected by a preposition are always unlike in from one to three respects, as follows: — 1. Unlike properties: He is a man of distinction; He came to stay; Man is born to die; We all fell to eating. 2. Unlike j^arts of speech: He lives /(*r glory; He is honest in purpose; It is ripe enough to use; He acted agreeably to instruction; Time to come is called future; He differs /wra you. 3. Unlike constructions: I should laugh to cry (observe that while laugh is verbal complement to should, cry is substantive in use, being the object of the preposition to); It came from a citizen o/ Chicago. (Of Chicago is adjec- tival, in construction, to citizen, while from citizen, on the other hand, is ad- verbial to came.) C. Conjunction. — A connective — between words, phrases, or sentences — having no governing power; as, James and Henry remain; Over the fence and across the river is out of danger; You may go, hut I will remain. To the Teacher.— The only way thoroughly to understand the preposition is to study it side by side — contrast it — with the conjunction, its co-ordinate class. Both are connectives; but the preposition connects unlike, the co-ordinate conjunction like, parts. With the pos- sible exception of the principle of "Be and Same Meanings," this principle of difference be- tween the preposition and the conjunction, is the most effective of all means in the solution of grammatical difhcultics. (See Demonstration XXX., d 1, 3). d. The antecedent term, or base, may be a noun, pronoun, verb, adjective, or adverb; as. They are men of honor; It was he on the stage; He went to Rome; He is honest in that belief; He acted agreeably to instruction. THE WORDS CLASSIFIED. 475 6. The objective is always a noun or other part used as a noun. f. Whatevek may connect an objective to a verb or to a nominative, is a preposition. What cannot be used to connect either of such parts, cannot become a preposition in any circumstance. This principle is of the utmost importance in the solution of grammatical difficulties. By means of it, the student will see that — g. Like, worth, a, adown, instead, because, near, nigh, concerning, pending, and notwithstanding, which have so perplexed teachers, sometimes become prep- ositions; since they are used to connect the objective to a verb; as, Qu'il yourselves like men; It is wortJi while; The hat is worUi, a dollar. That girl, for whom your heart is sick, Is three times wortJh them all. — Termyson. He went a fishing; He fell a bleeding; It floats adown the air; Let the thistles grow instead of ■vj\\e&\.; The spirit is life because of righteousness; Because of these things cometh the wrath of God. Instead of mounting barbed steeds. He capers nimbly in a lady's chamber. They speak concerning virtue; Pending the suit, he left the country; I shall go, notwithstanding X\iQ rain; You have failed, notwithstanding the aid received; Drunkenness increases notwithstanding the law, and in spite of the oflBcers; The principles and beginning of a moral government over the world may be discerned notwithstanding and amidst all the confusion and disorder of it. — Butler's Analogy. h. But, Save, and Except. — By this same principle (see b and f above) it will be seen that these words may become prepositions, since they are used to connect the objective to a nominative; as, — Whence all [ ] but him had fled. They are all gone except him. All [ ] were gone save him who now kept guard. Who should be king save him who makes us free. i. But, except, and save are, however, usually conjunctions; as, There is none other but he. — Bible. Place me on Sunium's marble steep. Where nothing save the waves and I May hear our mutual murmurs sweep. — Byron Few can save he and I. — Id. There was nobody at home but I. — Walker. Who can it be but perjured Lycon ? He did nothing but laugh. You can not but conquer. There was no man but prophesied revenge. There is no beast so fierce but knows some touch of pity. 4,Y6 THE SYSTEM METHOD. A man cannot steal but his countenance accuses him. He would steal but for the law. I could not live but for my hope of eternal life. No one but he will undertake it. All the conspirators save only he, Did that they did in envy of great Csesar. — Shakspeare. Except a man be born anew, he cannot see the kingdom of God. But except ye repent, ye shall all likewise perish. He is very tyrannical except over the Catholics. He never fought except in his own defense. None knew thee but to love thee. I do entreat you, not a man depart, Save I alone, till Antony have spoke. — Shakspeare. That mortal dint. Save He who rules alone, none can resist. — Milton. J^ote.— Simple examples might have heen given to illustrate the use of these knotty ypords— but, except, and save— as conjunctions; but because the author desired to pass by and leave no difficulty unexplained, he has here chosen the extraordinarily difficult sentences above. Too commonly, authors of text-books, passing by the real difficulties, explain the simple things only; so that not until he has left the school-room, and undertaken the doings of life, does the student discover that he has really taken only a toy-like and primary course in grammar, and is helpless so far as those things are concerned in which help was most needed. i. "Used with That. — When placed before the conjunction that or be- fore a nominative, but, save, except, and notwithstanding are conjunctions; as, — I c'uld not believe but that I was in hell; But except ye repent, ye shall all likewise perish; Except a man be born anew, he cannot see the kingdom of God; There is no difference excej^t that some are heavier than others; I should go except that I fear the storm; Adverbs have no modifications except that a few are compared. — Goold Brown. k. Except a Pakticiple. — Used with a nominative, except is an apoc- opated perfect participle; as, — God and his Son except [=excepted], Naught valued he, nor shunned. — Milton. Richard except, those whom we fight against Had rather have us win than him they follow. — Shakspeare. Compare, " Thunderbolts ea;cepfe(Z, quite a god." — Coioper; and "Hiley ex cepted, these grammarians," etc. — G. Broion. 1. Is To Before the Infinitive a Preposition? — See Demonstration XXX., j. m. As is never a preposition. (See Demonstration XV., "As" a. n. Two prepositions are often written together to connect the object ive and the antecedent term; as, He came from among the IWethodists; The spirit is life because 0/ righteousness; Let the thistles grow instead 0/ wheat. TEE WORDS CLASSIFIED. 4:17 0. Plus and Minus. — Plus is a conjunction and minus, an adjective. (See Demonstration XX., 1.) p. Prepositions become adverbs; as. He put her by; He died not long before; He dwells above; The season is over; She was laughed at; He gave up that plan; The cakes were done liberal justice to; He was made fun of. Jus- lice and fun are in the objective case, governed by done and made; to and of are accessory to done and made; therefore they are adverbs. Prog. XIX: /. Preposition. 2. Objective. 3. Antecedent 4. D-issimi/arities. Term, or Base. a. Parts of Speech. b. Properties. c. Constructions. PARTICULARS : Men are born to i die. The progress is to ^ accuracy, to ^ sldll, to * truth, from ^ blind force. He lived to ^ purpose. A mutual relation leads us to'' respect each other. Did you vote for ^ or against the measure? The principle operates in ^ and through i" the system. Alexander was ambitious of " glory. I could not agree with i^ him in ^^ that opinion. He is angry with i* me. He has a dislike for i^ us. He is eager for '^'^ riches. I shaU stay at ^'^ home, for I have need of ^^ rest. He walked into i^ the gar- den. He is skillful in 20 drawing. He put the knife into ^^ his pocket. Your weeping sister is no wife of*^^ mine. We differ from^^ you. I heard the echo along ^^ the hills. We hve in "^ a viUage. I am obliged to -*' you. It was sold at a sale by ^"^ auction. He divided the estate among -^ four children. True happiness is an enemy to "3 pomp and noise. He lives in 3* Chicago, on ^i Wabash Avenue. Our clock strikes when there is a change from '''" hour to ^^ hour ; but no hammer in ^^ the horo- loge of Time peals through ^^ the universe when there is a change from ^^ era to ^^ era. — Carlyle. He died of ^* the measles. Joseph's pronunciation is very different from^^ yours. Break it aU to *o pieces. The lectures have been pubhshed in *i full. The practice is of *2 late uncommon. He spoke in ^^ pubUc. In ^ short, he is a saint. In *» fine, we, as a people, hve too fast. From*® the first, his conduct has been suspicious. Ere *'' long, all upon *8 the earth must leave it. Life is, at ^^ best, very short. At ^o last, the passengers forsook the burning vessel. Quit yourselves hke ^i men. That girl for whom your heart is sick, Is three times worth ^^ them a.]l.—Te7i7iyson. He feU a 53 bleeding. He went &^ fishing. It floats adown ^^ the air. The spirit is hfe because s" of ■''' righteousness. He came near s* me. Thy spirit is aU afraid to govern thee near ^9 him. They speak concerning ''^ virtue. I shall go, notwithstand- ing ''1 the rain. They were all gone except ^^ him. AH were gone save ®^ him who now kept guard. You have failed, notwithstanding,®* too, the aid received. Let the this- tles grow instead®'^ of ®® wheat. ''^"^^®:l^^'^ 478 THE SYSTEM METHOD. DEMONSTRATION XX. FACT MATTER : a. Conjunction. — A connective — between words, phrases, or sentences — having no governing power; as, He is diligent but unsuceessfol ; James and Mary go to school; He will succeed if he try; Over the fence and across the river is out of danger. (See Demonstration XIX., b, c; and Demonstration XXX., b, c, d.) b. Classes. — Of grammatical divisions of conjunctions there are two, — co-ordinates and subordinates. C. Co-ordinate Conjunction. — A connective between like parts — words, phrases, or sentences; as. He is both learned and wise; We find them in the woods and by the streams; He goes, but I remain. (See Obs. a below.) d. Subordinate Conjunction. — A connective between modifier and thing modified; as. We regard hiffl «s dishonest; The English hate the French people as frivolous; A noun is a name; as, man, eagle; It is used as a coajunction; Ye shall be «s gods; We presented otirselves «s petitioners ; He was regarded as dangerous; Beelzebub, tJian whom none higher sat, with grave aspect rose; These I brought her as pitying her usage; He is wiser than me; He will succeed ^/hebe diligent; He will maintain his integrity though he lose his estate; i/he drinks, he swears; I fear fe^: he should arrive; He will go provided his fare is paid; Tltat he will succeed is evident. Note 1.— The word as is to the grammarians, perhaps, the most unmanageable word in the English sentence. Examples of the gnarly as are, I chose him as reliable; The English hate the French [people] as frivolous ; Ye shall be as gods ; lie came as a business man. But let the true doctrine of the subordinate conjunction be once understood, and this as is easily explainable. Now, the subordinate conjunction connects modifier and modified, governed and governing. And because as so connects modifier and thing modified, or governed and thing governing, — because as connects reliable and him, frivolous and \_pco2)le'\, gods and be, man and came, — it is to be regarded in each case as a pure subordinate conjunction, not as a preposition; for this governs an objective, and cannot stand between modifier and thing modified, or governed and thing governing. Other examples of as as a subordinate conjunc- tion are, A noun is a name ; as, man, eagle ; It is used as a conjunction ; lie acted as governor ; I chose him as the most worthy [one]; We presented ourselves as petitioners; He was re. gardcd as dangerous; Brutes have no sense of actions as distinguished from events; These I brought her as pitying her usage; Arnold's success as teacher['s] was remarkable; As mayor of the city, I felt much aggrieved. Notei. — Until the student comprehends the true nature of the subordinate conjunction as a link between word modified or governing and part modifying or governed, he must dizzily stagger whenever any of that multitude of sentences containing as and than as subordinate conjunctions, shall be presented to him for explanation or justification. Among the difficul- ties tinsolvable, except through a true understanding of the subordinate conjunction, is the expression "than whom" in such sentences as "Solomon, than lohom there never was a wiser, declares wisdom more precious than rubies;" also the words "than rubies." See Demonstration XXX., m, where it is shown, — 1. That adjectives and adverbs may govern the objective case; 2. That, since the word than links together the objective governed and the adjective gov- erning that objective, it is therefore a subordinate conjunction, and whom the object of the comparative adjective inser. THE WORDS CLASSIFIED. 4^9 OBSEJEt VjL TIONS : a. Observe that co-ordinate conjunctions alwaj'S connect parts which are each referred to a common — the same — third part; as, Peter and John went to the city. Here, the conjunction and connects Peter and John, each of which is nominative to ioent. In "They go to see and be seen," and connects see and be — two infinitives, each objective to the preposition to. And so always. b. The whole class of conjunctions has been commonly divided into two, disjunctives and coijulatives, though this division has nothing at all to do with the science system of grammar. Defined, these classes are, — C. Disjunctive Conjunction. — One signifying that the connected parts are put in contrast, or some sort of opposition; as. He was diligent, but un- fortunate; Either James or Henry will stay. d. Copulative Conjunction. — One signifying that the connected parts are put without contrast, or opposition; as, Birds chirp and sing; I I believe tJiat he will be elected. e. Two conjunctions often come together. They are not, however, to be regarded as one conjunction, because they form one phrase, but as two; for no word is so small that it has not in itself all the essential magnitude of some "part of speech." Examjiles are, — As if: He blinks the question, as if he hated to answer. As tJiouffh : He is always nervous, as thougli he expected in an instant some dreadful calamity. And yet : We may be playful, and yet innocent. f. Finding- Both Parts. — It is sometimes difficult to find what are the parts joined by the conjunction. This happens, — 1. When the subordinate conjunction that links, or incorporates, a subor- dinate sentence in a complex sentence; as. That he will succeed is evident. Here, that is a part of two sentences, — "That he will succeed," and " That he will succeed is evident." Mark how peculiar is the joining of these sen- tences. They are joined, not between succeed and is, but by tJiat at their very beginnings, thus: — That he will succeed is evident. So far from being "really an introductory expletive" (as taught by many grammarians) is this thai, that it is the strongest possible sort of a conjunc- tion; since it ties part to whole — ties the part, "That he wiU succeed," to the whole of which it is a part. The relative ichoever makes the same solid kind of union between the part, "Whoever studies," and the complex whole, "Whoever studies will learn." To denote its peculiar and substan- tial binding power, this subordinate conjunction, that, may be called, as it often is, the substantive conjunction. 480 THE SYSTEM METHOD. 2. When the conjunction introduces a sentence torn away from, but con- nected in sense with, a preceding sentence, or thought; as, — ■ " [ ] And the eyes of the sleepers walked deadly and chill." " [ ] And Jehoshaphat, his son, reigned in his stead." g. Adverbs Become Conjunctions. — Many words ranked as adverbs in grammatical treatises, are employed in a few cases like conjunctions; and if, in those cases, the word has not a distinct adverbial value in addition to the office of a conjunction, it must be then parsed as a conjunction. Ex- amples are, — Also: The earth is also (adv.) clothed with verdure. See articles d and e ; also g and h. Besides: That man who does not know those things which are necessary for him to know is but an ignorant man, whatever he may know besides (adv.). I do not desire to go; besides (conj.), I have not now the time to go. Else: He could find it nowhere else (adv.). I have lost my money, else (conj.) I could help you. Indeed: The young man is indeed (adv.) culpable in that act, though, indeed (conj.), he conducted himself very well in other respects. However: We must, liowever (conj.), pay some deference to the opinions of the wise, liowever (adv.) much they are contrary to our own. Nay: I tell you nay (adv.); but except ye repent, ye shall all likewise per- ish. His prudent conduct may heal the difference; nay (conj.), may prevent any misunderstanding in the future. Nevertheless: It is a dreary road; but we must march on nevertheless (adv.). I am crucified with Christ; nevertheless (conj.) I live. Now: I have noio (adv.) shown the consistency of my principles. Noio (conj.), Barabbas was a robber. Otherwise: Thy father was a worthy prince. But Heaven thought otherwise (adv.). I have lost my money, otherwise (conj.) I could help you. So: It was so (adv.) cold that I nearly perished. Let cares like a wild deluge come, ******** So (conj.) I but safely reach my home. — Watts. Then: On these facts, then (conj.), I then (adv.) rested my argument. Too: I found, too (conj.), a theater at Alexandria, and another at Cairo; but he who would enjoy the representations must not be too (adv.) particu- lar. Well: TFe/; (conj.), proceed with the speech which you have so we^Z (adv.) begun. Why: Why (adv.) do you trust your character to be evolved by accident? Why (conj.), these are testimonies of what the unfriended may do. THE WORDS CLASSIFIED. 481 Consequently, lience, likewise, whence, wherefore, etc., are other adverbs that sometimes become conjunctions. h. Seeing, Provided. — These words both perform the office of a con- junction. But another part of speech is not to be called a conjunction be- cause, in certain circumstances, it may take upon itself the connective office. In the sentence, "A man loho is honest will prosper," who is a pronoun, notwithstanding that it also performs the office of a conjunction. (See Demonstration XXX., 1, for "Change of Name.") Not unless, in per- forming the office of a conjunction, the participle can be shown not only entirely to have lost its own character, but also to have acquired the dis- tinctive and exclusive uses of something different from a participle, are we justified in changing the name. In the following examples, seeing is a par- ticiple, and retains the adjectival characteristic of a participle in referring, in each sentence, to some noun or pronoun: — It seems to me most strange that men should fear, [All or they] Seeing that death, a necessary end, Will come when it will come. — ShaJcspeare. I know that thou fearest God, [I] seeing thou hast not withheld thy son; Wherefore come ye to me, [all or ye] seeing ye hate me ? Hinder me not, [ye or all] seeing that the Lord hath prospered my way; [We] Seeing, then, that we have a great High Priest that is passed into the heavens, let us hold fast our profession. 'Bnt provided having lost the distinctive use of the participle, and having assumed, in many sentences, the office of the conjunction exclusively, it should be regarded as such; as, He will go provided his fare is paid. i. As Follows, As Appears, etc. — See Demonstration XV., "As," 3. j. But, Save, and Except. — See Demonstration XIX., Obs. f and g. k. Namely, Viz., As, etc.- — In the following sentence, namely is a con- junction: "I purchased the following articles; namely, tea, coffee, sugar, and raisins." Tea, coffee, sugar, and raisins are each in distributive apposi- tion with the noun articles, which is called the subject, or base, of the ap- positives. And so as in the sentence, " A noun is a name; as, man, eagle." 1. Plus and Minus. — In such sentences as, " Two plus three are five," plus is a co-ordinate conjunction, connecting like parts, tioo and three. '{Jn- l\\eplus, minus is always accessory to a noun; it is therefore always an ad- jective; as, a mmtts quantity; Five minus two is three. In this sentence, five is nominative, subject of is; two is objective, object of the adjective minus, which belongs to five; and three is a predicate nominative, separated from five by the Be-word, is. See Demonstration V. Prog. XX: /. Conjunction. 2. C/ass. 3. Parts Connected. 4. Similarity. a. Co-ordinate. a. Words. \>. Subordinate. b. Phrases. c. Sentences. 31 482 THE SYSTEM METHOD. PARTICULAJRS: Peter and ^ John went to the city. Birds chirp and ^ sing. My father gave me serious and^ excellent advice. We find them in the woods and* by the streams. We are fearfully and ^ wonderfully made. This principle operates in and ^ through him. Henry died loved and '' respected. Art is long, and ^ time is fleeting. He was not only rich, but ^ generous. Not that I so affirm, though i" so it seems. They no sooner looked but ^^ they loved. He must be either ^^ for or ^^ against us. I shall not venture, lest i* I be di'owned. A great man wiU neither ^^ trample on a worm nor^^ cringe before a king. I shall go notwithstanding i'' it rain. I care not whether '^ it rains or ^^ snows. He wiU go provided '^^ his fare is paid. Do you know whether ^^ he has come? As ^^ he was ambitious, I slew him. He fell as ^^ though ^^ he had been shot. A noun is a name ; as, ^^ man, eagle. The English hate the French as ^^ frivolous. Ye shall be as ^'' gods. He came as ^^ an embassador. Freely we serve, because ^^ we freely love. He is both 2* learned and ^^ wise. People trem- bled vrith fear, even^^ beasts were uneasy. I, even^^ I, am the Lord. I hate him; for 3* he is a Christian. I shall not go if ^^ it rain. Few can save ^^ he and I. They submit, since 3'' they cannot conquer. So^* the work is done, is it? London is larger than 29 New York. He is taller than *" I. He gave to him more than ^^ to me. That ^ he went is certain. I was so cold that *3 I nearly perished. If aU this be so, then ** man has a natural freedom. He blushes ; therefore, *» he is guilty. He blinks the question as *^ if *'' he hated to answer. We may be playful, and*^ yet** innocent. That'''' you have wronged me doth appear in this. I do not desire to go; besides ^^ I have not the time. I have lost my money, else ^2 i could help you. Com- monly, however, ^^ these words do not require to be punctuated. Now,^* Barabbas- was a robber. Let cares hke a wild deluge come, So ^^ I but safely reach my home^ — Watts. On these facts, then, ^^ I then rested my argument. I found, too, ^'^ a theater at Alex- andi'ia, and another at Cairo. WeU, ^^ this is a fine morning. Well, 69 proceed with this speech which you have so weU begun. Some words in English meaning the same thing are spelled differently ; namely, ^^ inquire, enquire ; jail, gaol. You may stop' here, or ^i we will go with you. The vase fell from the mantle, yet "^ was not broken. There is no difference except ^^ that "* some are heavier than ^^ others. THE WORDS CLASSIFIED. 483 DEMONSTRATION XXL FACT MATTER : a, Interjection. — An exclamatory word that cannot enter into the body of a sentence ; as, Oh! am I not happy? b. Principle XVII. — A word by nature independent is an interjection. C. Nouns and Pronouns Independent. — Nouns and pronouns are often used like interjections ; but they do not, for that reason, become in- terjections, but are still nouns or pronouns, in the nominative case; as. No, sir! I thank you! lercy! is the man killed? Thou fool ! six feet of earth is all thy store, And he that seeks for all shaU have no more. — Bishop Hall. d. There is one exception — that of the objective me — to the principle that a noun or pronoun independent is in the nominative case. Examples are, — Ah Bie! that I the judge's bride might be. Me miserable ! which way shall I fly? Infinite wrath and infinite despair. e. Verbs used like interjections are, likewise, still to be regarded as verbs, not as interjections; as, There are, say, a hundred soldiers. But hark! he strikes the golden lyre; And see! the tortured ghosts respire. f. Article c above is to apply only to such nouns and pronouns, as, having taken on the function of the interjection, have yet not lost their own functions as noun or pronoun. (See note under Prog. XXI., Work- Book No. 20.) Observe that if, in assuming the characteristics of an inter- jection, the noun or pronoun entirely loses its own characteristics as noun or pronoun, it is then to be regarded, in parsing and branching, as an interjec- tion; for what has lost the peculiar characteristics of a noun or pronoun, and, in the losing, acquired the peculiarities of an interjection, has certainly ceased to be noun or pronoun, and begun to be interjection; as. What! is the man killed? It is plain that what cannot here be called a noun; for it is never noun, even when forming part of the sentence body proper; and if what is never noun inside the sentence body proper, much less can it be noun when out- side the frame-work of the sentence, as it is here used. Being neither fun- damental nor substantive in character, it cannot be nominative independent, as noun or pronoun ; for nominative independent is a sub-branch included within fundamental substantive. (See the Grammatical Work-Tree Complete, p. 494.) And it can be no interrogative pronoun; for it is part of no direct 484: THE SYSTEM METHOD. question, and is connected with no verb. (See Dem. XIV., e, f.) It cannot be a personal pronoun; for neither is it always of the same grammatical number, nor can it be accompanied with a pointer. (See Dem. XIV., e, b, c.) It is not a relative; for there are not here the two necessary verbs which it might link together. (See Dem. XIV., f; also Dem. XV., p. 457.) Therefore, since it has totally lost its peculiarities as a substantive, and, in doing so, has acquired the distinctive features of an interjection, what is an interjection. Prog. XXI: /. Interjection. 2. Principle. PARTICULARS ; False wizard, avaunt ! ^ Ugh ! ^ how cold it is. Alas,^ then, for the homeless beggar old. Tut ! * our horses they shall not see. — Shakspeare. Heigh-ho ! ^ if it be not four o'clock, I'll be hanged. — Id. Whew ! ^ a plague upon you all. — Id. Zounds, '^ wHl they not rob us? — Id. O,* we are undone! O,^ what portents are these! Fie,!*' cousin Percy! how you cross my father! Ha,ii ha,i^ ha! i^ most ex- cellent, in faith! — Shakspeare. Ha,i* cousin, is not this a gallant prize? — Id. Tush!i^ tush!i^ man, I made no reference to you. Ah, i'^ pensive scholar, what is fame? Stand, ho! ^^ Hark! i^ he is arrived. Lo,20 yonder his body lies. O,^^ it is monstrous, monstrous ! Alack,^^ j ^m afraid they have awaked. Ha ! ^3 they pluck out mine eyes ! Hark ! ^^ I hear horses. Halloo,^^ travelers, did you explore these caverns? Poh,^^ poh,^^ I never saw you before. Hist,^* be silent. Alas,^^ the way is wearisome and long. Alackj^" 'tis he. 0,^^ let me not be mad. "Whoop,^^ Jug! I love thee! Ha!^^ sayst thou so? Alas,^*sir! are you here? O,^^ sir, you are old. Hewgh ! 3® give the word. O,^^ look upon me, sir ! O,^* save him, save him ! Ha ! ^9 what is it thou sayst? Ah *" me ! I feel my eyes grow heavy. O *^ Spain ! thou wert of yore the wonder of the realms. Lo! ^ wider grows the stream. O*^ ho,^* O*^ ho ! *^ would it had been done ! Hark,*'? hark ! ^^ the watch-dogs bark. Ah ! ^^ wela- way ! ^^ seasons flower and fade. SiEOoisriD St:e:p_ DEMONSTRATION XX IL In the development of every branch of learning, there are two steps, — (1.) classifying the particulars; (3.) organizing, or branching, the classes so formed into science system. (See pp. 29-31, 66-78, and 160,161.) Thus far we have been wholly engaged in classifying — making classes out of — the words. Of the word we have formed eight classes, — nouns, verbs, pro- nouns, adjectives, adverbs, prepositions, conjunctions, and interjections. And all words whatsoever, every word part of every sentence whole, is included in, and to be explained as belonging to, one of these eight classes. But we are now to take the second step: we are now to branch these classes, together with their higher classes and their sub-classes, into science system — into a grammar-system tree; for we cannot determine whether our language is correct — whether a word is built in the sentence as the right form in the right way — by knowing merely to what class, or "part of speech," that word belongs. We have to see to what branch the word be- longs — through what continuous branch it may be traced. To know that any given sentence is correct, the student must be able mentally to see each of its words as belonging in branch and sub-branch running entirely up to the tree-top branch. Take the sentence, "He has went." We may know that Jie is a pronoun, and that has and went are verbs; but to know merely to what part of speech the words belong is not to know whether the sen- fence is correct. The only way we can know a sentence to be correct (^. «., system -fitting) is to know that each of its words fits the grammar systevi, — to know that each word possesses those attributes that render it traceable through the grammar- system tree. (Refer to the " Grammatical Work- Tree Complete," p. 494.) We see, accordingly, — 1. That Ae=fundamental, substantive, nominative, essential. 2. That /j(is=fundamental, verb, finite, variable, indicative. 3. That we^ii^fundamental, verb, finite, ? Here we discover went to be incorrectly — ungrammatically — built in the sentence; for it is not sys- tem-fitting; it is not congruous with, belongs in no branch of, the grammar system. Went must be called finite, for going and gone are the only parti- [485] 486 THE SYSTEM METHOD. ciples, and go is the infinitive. But, if we call it finite, it must belong in one of the three branches of finite, and be either an indicative, an impera- tive, or a subjunctive. But it cannot be either of these; for each is built, in the sentence, next to and governed by a nominative; vrhile this went is not only disjoined from the nominative he by has, but it is not governed by he — it is accessory to has. The building of went is therefore erroneous, and the sentence is incorrect. It should be corrected to read, "He has gone;" for gone is so branchable completely through the grammar science system; thus: G'o?2/e=fundamental, verb, infinite, participial, verbal complement — to has. Again; take the sentence, " It vs^as him. " It and was are branchable, or traceable, through the grammar-system whole from trunk to topmost branch, and are thus shown not to be incongruous with the grammar sys- tem, but to fit it. But any attempt to find attributes, or class-marks, in Mm, to run it through the grammar-system tree, will fail: i?im=fundamental, substantive, objective, ? But it belongs in no branch of objective, unless it is predicate; but if it belongs in branch predicate, it must be in the same case as iY, the nominative! And since him is an objective, and cannot be the same case as the nominative it, the sentence must be incorrect. Thus, we see by branching the classes of words together into a science- system of grammar, that we may easily determine what is and what is not good grammar; whereas, no amount of study and learning concerning the unbranched classes — the separate classes known as "parts of speech" — would give us such power. Thus our knowing how to know, or think, a word in system, enables us quickly and understandingly to decide what is ungrammatical in a sentence; for it is this fact alone — the fact that the words of a sentence are all system-fitting — that makes the sentence correct. Said Prof. Swing, of Chicago: " An effort is now being made in schools to teach language without teaching grammar. It seems an unreasonable efEort. There is no better basis of correct speech than a knowledge of what makes it correct." But it is the perfect accord of each word part of a sen- tence with the word system, or science, — the science system of grammar, — that makes a sentence correct; therefore, that of which we must get knowl- edge, in order that we may speak correctly, is the science system of gram- mar. Now, the grammar-system tree is not only the learner's instrument, or tool, but it is also the long-sought-for test by which to determine what is and what is not grammatical, i. e., grammar-system-fitting, language. The simple rule, or method, of testing the grammar-system correctness — gram- mar-system fittingness — is this: Every correct word must fit or belong in some topmost branch, and therein, in every higher, or including, branch. Every word found thus to fit — thus to possess all the attributes making up any one complete branch running from tree trunk to tree top — is correct, so far as its grammar is concerned. Every word not so fitting some branching and sub-branching part of the grammar system, is incorrect — unfitting in the grammar system. BBANCHING THE CLASSES. 48T If it be asked whether the grammar-system tree can be made to con- demn such errors as, "John hurt herself," "It were they," I answer. No; grammar cannot be made responsible for the misuse of words as to their m,eanings. The dictionary is the only proper authority on the proper mean- ings of words. Herself is not an error of grammar system; were it so, it would be condemned by the grammar-system representative, the tree. It is an error of the b7-ain "system" of the author of the sentence. It is not to be called ungrammatical, except merely as it is nonsensical. It is proof of this statement that sound-minded children who know nothing whatever of grammar will at once correct this nonsensical sentence to "John hurt him- self." And so with "It were they." Were is not a grammatical mistake merely; it is a dictionary mistake to use the word were in lieu of the declar- ative was. On the other hand, "John hurt he," and "It was them" are grammatical errors, and as such they are easily condemned and corrected "by the tree. a. Grammar Defined. — The science of the word — a part of which the sentence is the including whole. (See the next Demonstration for "The "Word the Science Unit of Grammar.") b. Word. — A sign expressing an idea; the part of which the sentence is the whole. The word is the science unit, or tree trunk, of the grammar system; it is the whole of which the etymon is the part. e. Fundamental. — In grammatical science, the class-branch of which verb, noun, and pronoun are sub-branches. d. Adjunct. — That grammar-system branch, or class, including the "branches adjective and adverb. Adjuncts are often called " qualifiers " or "modifiers." e. Connective. — In grammar science, the higher branch, or class, of which the subordinates are preposition and conjunction; but, in a loose way, the term " connective " is made to include not only preposition and conjunc- tion, but also relative pronoun and conjunctive adverb. f. Transitive Use of Verb. — That use of the verb in which the ac- tion passes upon or results in an object named in the sentence, but not con- nected to the verb by a preposition; as, I was killed in the capitol; Brutus killed me; James was struck by Henry; Henry struck James; He believes God; He walked the plank; She sings soprano; He dreamed a dream; A dream was dreamt; The lesson is recited; James walked the horse down the street; Henry played himself weary; He lorded it over them; Be it enacted; They ran a race. iVoie.— Mark that it is not necessarily true that " the object of a transitive verb is in the objective case." In the sentence, "I was killed," the object— that upon which the action passes — of the transitive verb killed is the nominative /. And so in " The lesson is recited," lesson, though the logical object of recited, is nevertheless a nominative case. g. Intransitive Use of Verb. — That use of the verb in which the action passes upon no object named; or, if so, upon an object disjoined from the verb by a preposition; as, I was killed in the capitol; He believes in God; He walked ore the p?a?iA;/ He writes well; John studies diligently; She 488 THE SYSTEM METHOD. sings beautifully; She was dreaming; He recites well; The timber splits easily; I love to play; Be thou gone; Be it enacted; He desires to go. h. Verbs usually intransitive, are transitive when resulting in, or gov- erning, an object of kindred signification to the verb; as. He ran a, race; He ]e \-? 1 •V -&% t ■« m g rtiXv, •C r^/ dVe- / %sX GRAlVtlvrATICAIv ^ArORK:-TF^.K:E:; INCOlVrFLETE. Copyright, 1886, by I. E. WILSON. dies a righteous death. Other verbs, usually intransitive, are transitive when used in a causative sense; as. The boy flies his kite; The tempest sank the vessel; He trots his horse. BRANCHING THE CLASSES. 489 Prog. XXII.— Branching by the Tree Incomplete.— That the perfectly organized tree of grammar constitutes an invaluable working-guide for learners, has been fully set forth elsewhere. Such a work-tree will be intro- duced as constituting Prog. XXIII. and each program thereafter. But for the purpose of a review, and as a guide in distinguishing the "parts of speech," the foregoing representative of the science system of grammar, though incomplete, will be found a serviceable instrument. For the method of using this tree in preparing and reciting a system- guided grammar lesson, see the work-book, "The System Method," No. 20, Prog. XXII. Of the following particulars, let the student first work up the verbs marked, as a lesson on transitive and intransitive verbs; afterward, all the words in each sentence may be assigned as an exercise in distinguish- ing the parts of speech. PARTICVLARS : I smell 1 roses. Good people hate ^ vice. You own ^ a horse. The multitudes perish.* The rainbow appears ^ beautiful. The leech adheres ^ to the skin. The wax sticks ^ fast. James walked * his horse around the lot. Pharaoh dreamt ^ a dream. The boy flies i" his kite. The boys are ^^ playing ^^ ball. The earth grows ^^ plants and trees. The minister dies ^^ a righteous death. The tailor sews ^^ neatly. A woman toils.^^ A man works.^^ Man's works decay.i* The tempest sank ^^ the ves- sel. The timber splits ^^ easily. The passengers crowded ^^ the car. The author En- glished ^^ the sentence. Calpurnia, here, my wife, stays ^^ me at home. The heavens speed 2* thee in thine enterprise. Thus I roamed 25 me about. He ran ^6 the knife into his leg. He died ^7 a horrible death. They sat^s them down upon the yeUow sand. Was ^^ it a dream we dreamed ? 3" Paul writes ^i a good hand. He out- did ^2 himself. We lowered 33 the bucket. He faced 3* the foe. He walked 35 himself away. Trust 36 God and be 37 doing, 38 and leave 39 the rest with him. Is *<> it true, as is 1* said,^ that mood refers *3 to the manner of assertion ? Son, go ^ work *5 to-day in my vineyard. Up ! up ! my friend, and quit ^^ your books. He saw *7 me, and yielded; *8 that I might *9 say, so with the hook-nosed fellow of Rome, I came,5i saw,^^ and overcame. S3 Let 5* it be 55 who it is : 56 for Eomans now Have 57 thews and limbs hke to their ancestors ; But, woe the while ! our fathers' minds are 58 dead, And we are 59 governed ^^ with our mothers' spirits. — Shakspeare. And why should 6i Csesar be ^3 a tyrant, then ? Poor man ! I know ^3 he would 6* not be 65 a wolf, But that he sees 66 the Romans are 67 but sheep ; He were 63 no lion, were 69 not Romans hinds. — Id. The valiant never taste 70 of death but once. — Id. But what of Cicero? Shall we sound 71 him? — Id. I doubt 73 not of your wisdom. — Id. And now, Octavius, listen, 73 great things. — Id. 490 TEE SYSTEM. METHOD. But when they should ^* endure '^^ the bloody spur, They fall '^ their crests, and, like deceitful jades, Sink " in the trial.— /d. See '^^ he is ''^ arrived.^" — Id. For every tear he falls,^! a Trojan bleeds. — Id. Come, come, and sit*^ you down. — Id. Then why should ^^ j^g despair ^^ that knows to court ^^ it, With words, fair looks, and liberaUty. — Id. A man may fish ^^ with the worm that hath ^'^ eat ^^ of a king, and eat ^s of a fish that hath 90 fed 91 of that worm. — Id. DEMONSTRATION XXIIL FACT MATTER: a. Construction in the Sentence. — Tlie building of the word into the sentence, or the part itself built in. The sentence is a complex and wonderful building. It has bases, or essentials, corresponding to the main timbers in the frame-work of a residence. It has also accessory parts, those attached to, and dependent upon, these base or essential parts. There is also an independent, or absolute, part in some sentences, corresponding to an ornamental stone or tree in the yard, which stone or tree is independent of the building, yet naturally belonging with the building. There are, still, connecting parts, — the preposition and the conjunction. Prepositions and subordinate conjunctions join accessory to essential; co-ordinate conjunc- tions, accessory to accessory, or essential to essential. We have, therefore, of constructions the four kinds following: — b. Essential Element. — One necessary to the constitution, or exist- ence, of a sentence; as. He was ordered to go; It is I; Plato, thou reasonest well; The hack having gone, we were obliged to walk; The mountain rose, hight above hight; Peace, ho! iiear Antony, most noble Antony. C. Accessory Element. — One not essential to the frame-work of the sentence, but simply attached to another part. Thus, in "He was or- dered to go," neither ordered, to, nor go is really essential to the frame-work of the sentence; since he and was form that frame- work themselves, and are the two essentials of the sentence. Other examples are. It is I; Plato, thou reasonest well; The hack having gone, we were obliged to walk; The mountain rose, hight above hight; Peace, ho! hear Antony, most noble Antony. d. Independent, or Absolute, Element. — One neither essential to the frame-work of the sentence nor attached to another part; as, Plato, thou reasonest well; Peace, ho! hear Antony, most noble Antony; Sir, I beg you to give me a bit of bread; He that hath, to him shall be given; The hack BRANCHING TEE CLASSES. 49 1 having gone, we were obliged to walk; to die, how can I? To ask being forbidden, we proceeded without permission. e. Connective Element. — One used like a preposition or conjunc- tion; as, He went through France and Italy; Sir, I beg you to give me a piece of bread; The mountain rose, hight above hight; Man is born to die; Birds chirp and sing; You may go, but I must remain. f. Infinitive Constructions. — The infinitive is used in two different ways as a verbal complement, and as a substantive object of the preposi- tion to. The verbal comjilement uses are the assertive and the objective, as exhibited in the following explanation: — g. Assertive Complement. — This is the use of the infinitive to com- plete the assertion begun by %cill, shall, may, can, must, do, etc. ; as, I sliall love; Thou mayst love; He dares not undertake it; We can learn; I had rather be a kitten and cry mew. h. Objective Complement. — This is the use of the infinitive standing in the relation of an objective to the antecedent verb, from which it is sepa- rated by a noun or pronoun in the objective case; as, I heard him sing; I feel my pulse beat; They would have us give up our rights; She saw her beauties, star by star, expire. — Byron; I have watched thy current glide. — Longfelloio; I have/e??; my heart beat light. — Id.; I see its sparkling bubbles swim. — Id. i. Substantive Use.— The use of the infinitive as the object of the preposition to; as, Man is born to die; It is high time to wake out of sleep; I want to be a scholar; The Son of Man is come to seek and to save that which was lost; She is old enough to go to school; A mutual acquaintance leads us to respect each other; He loves to ride; She desired him to go; I rejoice to hear it; To enjoy is to obey; It is to be remembered; I have faith to believe; I should laugh to giggle. j. Object of Op or a Comparative. — The substantive infinitive is sometimes the object of of or of a comparative adjective or adverb; as. That is calculated to embitter instead c/ conciliate; He would rather go than stay ; I would better die than live falsely. Life seems a jest of Fate's contriving. Only secure in every one's conniving, A long account of nothings paid with loss. Where we poor puppets, jerked by unseen wires. After our little hour of strut and rave. Are tossed pell-mell together in the grave. — Lowell. k. Pahticipial Constructions. — Like the infinitive, the participle is an infinite mood, since it never takes a subject (see Demonstration SXIV.). And the participle and the verb in the participial mood are identical. It is used in four different constructions, as they here follow: — 1. Verbal Complement Use. — This is its genetic use; as. The day is ending; The night is descending; The bell is pealing; He is universally respected; 492 TEE SYSTEM METEOD. It may be known by its use; Bills are requested to he paid in advance; He or- dered the bridge to be broken down; Jolin is cutting wood; Be ye reconciled to God. m. Substantive Use. — Of substantive participles, there are three varieties or divisions, — the subject participle, the predicate participle, and the objective participle. 1. Subject farticiple : Giving ^s receiving; Stealing ^s lying; Forgetting the books delayed us; Doing is not sleeping. 2. Predicate participle : Giving is receiving; Stealing is lying; Doing is not sleeping; I consider preaching to be teaching. 3. Objective participle : I felt K^'e singing; I consider preaching to be teaching; We could not avoid breaking our promise; We all fell to eating; By culti- vating cheerfulness, we become happy; Dying for principle is a Idglier virtue than scolding. n. Adjectival Use. — Examples are. Being defeated, Tie retired; The darting sioalloios soar and sing; The rising morn has but the stars; I hear the bluebird prophesying spring; There, enraptured, he wanders; I feel my pulse beating; They stand by the throne all transfigured; Kneeling, /reread then the prayer of consecration. 0. Adverbial Use. — Examples are. They came pleading for peace; He came tumbling down stairs; The Son of Man came eating and drinking; The STLQ-w fell hissing in the brine; The tide came rushing in the harbor. p. The Word is the Science Unit op Grammar. — It has been else- where shown, in this first volume, that every real science is a system of what is the part of an including whole. Of orthography the science unit is letter, of which the including whole is etymon. Of etymology the science unit is etymon, of which the including whole is word. Of grammar the science unit is word, of which the including whole is sentence. Thus it is that orthography, etymology, and grammar are three sciences, of which the science units — the tree-trunk units — are letter, etymon, and word; in which, if we add sentence, we see a congeries in which the lowest part is the letter, and the greatest whole, the sentence; and in which etymon and word are wholes for orthography and etymology, and parts for etymol- ogy and grammar. Now, since the tree-trunk, or focal unit, of grammar is the word, and since it is the word — not the letter nor the etymon nor the sentence — that we distribute into nouns, verbs, pronouns, adjectives, ad- verbs, prepositions, conjunctions, and interjections, therefore, every word, therefore every single word of itself, is some part of sjjeech and to be treated as such. Let not the slightest heed be given, therefore, to the coxcomical science of any grammaticaster who teaches that "to before an infinitive is not a preposition;" that "for before the objective subject (!) of an infin- itive" " is not a preposition," "but is merely an introductory expletive,*" % BBANGHING THE CLASSES. 493 that "certain combinations of toords [as if, as well as, except that, forasmuch as, but also, but likewise, notwithstanding that, not only, etc. — Harvey's Revised Grammar, p. 129] should be parsed as conjunctions or conjunctive adverbs;" for, since the word is the system-unit — the thing studied, classi- fied, and branched into system— of grammar, grammar is as much bound to explain every word separately as zoology and botany — in which animal and plant are the units studied and systematized — are bound to explain every animal and every plant separately. What sort of a botanist would he be who should pretend to explain a banana and a lily together as one plant! Such sort of grammarian must he be who pretends to explain as well as as one conjunction. But what shall we say of that scientist, or systematizer(!), who, by treating can go, should go, etc., as one verb, makes it absolutely impossible to develop or use any such thing as a real system of grammar! (See "Learner's Process of Work Benighted," "One Verb Consists of a Single Word," and "Two Words as One," pp.. 377, 387, 393.) Note. — Of course, in the case of an appositive noun or pronoun, where the grammatical properties of base and appositive are identical, the disposing of two words as one is not at all to be condemned. (See Work-Book No. 20, Prog. IV., word 78.) q. A Mental Thee in the Memory. — It has been pointed out that the only way we may really know that any word in the sentence is grammatic- ally correct, is to know that it is system-fitting. But before we can know whether a word fits the grammar system, we must understand the grammar system. And before we can so understand the grammar system that we shall be able to use it instanter, as in daily life we always need and must use it, a perfect mental image of it must be sunk in the memory. The lack of this mental grammar system on the part of learners, clearly explains their inability to know and decide whether a sentence is correct language. Prog. XXIII.— Branching by the (Jrammar- System Tree.— The following is believed to be the perfect representative of the science system of grammar. As such, it constitutes the learner's only possible instrument in knowing and remembering how to solve any grammatical— grammar-system — prob- lem. It is hereafter to be used by the learner in "doing" the system of grammar, and is to be learned in the using. [494J Copyright, 18S6, hy I. E. WILSON. BRANCHING THE CLASSES. 495 PARTICULABS : He might havei been^ elected.^ How old must a man be* to be ^ a bachelor? Please to give ^ us bread to eat J He is anxious to learn ^ how to draw.^ Man is born i** to die.^^ I should laugh i^ to cry.i^ -phe book is to be i* studied.i^ He was ordered ^^ to flre.^^ The beef was ordered i* to be ^^ fried.^*^ We saw him fall.^i Let him be 22 C^sar. AU were sealed ^3 with the seal which is never to be 2* broken.^^ Let us be^s going.^^ Let it drive.^* To be ^9 good is to be ^o great. Time to come ^i is called ^2 future. Thou mayst love.^^ I want to be ^* quiet and to be ^^ let ^^ alone. It is to be 2^ remembered. 38 I feel my pulse beat.^* He has come *" to work.*i I ha rather be ^^ a kitten and cry *3 mew. They would have ** us give ^^ up our rights. He has laughed*^ to see*^ us suffer. *3 I see its sparkhng*^ bubbles swim.s" To enjoy ^i is to obey. ^2 M&n must die.^^ Giving 5* is receiving.^^ I consider preaching ^^ to be 5^ teaching. 58 BiUs are requested ^^ to be ^^ paid ^^ in advance. We aU feU to eat- ing. ^2 He came tumbling ^3 cIq-j^ti stairs. They came pleading ^* for peace. Life seems a jest of Fate's contriving, ^^ • Only secure in every one's conniving, ^^ A long account of nothings paid ^^ with loss. Where we poor puppets, jerked ^^ by unseen wires, After our Uttle hour of strut ^^ and rave,™ Are tossed '^^ peU-mell together in the grave. — Lowell. See it lightening. ^2 j j^ad better go. '''3 Had it not been ^* for Dryden, we had never known ''5 a Pope. Be it enacted, ^^ That schools shall be''"' estabhshed.^8 I hear the bluebird prophesying '^^ spring. Let him be 80 punished.^i Be so good as to go.82 I prefer rather to go 83 than to stay. 3* Being 33 let 36 go, 37 they went their own way. They kept him going.38 I do not like 39 her singing. ^'^ I was opposed ^^ to his writ- ing ^2 tiie letter. My son is going ^3 ^ be 9* married. ^^ Truth, crushed ^^ ^^ earth, shall rise''' again. By singing, ^8 birds delight us. He is compelled '^ to go.i"" Close beside her, faintly moaning,!"! fair and young, a soldier lay. Torn 1^2 with shot, and pierced !''3 with lances, bleeding !"* slow his life away. — Whittier. He is said^os to havei"^ gonc^^^ He is beheved "3 to have i"' been '^^^ caught."^ He could not avoid 112 giving ii3 offense. The beginner should be"* accustomed ^^^ to giving 116 aU the reasons. He would sooner die i^'' than askii3 a favor. His being ^i* a foreigner should not have 120 prevented 121 his being 122 elected. ^23 Supposing 12* he were here, what could you do? 125 The branch, being ^26 broken ^27 off, fell. His dancing 128 set us all to laughing. i29 He wishes to do i30 something more than eat ^31 and sleep.132 Better not to be ^33 at all than not be noble. 49G TEE SYSTEM METHOD. DEMONSTRATION XXIV. FACT MATTER : a. Variable Verb. — One so built in the sentence tliat if I, thou, and he be substituted for its nominative, its form will vary; that is to say, a variable verb is a verb in the indicative mood. b. Invariable Verb. — One that does not change its form to accord to changes in the person or the number of the subject-nominative; that is to say, one not in the indicative mood. Examples are, I shall go if it be possi- ble; He was ordered to go; Be it as it may, I must go. C. What Verbs Have Subjects? — Indicatives, imperatives, and sub" junctives, those that constitute the verb-half of the frame-work of the sentence; for it is tHe essential, or finite, verb-half of which the subject- half is the subject. It will be observed that some essential verbs, the in- dicatives, change to accord to changes in the person or the number of the subject-nominative. Thus, in "I can go," the indicative can changes to canst, when /is changed to t7iou; but the verbal complement infinitive go is not altered by changing I to thou. Now, when a verb constitutes the es- sential verb-half of the frame- work of a sentence, it is said to have a sub- ject; and when it thus has a subject, it is said to be z. finite verb. In this way, we see that the finite verbs are the indicatives, the imperatives, and the subjunctives. The infinite verbs, the infinitives and the participles, cannot form this essential verb-half of the sentence frame-work, and cannot, therefore, have grammatical subjects. And observe that not the infinitives and the partici- ples alone, but also all imperatives and subjunctives, are invariable verbs, the only variable verb being the indicative. This variation in the indica- tive, to make it accord to the change of person or number in its subject- nominative, is what has been called the "person and the number of the verb." " So far as the verb has different forms of person and number, the form used is of the same person and number with the subject, being, therefore, always in the third person if its subject is a noun: thus, — "I give, not I gives; thou goest, not thou go or goes; "He runs, not he run; the man runs, not the man runnest; "We are, not we am or is; the men were, not the men was. "This is also expressed by saying that the verb agrees with its subject in number and person, or that the subject governs the verb in number and person; that is, requires the verb to be of a certain character in these re- spects: the subject being given, the verb is compelled to correspond with it in number and person." — W. B. Whitney . BRANCHING THE GLASSES. 4,97 "As the verb is the essential part of every sentence, or the part that makes the assertion, the subject of the sentence is also called the subject (or subject nominative) of the verb." — Id. d. Finite Verb. — One used as the essential verb-half of the sentence frame-work. e. Infinite Vekb. — One not used as the essential verb-half of the sen- tence frame-work. f. PpaNCiPLE VIII. — A variable finite verb is in the indicative mood. g. Principle IX. — An invariable finite verb is in the subjunctive or the imper- ative mood. h. Principle X. — An infinite verb accessory to to, or to another verb and justifying with to, is in the infinitive mood. i. Principle XI.— An infinite verb ending in ing, or not justifying with to, is in the participial mood. k. The Current Rule Erroneous. — The indicative is the only mood in the English language that has variations for person and number. Hence, contrary to the practice of many teachers, consistency would aban- don the distinction of person and number when parsing all moods except the indicative; for what sort of science is it that distinguishes between he and be — between he and itself! Rather, is it not the fancy and fuss of nonsense to be making such a distinction? The conjugation in the sub- junctive runs, I be, thou be, lie he, between which forms of be there is no dis- tinction. Thus it is seen that the rule should read, " An indicative must agree with its subject in person and number," and not, " A finite verb must agree," etc. The conjugation correctly continues. Hove, thou love, lie love; I do, thou do, lie do; and so in the subjunctive mood of every verb in the English lan- guage, none having any change whatever. If the reader will draw two lines upon a slate, refer to the exami^les of the subjunctive present already given, take those verbs and write them so that the verbs in the subjunc- tive will come between these lines, thus: — far from home, he will have friends, drink at the festival, she will renounce him. or come, do not fail to send me word, his estate, he will maintain his integrity, go, I will inform you, — in this way he will observe that no termination whatever appears at the right of the second line, but that the form of the verb is always the simple infinitive form, the one used with I; thus, I go, I love, etc. The reader can now see that in all verbs the subjunctive present is the same for all persons and numbers. Hence, except for the indicative, the absurdity of the rule, "A verb must agree with its subject in person and number." For, if the first person of the subjunctive is the same as the 32 1. Though he get 2. If he do 3. Whether he go 4. Though he lose 5. If I do 498 THE SYSTEM METHOD. second person, and the second tlie same as the third, how can there be any difference among them? But if all three are the same, and any one "agrees," will not either of the other two agree? Nothing but the sagacity of a grammarian could detect a difference between things that are the same. Nothing but the wisdom of a grammarian could perceive the necessity of a rule of agreement where disagreement is impossible! 1. The SuBjtJNCTrvE will Never "Fade out." — More vicious teach- ing is the doctrine that the subjunctive " as a form of the verb" is of no practical value, and will eventually "fade out." The history of the sub- junctive through the Greek, Latin, and Teutonic languages clearly shows it to be not less persistent as a form of the verb than is the objective case of the pronoun. Its vitality may be illustrated in this way: — 1. If his feet be large, he will not sell well. 2. If his feet are large, he will not sell well. Now, if I am trained myself to represent facts undetermined, contingent, and unknown, by the subjunctive present, as in the first example, and events at present determined in fact, by the indicative present, as in the second example, I easily conclude, — 1. That in the first examjDle a colt is referred to; but — 2. That it is a liorse referred to in the second. For the future size of the feet, alone, might it be, that could be undeter- mined and unknown. The size of a full-grown animal's feet alone, might it be, that could be at present determined in fact. Evidently, not till the distinction itself between the colt and the horse is abandoned, will the means of that distinction be abandoned. Again, take — 1. If that is your opinion, I shall report it. 2. If that be your opinion, I shall report it. Here, also, if I am trained myself to represent events undetermined and future by the subjunctive imperfect, as in the second sentence, and events at present determined in fact, by the indicative imperfect, as in the first sentence, I conclude, — 1. That in the first sentence the opinion is already formed at the time of speaking; but — 3. That in the second sentence the opinion is not already formed, but future, and therefore undetermined and contingent. Prog. XXIV. — Branching by the Grammar- System Tree. PARTICULARS : The enemy advancing,^ the vanguard retreated.^ They refusing ^ to coTiply,* I advanced.^ He thanked^ us for saving^ his life. He wist* not what to say. ^ They kept 1" him going." We all fell ^'^ to eating.^^ He could i* not avoid ^^ giving ^^ offense. Great Nature spoke; i'^ observant man obeyed; ^* Cities were^^ formed; ^^ societies were^^ made.^" Time never ceases ^^ flying."* There the wicked cease ^^ from troubling. ^^ When BBANGHING THE GLASSES. 499 does 2'' he purpose ^^ to go.2' To love ^^ our neighbors as ourselves is •''^ divine. The cars having ^^ arrived,^^ we departed.^* Csesar, having ^^ crossed ^^ the Rubicon, gave '^'' battle. To speak ^* plainly, your habits are ^^ your worst enemies. Think *<* you I am *i no stronger than my sex. Being ^^ so fathered ^^ and so husbanded ? ■** To be *^ at war with one we love,*'' Doth *^ work *^ like madness in the brain. — Coleridge. She gave *^ me of the tree, and I did ^^ eat.^^ Truth, crushed ^^ to earth, shall ^^ rise ■''* again. What was ^^ it that moved ^6 pale Cassius to consi^ire ? •'''' Who could 5* refrain, 5^ That had''" a heart to love,"! and in that heart Courage to make "^ his love known? ^3 — Shakspeare. We saw^* the storm approaching. ''5 I felt^^ my heart beating. ^^ Remembering''^ matter pays ^^ the debt. Searching^" the window for a flint, I found ^^ this paper, thus sealed ^" up. I do '''^ entreat ''^ that we may ''^ sup ™ together. No man can '^'^ do ^^ these miracles except God be '^^ with him. My Prometheus, which has ^° long been ^i finished, ^2 is ^^ now being ^* transcribed.^^ I knew ^^ my lesson before the clock had ^^ struck, ^s That very law that molds ^^ a tear, And bids It trickle ^^ from its source, That law preserves the earth a sphere, And guides ^^ the planets in their course. — Rogers. I want^^ to be 33 quiet and to be^* let^^ alone. Had**^ we known ^'^ this, we could '^ have 39 aided i"" you much. Hear i^i me, for I wiU 1°^ speak.^''3 j g^y i04 g^ch of j-ou shall 1"^ be^"^ rewarded. ^"^ It matters 1"^ little, ultimately, how much a laborer is^o^ paid 11" for making ^ anything ; but it matters i^^ fearfully what the thing is "3 -n-hich he is 11* compelled 11^ to make.n^ — Ruskin. I haveH^ long beenH* accustomed, n^ as all men engaged i^" in work of investigation musfi^i be,i~2 to heari33 my statements laughed 1-* at for years, before they arei^^ examined i-^ or believed;!"^ and I am i^* generally content to wait i^^ the public's time. — Id. Simonides, being i3" asked i^^ by Dionysius what God was,!^^ desired i^^ a day's time to consider 13* of it before he made i^s his reply. — Addison. I was i36 born i3^ to a small hereditary estate, which, according i33 to the tradition of the village where it lies,i39 wasi*" bounded i*i by the same hedges and ditches in William the Conqueror's time that it isi*^ at present. — Id. Long in its dim recesses pines the spirit, Wildered and dark, despairingly alone ; Though many a shape of beauty wander i*3 near it, And many a wild and half -remembered tone Tremble i** from the divine abyss to cheer i*^ it, Yet still it knows ^*^ that there is only one Before whom it can i*^ kneel i*^ and tribute bring, i*" At once a happy vassal and a King. — Lowell. 500 THE SYSTEM METHOD. What we call ^^^ Nature — all outside ourselves — Is 1^1 but our own conceit of what we see/^'-^ Our reaction upon what we feel; ^^^ The world 's a woman to our shifting ^^* mood, Feeling ^^^ with us, or making ^^^ due pretence ; And therefore we the more persuade ourselves To make '^^'' all things our thought's confederates. Conniving ^^s ^yith us in whate'er we dream. 1^9 — Jd. Our slender hfe runs i^" rippling ^^i by, and glides ^^^ Into the silent hollow of the past ; What is 163 there that abides i''* To make 165 the next age better for the last? Is 166 earth too poor to give i6^ us Something to live 16* for here that shall 16* outlive i™ usi Some more substantial boon Than such as flows i^i and ebbsi'^ with Fortune's fickle moon? The little that we see i^3 From doubt is i '* never free ; The little that we do,"^ Is 1^6 but half-nobly true ; With our laborious hiving i^^ What men calli'^^ treasure, and the gods calli'^^ dross, Life seems i^" a jest of Fate's contriving, i*i Only secure in every one's conniving,!^^ A long account of nothings paid i^^ with loss, Where we poor puppets, jerked i^* by unseen wires. After our little hour of strut i^^ and rave,i^6 With all our pasteboard passions and desires. Loves, hates, ambitions, and immortal fires. Are 1^' tossed i*^ pell-mell together in the grave. — Id. If thou thouesti^^ him some thrice, it will not be amiss. DEMONSTRATION XXV. FACT MATTER: a. Construction op Names. — In construction, nouns and pronouns, like verbs, are essential, accessory, or independent. (See Demonstration XXIII.) b. It, You, That, What, and As. — Review Demonstration XV. C. Directions for Finding the Case. — See Demonstration V., g, h, i, j; also Demonstration VI., a, b. BRANCHING THE CLASSES. 50l d. The Offices of Nouns and Pronouns. — The specific part per- formed in the sentence. Of the oflices of nouns and pronouns there are five, — subject, predicate, appositive, adnominal, and object. e. Subject. — The term "subject" has been taken from the province of logic, and is of questionable value to the science system of grammar. Subject is from sub, under, andjacere, to throw, and refei's to whatever is thrown under the extension or dominion of something else. I have shown elsewhere many times (see pages 15-40 inclusive) that every thinking act is an act thinking particular, or subject, under the extension of some known class; hence, the logical definition of subject: Thatwldch is thrown, or thought, under the extension ofknoion class; or, commonly, "That concern- ing which anything is affirmed or denied." — Watts. f. GrammaticaIj Subject. — The following are three definitions of a grammatical subject: — 1. The substantive half of the frame-work of the sentence. 2. The substantive essential to the constitution of a sentence. 3. That which determines the form and j)osture of the essential verb. Examples are. It is they; It is I; What am 1? Who is he? Who are they? What hurts the man? Bills are requested to be paid in advance; Thine [ ] is the kingdom and the power and the glory; His pavilion were dark waters and thick clouds of the sky; It is sweet to die for one's country; Abide ye here; Somebody call my wife; Which is yours [ ]? g. A Blunder. — The grammatical subject is much more than merely "that of which something is affirmed." It is that which determines the form and posture of the verb. Grammar is the system, or science, of the word, — the structural part of which the sentence is the whole; and every grammatical distinction is based on the difference of structure or posture among these word-parts of the sentence whole: nearly as human anatomy is based on the difference of form and posture between different organ- parts of the man whole. Two different words may be like in form, just as the two eyes or the two hands are like in form; but, if so, they will be un- like in their posture in the sentence whole, just as either the two eyes or the two hands are unlike in their posture in the man whole, one eye and hand being left in posture, the other being right. Now, there is a corre- spondence between subject and verb as between the two hands, and between the two eyes. Given the right hand, then the form of the left, as well as its posture in the man whole, is easily determined. It is in this way that from the subject we determine what both the form of the verb and its posture in the sentence whole will be. Having never seen what part and whole in grammar mean; having never known how grammar is a real science system of the word, a part of which the sentence is the including and mirror whole; but metaphysically trying to solve the problems of grammar by vague principles of logic, many grammarians seem unable to distinguish the subject of the simple sentence. Thus, Thos. W. Harvey, in his "Practical Grammar of the English Lan- 502 THE SYSTEM METHOD. guage," page 63, declares that ■zc/wcA, in the sentence, "Which is yours?" "is used as the subject of the sentence"! Now, for convenience alone, we will suppose that yours has reference to a knife; for, being in the possess- ive case, doubtless it must limit, or possess, something. Then, for proof, change the singular, knife, with which the singular verb is agrees, to the plural knives, and the verb must change to are, and the sentence, will read, " Which are your knives?" But, with the rest, Mr. Harvey teaches, in his 13th rule, that that word with which the verb agrees, is the subject. There- fore, of the last sentence, the subject is knives; and of the first, the subject is a singular noun understood, and not the pronoun which. Had Mr. Har- vey discovered that in the proper scheme of grammar, the subject is the substantive essential, — that which determines the form of the verb, — he could not have committed himself to so laughable a blunder. But this er- ror is the sequel of another of the same anti-form, or "logical" (!), sort, found on page 63 of the same book, where it is declared that hers is in the neuter gender! Once in that error, this author had no form-and-posture test — had no principle except that of "logical" affirmation — by which to determine what is the subject in "Which is yours? " Very few have a just conception of what a grammatical subject is. h. Grammatical Predicate. — "Predicate" is from the li^iim prcedi- care, to cry or proclaim. Hence the definition: Those words which repre- sent what is cried, or asserted, of a subject; as. Chalk is white; John is a scholar; Brutus speaks; It is I; What am I? Somebody ring the bell; Long live the king! i. Predicate Noun, Pronoun, Adjective. — Any noun, pronoun, or adjective in the predicate after a copula (see Demonstration XXVI.); as, It is I; Hassan is my name; He is true; We found him to be true; They took him to be an enemy; They hated him as being an enemy. (See Demonstra- tion v., g, h, i, for " The Case Determined" and "Be and Same Meanings.") j. Appositive (= ad, "to," and position: close position). — The office of a ^ name accessory to another and meaning the same thing; as, I, thy father-in-law Jelhro, am come unto thee; Senator Conkling resigned; He did it himself; Peace, prosperity, happiness, — all these things were sacrificed; He is crippled, hand and foot; The stars, they shall shine forever. (See Demon- stration VI., b.) k- Adnominax (= ad, "to," and nominal, "name:" to a name). — The oflace of a ^name in the possessive case; as, Our gain is William's loss; Henry's being a student is no reason for his being a savage. 1. Object. — The office of a ^name accessory to any verb not a copula, or to a preposition, an adjective, or an adverb; as, I paid him the money; They called her Mary; We shall remain one hour; He stopped at Rome; The lake is ten miles long. '^Note. — Name, as here used, includes both nouns and pronouns. BRANCHING THE CLASSES. 503 Prog. XXV: /■ Substantive. 2. Class. 3. Case. 4. Construction. 5. Office. a. Noun. a. Nominative, a. Essential. a. Subject. b. Pronoun. b. Pofssessive. b. Accessory. b. Predicate. c. Objective, c. Independent, c. Appositivc. d. Adnominal. e. Object. PAHTICULARS : An idle brain ^ is the devil's^ workshop.^ He* was paid money.^ Some crim- inal^ is being tried for murder.'' Milton!^ thou^ shouldst be living at this hour.^'* Fiddlesticks ! 1^ it isn't worth the powder. ^^ Hope^^ deferred maketh the heart i* sick. — iSolomon.^^ Mr. Jones/^ the banker,!^ has gone to Europe.^^ Conscience!^' how it 2" rains! I,^^ Richard Roe,^^ do solemnly swear. He^^ scaled Mt. Blanc, ^* — a daring feat.^^ Ho ! gunners,'^^ fire a loud salute.^'' Come hither, my little daughter.^* A Daniel^' come to Judgment! yea, a Daniel.^" I^^ resolved to practice temper- ance^^— a resolution^'' I have ever kept. There is no harm^* in children's ^^ playing by the roadside.^^ Paul 3'' being a Roman,^* they^' feared to kill him.** Bingo," why Bingo !*2 hey, hey — here, here! His*^ teeth,** they*^ chatter, chatter still. Shame*® being lost, all virtue *''' is lost. Caesar,** thou*' dost me wrong. ^^ — Shaks- peare.^^ Leonidas,^^ Cato,^^ Phocion,^* TeU,^'' — one peculiarity^® marks them^'^ all. — Emerson.^^ What ! Celso,^' thou *" turned courtier? ®i Well, John,*'^ I ®^ hope you "* did not laugh. He®^ caused himself''® to be proclaimed king.®'' O mighty Caesar! ®s dost thou®^ lie so low? My™ friend, ''i Mr. Ogden,''^ will be here to-night. Peace, '''^ ho! hear Antony,''* most noble Antony.''^ The Pilgrim Fathers,'' ®where are they?''^ You ''* Frenchmen ''' are livelier than we ^'^ Englishmen. ^i I ^^ am the true vine,*^ and my** Father *5 is the husbandman.*® Take what*^ you want. It** is important wh-at*' he is. That book'" is hers,'^ not yours.'^ That is a picture'* of my'* father's.'^ Ah me! '® that I the Judge's''' bride'* might be. She soured to what" she is. Their 100 stay,ioi on him "2 theyi®* depend. Sister i'* Maryio^ is dead. I found the urchin i'® Cupid ^ot sleeping. Now the bright morning star,i®* day's ^oa harbinger,iio Comes dancing from the East.^*'- Me 11^ miserable ! which way ^i* shall I fly. Infinite wrath i^* and infinite despair.^i^ — Milton.^^^ "Gibbon's Rome."i" "Bakery"* and Restaurant.""' The wiU! *20 tj^e willli^i we wiU hear Caesar's ^^^ will-i*^* I^*^* do not care a straw. ^^^ I am the sister i^s of one 12^ Claudio.i~* Paul,i"' the apostle,i*o -^^as a true Christian.!*^ Shakspearei*^ excels as a dramatic poet.^^* Their i** betters i*^ cannot be found. They ^^s tore him, limb 1*' from limb.^** The mountains i*' rose, hight i*o above hight."^ I pitied the proud son i*^ returning a beggar.^** Liberty v/as theirs '** as men.i*^ He i*® destroyed, the rest will yield. He^*'' was paid money.^** Let him^*' be Csesar.^so The potatoes ^^^ measured a bushel. ^''^ Let the angels ^^s be our guides, i^* Pit'ticus ^^^ was offered a large sum^^® by the king.^'''' The program i^* read, Music,^^' Toasts,i®o Supper.i®! A noun is a name ; i®2 as, eagle,^®* man.^®* We purchased these articles; to-wit, carpet,i®5 lamp,i®® and book-desk.^®'' Do what^®* is right. Theyi®' found the party 1'® to be Henry.i^i jjg ^y^s made fun^^ of. I consider Dr. Johnson i''* as an excellent moralist.^'* Whoever i''^ did it^'® ought to be punished. It*'''' struck ten before it began to rain. Whatever *''* he i" does, you i*® mind what 1*^ I tell 504 THE SYSTEM METHOD. you.^^^ My ^^^ name i^* is Hassan.i^s Tq reign is worth ambition.i^s 1 187 discoyered him 1^8 to be a scholar. iS9 The learned pagans"" ridiculed the Jews^i for being a credulous people.i^^ His "^ being a lawyer i^* was no reason "^ for supposing him i^^ to be a rascal."^ Sir Philip Sidney ^^^ lived and died the darling"" of the court.^oo 1201 asked what 202 she^o^ knew about the matter.^o* It cost him^os a dollar.^os "VVeSOT yfCYQ defeated several times.^os Two times 209 two^" are four.'^n He con- templates a return 212 home.^ia The instinct 21* to protect one's 2i5 self2i6 comes by nature.2" Evil2i8 hides behind one's 2i9 back.220 This one is a great deal 221 better. The lake is five miles 222 long. They left the job 223 a botch.224 This law 225 preserves the earth 22s a sphere. 227 God 228 called the firmament 229 heaven.23'' It is not worth while.231 Who is a strong Lord 232 jjije thee? 233 His salvation 23* is nigh them 235 that 236 fear Him.237 He staid in Rome 238 five months.239 He came an hour 2*0 sooner. "We 2" shall start one hour 2*2 hence. Yours 2*3 is the oriole,2*i mine 2^5 {g the canary.216 Avoid such as 2" are vicious. He collected as much as2iSisdue. The positions were such as 2i9 appear incontrovertible. The conditions 250 are as 25i foUow. The conditions of the agreement were as 2^2 follows. Be courteous to your 253 betters.25* They love each other.255 One256 should defer to his 257 superiors.258 The possessives 259 are formed by adding the apostrophe to the nominatives.2eo What 261 is two thirds 262 of a unit? 263 She264 was promised the jewels.2C5 A better proof 266 of his 267 being a rogue 268 is not desired. DEMONSTRATION XXVI. Orammatical, or Dependent, Coni^tructiong. — Mark the difference between construction and case : construction pertains to tlie way a word-part is built, or woven, into the sentence; case is the postural form which the word acquires by being so built in as a part of the sentence whole. (See Demonstration XXV., g) Of the constructions, there are two varieties: (1.) Those which are purely grammatical, the essential and the accessory constructions; (2.) Those which are rhetorical, that is, independent of the real sentence whole and all of its parts. The cases of nouns and pronouns in the dependent (essential and accessory) construction are three, — the nominatim, the possessive, and the objective. a. Nominative. — That form or posture which a name acquires in be- coming either the subject or the predicate of a finite verb. Of nominatives, there are three branch classes, as follow here: — \. Essential, or Subject, Nominative: I exist; Who struck me? wherein the finite verbs exist and struck govern the case of the subjects / and who, requiring them to be nominative, in so far as such forms as " My exist" or "Me exist" are not allowable. (See Demonstration XXV., e, f, g.) 2. Predicate Nominative: It is they. I must be myself. He became a laioyer. He desires to be a lawyer. James is said to be a good boy. His being BRANGHINQ THE CLASSES. 505 a student was no reason for liis being a savage. Ye shall be as gods. He came as a business man. I did not bear of bis becoming a convert to tbat faith. It was to be his Jiorse. To be a good scholar requires much labor. (See Demonstration VI., a, b.) 3. A2>positlve Nominative. — These are subdivided as follows: — a. Explanatory ajjpositive: Johnson, the lexicographer, was a very ec- centric man. We, the people, are lovers of republicanism. Milton, the poet, was blind. I, ih.j father-in-laio Jethro, am come unto thee. b. Restrictive appositive: The poet Milton was blind. The emperor Ardonius wrote a good book. The word 2)hilosop7ier signifies " lover of wisdom." Thou river, roll. The river Euphrates is in Asia. I, thy father-in-law Jethro, am come unto thee. e. Repetitive appositive: The Lord himself shall descend. He himself was the writer of the editorial. We can manage the business ourselves. d. Distributive appositive: He is crippled, hand and/oo^. He is ruined, mind and heart. The allies — two thirds of them — invaded the country. Peace, prosperity, happiness, — all things were sacrificed. e. Resumptive appositive: It was Virgil, he who wrote the ^neid. A great and true man, the king sat upon the throne of his ancestors. Harry's flesh, it fell away. The stars, tJiey shall shine forever. The orphan of St. Louis, lie became the adopted child of the republic. The wind, it waved the willow boughs. You are too humane and consid- erate, things which few people can be charged with. By telling stories, a practice of which he was fond, he became popular with the children. Cortez, he who conquered Montezuma, was a Spaniard. Enow ye that the Lord, he is God. Mary gave the old man a cup of water, a kindness which he seemed to appreciate. The Pilgrim Fathers, where are tJiey'i b. Objective. — That form and posture which a name acquires in be- coming the objective predicate of an infinite copula, or the object of a prep- osition, adjective, or adverb, or of any verb not a copula. Of objectives, there are seven branch classes, as follow below: — - 1. Predicate Objective: They desired him to become a lawyer. Let him be Cmsar. Let him reign king. We thought it to be them. They hated him as being an enemy. I saw the child growing up the pride of the family. I pitied the proud son returning a beggar. 2. Factitive Objective. — One that names what is the outcome of the verb's ac- tion on a direct object (see 4 below); as, They made the log a boat. They chose her queen. He left the job a botch. That law preserves the earth a sphere. God called the firmament heaven. They did not esteem themselves men. 3. Appositive Objective: Elizabeth ordered Mary, Queen of Scots, to be be- headed. What a word escaped thee, thy lips. They beat him merci- lessly, head and body. I purchased the following articles; namely, tea, sugar, coffee, and raisins. Massachusetts censured her ablest statesman, Charles Sumner. 506 TSE SYSTEM METHOD. 4. Direct Objective.— The direct object of a transitive verb; as, I paid him his wages. He made me a coat. Vice emits a breath every moment. They did not esteem tJiemselves men. He vv^as paid money. 5. Indirect Objective. — One joined to a verb in the relation signified by to or for; as, I paid liim the money. He made me a suit of clothes. James gave me the book. I sent mj friend the book. We forgive our friends their faults. 6. Adverbial Objective.— This objective sustains the relation of an adverb to the verb, adjective, or adverb which governs it. The following are examples of the objective governed by verb, adjective, and adverb re- spectively: — a. Object of a verb : Yice emits ahvehtheYerj moment. We stayed one hour. It fell a great distance. The horse iceighs a thousand pounds. The coal measures twenty busJiels. The building /to?? te two st7'eets. I do not care a straio. b. Object of an adjective : The lake is ten miles long. He is not a w7dt wiser. The tower is ninety feet high. The hat is ioortha. dollar. It was a little more than two oniles. I wish they were a whole Atlantic [ocean'] broad. — Tennyson. My father, than whom no man is wiser, ajjproves that course. He is wiser than me. (See Dem. XXX., m.) c. Object of an adverb : He came an hour sooner. An hour hence we will start. They watched all night long. This one is built a great deal bet- ter. It was a long time ago. One hundred years ago, railroads were unknown. (These last two sentences will be more clearly understood after pondering the office of ago in the sentence, "He died long ago.") I should be guilty of ingratitude, than whicli nothing is more shameful. Beelzebub, than whom none higher sat, with grave aspect rose. It weighs a little more than three pounds. Than and as, than whic7i noth- ing has wzwe perplexed the grammarians, are subordinate conjunctions. (Bee Dem. XXX., m.) Whom does he honor more than 7ne ? d. i\7by pleonasm. 'Cortez, he . . . was a Spaniard;' he is in the nominative case by apposition with Cortez.''— Holbrook' s Complete English Grammar, pp. 35, 168. But if he is in apposi- tion with Cortez, then Cortez must be the surbject of ivas, and so Mr. Holbrook avers. But a child will see that stars sustains the same relation to shall shine that Corlez sustains to was; hence, stars is nominative not by pleonasm, but because it is the subject of shall shine, just as he declares Cortez is the subject of was. And thus appears the contradiction. 508 ^^■E' SYSTEM- METHOD. 4. By Accompanying Circumstance : The 7iack having gone, we were obliged to walk to the railroad. The sun being risen, we pursued our journey. Honor being lost, all is lost. The ship came into harbor, colors flying and guns firing. Flowers of all heavens grow side by side. The mount- ains rose, higJit above hight. Century after century, the earth has wheeled in its orbit. They walk hand in hand. They tore him limb from limb. He destroyed, the rest will yield. — Butler. They being absent, the cause was not decided. — Id. He rushed down the hill, heels over head. Heart to heart, we '11 sail together. Hand in hand for aye and ever. Her wh^el at rest, the matron thrills no more With treasured tales and legendary lore, 5. By Inscription. — The chief word in inscriptions on monuments, coins, etc., the chief word in titles of books, magazines, newspapers, etc., the chief word in signs of business houses, labels for goods, or packages, also the signatures of letters, superscriptions on envelopes, headings of pages and subjects, are in the nominative case, and are independent by inscription. Examples are, ' ' Webster's Unabridged Dictionary; " ' ' Gib- bon's Borne;" "Bakery and Restaurant" (sign on a business house); "United States of America" (inscription on a coin); "The Noun" (the heading of a discussion of that subject). Besides the nominative, the only case used in the independent construc- tion is the objective of the pronoun /. For examples, see under the article next below. 6. Me Independent. — The objective me is often in the independent construc- tion, by exclamation; as. Ah me! that I the judge's bride might be. — Wliittier. O me, why have they not buried me deep enough? — Tennyson. Dear me, what shall we do? Me miserable ! which way shall I fly. Infinite wrath and infinite despair. — Milton, Prog. XXVI.— Branching by the Gframmar-System Tree, FABTICVLAMS: Which 1 is yours? ^ (See Dem. XXV., g.) It ^ was to be his horse.* He ^ staid anhom-.^ I ^ paid him Hhe money. ^ He i" was paid money," To be a good scholar ^2 requires much labor.i^ "VVe,i* the people,i^ are lovers i** of repubUcanism.i'' -pi^e horse 1* weighs a thousand pounds.^^ Than^^ and as^"^ than which ^'-^ nothing has more perplexed the grammarians,'^^ are suboriiinate conjunctions.^* Solomon,"'' than whom ^^ there never was a wiser man,*^^ declares wisdom ^^ more precious than rubies.^^ Harry's flesh,30 jt 3i fgU away. The stars,^^ they^s shall shine forever. He^* is crip- pled, hand ^5 and foot.36 He 37 is ruined, mind 38 and heart. s^ The Lord *" himseK " BRANCHING THE GLASSES. 509 shall descend. James *^ is said to be a good boy.*^ His ** being a student*^ was no reason*^ for his being a savage.*^ Ye*^ shall be as gods.*' (See Dem. XX., d, note 1.) He ''° came as a business man.^i The poet ^^ Milton ^^ ^yas bhnd. Milton, 5* the poet, ^5 was blind. The allies ^^ — two thirds '^^ of them ^^ — invaded the country.^^ Thou ^" river, 61 roll. The Pilgrim Fathers,^- where are they? ''^ Know ye ^^ that the Lord,''5 he ^^ is God.^''' The word ^^ philosopher ^^ signifies " wisdom-lover." ^^ Plato, '^^ thou '^2 reasonest well. Nonsense ! '^^ it ^* can't be done. Most noble C^sar ! ''^ we ^"^ '11 revenge his '^'' death. '''^ He ''^ that ^^ is holy, let him *i be holy stiU. Cortez,*^ he ^^ was a Spaniard. 8* The river ^^ Euphrates ^^ is in Asia.^'' I^^ must be myself. ^^ He^" desires to be a lawyer.^^ As your attorney, ''^ I should know it.'^ (ggg Dem. XX., d, note 1.) Did you hear of his^* becoming a convert?'^ I,^^ thy father-in-law ^'^ Jethro,'* am come unto thee.'^ Hei"" himself ^^^ was the wi-iter.io^ peace,!''^ pros- perity,i°* hapiDinesSji^s — all things ^"^ were sacrificed. Let himi"^ be Csesar.ios Beel- zebub,!"' than whom i^" none higher sat, with grave aspect ^ rose. I^^^ should be guilty of ingratitude,!!^ than which!!* nothing i!^ is more shameful. Thou!!^ hast been wiser all the while!!'' than me.!!^ The hack!!' having gone, we!^" were obliged to walk. Ah me ! !2i that I !22 the judge's i^s bride !2* might be.— WfiittierJ^^ O me,!26 why have they!^^ not buried me!^^ deep enough? — Termyson.^^^ Heart i-'" to heart !^! we !^^ '11 sail together, Hand !^^ in hand !2* for aye and ever. It !3^ fell a great distance, i^^ He ^^'^ came an hour !38 sooner. An hour !3' hence, we !*•> will start. The coal !*! measures twenty bushels.!*^ I !*3 do not care a straw.!** It !*5 was a little !*6 more than two miles.**^ He!*^ is wiser than me.!*' I ^^'^ wish they!^! were a whole Atlantic [ocean] ^^'^ broad. — Tennywn.^-'^ Let him!^* reign king.!^^ We ^^^ took it !■" to be them.!^* They i^' hated him!"' g^g being an enemy.!''! The tower i^^ is ninety feet !83 high. He !'^* is not a whit '^'^^ wiser. The hat ^'^^ is worth a dollar.is''' He !6S is worthy praise.!"' It !^' was a long time !''! ago. They i^^ watched all night ^''^ long. It !''* was Virgil,!''^ j^e i76 ^iho !'''' wrote the iEneid.!^* You !''' are too humane and considerate, things!^' which !^! few people i^^ can be charged with. Cortez,!^'^ he!** -who!*^ conquered Montezuma,!^^ -^^g^g ^ Spaniard. !*7 Mary!** gave the old man!*' ^ cup !'" of water,!'! a kindness !'2 which !'* he !'* seemed to appreciate. 1 1'^ saw the child !'^ growing up the pride !'^ of the family.!'* j i99 pitied the proud son ''^" returning a beggar.^'! There I ^'^ brought her ^'^ as pitying her 2'* hard usage. ^'^ j^ 206 is a little 2'^ more than two feet'^'* long. Well, John,^" I'^!' hope you^!! did not laugh. Gad, ^!^ a troop 2!^ shall overcome him. ''^i* O mighty Caesar !^!^ dost thou^!^ lie so low? The sun"!''' being risen, we ^i* pursued our journey.''^!' Honor ^^' being lost, all is lost. The mountains^-! rose, hight^^^ above hight.^"* The ship^^i came into harbor,225 colors ^-^ flying and guns ^"^ filing. Her 22* wheel ^^' at rest,^"^' the matron ^^! thriUs no more With treasm-ed tales ^*^ and legendary lore.'^^* They ^** chose her 2*^ queen. 2*^ He '^^'' made me 2** a coat.^^' They ^*' did not esteem themselves 211 men.'^*^ 1^*3 sent my fi-iend^** the book.2*5 We 2*6 forgive our 2*7 friends 2** their 2*' faults. ^^9 That law^^* preserves the earth ^^^ a sphere.^^s ^ noun ^^i is a name;^'^ as, man,^^^ eagle."^'' They "5* beat him^^' mercilessly, head^"' and 510 THE SYSTEM METHOD. body.261 1262 purchased the following articles; 263 namely, tea,364 sugar, 265 cofifee,266 and raisins.^fiT Tijig one is built a great deal^es better. A hundred years 369 ago, raih-oads ^^o were unknown. Two times ^^i two ^''^ are four.^'i'a jjg "^* talks of a return ""'^ home.2''6 Once six 2" is six.^^s (See d, note, p. 506.) It 2-9 is fully twice the amount.sso Shellcy['s],28i the poet's 282 writings 2S3 are appreciated by few. Liberty 28* was theirs 285 as men ['sj. 286 There is no harm in women's 287 studying politics.288 Henry's 289 being a student 290 was no reason 29i for his 292 being a savage.293 They 29* walk hand 295 in hand.296 He 297 destroyed, the rest 298 will yield. — Butler. -^"^ He^oo rushed down the hiU,30i iieels ^^ over head.^os Me 301 miserable! which way 305 shall I^oo fly, Infinite wrath ^o^ and infinite despair.308 — Milton.^^^ Dear uie l^io what 3" shall we ^12 do? Century ^^^ after century, 81* the earth ^is has wheeled in its ^le orbit.s" Liberty ! 8I8 freedom ! 3i9 tyranny 820 is dead ! The pulses 821 of thought 822 that 823 go to the borders 82* of the universe,825 let them 82* proceed from the bosom 827 of the household.828 — Emerson.^'^^ Sir,880 j 331 T^^g you 882 to give me 883 a bit 884 of bread. 835 Alfred 880 ['s], the gi-eat Saxon's 837 court 838 was at all times 839 open to learned men. 3*0 "\;v^hat8ii ^o you 3*2 mean, you 8*3 blockhead? 3i4 stop at Putnani's,3*5 the publisher 3*6 ['s]. The rifle 3*7 Is my 3*8 friend's,3*9 the hunter's.850 This did not prevent John's being inaugurated Duke 35i of Normandy.352 The cap- tain 353 of the i^-wZtoji's 35* wife 355 (lied yesterday. 356 (See Dem. V., Obs. c.) It 857 weighs little 358 more than three pouiids.359 My860 father, 36i than whom 362 no man 863 is wiser, approves that course.364 Henry 865 jg taller than James.866 True happincss,867 than which 868 nothing 869 is more coveted, is rarely enjoyed. They 370 are stronger than we.87i (This sentence is not whole, and are must be supplied; for, since an ad- jective governs not a nominative, but an objective, the nominative we cannot be the object of the adjective stronger.^ and is therefore the subject nominative of are under- stood, without which the sentence is not whole.) She 872 is worth more than them 873 aU. The debt 87* is greater than what 375 I cim pay. Less judgment 376 than wit 877 is more sail 878 than baUast.379 — Pemi's^^^ 3£aximfs.^^^ Entreat him 882 as a father, 883 and the younger men 884 as brethren.885 More men 380 than women 387 were there. ( Women is the object of the adjective more : Men more than women were there.) BRANCHING THE CLASSES. 511 DEMONSTRATION XXVIL FACT MATTER: a. Copula. — Any form of the verb Be (see Demonstration V., g, h) or any other connecting the same cases; as, She is queen; She reigns queen; He was elected president; They desire him to become a lawyer; To be a scholar requires toil and time; Henry's being a student was no reason for his being a savage; Among the stones I stood a stone. b. Principle I. — A name essential is in tlie nominative case. c. Principle II. — A name accessory to an infinite copula not preceded by an objective, is in the nominative case. d. Principle III. — A name appositive to another is in the same case. e. Principle IV. — A name separated by a copula from another name meaning the same thing, is in the same case. f. Principle V. — A name accessory to a preposition, adjective, or adverb, or to any verb not a copula, is in the objective case. g. Principle VI. — A name having an (') or a changed structure indicating possession, is in the possessive case. h. Principle VII. — Me excepted, a name independent is in the nominative case. i. The Independent, or Absolute, Case. — In the sentence, "I beg your pardon, madam," the word madam is independent in relation, or use; since, even when it is dropped, there remains a complete sentence whole, "I beg your pardon." Therefore, is not madam in the independent, or absolute, case? (for it is declared by Harvey, Clark, Greene, and others that "case is relation;" by Reed and Kellogg, that "case is [sometimes] use or office") — No; examine the sentence, "Ah me! that I the judge's bride might be." Me is here in the independent, or absolute, relation or use. If consistent with themselves, these authors must, therefore, declare me to be in this so-called independent, or absolute, case. If being in the independent relation puts a noun or pronoun in the independent case, then me, which is in the independent relation, must be in the independent case. But these authors are forced by numerous facts hostile to their doctrine of case, to declare me to be objective, not independent, or absolute. " Nom- inative /, possessive my, objective me," say they all. And thus do they contradict their own plain teaching, since they teach, — 1. That because madam is in the independent, or absolute, relation, it is in the independent, or absolute, case; 2. That because me is in the independent, or absolute, relation, it is not in the independent case, but that it is objective! It is the form, not the relation, or use, of me that constitutes it an objective case; and these authors acknowledge this plain fact by putting me 512 THE SYSTEM METHOD. in their tables of the pronoun forms, where they have, "Nominative I, possessive my, objective me;" and it is a direct contradiction to teach, — 1. That a noun or pronoun is objective because it is in the objective relation, or use; 3. That we is objective not because it is in the objective relation, or use, but because it is the objective form, (See " The Current Doctrine of Case," p. 388; also j below.) Note.— Ml grammarians agree that "case is v^property of uouns and pronouns." But a "property of a thing is that which is inherent in [as its form and posture] and naturally- essential to it." — Webster. "A peculiar quality; that which denotes something essentially conjoined to the essence of a thing." — Worcester. But the relation, or use, of a thing is neither "inherent in " it nor "joined to its essence." Therefore, since case is &property of nouns and pronouns, it must be something very different from their " relation, or use," as to other words. i. The Current Doctrine of Case. — See page 388. See also Dem- onstrations V. and XXV., g. j. Cases Dropped from English Nouns and Pronouns. — The pro- noun ^oho is here declined to show how many cases — the dative and the ablative — have been dropped from the English language since the four- teenth century. — Anglo-Saxon. Latin. Greek. English. Si7ig. Flu. (n.) Sing. Flu. Sing. Flu. Sing. Flu. Norn.: hwa, hwaet, qui, qui, Of, ol, who. who. Pass.: hwaes, hwaes, cujus, quorum, ov, div, whose. whose. J)at.: hwam, hwam, cui, quibus, w, olg. Obj.: hwone, hwaet, quem. quos. Of, ovc, whom. whom. Abl.: hwi, quo, quibus. From this table it will be seen that while the English has but three forms, the Anglo-Saxon had, for its relative, Jive. Nouns in English have but two forms, but they have three postures in the sentence whole. (See Demonstration XXV., g.) And because English nouns assume precisely the same postures as parts of the sentence whole as do English pronouns, they are necessarily branched into the same case divisions; for the case of a word is its form as arising out of its posture in the sentence whole. And observe that two nouns are not necessarily the same identical case-thing, or form- thing, simply because they are alike in form; for these two like forms may be, may arise out of, different postures in the sentence whole. It is in the same way that the two eyes are not the same identical form- thing simply because they are alike in form; they are ttco forms — two differ- ent things. This we know because they are differently postured in the man whole, the one being right, the other left. Observe that two different foot- prints or twins are not one and the same print or person simply because they may be precisely alike. Therefore, of the word madam referred to under h of this Demonstration, it is to be observed that, since there are in English but three cases, and since madam is neither possessive nor objective. BRANCHING THE CLASSES. 513 and since it is the naming, or nominating, form of the word, we decide it to be a nominative noun. These remarks will apply to all nouns in this so- called independent, or absolute, case. Prog. XXYII.— Branching by the Grammar-System Tree. PARTICULAJRS : To be an Englishman i is no easy matter.^ I^ am this aay* weak, though anointed Idng.^ Amos,^ the herdsman ^ of Tekoa,* was not a prophet's'' son.^" Ar- nold's ^^ success 1^ as teacher ^^ was remarkable. Being a scholar i* is not being an idler.i^ His ^^ original purpose ^'^ was lost sight i* of. He promised us ^^ each a pres- ent.^** To be a wise man ^^ is to be misunderstood. He was made fun "^ of. I care not for what ^3 others^* say. Labor "^ makes one^® merry. Te^''' shall be as gocls.^^ He spoke as one^' having authority. Every son 3" was regarded as a slave. ^^ When the quantity ^^ is taken twice as a factor,'^^ the product 3* is called the second power.^^ The warrior fell back upon the bed ^^ a lifeless corpse.^^ I had a suspicion ^* of the fellow's 3^ being a swindler.^o I *i wish they*^ were a whole Atlantic [ocean] *^ broad. He came as a teacher.** As mayor *5 of the city, I feel much aggrieved. Liberty *^ was theirs *^ as men.** She was named Penelope.*^ They did not esteem them- selves ^^ men.51 We forgive our ^3 friends ^^ their 5* debts.^* This law ^^ preserves the earth ^^ a sphere.^* It •''^ was to be his ^^ horse. •'^ James ^^ is said to be a good boy.^^ I did not hear of his ^* becoming a convert ^^ of that faith.*'' I pitied the proud son ®^ returning a beggar.** I saw the child ^^ growing up the pride ''" of the family.'''^ We,'''^ the people ''^ of the United States,'''* are lovers ''^ of republicanism.'''* Our whole company,''^ man'^* by man,'^^ ventured in. They tore him,*" limb*^ from limb.*^ He is crippled, hand *3 and foot.** He is ruined, mind *5 and heart.** What a word escaped thee,*''' thy ** lips.*^ The mountain rose, hight ^^ above hight.^^ Peace,^^ prosperity,^^ happiness, — all these things^* were sacrificed. The Lord^^ himself,^* heaven's "^ own high king,^* shall descend. Harry's ^' flesh, i*" it ^"^ fell away. Gad,^^^ a troop ^"^ shall overcome him. The orphan i"* of St. Louis,!''^ he became the adopted chil 1** of the Republic."'' They scaled Mt. Blanc,"*— a daring feat. "^ Knowye"" that the Lord,"^ he 1^^ is God.^^^ An hour"* hence we will start. He died a long time ^15 ago. Two times ^i* six"''' are twelve."* The son has double the wealth"' of his father.!"* Plato, ^^^ thou ^^^ reasonest weU. O mighty Caesar ! ^^^ dost thou i^* Me so low? WTiat! wiU these hands ^-^ jjeygj. ^,g clean? What i-* do you i"''' mean, you i-* blockhead ? 129 Peacej^^" ho! hear Antony,!^^ most noble Antony.i^s Tiie Pilgrim Fathers,!"^^ where are they? i** The pulses i*^ of thought that^^* go to the borders of the universe, let them^^^ proceed from the bosom of the household. He^^* that^'^ hath, to himi*" shall be given. He i*i destroyed, the rest wiU yield. — Nohle Butler .^^"^ They !** being absent, the cause cannot be decided. He rushed down the hill, heels i** over head.i*^ The ship i** came into harbor, colors !*'' flying and guns i** firing. The sun "9 being risen, we pursued our ^^^ journey.i^^ Ah me ! ^^s, that I the judge's bride ^^^ might be. — WMttier}^^ O me ! 1^5 what i^s profits it 1^7 to put an idle case? i^* -Tennyson.^^'^ Me 1*" miserable ! which way i*i shall I fly. Infinite wi'ath^''- and infinite despair.i*^ — Milton. 33 514 THE SYSTEM METHOD. Man is such as i"* his affection ^^^ and thought ^^^ are. There is as gi-eat a variety '^^'^ asi''^ can be found in any other country.i''9 Tell what^™ man is. Simonides,!''^ be- ing asked by Dionysius what ^^^ God was, desired a day's '^''^ time to consider of it i''* before he made his reply. Mr. Locke i''^ retiring to a window and writing, my^'''*' Lord !''■'' Anglesey I'^s desired to know what^™ he was writing. The wicked shall per- ish, as 1^0 the world 1^1 knows. She soured to whati^^^ she is. My fathcr,i83 than whom IS* no man is wiser, approves that course. That that that i^'' that i^^ that lady parsed is wrong, is plain. As many as ^^^ choose may remain. What man i^* enters, dies. The modern Avorld is theirs.i^^ Love is a silly infatuation, depend upon \\.y^^ Whom 1^1 the shoe fits, let him put it on. "In that and all things will we show our^^^ duty." — SJiakspmreP'^ "We doubt it"* nothing; i^^ heartily fareweU." — Id. That is a picture "" of my father's."'? She is a wife "» of my brother's."^ O, I see that nose of yours.-"** Some one""! has called the eye"''^ the window ^"^ of the soul. His- tory is the action and reaction of these two — nature ^"^ and thought."'^**^ ^ tropical memory makes ""^ man an ahnanac.^o^ The Duke""^ of Norfolk's^o^ pai-k^io jg fifteen miles ~" in circuit."!^ The lamp of a man's fife -^^ has three wicks,^!* — braiu,^!^ blood,^!^ and breath.-i^ ^ noble boy ! ^i* who ^" would not do the right? ^^o Faith, there has been much to do on both sides. It ^^^ is certainly as easy to be a scholar "^^ as a game- ster.''^'^^ Show yourselves true Romans.^^*— J^mZjms CmsarP'^ Bonaparte relied upon his own sense, and did not cai'e a bean^-'' for other people's. ^^'^ Hour"^^ for hour,^"' the risk on a steamboat is greater. Fare you ^'^^ well a thousand times ! "^^ a thousand times ^^' farewell ! Leave me here a little."^^ Sir,"^* March '^^'^ is wasted fifteen days.^^^ Our youths and wildness shall no whit 2^'' appear, But all be buried in his gi-avity. You spurned me such a day ; ^^^ another time,^^' You called me — dog.^**> Rome,^*i I have been thy soldier forty years: ^*^ Woe 213 to the hands that 2" shed this costly blood. Kneel thou^*'^ down Philip, 2*6 bvit arise more gi'cat; Arise Sir "*^ Richard ''*8 and Plantagenet. This announcement is made once 2*' for all. Once^^o one"''i is one. 2^3 They were refused the protection ^53 of the law. I was asked that question ^s* yesterday. ^^^ He will be taken care 2"" of. He collected as much as 2^7 jg due. Be of the same mind, one 25* toward another.259 All departments of life at the present day — ^trade,^^'' pol- itics,26i sciencej^GS or religion, 2''3 seem to feel and to labor to express the identity of their law. You are too humane and considerate, things ^^i few people can be charged with. — Pope.^^^ If thou 266 thouest him some thrice,26'' it will not be amiss. He 268 de- stroyed, the rest will yield. I shall not lag behind, nor err The way, thou260 leading.— Jllft7to?(..2™ On these and kindred thoughts intent, I lay In silence, musing by my comrade's side. He 2'i'i also silent. — Wordsworth, BRANCHINO THE CLASSES. 51 5 The man"^^ that^^^ stands by himself, the universe stands by him also. The boy,''^'^* how sweet to him is his fancy ! They think that I am somewhat.^'^ Cease to glide a sunbeam by the blasted pine, To sit a star-"' upon the sparlding spire. This armor was Sir Ralph's at Ascalon: A good knight he ! ^'^'^ — Tennyson. But it^'^* turns out, that what^^^ he has to say is of that weight as to withdraw some attention from the vehicle. — Emerson. You write very carelessly, a habit ^^^ you must correct. Thy spirit. Independence, ^^^ let me share, Lqj.(J282 q{ ^jjg lion-heart and eagle-eye: Thy steps I'll follow with my bosom bare, Nor heed the storm that howls along the sky. DEMONSTRATION XXVIIL In view of the fact that, in the new science system of grammar, the old rules of agreement between verb and subject have been embodied in a single principle, stated in two below; and out of respect to the old teaching, we give below a special program which may be introduced advantageously next after passing over the syntactical of names, which we have just finished. No rule, except so far as it is the statement of a principle, or law, man- ifest to the understanding, can, from the nature of things, be worthy of the slightest respect. There is no law of perfect grammatical agreement be- tween verb and essential name, or subject, as is frequently shown below in the items of fact matter; for, while it is true in general that the subject de- termines, or molds, the structural of the verb, we cannot assert that they must be identical as to grammatical number. Language is thought-tracks, nothing different. Not of necessity as things are, but as they are thought to be, is the language; for language is the track of the thought, not of the thing itself. Now the verb is one thought-print, the subject another, of the same thing; and we cannot more truly assert that the verb must grammatically agree with the essential name, or subject, than we can assert that one bird- track must exactly agree with another. One track is not made by the other used as a pattern; the bird makes both. And precisely, the verb is not made by the subject used as a pattern, but the thought makes both verb and subject; and in either case, the tracks may differ somewhat. The es- sential truths here are these: — ' 516 TEE SYSTEM METHOD. a. More than one subject-unit requires a plural verb. b. A single subject-unit requires a singular verb. It will be seen that these are items of mere fact matter, just as the fact that me is objective is an item of mere fact matter. We are, therefore, not to ask why more than one subject-unit requires a plural verb, just as we are not to ask why me is objective, or why man has only two eyes. It is the fact overt and ultimate, and is not to be stated as a rule or principle. (For person and number in the verb, see Demonstration XXIV., c.) FACT MATTER: a. Principle 8, a. — More than one subject-unit requires a plural verb; as, James [one] and Henry [another] are studious; Two [2] and three [3] are equal to five; Here the boys [two or more], and especially [was one] Charles, were very much interested in the tools; The good [more than one] are great; The committee [one and others] were unable to agree; The pub- lic [individuals, companies] are invited to attend. b. Principle 8, b. — A single subject-unit requires a singular verb; as, A hue and cry [one pursuit] was raised; Bread and milk [one food] is good food (better, "hue-and-cry," "bread-and-milk "); Each book and paper [each one] is kept in its place; Every man, woman, and child [every one'\ was last; No book or paper [no one\ is out of its place; Thine [thy one] is the kingdom and the power and the glory; John [one], and James [one is] also, is here; John [one], and James [one is] too, is here; John [one], but not [is] James, is here; John [one], as well as [is] James, is here; Pompey [one], as well as [was] Caesar, was a great general; Virtue [one], and not [do] riches, constitutes the hapjsiuess of a nation; John [one], or James [one was], was here; Neither thou [art] nor I am to blame; I [am] or thou art to blame; Something attempted, something \one thing] done, has earned a night's repose; The scholar and the poet [one man] was also the Christian and the patriot; The wheel and the axle [one machine] was out of repair; The wages of sin [one consequence] is death; The public [one whole com- munity] is invited to attend; The poetry and eloquence of the Augustan age [one subject] was assiduously studied. C. The following plurals represent sometimes more than one, some- times a single subject-unit; hence they are written sometimes with a plural, and sometimes with a singular verb: species, bellows, pains, means, amends, news, alms, riches, odds, wages, series, suds, measles, hysterics, rickets, mumps, tidings. d. These plurals, when referring to the particulars of which the science treats, or to the parts embraced in the scientific treatise, represent more than one subject-unit, and must, therefore, be written with a plural verb; but when denoting the science itself alone, but one subject-unit is denoted, which requires the singular verb : mathematics, magnetics, mechanics, mne monies, metaphysics, phonics, phonetics, physics, pneumatics, poetics, pol- itics, statistics, analytics, statics, ethics, dioptrics, catoptrics, optics, aero- BRANCHING THE CLASSES. Sir nautics, economics, hydrostatics, hydraulics, gnomonics, hermeneutics, hydro-dynamics, aesthetics, etc. Ethics, with atheism, are impossible. — Disraeli. e. The following plurals represent more than one subject-unit, and are, therefore, always written with a plural verb ; tongs, shears, scissors, pincers, tweezers, nippers, snuffers, trousers, pantaloons, cattle, thanks, billiards, oats, vitals, entrails, victuals, nuptials, orgies, ashes, sweepings, trappings, assets, calends, nones, ides, eaves, fire-works, withers, obsequies, annals. The following, though having synonyms in the singular, represent al- ways more than one subject-unit ; hence they require plural verbs : — .__ Spectacles = Greens = Colors = Matins = Annals = Bellows = Drawers = Arms =: Checkers = Species = Heaves = Hives = Hangings = Hypochondria = Vespers = Compasses = Dominoes = Lees == Remains = Manners == Letters = Morals = Goods = Drops = Reins = Gallowses = Goggles = Institutes = instrument, food, ensign, worship, history, air jDropeller, garment, war, game, kind, disease, disease, • drapery, disease, evening service, instrument, game, sediment, corpse, behavior, literature, decorum, furniture, medicine, back, support, instrument. sights. plants. paints. mornings. daily prayers. cries. boxes. limbs. pieces. coins. movements. bee boxes. suspensions. parts of abdomen. evening stars. indicators. hoods. sides. relics. modes. printed characters. lessons. benefits. globules. straps. hanging instruments. motions of the eye. institutions. = book, f. The singulars given below, with others like them, represent some- times a single, at other times more than one, subject-unit. The verbs written with them are, therefore, sometimes singular, at other times plural, to correspond to the number of subject-units : — Youth = stage of life, young persons. Sail = canvas, collection of ships. 518 THE SYSTEM METHOB. Horse = animal, horse-soldiers. Foot = limb, foot-soldiers. StaflE = support, corps of officers. Fry = thing fried, swarm of fishes. Head = part of body, several individuals. Shot = act of shooting, several balls. Brace = support, two individuals. Yoke = neck-frame, two individuals. Span = bridge, two individuals. Flight = act of flying, flock of birds. People = tribe, human beings. g, Equitocal Nouns. — The following nouns are common, or equivo- cal, in number, and are to be identified as singular or plural by some other structure in the sentence with which they agree : sheep, deer, corps, swine, vermin, hose, fry, gross, grouse, neat, public, Esquimaux, Japanese, Cyclops. h. Contrary to analogy, such names as the following, when used as collective nouns, and limited by cardinal-numeral adjectives denoting more than one, are written in the singular form : head, score, dozen, gross, sail, yoke, ton, pair, couple, brace. But when in the singular form and pre- ceded by either a cardinal-numeral or the article, and not separated by of from the succeeding noun to which it is accessory, dozen, of the words above mentioned, as well as hundred, thousand, etc., are numeral adjec- tives, not nouns; as, " two dozen eggs ; " "a dozen men;" "Five hundred were slain ; " " Ten thousand men were taken prisoners." In " Ten thonsand men, women, and children," etc., thonsand is still a numeral adjective belong- ing to persons understood, with which men, women, and children are each in distributive apposition. Prog. XXVIII. — Branching by the Grrainmar- System Tree. PARTICULARS : This is 1 flour; the rest is ^ bran. Give^ me your hand. Were* he my son, he would ^ not shed ^ a tear. The ornament of a house is ' the friends who frequent * it. The public is ^ more disposed'" to censure 11 than to praise. ^^ How are^^ the mighty fallen ! 1* If any one call, ^^ tell'^ him I am" not at home. The public are '^ respectfully invited i^ to attend.^ Who dainties love 2' shall ^'-^ beggars prove.^^ Had ^* ye believed ^^ Moses, ye would "^^ have ^'' believed ^^ Me. He is ^^ a fool, as all the world knows. ^^ Having ^i now finished ^^ the work assigned ^^ me, I retire^ from the great theater of action. The multitude pursue ^^ pleasure as their chief good. Three times two are ^^ six. A hue and cry was ^^ raised. ^^ Two and three are ^* equal to five. Bread and milk is ^^ good food. To learn" is *^ a duty. To have ** succeeded ** in an undertaking cheers ^^ the heart. I am ^^ to go *^ into the country. Did ^^ I forget *^ to mention ^o that fact? Each book and paper is ^^ kept ^^ in its place. He was^^ obliged^* to return. ^^ Every man, woman, and child was^* lost.^'' Here the boys, and especially Charles, were^s very much interested ^^ jq the tools. She stoops ^^ to conquer. ^^ Schools are ^ established ^ to educate ^* the ris- BRANGHING THE CLASSES. 519 ing ^ generation. The committee were ^ unable to agree. ^^ He saw ^^ his comrade fall. ®' Soon peace shall '" come " with all her smiling ^^ train. The public are '^ in- vited "* to attend. '^ John, and James too, is '* here. They met on the day ap- pointed. " Truth, crushed '^ to earth, shaU " rise ^ again. Cease *^ from anger, and forsake '^ wrath. Pompey, as well as Caesar, was *^ a great general. The Son of Man came ^^ eating ^ and drinking. ^ May ^^ your shadow never be ^ less. Neither thou nor I am^' to blame. ^ Something attempted, ^^ something done, *^ bas^^ earned 8* a night's repose. The scholar and the poet was** also the Christian and a patriot. The wheel and axle was *® out of repair. The wages of sin is*' death. The reading *^ of the report occupied *' an hour. The majority were'™ disposed ^"^ to adopt ^"^ the measure which they at first opposed, i"* The public is '"* cordially Invited i"* to attend. ^"^ The poetry and eloquence of the Augustan age was ^'^^ assid- uously studied. ^"^ Each of the errors is ^^ found ^^^ at the beginning, m Several years' interest is ^^^ due at present. Each man, each woman, each child, has ^^^ a duty to discharge. 1^* Either I or they are ^^^ to be ^^^ promoted. ^^^ The world are ^^^ spectators of your conduct. The world is ^^^ round. Man, proud man, Drest^^ in a httle brief authority, Plays ^^' such fantastic tricks before high Heaven As makes ^'■^ the angels weep. ^'^^ — Shakspeare. DEMONSTRATION XXIX. FACT MATTER : a. Constructions of the Adjective. — The adjective is always of the accessory construction; it is never essential. Of the accessory construction of adjectives, there are four varieties, — adnominal, appositive, j^redicate, factitive. b. Adnominal (^a£?, "to," and nominal, "name": to a name). — The office of an adjective prepositive, and not disjoined by a comma; as. The beautiful flower was frozen; He is the very man; This young man was a brave soldier. C. Appositive {=ad, "to," and position: close position). — The office of an adjective postpositive, or prepositive and disjoined by a comma or other mark; as. Goldsmith wrote nothing so forceful and sweet; Who else is it? Young, handsome, and clever, the page was the darling of the house; A milk- white hind, immortal and unchanged, fed on the lawns; Fond of splendor, he was indifferent to neatness. Note. — An infinite verb, when used hke a noun, is often limited by adjectives and adjectival possessives. Thus, in "I was opposed to JoM's wi-iting the letter," and "Virtue consists in being good,^^ the substantive participles writing and being are modified by the possessive Johri's and the appositive adjective good respectively. Ob- 520 THE SYSTEM METHOD. serve that adjectives and possessives do not modify verbs as verbs, but that it is be- cause the verb is itself used like a noun that it may be modified by the adjective, or the adjectival possessive. Were this principle better known, that adjectives (noun-modifiers) may also mod- ify whatever is used like a noun, it would beautifully solve another problem of gram- mar hitherto xmexplained; namely, in the sentence, "To be good is to be great," what do the adjectives good and great modify? Once established that, in the sentence above, good may modify the substantive verb being, — and it can modify nothing else, — it foUows, in this sentence, that good and great may modify the substantive verbs he, which clearly explains the difliculty what to do with good and great. They are appositive adjectives belonging to the two infinitives be. This principle is confirmed not only by numerous examples in the English, but by the analogy of both the Greek and the German. In the following examples, the words are given in their Enghsh order, that the examples may be the better understood: — Greek : to afxagTapeLV hoTL ovdlv -QavfiaaTov ap-QgunovQ bvrag. The err is no wonder, men being. The meaning is, "The error is no wonder, they being men;" but observe that instead of the noun error, limited by the adjective the, the Greeks make use of the substantive verb, the infinitive, err, d/j.aQTdpEi.v, which is modified by the adjective the, to. The testimony here, therefore, is that because the infinitive err is used like a noun, it may be modified by the adjective. German: ©aS "Sitxim tft eine augeuel^me 2[3eiDeguug. The ride is an agreeable exercise. ©a§ Seffit enniibet bie Slugeit The read fatigues the eyes. Here the infinitive verbs ride and read, because used as nouns, being the subjects of is a.nd fatigues respectively, are each modified by the adjective the. The testimony is again that a verb, when used as a noun, may sometimes be modified by an adjective. If it be inquired whether we may not explain good and g7-eat by assuming an el- lipsis, the writer answers. No. The reasons are, briefiy told, as foUows : — 1. It has been shown (Dem. XXX., m, 1, note 3), (1.) That everywhere in science, we are by law bound to explain things as they are, since to explain them as they are not is not to explain them; (2.) That by law things explained by science — whether sentences or words or etymons or animals or solutions or plants or molecules or else — are in wholes; so that the only way to justify our assumption that some part is miss- ing, and therefore to be supplied, is to prove that such part is missing, that the sen- tence is not a sentence whole. But now mark: when it is once proven that an adjec- tive may modify averb,when that verb is used like a noun, it becomes impossible to prove that "To be good is to be great" is not a sentence whole! for the only ground for supposing it not whole, is the supposition that the parts modified oy good and great are missing. But that supposition is groundless; for I have shown that they may modify the substantive infinitives be. By such examples as, "Virtue consists in beino- good," and by the analogy of the Greek and the German it has been shown that an adjective may modify a verb used like a noun. Since, therefore, good and great may be explained as modifying the substantive verbs be, the sentence is proven to be whole, and no part can be missing. The assumption of an eUipsis, that is, of BRAKGHma THE GLASSES. 621 a missing part, is therefore as impossible and as ridiculous as the assumption that to a man whole having two eyes, two thumbs, and eight fingers, should be supplied an additional eye, thumb, or finger ! Such assumption is against law, therefore lawless, laughable, lunatic. 2. We should be wary about supjjlying; for it is by assuming that privilege that we curse the science of grammar with interminable and distracting disputes. Only by withholding from grammarians this arrogated privilege and outrageous practice of assuming that some part is missing, u7iUl he may prove it., may we hope to escape this distraction. 3. To supply a part assumed to be missing, is not to solve, but to evade the diffi- culty, as is shown in Demonstration XV. ; for equivalency in meaning is not necessa- rily sameness of granmaatical construction. d. Predicate. — The office of an adjective disjoined, by a copula, from the substantive to whicli it refers; as, Fond of splendor, he loas iudifferent to neatness; We found him to he true; The book is growing dull; He was able and ^tadions. e. Factitive. — A postpositive adjective made accessory to a direct ob- ject, through a transitive verb; as. He painted the box blue; He tallied him- self \\okk^; She boiled the e£^(/s hard; The jury found [declared] him guilty; "Royal Glue" mends everything solid. f. CoNSTRXJCTiOKS OP THE Adverb. — Of the adverb, there are two con- structions, — the modifying and the conjunctive. ff. Modifying Adverb. — One not performing the office of a connec- tive ; as. The soldiers marched forward; She came forth; The miners toiled eaily and late, fared paorly, lodged rudely, and, finally, received very little gold. h. Conjunctive Adverb. — One that performs the office of a connective in addition to that of an adverb; as, The rocks remain where they fell; The wind bloweth AvUere it listeth; The time has come Mhca we should be going; It was in the early spring that he came; Friendships are best when they are old; We shall reap as we sow. i. Principle XH. — Any word not a name or a verb, added to a name, is an adjective. j. Principle XHI. — Any word not a name or a verb, added to a verb, an ad- jective, or an adverb, is an adverb. Prog. XXIX. — Branching by the Grammar- System Tree. FARTICULABS : The canary's song is sweet, ^ clear, ^ and musical. ^ Were the* cherries ripe?^ How^ often'' have you been there?* He studies sometimes^ very'" well. '^ My pencil is long ^* and round. ^^ The ^* English '^ are positive, ^^ methodical, " cleanly,^® and formal. ^^ The 2" rose is the fairest ^^ of flowers. They live very^^ happily. 2^, Come often, 2* and stay long. ^^ It is full enough. ^^ The^^ book is where 2* you laid it. I have forgotten my part, and I am out.^* Many ^'^ of these ^^ books are worthless. ^^ Which ^* book do you desire ? Go to the ^* first 35 tree be- yond. The price of a virtuous woman is far ^6 above rubies. It is pretty ^^ nearly ^^ 522 ^'^^' SYSTEM METUOl). finishea. Without the ^9 other, *» each « is useless. ^'^ Quiet, ■'^ sensitive, ** devout, *^ he M'as predisposed to melancholy. The« English" hate the French as frivolous. ^^ None*" is so deaf 5" as he that will not hear. He came unto his own,^! and his own 52 received him not. The lunatic paces to^^ and fro 5* continually. 55 The^^^ louder 5^ the thief talked of his honor, the^s faster 59 we counted our spoons. Lu- ther died where "^^ he was born. He was made fun of. ^^ Young, e^ handsome, ^^ and clever, «* the page was the darling of the house. He is precisely ^^ what he seems to he. Whither «« I go, ye cannot come. I am as^^ tall^^ as you. To be right fi9 Is better™ than to be president. The boat started long" before noon. I shall see you then,'^ when'^ you come. We found the report to be true. '* Man ought to go upright ''5 and vital. '^ Who else" is it? The book is growing dull. '^ She boiled the eggs hard.™ They have broken up^" housekeeping. What^i a^^ piece of work is man ! She robbed me of my hopes, my heart, my all. ^^ I had as 8* lief ^5 go as stay. Please sing for us just^^ this^'? once. He must needs ^^ fail at the outset. The sun is up. ^* Go to ; 90 homo is a common name to all men. Wher- ever 9^ snow falls, there 92 is usually ^^ civil freedom. Then, «* who knew the force of those 35 dire 96 armsl At last, 97 the passengers forsook the burning vessel. Life is, at best, 98 very 99 short, i"" He has no")! ambition at all. 102 Above all,i03 te^ch your children to be useful, i"* These lectures have been published in full, i"^ The practice is, of late, ^'^^ uncommon. W7 The Lord is king, be the people never ws so^"^ impatient. "0 What"i by this and what "2 ijy that, he succeeded. My stay there"* was short."* Not "5 a drum was heard, not"" a funeral"^ note. To equal which, the tallest"** pine were but a wand. The thunder afar"^ roused up 120 the soldier. Ellie went home, sad 121 and slow. 122 Yet let noti23 one 12* heart-beat go astray. 125 Farthest 126 from him is best, i^? Herei28 we may reign secure. i29 You fine downi*" your distinction till there ^^^ is nothing left. If thou thouest him some 132 thrice. It will not^^^^ be amiss. ^^^ It is a thing to walk with. 1^5 i looked in 136 on iiim as^^^ I came from school. The country whence ^^s he came is desolate. ^"'■^ The year when"" Csesar was born is uncertain. '*! He painted the box blue. "2 Where "=* were you when"* I called? To be good "5 is to be great, i'" The son has double "'' the wealth of his father. On these and kindred thoughts intent, "* I lay In silence, musing by my comrade's side, He also silent. i*9 — Wordsworth. What'50 wretchedi5i creature of what'52 wretched ^53 kind, Thau man more weak, '^i calamitous, i55 and blind. ^56 — Homer. How weary, 15T stale, '58 flat,i59 and unprofitable, 1™ Seem to me all the uses of this world. — Shakspeare. ^.MSiQj2M^«',i^^^}/Ucnnr\^ BEANGIIING THE CLASSES. ' 523 DEMONSTRATION XXX. FACT MATTER: a. Principle XIV. — A connective governing an objective is a preposition. (Review Demonstration XIX.) b. Principle XV. — A connective between !il(e parts is a co-ordinate conjunc- tion. (See Demonstration XX.) C. Principle XVI. — A connective between modifier and word modified is a subordinate conjunction. d. The Conjunction and the Preposition Compared. — The follow- ing are the likeness and the differences of these two classes :— Likeness : Both are connectives between word and word, or word and phrase, or word and sentence, thus : — • Preposition : Man is bsrn to die ; He ventured to within danger , It is jnst, m that it is right. Conjunction : James is loved and respected ; Be so good as to go ; I would go rather than stay ; He will come if it is possible ; He is taller than I am. Unlikenesses: Prepositions govern an objective ; conjunctions govern noth- ing. The conjunction is built in between two parts, neither of which it governs ; the preposition is built in between two parts, one of which it forces, if it be a noun or pronoun, to take the objective case. The co-ordinate conjunction is always built in the sentence whole be- tween parts like in some respect or respects, and neither of which mod- ifies or governs the other ; the subordinate is always built in the sen- tence whole between parts unlike in some respect or respects, one of which always governs or modifies the other. Prepositions are built in between unlike parts, one of which must always be an objective and be governed, not by the other part to which it is connected by the preposition, but by the connective itself, the preposition. Thus, in "James is \oveA and respected," the co-ordinate and joins loves and respected, which are like in class, tense, mood, and con- struction, and neither of which modifies or governs the other. But in "I chose liim as honest," the subordinate as joins liim and lionest, which are U7blike in class, and of which the one, honest, modifies the other, him. So in " He is wiser thani I am," " Solomon, than2 whom there never was a wiser man, says," "Be so good as i to go," " I would go rather tMnsStay," "I am doabtfnl as2 to liim," and all such knotty sentences containing the so-called "difficult" subordinates as and than, than,i than,2 as,i than,s as,2 join respectively wiser and I am, wiser and whom, so and to go, rather and stay, doubtful and to him, the two parts in each pair of which being unlike in some respect or respects ; and in each case the one part modifies or governs the other. 524 THE SYSTEM METHOD. A true understanding of the conjunction as contrasted to the preposi- tion, but more especially of the subordinate, as contrasted to the co-ordi- nate, conjunction, is the best part of such skill as is to be employed in the solution or explanation of the more difficult grammatical problems. e. Is To Before the Infinitive a Preposition ? — The origin of to is not unknown. It is the same word, which, in Anglo-Saxon, was placed before the gerundial infinitive ; as, god to etanne=good to eating. But aft- erward the to was placed before the simple form of the infinitive : god to eatan:=good to eat. Just how and why, about seven hundred years ago, the to changed from preceding the gerundial infinitive, and began to be used before the present simple infinitive, we do not know ; and it does not affect our purpose here. And how, though used in Old English only between sentence parts, by a long succession of steps, each answering a new but felt need of expression, the to came to be employed as a connective be- tween part and whole, we need not explain. We desire only to prove that to is a preposition. f. Is To A Part of the Present Tense ? — Says Goold Brown, "The present, or the verb in the present tense, is the part from which all the rest are formed. It is commonly considered the root, or simplest form, of the English verb." — Grammar of English Grammars, p. 361. "The present tense is the simplest form of the verb." — Thos. TV. Harvey. And Holbrook, the school-master, declares that the "present [tense] is a root." But the signification of the wood root, as set forth in all English dictiona- ries, is radix, radical, simple, single, primitive, underived, uncompounded. Take, then, the sentence, " To die is to sleejD." It must be evident to all that to, considered by itself and apart from sleep, is a radix ; that both to and sleep are radical, simple, single, primitive, underived, and uncom- pounded words ; that to of itself is a root or radix. It is just as truth- fully said, too, that sleep, considered by itself, apart from to, is a simple, underived, uncompounded root. But tense belongs to verbs, never to another class; hence, to be in the present tense, a word must be, (1.) a verb, (2.) a root. Any ordinary stu- dent will now see that to sleep, the parts both taken together, cannot be the present tense, the counter assertions of scores of grammarians to the con- trary notwithstanding; for to sleep is not a single, uncompounded root. To sleep is two roots, a compound, if taken both parts together; and it is but folly to suppose that to can be any sort of a verb. Let him who fancies that he perceives some verbal force in to, determine its principal parts (see Demonstration VII., e); he will discover only these: I to now! I toe(Z yes- terday ! I have toed! But the same test, which is the one usually employed, shows sleep to be a real verb: I sleep now, I slept yesterday, I have slept. And since sleep taken by itself, considered apart from the to, is, (1.) a verb, (2.) a root, thus according exactly to the two requirements of the present tense, we may safely conclude that sleep is the present tense. Plainly, then, the to forms no part of the tense. BRANGHINO THE CLASSES. 525 g. Is To A Part of the Infinitive? — Those who oppose the doctrine that to before the infinitive is a preposition, teach that though the to is placed before the infinitive, we are to present the mood as belonging to the associated parts, to the to and the verb mutually, thus: — "To die i?, to sleep: To sleep is. a. verb, irregular, intransitive, [principal parts] sleep, slept, slept, active, infinitive [mood], present," eic—Holbrook, p. 178. "He attempted to ascend the mountain: To ascend is a verb, regular, transitive, active voice, common form, infinitive mood, present tense." — Harvey, p. 101. Thus, by the authority of these authors, both the to and the verb are included in the infinitive mood. I quote from these two only, for 1 have not others at hand; but these, and a score of others in use to some extent or other in the United States, very feebly betray the doctrine that to enters into the infinitive mood, on this wise: — "To before the infinitive is not a preposition, but is merely the sign of the infinitive." — Holhrook, p. 107. " The infinitive may usually be known by the sign to, placed before it." — Harvey, p. 74. For, if to is a part of (included in) the infinitive, surely it cannot be "before the infinitive"! or "placed before it"! Why, if to is included in the infinitive, and still comes "before the infinitive," as I have just quoted, then it comes before itself! But this, being impossible, may be said to be infinitely absurd. When a writer argues that he can get before himself, he is only crazily "beside himself." Thus the to cannot be included in the mood, by virtue of the fact that it is placed before the verb to which the mood belongs. h. Is To A Part op the Verb? — To is never joined to the verb by a hyphen, neither is there an instance where to and the verb are written to- gether. It is joined to three words only; namely, day, morrow, night; thus, to-day, to-morrow, to-night. But these are not verbs, but nouns. And the scientific method must needs deal with to as ichatit is, a single, simple, sep- arable, radical word. If it is a part of the verb, it will remain a part through all the modifications; since all changes in the verb to correspond to change of tense, mood, etc., are made at the right hand part — at the ending of the verb. These are called changes in the endings of the verb. What word, then, having to before it, will form the past tense? — Not " to slept," for these grate upon the ear and will not justify. The student will now see that to cannot be used before the past tense. And in the same manner, he may determine that to cannot be apart of the verb. i. The To Connects Odjective to Base, or Antecedent, Term. — We have seen (Demonstration XIX., b, 1, 2, 3) that the peculiarity of a preposition is that it connects unlike parts — an objective and an antecedent term. And the word to answers aptly to this definition. Examine the fol- lowing sentences: — 526 THE SYSTEM METHOD. 1. I'me to come is called future. 7. Young people in this country are 2. He went to Rome. •prone to melancholy. 3. He was ready to die. 8. I lioiied to see. 4. He came to me. 9. "We are 'born to die. 5. We all fell to eating. 10. He is about to go. 6. The fruit is ripe e/ioMf/Zi to -u.se. 11. ~B.q aciedL agreeably tohis instruction. 12. She y^as deaf to blessing. From these examples it may be clearly seen that to is the purest of prepositions; for it is used as a connective between a larger variety of un- like parts than any other preposition in the language. Thus, it is a link between — 1. Noun — verb, as in 1. time to come. v 2. Verb — noun, as in 2. «'c?i< to Rome. 3. Adjective — verb, as in 3. ready to die. Al. Verb — pronoun, as in 4. cnme to me. 5. Indicative — participial, as in 5. fell to eating. 6. Adverb — verb, as in 6. enougli to use. 7. Adjective — noun, as in 7. 'prone to melancJioly. 8. Past — present, as in 8. hoped to see. 9. Particijjial — infinitive, as in 9. born to die. 10. Preposition — verb, as in 10. about to go. It. Adverb — noun, as in 11. agreeably \o instruction. 12. Adjective — particijile, as in 12. deaf io blessing. It is the very habit of to to connect parts that differ in character or cir- cumstance, and that is sufficient to prove it to be a preposition. In these examples it would be absurd to sujopose it to be a conjunction, since con- junctions connect not such unlike parts. But all agree that to is not a noun, pronoun, verb, adjective, adverb, conjunction, or interjection. It is therefore a preposition; for what has the varied uses, the peculiar character, and the precise habits, of a preposition, and belongs not to another class, must be a preposition. j. To Before the Infinitive is Identical with the Preposition To. — Scrutinize these sentences: He will move your hearts io pity. — Shakspeare; It must be a bitter experience to be more accustomed to hate than to love; Each wooed the other to love. One thing will not be disputed: the value, or office, of love is exactly eq-ual to itself, call love verb or noun, as you please. Then to before the verb is identical with to before the noun. But all agree that to before a noun is a preposition. Therefore, it can be nothing else before the verb. k. We have now demonstrated that to forms no part of either the tense, mood, or verb. We have given proof that it is a separable word, which proof we were not bound to give; because all eyes behold it, and all tongues speak it, as a separable word. We have seen that it belongs not to the nouns, pronouns, adjectives, verbs, adverbs, conjunctions, or interjec- BRANCHING THE CLASSES. 527 tions. We have shown that to performs the peculiar office of a preposition in connecting unlilie parts, — accessory to base,- — and that it accords in all particulars to the definition of a preposition. We have educed examples showing that it cannot be distinguished from to, the preposition — nay, showing that it is identical with it. And, finally, we have exhibited con- traries and a contradiction in the teaching of those authors that deny to to a prepositional value and character. We can, therefore, safely conclude that to before the infinitive, or a verb in the infinitive mood, is a prep- osition. 1. Change of Names — To as an Introductory Preposition. — Because prepositions must be defined as connectives, or signs of relation between objects, the occasional use of to without an antecedent term has given rise to a storm of doctrines. Many imagine that because the circumstances have now been changed so that to can be used without an antecedent term, as in " To steal is base," the name, preposition, ought also to be changed. But there is aicell-cstahlishecl rule for change of names, which ought to be understood: When the character of an object changes, the old name is re- tained, unless, in changing, the object assumes the character of another totally diiferent, when the name also is changed. Thus, originally, a tum- bler was a drinking vessel with a convex base, causing it to tumble over on its side; and though the character of the object has now changed so that it is made with a flat base, and used for every sort of drinking, still we retain the name "tumbler," because the object has not entirely lost the original use or assumed the character of another object totally difCerent. "Mounte- bank " was originally one who mounted a bank or bench in a public place, vending medicines and boasting of their wonderful power to cure; and though the character of the object is changed so that a mountebank is de- fined, oftener, to mean a pretender or a quack, the term "mountebank " is still retained, and may be used, without the idea of a bank or bench, to ap- ply simply to a pretender; because, in assuming the character of a pretender the mountebank has not changed to something totally different from itself. And precisely so with to as an introductory preposition in such expres- sions as, "To sfeaZis base: " although the prepositional character is some- what changed, so that to is used without an antecedent term, still the name preposition is properly retained; because tlie to has assumed the character of nothing totally different from a preposition. It has been shown that it can- not be assigned to any other part of speech; and, until it assumes the char- acter of another, it will be most fitly called a preposition. But let the student observe that conjunctions must be defined, also, as connectives. Still, some are used to introduce independent statements; as, "And they said there is no hoj^e." — Bible. In the same way, prepositions, though necessarily defined as connectives, include a few words that may be used to introduce independent statements; as, To steal is base. But let it not be supposed from what has been said, that to is not strictly a connective. True, this to is not a connective between parts: it is between part and whole. 528 THE SYSTEM METHOD. that to is a connective — precisely as the subordinate conjunction that is a connective, not between parts, but between part and whole. (See Demon- stration XX., Obs. f.) The to joins the part steal, not exactly to, but rather in, the sentence whole, "To steal is base." Now mark that if the to be missing (and the sentence be written, " • ■ steal is base "), it is missing not only as a part of the sentence whole, but also as a part of the subject phrase, to steal; precisely as, if the eye or ear is missing, it is not only missing as a part of the man whole, but also as a part of the head. And mark that when you supply to to the imperfect sentence, " ■ steal is base, you supply it both as a part of the subject, to steal, and as a part of tlie whole sentence. To steal is base, thus joining such subject part, not exactly to, but rather in, such whole. It is thus shown how to is strictly a convective. This to parsed would be: To=preposition, showing the relation between its object (the part steal) and the sentence whole. m. The discovery of the true nature of the subordinate conjunction — that it joins modifier or governed to modified or governing — not only ex- plains the difficult as, but it is also the true solution to the difl^culties of than which, than whom, and than me, than which nothing, perhaps, has more perplexed the grammarians. Of sentence wholes in which these ob- jectives, ivhom, which, me, etc., occur, the following are examples : — "Whom does he honor more than me?" "Beelzebub, than ivhom none higher sat, with grave aspect rose." "I should seem guilty of ingratitude, than which nothing is more shame- ful." " Thou hast been wiser all the while than me." That these sentences are not bad English will be perceived by any stu- dent who has read good English authors. Of than whom and than which, Goold Brown, than whom no English writer, perhaps, has read more ex- tensively, says they " may be quoted from many of our distinguished writ- ers." — Grammar of English Grammars, page 676. So also A. H. Welsh : '■Than whom is an instance of grammatical vice, which, from being long endured, is now, from its long continuance, likely to be embraced." — Essen- tials of English, page 208. Thus, though this objective after than " has not been satisfactorily explained" to be in accordance with the system of grammar, yet it is " embraced" because of its " long continuance" and al- most universal adoption among "distinguished writers" of English. What is so nearly universally adopted by the " distinguished writers" can hardly be spurious grammar; for it is that very "long" continued use among " many distinguished writers " by virtue of which alone any speech is or becomes good English. Such sentences, including these objectives, whom, which, 7ne, etc., are therefore to be explained as they stand, unless it be impossible so to explain them — unless it may be proven that such than whom, than me, etc., are out-systemed from, or not in harmony with the prin- ciples of, the science system of grammar. BRANCHING THE G LASSES. 529 Let it now be shown, (1.) That adjectives and adverbs may sometimes govern the objective case ; (2.) That the subordinate conjunction connects word governing and part governed. It will therein be seen that, in all these sentences, the objective is governed by the comparative adjective or the comparative adverb, as the case may be, to which the objective is joined by than; and that i/iaii is a pure subordinate conjunction, easily explained and easily understood. Further, therein it will be made plain that, so far from being " an instance of grammatical vice/' than whom is, goodi, system- fitting, forcible English. 1. Adjectives and Adverbs Govern the Objective. — Not verbs and prepositions alone, but adjectives and adverbs, often govern the objective case. (See Demonstration XXVI., b. 6, b, c ; also Demonstration XVIII., Obs. j.) Indeed, it is part of the system of universal grammar that ad- jectives and adverbs, especially comparatives, may govern the objective. In the following examples, each word is given its English order, that the reader, even though unfamiliar with Latin, Greek, and German, may know what word is the adjective or adverb governing, and what the objective governed. The objective is also distinguished by a superior {^), the ad- jective or adverb by a superior (i) : — Latin: Perducit duas fossas quindecim pedes ^ latas.^ He constructs two trenches thirteen feet 2 wide.* Note 1. — The objective /eei! is the ohject of the adjective wide, precisely as in Latin, the objective pedes is the object of the adjective latas. Latin: Ipse collocat sues proprior^ montem.^ He himself collects his own nearer i the mountain.^ Note 2. — The adverb nearer governs mountain in the same way in which the Latin ad- verb proprior governs montem ; and observe that either mountain or montem is a noun in the objective case. Mountain is object of nearer just as years is object of the adverb later in the sentence, "He died many 2/«fflrs ^a^er." Mark that "he died years," is absurd. The sense is, "He died later,"" and years tells how much later; so that years is the object of the adverb later, and not of the verb died. Greek: 'Eirrt a^coc^ kiralvov.^ He is worthy i praise.^ Note 3.— Since the objective knaivov is the object of the adjective a^ioq, the English objective praise must be the objective of the adjective worthy ; for the translation is strictly literal. That worthy is an adjective, all agree. That praise is objective, is certain ; for it neither refers to the same thing as the nominative he, nor has the possessive sign. (See Demonstration V., h. i.) Of what, then, may this objective, praise, be the object?— Of the adjective worthy, evidently. For, since it can be the object of neither he nor is, if it is not the object of the adjective worthy, it is the object of nothing 1 But whatever is the object of nothing is not an object! just as whatever is the backbone of nothing is not a backbone. To suppose praise Ta-a.y be held, in the sentence, in the objective "without a governing [a holding] word," as does T. W. Harvey, is like supposing Mohammed's coffin may be held half way between earth and heaven without being heldi But may we not assume that the sentence is not whole, and, accordingly, supply the word of to govern praise, without raising the question whether praise is governed by, is the object of, the adjective ivorthyf—T^o, indeed; for if praise is the object of worthy, the sentence is whole as it stands 1 So that, by not raising that question, we might commit the 34 530 THE SYSTEM METHOD. ridiculous TDlunder of calling what is a whole, a part only ! of calling what is a whole man, a man with an ear ofE 1 If it were understood among authors that every real science ex- plains—is by law bound to explain— things as they are ; namely, in parts and wholes; that is, that botany explains plant part in tlie light of plant whole, that zoology explains animal part, or organ, in light of animal whole, that orthography explains word part, or letter, in light of word whole, that grammar explains sentence part, or word, in light of sentence whole, etc., etc., — if this elementary truth, that learning consists in mirroring whole in part and part in whole, were known among teachers and authors, we should then see that it is the same sort of nonsense to assume that something is to be supplied to a sentence before it is a sentence explainable, as it is to assume that something is to be supplied to an animal or a plant before it is animal or plant explainable 1 that is, it is supposing something is to be sup- plied to an animal before the animal is an animal I Think of a teacher of zoology, who, without iirst proving that a rabbit is not a rabbit whole, should assume that a cat-like tail should be supplied, because that part is to be found m other animal wholes ! or think of a teacher of orthography, who, without first proving that a word — the word " sure," for instance — is not a word whole, should assume that an h should be supplied (thus making up the spelling " shure " !), because that ^-part is to be found in other word wholes I What sort of a grammarian, then, is he, who, without first proving that a sentence (the sentence, "He is worthy praise," for instance) is not a sentence whole, should assume that 0/ should be supplied (!) because that o/-part is to be found in other sentence wholes 1 Give heed : all things can be explained only as they are ; for any explaining of things as they are not, is not explaining. But by law there is no thing without parts, and parts taken together form a whole ; therefore by law we are compelled to assume all things to be ex- plained, including sentences and all else, to be wholes, until we may have proven them to be not whole: so that when we find an animal, a plant, a word, a sentence, to be explained, we are bound by law to assume, not that something is to be supplied (!) but that we have found an animal whole, a plant whole, a word whole, or a sentence whole, as the case may be. It is the law that things are part-and-whole things; so that unless it is first proven that a sen- tence is not a sentence whole, it cannot be assumed that something is to be supplied. Greek: "E/tsve evrbg^ tuu b^uv.^ He remained inside ^ the boundaries.^ Jfote 4.— Since the objective oguv is the object of the adverb evtoq, it follows that bound- aries may be the object of the adverb inside; for the translation is literal. If, indeed, it be granted— what an objector might contend for— that inside is a preposition rather than an ad- verb, that fact, if such it may be called, in no way stands opposed to the fact that the Greek adverb evtoq governs the objective noun bguv. It therefore militates not against the truth that adverbs sometimes govern an objective. Oerman: ©ie SOf^auet ift jicanjig^ gu| lang.^ is twenty^ feet long.^ TDar iedjjig^ 3af)te aW.i was sixty ^ years old.^ Note 5.— These sentences, in which feet and years are the objects of the adjectives long and old, are of themselves a demonstration that, in either German or English, nouns in the objective may be governed by adjectives. English: I wish they were a whole Atlantic^ [ocean] broad. i — Tennyson. He is not a whit^ wiser. ^ — Huxley. It is not a pound ^ heavier. ^ Note 6.— These sentences are illustrations of nouns in the objective case governed by adjectives. And the following are instances in English of adverbs governing an object :— He came an hour 2 sooner. 1 This house is built a great deal^ better. 1 He died several years ^ ago.' We shall start an hour ^ hence.^ ©ie SOfiauet The wall ©ie grau The woman BRANCHING TEE CLASSES. 531 From such proofs it may be seen that not in Latin, Greek, and German alone, but also in modern English, an objective noun may be sometimes governed by, be the object of either an adjective or an adverb ; so also the objective in Anglo-Saxon, the syntax of which closely follows the syntax of the Latin. In either, adjectives denoting want, plenty, nearness, compar- ison, pleasure, pain, trust, distrust, etc., as well as adverbs of quantity and place, take after them a noun which they govern in the objective. Note 7. — If the reader suppose ago in the sentence, "He died several years ago," to be an adjective modifying 2/ears, instead of being an adverb belonging to died, he may see his error by pondering the sentence, " He died long ago ; " wherein he will be forced to consider ago a pure adverb belonging to died. But to see ago as an adverb here is to know ago as an adverb there also — an adverb governing the noun years in the objective. Mark that except as years is the object of the adverb ago, it is objective without a cause ! 3. The Subokdinate Conjunction Connects Word Governing and Part Governed. — Though in a way seeing it to be the law of universal grammar that adjectives and adverbs, especially comparatives, may govern the objective case, nevertheless grammarians have hever discovered how to explain the expressions than ivhom, than luhich, than me, etc., after compar- atives. "A relative after than is put in the objective case; as, 'Satan, than whom none higher sat.' This anomaly has not been satisfactorily ex- plained." — Dr. Bullion^ s Analytical and Practical Orammar. Now, the reason this than whom and than which has not been satisfac- torily explained is this : the grammarians have not understood the true nature of the subordinate conjunction, — that it always links together mod- ifier and word modified, or governor and part governed. Once seen that all subordinate conjunctions stand between two parts, one of which either governs or modifies the other, and the obstacle in the way of clearly ex- plaining than whom and than which is overcome ; for that one supposed insurmountable obstacle is the word than, standing between the relative and the comparative adjective or adverb which governs the relative in the ob- jective. But in Demonstration XX., c and d, note 1, it is shown that, so far from really being an obstacle in the way of explaining the relative objective as the object of the comparative adjective or adverb, the than is actually the best possible proof that the relative is the object of such adjec- tive or adverb; since any and every subordinate conjunction performs that very office of linking together the governor and the governed. Take the sentence, "Beelzebub, than whom none sat higher, rose." It will be seen that this than joins together the comparative adverb higher and the objective whom, thus being in the very habitation of the subordinate conjunction — between governor (higher) and governed (whom). 3. Than Whom, Than Which, Than Me. — It is thus shown that than whom and than which are not only good English, and in harmony with the prin- ciples of universal grammar, but, moreover, that they are easily explained as such. Not that than whom is bad English, but that the grammarians have not understood the subordinate conjunction, is the difficulty. As for than me, it has nearly the same currency among "distinguished" authors, as has 532 THE SYSTEM METHOD. than wJiom and than which; and it is. beautifully explained in precisely the same way. If " He is wiser than me " is not good English, it cannot be because it is not good grammar easily exjolained. 4. Objections to Than "Whom, Than Me, etc. — There is but one objection, and that is simply this : the inability of the objector to under- stand than whom, than me, etc., — his inability to explain them as being in accordance with the principles of grammar. Not to detain the reader longer here, I leave him to decide whether the explanation above given is not sound, clear, and conclusive. The following js the feeble nature of the ar- gument against the objective after than, as quoted from A. H. Welsh, author of "Development of English Literature and Language": — "An attempt has been made to justify 'than me' hy appeal to the awkward phrase ' than whom ' :— " ' Which, when Beelzebub perceived, than ivhom, Satan except, none higher sat." — Milton. ' Than who ' would be be more consistent, more accurate, and, even to unfamiliar ears, would not be intolerable. 'Than whom' is an instance of grammatical vice, which, from having been endured, is now, from its long continuance, likely to be embraced. If ' than me ' [and ' than whom ' and ' than which '] is proper, why not ' as me ' [ ' as her,' etc.] ? Yet who does not receive a verbal shock from Trollope's question, 'What would be the feelings of such a woman as herf' "—Essentials of English, p. 208. Why grammarians generally, with Mr. Welsh, though acknowledging it to be "embraced" by the most distinguished authors, should suppose than wJwm to be improper, is plain; for they have known neither the subor- dinate conjunction as a link between governor and governed nor the ob- jective as the object of adjective and adverb; they could therefore explain neither than nor lohom. But to admit this beautiful phrase than whom to be "embraced" by the distinguished, and yet to report it as an "intolerable" "instance of grammatical vice," — to report it to be "embraced" because it is "intolerable"! — is to report what is not improbably untrue. The evident reason that "as me" is not good grammar, is that the sub- ordinate conjunction as is ignorantly jiut in between me and a word (the noun woman) that cannot govern that m.e; whereas, it is the law of grammar that every subordinate conjunction must stand between words, one of which governs the other; just as than in "Solomon, than whom no man is wiser," etc., is built in between whom and the adjective wiser that governs whom: " no man is wiser than whom." And thus it is that a true understanding of than and as as subordinate conjunctions, not only justifies the beautiful system-fitting than whom, but it likewise, by the force of reason as guided by system, condemns "asms," "as her," etc. "As her" is wrong for the same reason that other sentence parts be- come wrong, — because they are not explainable by, orbranchable through, the grammar-science system. (See Demonstration XXII.) But Mr. Welsh's effort to make "as her" grate upon the ear with a "verbal shock" must remain entirely ineffective until he so modifies his sentence, "He than who no man is wiser," etc., that the English ear may not perceive the "than who " I BBANGEINO THE CLASSES. 533 5. The Common Explanation op Than Whom, Than Which, etc. — That the reader may grasp the importance of understanding the true doc- trine of the subordinate conjunction, and that he may not want wherewithal to shatter the feeble fortifications of teachers, who, from infirmities or ha- tred for truth, shut their eyes against light, I will now mention the objections to the use of than who after comparatives, by pointing out something of the fallacy by which it is commonly justified, and by which it is sought to con- demn than whom. I quote from Alfred Holbrook's "Complete Enr^lish Grammar," pages 174 and 46, sections 1120 and 298, quoting from this work only, because no others have given the common explanation of than whom in a form so complete and concise: — " Wliom is used for who ; ' Solomon, than whom there never was a wiser man, says,' etc. " Whom is (1.) pronoun, (2.) relative, (4.) its antecedent 'Solomon,' (5.) with, which it agrees in the 3d, singular, masculine ; objective, used by enallagc for the nominative 'who,' and is equivalent to 'and he.' By expanding, and supplying the ellipses [298] , the above sen- tence reads, 'Solomon, and there never was a wiser man than A« [was], says,' etc., thus separating 'whom' into its two elements, the [co-ordinate] conjunction and, and the pro- noun he. It is thus shown that the objective 'whom' is used by enallage for, and really should be [298], the nominative who, and is not the object of the conjunction 'than' as some grammarians affirm." [The numbers employed in the discussion following refer to sections of Mr. H's " Complete English Grammar," quotations from which are indica- ted. Sentences 1st and 2d refer resiDectively to the complex sentence con- taining whom, and to the expanded compound sentence containing not whom, but and lie in its stead.] a. At the very outset, without proving that the sentence is not a sen- tence whole, our author assumes an ellipsis ! precisely what the zoologist does, who, without first proving that an animal, say a rabbit, is not a rabbit whole, assumes that a cat-like tail should be supplied, simply because that part may be found on other animals! See note 3 above, where it is shown that the scientist is by law bound to ex]3lain all things as by law they are, — as things whole, — and that, therefore, until it is first proven that a sen- tence is not a sentence whole, it cannot be assumed that something is to be supplied. Never having known grammar as a class-branch, science-system tree, learned only by consciously mirroring word (sentence part) in sentence whole; never having known what a grammar-system tree trunk means, or what it means to focus, or trunk, a system on what is the part (word) of an including whole (sentence); consequently, never having perceived that every scientist, grammarian, orthographer, botanist, zoologist, is by law bound to assume and explain things as by law they are, namely, things whole, until they are proven not to be whole, he has, at the outset, com- mitted the same ridiculous folly committed by a teacher of orthography, who, without first pTOwm^ that the word, say sugar, is not whole, assumes that some part, say h, should be supplied (thus giving shugar!), simply because that 7i-part may be found after s in other word wholes! 534 THE SYSTEM METHOD. b. Sentence 1st is a complex sentence, as Mr. H. agrees; for it "con- tains the complete subordinate sentence," (" than whom there never was a wiser man,") " as a constituent part " (1002). To prove that than wJiom, etc., is a subordinate sentence, we have only to quote Mr. H. as saying, 1. Whom is a relative (1120); 2. It is therefore a subordinate connective (1038, Ex.), and connects clauses (236); 3. Solomon is the antecedent of tohom (1120). Now, since a "relative clause" joined to an "antecedent" can never be aught but a subordinate sentence, therefore this than whom clause is a sub- ordinate sentence. It is thus demonstrated, by Mr. H. himself, that this than whom clause is a subordinate sentence; it is therefore likewise demon- strated that the complete sentence is a complex sentence, since it " contains this subordinate sentence as a constituent part" (1002). c. Thus does Mr. H. testify that the sentence of which tJian whom is a part, is a comiDlex sentence; but before than whom is explained, he directly contradicts this testimony by declaring it to be part of a compound sentence! For observe: sentence 2d cannot be a comjalex, but must be called a com- pound, sentence, in so far as it "contains two simple sentences" (1005) joined by and, a co-ordinate conjunction (1120, 631). It is thus seen that, in assuming "ellipses" and "expanding" (!) sentence 1st, he has actually changed it into a compound sentence! The subordinate sentence, the than whom clause, which he declares is a "constituent part" (1002) of the com- plex sentence whole, he tears out of that complex sentence whole, and, by a marvelous "expanding" process, transforms it into one of two co-ordi- nate (1008) and independent sentence wholes, joined by the co-ordinate connective and, thus forming of the two co-ordinate sentences a compound sentence (1005)1 The reader is familiar with how, after causing a deep sleep to fall u^jon Adam, the Creator tore from the complex man whole a rib, which, by a wonderful "expanding" process, he transformed into a beautiful woman — one of two co-ordinate and independent man wholes. It is something of that kind, no doubt, so marvelously employed by Mr. H., in transforming this subordinate-sentence part into a co-ordinate-sentence whole! evidently, since both alike are a miraculous transforming of a jjart into a whole! Give an orthographer the complex word whole, redder. How much would the reader not be elated at his wisdom, should he, without first prov- ing that redder is not a word whole, assume an "ellipsis," and, accordingly, tear out of the complex word whole {redder) the subordinate part der, gravely declare it "expanded" into deer, thus suddenly discovering that the complex word redder is not the complex word redder, but that it is a com- pound word of two co-ordinate parts — " red deer "I As well pretend to ex- plain the subordinate-word part der by tearing it out of the complex word whole redder, and thus assuming it to be " expanded" into a two-horned deer, as to pretend to explain the subordinate-sentence part, than whom, etc., by tearing it out of the complex sentence whole and assuming it to be mirac- ulously transformed into a co-ordinate and independent sentence whole! BRANCniNQ THE GLASSES. 535 Give a zoologist a comiDlex animal whole, say a wasp, to explain. How would you not be almost fascinated by his learning, should he, without first proving that the insect is not an insect whole, assume, in his "entomolog- ical erudition " a " hymenopterous " "ellipsis," and, accordingly, tear out of the complex wasp whole the subordinate part abdomen, and assume it to be "expanded," or transformed, into an independent wasj)! d. But after the escapade, — after having so miraculously "expanded " the subordinate-sentence p«?'< into a complete sentence whole, — our author leaves w/tom entirely unexplained! " Whom should be loho," he says; but the mere beginner in literary studies, knows that such a sentence as, " Sol- omon, than who there never was a wiser man, says," is found in no author of literary taste or fame, nor, indeed, in the less learned writers. " Whom is used by enallage for [in place or stead of] the nominative who," he con- tinues; but how can whom be used in the place of who, when who itself was never used in that place? Thus, by contrasting the common explanation condemning than whom, with that which justifies it, we are not only able to detect the unsoundness of the received explanation, but we also attain, (1.) the true theory of sub- ordinate conjunctions; (2.) the true understanding of the objective as gov- erned by adjectives and adverbs, esjjecially by comjjaratives separated from their substantive objectives by than, (3.) and, through these, the only solu- tion of than whom not unsound and unclear. Therein we attain the truest and simplest explanation of than and as, than which, as subordinate con- junctions, nothing in grammar is more generally misunderstood. Prog. XXX. — Branching by the Grammar-System Tree, Note. — For how we may know what correct language is, see Demonstration XXII. The advanced student should not be unable to reproduce the arguments found on pages 365-402 inclusive ; and, if possible, he should read the entire educational discussion, pages 15-145 in- clusive. For the two great steps — classifying and branching — to be taken in mastering a branch of learning, read pages 29-31, the preface, and page IGO. FARTICULABS: 1. No parallel can be found in the annals of the world. 2. The time for action came and passed. 3. He reports that the king is dead. 4. Whom does he honor more than me ? 5. Whither I go, ye cannot come. 6. I have come that I may see. 7. It is strange that you should think so. 8. The fact that he killed her is apparent. 9. It is just in that it is right. 10. We are sorry that it is so. 11. He was so weak that he feU. 12. That the earth is a sphere is easily proven, 13. Be so good as to go. 14. He is taUer than I am. 536 TEE SYSTEM METHOD. 15. I chose him as honest. 16. I would go rather than stay. 17. I am doubtful as to him. 18. To be good is to be great. 19. To be a Sumner is to be a scholar. 20. To steal is base. 21. Beelzebub, than whom none higher sat, with grave aspect rose. 22. He constructs two trenches thirteen feet wide. 23. He collects his own nearer the mountain. 24. He died many years ago. 25. He is worthy praise. 26. He came an hour sooner. 27. This house is built a gi'eat deal better. 28. Vice is a monster of so frightful mien, As, to be hated, needs but to be seen. — Pope. 29. Solomon, than whom there never was a wiser man, says, " Wisdom is more precious than rubies." 30. Unless I am mistaken, it was he. 31. If you are honest, j'ou wiU be respected. 32. Ye shall not touch it, lest ye die. 38. The teacher praised you more than me. 34. This is a proof that he never came. 35, Man never is, but always to be, blest. 86. He is not as clever as you are. 37. Be it a trifle, it will be well done. 38. I should be guilty of ingratitude, than which nothing is more shameful. 39. As I began, so I wiU finish. 40. They chose Henry rather than him. 41. More women than men were present. 42. There I brought her as pitying her hard usage. 43. To be innocent is to be not guilty, but to be virtuous is to overcome our evil inclinations. 44. His zeal was wondered at. 45. The servant was spoken to by his master. 46. Gray hairs are Death's blossoms. 47. What if the foot, ordained the dust to tread. Or hand, to toil, aspired to be the head? — Pope. 48. You have confessed yourself a spy. 49. Virtue consists in being good. 50. He has not been here since then. 51. A wandering harper, scorned and poor. He begged his bread from door to door. — Scott. 52. He saidthat that that that that man said, was not that that that that man should say. 53. The English hate the French as frivolous. 54. Ye shall be as gods. 55. A noun is a name; as, man, eagle. BBANCHINa TEE GLASSES. 537 56. It is used as a conjunction. 57. He was regarded as dangerous. 58. Arnold's success as teacher was remarkable. 59. The circumstances are as follows. 60. Assume it as a fact. 61. She reigns as queen. 63. A greater than Solomon is here. 63. It shall be more tolerable for Sodom and Gomorrah in the day of Judgment than for that city. 64. These things I say that ye might be saved. 65. His stay here will be short. 66. I see men as trees walking. 67. As many as desire may go as soon as they choose. 68. The messenger comes running. 69. He rode seated between two officers. 70. It is I, Hamlet, the Dane. 71. I sang cheerily all day long. 72. Dying for principle is a higher virtue than scolding. 73. The blessed to-day is as completely so As who began three thousand years ago. — Pope. 74. He is taUer than John. 75. He is as good as she. 76. We listen when pleasure caUs. 77. The year when Chaucer was born is uncertain. 78. I shall not lag behind, or err The way, thou leading. 79. He that hath ears to hear, let him hear. 80. It is pleasant to see the sun. 81. I am pleased because this has happened, but I should have been disap- pointed if it had fallen out otherwise ; and I think that even now some of my real or supposed friends will be more surprised by the arrangement than satisfied with it. 82. History tells us that Socrates said that he was declared by the oracle to be the wisest of men merely because he knew that he knew nothing. 83. For me to labor and you to be idle would be unjust. 84. Better to reign in hell than sei^ve in heaven. 85. Where he was born is uncertain. 86. He traded with what capital he had. 87. Six times two are twelve. 88. Twice one are two. 89. He is a great deal worse. 90. The country was not a cent richer. 91. I wiU not be a step behind. 92. His spear was six feet long. 93. It is the foible of American youth, pretension. 94. It is a curious fact in modern history, the gi'owth of the machine shop. 95. It is James and Henry. 538 THE SYSTEM METHOD. 96. It was for Alexander to die in the prime of life. 97. It was the day that we came home. 98. It was in this tree that the rabbit ran. 99. Let us smoke, as it were, the pipe of peace. 100. He rushed down the hiU, heels over head. 101. They tore him limb from limb. 102. They go hand in hand. 103. Bit by bit the coming years steal all from us but woe. — Lowell. 104. He is crippled, hand and foot. 105. I purchased the following articles ; namely, tea, coffee, and sugar. 106. I would that I were young again. 107. Add the tens as if they were units. 108. If the sum can be expressed by one figure, %vrite it under the column added. 109. The man is, as it were, clapped into jail by his consciousness. 110. Be it enacted, that schools shall be established. 111. I know him to be honest. 113. This is a subject on which to show your powers. 113. He was so blind as not to see the danger. 114. The difficulties were so great as to deter him. ;" 115. He knew better than to resist. 116„ He would sooner go than stay. s^ 117. He had more money than he knew what to do with. \ 118. The time was that I hated thee. \ 119. He is conscious that he has done wrong. 120. It is to this place that the guUs resort. ■ ' 121. What man enters dies, 122. I have nothing to say. 123. She soured to what she is. 124. He is a fool in that he believes others to be as deceitful as he is. 125. They loved each other. 126. They hate one another. 127. He has more than atoned for his fault. 128. He more than atones for his fault. 129. They regarded him as being dangerous. 130. He is acting as governor. 131. As mayor of the city, I feel much aggrieved. 132. Shakspeare excels as a poet. 133. The beggar is greater as a man than the man is merely as a king. 134. I had as lief they would put ratsbane in my mouth as to attempt to stop it with security. 135. In this sense, the word means to place in an equal state as to obUgation. 136. John, as well as James, is here. 137. Five less two are three. 138. They move nearer the grounds. 139. He is a man worthy to be admired. 140. He is a man worthy admiration. SBANCEING THE CLASSES. 539 141. We were deceived as to his honesty. 142. As for me and my house, we will serve the Lord. 143. As for me, give me Uberty or give me death. 144. He did nothing but laugh. 145. I could not Uve but for my hope of eternal Ufe. 146. He gave you more than me. 147. There is nobody at home but I. 148. There was no man but prophesied revenge. 149. Tou can but try. 150. Every one can master a grief but he that has it. 151. But and if that servant say in his heart. My Lord delayeth his coming, the lord of that servant will come at a day when he looketh not for him. 152. He never fought except in his own defense. 153. Except a man be born again, he cannot see the kingdom of God. 154. There is no diderence except that some are heavier than others. 155. The smoother the surface, the deeper the water. 156. Better not to be at all than not be noble. 157. Evil, as well as good, hath its errands. 158. I shaU see you, then, when you come. 159. The wiser he grew, the humbler he became. 160. He did not decide the question as to who was the owner. 161. As to what is the usual course in such matters, I can give no opinion. 162. I will not be a step behind. 163. The painter flattered her a Uttle. 164. It does not matter one marble spUnter. — RusMn. 165. I would not care a pin, if the other three were in. — Shakspeare. DEMONSTRATION XXXI. FACT MATTES,— Analysis, or Dissection : a. Analysis. — Real, or true, analysis is discovering the attributes of the words (the parts) of the sentence that cause those words to belong in classes; and this discovering of the attributes of the words, when done as a system-led process, is branching— an up-the-tree process. For a fuller explanation of what true analysis is, see pages 101 and 113; and for exam- ples of analysis as such a system-guided process, see Work-Book No. 20, Programs XXII. and XXX. True analysis "is a tracing of things [words] to their sources [classes]." — Webster. And mark that so far as this "tracing (or branching) of words to their classes" is a system-led (system-like) proc- ess, it is a branching process, led by the branch guide, the tree. True analysis, in grammar, is analysis of the word, not of the sentence. But 540 THE SYSTEM METHOD. what is commonly called analysis, in grammar, is in reality sentence dis- section (from dis, apart, and sectum, to cut); for it is but a cutting apart of tlie joints between tbe parts of the sentence, as may be seen in any system of diagramming. b. When to be Learned. — Not as a primary, but only as a higher, study; not in the primary, but only in the advanced grades of schools. Since in disengaging the elements from each other, each word, or part, must be reviewed in its various and complex relations with every other part, or word, and with the sentence whole, for young children the study of "anal- ysis" is quite impossible; for such complex undertakings can be well per- formed only by minds understanding what the parts are, to what classes they belong. This grammatical exercise, called analysis, may be presented to students, with some assurance of their success, at any time after the reduc- ing of the particulars to classes (see Prog. XXII., page 485) is accomplished, but not before with safety. For, by the very nature of the process, unless conducted as a mere toy-like or rote exercise, the mind is compelled to hold the attention upon a multiplicity of parts at one and the same time. The only efficient "analysis" is such as ifeexpressed by continued and coherent speech on the part of the learner. With young children, or children younger than twelve to twenty years, this also is quite impossible, except as it is carried on by rote, which is worse than useless. Coherent speech con- tinued is impossible, until that finishing acquisition is attained, namely, the power of seeing, or mirroring, whole in part and part in whole. And that student, who, in analysis, has discursive thought and speech in reporting upon his diagrammed analyses, may be assumed to be well acquainted, not only with the particular sentence diagrammed, but with the laws, or prin- ciples, pertaining to all its parts. Success at discursive and coherent speech upon the general, or whole, the particulars of which have been previously studied, is the proper principle of, and evidence for, promotion. C. Word. — The whole of which the letter is the part. d. Element. — Any part of a sentence. Of the elements of sentences, there are four general kinds, — essential, accessory, connective, and inde- pendent. e. Essential, Element. — The verb and substantive forming the frame-work of a sentence: as, He can swim; Who am I? Plato, t&OB reasonest well; He crushed, the others may be persuaded; Preaching is teaching; For- getting the books delayed us. f. Connective Element. — A preposition or conjunction; as, Man is born to die; He differs from you; James and Henry remain; You may go, bat I will remain; I will go if it be possible; Whom does he honor more than me? I consider him as honest; Ye shall be as gods; It is right in that it is just; That he will succeed is evident (See Demonstration XX., Obs. f, 1); Less judgment than wit is more sail than ballast. BBANCHING THE CLASSES. 541 g. 4. Independent Element. — One limiting no other in the sentence; and not essential; as, Mercy! is the man killed? Plato, thou reasonest well, He destroyed, the rest will yield; Ah me! that I the judge's bride might be; Spain! thou wert of yore the wonder of the realms; Whoop, Jug! I love thee! The ship came into harbor, colors flying and gnns firing; The mount- ains rose, hight above hight; They sit side by side; Whatever is, is right. — Pope; " Respectfully yours, Henry Mendendall " (signature to a letter). h. Accessory Element. — Any element attached to another; one not essential to the frame-work of a sentence; as, He can swim; Who am I? Plato, thou reasonest well; He was ordered to go ; He is true. Of accessory elements, there are six divisions, — the adjectival, the ad- verbial, the adjectival, the infinitive, the participial, and the predicate. 1. Adjectival Element. — One limiting a substantive; as. The good man is dead; Changing his mind, he painted it white; His being false is shameful; To be good is to be happy {good and happy may be said here to limit the substantive verbs be); I regard him as honest; Being defeated, he re- tired; The boy who laughs, grows fat; The apostle Paul persecuted the Church; He talks of a return [to] home; The wicked shall perish, as all the world know; Time to come is called future; I feel my pulse beat- ing. The Pilgrim Fathers, where are they ? He is ruined, mind and body ; A noun is a name ; as man, eagle. 2. Objective Element. — The direct or factitive object of a transitive verb; as, They called her Mary; They paid him money; We forgive our friends their debts; We saw him; I believe whoever believes me. 3. Adverbial Element. — A word, or an objective with or without a preposi- tion, employed like an adverb; as, Man is wonderfully organized; He stopped at Rome; It led them to a mutual respect; They remained one hour; He came an hour sooner; I paid him the money; He starts to-mor- row; Whom does he honor more than me? Less judgment than wit is more sail than ballast; The wheat measured more than three bushels; I shall go if it be possible; It is right in that it is just; I would better die than live falsely; I would rather go than stay; The lake is ten miles long. 4. Infinitive Element. — An infinitive used as a verbal complement; as, I shall live; Thou mayst love; I had rather be a kitten and cry mew; I heard him sing; I feel my pulse beat; I see its sparkling bubbles swim. 5. Participial Element. — A participle used as a verbal complement; as, The day is ending; I have written; It is to be remembered; He is universally respected; Be ye reconciled to God; Bills are requested to be paid in advance. 0. Predicate Element. — Any substantive or adjective in the predicate after a copula; as. He is true; We found him to be true; It is they; Doing is not sleeping; Hassan is my name; They took him to be an enemy; They hated him as being an enemy; I consider learning to be branching, i. Simplicity. — The parts of a sentence are also distinguished accord- ing to the principle of simplicity, into simple, complex, and compound. 542 TEE SYSTEM METHOD. 1. Simple Element. — One not limited by any other; as, The cherries are ripe. 2. Complex Element. — One taken with its modifiers; as, The cherries are very ripe. 3. Compound Element. — Two elements joined by a co-ordinate conjunction; as, Man is fearfully and wonderfully made. j. Base. — That word to which a modifier is attached; as, The cherries are very ripe; We met a spry young mail; He will come if it be possible; We consider him as worthy; See the goose standing on one foot; See the goose standing on one foot; He is blessed with a good home; Caesar might have been crowned; Caesar might have been crow7ied. iVofe.— Whenever it happens that the base is either the essential verb or the essential substantive, it is more properly called, not "base," but " the essential element; " since ju- dicious teaching will fail, at no favorable circumstance, to fix the learner's thought upon what is essential in the sentence whole. No competent explanation will leave undistinguished, on the student's part, what is essential, and what is merely accessory or connective or inde- pendent, in the sentence. In "The cherries are ripe" and "Caesar might have been cro^vned," we speali, not of cherries, are, Ccesar, and might as bases, but of each as being an "essential element," though, to be sure, each of these four words is a base. Moreover, in "He is true," "It is John," "We proved him to be false," " Giving is receiving," "It is he," we speak not of true^ John, false, receiving, he, as " elements " at all, but merely as being " predicate adjec- tive," "predicate noun," "predicate pronoun," "predicate participle," or "predicate sub- stantive,'" according as the case may be, and according to our choosing. k. Class. — Elements are further distinguished in relation to their bases, into first-class, second-class, and third-class elements. 1. First-Class Element. — One whose base is a single part of speech; as, The lake is very rough; We met John Jones; Caesar might have been crowned. {Have been crowned is to be considered an element of the first class, be- cause its base is the single word have.) I must go; The day is ending; Being defeated, he retired. 2. Second-Class Element. — One whose base is the object of a preposition; as. He came to stay permanently; We find them in the woods and by the streams; Time to come is called future. 3. Third-Class Element. — A subordinate sentence, or clause; as,. The boy who laughs, grows fat; Whatever is, is right; That he will succeed is evident; Many things depend on who built the house. Note.— Let it be carefully observed that a third-class element may be, and often is, in- cluded within a second-class element. Thus in the last sentence above, while " Who built the house " is a subordinate sentence, and therefore an element of the third class, yet "on who built the house," taken as a whole, is an element of the second class, since its base, "who built the house," is the object of the preposition on. 1. Sentence. — The whole of which the word is the pa'-t. Sentences are divided according to the principles of simplicity, rank, and assertion. m. As TO Simplicity. — As to simplicity, sentences are simple, com- plex, compound, parLial compound, and complex compound. 1. Simple Sentence. — One containing but one verb and one subject; that is, one containing but two essential elements; as, He ordered the bridge to be broken down; The boy grows fat BRANCHINO THE CLASSES. 543 2. Complex Sentence. — One containing a third-class element; as, The boy who laughs, grows fat; Whatever is, is right; What man enters, dies; If he does not hear, I am mistaken. 3. Compound Sentence. — Two or more sentences joined by a co-ordinate conjunction. 4. Partial Compound Sentence. — One whose essential substantive has two or more essential verbs, or one whose essential verb has two or more essential substantives; as, John went and returned; John and James went. 5. Complex Compound Sentence. — A compound sentence containing one or more complex sentences; as. The dispatch which was sent last night was received, and the news has created a joy of which we had not dreamed. n. As TO Rank. — As to rank, sentences are principal, subordinate, leading, and co-ordinate. 1. Principal Sentence. — One forming but not limiting either the part or the whole of a complex sentence; as. The boy who laughs, grows fat; That he will succeed is evident; Whoever studies will learn. Note.—Yifii it be observed that while, in the sentence, "The boy who laughs grows fat," the principal sentence, "The boy grows fat," forms but part of the complex sentence whole, on the contrary, in "Whoever studies will learn," the principal sentence includes the entire complex sentence; for the subordinate sentence, " Whoever studies," is itself the subject of, and is thus incorporated in, the principal sentence, "M^ioever studies will learn," which is the same as the whole complex sentence. 2. Subordinate Sentence — One either forming or limiting some part of a complex sentence; as. The boy who laughs, grows fat; That he will succeed is evident; Whoever studies will learn. (See Demonstration XS., Obs. f.) Note.— Observe that while, in the complex sentence, "The boy who laughs, grows fat," the subordinate sentence, "Who laughs," merely ?i»'i^s the word "boy," a part of the com- plex sentence whole, on the contrary, in the sentence, " That he will succeed is evident," the subordinate sentence, "That he will succeed," actually /orwis a part (a subject part) of the complex sentence whole. 3. Leading Sentence. — The first of the two or more simple or complex sen tences contained in a compound sentence. 4. Co-ordinate Sentence. — -Any sentence contained in a compound sentence not of unequal rank with the leading sentence. 0. As TO Assertion. — As to assertion, sentences are divided into the classes declarative, interrogative, imperative, and exclamatory. 1. Declarative Sentence. — One that does not ask a question, and whose verb is either an indicative or a subjunctive; as, James arrived yesterday; I will go if it is possible; He will maintain his integrity though he lose his estate; He asked who the man was. 2. Lnterrogative Sentence. — One that asks a question, and whose verb is indicative; as. Did James go to the city? He asked, " Who was the man? " 3. Lmperative Sentence. — One whose verb is in the imperative mood; as, Run to your houses; Fall upon your knees; Somebody ring the bell; Hal- 544 THE SYSTEM METHOD. lowed be thy name; Be he a king, he is still a dependent creature; May you ever prosper; Exalt we his name; Be this our motto. IV. Exclamatory Sentence. — One expressing an outcry of pain, surprise, anger, or the like; as. How are the mighty fallen! DIAGRAMMING— T\\e System of Diagrammmg Explained: 1. The two essential elements are joined by the vertical tie, f 2. The objective element is set off by a vertical bar, |, the adjectival element by the same bar with its top slanting to the left, and the adverbial element by the same bar with its top slanting to the right. 3. The infinitive element is set off by the adverbial bar with a hook to the left, ]/ , and the participial element by the same adverbial bar with a hook to the right. 4. The predicate substantive is set off by a horizontal bar, , to which the predicate adjective adds the adjectival bar, \. 5. Relative pronouns, conjunctive adverbs, and prepositions and their objects are underscored with a single line, and conjunctions are inclosed in parentheses. 6. Words to be supplied to complete a sentence whole are inclosed in brackets, [ ]; and factitive adjectives and substantives are set off as are predicate adjectives and substantives, but they are put adjacent to the direct object. 7. The bar is lengthened to include two or more elements, and the 8. Independent elements are placed above, and separate from, the sen- tence body proper, and any modifier of a sentence whole is joined to the tie by a connecting line. Prog, XXXI. — Complete Program for Written and Oral Analysis. ^™P^f 1 declarative . , complex interrogative IS a compound imperative partial compound exclamatory 1^ complex compound J |^ •' is the subject, sentence. 2. Of which 3. Of which is the essential element, 4. Unmodified; or modified by , f adjectival ■] 1 objective | •{adverbial J- element of the I infinitive | [participial J 6. Of which , the base, is [here repeat 4 and 5] . 5. A \ ''"•?!'' ( complex first second third class. 7. Of which sentence, also, -is the predicate, BBANGHINa THE GLASSES. 545 8. Of which is the essential element, 9. And a predicate ] ^ ^^ ^T.- > [here, if necessary, repeat 4 and 5]. ^^ j is I \^ connective, I is also ) I an independent element. Note. — To beginners, unless they are thoroughly grounded in the work of branching, the teacher may assign such simple sentences as will require only a part of the ten steps of the program above. Thus, the simple sentence, "Flowers bloom," would require but four (1st, 2d, 4th, and 7th) of the ten steps : Flowers hlbom (1.) is a simple declarative sentence, (2.) of which^?owers is the subject, (4.) unmodified. (7.) Of which sentence, also, bloom is the pred- icate, (4.) unmodified. But to students having been measurably successful in branching by the work-tree com- plete, this entire 10-step program may safely be presented at the outset of the work in this dissecting work called analysis. In his first exercises, the learner should be directed to com- mit this work to writing. TA^TICTJIjAnS: Note. — In so far as a supply of sentences, thoroughly exhaustive for parsing or branch- ing or analyzing (dissecting), has already been provided in the thirty demonstrations already given, we shall give here only such and as many sentences as are needed to illustrate the sys- tem of diagramming above explained, and demonstrate its adaptability to every species, even the most difficult, of the English sentence. Attention is invited to this system of diagram- ming as ej'pressing the greatest possible number of things which it is desirable to make sa- lient in the explanation of the sentence. 1. The fox caught the hen. 2. We found three pairs of scissors in the wheat. Diagramming : r- fox \ The r ^^ / In wheat \ the 2. . caught I hen \the Analysis : 1. "The fox caught the hen," is a simple declarative sentence, 2. Of which the fox is the subject, 3. Of which fox is the essential element, 4. Modified by the, 5. A simple adjective element of the first class. 6. 7. Of which sentence, also, caught the hen is the predicate, 8. Of which caught is the essential element, 4. Modified by the hen, 5. A complex objective element of the first class, 6. Of which hen, the base, 4. Is modified by the, 5. A simple adjective element of the flurst class, 35 Uhree \ ^ found Uhree I pairs \ Qf scissors 546 THE SYSTEM METHOD. 3. Csesar miglit have been crowned. 4. The ship which sailed yesterday was laden with salt. Diagramming . . "Caesar sailed / yesterday . might l/have A been A crowned L was / laden / with salt 1. "Caesar miglit have been cro^vned," is a simple declarative sentence, 2. Of which Ccesar is the subject, 3. 4. Unmodified. 5. 6. 7. Of which sentence, also, might have been crowned is the predicate, 8. Of which might is the essential element, 4. Modified by have been crowned, 5. A complex infinitive element of the first class, 6. Of which have, the base, 4. Is modified by been crovmed, 5. A complex participial element of the first class, 6. Of which been, the base, 4. Is modified by crowned, 5. A simple participial element of the first class. 1. "The ship which sailed yesterday was laden with salt," is a complex declara- tive sentence, 2. Of which the ship which sailed yesterday is the subject, 3. Of which ship is the essential element, 4. Modified, first, by the, 5. A simple adjective element of the first class, And secondly, by which sailed yesterday, 5. A simple adjective element of the third class, 2. Of which which is the subject, 3. 4. Unmodified. 5. 6. 7. Of which subordinate sentence, also, sailed yesterday is the predicate, 8. Of which sailed is the essential element, 4. Modified by yesterday, 5. A simple adverbial element of the first class. 6. 7. Of which sentence, also, was laden with salt is the predicate, 8. Of which was is the essential element, BRANGEING THE CLASSES. 54^ 4. Modified by laden with salt, 5. A complex participial element of the first class, 6. Of which laden, the base, 4. Is modified by with salt, 5. A simple adverbial element of the second class. 5. The business is such as concerns me. 6. The conditions of the agreement were as follows. Diagramming : r- business NThe }^^^ ^ conditions \ „ ^ X^-.. \ of agreement \tne \ such L J 1 . L were I L concerns I me L follows Analysis : 1. "The business is such as concerns me," is a complex declarative sentence, 2. Of which the business is the subject, 3. Of which business is the essential element, 4. Modified by the, 5. A simple adjective element of the first class. 7. Of which sentence, also, is such as concerns me is the predicate, 8. Of which is is the essential element, 9. And business, understood, a predicate substantive, 4. Modified first by such, 5. A simple adjective element of the first class, And secondly, by as concerns me, 5. A simple adjective element of the third class, 2. Of which as is the subject, 4. Unmodified. 7. Of which subordinate sentence, also, concerns me is the predicate, 8. Of which concerns is the essential element, 4. Modified by we, 5. A simple objective element of the first class. 7. Buy what you want. 8. The wicked shall perish, as all the world knows. Diagramming : - [ y°» ] •7 p you . Buy I The .[peoph]\ \ wicked r'^'^'-Itl\ , N _ want I what L shall /perish Analysis : 1. "Buy what you want," is a complex imperative sentence, 3. Of which [youj is the subject, 4, Unmodlfiied. all the knows! as 548 THE SYSTEM METHOD. 7. Of which sentence, also, buy what you want is the predicate, 8. Of which buy is the essential element, 4. Modified by what you want, 5. A simple objective element of the third class, 2. Of which you is the subject, 4. Unmodified. 7. Of Tifhich subordinate sentence, also, want what is the predicate, 8. Of which want is the essential element, 4. Modified by what, 5. A simple objective element of the first class. 8. "The wicked shall perish, as aU the world knows," is a complex declarative sentence, of which the wicked \peo2}le~\ is the subject, of which jDeople, understood, is the essential element, modified by the and wicked, two simple adjective elements of the first class. Of which sentence, also, shall perish is the predicate, of which shall is the essential element, modified by perish, a simple infinitive element of the first class. Of which complex sentence, also, the principal-sentence part is modified by as all the world k?iows, a simple adjectival element of the third class, of which all the world is the subject, of which world is the essential element, modified by all and the, two simple adjectival elements of the first class. Of which subordinate sentence, also, knows as is the predicate, of which knows is the essential element, modified by as, a simple objective element of the first class. As is also a subor- dinate connective. 9. If thou read this, O Caesar, thou mayst live. 10. It is important what he is. Diagramming : O, Caesar _ thou 9. (If) ^ It _ thou _ mayest /live / ID -he - is what important _ read I L J \ this Analysis : 9. "If thou read this, O Caesar, thou mayst live," is a complex declarative sentence, of which thou is the subject, unmodified. Of which sentence, also, mayst live if thou read this is the predicate, of which mayst is the essential ele- ment, modified by live if thou read this, a complex infinitive element of the first class, of which live, the base, is modified by thou read this, a simple adverbial element of the third class, of which thou is the subject, unmodified, and read this [paper] the predicate, of which read is the essential element, modified by this [jjaper] , a complex objective element of the first class, of which 2Mper, understood, is the base, modified by this, a simple adjectival element of the first class. Coenar are two independent elejnents, and if is a subordinate connec tive, joining its clause to live. BRANGHING THE GLASSES. 549 10. " It is important what he is," is a complex declarative sentence, of which it what he is is the subject, of which it is the essential element, modified by what he is, a simple adjectival element of the third class, of which he is the subject, unmodified, and is what is the predicate, of which is is the essential ele- ment, and what a predicate substantive. Of which principal sentence, also, is important is the predicate, of which is is the essential element, and important a predicate adjective. What is also a connective. 11. The book is where you laid it. 12. It is right in that it is just. Biagramm,ing : r- book \ The It 11 { laid where . 12 just 'it Lis Z A right Analysis : 11. "The book is where you laid it," is a complex declarative sentence, of which the book is the subject, of which book is the essential element, modified by the, a simple adjectival element of the first class. Of which sentence, also, is where you laid it is the predicate, of which is is the essential element, modified by where you laid it, a simple adverbial element of the third class, of which you is the subject, unmodified, and where laid it the predicate, of which laid is the essential element, modified by where, a simple adverbial element of the first class, and also by it, a simple objective element of the first class. Where is also a subordinate connective. 12. "It is right in that it is just," is a complex declarative sentence, of which it is the subject, unmodified. Of which sentence, also, is right in that it is just is the predicate, of which is is the essential element, and right a predicate ad- jective. Is is also modified by in that it is just, a simple adverbial element of the second class, of which it is just, the object of the preposition in, is a simple element of the third class; of which if is the subject, unmodified, and. is just the predicate, of which is is the essential element, and just a predicate adjective. That is a subordinate connective. 13. She boiled the eggs hard. 14. They elected him president. Diagramming : f-She —They 13 J4-. . elected I him — president - boiled 1 eggs . A hard \the Analysis : 13. " She boiled the eggs hard," is a sim. declar. sen., of which she is the 650 THE SYSTEM METHOD. subject, unmodified. Of which sentence, also, hmled the eggs Tiard is the predicate, of which hoiled is the essen. ele., modified by the eggs hard, a compl. obj. ele. of the 1st class, of which eggs is the base, and hard a factitive adjec- tive. Sggs is also modified by tlie, a sim. adj. ele. of the 1st class. 14. " They elected him president," is a sim. declar. sen., of which they is the subject unmodified. Of which sen. also, elected him president is the predicate, of which elected is the essen. ele., modified by him and president, two simple obj. eles. of the first class, of which him is the direct, and 2^'^'^sident the factitive, object. 15. We could not conceive what liad made him v^hat we found him. 16. I am doubtful as to your statement. 17. His reasons are as grains of sand. Diagramming : We ' 15, [ what / I r had l>' made I him — I , L found I him _ what it — — I — I i^ reasons \ His 16. (as) A doubtful / to statement 17. (as) . _ are grains \ of sand Analysis: 15. "We could not conceive what had made him what we found him," is a compl. declar, sen., of which we is the subject, unmodified. Of which sen., also, could not conceive (and all that follows) is the predicate, of which could is the essen. ele., modified first by not, a simp. adv. ele. of the 1st class, and secondly, by conceive what had made (and all that follows), a compl. infinitive ele. of the 1st class, of which conceive, the base, is mod. by what had made (and what follows), a sim. obj. ele. of the 3d class, of which what is the subject, unmodified, and had made (and what follows) the predicate, of which had is the essen. ele., mod. by made him (and what follows), a compl. participial ele. of the 1st class, of which made, the base, is mod. by him (and what follows), two sim. obj. eles., of which him, the direct object, is of the 1st class, and what we found him, the factitive object, is of the 3d class, of which we is the sim. sub., unmodified, and found him, what the predicate, of which found is the essen. ele., modified by him and what, two simple obj. eles. of the 1st class, of which him is the direct, and what the factitive, object. The whats are also subordinate connectives. 16. " I am doubtful as to your statement," is a sim. declar. sen., of which J is the subject, unmodified. A>n doubtful as to your statement is the predicate, of which am is the essen. ele., and doubtful a predicate adjective, mod. by to state- nRANCHlNCf THE CLASSES. 551 ynent, a sim. adv. ele. of the 2d class. As is a subordinate conjunctiou, joining to statement (the modifier) to the thing modified, doubtful. 17. " His reasons are as grains of sand," is a sim. declar. sen., of ■which his reasons is the subject, of which reasons is the essen. ele., mod. by his, a sim. adj. ele. of the 1st class; of which sentence also, ai-e as grctitis of sand is the predicate, of which are is the essen. ele., and grains a predicate substantive, mod. by of sand, a sim. adj. ele. of the 2d class. As is a subordinate conjunction (connecting are, the governor, and grains, the word governed — in the nomina- tive). 18. We chose Mm as honest. 19. A noun is a name; as, man, eagle. 20. Liberty was theirs [ ] as men['s]. 21. Arnold's success as teacher ['s] was remarkable. Remarks: 1. Honest ia an adjectival modifier of him, and as is a subordinate conjunction. (See Demonstration XX., d, notes 1 and 2.) 2. JIan and eagle are adjectival to name, and this second as is also the subordinate con- junction. (See p. 478.) 3. The 's is omitted from men and teacher, because of the fact that, as appositives to theirs and Arnold's, they stand less directly connected to the nouns limited, {jaossessionl and success, than do theirs and Arnold's. Before parsing, branching, or analj-zing, the 's is to be supplied to men and teacher, which words, otherwise, do not fit, and are unexplainable by, the system of grammar, except as "figures of speech." But to explain a word is to show how it is law-like, how it fits, or belongs in, the science-tree system of grammar ; therefore, to pretend to explain men and teacher, by sajang that they " are examples of the ' figure of speech,' called enallage," is to pretend to show that men and teacher are law-like things by showing that they are lawless ! It is proving that these words fit the system by proving that they do not fit the system! being "examples of enallage." lien, and teacher here in these incomplete sentences are what an arm would be in an incomplete man, —a man with a liga- ment absent by which arm is attached to shoulder. Such missing ligament (which is here the missing 's) must be supplied before such arm can either be made to fit the man whole or to accord with, and be explained in the light of, the arm system. So here: the missing 's must be supplied before such word (man or teacher) can either be made to fit the sentence whole or to accord with, and be explained in the light of, the word sj'stem — the science-tree system, as sho-wn on page 494. Whenever it is seen, by grammarians, that grammar is a tree system of the word, students in the schools will immediately discover the organisms (animals, plants, etc.) of nature as its indispensable aids,— grammatical electric lights, as it were, — and grammar will take on an efficacy heretofore little dreamed of. Diagramming : ^ Liberty (as) Success \ Arnold's \ "teacher's 20. (as) _ was -T possession l \ theirs V men's 21 r was i remarkable 20. "Liberty was theirs as men ['s]," is a sim. declar. sen., of which liberty is the subect, unmodified. Of which sentence, also, ivas theirs as nien^s is the predicate, of which was is the essen. ele., and possession, understood, a predicate noun, modified by thei7-s as men's, a compl. adj. ele. of the 1st class, of which 552 THE SYSTEM METHOD. theirs, the base, is mod. by men's, a sim. adj. ele. of the 1st class. As is a sub- ordinate conjunction, joining rnert's to theirs, with which meri's is in apposition. 22. He is taller than I. 23. Whom does he honor more than me? 24. Less judgment than wit is more sail than ballast. 25. I would better die than live falsely. 26. I would rather go than stay. 27. Beelzebub, than whom none higher sat, with grave aspect rose. Eemarks : 1. In the 22d, / [ami is an adverbial modifier of the adjective taller, to which it is joined by the subordinate conjunction than. 2. In the 23d, »ie is the adverbial object of the adverb more, to which the same subordi- nate than joins it. 3. In the 26th, the infinitive substantive stay is an adverbial objective of the cojpparative adverb rather, which modifies would. Diagramming : (than) ^ — judgement \ less / wit 24. ( than ) sail \ more / ballast 25. Zdie (than) ter/live /falsely . Beelzebub 27. none (Ihan) / whom grave Arialysis : 24. "Less judgment than wit is more sail than ballast," is a sim. declar. sen., of which less judgment than ivit is the subject, of which judgment is the essen. ele., mod. by less than wit, a compl. adj. ele. of the 1st class, of which less, the base, is mod. by wit, a sim. adv. ele. of the 1st class. Than is a subor- dinate connective. Of which sentence, also, etc. 25. " I would better die than live falsely," is a sim. declar. sen., of which / is the subject, unmodified, bmA would better die than live falsely is t\x& predicate, of which would is the essen. ele., mod. by die, an infin. ele. of the 1st class, and also by better than live falsely, a compl. adv. ele. of the 1st class, of which better, the base, is mod. by live falsely, a compl. adv. ele. of the Ist class, of which live, the infin. base, is mod. by falsely, a sim. adv. ele. of the 1st class. Than is a subordinate conjunction. 37. " Beelzebub, than whom none higher sat, with grave aspect rose," is a compl. declar. sen. , of which Beelzebub, than whom, none higher sat, is the subject, of which Beelzebub is the essen. ele., mod. by than whom none higher sat, a sim. adj. ele. of the third class, of which none \one'\ is the subject, of which one, un- derstood, is the essen. ele., mod. by none, a sim. adj. ele. of the 1st class. Of BBANonma the classes. 553 which subordinate sentence, also, sat higher than whom is the predicate, of which sat is the essen. ele., mod. by higher than whom, a compl. adv. ele. of the 1st class, of which higher, the base, is mod. by whom, a sim. adv. ele. of the 1st class. Than is a subordinate conjunction, joining whom, the word governed, and higher, the adverb that governs whom (see Demonstration XX., d, and XXX., m.). Of which sentence, also, etc. 28. Virtue consists in being good. 29. To be good is to be happy. 30. His being false is shameful. 31. They paid him the money. BemarJcs : 1. In the 28th, good is best explained as an adjective appositive to, and mod- ifying, the substantive participle being. And, similarly, explain good and happy, in the 29th, as appositive adjectives modifying the substantive infinitives be. (See Demonstration XXIX., c, note.) In the same way, also, the adjective false is appositive to, and modifies, the sub- stantive verb being., in the 30th. 8. In the Slst, him is branched and diagrammed as an adverbial object of paid. In like manner, explain every indirect object, — that is, without supplying something. Diagramming : .^ , \ , \His _ To be Vgood- 29. \ —being \ SO / to be V happy L is ^-A shameful Analysis : 29. '.' To be good is to be happy," is a sim. declar. sen., of which to be good is the subject, of which to be, the infinitive phrase, is the essen. ele., of which he, the infinitive base, is mod. by good, a sim. adj. ele. of the 1st class. Of which sentence, also, etc. 30. " His being false is shameful," is a sim. declar. sen., of which his being false is the subject, of which the substantive verb being is the essen. ele. , mod. by his sjadi false, two sim. adj. eles. of the 1st class. Of which sentence, also, is shameful is the predicate, of which is is the essen. ele., and sham,eful a pred- icate adjective. 32. She is eighty years old. 33. He is not a whit wiser. 34. That he will succeed is evident. 35. I consider preaching to be teaching. 36. The mountains rose, hight above hight. 37. Changing his mind, he painted it white. Remarks : 1. See Demonstration XXX., wherein it is shown that nouns and pronouns are often the objects of adjectives and adverbs — that the nouns years and whit, in the 32d and 33d, are the objects of the adjectives old and wiser respectively. 3. For an explanation of the subordinate conjunction that, in the 34th, see page 479. 554 THE SrSTElii METHOI). 3. In the 36tli, the first word hight is nominative in ttie independent construction by ac- companying circumstance. (See page 473, h, and Demonstration XXVI., a, 4.) 4. In tlie 37tli, the participle changing is an adjectival element modifying he, the sub- ject. Diagramming : (That^ ,he L wil hight ' 34- will K succeed _ is _A evident Analysis : 35. sider J /t( above hight mountaius \ The consider J preaching to be ^-teaching 36. 34. " That he will succeed is evident," is a compl. declar. sen., of which that he will succeed is the subject of the 3d class, of which he is the subject, un- modified, and will succeed the predicate, of which vnll is the essen. ele., mod. by succeed, an infin. ele. of the 1st class. That is a subordinate conjunction. Of which sentence, also, etc. The System Method— No. 19. THE STTJIDEnSTT'S Work-Book of System Graramar, Grade B, FOR THE USE OP Public Schools, Normal Schools, Teachers' Institutes, and Colleges, AND ESrECIALLT ADAPTED TO Saiazooxj IB^^iPOsiTionsrs IN "WHICH THE STUDENT LEARNS AND REMEMBERS UNDER THE GUIDANCE OF THE "LEARNER'S WORK-TREE" BY BRANCH- ING THE WORD-PARTS OF SENTENCES THROUGH THEIR CLASSES, BRANCHED TOGETHER AS CLASS-BRANCHES OF A GRAMMAR- SYSTEM WHOLE. '>m^^T- BY ISAAC ELDRIDGE WILSON, INVENTOR OP THE LEARNER'S ■WORK-TREE, AUTHOR OP THE SYSTEM METHOD, A SERIES OP TEXT-BOOKS FOR SCHOOLS, A SERIES OP STUDENT'S WORK-BOOKS, ETC. •THE MAN MAY TEACH BY DOING, AND NOT OTHERWISE."— EMERSON. CHIOA.OO, Ilvlv.: Learner's Work-Tree Company. 1SS6. ^0 ^HB ^BagHBf^. The lessons to be assigned to primary or intermediate classes doing the work of this (No. 19) work-book are given in Appendix A. The particulars to be assigned to and used by higher classes, those doing the work of Work- Book No. 20, are given in "The System Method," No. 6, which is a text- book of higher grammar. This text-book (No. 6) will also provide all helps — definitions, explanations, lists, statement of principles, etc. — that the teacher may need to enable him to direct either his higher classes through the work done in No. 20, or his lower classes through this work of No. 19. And since every difficulty that can perplex teacher in directing, or student in doing, this work is somewhere fully solved in^both Work-Book No. 20 and the advanced text-book (No. 6), it is necessary to give in this (No. 19) work-book only a few examples, or particulars, under each program. For any difficulty that may arise in teaching this primary and interme- diate work of No. 19, if that difficulty is not explained herein, refer to Work-Book No, 20 and to " The System Method," No. 6. CoPTKiGHT, 1886, BT I. E. WILSON. All Rights Reserved. Any Infringement will be prosecuted to the full extent of the Law. [556] SYSTEM GRAMMAR— GRADE B. 557 (a.) Propositions. — To hasten the development of the perfect method of learning and teaching the science system of grammar, the author of "The System Method" challenges any educator of the United States, Can- ada, England Germany, or France to prove either of the following propo- sitions: — 1. That this system method is not a better method than he himself uses. 2. That this system method is not the true method. (b.) Conditions. — 1- Such educator shall compress his argument mio not to exceed 150 8vo. pages of a 200-page discussion. 2. The author of "The System Method" to have the last 50 pages of such 200-page discussion, in vfhich to reply. 3. The decision to be made by five judges, three of whom shall be .Tames Russell Lowell, of Boston, A. Wilford Hall, of New York, and the State Superintendent of Indiana; these three to choose a fourth and a fifth. 4. The judges shall make no qualification, but render their decision af- firming or denying the propositions as they stand. 5. The expression, "this system method," of the propositions, to be construed to mean that method of learning and teaching the science system of grammar exemplified in these (ISTos. 19 and 20) Work-Books, and advo- cated in this Volume I. of "The System Method," together with the au- thor's 50 last pages of the 200-page discussion. 6. Such educator shall make no use of the Learner's Work-Tree or any counterfeit in establishing any argument. (C.) Offer. — 1- As a summons to earnest exertion, the author of "The System Method " offers to print and deliver to such educator, for his own disposal, and free of all cost to him, 1,000 volumes of such 200-page discus sion, neatly bound in cloth. 2. The period of this offer extends from July 1st, 1886, to July 1st, 1888. 3. Such educator shall prepare his 150 first pages in proper form to be examined by the judges, in MS. or print, as he may choose, Appendix 4- Lessons to be Assigijed. -7^®^-''^^.'-^®^' Prog. I. — Jdhn,^ grammar,^ birds,^ St. Louis,* horses,^ stillness,® Washington,''' health,^ dog,^ countries,^'* life,^i Milton,!^ minds,^^ bonnet,^* author,i5 wolves,^® Susan,''' president,!^ Chicago,'* Job,^** goodness,^' waters,^^ Johnson, ^^ croquet,^* empire,^^ winter,^® Italy,^''' cowardice,^* Shakspeare,^® companies, ■'•' class,^' Cincinnati, ^^ meek- ness,33 machine,^* Irishman,^^ EngUshmen,^® Webster, 3''' Isabella, ^^ Scotland, ^^ coffees,*" rashness,*' school.*^ Prog. II. — ^Foxes,' man,^ fans,^ pan,* men,^ Csesars,® mallets,''^ baths,^ tooth,' Lincoln,"* mice," children,'^ ox,'^ paleness,'* Ptolemies,'^ teeth,'® San Francisco,''' beliefs,'* theories," Maries,^" hoes,^' task,'"'^ judgment,^^ Hudson,^* feet,^^ river,^® houses,^'' Indianapolis,^* author, ^^ goose,-'" driness,^' Harry, ^^ shiness,^^ negroes,^* Lucases,^* Mackintoshes,^® country, S'' Rufuses,^* Ohios,^* Sicilies,*" France,*' violence, *'^ ladies,*^ loaves.** Prog. III. — Sarah,' pencils,^ sugar,^ work,* softness,^ woman,® ^^''iinom 7 ,r,vi landlord," Nancies,'" boxes," uncles, '^ animal,'^ enemy,'* Wednesday, cousin,'* feet,'" Fanny,^" delays,^' mother, ^^ others,^^ Kentucky,^* E TT : 9.7 ,•„! — J !>S f_,- -I 9Q _:„4... Sft i„_ji_,i.. SI _;„4. S2 i, ^.^ ate,*" flew,*' got,*^ goad,** hang,** hung,*' creep,*® cringed,*^ crying,** chosen,*" get, proven,^' proved, ^^ spell, ^^ spill,^* gave,^^ read, 5® crow,^^ cut, 5* knit,^" lie,®" lay, hewed, ®2 latch,®* leap,®* look,®^ learned,®® lied,®^ light,®* may,®" must,''" paid,'" ought, might,''* quitted,^* riding, ''^ rung,'® seen,''' should,'* see,'" shone,*" shining,*' shoeing, show,** sat,** shut,*5 written,*® spun,*' taught,** work,*" swimming,"" swung,"' swear, swarm,"* will,"* wax,"^ waked."® Prog. V. — Chide,' forgotten,^ must,* mocked,* set,^ sat,® woven,'' wed,* wish," stay,'" wet," owe,'^ ought,'* frozen,'* choose,'^ chew,'® cleaving,''' clashed,'* gave,'" beseech, 2" hanging,^' read,^^ broken,^* built, ^* digging, ^^ could, 2® done,^'^ hidden,^* knelt, 2" teaching,*" saw,*' sawn,*^ seeking,** sevnng,** search,*^ risen,*® swimming,*'' teach,** knitting,*" did,*" flown,*' flowed,*^ met,** wasted,** wished, *'5 passed,*® pull,*'' put,** storm,*" rode,-^" shall, 5' spoil,^^ swear, 5* rowed,^* waxing, ^^ forbear,^® whetted, ^^ wept,^* wrote,^" wrought,®" singing,®' fallen, ®2 must,®* can,®* whet,®^ wilt,®® win.®' Prog. VI. — I wish ' to see ^ him before he goes * to the city. Mr. Russell goes * to Chicago next week. To-morrow is^ Thursday. If I had® a pen, I would'' write.* Cities were" forming.'" He was" speaking. '^ The clerk is '* reading '* a decision, rendered,''^ written,'® sealed,''' and delivered'* to him by the judge. When the train [558] 8T8TEM GRAMMAR— GRADE B. 559 arrives,!^ we are^" to start.^^ Columbus discovered ^^ America. When the mail arrives,23 we will ^* hearts the news. The pupils did ^^ succeed. 2'' Venus can ^^ be ^^ seen ^^ to-night. I shall ^^ go ^^ when he comes. ^^ The assassins wiU ^^ be ^^ hanged. ^^^ If the storm had'^''' not abated,^* the ship would ^^ have^" heen^i sunk.*^ The grain should *3 have** been*^ gathered*^ before the rainy season began. *^ The laws shall** be*' obeyed. 5*^ The assassins, having ^^ been^^ condemned,^** were^* executed. ^5 Cease, 56 then, to do ^7 evil. The lion was ^^ killed. ^9 I had ^o better go.^^ The child may ^^ have ^^ fallen 6* into the well. Dr. Livingston has ®^ explored ^^ a great part of Africa. The ship sails 6''' next week. The boy is** punished.*' Henry could™ speak '^'^ fluently. Mary would '''^ go "^^ if she had '^* the means. Prog. VII. — For gi-aded particulars to be classified by this program, refer to "Particulars," under Prog. IX., of "The System Method," No. 6. Since proper words to be assigned are there given, they are here omitted. Prog. VIII. — For proper words to be worked up by this program, see "Particu- lars under Prog. X., of " The System Method," No. 6. Prog. IX. — For graded particulars to assign to your classes, see "Particulars," Prog. XI., of "The System Method," No. 6. Prog. X. — He was ^ ordered ^ to go. ^ If he go,* say^ good-by. I can* swim.'' We saw* him fall.' I remember,!" 'jyjien you visited 11 us, we had^^ just built ^^ our new house. We believe 1* diligent study wiU ^^ bring 1* a rich reward. Peace may 1^ be 1* restored 1' in this way, but other measures might ^o have^^ succeeded "^ better. He stood -3 gazing 2* into the abyss. The sound of a shaken ^^ leaf shall ^* chase ^'^ them. The way to avoid ^* bad company is^' to find^" something to do.^^ We could ^^ not avoid 33 giving ^* offense. Here I will*^ stand 3* tiU Caesar pass 37 along. The thieves, having 3* hidden*' their stolen*" treasure in the ground, hid*i themselves in the wood. If he was there, I did *2 not see *3 him. If he ever came,** we knew *5 it not. If ever he come,** we shall *7 know** it. If it rains,*' why do s' you go ^^ out? We found 5^ the water frozen. 53 If to-morrow be 5* fine, I will^^ go.^s Though this seems ^^ improbable, it is 5* true. Take 5' heed that thou speak *" not to Jacob. Prog. XI. — Draw^ a pendulum to a given ^ point on one side, and it will* swing* to a corresponding 5 point on the other side. Happiness be* yours. I care^ not whether it rains* or snows.' Unless thou yield,!" this sword shaUii end^^ thee. Lest thou lose !3 thy health, govern 1* well thy passions. I ought ^^ to have^* gone.^'' I am !* pleased !' to have 2" met^^ you. We expected^ that he would ^3 arrive^* last night. He would^5 uot have dared^* to do^^ tiiat. Man is 2* born 2' to die. 3" Do*! you hear 32 the children singing? 33 Go 3* not, lest thou perish. 35 Exercising** the mind develops 3'i' business power. Bills are ** requested 3' to be *" paid *! in advance. Unless he return *2 soon, he may** be** locked *5 out. Complain** not of hard les- sons, but overcome *^ difficulty by hard study. Though he ventured ** far, he was *' not injured. 5" Having 5! borne 5^^ the burden of the day, he welcomed 5* the night. Rejoicing 5* In hope, we press 55 forward. By conquering 5* self, we conquer 57 all things. If you begin 5* a thing, push 5' it through. To have *" learned *! to restrain *2 passion is ** better than to be ** a king. Though having *5 been ** overpowered, *7 yet he stands ** facing *' the foe. Prog. XII. — We^ shall see him^ when he* returns. What* does he 5 want? We * thought it 7 to be them.* We ' took it i" to be the man who ^^ had gone before 560 THE SYSTEM METHOD. us.^^ Whom 13 (Joes he^* honor more than me? ^^ It ^^ is well with him^'' who ^^ dies a true Christian. This same child is he^* who^'^ reigns. Who^^ should be king save him^^ who 23 makes us^* free? I^^ that^® speak to thee^^ am he.^* The apples which 28 you 3" gave me ^i were sour. He ^^ that ^3 getteth wisdom loveth his 3* own soul. He 35 himself 36 performed the task. They 37 themselves 33 will go. Thou 39 crownedst him *" with glory and honor. My^^ duties to them^ require my *3 -Krhole attention. We ** desire not the knowledge of thy ^^ ways. He ^^ who *'' lives carelessly will die with regrets. Who ** shall separate us ^^ from the love of Christ ? Who ^^ is he^i that ^^ can change a hair of his53 head? His ^ story interests them; ^^ his ^^ nar- row escapes excite their ^^ surprise. Prog. Xni. — Who ^ gave thee^ that authority? 3 Mary* bought the apples,^ and gave them 6 to her ^ sister.* The book^ that i" lies on the table ^^ is mine.^^ \Vhomi3 did you i* suppose me ^^ to be? Only they ^^ who '^'' are temperate are happy. Who ^^ found the man ^^ that ^o stole the horse? ^i They ^^ who ^3 are temperate in all things 2* live long. The Hon ^5 that "^ you ^7 saw, killed his ^3 keeper.^^ The boots 3" which 3i uncle 32 gave me,33 1 34 gave my 35 brother. 36 What 37 have 1 38 to do with thee? 39 I lo myself *i know who*^ stole his*3 purse.** We*5 know whose *^ house*'' was robbed. You *3 have guessed which *9 belongs to me.^'' What 5i was said, and who 52 said it? 53 When Waterloo 5* had been lost, Napoleon 55 gave himself 56 ^p iq England. 57 Prog. XIV. — He ^ thanked 2 us 3 for saving * his 5 lif e.^ Time "^ never ceases * fly. ing.9 Please ** to give ^^ us ^^2 bread ^3 to eat.** Man *5 is i6 born *'' to die.** The beef 19 was 20 ordered 21 to be 22 fried.23 Let 2* him 25 be 26 Caesar.27 Mr. Jones,28 the banker,29 has 3" gone 3* to Europe. 32 Adolphus,33 the heroic lad,3* saved 35 his 36 drowning 37 brother, 3* Augustine. 39 It *" comes ** from Mr. Carpenter, *2 my *3 friend.** CEesar,*5 having *6 crossed *''' the Rubicon,** gave *9 battle.5'' 1 5i see 52 the sparkling 53 bubbles 5* swim. 55 156 consider 57 preaching 5* to be 59 teaching. 60 I6I bear 62 the bluebird 63 prophesying 6* spring. 65 Jane 66 drew 67 a bucket 6* of water 69 from the well.'''* She'''* was''2 j][amed''^3 peupjope.'^* The convention '''5 was''6 generally consid- ered '''^ to be ''^ a failure. '^8 London 36 is the largest city *! in Europe.32 He 33 caused ** himself*^ to be 36 proclaimed 37 king. 3* They 39 elected 96 him 9* president.92 He 93 made 9* the log 95 a boat.96 Though he ^^ slay 9* me, 99 yet will *<"> I *6* trust i»2 him.i03 They *»* found i<'5 the party *66 to be i6^ Henry, i"* God *69 caUed **« the firmament "* heaven. 1*2 A better proof **3 of his *** being "5 a rogue **6 is "^ not desired.*** If he **9 steal 120 aught, 121 1 122 wiU 123 pay i2i the theft. i25 if he *26 be 12^ better, you 12* may i29 stay *30 with him 13* an hour.*32 1 133 should *3* laugh i35 to cry.i36 it i37 ^^as i38 to be i39 his 1*0 horse.*** If it 1*2 be i*3 not now, yet it *** wiU **5 come.**6 He i*^ must *** fall i*9 except God *5» be 15* with him.*52 That book *53 is hers,*5* not yours.*55 The potatoes *56 measured *57 a bushel.*58 1 159 consider *66 Dr. Johnson *6* to be*62 an excellent man.*63 If he *6* does *65 not hear,*66 1 167 am *6* mistaken.*69 Truth,*''^ crushed *'''* to earth,i72 shall *''3 j-ise i74 again. By singing,*'''5 birds ^''^ delight ^'''^ ms™ If your *'''9 messenger**" find*** him**2 not there, seek**3 him*** in the other place **5 yourself. **6 So man**^ lieth *** down and riseth **9 not : till the heavens *96 be no more, they*9* shall *9* not awake 19* nor be raised i9* out of their *95 sleep.*96 if a *97 man i98 die,*99 shall 200 he 201 Mve 202 again ? Let 203 the angels 20* be 205 our 206 guides.207 Prog. XV. — A* beautiful 2 white 3 horse; black* squirrels; three 5 sad 6 lives; Brussels ''' carpet ; the * same ^ man ; reverent,*" cautious,** and valiant *2 Themistocles ; SYSTEM OBAMMAR— GRADE B. 561 large, ^3 fine^* apples; heavy ^^ guns; redi^ and blue^'' tape; the^^ old^' historic^" oak; five^^ large, ^^ ripe,^^ luscious, 2* mellow ^^ peaches; immense ^^ suspensions^ bridges; many^* intelligent*^* American ^^ citizens; a^i large, ^^ rickety, ^^ wooden 3* building; the ^^ first, 3*' second,^''' and third ^^ houses ; Which ^^ man and what ■*•' thing? They loved each *i other ; ^ Jamaica ^^ pepper ; those ** houses yonder, *5 adjacent *^ to the *^ common ; Arabian ** horses ; his own *' brother ; this 5" very ^i man ; nobody else;^^ wild^s geese; several ^^ sour,^^ unripe, 56 Isabella ^^^ grapes; broad, ^8 green ^9 pastures ; those ^^ brilliant, ^^ crimson ^^ clouds ; these ^^ five ^* modal ^^ adverbs ; that ^® thin, ^^ fleecy fi* cloud; two ®* busy ^^ actors ; every ^^ severe ^^ affliction; much'''^ infe- rior ''* grain ; the ^^ fifth "^^ military '''' company ; an ''^ older ''^ son ; the ^^ fourth *^ gloomy 8S day ; many ^3 happier ^^ families ; each ^^ flowery *^ field. Prog. XVI. — ^There^ they lived very^ happily.^ He stutters unavoidably.* The swallow darts continually,^ here •* and there.'' How* often * have you been there? ^^ Man is fearfully ^i and wonderfully ^^ made. Sometimes ^^ Henry studies very 1* weU.^^ Where ^® does he live? — There.^'' The ^^ oftener ^^ I see him, the '** more ^^ I like him. He is very '-^2 easily ^^ offended. Be more^* cautious. How^^ and where ^® shall we find him? Study more ^^ earnestly. ^8 The men toil early ^9 and late.^" The pendulum swings to 31 and fro.^^ We poor puppets are tossed pell-meU^s together ^ in the grave. Loud 35 and long 36 the piper blew. It is pretty 37 nearly 33 finished. He died long 39 ago.*** We came much *i sooner. *2 He is undoubtedly *3 and emphatically ** an ear- nest man. Even *5 then *^ the morning cock crew loud.*'' I am surely ** not ** deceived. Elsewhere ^^ the storm raged terrificaUy.5i I wind about 5^ and in 53 and out.5* The ship is, accordingl3',55 not 56 to sail. Somewhere 5^ he will certainly ^3 succeed. They rushed 59 forward headlong**' down the stairs. Pkog. XVII. — They always ^ Uved together ^ very 3 pleasantly.* Those 5 more® cowardly '' men wiU flee away. 3 The book is where® you laid it. Where ^^ did you lay it? The next 11 neighbor's apples are ripe.12 He seldom ^3 goes home sober. 1* I know when ^5 he wrote the letter. It was there ^^ when i'' I saw it. When ^3 did you see it? He remains while ^^ I am gone. Never '^^ two ^^ loved as '^ they do. Where ^3 the^* tree falls, there ^5 it -^viu Ue. He certainly ^6 behaves badly.^^ Dark,^* heavy ^^ clouds are slowly 3" gathering above. 3i Those 3- brilliant 33 crimson 3* clouds will very 35 soon 36 dissolve. There ^"^ were six 38 extraordinarily 39 large *** houses. You will un- doubtedly *i be very ^ cordially *3 welcomed. The HeU-Gate ** reef was slowly *5 drilled away.*^ A*'' furious *8 equinoctial *9 gale nas just 59 swept by.5i We now^^ travel still53 more^* rapidly. 55 The 5<> first 5'i' three precious ^8 morning 59 hours were conse- quently *" wasted. A ^^ somewhat *^ dangerous ®3 pass had been reached quite ^* unex- pectedly."^ This "'» pecuUar ^^ nerve passes to the "8 back of the ''9 eyeball, and there "'^ spreads out. '1 Education implies the fulF^ ^nd harmonious ''3 development of aU'''* the '^5 faculties. 7® The '^ Dead ^3 gga, dark '9 and misty 39 and almost 3i sacred, 82 is seen from the 33 Mount of Ohves. Some 3* men sin deliberately 85 and presumptuously. 36 A 8'!' few 38 old 89 buildings stiH 99 stand in the 9i deserted viUage. The 93 old 93 oaken 9^ bucket stiU ^^ hangs in the 96 weU. Few 9^ honest, 98 industrious 99 men fail of success in life. Do not i<"* use adverbs needlessly. i9i Prog. XVIIL— He^ went 2 with me 3 to the* fields,5 and there 6 we '^ filled 8 our 9 baskets ^9 with berries.^i Philip ^^ and Thomas ^3 went 1* to the i5 city ^^ together,^'' but each '3 came ^9 back ^9 by himself. ^i I "" wiU^s get 2* the ^5 parcel s" if he ^^ sends^* 36 562 THE SYSTEM METHOD. it.~9 The man ^^ had ^i great ^3 sorrow ^^ at the ^ news ^^ of the ^^ death ^^ of his ^^ son.39 The *" vine *i is ^'^ full ^^ of grapes/^ and the *^ tree *6 is *7 laden ^^ with plums.*^ Demosthenes s" and Cicero '^i were^^ celebrated ^s orators.^* Honest^s industry's generally 57 insures 's success.'^ When'^o soldiers ^^ disobey ^2 the ^^ captain,^* he ^^ punishes «« them." The ^^ girl 69 forgot "'^ the '^^ new ^'^ book ; ''^ so she ''^ ran ^^ back^^ for it.''''' Lucy ''^ wet ''^ her *<> feet ^^ in crossing ^^ the ^^ brook ; ^i she ^^ should ^^ not ^'' have 8S attempted 8^ to cross «» it 9i until she 92 had 93 learned 9* its 95 depth.96 Emma ^^ said 98 the 99 bag^^o was "^ hers;i03 she^os had"* bought "5 jt.ioe The "'^ other "S girls 1"* said 110 it"i was"^ theirs."^ Elizabeth's"* opinion ii5 igiie wrong, ii'' if she "8 thinks "^ my ^^o father i^i is 122 not i-'3 a 124 good i25 man.i26 gome i27 pupUs' ^28 attention i29 is iso easily i^i diverted ; i»2 they ^^s aUow ^^ it iss to wander "6 on the ^^T sUghtest 138 occasion.139 Do 1*0 you 1*1 cultivate 1*2 thei*3 habit 1** of close i*5 atten- tion ?"S Being 1*^ new 1-8 and strong,i*9 thei^o carriage I'l wasi52 roti^s broken.i'* They i^' found i'^ in it i'^ sixteen 1^8 eggs.i59 Prog. XIX. — He walks up 1 and down 2 the street. Few honest, industrious men fail of 3 success in * life. Cotton is raised in ^ Egypt, in ^ India, and in ^ the United States. She gazes at 8 him from 9 the window. They labor from i" morning till n night. He is devoted to 12 study. He has gone to i^ Pensacola. Man is born to 1* die. The book lies before i' him on i^ the table. By " so doing, he gained the friendship of I8 his enemy. He walks with i9 his hands in 20 his pockets. He went with 21 me to 22 the fields, and there we filled our baskets with 23 berries. The m"an went by 2* crooked paths ; through 25 them he finally escaped. A fresh stream of 2^ water rims with 2'' swift- ness by 28 the house through 29 the woods into 30 the river. John Simpson came from 3i New Brunswick to 32 Princeton, in 33 a wagon, with 3* a bag fuU of 35 apples. The way from 36 home to 37 school is one mile in 38 length. The stars at 39 night give Ught to ** travelers on*i the road. The leaf of *2 some kinds of *3 plants is good for** the cure of *5 disease. A bird was sitting on *6 a branch of *''' a tree with *8 a twig in *9 its mouth. Sarah went to 50 church in 5i the morning and to 52 school in 53 the after- noon. I saw a squirrel on 5* the fence, eating nuts with 55 his teeth. After 56 drink- ing water in 57 the brook, the ox went into 58 the meadow. The man and woman lived in 59 a cottage at 69 the foot of 61 the hiU, by 62 a stream of 63 water. Pkog. XX. — Laura wiU go, but 1 Alice wiU stay. Virtue is praised and 2 neg- lected. Neither 3 George nor * James will go. Either 5 George or 6 James will go. He is happy because 7 he is good. I wiU stay unless 8 you wiU go. Phihp and 9 Thomas went to the city together, buti'^ each came back by himself. I will get the par- cel if 11 he sends it. He is either 12 a knave or i3 a fool. Ask James if 1* his is ready. I care not whether i5 it rains or 16 snows. I fled because i7 I was afraid. You may remain till 18 I return. Take heed lest i* ye fall. James will go if 2" you call. I fear that 21 1 shaUfail. MiUions of spiritual creatures walk the earth unseen, Both 22 when we wake and 23 when we sleep. — Milton. Some murmur when their sky is clear And 2* wholly bright to view. If 25 one small speck of dark appear In their great heaven of blue. — Trench. SYSTEM GRAMMAR— GBADE B. 563 He came after ^^ you left. They kneeled before ^^ they fought. Warren is both "^ loved and ^^ respected. He can never be a scholar ; for 2" he will not study. Since ^^ he must stay, I wiU go. I had not a friend or ^^ a toy, but ^^ I had Aladdin's lamp. The wind blew, but ^ the house stood firm. The vase fell from the mantel, yet ^^ was not broken. Tou love him because ^^ I do. If ^^ I were not Alexander I would be Diogenes. Prog. XXI. — Alas ! ^ I do not know. Pooh ! ^ I do n't care. Hist,^ be silent. Alas,* the way is wearisome and long. 0,^ ye simple, understand wisdom. Humph ! ^ such pretense ■will furnish a very poor excuse. Ah ^ me! that I the judge's bride might be. Hah ! * it is a sight to freeze one ! Ah ! ^ ha ! i" you thought me bUnd did you? Eh ! i^ are you sure of it? Pshaw !^^ what a blunder! Long Uve Lord Robin! huzza I^^ huzza !i* Alack,!^ what trouble was I then to you! Alas,i^ poor Milan! 0,^^ woe the day! Heigh, ^^ my hearts! Oh,i^ that you bore the mind that I do! Ho,^" awake! Lo,^^ how he mocks me! O,^^ look, sir, look, sir! Ha,-^ ha! 2* what things are these, my Lord Antonio 1 Tut!^^ our horses they shaU not see. Heigh-ho ! ^® if it be not four o'clock, I am mistaken. Zounds ! ^^ wiU they not, rob us? Prog. XXII. — States rise, flourish, and decay. Men, women, and chil- dren stare, cry out, and run. The horses and the cattle were fastened in the same stables, and were fed with an abundance of hay and grain. Spring and summer, autumn and winter, rush by in quick succession. The Hebrew is closely allied to the Arabic, the Phoenician, the Syriac, and the Chaldee. Laura will go, but Alice will stay. The robin and the wren are flown, and from the shrub the jay, And from the wood-top calls the crow, through all the gloomy day. He is ready to give assistance to whoever gives assistance to him. I am pleased with what he has done. The dog in the manger would neither eat hay himself nor suffer the ox to eat. John the Baptist came neither eat- ing bread nor drinking wine. A wise son maketh a glad father, but a fool- ish son is the heaviness of his mother. He not only preached this doctrine, but he also practiced it. The Alps, piled in cold and still sublimity, are an image of despotism. No scene is continually loved except one rich by joy- ful human labor. All those things for which men plow, build, or sail, obey virtue. All night the dreadless angel, unpursued, Through heaven's wide campaign held his way: till Morn Waked by the circling Hours, with rosy hand Unbarred the gates of light. — Milton. The ship which sailed yesterday was laden with salt. The lady that you met has gone to Nashville. She sat under the tree and read the book. Alexander and Caesar were great conquerors. I have read the book which you lent me. He is the man who stole your purse. That is the boy that stole the apples. He who made the world now governs it. Franklin, who 561 THE SYSTEM METHOD. afterward became a distinguished statesman and philosopher, learned his trade when he was a boy in the printing-ofifice of his brother, who was a printer in Boston. Prog. XXIII. — Csesar might ^ have^ been ^ crowned.^ How old must ^ a man be ^ to be ^ a bachelor ? He is ^ anxious to learn ^ how to draw.^<* Man is ^^ born ^^ to die.i3 They made ^* him laugh. i^ The sun seems ^^ to be" rising.i^ Stars rose^^ and set.^" The patient will^^ soon have^^ become ^^ strong. The liquor is ^^ fer- menting. ^^ When the work has'''® been ^7 finished,-* the laborers shall '^^ be^** paid.^i The slowly falling ^^ snow is^^ melting. ^^ The eggs were^^ ordered ^^ to be^''' fried.^* All were ^^ sealed ^'^ with the seal which is ^^ never to be ^^ broken. ^^ If you are ** right, I am*5 mistaken.*® He will*^ maintain** his integrity though he lose*^ his estate. If hebe^*^ hearing ^^ his class, do^^ ^ot interrupt ^^ him. I am 5* going ^^ to write. 5® I am^^ on the point of writing.^* Let ^^ us be®" going. ®i If he deny ®2 the charge, I will ®^ prove ®* it. If he denies ®5 the charge, why does ®® he not prove ®'? it? If he is ®* better, I have ®^ hope of his recovery. If thou read ''" this, thou mayst ''^ live ^^ If thou dost'''^ not, he will.'^* The practice of stating '^^ reasons in advance can ^® not be '''^ too strongly insisted ^* upon. I see''^ the sparkHng*'' bubbles swim.^i Bills are*^ requested*^ to be ** paid ^^ in advance. I do *® not like *'' her singing.** I was*^ opposed 8" to his writing ^^ the letter. My son is^^ going''* to be^* married. ^^ If hishe.id ache,^® do^^ not ask^* him to go.®^ Prog. XXIV. — Earth ^ praises God^ with a thousand voices.* He* is a man^ who® understands his'^ business.* Who^ is here? Who,'^^ whicJi,^'^ what,'^^ ihat,^^ and as ^* are called relative pronouns.^^ He^® is a prince i'' among princes.^* A pro- noun ^^ used as the subject^" of a proposition^^ must be in the nominative case.^^ Whence art thou 1^^ Mother 2* said she ^^ would propose a plan.^® She ^'^ laid the . plan^* before the children ^^ themselves,*" and suggested the following Wednesday*^ for a holiday*^ on the river.** Her** mother *° was engaged that moment*® in teach- ing her*^ music.** Let him*" be Csesar.*" They*^ found the party *2 to be Henry.** The possessives ** are formed by adding the apostrophe *5 to the nominatives.*® John- son,*^ the lexicographer,** was a very eccentric man.*" He 5" made me ^^ a coat.^^ They hated him 5* as being an enemy.^* Massachusetts ^^ censured her ^® ablest statesman,^'' Charles Sumner.^* It 5" fell through the air®" to the ground. John®^ is the name®^ that ®* he ®* answers to. Vice ®^ emits a breath ®® every moment.®'^ I ®* did not hear of his®" becoming a convert ''" to that faith.'^^ I,''^ thy father-in-law^* Jethro,''* am come unto thee.'^ The word''® philosopher''"'' signifies "lover''* of wisdom."''" They*" chose her*^ queen.*^ I** pitied the proud son** returning a beggar. *° He*® made me *^ a suit** of clothes.*" They"" did not esteem themselves "^ men."^ It"* fell a great distance."* They "^ stayed until night"® under shelter."'' Prog. XXV. — They ^ paid him^ money.* They * called him ^ a hero.® I '' love thee,* old England." Nouns ^" and pronouns ^^ have three cases,^^ — the nominative,^* the possessive,^* and the objective.^^ The fur^® which i'' the monarch i* wears once warmed a bear.^" Behold a king 2" here, him^i whom^^ all men^* praise. He 2* was a king 25 that 2® loved his^^ people. 2* The work 2" of the school *" fell into the hands *i of his*2 son,** Charlie Flower.** Tou*^ should ask the gentleman*® his*'' name.** A life*" of virtue*" is a life*i of happiness.*^ Socrates's** teachings** were in ad- SYSTEM GBAMMAR— GRADE B. 565 yance*^ of his*^ age.*^ There is no harm*^ in women's*^ studying politics.^*' Hen- ry's 5^ being a student-''^ was no reason ^^ for his^* being a savage. '^^ My^® own, your^'^ cousin's^* prayer ^^ is that you^° do it^^ not. The rifle*''-' is my^^ friend's,^* the hunter' s.^^ We,^'' the people,^^ are lovers^* of republicanism.^^ The Lord'''*' himself ^^ shall descend. Who''^ struck me?^^ James'''* is said to be a good boy.^^ The court ^^ pronounced him^'' a guilty man.'^^ We ^* can manage the business*** ourselves. ^1 The emperor ^^ Antoninus ^^ ^rote a good book.** The river *^ Euphra- tes ^^ is in Asia.*^ This is the man** that** I'** spoke of. Than^^ and as^^ are generally subordinate conjunctions.^^ He "* was paid money.^^ She *® was named Penelope.9''' I** sent my»9 friend^*'*' the book.ioi Wei*>2 forgive our^o^ friendsio* their ^^^ faults. ^'^^ The coal ^**'^ measures twenty bushels.^"* I ^"^ do not care a straw. 1^** The building ^^^ fronts two streets.^^^ Pkog. XXVI. — I read often,^ you read oftener,' he reads oftenest.^ The* beautiful^ evening^ star is sinking toward the ''' western * horizon. English* nouns have only ^•^ these i'^ two ^^ numbers. Other ^^ written plurals are irregular. i* The^* plural ^^ number of other ^''' similar^* nouns is formed in the^* same'''* way. We thought them honest.^^ He should read it again ^^ and again. ^^ Many 2* tall,^^ stately ^^ elms, graceful^''' in outline, and symmetrical^* in form, grew on the ^* lawn. In yonder** cot, the*^ village *2 maiden kept her school. How*^ wonderfully** are we made! FloAvers are peeping out*^ everywhere.*^ The*^ heart of the** solitary** little ** lad sank within him. It had once*^ known better *2 days. That** bony** tree Is a*^ buttonwood. The*^ king scowled ominously.*''' You can not** go where** we do. Delays are dangerous.^* The^^ gentle ^^ rain refreshes the^^ thirsty 5* flowers. Puss is still ^^ living, and has just^^ completed her ninth ^'^ year. Formerly,^* people traveled in four-horse ^* coaches. Then ^* men sometimes *■* trav- eled during the"^ whole ^* day across one^* prairie. That beautiful ^^ Turkish ^^ car- pet is new.*^^ A valuable^* English^* watch was stolen. Idle'''* boys seldom '^^ become useful '^^ men. Three ''* yoke of oxen were in each '^* plow. This '^^ is the same ''^ book that I had. Prog. — XXVII. — The valiant soldiers rushed into ^ the jaws of ^ death. Point out both* the nouns and* the pronouns. Do you love him because^ Idol The winds blew, but ^ the house stood firm. I thank thee for ^ thy love to * me. We must either * obey or i** be punished. He knew that ^^ he was wrong. Wisdom is better than ^^ riches. Take heed lest '* ye fall. Mary and i* Susan prefer white or ^^ blue ribbons. Except ^^ ye repent, ye shall all likewise perish. He labors ; consequently i'' he succeeds. I shall go, notwithstanding i* it rains. It is easily proven that ^* the earth is round. If ^* we follow nature, we shaU never go astray. Though ^^ hand join hand, the wicked shall not go unpunished. Unless ^^ he go, I shall remain. Unless ^* he retreat, his army will be sacrificed. Ye will not .come unto Me that ^* ye may have life. Ye receive Me not because ^^ ye know Him not. Since ^^ the soil is rich, the corn will grow. He does not deny that ^^ he borrowed the money. Virtue gives us more pleasure than 2* pain. It costs less than 2* you suppose. Ye shall be as** gods. 5f5(] TUE SYSTEM METHOD. PnOG. XXVIII. — His illness arose from breatliiug impure air. Alas ! I do not know what may happen. The slowly falling snow is melting. I am on the point of writing. I will have been gone two hours. The clouds form a curtain to the setting sun. As the sun was setting, his form ap- peared to change. It is growing cold. To wait is often tedious. To steal is base. They paid him money. How long we shall stay is not decided. The house which you painted white is sold. He stood where I could see him. Such written plurals are regular. That life is long which answers life's great end. By others' faults, wise men correct their own. If thou dost not, he will. English pronouns have no other cases. Nouns and pro- nouns have three cases,— nominative, possessive, and objective. He is a pronoun used as a subject, and is in the nominative case. Beauty's tears are lovelier than her smiles are. If one writes or pronounces in succession the positive, comparative, and superlative forms of an adjective, he is said to compare the adjective. The words less and least are often used with the simple form of adjectives to form phrases of comparison. The habit is to be acquired chiefly through imitation. He thus learns principles through applying them. If his head does not ache, you should ask him to go. If he refuses the gold, I will not send him the silver. If he is true, I have no doubt he has friends. I will trust in him, though he slay me. His dancing set us all to laughing. If he is alive, he would write to us. That thing of which something is said is always an object of thought, and must be distinguished from the word which represents an object of thought. There the wicked cease from troubling. Sweet are the uses of adversity. Your spirits are too bold for your years. He had grown to be quite elegant. Waste not a sigh on fortune changed. Two good friends had Hiawatha, Singled out from all the others. Bound to him in closest union. They intend starting to-morrow. I enjoy inhaling the crisp air. The rat- tling of musketry is said to cause horses to grow restive. We are going to try reading him to sleep. They danced themselves out of breath. SYSTEM GRAMMAR— OR ABE B. 567 Prog. I : 1. John 2. Grammar 6. Stillness 7. Washington 10. Countries 11. Life 12. Milton 13. Minds Prog. II: 1. Foxes 2. Man 3. Fans 6. Cajsars 9. Tooth 10. Lincoln 11. Mice 14. Paleness /. Noun. a. Common. I). Proper. proper, common, common, proper, common, common, proper, common. 2. Number. a. Singular. b. Plural. singular, singular, singular, singular, plural, singular, singular, plural. 3. Thing Named. a. Material. b. Immaterial. material. material. immaterial. material. material. immaterial. material. immaterial. /. Noun. 2. Number. 3. Thing Named. 4. R. for Sp. a. Common. b. Proper. common, common, common, proper, common, proper, common, common, a. Singular. b. Plural. plural, singular, plural, plural, singular, singular, plural, singular. a. Material. b. Immaterial. material, material, material, material, material, material, material, immaterial. a. Paxle . b. Irregular. rule 11. irregular, rule 10. rule 10. irregular, rule 10. irregular, rule 11. Prog. Ill ! 1. Sarah 2. Pencils 8. Sugar 5. Softness 6. Woman 10. Nancies 12. Uncles Prog. IV: 1. Abide 2. Arose 3. Asked 4. Am 8. Beginning 14. Bereft, r. 18. Buy 19. Bade /. Noun. a. Common. b. Proper. proper, common, common, common, common, proper, common. 2. Number. 3. Gender. a. Singular. b. Plural. singular, plural, singular, singular, singular, plural, plural. /. Verb. 2. Principal Parts. = verb, abide, abode, abode, = verb, arise, arose, arisen, =: verb, ask, asked, asked, == verb, am, was, been, = verb, begin, began, begun, = verb, bereave, bereft, bereft, = verb, buy, bought, bought, = verb, bid, bade, bidden. a. MaHCuline. b. Feminine. c. Neuter. d. Common. feminine, neuter, neuter, neuter, feminine, feminine, masculine. Regularity. a. Regular. b. Irregular. irregular, irregular, regular, irregular, irregular, irregular, irregular, irregular. R. for Sp. a. Rule . b. Irregular. rule 10. rule 10. rule 10. rule 11. irregular, rule 11. rule 10. 4. Tense. a. Perfect. b. Imperfect. imperfect. perfect. perfect. imperfect. imperfect. perfect. imperfect. perfect. 568 THE SYSTEM METHOD. Prog. V: /. Regularity. 2. a. Regular. b. Irregular. Principal Parts 5. Inflection. 4. Tense. a. Complete, a. Imperfect. b. Defective, b. Perfect. c. Redundant. 1. Chide = irregular, chide, chid, chidden. etc., redundant, imperfect. 2. Forgotten = irregular, forget , forgot, forgotten, etc., redundant, perfect. 3. Must = irregular, must, defective, imperfect. 4. Mocked = regular, mock, mocked, mocked, comple e, perfect. 6. Sat = irregular, sit, sat, sat. complete, perfect. 10. Stay, r. = irregular, stay. staid, staid. redundant, imperfect. 11. Wet, r. == irregular, wet, wet, wet, redundant, imperfect. 13. Ought = irregular, ought 1 defective, imperfect. Prog. TI: /. Regularity. 2. Inflection. 3. Action. 4. Tense. a. Regular. b. Irregular. a. Complete. b. Defective. c. Redundant. a. Finished. b. TJnflnisbec a. Imperfect b. Perfect. 1. Wish = regular. complete. unfinished, imperfect. 3. See = irregular, complete. unfinished. imperfect. 4. Goes = irregular. complete. unfinished. imperfect. 6. Had = irregular, complete. finished. perfect. 7. Would = irregular. defective. finished, perfect. 8. Write = irregular. complete. unfinished. imperfect. 10. Forming = regular. complete. unfinished, imperfect. 11. Was = irregular, complete, finished. perfect. Prog. VII: /. Inflection. 2. Time. 3, Tense. 4. Event. a. Complete. b. Defective. c. Redundant a. Present. b. Past. c. Future. a. Imperfect. b. Perfect. a. Determined. b. Undetermined. 1. Be = complete, future. imperfect. undetermined. 2. Is = complete, present. imperfect. determined. 3. Do = complete. present. imperfect. deternained. 4. Eead = complete. future. imperfect. undetermined. 5. Mayst = defective, present. imperfect, determined. 6. Rose = complete. past. perfect, determined. 9. WiU = defective, future. imperfect. determined. 10. Lose = complete. future, imperfect, undetermined. Prog. VIII /. Inflection. 2. Time. 3 . Event. ^, Mood. a. Complete. b. Defective. c. Redundani a. Present. b. Past. \,. c. Future. d. Equivocal. a. Determined. b. Undetermined a. Indicative. . b. Subjunctive. 1. Is = complete. present. determined, indicative. 2. Must = defective, equivocal, determined, indicative. 3. WiU = defective, equivocal. determined, indicative. 7. Exert = complete, future. undetermined, subjunctive. 8. Be = complete, present, txndetermined, subjunctive. 12. Has = complete. present. determined. indicative. 15. Desires = complete, present, determined, indicative. SYSTEM GRAMMAR— GRADE B. 569 Prog. IX 5. Is = 6. Have = 7. Be = 9. Thundered = 10. Would = 13. Had = 16. Musti = /. Inflection. 2. Tense. a. Complete. b. Defective. c. Redundant. complete, complete, complete, complete, defective, complete, defective. a. Imperfect, b. Perfect. imperfect, imperfect, imperfect, perfect, perfect, perfect, imperfect, 3. Time. a. Present. b. Past. c. Fature. d. Equivocal. present, present, future, equivocal, equivocal, present, past. 4. Mood. a. Indicative. b. Subjunctive. indicative. indicative subjunctive. subjunctive. indicative. subjunctive. indicative. 1 Though in the imperfect, or present, tense, nevertheless must refers as the present may refers to past time, in " He may have gone." to past time; just Prog. X % 1. "Was 3. Ordered 4. Go 5. Say 6. Can 7. Swim Prog XI: /. Inflection. 2. Tense. 3. Event. a. Complete. b. Defective. c. Redundant, complete, complete, complete, complete, defective. a. Imperfect, a. b. Perfect. b, perfect, perfect, imperfect, imperfect, imperfect, Determined. Undetermined. determined, determined. 4. Mood. a. Indicative. b. Subjunctive. c. Imperative. d. Infinitive. e. Participial. indicative, participial. redundant, imperfect, undetermined, subjunctive, determined, imperative, determined, indicative, determined, infinitive. /. Regularity. 2. Inflection. 3. Tense. 4. Mood. a. Regular. a. Complete. b. Irregular, b. Defective. c. Equivocal, c. Redundant. 1. Draw 2. Given 8. Will 4. Swing 5. Corresponding 6. Be Prog. XII : 1. We 2. Him 4. What 8. Them 11. Who 12. Us 13. Whom = irregular, = irregular, = irregular, = irregular, = regular, = irregular, /. Pronoun. 2. a. Personal. b. Relative. c. Interrogative. personal, personal, interrogative, personal, relative, personal, interrogative. complete, complete, defective, complete, complete, complete, Person. a. First. b. Second. c. Third. first, third, third, third, third, first, third. a. Imperfect. b. Perfect. imperfect, perfect, imperfect, imperfect, imperfect, imperfect, 3. Number. a. Singular. b. Plural. c. Equivocal. plural, singular, equivocal, plural, singular, plural, equivocal, a. Indicative b. Subjunctive, c. Imperative. d. Infinitive. e. Participial. imperative. participial. indicative. infinitive. participial. imperative. 4. Case. a. Nominative. b. Possessive. c. Objective. nominative. objective. objective. objective. nominative. objective. objective. 5Y0 THE SYSTEM METHOD. Prog. XIII : /• Substantive. 2. Person. a. Noun. a. First. b. Pronoun. b. Second. c. Third. 3. Number. 4. Gender. 5. Case. 1. Who = pronoun, 3. Authority =: noun, 7. Her 10. That 12. Mine 13. Whom 18. Man = pronoun, = pronoun, =: pronoun, = pronoun, = noun, third, third, third, third, first, third, third. a. Singular. b. Plural. c. Equivocal. equivocal, singular, singular, singular, singular, singular, singular, a. Masculine. b. Feminine. c. Keuter. d. Common. common, neuter, feminine, neuter, common, common, masculine, Prog. XIV.— Branching by Grrammatical Work-Tree. a. Nominative. b. Possessive. c. Objective. nominative. objective. possessive. nominative. possessive. objective. objective. GRAlMlVEATICAIv ^?VORK-TR.E;Ei. Copyright, 1886, by I. E. WILSON. SYSTEM GRAMMAR— GRADE B. 571 In learning a branch of learning, there are two great steps to be taken: (1.) The objects, or particulars, of which the branch treats are to be dis- triljuted into classes; (2.) The classes thus formed are to be branched to- gether into a science-system whole — into a tree^ystem unity. In these first thirteen programs we have been wholly engaged in first-step work, classifying words into nouns, pronouns, and verbs. We will now branch these three classes, including their sub-classes, into the higher branch class fundamental (see tree), so that we may have a guide by which to practice in this second step of branching words into their science system, which, as elsewhere shown, is a tree system. For proof that the science-system tree is the only true guide and instrument in learning and remembering the grammar system, and for proof of its inestimable value as such guide, see pp. 29-31, 160, 161, and read pp. 1-134 inclusive, and study especially pp. 66-78 inclusive. Prog. XIY (Continued). — Branching of Fundamentals. 1. He = word, fundamental, substantive, pronoun, personal. 3. Thanked = word, fundamental, verb, transitive. 4. Saving = word, fundamental, verb, transitive. 5. His = word, fundamental, substantive, pronoun, personal. 6. Life = word, fundamental, substantive, noun, common. 9. Flying = word, fundamental, verb, intransitive. 11. Give = word, fundamental, verb, transitive. 12. Us = word, fundamental, substantive, pronoun, personal. 14. Eat • = word, fundamental, verb, intransitive. Prog. XV : /. Adjective. 2. Class. a. Qualitative. b. Quantitative. c. Demonstrative. 1. A = adjective. quantitative. 2. Beautiful = adjective. qualitative. 3. White = adjective. qualitative. 5. Three = adjective. quantitative, 6. Sad = adjective, qualitative, 8. The = adjective. demonstrative, 10. Reverent = adjective. qualitative, 13. Large = adjective, quantitative. Pr( 9g. XTI: /. Adverb. 2. Class. a. Temporal. b. Local. c. Modal. d. Intensive. e. Causal. 1. There = ■■ adverb, local. 2. Very = : adverb. intensive. 3. HapiDily = : adverb, modal. 5. Continually = = adverb, temporal. 9. Often = ■■ adverb, temporal, 11. Fearfully = adverb. modal, 3. Accessory to What ? horse. horse. horse. lives. lives. man. Themistocles, . apples. 3. Accessory to What ? lived. happily. lived. darts. have. made. 572 THE SYSTEM METHOD. Prog. XVII. — Branching of Adjuncts, or Accessories. 1. Always = adjunct, adverb, temporal. 2. Together = adjunct, adverb, local. 5. Those = adjunct, adjective, demonstrative. 6. More = adjunct, adverb, intensive. 7. Cowardly = adjunct, adjective, qualitative. 8. Away = adjunct, adverb, local. 10. Where ^= adjunct, adverb, local. 11. Next = adjunct, adjective, demonstrative. 13. Ripe = adjunct, adjective, qualitative. Prog. XVIII 1. He = 2. Went = 4. The = 5. Fields = 6. There = 8. Filled = 12. Philip = 18. Each = 23. Will = . — Branching of fundamental, fundamental, adjunct, fundamental, adjunct, fundamental, fundamental, adjunct, fundamental. Fundamentals substantive, verb, adjective, substantive, adverb, verb, substantive, adjective, verb. and Adjuncts. pronoun, intransitive. demonstrative. noun, local. transitive. noun, quantitative. intransitive. personal. proper. Prog. XIX : /. Preposition. 2. Object. 3. Antecedent Term. 4, Object. 1. Up 3. Of 5. In 11. Till 12. To 14. To 15. Before preposition, preposition, preposition, preposition, preposition, preposition, preposition. street, success, Egypt, night, study, die, him. walks, fail, raised, labor, devoted, born, lies. a. Koiin. b. Pronoun. c. Verb. noun. noun noun. noun. noun. verb. pronoun. Prog, XX: /. Conjunction. 2. Class. 3. Parts Connected. 4. Likeness of Parts. 1. But =:eonj unction, 2. And ^conjunction, 3. Neither ^conjunction, 4. Nor =conjunction, 7. Because=conjunction, 8. Unless =conjunction, 10. But =conjunction, 11. If =con junction. a. Co-ordinate. b. Subordinate. co-ordinate, co-ordinate, co-ordinate, co-ordinate, subordinate, subordinate, co-ordinate, subordinate, (sentences), praised, neglected, George, James, George, James, happy — he is, stay — you will, (sentences), get — he sends. a. Like. b. Unlike. like. like. like. like. unlike. unlike. like. unlike. SYSTEM GRAMMAR— GRADE B. 573 Prog^. XXI ; 1. Alas 2. Pool! 3. Hist 5. O 6. Humph 7. Ah 12. Pshaw 13. Huzza 15. Alack /. Interjection. interjection, interjection, interjection, interjection, interjection, interjection, interjection, interjection, interjection, 2. Principle. principle 17. principle 17. princii^le 17. principle 17. principle 17. principle 17. principle 17. principle 17. principle 17. Prog. XXII. — Branching of Words. To the Teacher. — It has been shown (pp. 1-145) that (1.) the partially-knowing thought thinks the object to be learned through one or two or three disconnected classes; (2.) the perfectly-knowing thought thinks the object through all its classes as branched together in a tree-class system, or unit. Therefore, to understand words (of which sentences are the including wholes) perfectly, we have to branch their branches (tree branches) together into what they then form, namely, a tree-system unit : so that by thus branching word up its tree through aU its classes (branches), we may learn to think it through aU its classes as thus branched together into such a tree-class system, or unit. Therefore, as a learner's guide, the grammar tree is strictly indispensable ; and, as a process to be practiced by the learner, this branching work cannot be less indispensable. (See pp. 29-31, 160, 161, 66-78, and note to Prog. XIV.) It has also been shown that in mastering grammar we have two great steps to take: (1.) To classify the words into their classes; (3.) To branch together the classes thus formed. But doing this second step develops a grammar-system tree, each branch and sub-branch of which is a branch and a sub-branch accordingly of the tree- trunk class, word. Now, it is not the first, but this second step alone, that makes overt the science system of grammar ; therefore, without this second step, by which the system (tree) of grammar is developed, we cannot possibly understand the system of grammar ; for previous to this second step, to us such system, or tree, can have no existence. Consider what a significant fact it is that no text-book of gra mm ar now used in the schools of either America or Europe goes further than the first step ! On page 29, to show the vast importance of this second step, by which this tree guide is formed, it is pointed out that without this second step and the tree, the learner is left without that one indispensable thing, a definite, system-guided, process of work ! and is accordingly forced to the folly of trying to learn what is a branch, or tree, of learning by reading books, another's conclusions, etc. But if learning is branching, as I have repeatedly demonstrated, to attempt to acquire it by reading merely, is the same sort of sense as attempts to get any other act, as writing or walk- ing, by reading merely ! GRAlVIIVlATICAIv ^A^ORK-TREE INTERMEDIATE Copyriglit, 188G, by I. E. WILSON. [5741 SYSTEM OBAMMAB— GRADE B. 575 In "The System Method," No. 6, Demonstration XXII., it is pointed out that the only possible way we may rationally know and prove that om* language or any sentence is correct, is by means of this grammar-system tree; and in "No Branching Guide Previously Discovered," p. 80, it is pointed out that by means of the science tree of grammar we develop that sort of thinking power which is able to reason about words, in the syllogism; that is, with the best possible understanding whether such words are rightly or wrongly built in the sentence whole — ^whether our language is correct. States = fundamental, substantive, nominative. essential. Kise = fundamental, verb. , finite, indicative. And = connective. conjunction, co-ordinate. Fastened = fundamental. verb , infinite, participial. In = connective, preposition. Same = accessory. adjective, adnominal. Closely = accessory. adverb, modifying. The = accessory. adjective, adnominal. Arabic = accessory. adjective, adnominal. Prog. XXIII. , — Branching of Verbs. 1. Might = verb. finite, indicative. 2. Have = verb, infinite (, infinitive. 3. Been = verb, infinite participial. 6. Be = verb, infinite , infinitive. 13. Born = verb. infinite , participial. 13. Die = verb, infinite , infinitive. 49. Lose = verb. finite, subji Jnctive. 51. Hearing ■ = verb. infinite participial. 53. Do = verb. finite, imperative. Prog. XXIV: /, Substantive (. 2. i Case. 3. Construct ion. 4. Office. a. Noun. b. Pronoun. a. Nominative. b. Possessive. c. Objective. a. Essential. b. Accessory. c. Independent. a. Subject. b. Predicate. c. Appositive. d. Adnominal. e. Object. 1. Earth = noun, nominative. essential. subject. 3. Voices = noun. objective, accessory. object. 7. His = pronoun, possessive. accessory. adnominal. 10. Who = noun. nominative. essential, subject. 17. Prince = noun, nominative. accessory. predicate. 20. Subject = noun. nominative. accessory, predicate. 30. Theinselves = pronoun. objective, accessory. appositive. 34. Her = pronoun. possessive, accessory, adnominal, 36. Moment = noun, objective, accessory, object. 5TG THE SYSTEM METHOD. Prog. XXV.— Branching of SubstantiTes. 1. They = substantive, nominative. essential. 3. Money = substantive, objective. object. 6. Hero = substantive, objective, factitive. 8. Thee = substantive. objective. object. 9. England = substantive. nominative, Independent. 13. Nominative = substantive. objective. appositive. 17. Which = substantive. objective, object. 31. Him = substantive. objective. appositive. 34. Charlie Flower = substantive. objective. appositive. Prog. XXYI.- Branching of Accessories, 1. Often = accessory. adverb. modifying. 4. The = accessory, adjective. adnominal. 6. Evening = accessory, adjective. adnominal. 10. Only = accessory. adverb. modifying. 11. These = accessory, adjective. adnominal. 13. Two = accessory. adjective. adnominal. 14. Irregular = accessory, adjective. predicate. 30. Same = accessory, adjective. adnominal. 33. Again = accessory. adverb, modifying. Prog. XXYII.- -Branching of ConnectiTCS. 1. Into = connective, preposition. 3. Both = connective. conjunction. co-ordinate. 5. Because = connective. conjunction. subordinate. 7. For = connective. preposition. 9. Either = connective. conjunction. co-ordinate. 11. That = connective,' conjunction. subordinate. 14. And = connective. conjunction, co-ordinate. 16. Except = connective, conjunction. subordinate. 18. Notwithstanding = connective. conjunction. subordinate. Prog. XXVIII . — Branching of Words. His = fundamental, substantive, possessive. Arose = fundamental, verb, finite, indicative. From = connective, jireposition. Breathing = fundamental, verb, infinite, participial. Impure = accessory, adjective 1, adnominal. Not = accessory, adverb. modifying. Happen = fundamental, verb, • infinite. infinitive. Is = fundamental, verb, finite. indicative. Gone = fundamental, verb, infinite. participial. The System Method.-No. 20. Work-Book ol System Graininar, Grade A, FOE THE USE OP Public Schools, Normal Schools, Teachers' Institutes, and Colleges, Soi3:oonij AND ESPECIALLY ADAPTED TO El:xi=osiTioie(ik\ therefore, measles is material. oi/M»ip= mumble; therefore, ?wuffy3S=mumblings, which is immaterial acts, i" As a repeated act, wages is immaterial, but as a thing valued, it is plural ; where it may be taken in either sense, as here, call it equivocal. See previous program, note 9. i^ We have " this molasses " and " these molasses ; " molasses is common. See Dem. II., Obs. g. 12 There is no occasion to use the spelling publics; for we need no such thing. Sec note 8 to previous program. BT8TEM QBAMMAB— GRADE A. Prog. Ill : 1- Class of Noun. 2. Number. a. Common, a. Singular, b. Proper. b. Plural. 1. Parlor = 3. Spider 3. Fly 6. Horses = 8. Children = 19. Aunt = 20. Ant 2-1. Flock 29. Death 34. Captain ■ 38. Spring 40. War 41. Necessity 43. Nature 46. Procrastination 48. Summer 51. Trouble 56. Death 57. Brother 60. Labor 68. Echo 73. Truth 78. Brook 86. Virtues 87. Earth 89. Everybody 90. Fox 91. Fox 95. Cousins 96. Cousins 97. Cousins 98. Teachers 101. Spirit 109. Earth 116. Child =common, =common, =common, =common, =common, =common, =common, =common, =common, =common, =common, =proper,^ =common, =proper, =common, =:common =proper, =proper, =common, =proper, =:proper, =proper, =:commonj =proper, ^ =common, =common =common =common =common =common t^common =common =common =proper, =common singular, singular, singular, plural, plural, singular, singular, singular,^ singular, singular, singular, singular, singular, singular, singular, ^ singular, singular, singular, singular, singular, singular, singular, singular, plural, singular, singular, singular, , singular, plural, , plural, plural, plural, singular, singular, singular. 3. Gender. 4. a. Masciiline. b. Feminine. c. Common. d. Neuter. neuter, common,^ common,'^ common, common, feminine, neuter,^ common,* neuter, masculine, neuter, masculine,^ neuter, feminine, 5 neuter, feminine, common, masculine,'' masculine, feminine,* feminine, feminine, masculine, feminine, neuter, masculine,^" masculine, 1" neuter, 1" masculine, feminine, common, common, neuter, common, masculine. 581 Personification. a. Personified. b. Unpersouified. unpersonified. personified. personified. unpersonified. unpersonified. unpersonified. unpersonified. unpersonified. unpersonified. unpersonified. unpersonified. personified.^ unpersonified. personified.^ unpersonified. personified. personified. personified. unpersonified. personified. personified. personified. personified. personified. unpersonified. unpersonified. unpersonified. unpersonified. unpersonified. unpersonified. unpersonified. unpersonified, unpersonified. personified. unpersonified. llf spider andj?2/ were not here personified, they would be regarded as neuter, in accord ance with Dem. III., Obs. d, f ; also Dem. IV., Obs. a. 2 See Dem. III., Obs. f. ^The plural is flocks. *See Dem. III., Obs. e. ^See Dem. III., Obs. a, b, c, d; also Dem. III., Obs. c. sproper nouns if singular, will not take the definite the before them. ? The gender of Death is deter- mined from his and brother; see Dem. HI., Obs. d. ssee Dem. III., Obs. b. » Virtues is an individualized class ; it is therefore proper. See Prog. H., note 4. w See Dem. III., Obs. f, g. 582 THE SYSTEM METHOD. Proff. IV: /. Class of Noun. 2. Person. 3 . Number. 4 . Gender. 5. Personification.^ a. Common. b. Proper. a. First. b. Second. c. Third. a. Singular. b. Plul-al. a. Masculine. b. Feminine. c. Neuter. d. Common. a. Personified. b. Unpersonifled. 1. Nero =proper. third, singular. masculine. unpersonified. 2. Tyrant =common, third. singular. masculine, unpersonifled. 3. Children =common, second, plural. common. unpersonifled. 6. Man =common. third. singular. masculine, unpersonifled. 7. John =proper, first, singular. masculine. unpersonifled. 8. Ocean :=proper,2 second. singular, common. personifled. 9. Trouble =proper, second. singular, common. personified. 12. Strangers ^common. third. plural, common. unpersonified. 13. God =proper. second. singular. masculine. unpersonifled. 16. Naught =:common, third. singular. neuter. unpersonifled. 17. Builders =common, second, plural, common. unpersonifled. 19. Fellow =common, third. singular. masculine, unpersonifled. 20. Sea ^common, second. singular, common. joersonifled. 21. Physician =common, third, singular, common. unpersonifled. 22. Creator =common. third, singular, masculine. unpersonifled. 24. Master =common, third. singular, masculine. unpersonifled. 28. Mortals =common, first. plural. common. unpersonifled. 34. Love :=common, third. singular, neuter. unpersonifled. 41. Time =common, third. singular. neuter. unpersonifled. 42. Wrinkles =common, third. plural. neuter. unpersonifled. 44. Snow-drifts 5 =common. third. plural, neuter. unpersonifled. 46. School-boys=common, third. plural. masculine. unpersonifled. 48. Hearts = common, third. plural. neuter. unpersonifled. 52. Children's :=common, third. plural. common. unpersonifled. 54. Knee =common, third. singular. neuter. unpersonifled. 55. Depths =common, third. jilural. neuter. unpersonifled. 59. John =proper, third. singular. masculine. unpersonifled- 60. John =proper, third. singular. masculine. unpersonifled. 61. Friend =common, third. singular. common. unpersonifled. 62. Somebody =common, third. singular. common. unpersonifled. 64. God :=proper, third. singular. masculine. unpersonified. 65. Heavens =common, third. plural. neuter, unpersonified. 66. Enterprise =common, third. singular. neuter, unpersonified. 68. Man =common, third. singular. masculine. unpersonified. 71. Daughter =:commou, third. singular, feminine. unpersonified. 72. Father =common, second, singular. masculine. unpersonified. 74. Pleasure = common, third. singular. neuter. unpersonified. 75. Company ^common. third. singular. neuter. unpersonified. iSee Dem. III., c, d. 2 See Dem. III., a, b, c, d. ■*See Dem. XXIIL, note under p; but two words of different properties or not appositives must be taken separately. See "Learn- er's Process of Work Benighted," " One Verb Consists of a Single Word," and " Two Words as One," pages 377, 387, and 393. SYSTEM GRAMMAR— GRADE A. 583 Prog. V : /. Number. 2. Gender. 3. Case. 4. Case Determined by.^ a. Singular, a. Masculine, a. Nominative, a. The apostrophe. ■ . b. Substituting pronoun, c. Be and Same Meanings. substituting pronoun. ^ the apostrophe.^ be and same meanings.^ be and same meanings. be and same meanings. be and same meanings. be and same meanings. be and same meanings.^ be and same meanings. be and same meanings. substituting pronoun. be and same meanings. be and same meanings. substituting pronoun. be and same meanings. be and same meanings. be and same meanings. be and same meanings. be and same meanings. substituting pronoun. be and same meanings. substituting pronoun, after to be. substituting pronoun, the apostrophe, substituting pronoun, be and same meanings, be and same meanings, be and same meanings, be and same meanings, be and same meanings, substituting pronoun, neuter, nominative, substituting pronoun. ' Study carefully Dem. V., j, g., h, i. 2 See Dem. V., i, j. ^King means the same as the objective himself, from which it is separated by be. * Outcast and sacrifice refer to the same things as she and Lady Jane Orey; and wanders and fell are equivalent to Be words : " She wanders [being] an outcast," etc. 6 xhe student here is to determine by the principle of " Be and Same Meanings" (see Dem. V., h) that straw is objective, but he is to go no farther. Neither is the teacher, even, to talk about sfraiu being "governed" by the "finite verb" care, until after the pnpil shall have studied and learned what is meant by " governing" and "finite verbs." Let the teacher heed this remark, ejn Syntax, we shall find that dollar is object of worth, which is a preposition ; see Dem. XIX., Obs. f, g. ' Feet is object of the ad- jective high; see Dem. XXVI., b, 6, b. sxhis Hassan is the name of a name; it is therefore neuter. ^See Dem. VI., a, 1, a, b, c. loxhe predicate noun is not necessarily of the same person, number, or gender as the nominative ; but it is of the same case. b. Plural. b. Feminine, b. Possessive. c. Neuter. c. Objective. d. Common. 1. Brain = singular, neuter. nominative. 2. Devil's = singular, masculine. possessive. 3. Workshop ' = singular, neuter. nominative. 4. Money = singular. neuter. objective,^ 6. Prince = singular. masculine. nominative. 8. King = singular. masculine. ^objective,^ 9. Outcast = singular, feminine, nominative,* 11. Straw = singular, neuter. objective, 5 13. Lawyer = singular. masculine. , nominative. 15. Henry = singTdar, masculine. objective. 30. Hat =: singular. neuter. nominative, 21. DoUar6 = singular, neuter. objective,^ 23. Feet^ = plural, neuter. objective. 24. Angels = plural, common. objective. 25. Guides = plural. common. objective. 27. Caesar = singular, masculine. nominative. 37. Emperor = singular, masculine. nominative. 43. Sacrifice = singular. neuter. nominative,* 50. Question = singular. neuter. objective, 59. Feet = plural. neuter. objective. 64. Hassan = singular. neuter,* nominative. 69. Man = singular. mascuUne, objective,^ 70. Lord =: singular, masculine, nominative. 72. Silver = singular, neuter. objective. 83. Mother's = singular. feminine. possessive. 91. Change = singular, neuter. objective, 93. Light = singular. neuter. nominative. 103. Prey = singular. common. nominative. 104. Eyes = plural. neuter,i<* nominative. 108. Instrum'ts = plural. neuter. nominative. 112. Sum = singular. neuter. nominative. 113. Ciphers = plural. aeuter, objective. 116. Years = plural. neuter. nominative. 584 THE SYSTEM METHOD. Prosr. VI: /. Number. 2. Gender. 3. Case. 4. Case Determined by- a. Singular, a. Masculine, a. Nominative, a. The apostrophe. h. Plural, b. Feminine, b. Possessive, b. Substituting pronoun. c. Neuter. c. Objective, c. Be and Same Meanings. d. Common. d. Apposition. i e. After Being or To Be. substituting pronoun. substituting pronoun, be and same meanings, after being. be and same meanings, be and same meanings, after being. after to be. be and same meanings, after to be. be and same meanings, substituting pronoun, substituting pronoun, be and same meanings, be and same meanings, the apostrophe, after being. be and same meanings, be and same meanings, substituting pronoun, be and same meanings, be and same meanings, be and same meanings, be and same meanings, be and same meanings, substituting pronoun, be and same meanings, substituting pronoun, be and same meanings, be and same meanings, the apostrophe, substituting pronoun, substituting pronoun, be and same meanings, be and same meanings, be and same meanings, be and same meanings, be and same meanings, be and same meanings, substituting pronoun.^ See Dem. VI. "See Dem. 1. Pagans = plural. common, nominative, 2. Jews = plural. common. objective, 3. People = singular, common, objective,^ 4. Lawyer = singular, masculine. nominative,^ 5. Reason = singular, neuter, nominative, 6. Rascal = singular, masculine. objective,^ 8. Man = singular. masculine, nominative, 9. Englishman * = singular. mascuhue, nominative, 15. Matter * = singular. neuter. nominative, 16. Grammarian := singular, common, nominative. 17. Mate = singular. masculine. nominative. 23. Luther = singular. masculine. objective. 24. Man = singular, masculine. objective. 27. Thing = singular, neuter. nominative, 33. Rascal = singular. common. nominative, 35. Fellow's = singular. masculine, possessive. 36. Swindler == singular. mascuhne. nominative, 37. Notice = singular. neuter. objective, 47. Two := singular. neuter. objective. 53. Cakes = plural. neuter. nominative, 54. Justice = singular. neuter. objective. 57. Man = singular, masculine. objective. 59. Use = singular. neuter. objective. 61. Factor = singular. neuter. nominative. 66. Music = singular. neuter. nominative. 69. Dr. Johnson ^ = singular, mascuUne, objective. 70. Moralist = singular. mascuhne. objective, 72. Son := singular. masculine, nominative. 73. Slave = singular. masculine. nominative, 76. Corpse = singular, neuter, nominative, 77. Duke ['s] = singular, mascuhne. possessive. 78. Norfolk = singular. neuter, objective, 79. Park := singular, neuter, nominative. 80. Miles = plural. neuter, objective, 82. PhiUp = singular. masculine, nominative. 84. , Plantagenet = singTilar, mascuhne. nominative. 85. . Gods = plural, mascuhne, nominative. 90. , Springs = plural, neuter. nominative. 95, . Husband = singular. mascuhne. nominative, 96 . Sir Philip Sidney= singular, masculine, nominative. 1 See Dem. VI., a, b. 2 See Dem. VI., a. 3 See Dem. V., h. * XXIII., p and note. SYSTEM ORAMMAB— GRADE A. 585 Prog. VII : /. Verb 1. Can 6 __ verb, 3. Play^ = verb. 3. Has = verb, 4. Written 5 = verb, 5. Love = verb. 6. See = verb, 7. Shine = verb, 10. Were = verb. 11. Reading = verb. 19. Must = verb. 20. Learn = verb. 21. Hoped = verb. 22. Have = verb, 23. Gone = verb, 24. Would = verb. 25. Have == verb. 26. Believed = verb. 35. Miglit = verb. 36. Have = verb. 37. Been = verb. 38. Arrested = verb. 48. Do = verb. 49. Let = verb, 50. Go = verb. 61. See = verb. 63. Rain = verb. 78. May- = verb. 79. Have = verb. 80. Stolen = verb. 84. Having = verb. 85. Been = verb. 86. Condem'd = verb. 87. Were = verb, 101. Is^ = verb, 102. Subdued^ = verb. 2. Principal Parts.^ a. Imperfect tense, b. Perfect tense, and c. Perfect participle. can,* could, play, -ed, -ed, have, had, had, write, wrote, writ'n, irregular, love, -ed, -ed, regular, see, saw, seen, irregular, shine, shone, shone, equivocal,* am, was, been, irregular, read, read, read, irregular, must, irregular, learn,learnt, learnt, equivocal," hope, -ed, -ed, regular, have, had, had, irregular, go, went, gone, irregular, will,* would, irregular, have, had, had, irregular, believe, -ed, -ed, regular, may,* might, irregular, have, had, had, irregular, am, was, been, irregular, arrest, -ed, -ed, regular, do, did, done, irregular, let, let, let, irregular, go, went, gone, irregular, see, saw, seen, irregular, rain, -ed, -ed, regular, may, might, irregular, have, had, had^ irregular, steal, stole, stolen, irregular, have, had, had, irregular, am, was, been, irregular, condemn, -ed, -ed, regular, am, was, been, irregular, am, was, been, irregular, subdue, -ed, -ed, regular. 3. Regularity^' 4. Tense.^ a. Regular. a. Imperfect, or b. Irregular. [present. c. Equivocal, b. Perfect, or past. irregular, regular, irregular, imperfect. imperfect. im^^erfect. perfect, or past. imjDerfect. perfect, or past. imperfect. perfect, or past. perfect, or past. imperfect. imperfect. perfect, or past. imperfect. perfect, or past. perfect, or past. imperfect. perfect, or past. perfect, orjjast. imperfect. perfect, or past. perfect, or past. imperfect. imperfect. imperfect, imperfect. imperfect. imperfect. imperfect. perfect, or past. imperfect. perfect, or past. perfect, or past. perfect, or past. imperfect. perfect, or past. 1 For the principal parts of any irregular verb, refer to Work-Book No. 18, Appendix B. 2 See Demonstration VII., d. 8 gee Demonstration VII., h, i, k. 4 See Demonstration VII., j. 5 For the reason why care and play, has and written, etc., cannot be taken together as one verb, see " One Verb Consists of a Single Word," " Two Words as One," and " Learn- er's Process of Work Benighted," pp. 387, 393, 377; see also Demonstration XXIII., p. 490. 6 A verb is said to be equivocal, when it can be truly called either regular or irregular. 7 Ob- serve how hopelessly the time doctrine of tense breaks down, when it is seen that is and subdued, the one a present, the other a past, tense, are used joined right together ! 586 THE SYSTEM METHOD. Prog. VIII : /. Principal Parts. a. Imperfect tense, b. Perfect tense, and c. Perfect participle. = can, could, = play, played, played, = have, had, had, = write, wrote, written, = am, was, been, 1. Can 3. Play 3. Has 4. Written 5. Was 6. Dreamed 11. Reaped 13. Will 14. Send 15. May 16. See 18. Is 19. Died 21. Shall 22. Inform 24. Were 25. Would 26. Be 28. Should 36. Might 37. Go 38. Were 39. Disposed 40. Could 41. Ask 46. Would 47. Seem 48. Have 49. Been 2. Regularity. 3. Inflection. 4. Tense a. Regular. b. Irregular. c. Equivocal. irregular, defective, regular, complete, irregular, complete, irregular, complete, irregular, complete. a. Complete, a. Imperfect. b. Defective, b. Perfect. c. Redundant. imperfect. imperfect. imperfect. perfect. perfect, redundant, perfect, complete, perfect. = dream, dreamt, dreamt, etc., equivocal, = reap, reaped, reaped, regular, = wiU, would, irregulaT, defective, imperfect. = send, sent, sent, irregular, complete, imperfect. = may, might, irregular, defective, imperfect. = see, saw, seen, irregular, complete, imperfect. = am, was, been, irregular, complete, imperfect. = die, died, died, regular, complete, perfect. = shall, should, irregular, defective, imperfect. = inform, informed, informed, regular, complete, imperfect. = am, was, been, irregular, complete, perfect. ^ wiU, would, irregular, defective, perfect. = am, was, been, irregular, complete, imperfect. = shall, should, irregular, defective, perfect. = may, might, irregular, defective, perfect. = go, went, gone, irregular, complete, imperfect. = am, was, been, irregular, complete, perfect. := dispose, disposed, disposed, regular, complete, perfect. = can, could, irregular, defective, perfect. = ask, asked, asked, regular, complete, imperfect. = will, would, irregular, defective, pei-fect. = seem, seemed, seemed, regular, complete, imperfect. = have, had, had, irregular, complete, imperfect. = am, was, been, irregular, complete, perfect. 50. Is ^ = am, was, been, irregular, complete, imperfect. 51. Punished^ = punish, punished, punished, regular, complete, perfect. 54. Hanged = hang, hung, hung, etc., equivocal, redundant, perfect. 67. Had = have, had, had, irregular, complete, perfect. 68. Been = am, was, been, irregular, complete, perfect. iNote.—Jf the advantage, aye, the educational necessity, of treating is punished as what they are, namely, two verbs, does not at first appear plain to teacher and pupil, both are assured that it will so appear after they have made the start in this direction. Until those who have been taught from the false principle of signification to consider two words as one verb, can escape the deep and ruinous deception of that error despite their bias, mood must remain to them an unsolved problem ; and infinitives and participles they can never under- stand. By this system method, however,— of treating each word as a separate verb,— the student may not only pierce to the heart of mood, but to him what in infinitives and partici- ples is abstruse, will become intelligible, short, and simple. See "One Verb Consists of a Single Word," "Two Words as One," and "Learner's Process of Work Benighted," pp. 387, 393, 377. SYSTEM GRAMMAB— GRADE A. SST Prog. IX: / . Regularity. 2 . Inflection. 3 Event. 4. Mood. a. Regular. b. Irregular. a. Complete. b. Defective. c. Redundant. a. Determined. b. Undetermined. a. Indicative. b. Subjunctive 1. Be :=irregular. complete, undetermined. subjunctive. 2. Is =irregular. complete. determined, indicative. 3. Do =irregular. complete. determined, indicative. 4. Read ^irregular. complete. undetermined. subjunctive. 5. Mayst =irregular. defective. determined. indicative. 6. Demands =regular. complete. determined. indicative, 7. Rose :=irregular. complete. determined, indicative. 9. Will ^irregular. defective. determined. indicative. 10. Lose =irregular. complete. undetermined. subjunctive. 11. Maintains=regular, complete. determined. indicative. 12. Loses =irregular. comj)lete. determined. indicative. 13. Do =irregular. complete. undetermined. subjunctive. 14. Shall =iiTegular, defective. determined, indicative. 15. Does =iiTegular, complete. determined. indicative. 20. Gets = irregular. complete, determined, indicative. 22. Get =irregular. complete. undetermined. subjunctive. 25. Have =irregular, complete. determined. indicative. 26. Are =irregular. complete. determined, indicative. 27. Find ^irregular. complete. undetermined. subjunctive. 28. Come ^irregular. complete. undetermined. subjunctive. 29. Do =irregular. complete, undetermined. subjunctive 30. Should =:irregular. defective. determined. indicative. 31. Aches =regular, complete. determined. indicative. 32. Must =irregular. defective. determined, indicative. 33. Be =irregular. complete. undetermined. subjunctive. 34. Is =irregular. complete. determined. indicative. 37. Refuses =regular. complete, determined. indicative. 38. Will =irregular. defective, determined. indicative. 30. Refuse ^regular. complete. undetermined. subjunctive. 40. Are ^irregular. complete. determined. indicative. 41. Will =irregular. defective. determined. indicative. 42. Be ^irregular. complete. undetermined. subjunctive 44. See =irregular. complete. undetermined. subjunctive 45. Shalt = irregular. defective. determined. indicative. 46. Will ^irregular, defective. determined, indicative. 47. Pass =regular, complete, undetermined, subjunctive 48. Undo ^irregular. complete, undetermined. subjunctive 49. Follow ^regular, complete. undetermined. subjunctive 50. Set =irregular, complete. undetermined, subjunctive 51. Slay =irregular, complete, undetermined, subjunctive 52. Will =regular, defective. determined, indicative. 53. Canst =irregular, defective. determined. indicative. 588 THE SYSTEM METHOD. Prog. X: /. Regularity. 2. Inflection. 3. Time. 4. Mood. a. b. Regular. Irregular. a. Complete. b. Defective. c. Redundant. a. Present. b. Past. c. Future. d. Equivocal. a. Indicative. b. Subjunctive. 1. Is = irregular. complete. present. indicative. 2. Must = irregular. defective. equivocal, indicative. 4. Loves = regular. complete, present. indicative. 6. Will = irregular. defective. future. indicative. 7. Exert = regular. complete, present. subjunctive. 8. Be = irregular. complete. present. subjunctive. 9. Is = irregular. complete. present. indicative. 10. Shrink = irregular, complete. future. subjunctive. 12. Has = irregular. complete. present, indicative. 13. Find = irregular, complete. future. subjunctive. 14. Do = irregular. complete. future. subjunctive. 15. Desires = regular. complete. present. indicative. 16. Steal = irregular. complete. future. subjunctive. 17. Have = irregular. complete, future, subjunctive. 19. Like = regular, complete. future. subjunctive. 21. Does = irregular, complete. present. indicative. 22. Aches = regular, complete, present. indicative. 25. Is = irregular. complete, present. indicative, 26. Find = irregular, complete. future. subjunctive. 27. Eefuse = regular. complete. future, subjunctive. 28. Holdst = irregular, complete. present. indicative. 30. Please = regular, complete. future. subjunctive. 32. Be = irregular, complete. present. subjunctive. 34. Didst = irregular. complete. past. indicative. 35. Must = irregular. defective, future. indicative. 36. Have = irregular, complete. future. subjunctive. 37. Do = irregular. complete. future. subjunctive. 38. Shall = irregular, defective. future, indicative. 40. Magnify = regular, complete. future. subjunctive. 41. Cut = irregular. complete. equivocal. subjunctive. 43. Gather = regular. complete, equivocal. subjunctive. 44. Can = irregular. defective. equivocal. indicative. 45. Art = irregular. complete. present. indicative. 46. Be = irregular. complete. present. subjunctive. 48. Can = irregular. defective. present. indicative. 49. Are = irregular. complete. present, indicative. 50. Appear = regular, complete,^ present. indicative. 51. Must = irregular. defective, future. indicative. 52. , Be = irregular, complete. future. subjunctive. 53. Should = irregular. defective, present. indicative. 54. Forget = irregular. complete, future. subjunctive. SYSTEM GRAMMAR— GRADE A. 589 Prog. XI: 1. Regularity. 2. Inflection. 3. Event. 4. Time. 5. Mood. a. Regular, a. Defective, a. Determined. a. Present, a. Indicative, b'. Irregular, b. Complete, b. Undetermined, b. Past. b. Subjunctive. c. Redundance. Supposed. c. Future. d. Equivocal. 1. Have =irregular, complete, undetermined, future, 2. Will =irregular, defective, determined, future, 3. Has =irregular, complete, determined, present, 4. Does =irregular, complete, determined, present, 7. Be =irregular, complete, undetermined, future, 8. May =irregular, defective, determined, future, 9. Thundered^^regular, complete, supposed, 10. Would =irregular, defective, determined, 11. Thundered =regular, complete, determined. 12. Did =irregular, complete, determined, 13. Had^ =irregular, complete, supposed, 14. Would —irregular, defective, determined, 15. Had =irregular, complete, determined, 16. Must* =:irregular, defective, determined, 17. Were =irregular, complete, supposed, 18. Should =irregular, defective, determined, 23. Loved ^regular, complete, supposed, 24. Would =:irregular, defective, determined, 25. Loved =regular, complete, determined, 26. Did =irregular, complete, determined, 27. Were —irregular, complete, supposed, 28. Would =irregular, defective, determined, 29. Was :=irregular, complete, determined, 30. Does =irregular, complete, determined, 37. Read =irregular, complete, undetermined, future, 38. Mayst =irregular, defective, determined, future, 39. Will :=irregular, defective, determined, future, 40. Pass =regular, complete, undetermined, future, 42. Died =regular, complete, supposed, 45. Was ^irregular, complete, determined, 46. Did =:irregular, complete, determined, 51. Had =irregular, complete, supposed, 52. Should =irregular, defective, determined, 53. May =irregular, defective, determined, 54. Makes =irregular, complete, determined, subjunctive. indicative. indicative. indicative. subjunctive. indicative, equiv'cal,^ subjunctive, equiv'cal.^ indicative. indicative. indicative. subjunctive. indicative. indicative. indicative. subjunctive. indicative. subjunctive. indicative. indicative. indicative. subjunctive. indicative. indicative. indicative. subjunctive. indicative. indicative. subjunctive, equiv'cal,^ subjunctive, past, indicative, past, indicative, past, subjunctive, equivocal, indicative, present, indicative, present, indicative. past, past, present, present, past, past,* present, present, present, present, past, past, present, past, past, present. 1 See Dem. X., a. Observe tbat in this sentence it does not thunder, whereas in the next sentence it does thunder. 2 '-Equivocal " here equals " either present or future." SMark how false the doctrine that " the past tense denotes past time ; " for here the past had, as Indeed often occurs, refers not to past, but to present, time ! See " The Correct Theory of Tense," and "Learner's Process of Work Benighted," pp. 384, 377. * Though in the imperfect, or pres- ent, tense, nevertheless must refers to past time: precisely as the imperfect (present) may refers to past time, in " He may have gone." 690 THE SYSTEM METHOD. Prog. XII: /. Principal Parts. a. Imperfect tense, b. Perfect tense, and c. Perfect participle. 2. Regularity. 3. Tense. 4. Mood. ^ a. Regular. a. Imperfect. b. Irregular, b. Perfect. c. Equivocal. 10. 14. 15. 16. 17. 18. 19. 20. 21. 23. 24. 25. 38. 39. 40. 47. 48. 49. 50. 51. 52. 53. 54. 55. 56. 62. 63. 71. 72. 83. 84. 85. 86. Study =study, -ed, -ed, regular, imperfect, Will =will, would, irregular, imperfect, Improve =improve, -ed, -ed, regular, imperfect. Am =am, was, been, irregular, imperfect. Have =liave, had, had, irregular, imperfect, Told =tell, told, told, irregular, perfect, Does =do, did, done, irregular, imperfect. Hear =liear, heard, heard, irregular, imperfect, Am =am, was, been, irregular, imperfect. Mistaken =mistake, -took, -taken, irregular, perfect. Might =may, might, irregular, perfect. Have =:have, had, had, irregular, imperfect. Been =am, was, been, irregular, perfect. Elected =elect, -ed, -ed, regular, perfect. Flee =flee, fled, fled, irregular, imperfect. Is =am, was, been, irregular, imperfect. Sail =sail, -ed, -ed, regular, imperfect. Go =go, went, gone, irregular, imperfect, Bid =bid, bade, bidden, irregular, imperfect. Do =do, did, done, irregular, imperfect. Should =shall, should, irregular, perfect. Be =am, was, been, irregular, imperfect. Flattered =flatter, -ed, -ed, regular, perfect. Let =let, let, let, irregular, imperfect. Be =am, was, been, irregular, imperfect. Cursed =curse, -ed, -ed, regular, perfect. Be =am, was, been, irregular, imperfect. Did =do, did, done, irregular, perfect. Call =call, -ed, -ed, regular, imperfect. Be =am, was, been, irregular, imperfect, Hallowed=hallow, -ed, -ed, regular, perfect. Be =am, was, been, irregular, imperfect. Be =am, was, been, irregular, imperfect. Known =know, knew, known, irregular, perfect, Rattling =rattle, -ed, -ed, regular, imperfect. Is =am, was, been, irregular, imperfect. Said =say, said, said, irregular, perfect. Cause =cause, -ed, -ed, regular, imperfect, iFor "How to Ascertain tbe Mood," see Dem. XII., e, f. a. Imperative. b. Indicative. c. Subjunctive. d. Infinitive. e. Participial. subjunctive. indicative. infinitive. indicative. indicative. participial. indicative. infinitive. indicative. participial. indicative. infinitive. particii^ial. participial. imperative. indicative. infinitive. imperative. imperative. infinitive. indicative. infinitive. participial. imperative. infinitive. participial. imperative. indicative. imperative. imperative. participial. imperative. imperative. participial. particii^ial. indicative. participial. infinitive. SYSTEM GBAMMAR— GRADE A. 591 Prog. XIII : / Regularity. 2. a. Regular. i). Irregular. Inflection. 3. Tense. a. Complete. b. Defective. c. Redundant, a. Imperfect, b. Perfect. 7. 9. 10. 11. 12. 13. Can = Do = Sing = Was = 5. Inclined = 6. Sleep = Hate 3 = Could = Laugh = Be = Be = Go = 14. Be = 15. ThougM^ = 18. Were® = Were = Commence = Do = Advance = Refrain = Love = Borrowed = Died = Loved = 19. 31. 32. 35. 40. 41. 46. 47. 48. 50. Be 51. Let 52. 53. 54. 68. 66. 68. Being Defeated Ring Sit Speak Had 77. Approacli'g = irregular, irregular, irregular, irregular, regular, irregular, regular, irregular, regular, irregular, irregular, irregular, irregular, irregular, irregular, irregular, regular, irregular, regular, regular, regular, regular, regular, regular, irregular, irregular, irregular, regular, irregular, irregular, : irregular, : irregular, regular. defective, complete, complete, complete, complete, complete, complete, defective, complete, complete, complete, complete, complete, complete, complete, complete, complete, complete, complete, complete, complete, complete, complete, complete, complete, complete, complete, complete, complete, complete, complete, complete, complete, imperfect, imperfect, imperfect, perfect, perfect, imperfect, imperfect, perfect, imperfect, imperfect, imperfect, imperfect, imperfect, perfect, perfect, perfect, imperfect, imperfect, imperfect, imperfect, imperfect, perfect, perfect, perfect, imperfect, perfect, imperfect, perfect, imperfect, imperfect, imperfect, perfect, imperfect. 4. Mood.^ a. Imperative. b. Indicative. c. Subjunctive, d. Infinitive. e. Participial. indicative.^ infinitive. infinitive. indicative. participial. infinitive.^ infinitive. indicative. infinitive.* infinitive. infinitive. infinitive. infinitive. participial. indicative.® subjunctive. imperative. imperative. imperative. infinitive. infinitive. participial. indicative. participial. infinitive. participial. participial. participial. imperative. imperative. infinitive. subjunctive. participial. iSee "The Potential Mood a Blunder," "Would Make the Verb belong to the Mood,^' ..Pot:ntl.'F"rd1tion.alse;'"L™^^^^ .For ''How to Ascertain the ^ood," see De m^ ^Ini ^llugk is ^ere verbal comple- tively as objects of the P-P^^^f '/^^^J:™- g™" i 1, m, nfo. e Tr^.^istbesubjunc licensed use of were, see Particular 315. Dem. XIII. 692 THE SYSTEM METHOD. Prog. XIV: /■ Class of Pronoun. 2. Person. 3. Number. 4. Gender. 5. Case. a. Personal. a. First. a. Singular, a. Masculine, a. Nominative. b. Relative. b. Second, b. Plural. b. Feminine, b. Objective. c. Interrogative, c. TMrd. c. Equivocal, c. Neuter. c. Possessive. d. Common. 1. It ^personal, third, singular, neuter,^ nominative. 2. He =personal, third, singular. masculine. ^ nominative. 3. I =personal, first, 1 singular. common. nominative. 5. It 2 ^personal, third. singular. neuter. objective. 6. Them 3 =personal, third. plural. common. objective. 3 9. Who ^relative. third. singular. common. nominative. 16. Whom ^interrogative, third. singular. common. objective. 17. You* =personal, second. plural,* common. nominative. 20. His =personal, third, singular. masculine. possessive. 21. Whose =:interrogative, , third. equivocal, common. possessive. 22. Which =interrogative, third. singular. neuter. nominative. 23. What =interrogative, third. equivocal, , neuter, objective. 28. Thou =personal, second. singular. masculine. nominative. 30. Thy =personal, second, singular, masculine. j)Ossessive. 36. Themselves=personal, third. plural. common. objective. 38. It 2 =personal, third. singular. neuter. nominative. 39. My =:personal, first, singular. common. possessive. 42. Who =relative, third, singular. masculine. nominative. 44. I =personal, first. singular, common. nominative. 45. That =relative. first. singular, common. nominative.^ 47. My =personal, first. singular. common. possessive. 48. Who =interrogative, third. plural, common. nominative. 49. You =personal, second. plural. common. nominative. 50. That =relative, second. plural. common. nominative. 58. Who 5 =:relative,^ third. singular. masculine. nominative. 54. You* =personal, second. pluraL* masculine. nominative. 55. Yourself ^personal, second, singular. masculine. nominative. 56. What =interrogative, , third. singular. neuter, nominative. 61. Ye =personal, second. plural. common. nominative. 62. Who =relative, second. plural. common. nominative. 66. Whose =relative. third, singular, masculine. possessive. 68. Whom =relative, third. singular. masculine. objective. 70. He =personal, third. singular, masculine. nominative. 71. Who ^relative, third, singular. masculine. nominative. 78. You =personal, second. plural. common, objective. 80. We =personal, first, plural. common, nominative. IOC ). Mine« =personal, first, singular. common. possessive.^ 1 Subject and predicate nouns and pronouns must agree in case, not necessarily in per- son, number, and gender. 2 gee Dem. XIV., a; also Dcm. XV., a. sgee Dem. V., h. * You is always grammatically plural ; see Dem. XV. 6 gee " Difference Between a Relative and an Interrogative," Dem. XV., p. 457. ^See Dem. XIV., Obs. d. SYSTEM QBA3IMAB— GRADE A. 593 Viog, XV : /. Class of Pro. 2. Person. 3. Number. 4. Gender. 5. Case. a pGrsonfil- a. First. a. Singular, a. . Masculine. a. Nominative. b'. Relative. c. Interrogative. b. Second. b. Plural. b . Feminine. b. Possessive. c. Third. c. Equivocal, c . Neuter. . Common. c. Objective. 1. It personal, third. singular. neuter. nominative. 2. It ^^ personal, third. singular. neuter, objective. 5. What , relative, third. singular. neuter. nominative. 6. He personal, third. singular. masctdine, nominative. 7. It _: personal, third. singular. neuter. nominative. 8. They = personal. third. plural. common. nominative. 13. Your ==: personal, second, plural. common. possessive. 19. Which relative, third. singular. neuter, objective. 21. We =^ personal, first, plural, feminine, nominative. 25. Ourself :=3 personal, first. singialar. common. objective. 26. What 3 relative. third, equivocal, 1 neuter. objective. 27. What 2 , relative. third, singular. neuter. nominative. 28. Which relative. third. singular. neuter. objective. 29. Which — , relative. third. singular. neuter. nominative. 30. Hers 3 = personal. third. singular. feminine. possessive. 32. Tours 3 _; personal. second. plural. common. possessive. 34. Ours ^=: personal. first, plural. common. possessive. 35. What __ relative, third. equivocal,^ neuter, objective.* 37. What , relative, third. singular. neuter. objective. 38. Who , relative. third, singular. masculine. nominative. 40. What relative, third. equivocal,^ neuter, nominative. 5 41. Who =: relative, third. plural, common. nominative. 43. Their =: personal, third. plural. common. possessive. 43. Who relative. third. singular. masculine. nominative. 44. What relative, third, equivocal,^ neuter. objective.^ 46. Thou _^ personal. second, singular, common. nominative. 47. Who _-r relative, second. singular. common. nominative. 49. Te = personal. second, plural. common, nominative. 50. That __ relative. second, plural. common. nominative. 53. Ourself -^ personal. first. singular. common. objective. 53. Whatever^ relative. third, singular, neuter. nominative. 54. Whoever = relative, third. equivocal,'- common. nominative. 55. It — personal. third. singular. neuter. objective. 56. What _ : relative. third. equivocal,^ neuter. objective. 57. What __ relative. third. equivocal. neuter. objective. 58. He — : personal. third. singular. masculine , nominative. 59. What _ -- relative, third, equivocal,^ neuter. objective. 60. Whoever = : relative. third. singular. common. nominative. 1 Here as elsewhere, " equivocal " is applied where neither singular nor plural is appli- cable 2 In' grammatical construction and properties, that is, in parsing, M-Aa^ is always identical with winch before if, see Dem. XV. ^See "Equivalency in Meamng is not Sameness of Grammatical Construction," Dem. XV. * Object of wanted, not got. ^Sub- j ect of prevented. « Obj ect of want. 594 THE SYSTEM METHOD. Pros'. XY. — Pronouns Continued. 69. \Vliatevcr= relative, third. equivocal, neuter, objective. 70. What = relative. third, equiA'oeal, neuter, objective. 1 71. What = relative. third, equivocal. neuter, nominative. 72. What = relative. third. singular. neuter, objective.^ 73. Whatever= relative. third. singular, neuter, objective. 74. Thy = personal. second. singular, common, possessive. 75. Whatever= relative. third. equivocal. neuter. objective. 76. What 3 = relative,^ third, equivocal. neuter. objective. 77. What = relative. third, equivocal. neuter, nominative. 78. As = relative. third, plural. neuter, nominative. 79. As = relative. third. plural. neuter. nominative. 80. As = relative. third. singular. neuter. nominative. 81. As = relative. third, singular. neuter, nominative. 82. As = relative. third. plural. common, nominative. 83. As = relative. third. singular. neuter, nominative. 84. As = relative. third. plural. common. nominative. 85. As* = relative, third. plural. neuter, - objective.* 86. As = relative. third, plural. neuter, nominative. 87. As c= relative, third, plural, neuter. nominative. 88. As = relative. third. plural. neuter. nominative. 90. As = relative. third. singular. neuter, nominative. 91. As = relative, third. singular. neuter. objective. 93. As = relative. third. singular. neuter, objective. 94. As = relative, third. singular. neuter. nominative. 95. What = interrogative, third. singular. neuter, nominative. 96. Thou = personal. second, singular. common, nominative. 97. What = relative, third, singular. neuter. nominative. 98. She = personal, third. singu'ar. feminine. nominative. 99. What = interrogative 1, third. singular. neuter, objective. 100. As = relative, third. equivocal. neuter. nominative. 101. As = relative. third. plural. common, nominative. 102. Theirs = personal, third. plural. common, possessive. 103. What =: interrogative :, third. singular. neuter. nominative. 104. That = relative. third, singular. neuter. objective. 105. What = relative. third, equivocal. neuter. objective. 106. It = personal. third. singular. neuter. nominative. 107. What = interrogative !, third, plural. neuter. nominative. 108. Whose = relative. third. singular. neuter. possessive. 109. Who = relative, third. plural. neuter,^ nominative. 110. Whose = relative. third. singular, neuter. possessive. 111. Whose = relative. third. plural. neuter. possessive. 1 Object of ileK. 2 Factitive object of /OMMcJ. See Dem. XXYI., b, 2. spor proof that wAai here is not interrogative see Dem. XV., "Difference between a Relative and an Inter- rogative.'" *It is proof that this as is a relative pronoun that it is modified by the adjective ^SeeDem. III., Obs. e. SYSTEM GRAMMAR— GRADE A. 595 Pro^. Xyi: 1. 2. 6. 9. 13. 15. 17. 19. 20. 21. 22. 26. 27. 35. 36. 37. 39. 44. 45. 46. 47. 48. 49. 50. 52. 53. 54. 55. 58. 59. 60. 61. 63. 65. 66. 77. 96. The Sweet Ripe A Long Blue This Very The English Positive The Three Fairest "Wise Some Worthless Either Three Which What Whatever No Business Snuff-box Enough First Beyond ^ This Many A The High-born Most Heartless Goods What 2 /, Adjective. 2. Class. 3. a. Quantitative. b. Qualitative. c. Demonstrative. Accessory to Wha adjective, demonstrative, canary's. adjective, qualitative. song. adjective. qualitative, cherries. adjective. quantitative. pencil. adjective. qualitative. pencil. adjective, qualitative. they. adjective, demonstrative. [question]. adjective. demonstrative, question. adjective. demonstrative. [people]. adjective, qualitative. [people]. adjective. qualitative. [people]. adjective, demonstrative, horse. adjective. quantitative. [horses]. adjective, qualitative. [flower]. adjective, qualitative. he. adjective. quantitative, [books]. adjective. qualitative. [books]. adjective. quantitative. [form] . adjective. quantitative. forms. adjective. demonstrative, book. adjective. qualitative. books. adjective, quantitative. soldiers. adjective. quantitative. connection. adjective. qualitative, connection. adjective. qualitative. factory. adjective. quantitative. men. adjective, demonstrative, tree. adjective. demonstrative. tree. adjective. demonstrative. many a year. adjective. quantitative. year. adjective. quantitative. year. adjective. demonstrative. [l^ersons]. adjective. qualitative. [persons]. adjective. quantitative, part. adjective. qualitative. [persons]. adjective. qualitative. be. adjective. qualitative, wealth. 1 See Dem. XXl., Obs. f, 4. sgeeDem. X^Til., Obs. m. ^Tlie grammatical construc- tion, or parsing, of good and great is the same as if the sentence read, "To[ward] good be- \ing'] is to[ward] great be\ing^ ; " see Dem. XXIX., c, note. 596 THE SYSTEM METHOD. Prog. XVn: /. Adjective. 2. Class. 3. Accessory to What i a. Qiiantitative. b. Qualitative. c. Demonstrative. 1. A = adjective, quantitative. day. 5. New-England.^ = adjective, qualitative, winter. 9. None = adjective. quantitative. weapon. 13. Lonelier = adjective. qualitative. thou. 15. What = adjective. qualitative. man. 16. A = adjective. quantitative, man. 17. Else = adjective. demonstrative, nothing. 18. What = adjective. qualitative. stufE. 20. Wise = adjective, qualitative. she. 23. Wisest = adjective. qualitative, [person]. 24. Wiser = adjective. qualitative. [person] . 25. Greater = adjective. qualitative. I. 26. True = adjective. qualitative. him. 31. Perfect = adjective, qualitative. [sense]. 32. AlP = adjective. quantitative. [possessions]. 33. Many = adjective. quantitative. [persons]. 36. Each 2 = adjective. quantitative, [person]. 37. An = adjective. quantitative, other. 38. Else = adjective, demonstrative. who. 40. Else 3 _ = adjective. demonstrative. nobody. 41. All* = adjective. quantitative. [concern]. 42. All* = adjective. quantitative. [concerns]. 43. Some = adjective, quantitative. [persons]. 44. Good 5 = adjective, qualitative. man. 45. A 5 = adjective, quantitative. man. 48. Many = adjective. quantitative, time. 49. A = adjective. quantitative, time. 53. Five« = ^adjective. quantitative, hundred men. 56. Thousand 7 = adjective. quantitative. [persons]. 57. Two = adjective. quantitative, dozen eggs.® 58. Dozen 7 = adjective. quantitative. eggs. 59. That = adjective. demonstrative, [life]. 63. Upright = adjective. qualitative. I. 66. Bitter = adjective. qualitative. apple. 70. Pale = adjective, qualitative, she. 75. Good* = adjective, qualitative. being. 1 SeeDem. XVI., Obs. h, i. 2 See Dem. XVII., 1. ^See Dem. V., Obs. c; also Dem. XVII., 1. *SeeI»em. XVn., 1, "All." 6 See Dem. XVII., m. esee Dem. XVI., Obs. k. 'See Dem. XXVIII., h. 8 This sentence is, in grammatical construction, this: "Virtue consists in good being;'' see Dem. XXIX., c, note. SYSTEM GRAMMAR— OBADE A. 59'7 Prog. XVIir- /. Adverb. 2. Class. 1. How = 2. Often 3. There 4. Sometimes = 5. Very 6. Well 7. Very = 8. Happily 9. Fluently 13. Enough 15. Where 17. Never 18. More 21. Out 23. Ominously 24. Far! 27. Fast 28. Right 1 29. Far! 32. Away 34. As 38. Not 42. Over 43. Again 52. Then 2 56. By 3 57. By' 62. Hence 63. Ago 65. There* 66. There 70. The 5 71. Louder 87. At 92. Why adverb, adverb, adverb, adverb, adverb, adverb, adverb, adverb, adverb, adverb, adverb, adverb, adverb, adverb, adverb, adverb, adverb , adverb, adverb, adverb, : adverb, : adverb, : adverb, : adverb, : adverb, = adverb, = adverb, = adverb, = adverb, = adverb, = adverb, = adverb, = adverb, = adverb, = adverb. a. Temporal. b. Local. c. Intensive. d. Modal. e. Causal. intensive, temporal, local, temporal, intensive, modal, intensive, modal, modal, intensive, local, temporal, intensive, local, modal, intensive, modal, modal, intensive, local, intensive, modal, modal, temporal, temporal, temporal, temporal, temporal, temporal, local, local, intensive, modal, modal, causal, 3. Accessory to What ? iSeeDem. XVm., Obs. n. 4 This word there is a formalized alogued as belonging in either of Dem. XVTII., Obs. 1. accessory to often, accessory to been, accessory to been, accessory to stutV.-iS. accessory to well, accessory to studies, accessory to happily, accessory to lived, accessory to speaks, accessory to full, accessory to laid, accessory to can. accessory to happy, accessory to am. accessory to scowled, accessory to beyond, accessory to falling, accessory to athwart, accessory to above. accessory to am. accessory to do. accessory to do. accessory to begin. accessory to begin. accessory to knew. accessory to offended. accessory to offended. accessory to start. accessory to unknown. accessory to is. accessory to is. accessory to louder. accessory to talked. accessory to laughed, accessory to speak. .See Dem. XVIII., Obs. c, d. 3See Dem. XVni., Obs. g i. adverb; nevertheless it is an adverb, since U canno be cat- the other seven parts of speech. See Dem. XXX., 1. See 598 THE SYSTEM METHOD. Prog. XIX : /. Preposition. . 2. Objective. 3 Antecedent 4. Unlikeness in. term, or Base. , a. Parts of Speech. b. Properties. c. Constructions. 1. Toi = preposition, die. born. properties. 2. To = preposition, accuracy. is, parts of speech.^ 5. From = preposition, force, is. parts of speech. 7. To = preposition. respect,^ leads, ^ properties. 8. For = preposition, measure. vote, parts of speech. 9. Through = preposition, system. operates. parts of speech. 11. Of = preposition. glory. ambitious, parts of speech. 13. In = preposition. opinion. agree. parts of speech. 14. With = preposition. me. angry. parts of speech. 15. For = preposition. us. has. parts of speech. 16. For = preposition. riches. eager. parts of speech. 18. Of = preposition, rest, need, constructions.* 20. In = preposition. drawing. skillful, parts of speech. 21. Into = preposition. pocket, put. parts of speech. 22. Of = preposition. [possession],^ wife. constructions. 23. From = preposition. you. differ. parts of speech. 24. Along = preposition, hills. heard. parts of speech. 26, To = preposition. you, obliged. parts of speech. 27. By = preposition, auction, sold, parts of speech. 28. Among = preposition. children. divided. parts of speech. 29. To = preposition, pomp, noise. enemy. constructions, s 31. On = prej)Osition, avenue, lives, parts of speech. 32. From = preposition. hour, change, constructions. 33. To = preposition. hour, change, constructions. 34. In = preposition. horologe, hammer. constructions. 37. To = preposition. era. change. constructions. 38. Of = preposition. measles. died. parts of speech. 39. From = preposition, [pronunciation], different, parts of speech. 40. To = preposition. pieces. break. parts of speech. 41. In = preposition. [state], published. parts of speech. 42. Of = preposition, [times]. is. parts of speech. 43. In = preposition. [manner]. spoke. parts of speech. 44. In = preposition. [phrase] , is. parts of speech. 45. In = preposition, fine, live. parts of speech. 47. Ere = preposition. [time]. leave. parts of speech. 48. Upon = preposition. earth. [persons], constructions. 49. At = preposition. [condition], is. parts of speech. 51. Like = preposition. men. quit. parts of speech. iSee Dem. XIX., a, b, c; also Dem. XXX., a, b, c, d. ^See Dem. XIX., b, 2. ^ Leads is In- dicative, respect infinitive. * observe tliat need is an objective element, object of have; while of rest constitutes an adjective element belonging to the noun need, sgee Dem. V., Obs. e. ^lEnemy is a predicate, tojiomp an adjective, element. SYSTEM GRAMMAR— OB ABE A. 599 Prog. XX: /. Conjunction.^ 2. Class. 3. Parts Connected.^ 4. Likeness of Parts.^ 1. And 5. And 6. And 8. And 9. But 10. Though 12. Either 13. Or Lest =conj., =couj., =conj., =conj., =conj., =conj., =conj., =conj., a. Co-ordinate. b. Subordinate. co-ord , co-ord., co-ord., co-ord., co-ord., co-ord., co-ord., co-ord.. a. Like. b. Unlike. Peter*— John, fearfully * — wonderfully, in * — through. Art is long* — etc., rich * — generous, I affirm* — it seems, for * — against, for — against. 14. 17. 18. 19. 21. 22. 23. 24. 25. 26. 27. 30. 31. 34. 36. 38. 39. ^couj., subord., venture^ — I be. Notwithstand'g =conj Whether Or Whether As As Though As As As Both And For Save So Than lo 40. Than 10 41. Than 10 42. That That As If Besides^ Else 9 Now 9 subord., :=conj., subord., ^conj., co-ord., =conj., subord., =conj., subord., =conj., subord., fell^ — he had, =conj., subord., fell^ — he had, go 5 — it rain, care ^ — it rains, rains * — snows, know^ — he has, slew^ — he was. 43. 46. 47. 51. 52. 54. =conj., =conj., =conj., =conj., =conj., =conj., =conj., =conj., =conj., =conj., =conj., =conj., =conj., =conj., =conj., =conj., =conj =conj co-ord., name* — man and eagle, subord., 6 [people] '' — frivolous, subord.,^ be''' — gods, co-ord., learned* — wise, co-ord., learned — wise, co-ord., I hate * — he is, co-ord., [persons] — he and I, co-ord., work is* — is it, subord., larger^ — New York [is], subord., taller 5 — I [am], subord., more — to me, subord., hs went^ — ho went is certain, subord. , so ^ — I perished, subord., blinks^ — he hated, subord., blinks^ — he hated, co-ord., I do* — I have, co-ord., I have* — I could, co-ord., [ ]^- — Barabbas, like. like. like. like. like. like. like. like. unlike. unlike. unlike. like. unlike. unlike. unlike. unlike. unlike. unlike. unlike. like. like. like. like. like. unlike. unlike. unlike. unll':3. unlike. unlike. unlike. like. like. like. iSee Dem. XX., a-d; Dem. XIX., b, c; Dem. XXX., c, d. ^See Dem. XX., Obs. a. ^See Dem. XXX., c. ^co-ord. conjs. do not — as do subordinates — join modifier and thing modified, but two parts each referred to the same third part. (See Dem. XX., Obs. a.) It is, in tlie case of co-ordinate conjunctions, these two parts here given as the "parts connected." ''Sub- ordinate conjunctions join modifier to tiling modified; it is these two parts given as "parts connected," in the case of subordinates. '' Ai is here a subord. conj., since it joins modifier, frivolous, to thing modified, people; see Dem. XX., d, note 1. ' Tliis as is a subord. conj., since it connects the word governed, gods, to the word governing, be. (See Dem. XX., d, note.) 8 gge Dem. XX., Obs. f. » See Dem. XX., Obs. f, 1, 2. >o gee Dem. XX., d, notes 1 and 2 ; also Dem. XXX., m, 1-5; 5, a, b, c, d. 600 THE SYSTEM METHOD. Prog. XXI: 1. Avaunt 2. Ugli 3. Alas 4. Tut 5. Heigli-lio 6. Whew 7. Zounds 8. O 10. Fie 11. Ha 15. Tush 17. Ah 18. Ho 19. Hark 20. Lo 22. Alack 25. Halloo 26. Poh 28. Hist 32. Whoop 36. Hewgh 38. O 39. Ha 40. Ah /. Interjection. . interjection, interjection, interjection, iuterjection, interjection, interjection, interjection, interjection, interjection, interj ection, interjection, interjection, interjection, interjection, interjection, interjection, interjection, interjection, interjection, interjection, interjection, interjection, interjection, interjection. 2. Principle. principle 17. principle 17. principle 17. principle 17. principle 17. principle 17. principle 17. principle 17. principle 17. principle 17. principle 17. principle 17. principle 17. principle 17. principle 17. principle 17. principle 17. principle 17. principle 17. principle 17. principle 17. principle 17. principle 17. principle 17. JVo^e.— See Dem. XVIII., Obs. c. The mere fact that a pencil may be dipped in ink and used as a pen is used, does not constitute such pencil a pen. The mere fact that one's coat may be used as a cushion does not make the coat become a cushion, so that we would be jus- tified in changing the name of coat and calling it cushion instead. (See the law for the '• Change of Names," Dem. XXX., 1.) And so the mere fact that a noun may occasionally depart from its regular— its system-fitting— use to assume that of an interjection, does not necessarily make such noun become an interjection, and justify the grammarian, who is a systematizer, in changing the name from noun to interjection. Accordingly, in such sentences as, " Nonsense! it can't be done," " The willt the will! we will hear Csesar's will," — in such sentences, nonsense^ will, etc., are not to be called inter- jections, but are to retain their own name, "noun;" because, though used like interjections, they have still not wholly lost the distinctive use of nouns. JVonsense and ivill are two nom- inatives in the independent construction by exclamation. (See Dem. XXVI., "Independent Constructions," a, 2.) If, however, in assuming the interj ectional use, the noun or pronoun totally loses its own peculiarities as noun or pronoun, it then actually becomes an interjection ; as, What! you left the man? In this sentence, what cannot be a noun, for it takes neither the added s nor the added ('), and is never found used as a noun in the body of the sentence proper. (See Dem. I., Obs. g; also Dem. II., Obs. k.) It is not an interrogative pronoun; for it is part of no question, and is connected with no verb. (See Dem. XIV., e, f.) It cannot be a personal pro- noun; for it can neither be used witli a verb nor accompanied with a pointer. (See Dem. XIV., e, b, c.) It is not a relative; for it connects not two verbs. (See Dem. XIV. f ; also XV.) Therefore, since what has totally lost the peculiarities of a pronoun, and, in doing so, has acquired the characteristics of an interjection, it is an interjection. SYSTEM GRAMMAR— GRADE A. 601 Prog. XXII .—Branching by the Grammatical Work- Tree. I = word. fundamental, substantive. pronoun, personal. Smell = word, fundamental. verb. transitive. Roses = word, fundamental, substantive, noun. common. Good = word. adjunct. adjective, qualitative. People = word, fundamental, substantive. noun. common. Hate = word. fundamental. verb, transitive. Vice = word, fundamental, substantive. noun. common. You = word, fundamental, substantive. pronoun. personal. Own = word. fundamental, verb, transitive. A = word. adjunct, adjective. quantitative. Horse — word. fundamental. substantive. noun. common. Multitudes = word. fundamental. substantive. noun. common. Perish = word. fundamental. verb. intransitive. The = word, adjunct. adjective, demonstrative. Rainbow = word, fundamental, substantive. noun. common. Aj^pears = word, fundamental. verb. intransitive. Beautiful = word, adjunct. adjective. qualitative. The = Avord, adjunct. adjective. demonstrative. Leech = word. fundamental. substantive. noun, common. Adheres = word. fundamental. verb. intransitive. To = word, connective, preposition. The = word. adjunct. adjective, demonstrative. Skin = word, fundamental. substantive, noun. common. The = word. adjunct. adjective, demonstrative. Wax = word, fundamental. substantive. noun. common. Sticks = word. fundamental. verb. intransitive. Fast = word, adjunct. adjective. qualitative. James = word. fundamental. substantive. noun. proper. Walks = word, fundamental. verb. transitive. His = word. fundamental, substantive. jjronoun. personal. Horse = word, fundamental. substantive. noun. common. Around = word, connective. preposition. The = word. adjunct, adjective. demonstrative. Lot = word. fundamental. substantive. noun. common. Pharaoh = word, fundamental. substantive. noun, proper. Dreamt = word. fundamental. verb. transitive. A = word. adjunct. adjective. quantitative. Dream = word, fundamental, substantive. noun. common. The = word, adjunct. adjective, demonstrative. Boy = word. fundamental, substantive. noun, common. Flies = word. fundamental. verb. transitive. His = word. fundamental, substantive, pronoun. personal. Kite = word, fundamental, substantive. noun. common. 602 THE SYSTEM METHOD. Prog. XXII (Continued).— Branching by the Tree. The = word, adjunct, adjective, demonstrative. Earth = word, fundamental, substantive, noun, common. Grows = word, fundamental, verb, transitive. Plants = word, fundamental, substantive, noun, common. And = word, connective, conjunction. Trees = word, fundamental, substantive, noun, common. The = word, adjunct, adjective, demonstrative. Minister = word, fundamental, substantive, noun, common. Dies = word, fundamental, verb, transitive. A. = word, adjunct, adjective, quantitative. Righteous = word, adjunct, adjective, qualitative. Death =: word, fundamental, substantive, noun, common. The = word, adjunct, adjective, demonstrative. Tailor = word, fundamental, substantive, noun, common. Sews = word, fundamental, verb, intransitive. Neatly = word, adjunct, adverb, modal. A == word, adjunct, adjective, quantitative. Woman = word, fundamental, substantive, noun, common. Toils == word, fundamental, verb, intransitive. A = word, adjunct, adjective, quantitative. Man = word, fundamental, substantive, noun, common. Works = word, fundamental, verb, intransitive. Man's = word, fundamental, substantive, noun, common. Works = word, fundamental, substantive, noun, common. Decay = word, fundamental, verb, intransitive. The = word, adjunct, adjective, demonstrative. Tempest = word, fundamental, substantive, noun, common. Sank = word, fundamental, verb, transitive. The = word, adjunct, adjective, demonstrative. Vessel = word, fundamental, substantive, noun, common. The = word, adjunct, adjective, demonstrative. Timber = word, fundamental, substantive, noun, common. Splits = word, fundamental, verb, intransitive. Easily = word, adjunct, adverb, modal. The = word, adjunct, adjective, demonstrative. Passengers = word, fundamental, substantive, noun, common. Crowded = word, fundamental, verb, transitive. The = word, adjunct, adjective, demonstrative. Car = word, fundamental, substantive, noun, common. The = word, adjunct, adjective, demonstrative. Author = word, fundamental, substantive, noun, common. Englished = word, fundamental, verb, transitive. SYSTEM GRAMMAR— GRADE A. 603 Prog. XXIII. — Branehing by the (grammar- System Tree Complete. 1. Have = verb, infinite, infinitive, verbal complement, assertive. 2. Been = verb, infinite, participial, verbal complement. 3. Elected = verb, infinite, participial, verbal complement. 4. Be = verb, infinite, infinitive, verbal complement, assertive. 5. Be' = verb, infinite, infinitive, substantive. 6. Give 1 =: verb, infinite, infinitive, substantive. 7. Eat 1 = verb, infinite, infinitive, substantive. 8. Learn = verb, infinite, infinitive, substantive. Born = verb, infinite, participial, verbal complement. Die 2 = verb, infinite, infinitive, substantive. Laugh ^ = verb, infinite, infinitive, verbal complement, assertive, Cry^ _. verb, infinite, infinitive, substantive. Studied = verb, infinite, participial, verbal complement, 17. Fire = verb, infinite, infinitive, substantive. 18. Ordered = verb, infinite, participial, verbal complement. 19. Be = verb, infinite, infinitive, substantive. 21. Fall = verb, infinite, infinitive, verbal complement, objective.* 22. Be = verb, infinite, infinitive, verbal complement, objective. 23. Sealed = verb, infinite, participial, verbal complement. 27. Going = verb, infinite, participial, verbal complement. 28. Drive = verb, infinite, infinitive, verbal complement, objective. 29. Be^ == verb, infinite, infinitive, substantive. 31. Come = verb, infinite, infinitive, substantive. 32. Called = verb, infinite, participial, verbal complement. 38. Love = verb, infinite, infinitive, verbal complement, assertive. 36. Let = verb, infinite, participial, verbal complement. 38. Remember'd=: verb, infinite, participial, verbal complement. 39. Beat = verb, infinite, infinitive, verbal complement, objective. 40. Come = verb, infinite, participial, verbal complement. 41. Work = verb, infinite, infinitive, substantive. 43. Cry® = verb, infinite, infinitive, verbal complement, assertive. 45. Give = verb, infinite, infinitive, verbal complement, objective. 47. See = verb, infinite, infinitive, substantive. 48. Suffer = verb, infinite, infinitive, verbal complement, objective. 49. Sparkling'' = verb, infinite, participial, adjectival. 50. Svrim = verb, infinite, infinitive, verbal complement, objective. 1 Object of to. which is a preposition; see Dem. XXX. ^ To equals toward: born toward death (die). i>ie is used like a noun, and is object of the to. ^ Laugh and cry, as here contrasted, are typical illustrations of the two constructions of the infinitive. *Saw t\\e falling of him. ^Be is object of the preposition to (Dem. XXX., e— 1) ; and good is an appositive adjective modifying the substantive verb be (see Dem. XXIX., c, note). 6"Had rather be and cry "=would rather, etc. Hadis used by enallage for would. '''Be- cause sparkling is here used like an adjective, does not necessarily prove it to be an ad- jective. That error is common among grammarians. See Prog. XXI., note. 604 THE SYSTEM METHOD. Prog. XXIII (Continued).— Branching of Infinite Yerbs. 63. Eating 1 = verb, infinite. participial. substantive. objective. 63. Tumbling t= verb, infinite. participial. adverbial. 64. Pleading = verb, infinite. participial, adverbial. 65. Contriving = verb, infinite, participial. substantive, objective. 66. Conniving = verb. infinite. participial. substantive. objective. 67. Paid = verb, infinite. participial, adjectival. 68. Jerked = verb, infinite, participial, adjectival. 69. Strut^ = verb. infinite. infinitive, substantive. 70. Kave^ = verb, infinite, infinitive. substantive. 71. Tossed = verb. infinite. participial. verbal complement. 72. Ligbtening = verb. infinite. participial, adjectival. 73. Go = verb. infinite, infinitive. verbal complement, assertive. 74. Been =: verb. infinite. participial. verbal complement. 76. Enacted = verb, infinite. participial. verbal complement. 79. Prophesying ;== verb. infinite. participial. adjectival. 80. Be = verb. infinite. infinitive. verbal complement. objective. 83. Go = verb. infinite, infinitive. substantive. 85. Being = verb. infinite. participial. adjectival. 86. Let = verb, infinite, participial, verbal complement. 87. Go 3 = verb, infinite. infinitive. verbal complement. assertfve. 88. Going = verb, infinite. participial. adjectival. 89. Like = verb, infinite, infinitive, verbal complement, assertive. 90. Singing = verb. infinite, participial. substantive. objective. 92. Writing =3: verb. infinite. participial. substantive, objective. 93. Going = verb, infinite. participial. verbal complement. 95. Married = verb. infinite, participial. verbal complement. 96. Crushed = verb, infinite, participial, adjectival. 98. Singing = verb, infinite. participial, substantive, objective. 101. Moaning = verb, infinite, participial, adjectival. 102. Torn = verb. infinite, participial. adjectival. 104. Bleeding = verb, infinite, participial, adjectival. 110. Been = verb, infinite. participial. verbal complement. 112. Avoid = verb. infinite. infinitive. verbal complement. assertive. 113. Giving = verb. infinite. participial. substantive, objective. 116. Giving = verb. infinite, participial. substantive. objective. 117. Die = verb. infinite, infinitive. verbal complement. assertive. 118. Ask* = verb. infinite. infinitive. substantive. 119. Being = verb, infinite, participial. substantive. subject. 122. Being = verb. infinite. participial. substantive, objective. 123. Elected = verb. infinite, participial. verbal complement. 124. Supposing = verb. infinite, participial. adjectival. 129. Laughing 3 ' = verb. infinite, participial, substantive. objective. 1 While we call— and not very impr operly— this substantive verb eating a "participle," it is really the old i gerundial infinitive, i and it therefore j ustifles with to. « See Dem. XXIII., j. 3 See Dem. SXIIL, g. i Ask is here the adverbial object ( 3f the comparative adverb sooner; see Di um. XXIII., j, SYSTEM GRAMMAR— GRADE A. 605 Prog. XXIV. — Branching by the Grammar- System Tree. 1. Advancing 2. Retreated 3. Refusing 7. Saving 8. Wist 11. Going 12. FeU 14. Could 15. Avoid 29. Were 20. Formed 23. Ceases 24. Flying 26. Troubling 28. Purpose 30. Love 82. Having 33. Arrived 84. Departed 38. Speak 40. Think 42. Being 44. Husbanded 46. Love 47. Doth 49. Gave 50. Did 55. Was 56. Moved 57. Conspire 59. Refrain 60. Had 61. Love 68. Known = verb, infinite, participial, = verb, finite, variable, =:: verb, infinite, participial, := verb, infinite, particiijial, =:: verb, finite, variable, = verb, infinite, participial, = verb, finite, variable, = verb, finite, variable, = v(Tb, infinite, infinitive, = vei'b, finite, variable, = verb, infinite, participial, =; verb, finite, variable, = verb, infinite, participial, = verb, infinite, participial, = verb, infinite, infinitive, = verb, infinite, infinitive, = verb, infinite, participial, := verb, infinite, participial, = verb, fiLnite, variable, = verb, infinite, infinitive, = verb, finite, variable, = verb, infinite, participial, = verb, infinite, participial. = verb, finite, variable, = verb, finite, variable, = verb, finite, variable, = verb, finite, variable, = verb, finite, variable, = verb, finite, variable, = verb, infinite, infinitive, = verb, infinite, infinitive, = verb, finite, variable, =1 verb, infinite, infinitive, = verb, infinite, participial, 65. Approaching = verb, infinite, participial, 66. Felt = verb, finite, variable, 67. Beating = verb, infinite, participial, 68. Remembering = verb, infinite, participial, 70. Searching = verb, infinite, participial, 73. Do = verb, finite, variable, 74. Entreat = verb, infinite, infinitive, 75. May = verb, finite, variable, adjectival. indicative. adjectival. substantive, objective. indicative. adjectival. 1 indicative. indicative. verbal complement, assertive. indicative. verbal complement. indicative. substantive, objective. substantive, objective, verbal complement, assertive. substantive. adjectival. verbal complement. indicative, substantive. indicative. adjectival. verbal complement. indicative. indicative. indicative. indicative. indicative. indicative. substantive. verbal complement, assertive. indicative. substantive. adjectival. adjectival. indicative. adjectival. adjectival. adjectival. indicative. verbal complement, assertive. subjunctive. 1 Observe that while the participle going ia'hoxe accessory — adjectival — to Aim, in such sentences as, "We made him go" and "Let him be Csesar," the infinitive?, gro and 6e, are said to be accessory to the verbs made and let, respectively. 606 THE SYSTEM METHOD. Prog. XXIV (Continued).— Branching of Verbs. = verb, finite, variable, indicative. = verb, infinite, participial, verbal complement. = verb, infinite, infinitive, verbal complement, objective. = verb, finite, variable, indicative. = verb, finite, invariable, subjunctive. = verb, finite, variable, indicative. = verb, infinite, infinitive, verbal complement, assertive. = verb, finite, variable, indicative. = verb, infinite, participial, verbal complement. = verb, finite, variable, indicative. = verb, infinite, participial, verbal complement. = verb, infinite, participial, substantive, objective. 115. Compelled = verb, infinite, participial, verbal complement. 116. Make = verb, infinite, infinitive, substantive. 119. Accustomed = verb, infinite, participial, verbal complement. 120. Engaged = verb, infinite, participial, adjectival. = verb, finite, variable, indicative. := verb, infinite, participial, adjectival. = verb, infinite, participial, verbal complement. =: verb, infinite, infinitive, substantive. =: verb, infinite, participial, adjectival. = verb, infinite, participial, verbal complement. = verb, finite, variable, indicative. z= verb, infinite, infinitive, substantive. = verb, infinite, participial, verbal complement. = verb, infinite, participial, adjectival. = verb, infinite, participial, verbal complement. = verb, finite, invariable, subjunctive. = verb, finite, invariable, subjunctive. = verb, infinite, infinitive, substantive. = verb, finite, variable, indicative. = verb, finite, variable, indicative. = verb, infinite, infinitive, verbal complement, = verb, infinite, participial, adjectival. = verb, infinite, participial, adjectival. = verb, infinite, infinitive, substantive. = verb, infinite, participial, adjectival. = verb, infinite, participial, adverbial. = verb, infinite, infinitive, substantive. = verb, infinite, infinitive, verbal complement, = verb, finite, variable, indicative. = verb, finite, variable, indicative. = verb, infinite, participial, substantive, 181. Contriving = verb, infinite, participial, substantive, 183. Paid = verb, infinite, participial, adjectival. 86. Knew 88. Struck 90. Trickle 91. Guides 96. Had 98. Could 99. Have 105. Shall 107. Kewarded 109. Is 110. Paid 111. Making 121. Must 124. Laughed 127, Believed 129. Wait 130. Being 131. Asked 133. Desired 134. Consider 137. Born 138. According 141. Bounded 143. Wander 144. Tremble 145. Cheer 146. Knows .147. Can 149. Bring 154. Shifting 155. Peeling 157. Make 158. Conniving 161. Rippling 168. Live 170. Outlive 171. Plows 174. Is 177. Hiving assertive. assertive. objective, objective. SYSTEM OBAMMAB—ORABE A. 60Y Prog. XXV: /. Substantive. 2. Class. 3. Case. 4. Construction. 5. Office. a. Noun. a. Com. or prop. a. Nominative, a. Essential. a. Subject. b. Pronoun, b. Per., re]., or b. Possessive, b. Accessory, b. Predicate. interrog. c. Objective, c. Independent, c. Appositive. d. Adnominal. 1. Brain :=noun, comiDon, 3. Devil's =noun, common, 3. Workshop =:noun, common, 4. He =pronoun, personal, 5. Money- =noun, common, 7. Murder :=noun, common. 8. Milton :=noun, proper. 13. Powder =:noun, common, 15. Solomon ==nouu, proper. 17. Banker =noun, common, 19. Conscience =noun, common. 30. It =rpronoun, personal. 33. Richard Roe 5=noun, proper, 35. Feat =:noun, common. 31. I =pronoun, personal. 33. Temperance :=noun, common, 33. Resolution =noun, common, 34. Harm =:noun, common, 35. Children's =noun, common. 37. Paul =noun, proper, 38. Roman =noun, proper. 41. Bingo =noun, proper, 43. His 44. Teeth 45. They 50. Wrong 55. Tell 60. Thou 61. Courtier 64. You 67. King 73. Mr. Ogden 73. Peace 76. Fathers 87. Wha.t =pronoun, personal, i^noun, common, i^pronoun, personal, =noun, common, =noun, proper, =ipronoun, personal, =noun, common, =pronoun, personal, =noun, common, =noun, proper, =noun, common, =noun, common, =pronoun, relative, e. Object. subject. adnominal. predicate. subject. object.^ object. nominative, essential, possessive, accessory, nominative, accessory,^ nominative, essential, objective, accessory, objective, accessory, nominative, independent.^ objective, accessory, object.* nominative, independent, nominative, accessory, appositive, nominative, independent.^ nominative, essential, nominative, accessory, nominative, accessory, nominative, essential, objective, accessory, nominative, accessory, nominative, essential, possessive, accessory, nominative, independent, nominative, accessory, nominative, independent, nominative, accessory, nominative, essential, nominative, accessory, objective, accessory, nominative, independent. ^^ nominative, essential, subject, nominative, accessory, nominative, essential, objective, accessory, nominative, accessory, nominative, independent, i^ nominative, essential, subject, objective, accessory, object.i^ subject. appositive. appositive.^ subject. object. appositive. subject. adnominal.^ predicate.' possessive, subject.^" appositive. object. predicate, subject, predicate, appositive. 'Accessory to the verb Is; the two essentials in this sentence are brain and is. 2 object of the transitive verb paid, notwithstanding the fact that paidi's, in the so-called "passive voice." sSeeDem. XXVIL, h, i; also Prog. XXI., note. * Object of the preposition worth. 5 See Dem. XXIII., p., note. « In apposition with the sentence, " He scaled Mt. Blanc." 'To the substantive participle pZaj/iwgr. SBy "accompanying circumstance." ^See Dem. VI., a. ^Subject of the verb chatter. They is appositive to teeth; see Dem. XXVI., a, 3, e. ''By pleonasm; see Dem. XXVI., remark, p. 507. '2 By exclamation. '3 of want; see Dems. XIV, and XV. 608 THE SYSTEM METHOD. Prog, XXV (Continued).— Substantives Classified. 95. Father's =noun, conamon, possessive, accessory,! adnominal.i 96. Me 2 =pronoun, personal, objective, independent. 98. Bride =nouu, common, nominative. accessory. predicate. 99. What =pronoun, relative, nominative, accessory. predicate. 101. Stay =noun, common, nominative. independent.^ 105. Mary- ^noun, proper, nominative, accessory. appositive. 107. Cupid =noun, proper, objective. accessory, appositive. 111. East =noun, proper, objective, accessory. object. 112. Me 2 =:pronoun, personal, objective, independent.^ 113. Way =noun, common. objective, accessory, object. 114. Wrath =nouu, common. nominative, independent. 117. Rome* =noun, proper, nominative. independent. 118. Bakery =:noun, common. nominative, independent. 120. WiU =noun, common. nominative, independent. 125. Straw- =noun, common. objective. accessory. object. 127. One r=noun, common. objective, accessory, object. 133. Poet =noun, common, nominative, accessory. predicate.^ 135. Betters ^ :=noun, common, nominative. essential. subject. 137. Limb =noun, common. nominative, independent.''' 140. Hight :=noun, common. nominative. independent.'' 143. Beggar =:noun, common. objective, accessory, predicate. 144. Theirs =pronoun, personal, possessive. accessory. adnominal. 145. Meii['s] 12 =noun, common. possessive, accessory. appositive. 146. He =:pronoun, personal. nominative, independent. 148. Money :=:noun, common, objective, accessory, object. 150. Caesar =noun, proper, objective, accessory, predicate. 152. Bushel =noun, common, objective. accessory. object. 154. Guides =noun, common. objective, accessory. predicate. 156. Sum =noun, common, objective, accessory, object. 159. Music =noun, common, nominative. accessory, predicate. 162. Name =noun, common, nominative. accessory. predicate. 163. Eagle =noun, common, nominative, accessory, appositive. 165. Carpet =noun, common. objective, accessory. appositive. 168. What ^pronoun, , relative. nominative, essential. subject. 171. Henry =noun, proper, objective. accessory, predicate. 172, , Fun =noun, common. objective. accessory. object. 8 186. , Ambition :=noun, common, objective. accessory, object.^ 192, . People =noun, common, objective, accessory. predicate. 213, . Home =noun, common, objective, accessory. object.i* 222, . Miles =noun, common, objective, accessory. object. 11 iTo some sucli noun as "possession" or " choice" understood; see Dem. V., Obs. e. 2See Dem. XXVII., h, i. ^ By pitjonasm. < imicpendent by inscription. ^ ^s ig a subordinate conjunction, joining^oe/! and excels; see Dem. XX., d, note 1. 6 See Dem. XVII., i. ' By ac- companying circumstance. ^Oimade. o Of the preposition worth, i" Of the preposition ("o to be supplied. " Of the adjective long; see Dem. XXX., uuder m. ^^Men is nominative by enallage for the possessive man's. SYSTEM GRAMMAR— GRADE A. 609 Pre )g. XXVI.— Branching by the Grammar- System Tree. 1. Which = substantive. nominative. accessory, predicate. 2. Yours = substantive, possessive. adnominal. 4. Horse = substantive. nominative, accessory, predicate. 6. Hour = substantive, objective, adverbial. 8. Him = substantive. objective. indirect. 9. Money = substantive. objective, direct. 11. Money = substantive. objective, direct. 15. People = substantive, nominative, accessory. appositive. 19. Pounds = substantive. objective, adverbial. 20. Than = substantive, nominative, essential. 21. As = substantive, nominative. essential. 23. Which = substantive. objective,! adverbial. 24. Conjunctions =: substantive. nominative, accessory, predicate. 25. Solomon = substantive, nominative. essential. 26. Whom = substantive, objective,! adverbial. 27. Man = substantive, nominative, essential. 29. Rubies = substantive. nominative,^ essential. 30. Flesh = substantive. nominative, essential. 31. It = substantive, nominative, accessory. appositiv e. 32. Stars = substantive. nominative, essential. 33. They = substantive, nominative. accessory. appositive.^ 35. Hand* = substantive, nominative, accessory. appositive. 39. Heart* = substantive. nominative. accessory, appositive. 41. Himself ^ = substantive, nominative. accessory, appositive. 43. Boy = substantive. nominative. accessory. predicate. 47. Savage = substantive, nominative. accessory. predicate.^ 49. Gods = substantive. nominative. accessory. predicate.'' 51. Man = substantive, nominative. accessory. j)redicate.'' 52. Poet = substantive. nominative. essential. 53. Milton == substantive. nominative. accessory. appositive. 54. Milton = substantive. nominative, essential. 55. Poet = substantive. nominative. accessory. appositive. 57. Thirds = substantive. nominative, accessory. apjiositive. 60. Thou = substantive. nominative. essential. 61. River = substantive. nominative, appositive. 62. Fathers = substantive, nominative. essential. 66. He == substantive, nominative. accessory, appositive. 69. Philosopher = substantive. nominative. accessory, appositive. 79. He = substantive, nominative, independent. 8 1 Objects of the adverb more and adj. wiser. See Dem. XXX., m. * Subject of are, which is to be supplied. ^ The pronouns it and they are resumptive appositives, since they resume flesh und stars. *ffawc? and/oo< are called "distributive" appositives, since they distribute he; so edsomind and heart, b Himself is called a " repetitive " appositive, since it repeats Lord. 6 See Dem. VI., a. '^s is a subordinate conjunction, joining the nominative gods and be, by hich gods is governed; see Dem. XX., notes 1 and 2. 8 By pleonasm. 39 610 THE SYSTEM METHOD. Pros. XXVI (Continued).— Branching of Substantives. 86. Euphrates = 89. Myself = 93. Attorney = 95. Convert = 96. I = 97. Father-in-law = 98. Jethro = 103. Peace = 106. Things = 109. Beelzebub = 110. Whomi = 114. Which 2 = 117. While 3 = 118. Me 3 = 119. Hack = 121. Me 5 = 130. Heart = 132. We = 136. Distance = 138. Hour 6 = 139. Hour 6 = 142. Bushels = 144. Straw = 146. Little 7 = 147. Miles •? = 149. Me 3 = 155. King = 158. Them = 161. Enemy = 165. Feet 8 = 165. Whit 3 = 167. DoUar = 169. Praise = 171. Time = 173. Night = 175. Virgil ^ 176. He = 177. Who = 180. Things = substantive, substantive, substantive, substantive, substantive, substantive, substantive, substantive, substantive, substantive, substantive, substantive, substantive, substantive, substantive, substantive, substantive, substantive, substantive, substantive, substantive, substantive, substantive, substantive, substantive, substantive, substantive, substantive, substantive, substantive, substantive, substantive, substantive, substantive, substantive, substantive, substantive, substantive, substantive, nominative, nominative, nominative, nominative, nominative, nominative, nominative, nominative, nominative, nominative, objective, objective, objective, objective, nominative, objective. nominative, nominative, objective, objective, objective, objective, objective, objective, objective, objective, objective, objective, objective, objective, objective, objective,^ objective,!'^ objective," objective, ^^ nominative, nominative, nominative, nominative, accessory, accessory, accessory, accessory, essential. accessory, accessory, essential. accessory, essential. adverbial. adverbial. adverbial. adverbial. independent.* independent.* essential. adverbial. adverbial, adverbial. adverbial. adverbial. adverbial. adverbial. adverbial. predicate. predicate. predicate. adverbial. adverbial. prepositional. adverbial. adverbial. adverbial. accessory, accessory, essential. accessory, appositive. predicate, appositive. jjredicate. appositive. appositive. appositive. predicate, appositive. appositive. iWhom is governed by, is the object of, the adverb higher; see Dem. XXX., m. * Ob- ject of tlic comparative adverb wore, s objects of the adjective wiser. * By accompanying cir- cumstance. 6See Dem. XXVII., h, i. 6 objects or the adverbs sooner and ^le/ice. 'Objects of the adjective more; sec Dem. XXX., m. 8 object of high; see Dem. XXX. 9 of the prep- osition ivorth. 10 Of the adjective worthy; see Dem. XXX., m, 1, note 3. "Of the adverb ago. 12 Of the adverb long. SYSTEM GRAMMAR— GRADE A. 611 Prog. XXVI (Coiitinued).-Branchiiig of Substantives. 203. Her 204. Her 20r. Little 208. Feet 212. Gad 213. Troop 217. Sun 218. We 220. Honor 221. Mountains 222. Hight 226. Colors 229. Wheel 233. Lore 235. Her 236. Queen 238. Me 239. Coat 241. Themselves 242. Men 244. Friend 256. Man 260. Head 264. Tea 268. Deal 269. Tears 271. Times 276. Home 281. Shelley['s]i2 285. Theirs 286. Men['s]i* 287. Women's 295. Hand 310. Mei6 313. Century 318. Liberty substantive, substantive, substantive, substantive, substantive, substantive, substantive, substantive, substantive, substantive, substantive, substantive, substantive, substantive, substantive, substantive, substantive, substantive, substantive, substantive, substantive, substantive, substantive, substantive, substantive, substantive, substantive, substantive, substantive, substantive, substantive, substantive, substantive, substantive, substantive, substantive. objective, possessive, objective,'- objective,^ nominative, nominative, nominative, nominative, nominative, nominative, nominative, nominative, nominative, objective, objective, objective, objective, objective, objective, . objective, objective, nominative, objective, objective, objective,* objective,^ objective,'" objective,^^ possessive, possessive, possessive, possessive, nominative, objective. nominative, nominative. direct. adnominal. adverbial. adverbial, independent.^ essential, independent.* essential. independent.* essential. independent.* independent.* independent.* prepositional. direct. factitive. indirect. direct. direct. factitive. indirect. accessory,^ appositive.® accessory, '^ adverbial. adverbial. adverbial. prepositional. adnominal.'^ adnominal.''^ appositive.^* adnominal. '5 independent.* independent.* independent.!'' appositive.s appositive. 1 Of the adverb more. 2 See Dem. XXX., m. » By pleonasm. * By accompanying circum- stance. 6yls is a subordinate conjunction ; see Dem. XX., d, note 1 ; and Dem. XXX., d, m. s Head and body distTibnte Mm. t Ifamely is a. siihordmaVe conjunction, s of the adverb fier- ier- see Dem. XXX., m. aObjectof the adverb ago. lOQf taken to be supplied: Two [taken] two t'imes, are four, n Of the preposition to, to be suppUed. 1= To writings. Shelley is nom- inative by enallage for the possessive Shelley's, is To right, to be supplied, hto theirs. Men is nominative by enallage for the possessive men's, is participles, when used like nouns, may be limited by either a possessive or an adjective; see Dem. XXIX., c, note. "See Dem. XXVn., h, i. 1' By exclamation. 612 THE SYSTEM METHOD. Prog. XXVI (Continued).— Branclimg of Substantives. 320. Tyranny = substantive, nominative, essential. 321. Pulses = substantive, nominative. independent.! 323. That r= substantive, nominative. essential. 326. Them = substantive, objective. direct. 329. Emerson = substantive, nominative. independent.^ 330. Sir = substantive, nominative. independent.^ 332. You = substantive, objective, direct. 333. Me = substantive, objective, indirect. 334. Bit = substantive, objective. direct. 336. Allred['s] = substantive, possessive. adnominal.* 337. Saxon's = substantive, possessive, appositive.5 343. You = substantive, nominative. independent.^ 344.. Blockhead = substantive. nominative. accessory, appositive. 345. Putnam's = substantive, possessive, adnominal. 346. Publisher['s] = substantive, possessive, appositive.'' 349. Friend's = substantive. possessive. adnominal. 350. Hunter's = substantive, possessive. appositive.''' 351. John's = substantive. possessive, adnominal. 352. Duke = substantive. nominative. accessory, predicate. 353. Captain['s]8 = substantive, possessive. adnominal.^ 354. Fulton = substantive, objective. .prepositional. 355. Wife = substantive. nominative, essential. 356. Yesterday 9 =: substantive. objective. adverbial. 359. Pounds = substantive, objective, ^'^ adverbial. 361. Father = substantive. nominative. essential. 362. Whom = substantive. objective,^" adverbial. 363. Man = substantive. nominative. essential. 365. Henry = substantive. nominative. essential, 366. James = substantive, nominative,!! essential. 368. Which = substantive. objective,!" adverbial. 371. We = substantive. nominative,!^ essential. 373. Them == substantive, objective,!'! adverbial. 375. What = substantive, objective,!^ direct. 377. Wit = substantive, objective,!^ adverbial. 379. Ballast = substantive. objective,!" adverbial. 381. Maxims = substantive, nominative, independent.!'' 383. Father = substantive, objective. appositive.!* 385. Brethren = substantive. objective. appositive.!* 387. Women = substantive. objective,!' adverbial. 1 By pleonasm 1. 2 By inscription. 3 By direct addrc ss. ^ To coxirt. Alfred is used by enallagc for Alfred's. 6 To Alfred\^s'\. 6 To Putnam's. Publisher is enallage for publis/i- er's. "> To friend's 8' ro wife. Captd, in is enallage for Captain's. Sec Dem. V., Obs. c; also Dem. XVII., 1. . sSeeDem. XVIII., Obs. b. 10 Of the adverb more. " Subject of is to be supplied. >2 0fthc ! adverb more. '^Sii bjectof are, to be supplied. >*0f the adverb morg. ii-Of pay\ "than what I can pay" is an adverbial element of the 3d class, limiting greater. "■•Of less and 7nori ': '' 'By inscription. iSThc two as's are subordinate conjunctions. '^Of the adjective more. 20 Object of wiser. SYSTEM GRAMMAR— GEADE A. 613 Pros XXYII.— BraucUiug by the Grammar- System Tree. predicate.^ preclicate. nominative, nominative, objective, nominative,^ possessive, nominative, objective,^ objective, objective, objective,* nominative, nominative, nominative, nominative, nominative, nominative, possessive, objective,^ nominative, nominative, possessive, possessive, nominative, objective, objective, objective, nominative, nominative, nominative, nominative, nominative, noradnative, objective, nominative, nominative, nominative, nominative, nominative, - 'Though is a subordinate conjunction joining / (word modified) and its momer anointed which is a copula-equivalent to a &.-word. ^To Arnold^s. Teacher is enallage for teach- :^T:Oflol 0/is\n adverb modifying ^... ^ Of say. ^^^ Dem. XX., d, notes o^ys a subordinate conjunction, joining one (thing governed ^y spoke) ''^"ZTx^i m " iil d 7^. is a subordinate conjunction. BOf the adjective ftroad .• see Dem XXX m. ^As.s asubordinate conjunction, lo i/.,i is enallage for men^s. hto rrght, to be supplied. i^By JcolX^ScnlurnsUnce. "To /... ^*To thee. ^^To peace, prospenty, ani happiness, which it resumes, is Resumptive appositive. >t By pleonasm. 1. Englishman = 3. I = 4. Day = 5. King = 13. Teacher['s] = 14. Scholar = 18. Sight = 19. Us = 30. Present = 23. What - 24. Others = 28. Gods 29. One "5 - 31. Slaved 35. Power = 37. Corpse = 39. Fellow's = 43. [Ocean] - 44. Teacher 9 = 4.5. Mayor 9 = 47. Theirs 9 - 48. Men['s]9 49. Penelope = 51. Men 53. Friends = 55. Debts 61. Horse = 63. Boy 65. Convert : 78. Man 81. Limb 83. Hand • 89. Lips Hight Things It 102. Gad 103. Troop 90 94 101 substantive, substantive, substantive, substantive, substantive, substantive, substantive, substantive, substantive, substantive, substantive, substantive, substantive, substantive, substantive, substantive, substantive, substantive, substantive, substantive, substantive, : substantive, : substantive, = substantive, : substantive, : substantive, : substantive, = substantive, = substantive, = substantive, = substantive, = substantive, = substantive, - substantive, = substantive, = substantive, = substantive, substantive. accessory, essential. adverbial. accessory, appositive.^ accessory, direct. indirect. direct. direct. essential. accessory, accessory, accessory, accessory, accessory, adnominal. adverbial. accessory, accessory, adnominal. 1^ appositive.^" accessory, factitive. indirect. direct. accessory, accessory, accessory, independent.^^ independent.!^ accessory, appositive.!* independent.12 accessory,!^ accessory, independent.!'^ essential. predicate. predicate.^ predicate. predicate. predicate. predicate. predicate, appositive. predicate. predicate. predicate, predicate. appositive.!^ appositive. appositive.!" 614 THE SYSTEM METHOD. Prog. XXVII (Continued).— Branching of Substantives. 104. Orphan = substantive, nominative, essential. 106. Child 111. Lord 112. He 114. Hour 116. Times 117. Six 118. Twelve 119. Wealth 121. Plato 128. You 130. Peace 135. Pulses 138. He 139. That 141. He 143. They 144. Heels 147. Colors 149. Sun 152. Me 156. What 164. As 168. As 170. What 171. Simonides = substantive = substantive = substantive; = substantive = substantive := substantive = substantive = substantive = substantive = substantive = substantive = substantive, = substantive = substantive = substantive = substantive = substantive = substantive = substantive = substantive = substantive = substantive = substantive = substantive = substantive 172. What = substantive 178. Anglesey ^= substantive 184. Whom 185. That 186. That 189. Theirs 194. It 197. Father's = substantive = substantive = substantive = substantive = substantive = substantive 199. Brother's ^= substantive 208. Duke['s]i»= substantive 211. Miles = substantive 218. Boy = substantive nominative, accessory, nominative, essential, nominative, accessory, objective, adverbial, objective,^ adverbial, nominative, essential, nominative, accessory, objective, direct, nominative, independent.^ nominative, independent.^ nominative, independent, nominative, independent.* nominative, independent.* nominative, nominative, nominative, nominative, nominative, nominative, objective. ^^ objective, nominative, nominative, nominative, nominative, nominative, nominative, objective,^ nominative, objective,^ possessive,^ objective, possessive,^ possessive, possessive, objective, ^^ essential. independent.^ independent.* independent.* independent.* independent.* adverbial. accessory, essential. accessory, essential. accessory, accessory, adverbial. essential. direct. adnominal.' direct. adnominal.^ adnominal. adnominal. adverbial. predicate, appositive.^ predicate. predicate. T3redicate. predicate, aiipositive.* nominative, independent.^^ ^ To Lord. 7/e is a resumptive appositive. ~ Of taken, to he supplied, s jjy direct ad- dress. ^By pleonasm. ^ By accompanying circumstance. * To iorcJ; but it would not be objectionable to take Lord and Anglesey together; since they are, in all respects, alike. ' Of the comparative adjective wiser,* see Dem. XXX.,m. ^ Of parsed. ^To possession or choice, to be supplied; see Dem. V., Obs. e. 'o See Dcm. V., Obs. c. i' Of extewcied, to be supplied. ■fi By exclamation. i3 See Dem. XXVII. , h, i. srsi ""EM ORj iMMAR— GRADE A. 615 Pr og. XXVIII.- —Branching by the Grammar- System Tree. 1. Is = verb, finite. variable. indicative. 3. Give = verb, finite. invariable. imperative. 4. Were = verb, finite. invariable. subjunctive. 5. Would = verb, finite, variable. indicative. 6. Shed = verb. infinite. infinitive. verbal comp.. assertive. 9. Isi = verb, finite, variable. indicative. 10. Disposed = verb, infinite. participial. verbal comp. 11. Censure = verb. infinite, infinitive. substantive. 13. Are = verb, finite. variable. indicative. 14. Fallen = verb. infinite. participial, verbal comp. 15. CaU = verb. finite. invariable. subjunctive. 16. TeU = verb, finite. invariable. imperative. 18. Aral = verb. finite. invariable. indicative. 19. Invited = verb. infinite, participial. verbal comp. 20. Attend = verb. infinite. infinitive. substantive. 22. ShaU = verb. finite. variable. indicative. 24. Had = verb, finite. invariable, subjunctive. 25. Believed = verb. infinite, participial. verbal comp. 26. Would = verb. finite. variable. indicative. 31. Having = verb. infinite, participial. adjectival. 82. Finished = verb, infinite. participial. verbal comp. 33. Assigned = verb. infinite, participial. adjectival. 37. Was 2 = verb. finite. variable. indicative. 38. Raised = verb. infinite, participial. verbal comp. 40. Is 3 = verb. finite. variable, indicative. 41. Learn = verb, infinite. infinitive. substantive.* 44. Succeeded = verb. infinite, participial, verbal comp. 47. Go = verb. infinite. infinitive. substantive. 49. Forget = verb. infinite, infinitive, verbal comp.. assertive. 50. Mention = verb. infinite, infinitive. substantive. 54. Obliged = verb. infinite. participial. verbal comp. 58. Were = verb. finite. variable. indicative. 60. Conquer = verb. infinite. infinitive. substantive. 64. Rising- = verb, infinite. participial. adjectival. 68. Fall = verb, infinite, infinitive. verbal comp.. objective. 70. Come = verb. infinite. infinitive. verbal comij.. assertive. 71. Smiling = verb. infinite. participial. adjectival. 73. Invited = verb. infinite. participial. verbal comp. 77. Appointed = verb. infinite. IDarticipial, adjectival. 78. Crushed = verb. infinite. participial. adjectival. 81. Cease = verJj, finite. invariable. imperative. 85. Eating = verb. infinite, participfftl, adverbial. 1 Public is singular or plural according as it represents one or more than one subject- unit. 2 "Hue and cry" represent only cue subject-unit, s" Bread and butter,'' one food. ^ Learn is the object of the preposition to, which joins the^aW, learn, to the sentence whole. 616 THE SYSTEM METHOD. Prog. XXIX.— Branching by the Grammar- System Tree. 1. Sweet =: accessory, adjective. predicate. 4. The = accessory, adjective. adnominal. 5. Ripe == accessory, adjective. predicate. 6. How = accessory. adverb. modifying. 7. Often = accessory. adverb. modifying. 8. There = accessory, adverb. modifying. 9. Sometimes = accessory, adverb. modifying. 10. Very = accessory. adverb. modifying. 11. Well = accessory. adverb. modifying. 15. English =: accessory, adjective. adnominal.^ 17. Methodical .= accessory. adjective. predicate. 23. Happily = accessory. adverb, modifying. 25. Long- = accessory, adverb, modifying. 26. Enough = accessory, adverb. modifying. 28. Where = accessory. adverb. conjunctive.^ 29. Out = accessory. adverb. modifying. 31. These = accessory. adjective. adnominal. 33. Which = accessory, adjective. adnominal. 35. First = accessory. adjective. adnominal. 36. Far == accessory. adverb. modifying.^ 37. Pretty = accessory, adverb, modifying. 40. Other = accessory. adjective. adnominal. 41. Each = accessory. adjective. adnominal. 43. Quiet = accessory. adjective. appositive. 48. Frivolous = accessory, adjective. appositive. 49. None = accessory, adjective. adnominal.* 51. Own = accessory. adjective, adnominal. 53. To = accessory. adverb, modifying. 54. Fro = accessory, adverb. modifying. 56. The 5 = accessory. adverb. modifying. 57. Louder = accessory. adverb. modifying. 58. The = accessory, adverb. modifying. 60. Where = accessory. adverb. conjunctive. 61. Of = accessory. adverb. modifying." 64. Clever = accessory. adjective. appositive. 66. Whither — accessory. adverb. conjunctive. 69. Right = accessory. adjective, appositive.'^ 71. Long = accessory. adverb. modifying.^ 83. All = accessory. adjective. appositive.^ 'So people, to be supplied. ^3oivls is and you laid, and modifies laid. ^Far mod- ifies the preposition «& ,_, o H ^ M m O ^ f O ^ 2i 0116, arlj:^ "^^^^/. '^^1% DEVELOPMENT OF ARITHMETIC. 643 8-20, and 8-21, as a whole, cannot be wrong, since they are per- fectly correlative to 7-16, 7-17, and 7-18. (b.) Limbs 21-36, 21-37, 21-38, 21-39, and 21-40 are five of what ought to be a whole of seven perfectly similar limbs, (1.) be- cause their cousin co-ordinates, 19-22, 19-23, etc., and 20-29, 20- 30, etc., are two such wholes of seven perfectly similar (system- fitting) parts; (2.) because they are the offspring of 8-21, and 8- 21, 8-20, and 8-19 are complemental thirds of what forms one or- ganic whole as complete as the animal or the plant whole. Since, then, limbs 21-36, 21-37, 21-38, 21-39, and 21-40 are lawful, i. e., system-fitting, these four unlawful, disorganizing, or systemless, limbs can themselves also become syscem-fitting, i. e., scientific, by becoming like 21-36, 21-37, 21-38, 21-39, and 21-40. They must^ therefore, read " 100 per cent^'' etc., instead of " 1 per cent," etc., and " 12 months," etc. The right-hand half of these Case III and Case IV (so-called) solutions must therefore be so changed that they will start at top with " 100 per cent," and stop at bottom with "rate." Whatever, therefore, is done in rectifying Cases III and IV and reducing both to one, one thing the tree avers must be done: 100 per cent must be made the right-hand member, or half, of the first of the three equations embraced in the solution form proper. Now of the three terms ($300, 22 m., and 833) mentioned in the 5th problem above, but one could constitute 100^ in any possible solution — the term $300. Placing this term, then, as equal to the 100^ to be placed at the right hand, the author proceeded thus since he could not proceed otherwise: — a. $300 = 100% b. 1 = i c. 33 = 33 X i = 11%- But if this is the organizing, or system-fitting, solution, it will, as repeatedly demonstrated by the tree, show a correspondence of parts with those solutions (Cases I and II) with which it is to form a complete whole of correspondents. It was therefore to be com- pared with Case I and Case II solutions. So compared, it was seen to be Case I turned end for end, and Case II turned upside down. Eut such was the very circumstance of harmony of parts upon which the construction of the tree was started and so far devel- oped. It is therefore the organizing, or science-system, method. And to complete it will be easy; since it must correspond to Case I, ^44 THE SYSTEM METHOD. of which it is merely a reversion, end for end. Since these three equations are the reverse of three of Case I, the complemental fourth here must likewise be the reverse of the remaining fourth of Case I. But this remaining equation of Case I was — 6 per cent x 2 2 '\^ O/ 12 — ^^/O- But this was developed, at the dictation of the tree, thus: Since 6% is the interest for 12 m., divide by 12 for 1 month's interest, and then multiply by 22 for 22 months' interest, which equals 11^ of principal. The jjrocess reversed would therefore be-^ 1 1 per cent X 1 2 — . Cio/ 2 2 ^/Cl which is the answer sought. "With this complemental fourth equa- tion supplied, solutions 5th of Case III and 7th of this so-called Case IV are thus corrected to the organizing method: — System-Method Solution to 5th. System-Method Solution to 7th. (^To find the rate.) (To find the time.) a. $300 =100% a. $300 = 100^ b. 1 = i b. 1 = i c. 33 = 33 X i = 11% c. 33 = 33 x i = 11% ^_ iipercentxi2 = g % ratc. X. ^ ^ m, x 1 1 _ £2 m. time. Thus was the fourth Case embodied in the third. And it has already been seen under Case II of this solution, that the supposed Case Y was brought under the reach of that Case (II). The law- breaking 4-limb branch fell into branch 8-21 as tAVO law-abiding, or system-fitting, parts, branch 8-21 being thus divorced of those in- congruent parts by reason of which it was near to be dismembered from its would-be co-ordinates, 8-20 and 8-19, and outlawed from the tree system. Percentage, gain and loss, commission, taxes, interest, true discount, bank discount, money exchange, as well as fractions, longitude and time, all comparative measures, compound and simple proportion, square root and cube root, were now substantially mas- tered by mastering simple proportion alone; since all could be solved by this one system method. All rules and formulas were eliminated fi'ora, system-driven out of, the science, as plundering caterpillars in the arithmetical tree, as vermin to be smoked out of the schools. Complete Scheme of Solutions. — We shall here give the complete system scheme of arithmetical solutions, by what the author has called the system method, side by side, that their uniform- ity may the more plainly appear. In it will be found, it is be- lieved, no two solutions incongruent with each other; nor will any DEVELOPMENT OF ARITHMETIC. 645 diversity appear that is not diversified uniformity merely, not incongruity. The solutions will be given in the order in which the subjects to which they respectively belong should always be taught. ISTor let it be erroneously suspected that these solutions do not form all the organizable types belonging to the complete system of arithmetic. ]N"o truly arithmetical problem can be found, the sys- tem solution to which is not precisely like some one of the solu- tions given below, for the following — PROBLEMS. a. Reduction Ascending.— 1. In 228 pt. how many quarts? 2. In 228 pt. how many bushels ? b. Proportion. — 3. If 13500 bricks are required to build a wall 200 ft. long, how many bricks will be required to build a wall 600 ft. long? 4. If it requires 13500 bricks to build a wall 200 ft. long, 20 ft. high, 16 in. thick, each brick being 8 in. long, 4 in. wide, and 2 in. thick, how many bricks will be required to build a wall 600 ft. long, 24 ft. high, 240 in. thick, each brick being 10 in. long, 5 in. wide, and 3.5 in. thick? C. Fractions : Case I. — 5. If an entire crib of corn is worth $100, what will f of it cost ? Case II. — 6. If f of a crib of corn is worth $75, what is the entire crib worth ? Case III.—I. S75 is what part of $100? d. No-Time Percentage : C'ase /.— 8. 11 per cent of$300 = what? Case IL—9. $33 is 11% of what? Case ///.— 10. $33 is how many per cent of $300 ? e. Time Percentage : Case I. — 11. What is the interest on $300 for 22 m. at 6%? Case 77.— 12. The interest at 6% for 22 m. is $33. What is the principal ? Case III.— 13. The interest on $300 for 22 m. is $33. What is the rate ? Case III.— 14. The interest on $300 at 6% is $33. What is the time ? 6^6 THE SYSTEM METHOD. SYSTEM-METHOD SOLUTIONS. 1st — Reduction. a. 2 pt. := 1 qt. b. 1 " = 1 c. 228 " = 228 X i = 114 qt. Bemark. — It will be observed that solution 2d is simply solution 1st three times repeated ; and that, therefore, learning solution 1st learns solution 2d. 3d — Simple Proportion. a. WaU 200 ft. long = 18500 b. b. " 1 " " = 2^0 X 13.500 = ip c. " 600" " = 600 X ^F = 40500 b. 2d- -Reduction. 2 pt. = 1 qt. 1 u = 1 228 u = 228 X h 8 qt. = 1 pk. 1 (1 = 1 8 114 u = 114 X i 4 pk. = 1 bu. 1 " = 1 14 u = 14 X i = 114 qt. = 14 pk. 2 qt. Mark that the left-hand part of solu- tions 3d and 4th are identical ; that sol. 4th is sol. 3d simply repeated six times ; that sol. 4th is therefore really mas- tered bv mastering sol. 3d. WaU 200 lon^ 1 " 600 20 high, 1 " 24 " 4th — Compound Proportion. 16 thick; brick 8 long, 4 wide, 3 thick 240 5tti — Case I Fractions. = $100 = 1 X 100 = $25 $75. ■| of crib XOP, I " " = 3 X 7tfi — Case III Fractions. $100 = f , whole. 1 — _i_ V A — -1- '- 100 '^ 4 100 10 13500 bricks. 1 " 1 " — 13 500t'>^8x 4x2 2 0x2 0X1 6 [= 13.5. 5 " 3.5 " = [ 13^5.b.x6 0^x2 4x2 4 =266605f. 6th — Case II Fractions. $75 i X 75 = $25 4 X 25 = $100. •| of crib 75 75 X TTo = I. part. Remark. — Observe carefully that Case III is Case I turned end for end., while Case II is Case I turned upside down. See exactly the same correspondency of parts among the three cases of percentage, in- terest, and discount below. Consider, then, how easy to learn Cases II and III after learning Case I. Nor let the reader pass by till he clearly understands how these three cases are merely one in different positions. Case I being a cat I on its feet, head north. Case III same only on back, head south, and Case II also same as cat I only on back. Consider especially how these three cases of fractions are identical with the following three cases of percentage, in- terest, and discount— how, therefore, learning fractions by this system method really learns percentage, interest, discount, exchange, etc., also; just as learning one quadruped in three positions learns cdl quadrupeds in the same circumstances. 8th- -Case I Percentage. (No-time division.) a. 100% = $300 b. 1 = 3 c. 11 = 11 X 3 = 33. 9ih — Case II Percentage. (No-time division.) 11% =$33 1 = J- X 33 = 3 100 =$300. DEVELOPMENT OF ARITHMETIC. 647 10th — Case III Percentage. Eemark.—See remark under the three cases of frac- CNo-time- division ) tions above. Ponder how it is the very purpose of science to put this unity in every subject studied, so that we can a. 8300 = 100% '■'see the one inthe many.'"— Sir Wm. Ilamilton. "The b. 1 = i mind only knows when it subdues [masters] its objects, „ 33 __ 33 -v 1 __ 1 1 ^ when it reduces the many to the one."— ^Mazagforas. And ^ so Plato, Leibnitz, Kant, Aristotle, Spencer, Ueberweg, and many others. But see how these three cases again are identical with the three cases into which this system, or organizing, method divides all interest, discount, exchange, and U. S. government securities, etc., so that while learning fractions learns also percentage, learning fractions and percentage learns interest, discount, etc. nth — Case I Interest, etc. (Time division.) ■^ 6 per cent x L^2 11/9/ A. • J- 2 = yi-yo a. 100% =$300 b. 1 =3 c. 11 = $33 13th— Case III Interest, etc. (Time division.) a. $300 = 100% b. 1 = i c. 33=33xi = ll% X. 11 p'^'- cent X 12 = (j^ rate. Take notice how not only the a-b-c parts of these solutions per- fectly articulate with each other when clasped in each other's arms, but that X of solution 11th is identical with x of solution 12th; that X of either the 13th or the 14th is the reverse of x of the 11th; and that a, b, c, of the 14th is a, b, c, of the 13th with no change, not even in position. The System Method Established — the Question Set at Rest. — What is true is a branch of all truth; and every branch has many fibers. Two facts colluded to establish this a-b-c solution form as the tree trunk of arithmetic, and thereby to put this organizing method forever beyond dispute as the scientific, natural, or system, method — and therefore the safe and easy bus- iness method — of solving all truly arithmetical problems. These facts were derived from the lingual and natural branches of the great tree. By these means it was established that the focus, or tree trunk, — 1. Of orthography is letter, a part of which etymon is the in- cluding whole. 2. Of etymology is etymon, a jDart of which word is the includ- ing whole. 12th- —Case II Interest, etc. (Time division.) X. 6 per cent x 22 1 1 /^ 12 ^^/O a. 11% = $33 b. 1 = 3 c. 100 = $300. 14th- -Case III Interest, etc. % (Time division.) a. $300 = 100% b. l=i c. 33 = 33 X i= 11% X. i2nuxii ^ 22 m. time. (348 THE SYSTEM METHOD. 3. Of grammar is word, a part of which sentence is the inchid- ing whole. 4. Of botany is plant-organ, a part of which plant is the includ- ing whole. 5. Of zoology is animal-organ, a part of which animal is the in- cluding whole. Now, since in these branches the units letter, etymon, word, plant-organ, and animal-organ are each a part articulating per- fectly with other parts to form the including whole, by the law of unity, whatever constitutes the system unit, or trunk, of arith- metic — a. Must not only form part of an including whole, but, as a part, it— b. Must articulate harmoniously with the other parts with which it forms such whole. The arithmetical tree had said that this a-b-c, or 3-step-equation, form of solution is the trunk of a perfect system. Then, de- clared the lingual and natural trees, every complex (though ra- tional) solution must be made up of such 3-step parts repeated — repeated as part to form the complete, or whole, solution of such problem. That every whole solution based on the system of arith- metic is but the repetition of this 3-step form, find here below demonstrated: — 1. What is 11% of 1300? [One a-b-c step only.] 2. What is the interest on |300 for 22 m. at 6% ? [Two a-b-c steps.] 1st Soluiion. 2d Solution. One Step like Case I. 1st Step like Sim. Proportion. a. 100% = $300 a. 12 m. of int. =6% b. 1 = 3 b. 1 " " = i c. 11 = 11 X 3 = $33.— J.MS. c. 22 " " =22X\ = 11%. Solution 1st is the form of all simple solu- 2d Step — like Case I. tions, being tlie a-b-c step taken but once. But see in 2cl, 3d, 4th, and 8th how this a-b-c step re- ^- 100% = ^300 peated forms the tvhole of even the most prolix b. 1 = 3 or k.nottj solutions— when they are fully written c_ ii -.. ii -y 3 = $33. Ans. out. 3, What rate of interest do banks make when they discount '^ "^/g 3 day notes at 2% a month ? [Three a-b-c steps.] 4, By discounting notes at 2% a month, a bank realizes 25^% interest on its loans. How long do the notes discounted run? [Two a-b-c steps.] DEVELOPMENT OF ARITHMETIC. 649 Note. — Since bank discount always exceeds simple interest by the interest on the interest for the time to run, it is evident that l^f-^ (^SjyV — 2% a month) is the sim- ple interest on 24% (2% a month) considered as a principal. We have then: The principal 24%, the interest l^W, the rate 25j2^5_ to find the time. We start by Case III of simple interest. 5. A man bought a farm for $6000, and agreed to pay principal and interest in 4 equal annual installments. The interest being 6 ^ , what is the amount of the equal annual payments ? [Four a-b-c steps.] 6. If it requires 13500 bricks to build a wall 200 ft. long, 20 ft. high, 16 in. thick, each brick being 8 in. long, 4 in. wide, and 2 in. thick, how many bricks, each 10 in. long, 5 in. wide, and 3.5 in. thick, will be required to build a wall 600 ft. long, 24 ft. high, and 20ft. thick? a. Wall 200 ft. long, 20 ft. high, 16 in. thick; brick 8 in. long, 4 in. wide, 3 in. thick, [= 13500 bricks. ■5^ It j^ u {< -^ a u -^ u u £( 1 " "1 " " 1 " " r_ 13500 b . X 8 x4x2 — 13 5 c. " 600 " " 24 " " 20 ft. " " 10 " " 5 " " 3.5 " " [= 13.5b^x6 |.0| 2^4x2 4 =, 266605 ^ 4th Solution. 1st Step like Case III. a. 24 = 100% b. 1 r= 2T X 100 = -V- C 1 -Ap- 5.0 4 V -2 5 210 26, Step like Siinpile Proportion. a. 25 j2_59% = 13 m. b. 1 = eis X 100 b. ^1^ = ^^i^^ X 12 = xoV(T c. -2jVT = 3100 X ToVo- = 3-1 m.—Aiis. 5th Solution. 1st Step like Case III. Note. — The interest paid 2d. yr. is less than interest paid 1st yr. by interest on"portion of principal paid 1st yr., and int. paid 3d yr. is less than int. paid 2d yr. by int. on portion of principal paid 2d yr. ; and so on. But the payments are to be equal. Therefore — 1st Step like Case II. a. 106%, prin. paid 3d yr. = 100%, prin. paid 4th yr. 3d Solution. 1st Step like Simple Proportion. a. 12 m. int. = 24% b. 1 " = 2 c. 2.1 " = 2.1 X 3 = 4.2. 2d Step like Case III. a. 95.8 =100% b. Sd Step like Simple Proportion. 2.1 m. = W/% -I In/ 2100 1000 ■L — 2.T X -jysr — -JT9" 12 = 12 X W# = 25 ^\%.—Ans. 1.00 106 10_0 1-06 b. 1 •' c. 100 " " = k%°e " 3d jr. 2d Stqj like Case II. 106%, prin. paid 2d yr. = ^."^^g prin. paid 3d yr. 1 u u loo_ " '■: ^ — 10 6';l-G 6 100 " " = j.oir.oe " 3d yr. 650 THE SYSTEM METHOD. 3d Step like Case II. a. 106 prin. paid 1st yr. = x.o6xT%6 P"ii- P'^^'^ '^^ J"- •u 1 U U JOO U 41 D- J- 106>Ll-06xl-06 c. 100 " " =T.o6i.76.T.o6" 1st yr. )6il.06«1.06 to. b. hj these solutions it was thus demonstrated that every problem to be ex- plained or solved by the system of arithmetic may be solved by means of this a-b-c solution repeated. And thus it was finally established, not only that this a-b-c part was part of an including whole, the complete solution, but that every a-b-c part of a solu- tion did harmoniously ar- ticulate with other a-b-c forms organizable through the tree system. The question whether the -S- step solution form was properly the tree trunk of the arithmetical system was permanently put at rest; for the evidence that the 3-Btep solution is the unit of the arithmetical system rested on precisely the same foundation as the proof that letter and word are the system units of orthography and grammar. And who is able to show that letter and word are not the science units on which these branches are branched? Whoever succeeds in such an undertaking, will succeed in proving that M'hile words are aspi- rates, vocals, dentals, diphthongs, etc. ! letters are nouns, verbs, con- junctions, subjects, predicates, adverbials, objects, etc.! — a tran- scending accomplishment. 4th Stejo like Simj^le Proioortion. a. 367. 3 fo = $6000 b. 1 = 3 6T.3- X 6000 = $16.3354 -|- c. 106 = $1633.54 -\-.—Ans. 6th Solution. 1st Step like Simple Proportion. a. Wall 200 ft. long = 13500 bricks. b. " 1 " := 2^0 X 13500 = if5- c. " 600 " = 600 l^A = 40500 b. Sncl Step like Simpde Proportion. a. WaU 20 ft. high = 40500 b. "1 " = 2^0 X 40500 = 2025 " 24 " = 24 X 2025= 48600 b. Sd Step like Simple Proportion. WaU 16 in. thick = 48600 " 1 " " = Jg X 48600 = 3037|- c. "240" " =240X30371 = 729000. 4th Step like Sim,ple Proportion. a. Brk. 7 in. long = 729000 b. " 1 " " = 8 X 729000 = 5832000 c. " 10 " " = 583200. 5th Step like Simpde Proportion. a. Brk. 4 in. wide = 583200 b. " 1 " " = 4 X 583200 = 2332800 c. '■ 5 " " =466560. 6th Stej) like Simple Proportion. a. Brk. 2 in. thick = 466560 b. " 1 " " = 2 X 466560 = 933120 c. " 3.5 " " = 2666055— J.,iA-. DEVELOPMENT OF ARITHMETIC. 651 " Short Methods." — All education is a taking out of the will and a putttng down into unconscious reflex action of the brain; therefore finished arithmetical training will — by the similarity- means, by always having solved all problems in the same way — have taken the a-b-c solution out of the will, and so sunk it into the reflex centers of the brain that the student will be unconscious that there is any form about it. He will then have condensed the solution above into — a. The 2d: " 11 times $3=$33 (^m.)"— a "lightning" method. b. The 3d: "24 x i|.%% = 25^^^ (^ws.)"— a "hghtning" method. But we are to reach these lightning (i. e., condensed) processes through system. We are therefore never to teach them until after the system method is fully mastered. No " lightning " process is safe or uninjurious that is not merely a condensation of the system, or scientific, process. Teach no isolated process. This the teacher need have no occasion to do, in fact, since the system, or organiz- ing, method, when simply condensed, or abbreviated, is invariably the shortest process possible. I would, then, teach no lightning method until after my pupils had mastered the system method. I would have all "lightning" work derived directly from this system method by condensing. Life preparation in school is properly divided into two, which are, — 1. Empirical Training of the Child = age 1 to 10. This first period is to be given to empirical education wholly. The whole object here is to sink the following activities out of the will into reflex action so that the child may (1.) walk, (2.) talk, (3.) read, (4.) write, (5.) sing, (6.) calculate (without symbols), and (7.) labor without any vexing of the will. These activities are those num- bered. Constant doing in these activities is to lead to the power of doing them while the will sleeps. It is also to gradually win the child from his natural repugnance to severe labor. Having thus acquired a love to " do his best,"— to put in a hearty effort,— he is to go to this second period, — 2. Rational Education of the Youth = age 10 to 25,— to begin this work in learning concisely by means of systems. But after having been consciously guided by system for the greater portion of thi^lO to 25 years of age, the system itself, though always em- g52 THE SYSTEM METHOD. ployed in doing, is to melt away from student's consciousness, just as our feet and hands, though constantly used, are rarely thought of. Now, when, like the hands, ears, and feet, the organizing method as guided by the system is well known and itself handled with dispatch, then, but not till then, should lightning processes be employed; and even then as a condensation merely of such system method as a " hop-step-and-jump " system method. But there are some specific questions which ought here to be answered: Is it desirable to reduce this rational a-b-c process to an empirical process, as the author has done in interest, discount, etc.? He thinks it is; for a business man must be able and ready to figure without nicety and convenience — in his head, on his shoe-heel, or thumb-nail. And when we have learned to add and substract, notation is naturally sunk into reflex action as thereafter an empir- ical process; when we have learned to m.ultipiy and divide, addition and subtraction are so treated; similarly, when we begin to reason in the a-b-c syllogism, multiplication and division are to be so far sunk out of the will centers that the will may be wholly engaged in the one matter of reasoning. And similarly always: since all solutions, simple and complex, are really wrought out by a repeti- tion of this a-b-c solution form, and since solving problems is forever a repetition of this process, it can and should be so sunk into reflex action as to render it, finally, an empirical process. The System Method— No. 22. THIS STTTIDEniTT'S Work-Book of System Arithmetic, Grade B, rOK THE USE OP Public Schools, Normal Schools, Teachers' Institutes, and Colleges, AND ESPECIALLY ADAPTED TO Soi3:ooxj IE]:2^i=^osiTionsrs : IN WHICH, BY MEANS OF THE 3-STEP SOLUTION, THE ARITH- METICAL TREE TRUNK, ALL PROBLEMS ARE SOLVED IN THE SAME WAY. BY ISAAC ELDRIDGE WILSON, INVENTOR OF THE LEARNER'S WORK-TREE, AUTHOR OF THE SYSTEM METHOD, A SERIES OP TEXT-BOOKS FOB SCHOOLS, A SERIES OP WORK-BOOKS FOR SCHOOLS, ETC. ' THE MAN MAY TEACH BY DOING, AND NOT OTHERWISE."— EMERSON. AUG 13 1886 y ^WASHI OHIOA.OO, ILIv.: Learner's Work-Tree Company. COPYRIGHT, 1886, BY ISAAC ELDRIDGE WILSON. ALL RIGHTS RESERVED. Any infringement will be prosecuted to the full extent of the taw. System Arithmetic- Grade B. COMPOUND NUMBERS. Reduction Ascending". 1. Keduce 192 pt. to bushels. a. 2 pt. = 1 qt. b. 1 " = i c. 192 " = 192 X i = 96 qt. a. 8 qt. = 1 pk. b. 1 " = i c. 96 " = 96 X i = 12 pk. a. 4 pk. = 1 bu. b. 1 " = i c. 12 " = 12 X i = 3 bu. — Ans. Remarlc —It will be observed that each of the 3-step, or a-b-c, parts of this 9-step solution is in itself a smaller solution whole, included within the larger 9-step solution whole. It is this 3 step solution part which forms the trunk of the arithmetical tree, page 642, and by which the system method undertakes to solve all arithmetical problems whatsoever in the same way Since this 3-^tep solution is to be the student's instrument with which to solve all prob- lems properly belonging to the system of arithmetic, and with which he is to displace formulas and rules, he should, at the outset, become thoroughly familiar with it. 2. In 228 pt. bow many bushels? a. 2 pt. = 1 qt. b. 1 " = i c. 228 " = 228 X i = 114 qt. a. 8 qt. = 1 pk. b. 1 " = - c. 114 " = 114 X i = l^pk. • • 2qt. 4 pk. = 1 bu. a b. 1 " = i U a = i4,x \ = Shu. . . 2ph c. 3. Eeduce 532 gi. to gallons. a. 4 gi. = 1 pt. b. 1 ■' = \ c. 532 " = 532 X i = 133 pt [655] 656 THE SYSTEM METHOD. a. 2 pt. = 1 qt. b. 1 " = 1 c. 113 " = 133 X i = 66 qt. . . Ipt. a. 4 qt. = 1 gal. b. 1 " = i c. m '•' = QQ X \: = 16 gal. . . 2qt. 4. In 31708810 sec. how many years? a. 60 sec. = 1 min. b 1 " = JL "•J- 6 c. 31708810 " = 31708810 x -^V = 528480 min. . . 10 sec. a. 60 min. = 1 hr. 1 6 b. 1 " = c. 528480 " = 528480 ^ = 8808 hr. a. 24 hr. = Id. b 1 " = -i- U. A 2 4 c. 8808 " = 8808 x 2^^ = 367 d, a. 365 d. = 1 yr. b. 1 " = -L "• -^ 36 5 0. 367 " = 367 x 3^3- = i 2/r. . . 2d. 5. Eeduce 121888 oz. to tons. a. 16 oz. = 1 lb. b. 1 " = -1- c. 121888 " = 121888 x iV = 7618 lb. a. 100 lb. = 1 cwt. b. 1 " = 1- "• ^ 100 c. 7618 " = 7618 X li^ = 76 cwt. . . 18 lb. a. 20 cwt. = IT. b. 1 " = JL "' ^ 2 c. 76 " = 76 X 2V = ^ ^- • • 16 lb. 6. How many miles is it through the earth from pole to pole, the distance being 41707308 ft.? First Solution. 5280 ft. = 1 mi. 1 " = 1 c. 41707308 " = 41707308 x tjW = 7899 mi. 588 ft.— Jtws. SYSTEM ARITHMETIC— GRAJDE B. 657 Second Solution. a. 3 ft. = 1 yd. b. 1 " = 1 c. 41707308 " = 41707308 x i = 13902436 yd. a. 5. 5 yd. = 1 rd. b- 1 '' - Jg c. 13902436 " = 13902436 x 5 ^ = 2527715 rd. . . S.S ff. a. 40 rd. = 1 fur. b. 1 " = -i- • 4 c. 2527715 " = 2527715 x J-q == 63192fur. . . 35rd. a. 8 fur. = 1 mi. b. 1 " = I c. 63192 " = 63192 X i = 7899 mi. N'ote. — The inquiry will arise whether " reduction descending " should not pre- cede "reduction ascending." To which query we reply, Yes ; reduction descending involves but two of the three equations in the 3-step solution ; it embraces naught, therefore (as may be seen from the following example), — Frdblem. — Reduce 3 bu. 2 pk. 2 qt. 1 pt. to pints. Solution. — b. 1 bu. = 4 pk. c. 3 " = 3 X 4 = 12 . . 14 pk. b. 1 pk. = 8 qt. c. 14 " = 14 X 8 = 112 . . 114 qt, b. 1 qt. = 2 pt. c. 114 " = 114 X 2 = 228 . . 229 i^t.—Ans. but simple addition and multipUcation, and is, accordingly, to be taught when and as these simple empirical processes are taught— empirically, and in the lower grades. On the other hand, reduction ascending, as may be seen in the solutions under that heading (p. 655), involves the 3-step solution complete; so that the reasoning process is identical with the reasoning process in solutions of problems in proportion, frac- tions, percentage, profit and loss, commission, insurance, taxes, interest, discount, money exchange, etc. Now, since it is the very undertaking of the arithmetician to put such system into all these problems that, if possible, they may be solved by one and the same process, — since it is the business of arithmetic to solve the greatest pos- sible number in the same class by the same method, — evidently the proper method of solution in reduction ascending is that method by which proportion, fractions, per- centage, profit and loss, commission, insurance, taxes, interest, discount, money ex- change, etc., are solved. Therefore, the successful teacher will be guided by these two principles : (1.) Reduction descending may be taught at any time during and after the time when simple addition and multiplication are taught; (2.) Reduction ascending should be taught by that solution, which, once learned for reduction ascend- ing, is also learned for all these remaining branches of arithmetic; namely, by the 3- stcp solution. In so far as reduction ascending is entirely like proportion, fractions, 42 658 THE SYSTEM METHOD. percentage, profit and loss, conunission, insurance, taxes, interest, discount, money- exchange, etc., to teach it otherwise than as these are taught is folly. To teach what is like proportion and percentage as unlike proportion and percentage, is to teach what is like a cat as being unlike a cat ! is to teach a wild-cat, not as a cat, but as a cow, horse, whale, or other unlike animal ! Let it nere be observed, once for all, that by " rational process" we mean in this book, the 3- step-solution process — the process by which all arithmetical problems not simple addition, subtraction, multiplication, and division problems, may be solved in one way. Any of the 3-step, or a-b-c, solutions contained in these (Nos. 22 and 23) work-books, will illustrate, then, what we mean by the " rational process," or the proc- ess of "rational arithmetic," as we may term it. The following four solutions will illustrate what we mean by the " empirical process." Observe that the rational proc- ess employs the three simple equations, and reasons from many to one and thence again to many; that the empirical process employs simple notation, addition, subtrac- tion, multiplication, and division: — 1st Problem. — Reduce 18 bushels to pints. Explanation. — Since 1 bu. = 4 pk., 18 bu. = 18 times 4 pk. = 72 pk. ; and since 1 pk. =8 qt., 73 pk. = 72 times 8 qt. = 576 qt. ; and since 1 qt. = 3 pt., 576 qt. = 576 times 2 pt. = 1152 pt. Or, since 1 bu. = 64 pt., multijsly 64 pt. by 18, which gives 1152 pt. as before. This is sometimes called Reduction Descending. — Raifs New Higher Arith- metic, p. 154. Empirical Solution. 18 bu. 4 72 pk. 576 qt. 2 2nd Problem. — Change 20 mi. 70 rds. 5 ft. to feet. Explanation. — 30 times 320 rds. = 6400 rds., to which the 70 rds. are added. Again, 6470 times 16| ft. = 106755 ft., to which the 5 ft. are added. Observe that multiplicand and multiplier are in- terchanged in the operation. — Wentworth and HilVs Practical Arithmetic, p. 149. 1152 pt.— ^ws. Em.pirical Sohition. 20 mi. 70 rds. 5 ft. 320 6470 rds. 161- 106760 ft. — Ans. 3d Problem. — Reduce £23 4 d. 3 far. to farthings. Explanation. — £1 equals 20 s. Therefore, £33 equal 23 times 20 s. ; but, as either factor may be made the multipUer, for convenience we say 30 times 23, or 460, s. We have now reduced the given num- ber to 460 s. 4 d. 3 far. 1 s. equals 12 d. Therefore, 460 s. equal 12 times 460, or 5520, pence, which with the 4 d. make 5534 d. We have now reduced the number to 5524 d. 3 far. 1 d. equals 4 far. Therefore, 5534 d. equal 4 times 5534, or 22096, farthings, which, with the 8 far., make 23099 ta,T.—Quackenbos's Higher Arithmetic, p. 195. Em,pirical Solution. £33 4 d. 3 far. 20 460 s. 12 5524 d. 4 23099 far. —Ans. SYSTEM ABIT EMETIC^ GRADE B. 659 4th. Pkoblem. — How many drams in 343 tons? Ex2jlanation.~l ton, as was learned from the Empirical Solution. table of equivalents memorized under Avoirdupois Weight, equals 512000 drams. Therefore, 343 tons ^^^^"^ equal 343 times 512000 drams. To multiply 343 and 686000 512000 together, is shorter than to multiply succes- „^^^ sively by 20, 4, 25, 16, and 16, according to the rule. _! MTien no intermediate denominations ai'e given, one 175616000 dr. — Ans. multiplication, by the number of equivalent units, may, as in this case, most easily effect the reduction. — Quack€nbos''s Higher Arithmetic, p. 195. COMPARATIVE MEASURES. 1. 20 lb. Troy = how many lb. Avoirdupois ? X. 20 lb. @ 5760 gr. = 115200 gr. a. 7000 gr. = 1 lb. Avoir. b. 1 " = 1 c. 115200 " - 115200 X ^^^ = 16i| lb. Avoir.— ^,25. 2. Change 4 cwt. 72 lb. 8 oz. to Troy pounds. X. 4 cwt. 72 lb. 8 oz. = 4721 lb. Avoir, y. 4721 lb. @ 7000 gr. = 33"b7500 gr. a. 5760 gr. = 1 lb. Troy. b. 1 " = _J c. 3307500 " = 3307500 X ^J^ = 574^^ lb. Troy.-^^^. 3. What is the value of a coflfee urn weighing 2 lb. 14 oz. Avoir. @ $1.80 per oz. Troy ? X. 2 lb. 14 oz. Avoir. @ 7000 gr. = 20125 gr. a. 480 gr. = 1 oz. Troy. b. 1 " = -1- c. 20125 " = 20125 X^ = 41ff oz. Troy, y. 41ff oz. @ $1.80 = ^lbA^.—A7^s. 4. A grocer bought 33 bu. of berries at $2.25 per bu., and sold them, through mistake, by wine measure, at 8/ per quart. What did he gain or lose ? X. 33 bu. @ 12.25 = $74.25 cost. y. 33 bu. @ 2150.4 cu. in. = 70963.2 cu. in. a. 57.75 cu. in. = 1 Avine qt. b. 1 " = _i_ c. 70963.2 " = 70963.2 X ^J.^^ = 1228.8 wine qt. z. 1228.8 qt. @ 8^ = $98,304 b. price. p. $98,304—174.25 = $24.054 gain.— ^w^. QQQ THE SYSTEM ME TROD. 5. An apothecary bought 14 lb. of opium by Avoirdupois weight, at 621/ per oz., and retailed it at 5/ a scruple. How much did he gain ? X. 14 lb. @ 16 oz. = 224 oz. y. 224 oz. @ 621/ = $140 cost. z. 14 lb. @ 7000 gr. = 98000 gr. a. 5760 gr. = 1 lb. Apoth. '-'•-'• 57 6 c. 98000 " = 98000 x 57W = 1^T2 ^^- ^poth. p. 17JI_ lb. @ 288 sc. = 4900 scruples, q. 4900 sc. @ 5/ = $245 s. price. r. $245 — $140 = $105 gSiin.—Ans. 6. How many wine gallons in a 3-bushel measure ? X. 3 bu. @ 2150.4 cu. in. = 6451.2 cu. in. a. 231 cu. in = 1 wine gal. h 1 " = -i— ^- ■■• 2 3 1 c. 6451.2 " = 6451.2 X all =27.92+ wine gal.— ^m. 7. A bin 8 ft. long, 2 ft. wide, 5 ft. deep, will hold how many bushels ? how many wine gallons ? how many pounds (Av.oirdu- pois) of distilled water.? X. A bin 8 ft. by 2 ft. by 5 ft. = 80 cu. ft. = 138240 cu. in. a. 2150.4 cu. in. = 1 bu. b 1 " = L-_ '■'• ^ 2 15 0-4: c. 138240 " = 138240 X -2x^-4 = 64.285+ h\i.—Ans. a. 231 cu. in. = 1 Avine gal. b 1 " = -J— 2 3 1 c. 138240 " = 138240 X ^ = 598.441 w. gixl—Ans. a. 27.72 cu. in. = 1 lb. (Avoir.) distilled water. b 1 '• = __i '-'• '- 2 7-72 c. 138240 '' =138240 x ar^.-^^^ 4987.01+ lb. dis. water. [ — Ans. 8. A tank 4 ft. wide and 3 ft. deep holds 4500 lb. of water. How long is it, water weighing 621 lb. per cu. ft.? a. 62.5 lb. = 1 cu. ft. b 1 " = _L_ "• ^ G 2 - 5 c. 4500 " = 4500 x -gi.g = 72 cu. ft. a. 12 cu. ft. of water = tank 1 ft. long. b 1 " " " — "1 a <; c. 72 " " " ^ « 72 X tV = 6 ft. long.— Ans, SYSTEM ABITHME TIC— GRADE B. QQi LONGITUDE AND TIME. ITote. — By an examination of tiie solution below, it will be seen that Class I solu- tions, in Longitude and Time, involve naught but the fundamental processes of addi- tion, subtraction, multiplication, and division. These Class T problems, therefore (in which the time is given to find the longitude), are to be taught and learned as what they are, — mere empirical processes, a simple application of what is generally called the "four fundamental processes," — and not as an application of the 3-step solution, which involves much more than the four simple empirical processes. CLASS I PROBLEMS. 1st Problem. — The difference of time between two places is 3 hr. 20 min. What is the difference of longitude? b. 1 hour = 15° c. 3 " = ,S X 15 = 45° b. 1 min. = 15' c. 20 " = 20 X 15' = 300' X. 450 -\- 300' = 50°— 4ws. Since they do not embrace the 3-step solution complete, and involve nothing over aud above the four fundamental arithmetical operations, these Class I problems cannot come within the branch Reduction Ascending as rational, or system-guided, processes, but belong in empirical arithmetic as a simple application of addition, sub- traction, multiplication, and division, to be learned during and after the time when these simple operations are learned. See the note, pages 657, 658, wherein is given an imperfect explanation of what is the difference between 3-step-solution problems and empirical-solution problems. Since these Class I problems, of Longitude and Time, are very similar to addition, subtraction, multiplication, and division problems, and since they are entirely dissimilar to reduction-ascending, proportion, fraction, percentage, gain-and-loss, commission, insurance, taxes, interest, discount, and money- exchange problems, and since they cannot be solved by the 8-step solution, therefore they are to be solved by the only other solution possible, the empirical solution illus- trated in the problems below solved. But since Class II problems, of Longitude and Time, are v£ry similar to reduction-ascending, proportion, fraction, percentage, gain- and-loss, commission, insurance, taxes, interest, discount, and money-exchange prob- lems ; and since they are entirely dissimilar to, and include much more than, mere addition, subtraction, multiplication, and division problems; since they cannot possi- bly be solved by these fundamental processes merely, without the S-step-solution proc- ess of reasoning in an organism from many to one and thence to many, therefore they are to be solved by the 8-step solution. The following solutions will still further illustrate what the empirical solution to these Class I problems is: — 2nd Problem. — When it is 9 o'clock at Washington, it is 7 min. 4 sec. past 8 o'clock at St. Louis. Find the difference of longitude. 663 THE 8T8TEM METHOD. Empirical Solution. Explanation.- Since every hour of time corresponds to 15° of long., and every min- 9 hr. min. sec. ^^^ ^^ ^^^^ y^ -^5/ ^j j^^g^ ^^^ g^^j.^ g^^. * end of time to 15" of long., there are 15 52 56 diff. in time. times as many deg. min. and sec. in the 15 difference of longitude, as there are hr. TZZ 777 ~„ -..^ . . min. and sec. in the difference of time. 13° 14 00 diil. m long. /~w J^e•^. • r/> Or, since 4 min. of time make a difference Or, 4)52 mm. 56 sec. ' of 1° of long., and 4 sec. of time a differ- 13° 14' / ence of 1 of long., there wiU he \ as many degrees of long, as there are minutes of time, and \ as many minutes of long, as there are seconds of time. — 2iobinson''s Complete Arithmetic, p. 235. 3d Problem. — The difference in time between two cities is 1 hr. 35 min. and 12 sec. What is the difference in longitude ? Empirical Solution. 1 hr. 35 min. 12 sec. Exp)lanation. — Multiply by 15 as in compound multipli- ]_^ cation. — White's Complete Arithmetic, p. 133. 23° 48' 0"—Ans. 4th Pkoblem. — The master of a ship finds his time (by the sun) to be 2 hr. 16 min, later than New-York time (indicated by his chronometer). New York being 74° 34" W., what is the longitude of the ship ? Em,X)irical Solution. 2 hr. 16 min. diff. in time. 74° 00' 24" 15 34° 00' 00" 34° 00' diff. in long. 40° 00' 24" W.— J.ws. 5th Problem. — A navigator finds that when it is noon where he is, it is 2 min. 1 sec. past 5 by his chronometer, which is set by (and indicates) Greenwich time. What is his longitude? Empiirical Solution. 5 hr. 2 min. 1 sec. diff. in time. 0° 00' 00" 15 750 30' 15" 75° 30' 15" diff. in long. 75° 80' 15" 'W.—Ans. 6tu Problem.— a and B sailed together from San Francisco (long. 122° 27' 49" W.). A kept his watch by San Francisco time, and B set his by the sun each day. After 10 days, A's watch was 4hr. 39 min. faster than B's. In what longitude were they? 4 hr. 39 min. diff. in time. 122° 27' 49'' 15 69° 45' 69° 45' diff. in long. 192° 12' 49" W. or 167° 47' 11" E.— Jws. SYSTEM ARITHMETIC— GRADE B. 663 CLASS II PROBLEMS. 1. The longitude of Cincinnati is 84° 29' 45" W.; of San Fran- cisco, 122° 27' 49" W. When it is noon at San Francisco, what is the time at Cincinnati ? X. 122° 27' 49" — 84° 29' 45" = 37° 58' 4" diff. of long, a. 15° = 1 hr. c. 37° = 37 X T^ = 2 hr. 28 min. a. 15' = 1 min. b 1' = J- c. 58' = 58 X 1I5 = 3 min. 52 sec. a. 15" = 1 sec. b 1" = -1- c. 4" = 4 X -jig- = -^ sec. y. 12 M. + 2hr. 31 min. 52 ^^ sec. = 2hr. 31 min. 52/g^see. p. M. [ — Ans. 2. When it is 6 a. m. at Boston, long. 71° 3' 50" W., what is the time at Cincinnati? X. 84° 29' 45" — 71° 3' 50" = 13° 25' 55" diff. of long, a. 15° = 1 hr. b 1° = -L "■ -^ 15 c. 13° = 13 X JL. ^ 52 min. a. 15' =1 min. b 1' = J- c. 25' = 25 X y^^ = 1 min. 40 sec. a. 15" =1 sec. b 1" — J- u. J. — J 5 c. 55" = 55 X ^ig- = 3| sec. y. 6 A. M. — 53 min. 43|sec. = 5 hr. 6 min. 161- sec. A. m. — \^Ans. 3. A telegram sent from Paris at 7 p. m. reaches New York in 2hr. 55 min. At what o'clock, New- York time, is it received? (For the longitude of these cities, see table, in Work-Book No. 23.) X. 74° 24" -f 2° 20' = 76° 20' 24" diff. of long. a. 15° = 1 hr. b 1° — -L c. 76° = 76 X 1I5 = 5 hr. 4 min. QQ4: THE SYSTEM METHOD. a. 15' = 1 min. b. 1' =1^ c. 20' = 20 X Jg- = 1 min. 20 sec. a. 15" = 1 sec. b. 1" = J- ^ 15 c. 24" =24 X iV = If sec. 7- 9 h. 55 min. p. m. (time at Pari [=4 hr. 4 5hr. 5 min. 21-|sec. 1. 381 sec. P. M. — Ans. SIMPLE PROPORTION. 1. If a farm of 160 acres rents for $450, how much should be charged for one of 840 acres? a. 160 a. = $450 b. 1 - = ^ X 450 = If c. 840 " = 840 X f I = 12362.50.- Ans. 2. If a cane 3 ft. 4 in. high held upright, casts a shadow 2 ft. 1 in. long, how high is a tree whose shadow at the same time is 25 ft. 9 in.? a. 25 in. shadow = 40 in, hight. b. 1 " " = 1 V 4.0 — 8 c. 309 " " = 309 X -I = 494| in. = 41 ft. 21 in. his^ht—Aois. o o 3. A garrison of 560 men have provisions to last during a siege, at the rate of 1 lb. 4 oz. a day per man ; if the daily allowance is reduced to 14 oz. per man, how large a reinforcement could be received ? a. 20 oz. per day = 560 men. b. 1 " " " = 20 X 560 = 11200 c. 14 " « " = J^ X 11200 = 800 X. 800 — 560 = 240 size of reinforcement. — Ans.^ Note. — If only 1 oz. were allowed, 20 times as many men (20 X 560 = 11200) could be maintained as if 20 oz. were given to each man. Then if 14 oz. were given each day, only Jj as many men (Jj X 11200 = 800) could be maintained. The size of the reinforcement which can be received is therefore (800 — 560) 240 men. 4. A clock which loses 5 minutes a day, was set right at 6 in the morning of January 1st. What will be the right time when that clock points to 11 a. m. on the 15th? SYSTEM ABITHME TIC— GRADE B. 665 X. 11 A. M. Jan. 15th — 6 a. m. Jan. 1st = 14 d. 5 hr. a. 24 hr. = 5 mm. lost. b. 1 " = 2^ X 5 =2T c. 341 " = 341 X 2^4 = 71 min. 21 sec. lost. y. 11a. m. -f- 71 min. 2-1- sec. = 11 min. 2i see. p, m. — Ans. 5. Two bodies free to move, attract each other with forces that vary inversely as their weights. If the weights are 9 lb. and 4 lb., and the smaller is attracted 10 ft., how far will the larger be attracted ? a. 4 lb. = 10 ft. moved, b. 1 " = 4 X 10 = 40 ft. c. 9 " = ^ X 40 = 4 ft. 5-1- in. distance moved. — Ans. Note. — The force of attraction varies inversely as the weight of the bodies; i. g., as the weight is increased, the distance moved is decreased, and as the weight is decreased, the distance moved is increased. Thus it is that a body weighing only 1 lb. is moved four times as far as a body weighing 4 lb. ; and that a body weighing 9 lb. is moved only one-ninth as far as a body weighing 1 lb. 6. If our present calendar gains on the exact year 2 hr. 26 min. 40 sec. in 400 years, how long will it be in gaining a day ? X. 2 hr. 26 min. 40 sec. = 8800 sec; 1 d. = 86400 sec. a. 8800 sec. gained = 400 yr. time. b 1 " " = _i V 400 = J- U. i — 88OOA ^^-l^' 2 2 c. 86400 " " = 86400 x 2V = 392733- yr. time.— Ans. iVote.— See p. 700. 7. A railroad company has work for 250 men 90 daj's. How many men must be discharged that the remainder may have em- ployment for 150 days? a. 90 d. = 250 men. b. 1 " = 90 X 250 = 22500 c. 150 " = ylo X 22500 = 150 men. X. 250 — 150 = 100, no. of men to be discharged. — Ans. Note. — If 250 men can be employed for 90 d., to finish the work in one d. would require (90 X 250) 22500 men. But the work is to occupy 150 d. ; therefore only yig- of 22500 men (number required to do it in 1 d.), or 150 men, can be employed. The number of men to be discharged is then (250 — 150) 100. 8. A man borrows $1750, and keeps it 1 yr. 8 m. How long- should he lend $1200 to compensate for the favor? QQQ TEE SYSTEM METHOD. a. $1750 = 20 m. b. 1 = 1750 X 20 = 35000 c. 1200 = jJq^ X 35000 = 291 m. = 2 yr. 5 m. 5 di.—Ans. Note. — To realize the same amount of interest from $1 as from $1750, tlie $1 must be on interest 1750 times as long (1750 X 20 m. = 35000 m.) ; but to gain the same amount of interest, $1200 must be on interest only xsVo ^^ ^'^'^S ^® ^1 (tAtt X 35000 = 291 m.) 9. If I gain $160.29 in 2 yr. 3 m., what would I gain in 5 yr. 6 m. at that rate ? a. 27 m. = $160.29 b. 1 " = -i^x 160.29 = 5.93| c. QQ " = 66 X 5.93| = $391.82 gsan.—Ans. 10. A has cloth at $3.25 a yard, and B has flour at $5.50 a ban-el. If, in trading, A puts his cloth at $3,621, what should B charge for his flour? a. $3.25 value = $3,625 asked. b. $1 " = ^ig X 3.625 = l.l-j% c. $5.50 " = 5.50 X 1.1^ = $6.13y6^ asked.— ^«6'. 11. If 16 per cent of a number is 80, what is 50 per cent ? a. 16 per cent = 80 b. 1 " " = Jg X 80 = 5 c. 50 " '' = 50 X 5 = 250.— ^Tzs. FRACTIONS. 1. The whole of a ship is worth $10800. What is | of it worth ? 1. Case I Solution. a. f of ship = $10800, the whole. b. 1 " " = i X 10800 = 1200 c. I " " = 5 X 1200 = $6000, the part.— ^?zs. Note. — Read and study carefully "The Sj'stem Method" following the solution to problem 3d below. 2. I of a ship cost $6000. What did the whole ship cost? 2. Case II Solution. a. f of ship = $6000, the part. b. I " " = i X 6000 = 1200 c. f " " =- 9 X 1200 = $10800, the whole.— ^?^s. iVote.— Read and study carefully " The System Method " following the solution to problem 3d below. SYSTEM ARITHMETIC— GRADE B. 66^ 3. $6000 is what part of $10800? 8. Case III Solution. a. $10800, the whole, = |, |, f, etc., the whole. ^- ^1 = 10800 c. $6000, the part, = 6000 x yimro = h ^^^ part.— ^ws. The System Method.— That these three solutions possess the organiz- ing virtue of the true system method, plainly appears in that solutions 1st and 2d are both learned in the study of either one alone, since solution 2d is identical with solution 1st turned upside down, and the only real difference between solution 3d and solution 1st is that solution 3d is solution 1st turned end for end. Thus it is that the mastery of three is well-nigh accomplished in the study of one alone ; pre- cisely as the perfect mastery of both lungs may be well-nigh accomplished in the study of one alone. And let this grand thought not escape the reader; namely, that unless both lungs or eyes are thus well-nigh perfectly learned in the study of one alone, the study of the one itself is not system-led, is not correct ; for all learn- ing and mental power consists in that very thing, that we shall see the system in what we study — that we may know, or have power over, many in knowing the one alone. Therefore, the very fact that solution 2d may be quite well mastered in the study of solution 1st alone, and that solution 3d is well-nigh mastered in learning either solution 3d or solution 1st, — this fact itself is the best possible evidence that these 3-step solutions are but the complemental thirds of the perfectly organizing method ; precisely as the very fact that both lungs are learned, in any trial method of study, in the learner's study of the one alone, would be the best possible proof that his studies, or solutions, of the two lungs are but the complemental halves of the per- fectly-organizing method; for, I repeat, herein is precisely what learning is, namely, the becoming able to know, or solve, or manage, the many in learning the one. Thus it is that this system method, by means of the 3-step solution, accom- plishes the very purpose of science, which is to put such a unity into the multitude of objects that we can manage all in the same way; or, " see the one in the many." — Sir Wm. Hamilton. " The mind only knows when it subdues its objects [solves its problems], when it reduces the many to the one [reduces the many solutions to the one solution]." — Anaxagoras. And similarly testify Leibnitz, Kant, Aristotle, Spencer, Ueberweg, Butler, and every educational reformer. But let it not pass un- observed that the 3-step solution not only organizes all fraction problems into three solutions, all of which are practically mastered in learning any one alone, — let it be observed that this 3-step solution not only reduces the many solutions of fractions to one simple animal, as it were, but that it also reduces all percentage problems, all gain-and-loss problems, all commission problems, all insurance problems, all interest problems, all bank-discount and true-discount problems, all money-exchange prob- lems, and all government-security and stock problems to the same simple animal form: so that learning fractions or percentage or interest or any other of these sub- jects practically learns them all. Refer to solution 1 (Case I Solution) above. Call f the right fore leg of a cat standing on the opposite bank, from you, of a clear body of water, so that you may QQg THE SYSTEM METHOD. see its picture plainly reflected from the water below ; and call f the left fore leg oi the same cat standing on the bank; then $10800 would be the right hind leg, and $6000 the left hind leg. Now, call solution 1 such a cat, head to the reader's left hand and with equation b for its body, and solution 2 will be the same animal reflected as a picture from the water below, and solution 3 will be the same under-the- water animal picture, on its back, as solution 2, but with head turned to the reader's (observer's) right hand. Thus we see that this 3-step solution is an organism, since its parts correspond to each other, and its different positions fit to each other, as nat- urally as do the parts (limbs and organs) of an animal whole, — as naturally as does the reflected picture to the animal pictured. (And it is an interesting truth that the fact that this 3-step solution is an organism, is proof as perfect as proof can be made that such 3-step solution is the perfect — that is, the organizing, or natural — - method of solution.) The 3-step solution, therefore, becomes, in a way, a sensible representative of what an arithmetical solution is. An arithmetical solution, then, is an animal, a quadruped, if you please. And, if so, it ought to be just as much easier and more rational to learn and understand a solution in arithmetic as such 3-step solution quadruped whole, than it is to learn and understand a solution as the disorganized something generally used in the schools, — just as much easier as it is easier and more rational to learn and understand a cat as an organized quadruped whole than it is to learn and understand a cat as a disorganized something — as parts disorganized and therefore parts of no whole! as parts that are not parts! (For what such learning is worth, see pages 90-94 inclusive.) Only Three Cases. — There are, of course, as many different cases as va- riations of the 3-step solution; for were there absolutely no variation for different problems, were all problems in fractions and percentage solved in absolutely the same method, why, we should solve them all in the never-varying way, and there could be but one case of either fraction or percentage subjects. But I have shown (see the article above, "The System Method") that the 3-step solution is like an an- imal organism, and that the three variations are but the same organism in different positions to the mental eye ; so that when we have found how many different posi- tions this 3-step organism may assume, we have found how many cases there are — how many cases will be employed in that method which solves aU problems in the same way. Now, it was shown in the figure of a quadi-uped beast, with its picture reflected from the water below, and reflected in two different positions, that there are just the three positions, or variitions, in this 3-step solution. These variations are already given for fractions, page 666, being there called Case I, Case II, and Case III solutions. Whatever problem, properly belonging to any other branch of arith- metic, does not prove to be, when solved, one of these three solutions, belongs in simple proportion. But this simple-proportion solution, as commonly found in frac- tions and percentage, is reaUy but two of the 3-step solutions locked up together into one. Such a problem, solved below, first, as two 3-step solutions, and, secondly, as such a simple-proportion solution, here follows : — Problem : i,f- is what part of -UJ-? SYSTEM ARITHMETIC— GRADE B. 669 Solved in Two 3-step Solutions. a. iii, the whole, = |, |, |, etc. "•11 — TTT c. 11 number compared, = 11 X tit = ttV- a. f, the whole, i= JJy hi ivii 11 c. -y-, number compared, = 13 X £¥¥ = is- — ^"«' Solved in One Simiiile-Proportion Solution. '^- 11 — 2 h 1 — iiy 2 11 u- J- — XTT A 2 • — TTT O _13. 13 V 11 11 Anv >-• 2 — 2" A TTT ■ — TS- — -^nb. In the same way we shaU find, in percentage and all the remaining subjects of arithmetic, that these same three cases, into which every remaining arithmetical subject will divide itself, wiU solve all problems except such as properly come under simple proportion. l¥liere Sbould Fractions be Taught ?— Fractions are to be taught precisely where they are found — in connection with whole things. Indeed, they can be learned no place else ; for a fraction is a fraction, and can be understood as such, only by virtue of its being part of some whole. Whoever undertakes to teach the parts fi'actions, as disconnected and separate from the wholes of which they are fractions, or parts, is justly chargeable with the same foUy as he who undertakes to teach the parts nose, eye, ear, etc., as disconnected and separate fi-om the whole (the man) of which they are parts. It has been pointed out, in several ways, in this book, how nonsensical such teaching and learning are. Consider what it is to study and vainly pretend to learn a part as a separate thing: suppose you had studied nose, ear, eye, brain, backbone, etc., a Ufetime as separate things, but not as parts, as they really are. Suppose, I say, that you had studied these things a lifetime, but knew them not as parts ; that is, had failed to learn them as parts of the head-whole and man-whole ; of course, then, you would not know the prominent thing in the middle front of the face to be nose ! for you learned nose not as part of the head, but as a separate thing! Neither could you know that the two little rolling balls in the sockets at the sides of the nose are eyes ! nor that the two things projecting from the sides of the head are ears ! nor that the backbone is any. part of a man ! for all these things you learned, not as parts, but as separate things ! How unreasonable, then, how laughable, how feeble, to talk about learning fractions whoUy by themselves, and not as associated with whole numbers, of which wholes they are parts ! Fractions are therefore to be taught as the learner is becoming familiar with whole numbers, in the lower school grades; but the pupil is there to learn empirical fractions merely, not the science system (i. e., the 3-step-solution system) of fractions. AU arithmetic, like every other branch of learning, is first to be learned empirically (as we learn notation, addition, subtraction, multiplication, division, cancellation, and reduction descending) , and then much of it is to be learned rationally (by the 3-step- solution process). To specify: First, the learner is to be taught fractions as parts of things, — apples, matches, sticks, crayons, etc., — as parts to be understood only as asso- 6Y0 THE SYSTEM METHOD. dated with whole things. This first teaching -will be done in the primary grades dur- ing, and especially in the last part of, the object-lesson exercises to be given to the beginner in arithmetic. Secondly, the learner is to learn fractions, as he must natu- rally do, in connection Tvith the exercises in short and long division; because here he is repeatedly using such expressions as, " |^ of 10 = what? " " \ of 200 = what? " Observe how this early study of fractions, in these primary object lessons, and in division, is to acquaint the learner with the simple fraction |, \, \, -5^, etc., as used in Eeduction Ascending, which is the first arithmetical subject wherein he uses the 3-step solution. Observe that in this first, or reduction-ascending, 3-step solution, the only fraction used is the very simplest (|, ^, j^q, etc.) of all fractions, the fi'ac- tion having the unit, 1, for its numerator, and with which the learner is already made familiar by reason of his object-lesson exercises in fractions, and his examples in sim- ple division. This, then, is the order in which fractions, empirical and rational, are to be taught and learned: (1.) Fractions in simple object lessons, to teach what they are and how to write, reduce, add, and subtract them; (3.) Fractions in division ex- ercises; such as, J of 20 = what? (3.) Fractions in Eeduction Ascending, by there using the simple ones (|, |^, -^ig, etc.) in the 3-step solution; (4.) Fractions (these same simple ones) in Proportion, simple and compound; (5.) Fractions as a branch of the science system of arithmetic; that is, fractions by the three cases of the 3-step solution; (6.) Fractions as occmTing incidental and side-show-like to all the remain- ing arithmetical subjects, — -percentage, gain and loss, commission, insurance, taxes, interest, true and bank discount, money exchange, etc. In a word, fractions are to be taught in the order in which the different arithmetical subjects are presented in the perfected "Arithmetical Work-Ti'ee," page 642, beginning with the right-hand lowermost limbs, and going right around toward the left thus : Reduction Ascending, Proportion, Umbs 2-4, 4-7, 7-16, 7-17, 7-18 ; 4-8, 8-19, 8-20, 8-21, with their sub- divisions. And of the six steps here above mentioned, the first four are wholly em- pirical, the fifth wholly rational, the last mixed. Lest the author should not be definitely understood, through the arithmetical work-tree and what has been said, in regard to the proper order in which to teach the different subjects of arithmetic, and as to what is chiefly empirical and what chiefly rational (by the 3-step solution) in arithmetic, he will here indicate the proper order and the proper division into empirical and rational : — a. EMPIRICAL PROCESSES. 1. Object lessons in numbers, whole and fractional. 2. Notation (writing) of numbers, whole and fractional. 3. Addition and subtraction of numbers, whole and fractional. 4. MultipUcation of numbers, whole and fractional. (This step would include also such problems as the 3d and 4th problems of "Inquiry 2d," page 721 .) 5. Division of whole numbers; fractions proper are ah-eady divided, since they are quotients. (This step would, of course, include also such improper-fraction problems as the 1st of "Inquiry 2d,'' p. 721.) 6. Cancellation of numbers, whole and fractional. (Such problems as the 4th and 5th, page 721, will also be included and may be reviewed here.) SYSTEM ARITHMETIC— GRADE B. 671 7. Reduction Descending of numbers, whole and fractional. (The 2d of "In- quiry 2d," page 721, would also be included in this step.) b. 3-STEP SOLUTION (OR RATIONAL) PROCESSES. 1. Reduction Ascending of numbers, by the 3-step solution. 2. Proportion (simple and compound) of numbers, by the 3-step solution. 3. Fractions, the three cases by the 3-step solution. 4. Percentage, the three cases by the 3-step solution. 5. Profit and Loss, the three cases by the 3-step solution. 6. Commission, the three cases by the 3-step solution. 7. Insurance, the three cases by the 3-step solution. 8. Taxes, the three cases by the 3-step solution. 9. Interest (simple and compound), the three cases by the 3-step solution. 10. Discount (bank, true, and trade), the three cases by the 3-step solution. 11. Miscellaneous, (a.) Money Exchange; (b.) Arbitration of Exchange; (c.) Stock Investments; (d.) Government Securities; (e.) AlUgation; (f.) Square and Cube Root; (g.) Algebraic Problems. Difference Between the Three Cases and Simple Pro- portion. — It may be seen from limbs 7-16, 7-17, and 7-18 of the tree, page 642, or from the first three solutions in fractions, page 666, that that which distinguishes each of the three cases in fractions is as follows: (1.) Case I begins, in left-hand member of equation a, with |, |, |, etc., and stops, in left-hand member of equation c, with a fraction: (2.) Case II is the reverse of Case I, beginning with a fraction, and stopping with f or |-, |-, etc. ; (3.) Case III begins with some number used as a whole, and stops with some number compared with that whole. And, similarly, what distinguishes the three cases of percentage may be seen likewise to be a difference in the left-hand ends, or halves, of the fii-st three 3-step solutions on page 677; for Case I solutions begin (in equation a) with 100% and stop (in equation c) with the rate. Case II solutions begin with the rate and stop with 100 % , and Case III solutions be- gin with the number used as a whole and stop with some other number which is com- pared to that whole. (See this same thing written on 8-19, 8-20, and 8-21 of the tree, page 642.) Now, whatever 3-step solution has not this threefold left-hand half so constituted that it thus corresponds either to one of the three fraction, or to one of the three percentage, hmbs above mentioned,— every such problem belongs in sim- ple proportion. Accordingly, among the Case I, Case II, and Case III solutions in fraction and percentage subjects, in this book, will be found "simple-proportion" solutions. By this explanation the author wishes to get it squarely before the reader's mind that there are such things as simple proportion of fractions and simple proportion of percentage and simple proportion of true and bank discount, of money exchange, etc., as well as simple proportion of whole numbers ; and that simple proportion of whole numbers alone is to be presented to the learner previous to when fractions, percent- age, etc., shaU have been presented and learned. But as soon as the thi'ee cases proper of fractions, percentage, discount, exchange, etc., have been mastered, then the simple proportion of such subject may be taken up and mastered. By simple proportion we do not mean, of course, the old equation of ratios, but the 3-step solution. gY2 THE SYSTEM METHOD. 4. What will f of a yard of cloth cost at $f per yard ? Case I Solution. a. I yd. = $1 ^- "8 8-^6 4 8 c. I " =^ 'J' X A = $!!•— ^'-'s- 5. What is I of i|? Case I Solution. ■*• 3 ~ 16 K 1 = 1 V 15 — 5 ^•3 3^16 16 c. I = 2 X tV = H-— ^*^«- 6. A 63-galloa cask is | full ; 9i gallons being drawn off, how many gallons does it contain ? Case I Solution. a. f of cask = 63 gal. c. I of cask = 5 X ^ = 39| gal. X. 39|- gal. — 91 gal. = 29|- gal. remaining. — Ans. 7. A bill of books amounts to $596|, but I get i off for whole- sale, and -f-^ off for cash ; what do I pay for the books? Case I Solution. ^•\ + -h = Th amount off. a. iff of bill - $2^ h 1 " " = _i_ V 2387 = 2387 *-^*150 150'^ 4 600 c. t¥o " " = 53 X Wf = $210fii amt. off y. $596| — $210 fi^ = $385|||- amt. \^QA±—Ans. 8. How much is f of a vessel worth, if the whole vessel is worth $29951.88? Case I Solution. a. 5. of vessel =$29951.88 b. i " " = i X 29951.88 = 4278.84 c. f " " = 5 X $4278.84 = $21394.20.— J tis. 9. A teamster hauled 7486 bushels of potatoes to a railroad depot, receiving in payment -^^ of them. How many bushels did he receive? Case I Solution. a. ifl-g- of the potatoes = 7486 bu. b. ^ " " " = xio X 7486 = 74.86 c. i|(^ " " " = 9 X 74.86 = 673.74 h\i.—Ans. SYSTEM ARITHMETIC— QBABE B. 673 10. If I of a yard of cloth can be bought for | of a dollar, what is that per yard ? Case II Solution. a. 1 yd. = $"5" b. i - = ixl = 1 c. f " = 3 X 1 = $H. -Ans. 11. "Il of what number equals 9i|? Case II Solution. a. 1^ of a number = Xi^ 7T 35X18 — TS" c. ^ << " = 72 X A = 20.— ^«s. 12. If |- of a bbl. of flour cost %^-^, what is the price per barrel? Case II Solution. a. f bbl. = $-V- b-l" — 1 y 3 1 —31 af" = 7 X li =110.85.— ^ws. 13. |- of 42 is -j^ of what number ? Case II Solution. X. f of 42 = 18 a. ^ of a number = 18 b. J_. « " = -1- X 18 = 9 c. li " <' =11x9= d^.—Ans. Note. — ^Equation x of the above solution is, in reality, a shortened Case I solu- tion. Made out in full, this solution would correspond to the solution of problem 5 above, which see. After Case I solutions are mastered, the shortened form used in the solution above may be employed. 14. A farmer sold I of his farm for $1645. At this rate, what was the value of the farm ? Case II Solution. a. I of the farm = $1645 \yiuu u = I X 1645 =5481 c. I " " " = 5 X 5481- = $2741|.— ^w*. 15. A graded school enrolls 208 boys, and y^^ of the pupils are girls. How many pupils are enrolled in the school? 43 (3Y4 THE SYSTEM METHOD. Case II Solution. X. 1|, the whole school, — -jL = ^, the boys. a. -jSg- of the school = 208 b. tV " " " = i X 208 = 26 c. 1^ " " " = 15 X 26 = 390.— ^TOs. 16. A man owning |- of a ship sells f of his share for |3480. At this rate, what is the value of the ship? Case II Solution. -sr 3 of 5. = _5_ X. Y «.H 9 2 1 a. ^5_ of the ship = $3480 b. JL « « " = i X 3480 = 696 c. |i " " " = 21 X 696 = 114616.— ^ws. 17. What part of 120 quarts are 31 quarts? Case III Solution. a. 120 qt,, the whole, = |, |, f, etc., the whole. c. 31 " the part, = 31 X y^-j- = -^V' *^® P^^^- — ^**** 18. ytt ^^ what part of y®^? Case III SohUion. a. -^-^, the whole, = |, |, |, etc., the whole. '-'• 1 7 7 8 c. YTT, the part, = 3 x -jq) = toj ^^® P^^*- — ^*^*- 19. A farmer gives a miller 13 quarts of grain for every 3 bush- els ground. What part of the grain does the miller receive ? Case III Solution. X. 3 bu. @ 32 qt. = 96 qt. a. 96 qt., the whole, = |, -|, f, etc. b 1 " = J- U. J. — 9 g c. 13 " the part, = 13 X -Jg- = if. — Ans. 20. If 9 gallons are drawn from a cask containing 63 gallons, what part of the whole remains ? Case III Solution. X. 63 gal. — 9 gal. = 54 gal. left, a. 63 gal. = f , |, |, etc. c. 54 " = 64 X -^ = f, part remaining. — Ans, SYSTEM ARITHMETIC— GRADE B. ^75 21. A pole whose hight was 80 ft. was broken off by the wind 48 ft. from the top. What part of the pole was left standing ? Case III Solution. X. 80 ft. — 48 ft. = 32 ft. remaining, a. 80 ft. = I, I, f , etc. b 1 " = -i- ^- ^ 8 c. 32 " = 32 X -^ = I left standing.— ^«5. 22. A man having $150, gave $25 for a robe, and |- of the re- mainder for a harness. What part of $150 had he left? Case III /Solution. X. 1150 — $25 = $125 remainder. y. I of $125 = $50 for harness, z. $125 — $50 = $75 money left. a. $150 = I, I, f, etc. b. l=Tio c. $75 = 75 X xl-o = ^, part left. — Ans. 23. A 63-gallon cask is I- full; 9-1- gallons being drawn oflF, how full will it be ? Case III Solution. X. I of 63 gal. = 39f gal y. 39f gal. — 9i- gal. = 29|- gal. = ^^ gal. remaining. a. 63 gal. = |, |, |, etc. b. 1 " = J3 c. ^fS- " = 2 39 X Jg. = 2||. Ml— Ans. 24. If I of a farm is worth $9000, what is j\ of it worth ? Simple- Proportion Solution. a. I of farm = $9000 b. 1 = I X 9000 = 13500 c. 3^ = ^5_ X 13500 = $5625.— ^?zs. Note. — Consult the article "Only Three Cases," page 668, where it is shown that such as the above and the following "simple-proportion" solutions are really two 3- step solutions locked up together in one. Let it be carefully explained to the young learner how from the statement that " | of a farm = $9000," we conclude that " 1 (whole) farm = f times $9000;" and so likewise carefully each of the following solutions. 25. If f of 9 bu. of wheat cost $13^ what will | of a bushel cost ? gY6 THE SYSTEM METHOD. Simple- Pro^iortion Solution. X. f of 9 bu. = 2T. bu. a. ^7- bu. = 1131 b. 1 " = ^ X 13|- = 2 c. -z- " = I X 2 = $1|.— ^?js. 26. One-eighth of a dime is what part of 3 cents? Simple-Proportion Solution. X. i-oflO/ = |/ a- 3/ = I, I, A etc. b. 1 =1 27. If I of a mill is worth $2500, what is f of the mill worth ? Simple-Proportion Solution. a. I of mill = $2500 b. 1 " = f X 2500 = 4500 b. I " = f X $4500 = $3750.— ^ws. 28. If I of a mill is worth $5225, what is f of the remainder worth ? Simple- Proportion Solution. X. f of I" (the remainder) = i- a. I of mill = $5225 b. 1 " = f X 5225 = 9405 c. ^ « = -g- X 9405 = $3135.— ^ns. 29. What will 15 hhd. of molasses cost, if 281 gal. cost $71? X. 15 hhd. @ 63 gal. = 945 gal. a. ^ gal. = $^ b. 1 « = ^x^ = i c. 945 " = 945 X 1 = %2m\.—Ans. 30. How many men will it take to do a piece of work in 12|- d. which 81 men can do in 6|- d.? a. ^- d. = 81 men. b. 1 " = ^ X 81 = 540 c. !L0 u = ^ >^ 540 = 42 men.— ^ws. SYSTEM ABITHME TIG— GRADE B. 677 PERCENTAGE. 1. What is 8^ of $500? 1. Case I Solution. a. 100% = $500 b. 1 = 5 c. 8 = 8x5== %4.Q.—Ans. 2. $40 is 8% of what? 2. Case II Solution. a. 8% = $40 b. 1 = i X 40 = 5 c. 100 = $500.— ^«5. 3. $40 is how man^^ % of $500 ? 3. Case III Solution. a. $500 = 100% b- 1 = 5-^ X 100 = i c. 40 = 40 X 1- = 8 %.—Ans. Note 1. — The important thing for the teacher to do in beginning any new sub- ject in arithmetic, is to make his pupils see that the subject, strictly speaking, is not new; but that it is the same old thing they have been before studying, except it has a new name. For instance, in the three problems above, change "8% " and "how many % " to "xfo" (common fraction) and " how many lOOths " (common fraction) , and these three cases become identical in both their form and solution with the first prob- lems of fractions, page 666. Now, this same old thing that solves all arithmetical problems in the same way, is the 3-step solution ; and the three cases of fractions, of percentage— of all these subjects — are but different positions, or variations, of the 3-step solution. (See " The System Method," " Only Three Cases," and "Difference between the Three Cases and Simj^le Proportion," pages 667, 668, 671, respectively.) Now, we are not only to learn all arithmetic in learning these three cases above, but we should endeavor to learn the three cases in learning one case alone ; for, as already shown, Case II solution is the same thing as Case I solution turned upside down, and Case III solution is also identical with Case I solution turned end for end. This 3-step solution as much remains the same thing in its three positions, or vai'iations, as a quadruped remains the same thing in three different positions. And mark well that solving a problem is merely finding a missing part of this 3-step-solu- tion organism, — finding a missing leg, if you please, — which missing part is always found in the right-hand member of equation c, and forms the answer to the problem. Indeed, in solving a problem of system arithmetic, the true arithmetician (system- atizer) is as much concerned in finding an organic missing part as is a comparative anatomist in determining from some given animal parts what another part should be to fit such animal whole. ' gY8 THE 8T8TEM METHOD. j^Qte 2. — Young learners beginning at percentage, and not having before become acquainted with the 3-step solution in fractions or proportion or reduction, may meet, at the outset, with some difficulty in determining what quantity to write as the right-hand member (or base term) of the first equation. Therefore the really suc- cessful teacher will never fail in his preliminary drill, on the day previous to the first recitation day, to get before his pupils these two facts: — 1. That which is to form the right-hand member, or base term, of equation a, will always be of the same denomination as the answer required; for the answer, the part missing, corresponds to this right-hand member of equation a precisely as hind leg corresponds to hind leg, or fore leg to fore leg. 2. Of the diilerent quantities given in the problems, commonly but one wiU thus be of the same denomination as the answer required. Ordinarily, therefore, his attention having been called to these facts, the student will have but little difficulty in determining what shall be the right-hand member (or base term) of equation a of his solution. (See "Compound Proportion," note 3, page 714; also, "Answer Found in Second Member," 2d note, page 714.) Ifote 3. — There are three ways of distinguishing the system, or truly organizing, method; (1.) It is that by which all problems are solved in the same way; (2.) It is that by means of which alone we may dispense with all rules and formulas; or, (3.) It is that which requires the least possible amount of memorizing. Let it be care- fully observed that there is just one device in which these three objects are accom- plished, namely, the 3-step solution ; for this not only solves all problems in the same way and does away with the necessity of memorizing rules and formulas, but it ren- ders all rules and formulas absolutely useless. As soon as the three variations (or cases) of the 3-step solution become known to arithmeticians, we shall then realize that aU rules and formulas are walls between the mind and safe conclusions, and are to be shunned as carrion to the sense, as vermin to be smoked out of the schools. That which is to take the place of rules and formulas is the 3-step solution, in rational arithmetic; in empirical arithmetic, the teacher's own doing (solving) act. Instead of directing the learner to memorize an isolated rule or formula, he shall say rather, to the learner, " Watch me, and then do as I do." " The man may teach by doing, and not otherwise." — Emerson. Precisely as the mother teaches the child to talk, write, sew, sing, etc., by saying, " Watch me, and then do as I do." 4. What is 35% of 6951b.? Case I Solution. a. 100% = 6951b. b. 1 = 6.95 c. 35 = 35 X 6.95 = 2^3.2^ Ih.—Ans. 5. Find 120% of $171.24. Case I /Solution. a. 100% =$171.24 b. 1 = 1.7124 c. 120 = 120 X 1.7124 = $20b.49.—Ans. SYSTEM ARITHMETIC— OBADE B. 679 6. 500 is 25^ of what number? Case II Solution, a. 25^ = 500 b. 1 = JL X 500 = 20 c„ 100 = 2000.— ^ws. 7. 827 is 120% of what number? Case II Solution. a. 120% = 828 b. 1 = ^ X 828 = 6.9 c. 100 = 690.— ^Ms. 8. What % of 980 mi. are 49 mi.? Case III Solution. a. 980 mi. = 100% b. 1 =^1^X100=^ c. 49 = 49 X ^ = 5%.— ^ws. 9. A merchant owes $15120, and his assets are $9828. What % of his debts can he pay ? Case III Solution. a. $15120 = 100% b 1 = 1— V 100 = 5 '-'• ^ 15120'^ -^"^ — TTS c. 9828 = 9828 X yf^ = 65%.— ^m-. 10. 600 is 20% more than what number? Case II Solution. a. 120% =600 b. 1 = ^ X 600 = 5 c. 100 = 500.— ^Ks. 11. The bread made from a barrel of flour weighs 35% more than the flour. Find the weight of the bread. Case I Solution. a. 100% = 1961b. b. 1 = 1.96 c. 135 = 135 X 1.96 = 264.61b.— ^«s. 12. 3f is what % of 18|? 680 TEE SYSTEM METHOD. Case III Solution. a. 145 ^ 100% I. 1 1 \/100 ^-2^ c. -^ = 29 X 11 = 20%.—Ans. 13. A man receives $1600 per year. He pays 18% for board, 81% for clothes, and 16% for incidentals. What amount does he save? Case I Solution. X. 100% — (18% + 81% + 16%) = 57|% saved. a. 100% = $1600 b. 1 = 16 c. 57| = 57| X 16 = $922^.— Am. 14. |i is what % of^s^f^? Case III Solution. a. ^\ = 100% b. 1 = ^0/ X 100 = 1871 c. H = li X 187.5 = 77^\%.-Ans. 15. Find | % of if of a hogshead. Case I Solution. a. 100% =ifhhd. b 1 — -J-^ V 16. = _4_ "• ^ — TOTF A 2 5 6 2 5 c- f = f X ffs = 2^0 lihd.— ^ns. 16. A farmer sold 3150 bu. and had 30% left. What was his entire crop ? Case II Solution. X. 100% —30% = 70% sold. a. 70% = 3150 bu. b. 1 = J^ X 3150 = 45 c. 100 = 4500 hn.—A7is. 17. A man sold a farm for $4563.20, which was 8% less than it cost. Find the cost. Case II Solution. a. 92% = $4563.20 b. 1 = ^ X 4563.20 = 49.60 c. 100 = $4960.-^*15. SYSTEM ARITHMETIC— GRADE B. 681 18. A grocei' sold from a hhd. containing 6001b. of sugar, \ at one time, and i of the remainder at another time. What ^ re- mained ? Case III Solution. X. 6001b. — (150+150) = 3001b. remaining. a. 6001b. = 100% b. 1 = ^ X 100 = i c. 300 = 300 X i = ^0%.—Ans. PROFIT AND LOSS. 1. Having used a carriage for one year, I sold it for $125, which was 25% below cost. What should I have received had I sold it at 10% above cost? a. 75% of cost = $125 b. 1 " " = ^ X 125 = If c. 110 " " = 110 X If = $183.331— ^«5. 2. How must a watch be marked that cost $120, so that 4% may be deducted from the marked piice and still gain 20%? a. 100% = $120 b. 1 = 1.20 c. 120 = 120 X 1-20 = $144 selling price. a. 96% of marked price = $144. b. 1 " " " = aV X 144 = 1.5 c. 100 " " " = $150.— ^ws. 3. A man bought a span of horses for $450, which was 25% less than their real value, and sold them for 25% more than their real value. Find his gain. a. 75% of real value = $450 b. 1 " " " = ^ X 450 = 6 c. 125 " " " = 125 X 6 = $750 selling price. X. $750 — $450 = $300 gain.— ^ws. 4. A merchant sold two coats for $25 each. On one he gained 20% ; on the other he lost 20%. Did he gain or lose on the two transactions? 682 THE SYSTEM METHOD. a. For 1st coat, 120% = $25 b. 1 = yfir X 25 = .20831 c. 100 = $20,831 cost of 1st. a. For 2d coat, 80% = $25 b. 1 = Jg. X 25 = .3125 c. 100 = $31.25, cost of 2d. X. $20,831 + $31.25 = $52,081 cost of both, y. $52,081 — $50 = $2,081 loss.— ^ws. 5. If 1 of an acre of land is sold for | of the cost of one acre, what is the gain % ? X. f s. price — l c. price = i gain. a. 1 cost, = 100% b. 1 " = 2 X 100 = 200 c. 1 " = 1 X 200 = 50%.— ^ws. COMMISSION. 1. A Louisiana planter sent a Chicago commission merchant 15 tons of sugar. The Chicago merchant pays freight at 75/ per cwt. and $5 for cartage. If the agent sells the sugar at 26f per lb., commission 21%, how much money shall he send the Louisiana planter ? X. 30000 lb. @ $.25 = $7500 amt. of sales. a. 100% =$7500 b. 1 = 75 c. 2i- = 21 X 75 = $187.50 commission. y. 300 cwt. @ $.75 = $225 freight. z. $187.50 + $225 + $5 = $417.50 total charges. p. $7500 - $417.50 = $7082.50.-^^65. 2. A commission merchant receives from his principal 50000 lb. of wool, which he sells at 50/ per lb., commission 21%. If he pays $125 freightage and $50 cartage, what are the net proceeds? X. 50000 lb. @ $.50 = $25000 a. 100% = $25000 b 1 = 250 c. 21 = 21 X 250 = $625 commission. y. $25000 — ($625 + $125 + $50) = $24200 proceeds.— ^ws. SYSTEM ARITHMETIC— GRADE B. ggg 3. A merchant buys, through his agent, 730 yd. of carpet at 11.25 per yd., commission |%. If the freight is $7.37, at what price per yd. must he sell the carpet to make a profit of 20%? X. 730 yd. @ $1.25 = $912.50 a. 100% = $912.50 b. 1 = 9.125 c. f = I X 9.125 = $6.843f commission. y. $912.50 + $7.37 + $6.843f = $926.713f total cost. a. 100% = $926,713 b. 1 = 9.267 c. 120 = 120 X 9.267 = $1112.04 s. price, z. 730 yd. = $1112.04 p. 1 " = yi^^X 1112.04= $1.523.— ^?zs. 4. A merchant sends his agent $6150 with which to buy cotton at 101/ per. lb., commission 2i%. How many pounds of cotton can the agent buy ? a. 1021% =$6150 b. 1 = yJL. 5 X 6150 = 60 c. 100 = $6000 amt. invested in cotton. a. lOi/ = 1 lb. b 1 — 1 c. 600000 = 600000 X yJ^^ = 58536ff lb. cotton.— Ans. INSURANCE. 1, Insured |- of a vessel worth $24000, and |- of its cargo worth $36000, the former at 2i%, the latter at 1^%. What is the pre- mium? X. I of $24000 = $15000 amount. a. 100% = $15000 b. 1 = 150 c. 2i =2^x1 50 = $337.50 premium on the vessel. y. I- of $36000 = $24000 a. 100% = $24000 b. 1 = 240 c. li = n X 240 = $270 premium on the cargo. z. $337.50 4- $270 = $607.50.— ^«s. gg4 THE SYSTEM METHOD. 2. An insurance company having a risk of $25000, at -^fo, re- insured $10000, at |-%, with another office, and $5000, at |%, with another. How much premium did it clear above what it paid ? a. 100% =$25000 b. 1 = 250 c. -2^ = ySg. X 250 = $225 premium rec'd by company. a. 100% = $10000 b. 1 = 100 c. f = 4 >< 100 = $80. a. 100% = $5000 b. 1 = 50 c. I = I X 50 = $37.50. X. $225 — ($80 + $37.50) =$107.50 premium cleared. —^?^s. 3. I took a risk of $45000; re-insux-ed at the same rate, $10000 each in three offices, and $5000 in another; my share of the pre- mium is $262.50. Wb at was the rate ? X. $45000 — $35000, amt. re-insured, = $10000 a. $10000 = 100% b- 1 = Twoo X 100 = T^o c. $262.5 = 262.5 x rk = ^%.—Ans. 4. Took a risk at 2% ; re-insured $10000 of it at 2J%, and $8000 at If %; my share of the premium was $207.50. What sum was insured ? X. 2i% of $10000 = $212.50 y. 1|% of $8000 = $140 z. $212.50 -f $140 -f $207.50 = $560 prem. on entire risk. a. 2% = $560. b. 1 = i X 560 = 280 c. 100 = $28000 amt. insured.— -^ws. TAXES. 1. The taxable property of a county is $486250, and the rate of taxation is 78/ on $100, that is, ■^%. What is the tax to be raised ? a. 100% = $486250 b. 1 = 4862.50 c- -rr^ = a¥(F X 4862.50 = $3792.75.— ^«s. SYSTEM ARITHMETIC— GRADE B. 685 2. A's tax is $50.46; he pays a poll-tax of $1.50, and owns |8704 taxable property. What is the rate of taxation ? X. $50.46 — $1.50 = $48.96 tax paid on property, a. $8704 = 100% h 1 = 1 V 1 no 25 c. 48.96 = 48.96 X ^^ = ^(^^,—Ans. 3. A man's income is 16% of his capital; he is taxed 21% of his income, and paj^s $26.04. What is his capital? a. 21% =$26.04 b. 1 = J^ X 26.04 = 10.416 c. 100 = $1041.6 income. a. 16% = $1041.6 b. 1 = Jig. X 1041.6 = 65.1 c. 100 = $6510.— ^ws. 4. Sold a lot for $7599, which covered its cost and 2% paid for tax. What was the cost? a. 102% =$7599 b. 1 = T^ X c. 100 = $7450.— ilws. b. 1 = -j^ X 7599 = 74.50 a. SIMPLE INTEREST. 1. What is the interest on $300 for 1 yr. 6 m. @ 6% ? Simple Proportion. N'ote i.— But in so far as 12 m. of int. = 6% time rate. ^^^^ ^"^^^ (simple-proportion) 1 -. ^^ ^j J 3-step solution has constantly 2 been employed by the learner C. 18 " " = 9 absolute rate. j^ reduction ascending, pro- Case /. portion, fractions, percentage, a. 100% = $300 principal. profit and loss, commission, b, \ =z 3 insurance, etc., the pupil may, c* 9 = 9 X 3 = $27 interest.— ^Tzs. ^^ *^^^ "'^^' ^^ vr<^^^'^^<^ *" be thoroughly familiar with it. It may, therefore, be reduced to an empirical process. (See p. 652.) Thus doing, instead of this double solution above we get the following truly organizing solution, which is the form of the system solution used in this work-book : — X. 6 per cent x 1 8 = 9 ^ absolutc rate (or amt. of interest) . a. 100^ = $300 principal. b. 1 =.3 c. 9 = 9 X 3 = 27 interest. — Am. 6g0 THE SYSTEM METHOD. 2^ote 2. — Since money loaned @ 6% is loaned 12 months for 6%, to reduce this time rate (6 % ) to an absolute rate, as in equation x, the simple operation is this : Divide by 12 to get the amount of the interest for 1 month, and then multiply by 18 (or the number of months) to get it for 18 months. We thus change "6% for 18 m." into 9% absolute, thus throwing the problem out of interest into simple per- centage. For, having found that "6% for 18 months" is identical with 9% abso- lute, all we have to do to solve the problem, is to find 9% of $300, the principal; and this is a simple problem in simple percentage. It is, in all respects, Uke the first on page 677. Note 3. — The three notes beginning on page 677 are as applicable to these three cases of interest as to the three cases of percentage there explained ; for these three and those three, this simple x equation excepted, are identical. 2. The interest for 1 yr. 6 m. @ &% is $27. What is the prin- cipal ? Case II. ^ 6percentxi8 = 9 ^ absolute rate (or amt. of interest). a. ^% = $27 interest. b. 1 = 3 c. 100 = $300 principal.— ^TOS. 3. The interest on $300 @ %% m $27. What is the time ? Case III. jy^fg 4.— It is easily seen that $27, the interest, is 9% of $300, the principal. And the explanation of the remaining equation (x) is easy, thus: 6% being the amount of 13 months' interest, 1% would be the amount of " (1^ X 13 =:) 3 months' interest, and conse- quently 9fo would be the amount of (9 x 3 =) 18 months' interest. The time, therefore, would be 18 m. 4. The interest on $300 for 1 yr, 6 m. is $27. What is the rate? Case 111. jy^^^ 5.— Equation x is thus explained: a. $300 principal = 100% since the amount of 18 months' interest is b. 1 = ■!■ 9%, one month's interest would be (y\^ X 9) C. 27 interest = 9%. -|%, and then 13 months' interest would be ■^ 9 ptr cent x 1 2 ^= Qo/ J^j^g (13 X i) 6 %, the yearly rate. — Ans. 5. What is the interest on $300 for 1 yr. 6 m. 18 d. @ 6% ? a. $300 principal = 100% b. 1 = 1 3 c. 27 interest = 9%. X. i2m.x9 ^ 18 m.- -Ans. X. 6 percent I 18-6 12 9.3% absolute rate. Mte e.— Since 18 a. 100% = $300 principal. if of a month = j% (or .6) of jj 1 __. D a month, 18 days, written as a c' 9.3 = 9.3 X 3 = $27.90.-Ans. '''''^''^ °' ^ °^°^*''' *' 'f "^ So always divide the days by ^, and then write the quotient as tenths (decimal) of a month. SYSTEM ARITHMETIC— GRADE B. ggT 6. "What is the interest on $840 for 1 yr. 8 m. 12 d. @ 8«^ ? X. s p^^ceny 2 0-4 = i^ Q ^bs. rate. iVb0 *19.60X,i, = $3 20;hence ^' 24-5 X i.'.uu — ^,3..^u the principal would be $320.— c. 100 = $320 principal. — Aas. [-^,js. 10. The interest for 1 yr. 5 m. 12 d. @ 8% is $40.60. What is the principal ? ^ 8_J^roent20_7^ = 11.6% ^^n^ t^An ^r^ Lightning Process'' a. 11.6% =$40.60 (of the system method). b. 1 = t|.6-X 40.60 = $3.50 $40.60 X tt.^ = ^3.50; hence c. 100 = $350 prin.— ^n*. $350.— ^m. 11. The interest for 2 yr. 3 m. 2 d. @ 6% is $3,381 What is the principal. X. 6percen^tx2 7 .0i£ = 13.51% '^Lightning Process" a. 13.51% = $3,381 (of the system method). b. 1 = -^^ X 3.381 = .25 ^^-^^^ ^ ^^-^ ^ ^■^^' ^^^''^ c." 100 = $25 prin.~i,z.. ' m.-Ans. 688 THE SYSTEM METHOD, 12. The interest on $300 @ Q% is $27.90. What is the time ? a. $300 = 100^ Note 8. — Of course, if the learner prefers, i^ 1 J he may write out equation x into the 3-step- ^p, ^^ %p, ^ , ^ „ _y solution form, thus: — c. 27.90 = 27.9 X i = 9.3$^. ' iomvQ9 -ion -. a. 6% interest = 12 m. X. 12^-^^9.3 = 18.6 m. = 1 yr. ^ / , ^ i >< 12 = 2 6m. 18d.—Ans. c. 9.3 " =9.3 x 3 = 18.6m.-^«s. 13. The interest on $300 for 1 yr. 6 m. 18 d. is $27.90. What is the rate? Ifote 9. — Or, if the learner prefers, he a. $300 =100^ may put this empirical process in equation b. 1 = "§■ X in the 3-step-solution form : — C 27.90 = 9.3 a. 18.6 m. interest = 9.3% X. 9^3P«:cen^JL2 =g^_ AnS. ^- ^ " " == T¥.6 X 9.3 = J ^^'^ ^ c. 13 " " =13Xi=6%.— ^»is. 14. What principal will amount to $552.60 in 5 m. 6 d. @ 6%? X. ■6 J"='^«e°^^5.2 — 2.6^ interest. iVb^e iO.— Since the principal always con- tains 100%, and since here the interest a. 10^.6 = $552.60 amt. contains 2.6%, the amount (of interest and "•■'■= TCT-e" ^ 5o2.6 = 5.386 principal) wiU contain 102.6% ; hence equa- C. 100 = $538.60.— ^?is. tion a. 15. What principal will amount to $609.20 in 4 m. 24 d. @ 4%? NoU 11. — It will be observed that these last two problems are identical with true- discount problems, the amount being the future debt, the principal the present worth, and the interest the true discount. TRUE DISCOUNT. 16. I have a note of $500 legally due in 3 months, and bearing 6^ interest. Find its present worth, money being worth (loaning at)8%. X. 6 per cent X 3 := li^ a. 100% = $500 principal. b. 1 = 5 c. 1011 ■= $505.50 face of note. X. 4rercen^tx4.8 ^ 1_6% a. 101.6% - $609.20 b. 1 - lol 6 X 609.20 -5.996 c. 100 = $599.60.— ^?zs. y- 8per^centx3 £% a. 102% = $505.50 face, or amt. b. 1 = 4.95588 c. 100 = $495.59 p. worth.— ^ws. The System Method— No. 23. THE STTJIDEIiTT'S Work-Book of System Aritlimetic, Grade A, rOB THE USB OF Public Schools, Normal Schools, Teachers' Institutes, and Colleges, AND ESPECIALLY ADAPTED TO SoSIOOXj lEjI^IPOSITIOIbTS IN WHICH, BY MEANS OF THE 3-STEP SOLUTION, THE ARITH- METICAL TREE TRUNK, ALL PROBLEMS ARE SOLVED IN THE SAME WAY. BY ISAAC ELDRIDGE WILSON, INVENTOR OP THE LBAENEB'S WOEK-TREB, AUTHOR OP THE SYSTEM METHOD, A SERIES OP TEXT-BOOKS FOR SCHOOLS, A SERIES OF 'WORK-BOOKS FOR SCHOOLS, ETC. ' THE MAN MAY TEACH BY DOING, AND NOT OTHERWISE."— EMERSON. CHIOAOO, ILL.: Leakner's Work-Tree Company isse. 44 COPYRIGHT, 1886, BY ISAAC ELDRIDGE WILSON. ALL RIGHTS RESERVED. Any infringement will be prosecuted to the full extent of the law. Sys(em Arithmetic— Grade ^, ^-r-i=r«-v®= REDUCTION ASCENDING, JVbte 1. — For note concerning the importance of becoming familiar with the 3-step solution at the very outset, on beginning the subject of reduction ascending, see Re- duction Ascending, Work-Book No. 23. See the same, also, for the solution of a number of interesting problems, by the system method. Note 2. — For when and how to teach reduction descending, see note on page 657; see also in "Where Should Fractions be Taught," page 669. 1. If sound travels 1 120 ft. per second, and there are lOi seconds between the lightning flash and the thunder clap, how far away is the thunder cloud ? X. lOi sec @ 1120 ft. = 11760 ft. a. 5280 ft. = 1 mi. b 1 " = —J— 5 2 S c. 11760 " = 11760 X 3tVo =^"^*- • • 1200 ft. a. 16.5 ft. = lrd. b. 1 " =t|.7 C. 1200 " = 1200 X TT.5 = ^TT ''^• Solved by Lida Oltman. 2. If 2 bu. 3 pk. of rye are required for sowing ?) acres, how many quarts are needed for sowing a rectangular field 484 ft. by 270 ft.? X. Field 484ft. X 270ft. = 130680 sq.ft. a. 272.25 sq. ft. = 1 sq. rd. '-'• ■*■ 2 7 2-25 c. 130680 " = 130680 X 2Ti2 5 = ^SOsq.rd. a. "320sq.rd. = 1 a. b 1 " = -i- "• -^ 3 2 c. 480 " = 480 X 3^^ = 1.5 a. X, 2bu. 3pk. = 88 qt. a. 3 a. = 88 qt. b. 1 " ^ i X 88 = 291 c. 1.5 = 1.5 X 291 = 44qt.— ^?is. Solved by Hattie Johnston. [691] QQ2 TEE SYSTEM METHOD. 3. What length must be given to a rectangular lot 20 rd. wide, to make it contain 3 acres ? X. 3 a. @ 320 sq. rd. = 960 sq. rd. a. 20 sq. rd. of lot = lot 1 rd. long. D. i 2 c. 960 " " = " 960 X yV = 4S^*^- ^o^g- — ^'^^■ Solved by Emma Highley. 4. A rectangular cistern 4 ft. in length by 3 ft. in width, has a capacity of 8 cubic yards. What is its depth ? X. Cistern 4 ft. long x 3 ft. wide X 1 ft. deep = 12 cu. ft. = | a. Cistern of | cu. yd. = cistern 1 ft. deep. [eu. yd. b. " 1 " = " f " c. " 8 " = " 8 X f = 18 ft. deep.— ^ws. 5. A mill-wheel is turned by a stream of water running 3 ft. per second in a channel 4 ft. wide, 9 in. deep. Find, in tons, the weight of water supplied in 3 hours if 4 cu. ft. of water weigh 249 lb. X. Stream 3 ft. by 4 ft. by 9 in. = 9 cu. ft. y. 3 hr. @ 60 m. @ 60 s. = 10800 sec. z. 10800 sec. @ 9 cu. ft. = 97200 cu. ft. a. 4 cu. ft. = 249 lb. b. 1 " - 1 X 249 = 62^ c. 97200 " = 97200 x 62i = 6050700 lb. a. 2000 lb. = 1 ton c. 6050700 " = 6050700 X -^^^ = 3025^^ tons.— ^ws. Solved by Dora Shepherd and Hattie Johnston. COMPARATIVE MEASURES. Note. — ^For the solution of less difficult problems than those following, see Work-Book No. 22. 1. I have a bin Hi ft. by 61 ft. by 4f ft. full of wheat. What is it worth at $1,875 per cental, counting 1 bu. equal to .6 centals? X. A bin 111 X 61 X 43 ft. = 34711 cu. ft. = 600210 cu. in. a. 2150.4 cu. in. = 1 bu. "• -^ ~ 2 150.1" c. 600210 " = 600210 X -^j^.^ y. 279.115 bu. @ .6 centals = 167.469 centals. z. 167.469 centals @ $1,875 = $314.004.— ^m. SYSTEM ARITHMETIC— ORABE A. 693 How many pounds of clover seed will the above bin hold ? of Iowa oats? Iowa flaxseed? dried apples? dried peaches? X. 279.115 bu. clover seed @ 60 lb. = 16746.9 \\).—Ans. y. 279.115 bu. Iowa oats @ 35 lb. = 9769.025 \h.—Ans. z. 279.115 bu. flaxseed @ 56 lb. = 15630.44 Vo.—Ans. p. 279.115 bu. dried apples @ 24 lb. = 6698.76 \h.—Ans. q. 279.115 bu. dried peaches @ 33 lb. = 9210.795 \h.—Ans. Solved by Anna Ilershman and Madge Brundage. 2. In |- lb. Troy, what fraction of a pound Avoirdupois? X. I lb. Troy @ 5760 gr. = 3600 gr. a. 7000 gr. = 1 lb. Avoir. b 1 " = — i— "• ^ 7 c. 3600 " = 3600 X t-Jqo = if ^^- Avoir.— ^ws. Solved by Etta Strong and Alonzo Highley. 3. Reduce .9 lb. Avoir, to the decimal of 5 lb. Troy. X. .9 lb. Avoir. @ 7000 gr. = 6300 gr. y. 5 lb. Troy @ 5760 gr. = 28800 gr. a. 28800 gr. = 1.00 b 1 « = i 2 8X00 c. 6300 " = 6300 X 2Tioo = -21875.— ^ws. Solved by Emma Ball and Geo. W. Vibber. 4. Eeduce 14° 11' 54" to the fraction of a right angle. X. 14° 11' 54" = 51114" a. 324000" = 1 right angle. "• ^ 32 4 0T c. 51114" = 51114 X ^^0^ = ^8^y^9_ right angle.— ^7^s. 5. What will it cost to carpet a room 131 ft. by 24 ft., with carpet f yd. wide, at $1.60 a yard? X. Eoom 131 ft. X 24 ft. = 324 sq. ft. a. -2_igq. ft. (3 of 9) = $1.60 b. 1 " = 2T X 1.60 = §^4_o c. 324 " = 324 X Yt^ = $76.80.— ^ws. Solved by E. H. Wintermute and Henry Brammier. 6. Reduce 1 cu. ft. to the decimal fraction of a bushel ; 1 w. gal. to the decimal fraction of a cu. ft. ; 4 lb. distilled water to the decimal fraction of a cu. ft. a. 2150.4 cu. in. = 1 bu. k 1 " = 1 "• ^ YTS'O-* c. 1728 " = 1728 X 2T5(7.4 = -8035 + bu.— ^ws. 594 THE SYSTEM METHOD. a, 1728 cu. in. = 1 cu. ft. kit' = __J . ^- ^ 17 2 8 c. 231 " = 231 X yJ^ = .133+ cu. ft— ^ws. a. 62.5 lb. dis. water = 1 q\\. ft. c. 4 " " " = 4 X -g4.5 = .004 CU. ft.— J«s. Solved by Matie E. Bowers and W. E. Arbingast. 7. If i of an inch of rain fall, Low many barrels will be caught by a cistern which drains a roof 52 ft. by 38 ft.? X. Eainfall 52 X 38 X 4V f*- = ^li cu. ft. = 71136 cu. in. a. 231 cu. in. = 1 gal. '-'• •*- — 2 3 1 c. 71136 " = 71136 X -J-y = 307.948+ gal. a. 30.5 gal. = 1 bbl. "• -■- 30-5 c. 307.948 " = 307.948 x 30.5 = 10.096+ hU.—Ans. Solved by Wm. Miller, Wm. Murpby, and Albert Peterson. "What depth of water will the above rainfall produce in a cis- tern 4i ft. by 8 ft. ? a. 36 cu. ft. rainfall = water 1 ft. deep. b. 1 " " = " Jg lAns. e. 411 " '' = " 411 X gV = 1-^'^3 + ft. deep.— Solved by Geo. Armentrout and Matie E. Bowers. 8. y\ of ^ pound of sugar is, in weight, what part of ^ of a pound of gold? X. j\ lb. @ 7000 gr. = 1500 gr. y. I lb. @ 5760 gr. = 2304 gr. '1 ?3n4- o-r = 230 4 a. z,oU'± gi. — 2X0^ b 1 " — 1 '-•• ^ 2 3 f c. 1500 " - 1500 X 2-^ = {^.—Ans. Solved by J. R. Morehouse, Harry Dawson, and Albert Ehreclie. 9. A druggist bought 3 lb. 8 oz. Avoir, drugs for $52.50, and made them up in pills of 5 gr. each, which be retailed at 36/ a dozen. What was his gain ? X. 31- lb. @ 7000 gr. = 24500 gr. a. 5 gr. = 1 pill. b. 1 " =i c. 24500 " = 24500 X ^ = 4900 pills. y. 4900 pills @ 3/^ each = $147 s. price, z. $147 — $52.50 = $94.50 gaiu.—Ans. Solved by John B. Coyne and Ella Taylor. SYSTEM AEITUMETIC— GRADE A. G'J.j 10. A clock loses 1 sec. every |- hr. If it is right at 20^^ min. past noon, May 1st, what is the day and hour when it will be 10 min. too slow? X. 1 second lost = | hr. y. 60 sec. lost (or 1 min.) = 60 X f = 90 z. 10 min. lost = 10 X 90 = 900 a. 24 hr. = 1 day. h 1 « _ 1 c. 900 " =. 900 X -Jj = 37 d. . . 12 hr. p. 12 M. 201 min., May 1st + 37 d. 12 hr. = 20 min. 20 [sec. A. M., June 8th. — Ans. Solved by Emma Miller and Tanta Mellick. 11. How far does a man walk in planting a field of corn 285 ft. square, the rows being 3 ft. apart and 3 ft. from the fences ? a. 3 ft. wide = 1 row. b. 1 " " =i c. 285 " " = 285 X i = 95 * X. 95 rows @ 279 ft. long = 26505 ft. a. 5280 ft. = 1 mi. b 1 " = 1 "• -^ 5 2 8 c. 26505 " = 26505 x yaVo = ^Th mi.— ^ws. Solved by W. C. Frampton and Jennie Drake. *Note. — There are in reality only 94 rows of corn in the field; but a man in planting the field must waJk from row to row at the ends, and this distance between the rows is equivalent to one row across the field. 12. If sound travels 1120 ft. per second, in how many seconds after the lightning flash will the thunder-clap be heard from clouds 2 mi. 160 rd. distant. X. 2 mi. 160 rd. = 13200 ft. a. 1120 ft. = 1 sec. k 1 u 1 0- -L — 1120 c. 13200 " = 13200 X ttVo = ^m sec— ^W5. 13. Out of a hhd. of 63 gal. leaked 3 qt. What fraction of the whole was lost ? X. 63 gal. @ 4 qt. = 252 qt. a. 252 qt. = ||f of the whole. U 1 <( = _1 " " " "• ^ 2 5 2 C. 3 « = 3 X YsT ~ 'ii ^^ ^^® whole. — Ans. (59(3 TEE SYSTEM METEOD. 14. A crib 15 ft. X 8 ft. X 7i ft. inside dimensions, is full of corn in the ear. If 2 bu. of ears make 1 bu. of shelled corn, what is the value of the whole at 92^ j^er shelled bu.? X. A crib 15 X 8 X 71- = 880 cu. ft. = 1520640 cu. in. a. 2688* cu. in. = 1 bu. (heaped measure). "• •*■ 2 6 8 8 c. 1520640 " = 1520640 X grW = 565.71 bu. in ear. J. 565.71 bu. in ear = 282.85 bu. shelled corn, z. 282.85 biT. @ 92/ = 1260.22.— ^%s. Solved by Wm. N. Orris and E. E. Eeynolds. * Note. — In measuring corn in the ear, the heaped measure is used, which is one- fourth larger than the stricken, or lever, measure. The heaped bushel therefore con- tains 3150.4 cu. in. -|- M of 2150.4 cu. in. = 2688 cu. in., which number is used in the solution. 15. If one bushel or 60 lb." of wheat makes 48 lb. of flour, how many barrels of flour can be made from the contents of a bin 10 ft. long, 5 ft. wide, and 4 ft. deep, filled with wheat ? X. A bin 10 ft. by 5 ft. by 4 ft. = 200 cu. ft. = 345600 cu. in. a. 2150.4 cu. in. = 1 bu. b 1 " — 1 ^'- -^ — 2 15 0-4 c. 345600 " = 345600 x ty^-a. = 160.714 bu. J. 160.714 bu. wheat @ 48 lb. flour = 7714.27 lb. flour. a. 196 lb. = 1 bbl. flour, b 1 " = -JL_ "• ^ 19 6 c. 7714.27 " = 7714.27 X ^ = 39.35-|- bbl. flour.— ^7^s. Solved by C. M. Pierce and Cora Cox. 16. A bin that holds 100.8 bu. is 7 ft. long and 6 ft. deep. How wide is it ? X. 100.8 bu. @ 2150.4 cu. in. = 216760.32 cu. in. = 125,44 a. 42 cu. ft. = bin 1 ft. wide. [cu. ft. b. 1 " = « -1- c. 125.44 " = 125.44 X ^ = 2.98 ft. wide.— ^ws. 17. A cistern 5 ft. by 4 ft. by 3 ft. is full of water. If it is emptied by a pipe in 1 hr. 30 min., how many gallons are discharged through the pipe in an hour ? SYSTEM ARITHMETIG—GBADE A. 697 X. A cistern 5 ft. by 4 ft. by 3 ft. = 60 en. ft. = 103680 a. 90 min. = 103680 cu. in. [cu. in. b. 1 " =^ X 103680 = 1152 c. 60 " = 60 X 1152 = 69120 en. in. a. 231 en. in. = 1 gal. c. 69120 " = 69120 X ^ = 29915- g-^\.—Ans. Solved by Kittie Wilson. 18. A pile of Avood containing 67i cords, is 90 ft. long and 12 ft wide. How high is it ? X. 67i e. @ 128 cu. ft. = 8640 ca. ft. a. 1080 cu. ft. = pile 1 ft. high, b 1 " = " L_ "• '- 10 8 c. 8640 " = " 8640 x ^^ = 8 ft. high.— Ans. LONGITUDE AND TIME, standard Unit, — The unit of time is the day, which is the time required by the earth to revolve on its axis. SideeejVX, day. — The exact time required by the earth to make one complete revolution on its axis. Solar day. — The exact time re- quired by the earth to bring the same point on its surface twice under the vertical rays of the sun. Asteonomical day. — The solar day beginning and ending at noon. CiTiL DAY. — The solar day beginning and ending at midnight. Sabbath day. — Every seventh solar day beginning and ending at sunset. Calendar Time. — In business transactions, 1 year is regarded as 360 days, 1 month as 30 days, and 1 day as gJg of a month, or j\-^ of a year. But for other civil purposes, including business transactions with the Government, a year is to be regarded as a calendar year of 365 or 366 days, a month as the calendar month of 28, 29, 30, or 31 days, according to the year and month referred to. The calendar months with the number of days they contain are as follows : — Winter. Summer. 12. December 31 days. 6. June 30 days. 1. January 31 " 7. July 31 " 2. February I ««^^7;>/«='%28 days. 8. August 31 " Spring. Autumn. 3. March 31 days. 9. September 30 days. 4. April 30 " 10. October 31 " 5. May 31 " 11- November 30 " 698 THE SYSTEM METHOD. Table showing the number of days from any day of — P > 2h ^ ^ ^ qq J* O < P To the same day of next 365 334 306 275 245 214 184 153 122 92 61 31 Jan. 31 59 365 28 337 306 276 245 215 184 153 123 92 62 Feb. 365 334 304 273 243 212 181 151 120 90 Mar. 90 120 59 89 31 61 365 30 335 365 304 834 274 304 243 273 212 242 182 212 151 181 121 151 April. May. 151 120 92 61 31 865 335 304 273 243 212 182 June. 181 150 122 91 61 30 865 334 303 273 242 212 July. 312 243 181 212 153 184 122 153 92 123 61 92 31 62 365 81 334 365 304 835 273 304 243 274 Aug^ Sept 273 242 214 183 153 122 92 61 30 365 834 304 Oct. 304 834 273 303 245 275 214 244 184 214 153 183 123 153 92 122 61 91 31 61 365 30 335 865 Nov. Dec. Remark. — In leap years, if the last day of February be included in the time, 1 day must be added to the number obtained from the table. Calendar. — A calendar is a register of the year, giving its divisions into months, weeks, and days, as kept in the common almanac. Of calendars now used there are two, — the Gregorian, established by Pope Gregory XIII. in 1582 and now used by all civilized nations except Russia, and the Julian, established by Julius Csesar b. c. 46, and now used by Russia alone. ©id Style and Xew Style. — "Old Style" is the style of counting adopted in the Juhan calendar. " New style " is the style of reckoning adopted in the Gregorian calendar. Between the two, there is now about 12 days' difEerence, which may be explained as follows : — By the Julian calendar, the earth was declared to complete its circuit around the sun (passing and returning to "Jan. 1 " in the figure) in 365 days and 6 hours. But the exact length of the year in mean solar time is 365 days, 5 hours, 48 minutes, and 50 seconds = 365 d. 5 hr. 48f noin. That is to say, starting at the posi- tion marked "Jan. 1 " in the figure, the earth passes through Winter, Spring, Sum- mer and Autumn, and returns again to the position "Jan. 1" in 11-J- minutes less than 365 d. 6 hr. Hence to the Romans (who used the Julian calendar) it was Jan. 1, of any new or incoming year, for 11^ min. before they recognized it — before they began to count it Jan. 1 of the new or incoming year. That is to say, the earth reached the end of the old year at the position "Jan. 1," and passed 11^ min. into the new year (toward b in the figure) before they counted it a new year — ^before .they called it Jan. 1. But a 2d year, it would be Jan. 1 for (2 XH^) 221 min. before they began to count it Jan. 1. In the 3d year, it was Jan. 1 for (3X11 J) 33J min. before they began to count it Jan. 1. In the 10th year, the earth went beyond the position "Jan. 1" into the new year for (10 X 11|) lUf mil- before they counted Jan. 1. Finally, in 250 years, it was (250 X HJ) 2791| min. = about 2 days be- yond "Jan. 1 " toward b, before they began to count Jan. 1. Now, in 1582, the earth SYSTEM ARITHMETIC— GRADE A. 699 went into January of a new year as much as about 10 whole days before they counted it Jan. 1. That is to say, the earth was out of the old circuit, or year, and far toward b in the new (or next) year, before the people counted January to have begua at all. Hence, in order that the count, or reckoning, of the time might correspond to the season, or earth's position in its path around the sun, in 1583 Pope Gregory XIIT. de- creed that October 5th should be counted October 15th. Those using the Old Style, or Julian calendar, are now about 12 days behind the season ; so that when we have July 4, Russia has June 22. Common Year, Leap Year. — A common year is any year of 365 days. A leap year is one of 366 days ; that is to say, one in which February is counted as having the extra, or 29th day. By the Gregorian calendar, the 29th day is added to February in such years as are divisible by 4, except such as, being divisible by 100, are not divisible by 400. Why Add the Extra Day, Feb. 29, to All Years Divisible by 4, Except iSuch as, Being Divisible by lOO, are not Divis- ible by 400? — The exact length of the solar year is 365 d. 5 hr. 48 1 miu. iu mean solar time. But for three years in succession, we count but 365 days to a year, giving February but 28 days. Thus, in counting it Jan. 1 at the end of 365 days, we begin about 5 hr. 48 1 min. too soon. A 2d year, we count it Jan. 1(2X5 hr. 48 ^. min) 1 1 hr. 37| min. too soon. The 3d year, we count it Jan. 1 (3 X 5hr. 48f min.) 17hr. 26i-min. too soon. The 4th year, we count it Jan. 1 (4 X 5 br. 48f min.) 23 hr. 15 i min. too soon. Hence we put in a day (Feb. 29) in this 4th year, in order to make the count or calendar correspond more nearly to the actual season, or the actual position of tlie earth in its path around the sun. Here, then, is the explanation of why we divide by 4 to determine whether a year is a. leap year. YOO THE SYSTEM METHOD. But when we put in a day (Feb. 29), we put in (24 hr. — 23 hr. 15| min.) 44f min. too much. That is to say, we then count it Jan. 1 not till 44| min. after the earth has completed its year, or circuit; or not till the earth has gone 44| min. into the new year. Hence, putting in an extra day once in four years, brings the counting 44| min. behind the season — in 4 years. Then in 8 years (2 X 4), the count be- comes (3 X 44|) 89| min. behind the season, or earth's position in its orbit. And in 100 years (25 X 4), the count becomes (25 X 44|) 1116| min. = 18 hr. 36| min. behind the season. Here, then, in the hundredth year, we drop, and do not insert, the extra day of February, by doing which we bring the count up with the season and (24 hr. — 18 hr. 36| min.) 5 hr. 28| min. ahead of it. Here, then, is the reason why the 100th year is made, in the calendar, a common year of 365 days. But the count is now, at the end of 100 years, 5 hr. 23^ min. ahead of the season. That is to say, at the end of 100 years we count it Jan. 1 5 hr. 23^ min. too soon. But in 400 years we should count it Jan. 1 (4 X 5 Jir. 231 min.) 21 hr. 33J- min. too soon, about 1 day too soon. Hence in the 400th year we again put in the extra day in order to make the count and the season agree. Here, then, in the 400th year, we insert the extra day again ; that is, we make every 4th centennial year a leap year. Putting in the extra day in the 400th year takes the season 2 hr. 26| min. again ahead of the count, as it was after adding the extra day to the 4th year. Hence it is that we divide years divisible by 100, by 400, to determine whether we add the extra day. By continuing this process of dividing by 4, 100, and 400, we may arrange a calendar correct for any number of years. Thus it is, finally, that the Gregorian calendar in- serts an extra day in such years as are divisible by 4 ; but not in such as are divisible by 100 unless they are also divisible by 400. Let it be observed that at the end of 400 years, by the Gregorian calendar, the season is ahead of the count 2 hr. 26| min. = ^-^ hr. Hence the error in the Gre- gorian calendar may be found thus : — a. -2^ hr. error = 400 yr. @f calendar. b. ^ = ^^2 X 400 = ^^ c. 24 hr. (1 d.) = 24 X Ht° = 3927-j-='x jr. of calendar. It is thus shown that the error in the Gregorian calendar is small, amounting to but 24 hours, or 1 day, in 3927^^^ years. Elementary Truths. — In 24 hours the whole circle of the earth (360°) passes under the rays of the sun. Hence, — a. In 24 hours, the earth revolves through 360.° b. In 1 hour, the earth revolves through Jj of 360° = 15.° c. In 1 min., the earth revolves through -^-^ of 15° = 15.' d. In 1 sec, the earth revolves through gL of 15' = 15." From these four items of fact matter, the tables for reduction may be made up. Different Places East and West Have Different Time.— One-haU of the earth is always in darkness, the other half always in the light. But the earth turns on its axis from west to east (causing the sun to appeal* to go fi'om east to west). That is, in the diagram, "W (morning) rotates up to M (noon), thence down to E (evening), thence into the night, and through it again around to morning at W. (The reader's head is supposed here to represent the sun.) Therefore places SYSTEM ARITHMETIC— GRADE A. 701 west of the meridian N S are in the fore- noon, and have earlier time than places east of N S, which are in the afternoon and have the later time. 'Wlien the smi is directly over M, at that point it is noon; at W, 6 o'clock in the morning; at E, 6 o'clock in the evening, of the same day. Oreenwicli (Eng.) llie 13ase. — Longitude is reckoned as so many degrees (up to 180°), minutes ('), and seconds (") east or west of the meridian of Greenwich, England. A de- gi-ee is 3^g of any circle. Hence 1° of the equator = g J-^of 24,899 statute miles = 69.16 statute miles. But since the parallels north and south of the equator are smaller circles than the equator, 1° of longitude measured on a parallel will be less than 1° of longitude at the equator. TABLE SHOWING THE CIRCUMFERENCE OF EVERY FIFTH PARAL- LEL, AND THE LENGTH OF ITS DEGREES IN ENGLISH MILES. Latitude. Circumference of Parallel. Length of Deg. of Longitude. Latitude. Circumference of Parallel. Length of Deg. of Longitude. Equator 0° 24,899 69.164 45° 17,636 48.988 5 24,805 68.903 50 16,030 44.545 10 34,538 68.120 55 14,814 39.760 15 24,056 66.833 60 13,481 34.669 30 33,406 65.018 65 10,552 29.310 35 23,580 63.731 70 8,541 23.726 30 21,581 59.948 75 6,465 17.957 35 20,418 56.718 80 4,338 12.049 40 19,100 53.055 85 2,177 6.048 45 17,686 48.988 Pole 90 0,000 0.000 While the prime meridian employed by the Enghsh and Americans is that of the National Observatory at Greenwich, the French and German geographers often use the meridian of the observatory at Paris. The Americans sometimes use, likewise, that of the National Observatory at Washington. Paris is 2° 20' east of Greenwich; Washington, 77° 2' 47" west of Greenwich. Table of Longitudes. — Since in the equator or any parallel there are but 860°, and longitude is reckoned both east and west, a place farther west than 180° west longitude, wiU be in east longitude, and vice versa. Hence longitude is reckoned up to 180° only. In the following table of longitudes, W. = longitude west from Greenwich ; E. = longitude east from Greenwich : — 703 THE SYSTEM METHOD Portland, Me...... ... Boston, Mass., New Haven, Conn.,. New York City, Philadelphia, Pa., . . . Baltimore, Md., Washington, D. C, . . Kichmond, Va., Charleston, S. C.,. . . . Pittsburg, Pa., Savannah, Ga., Detroit, Mich., Cincinnati, O., Louisville, Ky., Indianapolis, Ind.,. . . Nashville, Tenn., Chicago, 111 Mobile, Ala., Madison, Vila., New Orleans, La., . . . St. Louis, Mo., Minneapolis, Minn., . Des Moines, Iowa, . . . LONGITUDE. 18 W. 50 " a:-, " U " 3 " !59 " 13 Omaha, Neb., Austin, Tex., Denver, Col., . , Salt Lake City, Utah, . . , San Francisco, Cal., — Sitka, Alaska, St. Helena Island, Reykjavik, Iceland,. . . . Kio Janeiro, Brazil, . . . St. Johns, N. F., Honolulu, Sandwicli Is, Greenwich, Eng., Pans, France, Ptome, Italy, Berlin, German Emp.,. Yienna, Austria, Constantinople, Turkc St. Petersburg, Kussia, Bombay, India, Canton, China, Pekin, China, Manila, Philippine Is.,. Sydney, Australia, LONGITtTDE. O ' II 95 50 14 V/. 97 44 12 " 104 53 33 " 111 53 47 " 122 27 49 " 135 19 42 '-■ 5 42 " 22 " 43 20 " 52 43 " 1.57 52 " " 2 20 E 12 23 " 13 23 " 16 20 " 23 59 " 30 16 " 72 48 " 113 14 30 " 116 26 " 120 30 " 151 11 " Islaiitlers' I>ate tiine. — ^A, traveling east, and B, traveling west, start from Greenwich with the same time. But at New York, B tm-ns his watch back to agree with New-York time. At San Francisco, lie turns it back again. And, finally, at Manila, Philippine Islands, in longitude about 120° east, he turns his watch back once more to agree with Manila time. But the Spaniards there reckon Manila time from Greenwich west. Now, from Greenwich "west to Manila is (360° — 120°) 240.° And — 1° of longitude = Jj hr. time. 340° " " = 240 X tV = 16 hr. Hence, by Spanish count, Manila time is 16 hr. earher than Greenwich time, and B turns his watch back, in aU, 16 hr. After the manner of B, at Constantinople, A, who travels east, turns his watch up to agree with Constantinople time. At Calcutta, he again sets his watch forward. And, finally, at Canton, in longitude about 113° east, he turns his watch forward to Canton time. But Canton uses eastern time; that is to say, time reckoned from Greenwich east. Now, from Greenwich east to Canton is about 113.° And — 113° of longitude = 7 hr. 32min. of time. Hence, Canton time is 7 hr. 33 min. later than Greenwich time, and A turns his watch up, in all, 7 hr. 33 min. Consequently, the difference between A's and B's watches is (16 hr. -|- 7 hr. 33 min.) 33 hr. 33 min. That is to say, when B's watch at Manila is 10 min. past 13 Wednesday morning, A's watch at Canton is (10 min. past 13 Wednes- day morning -\- 23 hr. 33 min.) 43 min. past 11 Wednesday night. But 20 min. later, B would have (10 min. past 13 -f 20 min.) 30 min. past 13 Wednesday xsioxTATi%, SYSTEM ARITHMETIC—GRADE A. T03 while A would have (42 min. past 11 Wednesday night -|- 20 min.) 2 min. past 12 Thursday morning. B would date a letter " Wednesday^ Oct. 16 ; " A, " Thursday, Oct. 17." (The day is here assumed to be a "civil day," beginning and ending at midnight, not at sunset, as the Bible would reckon the day to begin and end.) Now, after the likeness of the travels of A and B have been the migrations east and west toward the Paciiic Ocean. The inhabitants of eastern Asia, Kamtchatka, the Japan Islands, the larger portion of the East-Indian Archipelago and Australia, hav- ing migrated east from Europe and Asia, use eastern time, that is, time reckoned east from Greenwich. On the other hand, the civilized inhabitants of California, the Sandwich Islands, and Polynesia, having emigrated from the east westward via New 704 THE SYSTEM METHOD. York, Cape Horn, and San Francisco, use Western time ; that is to say, they reckon their time from Greenwich west. Those emigrants coming from the west to the east, like A, use Eastern time, which is nearly one day faster than that used by those com- ing to the west from the east. Thus the islanders' date line is but an approximate division hue between those peoples using the Western, or earlier, time, and those using the Eastern, or later, time; that is to say, it is merely the general separation line between those peo- ples having migratec! eastward to their new homes and those having migrated westward to theirs. Since this line serves no other purpose than merely to represent to the eye, in a general way, what is the division line between unhistorical islanders using different dates, it is quite improperly called an "International" Date Line. This hne " differs somewhat from the 180th meridian. It may be traced from Chat- ham Islands, lat. 44° S., long. 177° W., and running north and west, keeping on the east of New Zealand, New Guinea, and Borneo, running through the Philippines, ap- proaching the China coast near Canton, and then bearing off to the northwest [north- east] outside the Japanese Islands, passes up through Behring Strait, and terminates in the north pole. On the west of this line the date is, in general, just one day in advance of what it is on the east." — Olney''s Science of Arithmetic. The map on pre- vious page wiU show the location and course of this date line. Note.— ^mce it is the prime purpose of this date line to divide the peoples accordingly as they actually reckon their dates by the Eastern or by the Western count, it is to be observed that it is incorrectly drawn ; for both the Alaskans and the Pitcairn Islanders reckon their time by the Eastern (later, or faster) count. Therefore, to be a true line of separ.ition be- tween the peoples using these two different counts, at the nortli it should be so deflected to the east as to bring Alaska on its west side ; and at the south it should likewise be so deflected far back toward the east as to bring Pitcairn Island on its west. See the article, "How Long the Day Lasts on the Globe," farther on. International Day Line. — But there is another line, which, running rom pole to pole through Behring Strait, and coinciding with the circle of illumina- tion, marks where each new day takes its start on the earth. This line is truly an interesting one to all nations, as locating the arrival and departure of each of the seven days of the week, — interesting especially as locating where the sabbath begins, and where ship-masters and voyagers are wont to drop or add a day in their reckoning of days and dates, according to which way they are sailing. If we should leave Indianapolis at noon on Monday, June 1st, and travel west- ward around the earth, keeping pace with the sun, the day would be to us Monday, June 1st, till we pass entirely around and return to Indianapolis, where the home peo- ple would be calling it Tuesday, June 2d ; and observe that our Monday noon would continue 24 hours, and Monday noon would change to Tuesday noon, when we arrived home, without an intervening night. And in like manner, however long we might be in passing around the earth from east to west, our count after returning, unless we add a day at the Day Line, would be one day behind the home people's count. On the contrary, if we pass around the earth from loest to east., our reckoning of days and dates, unless we subtract a day at the Day Line, comes out one day in advance of the correct count, as kept by our home people. Now this error of the voyager's is caused by his failure to make the change in his count when, in his voyage, he reaches the Day Line — caused by his ignorance in not knowing when (because of his sailing out of the old day into the territory of the SYSTEM ARITHMETIC— GRADE A. 705 new day) he should change his count. And before he may know where to change his count, he must understand, of course, where and what such Day Line is. "Ship- masters actually change their count at [or about] the 180th meridian from Greenwich, to accord with these facts. Thus, were you going from California to Canton, China, and crossed the 180th meridian on Monday, you would see posted in conspicuous places on the ship an announcement that, having crossed the 180th meridian, the date is changed from Monday to Tuesday. So, likewise. If you were returning from Canton to San Francisco, and should cross this meridian on Monday, you would see a notice of change of date to Sunday." Why the true Day Line cannot be identical with the date Une which separates the two different counts of dates used by the eastern Asiatics and some of the islanders of the western Pacific Ocean, it is easy to understand ; for that date line is so far de- flected to the west at the Philippine Islands as to bring it out of coincidence with the circle of illumination. Suppose the circle of illumination (i. e., the Day Line, the circular line that marks the division between the day and the night halves of the globe) coincides for a distance, with this date line, at Chatham Island (see the date line map, page 703) : it could not, then, coincide with it farther north, in the central Pacific ; for this Day Line, dividing, as it does, the illuminated half of the earth from the unlighted half, must run, in general, very nearly directly north and south. (Refer to any astronomy or physical geography, article " Circle of Illumination.") Why this international Day Line should be located in the Pacific Ocean, and pass through Behring Strait, near the ISOth meridian from Greenwich, may be under- stood by him who wiU attend carefully to these considerations : — Sunset, midnight, sunrise, and midday come from the east and go around the earth fi'om east to west; which fact is well known even to school children. Since these phenomena go on and on, in this order, round and round the earth, in a never-ceasing circuit, — since there can be no question about the fact that sunset, midnight, sunrise, and midday come in this order as to time, and travel thus around the earth from east to west, — the only question is, Where is that " east " from which they start ? Since these phenomena start in and come from the east, the only ques- tion is. Where is that "east" ? Now, there are three weU-known and universally- received facts by which we determine where that " east " is, in which these phenom- ena of each day and night take their start on the earth : — 1. Sunset, midnight, sunrise, etc., travel on the earth from east to west; for they are things that foUow with the sun. 2. These phenomena began before the world was discovered to be round, — be- gan when the earth was supposed to be flat, — and when North America and South America were unknown to the civilized world. 3. Mankind, the reckoners of these phenomena, originated in Asia; for there philology and aU history, sacred, profane, and geological, place the cradle of the race. Neither of these three statements needs any argument to establish it as fact. The first is well known even to school children. As to the truth of the second state- ment, it may be remarked that it was not till 1520 after Christ that, by Magellan's circumnavigation, the world was proven to be round. Thus M'e see that sunset, mid- night, sunrise, and midday had been taking their start out of this " east," over and over again, as each new day came upon the earth, for a period of about five thousand 45 706 THE SYSTEM METHOD. years previous to Magellan, during the whole of which time the earth was supposed by all mankind to be flat. We come, therefore, squarely to the question thus : To this early people living in Asia on aflat earth, what would he that '■^ east ''^ from which the sun comes — rises ? Now give heed : this indefinite east could have been nowhere 07i land in eastern Asia ; for, as surely as any of these early Asiatics ever attempted to find the sun on the land by journeying to it, he found naught but that the sun was still farther in the east ; just as men now know that when a man journeys east to find the rainbow, he finds naught but that the rainbow is — wt the east. Thus, since the sun was seen to come from farther east than the land extended, — since sunrise, midday, sunset, etc., came from an east beyond the land, — the only thing that could be known was that they started from somewhere in the watery east. Therefore, the source from which the days and nights came to our early Asiatic forefathers was the watery east of the Pacific Ocean. And mark you, it was because the Creator placed original man in Asia that the days in Asia, on a supposed fiat earth, thus came to man from the watery east, the Pacific Ocean. Had he placed original man in North America instead, these phenomena would have first appeared, not in the Pacific, but in the Atlantic, Ocean. Thus it is that the providence of God has located the beginnings of days pnd nights in the Pacific Ocean, whence they go westward around the earth. Therefore sunset, midnight, sunrise, and noon could not take their start west- ward on the earth from any point west of Behring Strait. Plainly, therefore, since they could not have begun their journey on this flat earth from any point west of Behring Strait, and since they go on the earth from east to west, they (and with them the day) came to central and western Asia from extremest eastern Asia ; from western Asia to Europe and western Europe ; and in the same way it was that they crossed the Atlantic Ocean to America; for, started westward, they could jiot go otherwise. That this reckoning of days and dates did start in the east of Asia, is as certain and as plain as that mankind, the day reckoners, did start in Asia and see their sun rise in the far and uncertain watery east. Thus it was sabbath in eastern Asia before it was sabbath in western Asia; sabbath in western Asia before it wits sabbath in western Europe ; and in the same way, on this flat earth previous to Magellan, it was sabbath in western Europe before it was sabbath in eastern America ; and similarly stiU, from east to west, it was sabbath in eastern America before it was sabbath in western America. And now mark, since sabbath came from the east to the west, — came on this flat earth from Asia to America, as mankind know,— it could not come from America to Asia. Why could not this sabbath reckoning have come westward from America to Asia? — Answer: Because on the flat earth there was no America east of Asia from which it could come. America was then a thing unknown, and a thing unlmown to mankind is to mankind nothing. (See pages 75-77.) So far as mankind are concerned, the appearances called sunset, midnight, sunrise, etc., each came from where God caused it first to appear to mankind ; namely, from the Pacific Ocean east of Asia. This appearance, called " day," was first seen, — was first an ap- pearance, — not in America, but in Asia; for it was originally brought to America by sailors from Europe, whither it came originally from Asia, as aU history and philology attest. The stalling sabbath did not, therefore, strike the earth at any point east of Behring Strait. But we have already seen that it did not first strike the earth at any SYSTEM ARITHMETIC— GBADE A. ^QT point west of Behring Strait ; it did, therefore, take its start fi-om a circle of illumi- nation running north and south through Behring Strait and the Pacific Ocean. And it may be observed that since the circle of illumination, bringing with it any new day, occurs earliest in the morning and latest in the evening where the days are the long- est, of those people living at the west side of this watery Day Line, they will see and count any new day first who live farthest north and south. There are two periods only, in each year, when the Day Line runs directly north and south — the two equinoxes. Where Do the Sabbath, STew Tears, etc., Begin and End ? — The day begins at the west side of the Day Line, and ends at the east side of the Day Line. (See the article " International Day Line " next above.) How Far the Day Stretches on the Globe.— It is now Sun- day, say, at Cincinnati : where on the globe does that Sunday begin and end, and what day of the week stretches over the remaining portion of the globe ? Answer : It is Sunday back toward the east from Cincinnati tiU we reach the location where it is midnight ; beyond that point, east to the Day Line, it is Monday, and it is Sunday also from Cincinnati west to the Day Line; for, by the usage of all time-reckoning nations (the Philippine Islands, Pitcairn Island, and Alaska excepted), in reckoning their days and dates from Asia, the cradle of the time reckoners, the time is to be counted later eastward and earlier westward till we reach the watery Day Line. How Long the Day Lasts on the Crlobe. — Since the day begins on the west side of the Day Line just twenty-four hours before it ends there, and since on the east side of the Day Line the day does not begin tiU when it ends on the west side, therefore, by the proper reckoning, each of the seven days should linger on the earth for just forty-eight hom's. But because the Pitcairn Islanders and the Alaskans, who came to their homes from west of the Day Line, have been derelict in rightfully dropping a day; and because the Spaniards of the Phihppine Islands, who came to their island home from Spain westward by May of Cape Horn, have been dereUct in rightfully adding a daj', — because of these derelict acts, the day, as counted by them, lingers upon the earth for the space of about 55 hours ; because it ends by their count in Pitcairn Island and Alaska about 31 hours before it begins in Phil- ippine Islands — where, of course, it lasts 34 hours. - The Day One and the Date Line in Computations. — We have already seen how each old day ends and each new day begins along a line (the circle of illumination) running north and south through Behring Strait and the Pa- cific Ocean. Therefore the correct method of reckoning the time east and west be- tween difEerent places on the round earth, will be as foUows : — a. Longitude is not reckoned for more than 180° either east or west. b. The time is later eastward as far as the international Day Line. c. The time is earlier westward as far as the international Day Line. d. A place immediately east of the international Day Line is 24 hours earlier iu time than a place immediately west of it. e. In questions involving places separated by the Day Line, the time must be reckoned the long way around. f. In questions involving places not separated by the Day Line, the time must be reckoned the short way around. Y08 THE 8T8TEM METHOD. g. The earth turns from west to east (causing the sun to appear to travel west). h. East and west are continuous circles (not straight lines) following the paral- lels and the equator. Exception. — But to this providential practice, as here stated and as generally ob- served by the nations of the earth, the Philippine Islanders, the Pitcairn Islanders, and the Alaskans count themselves exceptions; since the last two of these peoples count the day now to begin (or first appear), not where it first appeared (namely, eastward, beyond aU land, in the Pacific) , but with themselves ; and the Philippine Islanders reckon it to begin just westward of themselves, in China. Therefore, while rules a to f above are regularly to be used in aU cases, yet to find the time for these three peoples as actually counted by them, we shall have to remember that while the Philippines use Western time, both the Alaskans and the Pitcairn Islanders use East- ern time. The following problem involves this exception : — ProUem. — ^When it is 30 min. 2 sec. past twelve Wednesday morning, October 16, at Manila, Philippine Islands, what is the time at Canton? Between Canton and Manila the short way, are of longitude (120° 30' — 113*^ 14' 30") 7° 15' 30". The long way around is (360° — 1° 15' 30") 352° 44' 30." But- Short way = 7° 15' 30" = 29 min. 2 sec. difE. in time. Long way = 352° 44' 30" = 23 hr. 80 min. 58 sec. difE. in time. Hence, when it is 30 min. 2 sec. past twelve Wednesday morning at Manila, at Canton — a. Counting short way, it is 30 min. 2 sec. past 12 Wednesday morning — 29 min. 2 sec. = 1 min. past twelve Wednesday morning. b. Counting long way, it is 30 min. 2 sec. past 12 Wednesday morning -\- 23 hr. 30 min. 58 sec. = 1 min. past ttvelve Thursday morning. The results thus differing 24 hours, it is an important question of fact which is right. Let it be perceived that in determining the time as reckoned by any given place, it must needs be known whether that place reckons its time from Greenwich east, or from Greenwich west; that is, whether it uses "Eastern " or " Western " time. In this problem, since Philippine Islanders use Western time, and the people of China Eastern time, we reckon the greatest difference between Manila and Canton, finding the time at Canton to be properly 1 minute past 12 Thursday morning. Let not the student fail to note that Canton, though 7° 15' 30" west of Manila, has nevertheless later time, a truth directly confuting the usual but pernicious doctrine of the arithmetics that, of two places, that has the later time which is farthest east. IVaTlgators at Sea. — To determine the longitude at sea, navigators take with them a chronometer (an accurate time-keeper) set to give the time of Greenwich. They first ascertain the time for the point they are at by observing the sun with the sextant. The difference of time between the chronometer and the point of observation, reduced to longitude, gives the distance east or west of Greenwich. -^^M^^^ SYSTEM ARITHMETIC— QB ABE A. T09 LONGITUDE AND TIME. CLASS II PROBLEMS. Note 1. — For solutions to a number of class II problems, in longitude and time, see " Class II Problems," page 663, Work Book No. 22. Note 2. — For why class I problems (problems in which we are to find the longi- tude) of longitude and time are to be taught before these class II problems are taught, and in the lower grades, as an empirical process, see note under "Longitude and Time," page 661, Work Book No. 22. 1. The distance from Boston to Chicago is about 843 miles. When it is noon at Boston, what is the time at Chicago, a degree of longitude at Boston containing about 51 miles? a. 51 mi. = 1° b 1 " = -1-° "• ^ — 5 1 c. 843 " = 843 X ^ = 16ja^.° a. 15° = 1 hr. b 1° = J- ^- ^ 15 c. 16^° = 16^ X tV = llii'- 6min. TJ^^sec. \—Ans. X. 12 m. — ■ 1 hr. 6 min. 7Jy sec. = 10 hr. 53 min. 52l|- sec. a. m. Solved by Ella Miller, John Comer, and Alva Van Evera. 2. How much later does the sun set at San Francisco than at Philadelphia ? X. 122° 27' 49" — 75° 9' 3" = 47° a. 15° = 1 hr. b. 1° = JL ^ 15 c. 47° = 47 X JL. = 3 hr. 8 min. a. 15' = 1 min. b. 1' — 1 ^ — 3-5 c. 18' = 18 X -jig- = Imin. 12 sec. a. 15" = Isec. b. 1" = J- ^ 15 c. 46" = 46 X jig- = 3J5 sec. [sec. — Ans. y. 3hr. 8 min. + Imin. 12 sec. + 3JLsec. = 3hr. 9 min. 15Jg Solved by David Kelley, Emma Higliley, and Wm. Miller. 3. When it is 3 A. m. at Omaha, longitude 95° 56' 14" W., what is the time at Bombay, 72° 48' E.? X. 95° 56' 14" + 72° 48' = 168° 44' 14" diflf. of long. a. 15° = 1 hr. b. 1° = tV c. 168° = 168 X tV = 11 ^^- 12 min. YlO THE SYSTEM METHOD. a. 15' = 1 min. b. V=-h c. 44' = 4-4 X tV = 2 E a. 15" = Isec. b. 1"=!^ c. 14" = 14 X 3L ^ i4 sec. [p. M. — Ans. J. 3 A. M. + 11 hr. 14 mm. 56^4 sec. ■= 2hr. 14 mm. 56i|^ sec. Solved by Henry Peterson, Olive Morris, and J. A. Macy. 2;fote. — Since Bombay is east of the starting point, the meridian of Greenwich, and Omaha is west of the same point, the difference of longitude between these two places is the sum of their distances from the common starting point (95° 56' 14" ~\- 72° 48' = 168° 44' 14"). The time at Bombay is later than at Omaha, since the day reaches Bombay first, and travels thence westward to Omaha. (See the article, "International Day Line," p. 704.) 4. When it is five minutes after four o'clock on Sunday morning at Honolulu, 157° 52' minutes W., what is the hour and day of the week at Sydney, Australia, 151° 11' E.? X. 157° 52' + 151° 11' = 309° 3' diff. of long, a. 15° = 1 hr. 20 hr. 36 min. = 12 sec. [12 sec. a. m. Mon. — Ans. 5 min. A. m. Sun. + 20 hr. 36 m.in. 12 sec. = 41 min. Solved by Lizzie PeifEer, Henry Peterson, Charles Mason, and Daniel Dwyer. 5. When it is 6 A. m. Tuesday, at Pekin, China, 116° 26' E., what is the time at San Francisco, 122° 27' 49" W.? X. 116° 26' + 122° 27' 29" = 238° 53' 29" diff. of long. a. 15° = 1 hr. b 1° = -L c. 238° = 238 X tr = ^5 hr. 52 min. a. 15' = 1 min. c. 53' = 53 X tV = 3 rain. 32 sec. a. 15" = 1 sec. c. 29" = 29 X Jt^ = Ifl sec. [26 J-^ sec. p. m. non.— Ans. y. 6 A. M. Tues. — 15 hr. 55 min. 33i| sec. = 2 hr. 4 min. Solved by Harry Littler, Eugene Albaugh, W. 11. Childs, and Katie Welch. Note. — The time at San Francisco is earUer than at Pekin, because the day reaches Pekin first and travels thence westward to San Francisco. b. 1° = J- ^ 15 c. 309° = 309 X 3^ a. 15' =1 min. b. ^ 15 c. 3' = 3 X T-V = X J- 4 hr. 5 min. A. m. SYSTEM ARITHMETIC— GRADE A. YH DATE-LINE PROBLEMS. 1. The longitude of Marshall Island is 145° E.; of Tokio, Japan Islands, 141° E. When it is 3 p. m. Wednesday at Marshall Is., what is the time at Tokio? X. 360° — (145° — 141°) = 356° ditf. of long. a. 15° = 1 hr. b 1° = JL c. 356° = 356 X -Jg- = 23 hr. 44 min. diff. of time. \—Ans.^ J. 3 p. M. Wed. + 23 hr. 44 min. = 2 hr. 44 min. p. m. Thurs. Solved by Robert Knott, J. A. Nebergall, and Hattie Johnston. Kote. — These two places, though both in east longitude and only 4° apart, have their diiference in time reckoned the long way around, since they are on oppo- site sides of the date line. The distance the long way around is what remains of the circle of 860° after 4° are subtracted. 2. When it is 6 A. M. Tuesday at Sydney, Australia, longitude 151° ir E., what is the time at Mindanao, Philippine Islands, lon- gitude 124° E.? X. 360^ + (151° 11' — 124°) = 387° 11' diff. of long, a. 15° = 1 hr. b 1° = J- "• ^ 15 c. 387° = 387 X iV = 25 hr. 48 min. a. 15' == 1 min. b 1' = -i- "•-^15 c. 11' = 11 X J^ = 44sec. y. 6 A. M. Tues. — 25 hr. 48 min. 44 sec. = 4 hr. 11 min. 16 [see. A. M. Mon., time at Mindanao. — Ans. Solved by Hattie Dickey, Lulu Nebergall, Fannie Van Bibber, and Cora Cox. Note. — If Mindanao were on the same (west) side of the date line as Sydney, the difEerence of longitude between the two places would be 27° 11', and the time at Mindanao would be 1 hr. 48 min- 44 sec. earlier than at Sydney. But Mindanao being on the opposite (east) side of the date line, its time is 24 hr. earlier than if it were in the same longitude on the west side of the date line. Mindanao time is therefore (1 hr. 48 min. 44 see. -j- 24 hr.) 25 hr. 48 min. 44 sec. earlier than Sydney time, and the difference of longitude between the two places is thus seen to be 360° -f 27° 11' = 387 ° 11.' 3. The longitude of Yeddo, Japan Is., is 140° E. ; of Kalagan, Philippine Is., 126° 30' 50" E. When it is 11 hr. 55 min. p. m. Sun- day at Kalagan, what is the day and hour at Yeddo? Y12 THE SYSTEM METHOD. X. 360° + (140° — 126° 30' 50") = 373° 29' 10" diff. of long, a. 15° = 1 br. b 1° = -J- U. 1 J 5 c. 373° = 373 X yV = 24 br. 52 min. a. 15' = 1 min. b. 1' = yV c. 29' = 29 X Jg- = 1 niin. 56 sec. a. 15" = 1 sec. ^- 1" = iV c. 10" = 10 X tV = I sec. y. 11 br. 55 min. p. m. Sun. + 24 br. 53 min. 56| sec. = 48 [min. 56| sec. A. m. Tues. at Yeddo. — Ans. Solved by E. A. Holling, Jennie Drake, Wm. C. Frampton, and Emma Vale. Note. — This problem is precisely like the one above it, except that the place at which the time is given is on the east, instead of the west, side of the date line. If Yeddo were on the east side of the date line, the difference of longitude between the places would be 13° 29' 10,'' and the time at Yeddo would be 53 min. 56| sec. later than at Kalagan. But Yeddo is on the west side of the dateline; its time, therefore, is 24 hr. later than if it were in the same longitude on the east side. The time at Yeddo is therefore (53 min. 56| sec. -\- 24 hr.) 24 hr. 53 min. 56 1 sec. later than at Kalagan ; and the difference in longitude between the two places is (360° -\- 13° 29' 10") 373° 29' 10". 4. Tbe longitude of Palawan, Pbilippine Islands, is 118° 40' E.; of Auckland, New Zealand, 174° 17' E. Wben it is 10 min. 23 sec. a. m. Tuesday at Auckland, wbat is tbe time at Pala- W^an? X. 360° + (174° 17' — 118° 40') = 415° 37' diff. of long, a. 15° = 1 br. c. 415° = 415 X ^5 = 27 br. 40 min. a. 15' =1 min. b r = J- "• -^ 15 c. 37' = 37 X -Jg = 2 min. 28 sec. y. 10 min. 23 sec. a. m. Tues. — 27 br. 42 min. 28 sec. = 8 [br. 27 min. 55 sec. p. m. Sun. at Palawan. — Ans. Solved by Amia Brammier, John Miller, and John Comer. See note under problem 2. 5. Tbe longitude of Yenua Leon, Feejee Is., is 178° 30' E.; of Tananarivo, Madagascar Island, 47° 28' 20" E., Wben it is 4 p. m. Monday at Yenua Leon, wbat is tbe time at Tananarivo? SYSTEM ARITHMETIC— GRADE A. 713 X. 360^ — (178° 30' — 47° 28' 20") = 228 ° 58' 20' diff of a. 15° = 1 hr. [long b. c. 1° 228° _ 1 — rg" = 228 X yL -- = 15 hr. 12 min. a. 15' = 1 mill. b. c. 1' 58' — 1 = 58 X xV = 3 min. 52 sec. a. 15" = 1 sec. b. c. 1" 20" 1 — T5" = 20 X tV = 1^ sec. [53J -sec. A. M. Tues. — Ans y- 4 p. M. Mon. + 15 hr. 15 min. 531^ sec. = 7hr. 15 min. Solved byE. 0. Burnside , W. F. Hoskins and Lincoln Wright. note under problem 1. COMPOUND PROPORTION. 1. If it requires 13500 bricks to build a wall 200 ft. long, 20 ft. high, 16 in. thick, each brick being 8 in. long, 4 in. wide, and 2 in. thick, how many bricks, each 10 in. long, 5 in. wide, and 3.5 in. thick, will be required to build a wall 600 ft. long, 24 ft. high, and 20 ft. thick ? a. Wall 200 ft. long, 20 ft. high, 16 in. thick; brick 8 in. long, 4 in. wide, 2 in. thick [=13500 bricks. b. " 1 u 1 " 1 " " 2 " 1 " 1 " r L3.5 0b.x8x4x2 m f; hrint* L :i 0X2 0X16" ^"^-^ UllUlb. c. " 600 " 24 " 20 ft. " " 10 " 5 " 3.5 " |-_ i3_.^|0|x|4„x_2^j. _ 266605f bricks.— Jns. Explanation: Reasoning from Many to One. 1. If a wall 200 long requires 13500 brk., one 1 long will require fewer; hence divide by 200. " " " " 20. " " " 16. " more; " multiply " 8. " " " " " 4. " " " " " 2. We now have 13.5 bricks, the number required to build a wall 1 ft. long, 1 ft. high, 1 in. thick, each brick being 1 in. long, 1 in. wide, and 1 in. thick. 2d step : Reasoning from One to Many. 7. If a wall 1 long requires 13.5 brk., one 600 long will require more ; hence multiply by 600. 24 high " " " " " " 24. 240 thick " " " " " " 240. 10 long " " fewer; " divide " 10. 5 wide " " " " " " 5. 3.5 thick " " " " " " 3.5. 2. If a " 20 high 13500 " " 1 high 3. If a " 16 thick ' 13500 " " 1 thick 4. If bricks 8 long ' 13500 " " 1 long 5. If " 4 wide ' 13500 " " 1 wide 6. If " 2 thick 13500 " " 1 thick 8. If a "1 high 13.5 9. If a "1 thick 13.5 10. If bricks 1 long 13.5 11. If " 1 wide 13.5 12. If " 1 thick 13.5 'j'14 THE SYSTEM METHOD. Note 1. — For a shorter solution and explanation of this problem, see farther on, page 717. This shorter solution is but the system, or 3-step, solution above shortened. It is, if you please, a method of going through this 3-step solution at a "hop, step, and jump," since by it we jump over the b step, passing directly from "200 ft. long" to "600 ft. long," from " 20 ft. high " to "24 ft. high," from " 16 in. thick" to "20 ft. thick," etc., thus omitting the step by which the 3-step solution reasons down to the unit one, the measuring instrument. This hop-step-and-jump process of going through the 3-step solution may be practiced in the school-room or in business, for the sake of obtaining quick results, by those who have already learned to walk steadily through the 3-step solution; but no hop-step-and-jump, or "lightning," process is to be learned except after, and as the result of, a thorough familiarity with the walking process of the system method — because of the very fact that every such quick process is but the same thing as the system method expedited by some hop-step-and-jump device to shorten things. Note 2. — Answer 'Found in Second Member. The denomination desired in the result should always be placed in the second, or right-hand, member of the equation ; for, in the 3-step solution, we reason to that which is the denomination of the answer, a.nd from aU other terms. But we think and reason in words, and we read the words from left to right. Therefore, the quantity to which we read and reason (the denom- ination of the answer) must be placed at the right. Let it be observed that in every proportion problem there is one, and but one, number which is of the same denomination as is required in the answer; and let it be well known that, in solving any problem in proportion, this number must always be written as the right-hand member of the first equation. Thus, in the problem above, the denomination required in the answer is bricks, and the only number given in the problem of that denomination is 13500 ; which number is, accordingly, writ- ten as the right-hand member of the first equation, as the " base term " from which the solution proceeds. Thus, in solving the problem above, the first thing to do is to find this base-term number which is of the same denomination as the answer re- quired. Having found this number to be 13500 bricks, we then ask which of the two sets of numbers (for there are just two sets in every compound-proportion prob- lem) belongs with the 13500 ; and it is easily seen that it is not the " wall 600 ft. long, 24 ft. high, 20 ft. thick, brick 8 in. long, 4 in. wide, 2 in. thick " that contains the 13500 bricks, but that it is the " wall 200 ft. long, 20 ft. high, 16 in. thick, brick 8 in. long, 4 in. wide, 2 in thick " which contains, or equals, the 13500 bricks. (See equations a and c of the solution above.) Note 3. — The all-important thing in beginning any new arithmetical subject is to have the student see how to use what he has already learned; for absolutely all problems belonging to the system of arithmetic are, by the system method, solved in the same way — by the 3-step solution, repeated or unrepeated. See " Complete Scheme of Solutions,' page 644. See also page 650, where it is shown that the prob- lem above may be solved in six 3-step solutions, and that the solution above is but the same thing as the 3-step solution repeated six times. 2. "What is the weight of a block of granite 8 ft. long, 4 ft. wide, and 10 in. thick, if a similar block 10 ft. long, 5 ft. wide, 16 in. thick, weighs 5200 lbs.? SYSTEM ARITHMETIC— GRADE A. 715 a. Block 10 ft. long, 5 ft. wide, 16 in. thick = 5200 lbs. b. II 1 " 1 " 1 " = ff^sl-T"? = 6.5 lbs. {Ans. c. " 8 " 4 " 10 " = 6.5 lb. X 8 X 4 X 10 = 2080 lbs.— Solved by Albert Peterson and Eugene Albaugh. 3. If a vat 16 ft. long, 7 ft. wide, and 15 ft. deep holds 384 bbls., how many bbls. will a vat 17J ft. long, lOJ ft. wide, and 13 ft. deep hold? a. Vat 16 ft. long, 7 ft. wide, 15 ft. deep = 384 bbls. all i — Ts^T'lF — 35 ""^°- c. " ITJL " 101^ " 13 " = ■§-^%%5|2jjLi_3 =546 bbls. — Ans. Solved by B. E. Reynolds and Minnie Nation. 4. How many men will be required to dig a cellar 45 ft. long^ 34.6 ft. wide, and 12.3 ft. deep in 12 d. of 8.2 hr. each, if 6 men can dig a similar one 22.5 ft. long, 17.3 ft. wide, 10.25 ft. deep in 3 d. of 10.25 hr. each? a. CeUar 22.5 ft. long, 17.3 ft. wide, 10.25 ft. deep; 3 d., 10.25 hr. = 6 men. 1, u 1 il 1 U 1 U 1 U 1 U 6pn.i3il0._2_5 D- 1 i i J- J- 22-5xl7-3xl0.23^ [= ?/.2 5 men. c. " 45 " 34.6 " 12.3 " 12 " 8.2 " ='-°fi\'|*ffi.'^^ [= 9 men. — Ans. Solved by Lizzie Peiffer and E. B. Reynolds. 5. If 27 men, in 28 d. of 10 hr. each, dig a trench 126 yd. long, 21 yd. broad, 1^ yd. deep, how long a trench 2% yd. wide. If yd. deep, will 56 men dig in 25 d. of 8i hr. each ? a. Trench 2.5 yd. wide, 1.5 yd. deep; 27 men, 28 d., 10 hr. = 126 yd. long. b. "1 " 1 " 1 " 1 " 1 " = TJiAf^'^Y^f:^^^LL5. _ [.0625 yd. long. C. " 2.75 " 1.75 " 56 " 25 " 8.25 " ==.0625y|._...|5|_x^2_5^£8^5_ [= 150 yd. long. — Ans. Solved by Wm. H. Covell and R. J. Million. 6. If 6 men in 4 m., working 26 d. for a month, 12 hr. a day set the type for 24 books of 300 pp., 60 lines to a page, 12 words to a line, 6 letters to a word, in how many months of 24 d., 10 hr. a day, can 8 men and 4 boys set type for 10 books, 240 pp. each, 52 lines to a page, 16 words to a line, 8 letters to a word, 2 boys being equal to one man ? a. 6 men, 26 d., 12 hr. ; 24 bks., 300 pp., 60 lines, 12 words, 6 letters = 4 m. b. 1 " 1 " 1 " 1 " 1 " 1 " 1 " 1 " = r 4o>o-X6x2 6Xl 2 13-;^ L2?!l3 03i6 0XTZxi' ■^^oO'ff c. 10 " 24 » 10 " 10 " 240 " 52 " 16 " 8 " = ri 3n>.xl 0x240x5 2X1 6x8, 1 fi_l m ^„„ L — 54000X10x24x10 — — i.D-f- m. ^ns. Solved by Minnie Jennings and Lottie James. Y16 TEE SYSTEM METHOD. 7. If 18 pipes, each delivering 6 gal. per minute, fill 2 cisterns in 2 hr. 16 min., how many pipes, each delivering 20 gal. per minute, will fill a cistern 11 times as large as the first cisterns, in 3 hr. 24 min.? a. 3 Cisterns, 136 min., 6 gal. per min. = 18 pipes. \) \ (i 1 " 1 " " = i8p.x i3Gxfi — 7344 c. 7| " 204 " 20 " " = iA^fh^ = 131 pipes.— ^«s. Solved by Emma Vale and J. W. Nation. 8. How many half eagles, each weighing 5 pwt. 9 gr. and made of gold ^ pure, are equivalent to 1000 English sovereigns, each weighing 5 pwt. 3.274gr. and made of gold ii pure? a. Coins \\ pure, of 123.274 gr. each = 1000 pieces. b. " l(orif)" " 1 " " = jmio.fcx^. x 12 3.274 ^ 1130011. „ u 9 u u 1 on u u — 6_7aO_0 7JLLQ — q7Q1076 J„„ Solved by Wm. F. Hoskins, Hattie Hurd, and Frank S. Smith. Explanation. — A given amount of gold wiU make a less number of coins j| pure than if the coins are made only W pure; hence multiply by 11 and divide by 12. The same gold will make a greater number of coins of 1 gr. each only, than if each coin contains 123.274 gr. : it will make 123.274 times as many coins; hence multiply by 123.274. We thus get, by canceling, the number of coins produced from a given amount of gold when each coin contains 1 gr. only, and is made pure. Second Step : This given amount of gold will make a greater number of coins only Y^^ pure than if the coins are made ^| pure — a number of coins ^-^- times greater; hence multiply by -L"-, that is, multiply by 10 and divide by 9. And, finally, this gold will produce a less number of coins of 129 gr. each than if each coin contains only Igr. : it will produce a number of coins 129 times less; hence divide by 129. Thus performing this second cancellation, we get for the correct result 973^||| coins, or half eagles. 9. If the use of $3750 for 8 m. is worth |68.75, what sum is that whose use for 2 yr. 4 m. is worth $250 ? a. 8 months' use, $68.75 interest = $3750 principal. \) 1 " "1 " = $3750 P- X 8 __ :i8. c. 28 " " 250 " = ^.VtV2¥- = $3896.10. Solved by I. C. Gary, A. J. Paden, and D. tV. Cook. Explanation. — One month's use only, would require a greater principal to produce a given amount of interest than 8 months' use : it would require a principal 8 times greater; hence multiply by 8. It would require a less principal to produce $1 only, in a given time, than to produce $68.75; hence divide by 68.75. Then, by cancehng, we ascertain that it requires S.ff principal to produce $1 interest in 1 month. Second Step : 28 months' use would require but ^\- as great a principal to pro- duce a given interest, as 1 month's use would require; hence divide by 28. And, finally, $250 of interest produced, M'ould require 250 times as great a principal as it requires to produce but $1 interest; hence multiply by 250. SYSTEM ARITHMETIC— ORADE A. 71 Y 10. What sum of vaonQj \B ih.aX whose use for 3yr. at 4i^, is worth as much as the use of $540 for 1 yr. Sm., at 7%? a. 20 months' use, at 7^ = $540 principal. b. 1 " " « 1 = $540 X 20 X 7 c. 36 " " " ^ = ^4^H^^^ = $4:66M^.—Ans. Solved by Kittie Wilson, J. C. Games, and Barney Stecker. Explanation. — One month's use would require 20 times as great a principal to produce a given amount of interest as 20 months' use; hence multiply by 20. And one per cent would require 7 times as great a principal to produce a given interest as Ifo would require; hence multiply by 7. Second Step : 36 months' use, and at f % , would require 3L and | as great a principal to produce a given interest as 1 month's use and at Ifo ; hence divide by 36 and | ; that is, multiply by -j^- and |-. 11. If 15 men cut 480 sters of wood in 10 d. of 8hr. each, how many boys will it take to cut 1152 sters of wood, only I as hard, in 16 d. of 6 hr. each, provided that while working, a boy can do only |- as much as a man, and that -l- of the boys are idle at a time throughout the work? a. 480 sters, 10 d., 8 hr., -| hardness, | working capacity, |- acting force = 15 workers. ]j 1 " 1 " 1 " i " 1 << « i " » [-i5^is2|_i_o x^LiAAA = 6 workers. c. 1152 " 16 " 6 " f " f " " f " " = L^f-x-f W-H- = 24 workers.— JLws. Solved by Wm. E. Haskell, Mina Federlin, and Fred Wyman. The System Solution Shortened.— In note 1 to solution 1, page 714, reference was made to this page, where it was to be shown what the shortened process is by which the system-method process, of reasoning first down to unity and thence again out to many, might be shortened and expedited. Explanation was there given of why this shortened process is not to be presented to the learner before the fuU 3-step solution is learned. This shortened process is a systemless process, except as it is an outgrowth of the 8-step-solution, or truly systematic, process. The reason the 3-step solution is a system-like, that is, a scientific, process, is because it is identical with all other system-like solutions. The reason this shortened process (except as it is an outgrowth of the 3-step solution) is a systerafess process, is because it is out- lawed, or out-systemed, being unlike all other system-like solutions. Thus, while learning the 3-step solution is learning how to solve all arithmetical problems, learning this two-step solution (except as it is the result of the 3-step method) learns naught else except this particular sort of isolated solution. Of this shortened form of the system solution, the following are examples supple- mented by explanations : — 1. If it requires 13500 bricks to build a wall 200 ft. long, 20 ft. high, 16 in. thick, each brick being Sin. long, 4 in. wide, and 2 in. thick, how many bricks, each 10 in. long, 5 in. wide, and 3.5 in. thick, will be required to build a wall 600 ft. long, 24 ft. high, and 20 ft. thick ? 718 TEE SYSTEM METHOD. a. WaU 200 ft. long, 20 ft. high, 16 in. thick; brick 8 in. long, 4 in. wide, 2 in. thick [= 13500 bricks. b. » 600 " 24 " 30 ft. " " 10 " 5 " 3.5 " " r=i3 5aobxjx4xx^6ao X2 4X 240. _ 2666051- bricks. -4ns. Explanation.— ^mce a wall 600 ft. long will require more bricks than a wall 200 ft. long, multiply by 600 and divide by 200 ; since a waU 24 ft. high wiU require more bricks than a wall 20 ft. high, multiply by 24 and divide by 20 ; since a wall 20 ft., or 240 in., thick will requu-e more bricks than a wall 16 in. thick, multiply by 240 and divide by 16; since when the bricks are 10 in. long it will require a less number than when they are 8 in. long, divide by 10 and multiply by 8 ; etc., etc. 3. What sum of money is that whose use for 3 yr., at 4i%, is worth as much as the use of $540 for 1 yr. 8 m., at 7% ? a. 30 months' use, at 1% = $540 principal. b. 86 " " " 41% = I^^HFx-V^-^ = $466.66f.— 4ws. Explanation. — 36 months' use would require a less principal to produce a given interest in a given time than would 20 months' use; hence divide by 36 and multiply by 20 ; and a rate of 4^ % would require a larger principal to produce a given interest in a given time than 7 % would require ; hence multiply by 7 and divide by 4i (or multiply by f). See this same problem solved on page 717 as a 3-step solution. 3. How many half eagles, each weighing 5 pwt. 9 gr., and made of gold -f^ pure, are equivalent to 1000 EngUsh sovereigns each weighing 5 pwt. 3.274 gr. and made of gold \\ pure? a. Coins \\ pure, each of 123.274 gr. = 1000 pieces. {—Ans. v. II 9 14 U U 1 OQ U 1000PX 11X10X1 2 3-274 Q7Si016. D- To' -'•■^y 12"^F'X-T2 9 *''^3 4 8 3* Explanation. — A given amount of gold will make a greater number of coins if each is made only j^^ pure than if each coin is made \\ pure; therefore multiply by ■11^ and divide by -f^ [i. e., multiply by -L") ; and if each coin is to contain as many as 139 gr., a given amount of gold would produce a less number of coins than if each coin should contain only 133.374 gr. ; hence divide by 139 and multiply by 133.374. FRACTIONS. Note 1. — For " Where Should Fractions be Taught? " or what is the true order in teaching fractions, proportion, reduction, percentage, etc., see pages 669-671 of of Woi-k-Book No. 33, inclusive. See pages 673-676 for a large number of problems as solved by the 3-step solution of the system method. Note 2. — Like the two eyes, the two lungs, the five fingers, and similar manifold organisms in which the parts fit together and answer to each other in such harmoni- ous way that the study of the one part alone, — eye, ear, or finger, as the case may be, — so these three (Case I, Case II, and Case III) 3-step solutions fit together and answer to each other to form one organism whole: so that the study of either learns aU three alike. For how, by means of the three organizing 3-step solutions, the sys- tem method manages to solve aU problems belonging to the system of arithmetic in SYSTEM ABITHMETIG— GRADE A. 719 one and the same way, see "Complete Scheme of Solutions," " The System Method EstabUshed," "The System Method," and " Only Three Cases," pp. 644, 647, 667, and 668 respectively. Note 3. — ^For what is meant by "Simple-Proportion Solution," as a heading to the following solutions, see " DifEerence Between the Three Cases and Simple Propor- tion," p. 671, Work-Book No. 22. 1. The owner of -j^ of a mine sold -^ of his share for $40500. What should he who owns f of a mine get for | of his share ? Simple-Proportion Solution. a. ^2_7_ of mine = 140500 b. 1 « = iJ-f X 40500 = 165000 c. i " = i X $165000 = $55000.— ^ws. Solved by John Comer. 2. A merchant's assets are $4672, and his liabilities $5840. What part of his debts can he pay ? Case III Solution. a $5840 = I, 4, f , etc b. 1 --i- 2' 3' 4' 1 c. $4672 = 4672 x Wto = ^-—^ns. Solved by W. H. Childs. 3. If a man can dig a cellar in 22i- days, how many days would it take him to dig f of it ? Case I Solution. a. f of cellar = 22.5 d. b. 1 9 a = i X 22.5 2.5 c. -I ii = 5 X 2.5 = 12. i 12. 5 di.—Ans, Solved by Charles Mason. 4. A man sold |-| of a mill, which was |- of his share. What part of the mill did he own ? Case II Solution. a. |- of his share = |^ of mill. hJL '< l.v3_5 5_ "^•9 — 7-^54 — 54 c. f " = 9 X -5^4 = f of mill. — Ans, Solved by S. F. Witmer. 5. If f of -| of a ship is worth $2540, what is the whole vessel worth ? Y20 THE SYSTEM METHOD. Case II Solution. X 3. of 5. = -5_ -^' 4 "^ 9 12- a. 3^ of ship = $2540 b. JL " = i X 2540 = 508 c. if " = 12 X 508 =$6096.-^745. Solved by W. F. Wilson. 6. What is 3^^ of i^? Case I Solution. a. i 10 _ 11 10 — 84 L =^ _1_ X 1J_ 11 '-^* 10 10'^"8T^~840 U y-Q ^ -^ 840 2 8 0- -^«*- Solved by Wm. Murpby. 7. A man bought a cow for $33J, and sold her for |- of what she cost. How much did he lose ? Case I Solution. a. f of cow = $33J b. 1 ii — IN/IOO — 100 6" 6-^3 18 c. 5 u — 5 X \7 — $277 g, price. X. $33^ — $27|- = $5f \oBB.—Ans. Solved by Clara Hendricks. 8. Two persons engage in trade; A furnished -^ of the capital, and B, -j^; if B had furnished $4921 more, their shares would have been equal. What is A's capital ? Simple- Proportion Solution. X. -j^ — 1^ ~ A difference of shares. a. -^ of capital = $492f b. Jl^ " = 1 X 4921 = 246i c. ^ " = 7 X 246J = $17241 A's capital.— ^Iws Solved by Augusta Chandler. 9. If |- of a barrel of cider cost ^^j- of a dollar, what will ii of a barrel cost ? Simple- Proportion Solution. a. 7 bbl. = %-^ U 1 u — 8. V _9_ — 7_2 u. i 7-^11 77 P 11 a 11 N^ 72 — «36 J^Q Solved by Alonzo Hjghley. 10. If i of a pole stands in the mud, 2 ft. in the water, and -^ above the water, what is the length of the pole? SYSTEM ARITHMETIC— GRADE A. ^21 Case II Solution. ^* 36 V9 -f- I2y 36 a. -1-2- of pole = 2 ft. h -i- " — 1 V 9 — 2 c. If '' = 36 X 3^ = 4-A. ft.— ^ws. Solved by Carl Hnttig. 11. What Bumber is such a part of 125 as 16 is of 24? Case III. a. b. c. 24 = I f, 1, etc. 16 = 16 X Jj = |. Case /. a. 1 = 125 b. 1 = i X 125 = 41| c. 1 = 2 X 41| = 831 -Ans. Solved by I. B. Crawford and George Lindle. 12. If I of a 3^ard of cloth -J of a yard wide cost $|, what will |- of a yard If yards wide cost ? Simple- Proportion Solution. X. Cloth f yd. long by | yd. wide = -|i sq. yd.. V Cloth -^ '' " <( 7. a u — 3_5 u J- ^^^^^ 8 4 — 3T a. 1^ sq. yd. = 6| ■-^•32 — 21^5 105 c. If " = 35 X T-§T == H.—Ans. Solved by Daniel Dwyer. Inquiry 1st. — If the inquiry arises where the author would teach the proc- ess of dividing one fraction by another, by inverting the divisor and multiplying, he would answer. Nowhere. Francis W. Parker, of Chicago, declares it impossible, as a rational, or legitimate, process to divide one fraction by another. And it is certain that such process is not needed in the doings of life, either in the 3-step solution or in business. Let the reader inspect the solutions of this (No. 23) work-book, and note the entire absence of such fraction-by-inverted-fraction process, and more, even the absence of the sign of division. And let the reader here reflect how the system method (the 3-step solution) not only eliminates all formulas and rules, but all other dogmatic devices, of which the fraction-by-inverted-fraction device is an example. Inquiry 2d. — The inquiry wiU also arise where problems of the following sort should be taught: — 1. In $^3^ how many dollars ? 2. In I how many 63ds ? 3. In 13| how many 4ths ? 4. 8 X I = what ? 5. I X Hor f of f ) = what ? 46 Y22 THE SYSTEM METHOD. The answer is : Since problems like these are problems of addition, subtraction, multiplication, division, reduction descending, and simple proportion of whole numbers ; and since empirical fractions are to be learned incidentally in connection with these subjects, such fraction matters are to be taught incidentally in connec- tion with these. (See " Where Should Fractions be Taught? " p. 669.) PERCENTAGE. I7ote 1. — On beginning an examination, or study, of the following solutions, let the reader first attend carefully and very wakefully to notes 1, 2, and 3, under " Per- centage," Work-Book No. 22. Let him first consider also the simpler solutions there given. Note 2. — In connection with his study of the following solutions, the reader should study "Complete Scheme of Solutions," "The System Method," and "Only Three Cases," pp. 644, 667, 668, of Work-Book No. 22. 1. A man sold two houses at $2500 each ; for one he received 20% more than its value, and for the other 20% less. Eequired his loss. Case II Solution. a. 120% of cost of 1st = $2500 b. 1 = ^ X 2500 = 20f c. 100 = 100 X 20f = $2083. 33i cost of Ist. a. 80% of cost of 2d =$2500 b. 1 = gV X 2500 = 31^ c. 100 = 100 X 31i = $3125 cost of 2d. X. $5208.331 cost,— $5000, s. price, = $208,331 loss.— ^ns. Solved by Eugene Albaugh, Wm. Lenker, and Lewis Filloon. 2. A man contracts to supply dressed stone for a court-house for $119449, if the rough stone costs him 16/ per cu. ft.; but if he can get it for 15/ a cu. ft., he will deduct 3% from his bill. How many cu. ft. would be needed, and what does he charge for dress- ing a cu. ft.? Case I Solution. a. 100% = $119449 b. 1 = 1194.49 c. 3 = 3 X 1194.49 = $3583.47 amt. to be deducted. X. 358347/ deducted @ 1 cu. ft. = 358347 cu. ft.—lst Ans. J. 358347 cu. ft. = $119449 [ft. of dr.stone. z. 1 " = ^-^ X 119449 = $.331- cost per cu p. $.33J — $.16 = $.17i cost of dressing a cu. ii.—2d Ans. Solved by J. M. Barclay and S. P. Gary. SYSTEM ARITHMETIC— OBADE A. ^23 3. xV brewery is worth 4^ less than a tannery, and the tannery 16^ more than a boat. The owner of the boat traded it for 75% of the brewery, thus losing $103. What is the tannery worth ? Simple- Proportion Solution. X. 100% = boat ; 116% = tannery, y. 96% of 116% = 111.36% brewery, z. 75% of 111.36% = 83.52% received for the boat, p. 100% — 83.52% = 16.48% loss on the boat, a. 16.48% = $103 t>- 1 = T6^4¥ X 103 = 6.25 c. 116 = 116 X 6.25 = $725 value of tannery.— ^W5. Solved by Emma Vale, Dora Shepherd, Fred Wyman, and E. A. Holling. 4. A man sold two horses for the same price. On one be gained 25%, on the other he lost 25%, and thereby lost |20. What did each cost, and what did he sell them for ? Case II Solution. a. 125% of cost of 1st = 100% s. price. b. 1 = _i^ X 100 = I c. 100 =80% cost of 1st. a. 75% of cost of 2d = 100% s. price. b. 1 = ^ X 100 = I c. 100 = 1331% cost of 2d. X. 213i% — 200% = 131% loss. a. 131% =$20 b. 1 = ^ X 20 = f c. 100 = 100 X I = $150 s. ^vicQ.—lst Ans. y. 80% of $150 = $120 cost of Ist.—^d Ans. z. 133i% of $150 = $200 cost of 2d.— 3d Ans. Solved by S. P. Gary, Minnie Jennings, and Hattie Dickey. PROFIT AND LOSS. 1. Bought land at $60 an acre; how much must I ask an acre that I may deduct 25% from my asking price, and still make 20% on the purchase price ? a. 100% = $60 b. 1 =.6 c. 120 = 120 X .6 = $72 desired s. price. 7^4: THE SYSTEM METHOD. a. 75% = $72 b. 1 = J^ X72=|| c. 100 = 100 X f f = $96 asking price.— ^ws. Solved by I. B. Crawford, Daniel Dwyer, Ida Wood, and Anna Sclinier. 2. Invested $10000 ; sold out at a loss of 20 % . How much must I borrow at 4%, so that, by investing all I have at 18% I may re- trieve my loss? X. $10000 — $2000 loss = $8000 amt. left. a. 100% = $8000 b. 1 =80 c. 18 = 18 X 80 = $1440 gained on amt. left. y. $2000 — $1440 =$560 to be gained on what I borrow. a. 14% (18 — 4) of amt. borrowed = $560 b. 1 u u ^ _i^ X 560 =40 c. 100 " " = $4:000.— Ans. Solved by J. S. McFaddin, Mina Pederlin, and Granville Teter. 3. I sell Sit 8*^ gain, invest the proceeds, and sell at an advance of 121% J invest the proceeds again, and sell at 4% loss, and quit with 11166.40. What did I start with ? a. 100% == 108% proceeds 1st sale. b. 1 = 1.08 c. 112i = 112.5 X 1.08 == 121.5% proceeds 2d sale. a. 100% = 121.5 b. 1 = 1.215 c. 96 = 96 X 1.215 = 116.64% proceeds 3d sale. a. 116.64% = $1166.40 b- 1 = TxiFT X 1166.40 = 10 c. 100 = UOOO.—Ans. Solved by Joseph Chandler, J. A. Nebergall, and Lewis Filloon. 4. Bought stock at 10% discount, which rose to 5% premium and sold for cash; paying a debt of $33, I invested the balance in stock at 2% premium, which, at par, left me $11 less than at first. How much money did I first invest ? a. 102% of stock = 105% — $33 b. 1 a —10 5 3 3 102 102 c. 100 -^IfO $\\«. X. 11 70 $5_5o_ + $11 — 90%, amt. firs' a. 220 c/ n 70 = $336^ b. 1 17 N^3 63 — IftF^ 22 -^ 17 •'•"^ c. 90 = 90 X 1.65 = $148.50.-^^5. Solved by Harry Long, Minnie Nation, and A. J. Paden. SYSTEM ABITHMETIO^ORADE A. 725 j^ote. — This problem involves the algebraic process of " transposing terms " and " collecting terms." 5. I sold an article at 20% gain. Had it cost me $300 more, I should have lost 20%. Find the cost. X. 120% of real cost = s. price; also, 80% of supposed cost y. 120% " " = 80% of (100% + $300) [= s. price. a. 40% of real cost (120% — 80%) = $240 b. 1 '< " = 4V X 240 = 6 c. 100 " " = $600. — Ans. Solved by U. P. Long, E. O. Burnside, Anna Brammier, and Robert Knott. Note. — See note under problem 4 above. 6. I paid $125 for a horse, and traded him for another, giving 60% additional money. For the 2d horse I received a 3d and $25. I sold the 3d horse for $150. What was my % of gain or loss? a. 100% = $125 cost of 1st horse. b. 1 = 1.25 c. 160 = 160 X 1.25 = $200 cost of 2d horse. X. $25 + $150 = $175 s. price of 2d horse. y. $200 — $175 = $25 loss on 2d horse. a. $200 = 100% b. 1 = 1 c. 25 = 25 X I = 121% loss.— ^ws. 7. A horse and carriage were sold for $459, the horse at a gain of 20%, and the carriage at a loss of 10%. The horse cost 66|% of what the carriage cost. Find the cost of each. X. 90% + (120% of 66|%) = 170% s. price of both. a. 170% of carriage = $459 b. 1 " = ^ X 459 = 2.7 c. 100 " = $270 cost of carriage. — 1st Ans. y. 66|% of $270 = $180 cost of horse.— ^cZ Ans. Solved by Wm. N. Orris, Granville Teter, and John Miller. COMMISSION. 1. An agent sells 1100 barrels of flour at $4.50 per barrel, com- mission 2i%. He invests the proceeds in steel at li/ per lb., com- mission 11 %, after deducting $50 freight. What is his entire com- mission, and how many tons (2240 lb.) of steel does he buy? fj2Q TEE SYSTEM METHOD. X. 1100 bbl. @ $4.50 = $4950 s. price of flour. a. 100% = $4950 b. 1 = $49.50 c. 2-}- = 2i X 49.50 = $123.75 1st commission. y. $4950 —($123.75 + $50) = $4776.25 proceeds of flour. a. 1011% - $4776.25 b. 1 = ^L^ X 4776.25 = 47.0566 c. 100 = $4705.66 amt. invested in steel. z. $4776.25 — 4705.66 = $70.59 2d com. p. $123.75 + $70.59 = $194.34 entire com.— isi! Ans. a. 1-1-/ = 1 lb. steel. b. 1 = I X 1 = I c. 470566 = 470566 X | = 31 371 Of lb. steel. a. 2240 lb. = 1 ton. b. 1 =i^^To c. 3137101 = 3137101 X ^^^ = 140+ tons.— ^c? Ans. Solved by W. F. Wilson, Fred Wyman, S. F. Witmer, Granville Teter, Harry Littler, and Lida Oltman. 2. My agent bought 40 horses at $150 each, paid $25 for their keeping, and $80 for their transportation; he drew on me for $6315. What was his rate of commission ? X. 40 horses @ $150 = $6000 amt. invested. y. $6315 — ($6000 + $25 + $80) = $210 com. a. $6000 = 100% b- 1 = oV c. 210 = 210 X yV = 31% rate of com.— Tins. Solved by Hattie Micbacls, Delia Bond, Nellie Hickey, and Madge Brundage. 3. An agent of mine sold coi-n and then bought again at the same price, commission 3% for selling and 3% for buying. His whole commission was $12. What did the corn sell for ? a. 103% of corn bought = 97% of corn sold. h 1 " " r= _i_ V Q7 = SJ- D. i — Yu3 A J< — YWS c. 3 " " = 3 X yVa = 2^% com. for buying. a. 5yV3% of corn sold (3% + 2^^^%) = ^12 b. 1 u « ^ _i^03 X 12 = 2.06 c. 100 " " = $206 s. price of corn.— [^Ans. Solved by Emma Vale, E. E. Reynolds, Madge Brundage, and Mary McLane. SYSTEM ARITHMETIC— GRADE A. Y27 4. Sold flour at ^\% commission. Invested | of its value in coffee at 1|% commission. Eemitted the balance, $432.50. What was the value of the flour, the coffee, and my commissions? a. 100^ = I b- 1 =^ c. 11 = f X yl-g- = 1 % com. for buying coffee. X. 331% — {^ + 1) = 2^% remitted. a. 2^% = 8432.50 b. 1 = ^ X 432.5 = 15. c. 100 = $1500 value of flour. — 1st Ans. y. I of 81500 = 11000 value of coffee.— ^c^ ^TO5. z. ?j\% of $1500 = $52.50 1st com.— ^cZ ^ws. p. 1% of $1500 = $15 2d com.- .^^A Ans. Solved by Dora Shepherd, Minnie Jennings, Jennie Drake, and Emma Ball. 5. Sold a consignment of pork and invested the proceeds in brandy, commissions 4% for selling and 1\% for buying. The brandy cost $2304. What did the pork sell for, and what were my commissions ? a. 100% of brandy = $2304 b. 1 " = 23.04 [—1st Ans. c. 1^ " = li X 23.04 = 828.80 com. for buying. X. 82304 + $28.80 = $2332.80 proceeds of pork. a. 96% of pork = $2332.80 b. 1 " = -5^x2332.80 = 24.30 [—Sd Ans. c. 4 " = 4 X 24.30 = $97.20 com. for selling. y. 82332.80 + $97.20 = $::430 s. y.rlcc cA']un-k.—Sd Ans. Solved by Wm. C. Shafer, Eugene Derby, Anna Brammier, Ida Wood, and W. E. Arbingast. 6. An agent in Philadelphia sells 1850 bushels of wheat at $1.25 per bushel; having deducted $35 for freight, $25 for storage, and his commission, he remits to his principal in Chicago $2171.56i- What was his rate of commission ? X. 1850 bu. (a^ $1.25 = $2312.50 amt. of sales. y. $2312.50 — ($35 + $25 + $2171.561) = $80.93f com. a. 82312.50 = 100% b. 1 = Yari.To X 100 = ^2_^ c. 80.93f = 80.93f X 4^2^ = s^^^,_Ans. Solved by J. A. Nebergall, Lizzie Peiffer, and W. F. Wilson. 728 THE SYSTEM METHOD. 7. An agent sold wheat at 4^ commission, and invested the proceeds in sugar, at 2^ commission. His whole commission was $63. What was the cost of the sugar? X. 100^ — 4% = 96% proceeds of wheat. a. 102% of sugar = 96% of wheat. b. 1 " = _^ X 96 = If [sugar. c. 100 " = 100 X If = 94^% invested in y. 96% — 94^2^% = lif % com. on sugar. a. 5lf % (4% + lif %) = $63 b. 1 JJj^ X 63 = 10.71 [—Ans. c. 94^-2^ = 94-j2^ X 10.71 = $1008 cost. Solved by Harry Long, Wm. C. Frampton, Eugene Derby, Hattie Michaels, Mary Oxley, and J. S. McFaddin. 8. An agent sells flour on commission at 2%, and with the pro- ceeds purchases sugar at 3% commission. If he had received 3% for selling and 2% for buying, his whole commission would have been $5 more. Find the cost of the sugar. X. 100% — 2% =98% proceeds of flour. a. 103% of sugar = 98% of flour. b 1 " = 1 V 98 = 9 8 c. 100 " = 100 X ^-^ = 953iy^% invested in sugar. y. 98% - %^^\% = 2j%% com. on sugar. z. 2% (on flour) + 2^% (on sugar) = 4.^^ entire com. p. 100% — 3% =97% supposed proceeds of flour. a. 102% of sugar = 97% of flour. b 1 " r= 1 V 97 = 9 7 c. 2 " = 2 X -^-^j = l|-j-% supposed com. on sugar, q. 3% + l|f-% = 4f|-% supposed entire com. b. 1 = ^^ X 5 = 105^ IA71S. c. 95^0^ = 95J-i,\ X 105^=19996.— Solved by Tanta Mellick, Nellie Hickey, Fannie Van Bibber, Sarah Macy, Ida Wood, I. B. Crawford, and Wm. N. Orris. 9. My agent sold my flour at 4% commission. Increasing the proceeds by $4.20, I ordered the purchase of wheat at 2% commis- sion; after which, the wheat declining 3i% in value, I lose $5. What was the flour worth ? a. 102% of wheat = 96% + $4.20 b 1 " — 16 4- .70 17 ^ TT C. 100 " — 1AQ± I 7.0. 17 T^ 1 7" SYSTEM ARITHMETIC—GRADE A. 729 X. 1^ of wheat = ^% + $Jj- y. lA^c/^ + $ii - (W% + $Jr) + $5 = 100% + $4.20 z. 1-4^% - 1^% - 100% - $4.20 _ $1^ - $5 + $Jr a. — 460% of flour = _ $243.80 b. 1 " = Tk X 243.8 = $.53 c. 100 " =$53.— i%s. Solved by E. O. Burnside, J. M. Barclay, Hattie Dickey, Wm. E. Haskell, and G. T. Oxley. Note. — ThXa solution involves some algebraic operations, known in mathematics "transposing terms," " collecting terms, " and " changing signs." SIMPLE INTEREST AND TRUE DISCOUNT, Note 1. — For a full explanation of how the system method solves all simple, com- pound, and annual interest problems In precisely the same way as other problems are solved (by means of the 3-step solution), see Work-Book No. 22, pp. 685-688 inclu- sive. Consider especially the twelve notes there given in connection with the seven- teen solutions of interest and true-discount problems. Note 2. — For how to distinguish the truly organizing method from disorganizing, or systemless, methods, see note 3 under "Percentage," Work-Book No. 22, p. 678, For why the 3-step solution employed below is the truly organizing method, see " The System Method" and "Only Three Cases," pp. 667-668 inclusive, Work-Book No. 22. Note S. — For the important thing to be done by teacher on first assigning the subject of interest, or any other new subject, to his pupils, see note 1 under " Per- centage," page 677, Work-Book No. 32. 1. What is the interest on $1800 at 10% for 6 m. 24 d.? X. 10P"f^^6-8 = 5|% a. 100% = $1800 principal. b. 1 = 18 c. 5f = 51 X 18 = $102 interest.— ^ws. 2. The interest for 6 m. 24 d. at 10% is $102. What is the principal ? X. 10PY^.x6 8. ^ 51^ a. 5|% = $102 interest. b. 1 = ^V X 102 = 18 c. 100 = $1800 principal— ^?^s. 3. The interest on $1800 at 10% is $102. What is the time? a. $1800 = 100% ^ ,. ^, , . , ^. ..^ . . /'^ JSquation X Exiilained. — Smce 10% mter- "' -^ ~ TTFu X 100 = Yg (.gj. ig produced in 12 m., 1% would be pro- C. 102 = 102 X tV = ^l%- *iuced in J^- of 12 m., and then ^fo in 5| X. X'^Yii^^" = 6.8 m. — Ans. times Jg- of 12 m.=6.8 m.—Ans. fjSQ THE SYSTEM METHOD. 4. The interest on $1800 for 6 m. 24 d. is $102. What is the rate? a. $1800 = 100% Equation x Explained.— ^mce 6.8 m. b. 1 1800^-^ 18 produces 5|% interest, 1 m. would produce C. 102 = 102 X y 8 ^ ^l%- o.V of 5|%, and then 12 m. would produce ^ 5?^perct.x 12 _. 10^. Ans. 12 times Jg of 5f % = 10%. — Ans. 5. What principal will amount to $15200 in Syr. 8 m. @ 6^? X. 6 perct .^_i_o4 ^52^ (in interest). a. 152^ (in amount) = $15200 amount. b. 1 = 100 c. 100 (in principal) = $10000 prin. — Ans. 6. The amount is $28334, the rate 7, and the time 80 days. Find principal and interest. ^ ijp^^2,G% = 1| ^ (in interest). a. 101|% = $28334 amount. b. 1 = qIt X 28334 = 279 c. 100 = 127900 principal— ^ws. y. 1|% = 1 j X 279 = $434 interest.— ^ws. Note 1. — "It should be generally understood that the U. S. courts, and most other courts, hold the common method of reckoning a day as -g\-Q of a year as illegal. Several of the States have statutes making a day g-J-j- of a year. The proper way [theoretically speaking], therefore, to reckon time in computing interest, is to reckon the [years as] calendar years, and the exact number of days as 365ths of a year. At- tempts to collect claims with interest reckoned on the basis of a day as -^^-^ of a year, might endanger the entire claim in some of the States." — Olney''s Science of Arithmetic, p. 134. That is to say, in business life, the " common " custom of reckoning one day as ■Jg of a month and -gl^ of a year, difEers from the theory of the U. S. courts as ex- pressed in the statutes. But what is the common custom is the common custom, the statutes to the contrary notwithstanding. A thorough investigation would develop the fact that the practice of counting 360 days to the year is universal among business men in the United States, except in the case of U. S. securities. In U. S. bonds alone a day is to be counted as g-J-j- of a year. Ifote 2. — Interest computed by reckoning a day as -^^-^ of a year (a method em- ployed only in computing the interest on U. S. bonds) is called " accurate interest," or "exact interest;" while interest reckoning 1 day as -^\-^ of a year, as in the com- mon custom, is called " ordinary interest." Ordinary interest is y^- of accurate inter- est greater than accurate interest, while accurate interest is -^^ of ordinary interest less than ordinary interest. Therefore, since t^^ is about 1|%, to find the " accurate interest" subtract from ordinary interest ^\%. SYSTEM ABITEMETIG—QBADE A 731 7. ^1250 Boston, June 12, 1886. iSix months after date I promise to pay Knight, Adams, and Co., or order, Twelve Su7idred and Fifty Dollars, with interest at 6^, value received. S. B. Brown. Disc. Nov. 15, 1876, at 6^ (true discount.) X. 6 Percy 6.1 = 3.05% a. 100% = $1250 principal. b. 1 = 12.50 c. 103.05= 103.05 X 12.50 = $1288.125 face. a. 100.5% =$1288.125 face. b. 1 = ^^ X 1288.125 = 12.81716 c. 100 = $1281.716 present worth.— ^«s. Solved by Kittie Wilson, Harry Littler, Olive Morris, and Orra Barclay. Note. — The reason the three days of grace are here added in computing the true discount is because the debt discounted is a grace-bearing debt, a promissory note. Since Mr. Brown may not pay Dec. 12, — not till Dec. 15 (the last day of grace), — the purchaser discounting (purchasing) the note Nov. 15 may have to wait the full 30 days for Mr. Brown to pay. If, therefore, in purchasing the note, the purchaser (discounter) does not discount for the full 30 days, but only for 27 days, he may lose the difference between the interest on such supposed present worth ($1381.74) and the interest on the principal ($1250) for these 3 days. See the next problem below for a similar circumstance, problem, solution, and explanation. See also this same problem as a subject of bank discount, page 735. Indianapolis, Ind., September 16, 1886. Three months and 27 days after date, J promise to pay A. J. Smith, or order, Three Thousand Dollars, with interest at B%, value received. J. B. Sloan. The above note was purchased, at true discount, November 1, 1886, at 6%. What were the present worth and the true discount? „ 3 perct.x 4 -— -\ (y/ a. 100% = $3000 principal. b. 1 = 30 c. 101 = $3030 face, or amt. y. 6perct.x2.5 =, 1.25% a. 101.25% = $3030 flxce of debt. ^- 1 = T0T.25 X 3030 = 29.9259 c. 100 = $2992.59 p. worth.— ^ms. z. 1.25% == 1.25 X 29.9250 = $37.41 t. disc— ^ws. Solved by Emma Vale, Lincoln Wright, and C. W. Vance. Y32 THE SYSTEM METHOD. j^Qle i_ — The time of notes bought and sold by true discount is reckoned precisely as in simple interest and bank discount. When the time is expressed in months, cal- endar months are used in determining the day of maturity. "When the time is ex- pressed in days, the exact number of days is used. See note 3 under " Bank Discount." ], 1 U il — 1_ V 9300 3000 0.1. 31'^907 — 907 c. 12 " '' = 12 X 11^^ = 39^1.%.— Ans. Solved by W. F. Hoskins, Wm. C. Frampton, Minnie Jennings, J. M. Barclay, John B. Coyne, and Ida Wood. 8. At Avhat rate does a bank discount when ^ ''/g 3 day notes yield it 2% interest a month ? X. 2% each m. for 2.1 =4.2% disc, collected. a. 104.2 amt. disc. = 100% principal. b. 1 " '' = Tih.2 X 100 = -^0^ c. 4.2 disc. col. = 4.2 x T2.1 = ^iir% of principal, a. 2.1 m. of discount =^5^% of iDrincipal. Vv 1 a u In/ 210 — 10 00 '-'• -i — ¥-T -^"521 521 c. 12 " '' = 12 X Wt =23 JJj-^-— ^'^«- Solved by Emma Vale, Olive Morris, Mina Federlin, Robert Knott, Dora Shep- herd, and E. E. Reynolds. 9. By discounting notes at 20% per annum, a bank gets 22^% per annum interest. How long do the notes run ? Remark. — Since bank discount exceeds simple interest by the interest on the inter- est for the time to run, it is plain that while 22 1- would represent the amount of the bank discount, 20 would represent the amount of the simple interest, and the differ- ence (2i) would be the interest on 20 for the time to run. We have then : The prin- cipal 20, the interest 21, the rate 221 to find the time. We proceed thus: — a. 20 principal = 100% b. 1 " = 5 c. 2i interest = | X 5 = 2_5%. a.22i% of interest = 12 months. l~v 1 " — 2v19 — 8 D. 1 _ ^g. X i-. — y^ C, 2_5 u ^ 2_5 X _8_ = 6| m.—AoiS. Solved by Wm . X. Orris, Harry Long, E. O. Burnside, Lincoln Wright, and S. P. Gary. 10. By discounting a note at 2% a mouth, I get 25^^ interest per annum. How long does the note run ? a. 24% principal = 100% b. 1 = -2-^ 0. 1^ interest = 1^ x ^ = VtV^- 1. To Find True Discount. X. 12pe^r_ct.x 1-2,% a. 2% = $.40 excess. b. 1 = .20 c. 100 == $20 t. discount, SYSTEM ARITHMETIG^ORADE A. ■ Y3T a. 25^% of interest = 12 months. Vi 1 << a — 47 9 N/ 19 ^=: 479 p 2100 u ^ ^^ *"^ / : \ ^^ s » » ^ Oo X ''%...^- '- \.*^ cS ^ ^AO^ ^^^ °- ^' 9?. ^■'o, x-* % ^^ LIBRARY OF CONGRESS 019 760 015 9